Musawi 4th Edition

You might also like

Download as pdf or txt
Download as pdf or txt
You are on page 1of 792

f he Field of Pediatrics

Questions
ZUHAIR ALMUSAWI
1. What is the leading cause of worldwide under -five mortality rate (
A . Preterm birth complications USfo K|?
B. Pneumonia
C. Perinatal asphyxia
D. Diarrheal diseases
E . Malaria

2. Which of the following is the MOST common cause of under -5


developing countries? mortality ln
A. Diarrheal disease
B. Pneumonia
C. Malaria
i D. AIDS
E. Measles

3. Which of the following countries has the lowest rank of child mortality
rate in
the world ?
A. U .S. A.
B. United Kingdom
C. Cuba
D. Czech Republic
E. Japan

4. in 2002 an estimated l.S million under -5 deaths were caused by vaccine


preventable diseases.
Which of the following is the top
contributor ?
A. Haemophilus
influenzae B (Hib)
B. Measles
C. Rotavirus
D. Pertussis

1
r

E Tetanus
hich of the following conditions curative treatment i15 Possible but may
^succeed'?
nnt
A

A cystic
fibrosis
immunodeficiency
B Severe
c chronic respiratory failure
D Muscular dystrophy
E. Advanced cancer
following conditions, there is intensive long -term
6. For which of the treatment,
but premature death is still possible ?
A . Severe cerebral palsy
B. Cystic fibrosis
C. Tay - Sachs disease
D. Batten disease
E. Severe forms of osteogenesis imperfect

7 What is the FIRST line drug used in moderate pain management ?


A. Short - acting opioid
B. Acetaminophen
C. Ibuprofen
D. Salicylates
E . Codeine

8. Which of the following can be used as adjuvant drugs for bone pain ?
A. Amitriptyline
B Carbamazepine
C. Steroids
0- Codeine
Ketamine
9- Which of the following can be used in a 10-old child with life-threatening
lllness and excessive respiratory secretions?
A - Intramuscular hyoscyamine sulfate
B. Scopolamine patch
Intramuscular atropine
D- Oral dexamethasone

2
, nw

illness and dcPreSSl
A clonidine
* 1“* * antidepressant for children with life-threa,en
0n '
" I "

9 ivlethylphemdate
Q Benzodiazepines
0. Nortriptyline
F. Methadone
common cause of global Injury deaths to children,
11. What is the MOST
adolescents, and young adults?
A. Suicide
B. Road traffic injuries
C. Drowning
D. Fire and burn
E . Falls

12. Which of the following age groups is MOST vulnerable to poisonings ?


A. Infants
B. Toddlers
C. Preschool children
D- Schoolchildren
E. Adolescents
13. Which of the
following ,s an important precursor
A- Mental health problems of serious school violence?
B. Racial tensions
Alcohol use
poverty
E. Bullying
R hich f

^ SAch
B•
°001he follow!nB is a common
‘ avoidance
Academic failure
manifestation of bullying?

f '
0C al
a Lack ofProblems
friends
SuicWal ideation
- Vear- old
boy may need careful
15. A 1A screening for bullving jf
A. quiet ^
0 overly confident
C. depressed
D disabled
E. obese

can counsel parents to help their


l6 pediatricians children avoid exposure to
any form of media violence under age of
A. 4 years
B. 6 years
C. 8 years
D. 10 years
E. 12 years

17 . Which of the following is a manifestation of stress reactions in children <6


years exposed to war or terrorism?
A. Terrified affect
B. Truancy
C. Somatization
D. Depressive affect
E. Irrational fear

18. Which type of child abuse is difficult to prove inspite of its extreme harm ?
A . Physical abuse
B. Sexual abuse
C. Psychological abuse
D. Inadequate healthcare
E. Inadequate education
?
19 - What is the MOST common manifestation of child physical abuse
A. Hair pulling
B - Bites
C. Burns
D. Bruises
E. Fractures
inflicted bruises ?
is suggestive of
in
20- Which of the following features
4
toddler
Bruising in a u
A.
B. Bruising of shin
C. Bruising of nose
bruising
D. Patterned same age
E. Multiple bruises of
is MOSTLY inflicted?
fractures is
21. which of the following
A. Clavicular
B. Femoral
C. Supracondylar
humeral
0. Posterior rib fractures
E. Distal extremity

22 A single young mother


brought her 8-year-old daughter complaining from
days. The mother gives history that
genital redness and bleeding for the last 3
learning difficulties with
her daughter has social withdrawal, fearfulness, and
new -onset bed- wetting.
Of the following, the MOST likely cause of the genital complaint is
A. urethral prolapse
B. vaginal foreign body
C. accidental trauma
D. sexual abuse
E. vaginal tumor
apter 1

The Field of
Pediatrics
ZUHAIR ALMLKAVA /I Answers
t
l.( A) The leading causes
pneumonia per natal
of worldwide U5
MR are preterm birth fr
complications,
.'.
causes are linked to '
asphyxia ,
malnutrition.
diarrheal diiseases, and malaria . Many of these d
2.( A ) Causes of under-5
• a . .
mortality
J mortality
developing nations. In developing
AIM
differ /
great v betwepn
countries 66y
''L
developed and
Ch dren <5 yr d
V
*
t
resulted from infectious and parasitic
Qrw rxf h;irikrt
/> A A
diseases. Among
P asitic HI A
the 42 °"
countries having '
°° ^
eat S /
diarrheal disease accounted for 22% of deaths
pneumonia 21%, malaria 9%, AIDS 3%, and
measles 1%.
B.(E) Although the wealth of the United States places it in the 8th position with V
respect to gross domestic product ( GDP ) per capita ( 2016) in the world, the U.S.
child mortality rate is ranked 56th in the world, at 5.8 deaths per 1,000 live
births, which is higher than the United Kingdom (4.3), Cuba ( 4.4), Canada ( 4.5),
Czech Republic ( 2.6), and Japan ( 2.0).
£
4.(C) Top contributors were pneumococcus and rotavirus, followed by *
Haemophilus influenzae B ( Hib), measles, pertussis, and tetanus.
5.(E) Conditions for Which Curative Treatment Is Possible but
May Not Succeed r
progressive cancer or cancer with a poor prognosis
Advanced or
Complex and severe congenital or acquired
heart disease ;
MB) ConditLife
ionsand* Maintaining Q.
SSS! I
Prolonging
Possible
Cystic fibrosis
Severe immunodeficiency and/ or recipients (e.g., lung,
transplant candidates
High-risk solid-organ
S multivisceral )
or
Chronicsevere resp r atorv failure
.
Muscular dystrophy formation syndromes
^ t
duplications)

Severe chromosomal disorde 6


seizeavailable
d ®
convTrt
.7 (A) For moderate
intervals When
dose
acting
long-
0
SeVi
req
f

' r
formulation
uncontrolled pamjo, as
needed
e emPen
^.
t s 'ave
h
with doses
consider
for
breakthrou ^^^ gh
r

8.(C) amitriptyline, nortriptyline) and anticonvU|San


(e g ,
Antidepressants '
gabapentin , carbamazepine, topiramate) for neuropathic pain '
le g.,
bone pam
Steroids or NSAIDs for
'' da ives
Scopolamine
and hypnotics
patch: 1.5
for anxiety and muscle spasm
mg patch, change q72h (for children >8-12 yr 0 d) ,
[
9 t )
10 IB) Because of its immediate
and positive effect on mood, methylPhenidate
at end of life, when there may
may be an effective antidepressant for children
not be time for a traditional antidepressant to
take effect.
11.(8)
12.(B) Toddlers do not have the judgment to know that medications can be
poisonous or that some houseplants are not to be eaten.
.
13 (E) Bullying and weapon carrying may be important precursors to more
serious school violence.
.
14 ( A) Signs of a child being involved in bullying or exposed to school violence
include physical complaints such as insomnia, stomachaches, headaches, and
new-onset enuresis. Psychological symptoms, such
as depression, loneliness,
anxiety, and suicidal ideation, may
irritability, poor concentration, school
.
occur Behavioral changes, such as
avoidance, and , are
substance abuse
failUre S0C,al Problems and W
cdfriends can ,

^
ooaur '
VerlV COnfident lackine in empathy, or
'
having persistenUonduct
16.(0 These younger
from reality.
-
17 (A)
child 1
problem
0 not have

Manifestations of Stress n
^1 °

Screenin6'
the capacity to distinguish fantasy
0
ChMrenTs rYriSm'”* ° ' "
6
Urban Vi lence°
S
3nd Adolescents Exp0Se *
' SngVebIharviorSeParati°
-
' Terrifieddisorderst
6|eep

affect
6nt *
— n

lrnrnobi ,itv )
ing
V Regressive behavior
Expressions of helpless
Children 7-11 Yr ness and passivity
^ Decline in school performance
V Truancy
V Sleep disorders
Somatization
^V Depressive affect
Abnormally aggressive or violent
behavior
* Irrational fears
v Regressive and childish behavi
vior
^

Expressions of fearfulness,
withdrawal, and worry
18.( C) Psychological abuse includes verbal
abuse and humiliation and acts that
scare or terrorize a child . Although this form of
abuse may be extremely harmful
to children, resulting in depression, anxiety, poor self -
esteem, or lack of
empathy, child protective services seldom becomes involved because of the
difficulty in proving such allegations.
19.(D)
20.(D) Bruises are the most common manifestation of physical abuse. Features
suggestive of inflicted bruises include (1) bruising in a preambulatory infant
(occurring in just 2% of infants), ( 2 ) bruising of padded and less exposed areas
(buttocks, cheeks, ears, genitalia ), ( 3 ) patterned bruising or burns conforming to
shape of an object or ligatures around the wrists, and ( 4) multiple bruises,
especially if clearly of different ages.
extremity fractures
21.(D) Clavicular, femoral, supracondylar humeral, and distal
yr are most likely noninflicted unless they are mult,pie
in children older than 2
or accompanied by other signs of abuse.
22.(D)
Chapter 2 an d
, v
De elo pm en t, Be ha vior
Growth Questions

age by which external


genitals are distinguishable?
1 What is the fetal
A . 8 weeks
B. 10 weeks
C. 12 weeks
D. 14 weeks
E. 16 weeks

2. What is the approximate average term newborn weight ?


A. 3 kg
B. 3.2 kg
C. 3.4 kg
D. 3.5 kg
E . 3.75 kg

3. At what age, the baby starts to cruise ?


A. 4 months
B. 7 months
C. 10 months
D . 12 months
E - 15 months

4 . What is the
approxi te age of
A. At birth humerus head appearance
3 weeks
?
C. 6 weeks
D. 12 weeks
E . 6 months

5- At what r
a8e
native language
. Infants can
? discrimilnate
Rythmic
Patterns in native vs non-
A. 2 months
B. 4 months
C. 6 months
D. 8 months
E. 10 months

6. What is the age of achievement of object permanence (constancy)?


A. 4 months
B. 7 months
C. 9 months
D. 12 months
E. 15 months

7. Which of the following is of benefit for colic management in a 2 - month- old


crying infant?
A. Simethicone
B. Anticholinergic medications
C. Fennel extract
D. Chiropractic manipulation
E . Continuous carrying

8. Ali can sit on small chair; walks up stairs with 1 hand held; makes tower of 4
cubes; and can name pictures .
What is the expected age of Ali?
A . 12 months
B. 15 months
C. 18 months
D . 24 months
E. 30 months

9. Salem runs well, opens doors; climbs on furniture; makes tower of 7 cubes,
puts 3 words together; handles spoon well; and helps to undress.
What is the expected age of Salem?
A. 15 months
B. 18 months
C. 24 months
D . 30 months
E . 36months
10
I
; stands moment arily on
1 foot; makes
rides tricycle t
ower
10 Moha mmed age and sex ; helps in dressin g; and washe$ ha 0flr
f Moham med ?
expected age
What is the
A. 18months
B 24 months
C. 30 months
D. 36 months
E. 48months

on 1 foot ; throws ball overhand; uses scissors to c t out


with 2- 4 parts besides head“; teij
11. Ali can hop
.
pictures copies cross and square ; draws man
; and goes to toilet alone.
story; plays with several children
What is the expected age of Ali ?
A. 30 months
B. 36 months
C. 42 months
D. 48months
E. 60 months

12. Salma can skip; draws triangle from copy; names heavier of 2 weights;
names 4 colors; counts 10 pennies correctly; dresses and undresses; and asks
questions about meaning of words.
What is the expected age of Salma ?
A. 36 months
B. 42 months
C. 48months
D. 60 months
E. 72 months

A . 170 cm
B. 175 cm
C. 180 cm
D. 185 cm
E. 190 cm
14. A 4-year-old child should put in a typical sentence at least
A. 2 words
B. 3 words
C. 4 words
D. Swords
E. 6 words

15. Although 5% of preschool children will stutter, it will resolve in 80% of those
children by age of
A. 6 years
B. 7 years
C. 8 years
D. 9 years
E. 10 years

16. The upper-to-lower body segment ratio equals approximately 1.7 at birth,
and 1.0 after
A. 5 yr
B. 7yr
C. 9 yr
D. llyr
E. 13 yr

17. What is the MOST common cause of delayed teeth eruption?


A. Hypothyroidism,
B. Hypoparathyroidism
C. Familial
D. Idiopathic
E. Rickets

height of their 2 -year -


18. A couple came to you asking about the expected adult
is 70 inches tall. Your
old daughter, the mother is 63 inches tall and the father
midparental height will be
proper answer will be, the daughter ' s sex -adjusted
A. 56-64 inches
B. 58-66 inches
C. 60-68 inches
D. 62 -70 inches
E. 64-72 inches
12
you asking about the expected adult height 0f the
19. A couple came to i r 3.
year-old son, the mother iis_ 63
inches tall and the father is 70 inches tall '
Your proper answer will be, the
son’s sex-adjusted midparental height will be
K
*
'
A. 62-70 inches
B. 63- 71 inches
C. 64- 72 inches
D. 65- 73 inches
E. 66- 74 inches

, stereotyped, and rhythmic


20. A 1- year-old child presents with repetitive
movements in the form of head banging and body rocking. These behaviors
typically occur with the transition to sleep at bedtime, but also at nap times and
after nighttime arousals.
Of the following, the MOST important aspect in management is
A . ordering EEG
B. ordering MRI brain
C. referral to pediatric neurology
D. reassurance to the family
E. starting antiepileptic drug

21. Which of the following is suggested by poor linear growth in the context of
good BMI ?
A. Malnutrition
B. Celiac disease
C. Inflammatory bowel disease
0. Hypothyroidism
E. Renal tubular acidosis

22. In which of the following


conditions with short stature,
A. Familial short stature bone age is normal?
B. Constitutional delay
C. Hypothyroidism
D. Undernutrition
E. Celiac disease

23. Which of the followi


lng provide an
A. BMI accurate clinical iindex of adiposity?
B. Triceps
skinfold thickness
13
C. Hydrodensitometry
D. Bioelectrical impedance
.
E Total body water measurement

24. Which of the following is a cause of early exfoliation of teeth


?
A. Hypothyroidism
B. Hypoparathyroidism
C. Gum fibrosis
D. Hypophosphatasia
E. Crowding of teeth

25. At what age, children do understand that death


is irreversible and that it
may involve them or their families?
A. Children approximately 3 yr old
B. Preschool-age children
C. Early school- age children
D. Children approximately 9 yr
E. Pubertal age

26. Which of the following is TRUE regarding sleep duration and sleep patterns
in the first 2 months of life?
A . Average total sleep 15-18 hr
B. Prematures sleep less than full-term infants
C. Bottle-fed babies sleep longer than breastfed babies
D. Established nocturnal diurnal pattern
E. Sleep periods are separated by 2-4 hr awake

27. There is substantial overlap between the clinical impairments associated


with obstructive sleep apnea syndrome (OSAS) and the diagnostic criteria for
ADHD, including
A . Poor concentration
B. Increased irritability
C. Mood instability
D. Low frustration tolerance
E. Depression

28. Which of the following can accurately predict which children with snoring

have obstructive sleep apnea syndrome ?


14
clWMihi«!<*
c
"“’t
D. X - rav
^ ***
space

-—
post -na sal
E. MRI of frequent, and dism
presents with loud, PtlVf
old obese
A 12 year- . „ ith
- diaphoresis. He is mouth breathe ,- with
29. nocturnal
• rpstlfSS Sl 6Cp;
L headach. Oropharyngeal examination reveaied
enlarged tonsils -
*treatment of this boy is
, the tirst-line
Of the following
waiting
A watchful
B. weight reduction
C, positional therapy
pressure
o. continuous positive airway
E. adenotonsillectomy

of the following parasomnias occur during rapid eye movement


30. Which
sleep?
A. Confusional arousals
B. Sleep terrors
C. Sleepwalking
D. Nightmares
E. All the above

31. Which of the following is MOST successful in the management of Parttai


arousal episodes occurring on a nightly basis ?
A. Parental education
B. Reassurance
C. Avoidance of caffeine
D. Scheduled awakenings
£. Benzodiazepines

addition to the ^ ,
unp easant sensations in the legs, ir
in a car for prolonged perils ' ^ ° 8 n Vln8
' bed t0 sleeP at niBht or rld n6
'
during'he
falls out ofbed

^ ieht ** * reStleSS Sleeper' m°ves around, and eve"


IS
Of the following, the MOST likely diagnosis is
A. Leg cramps
B. Neuropathy
C. Arthritis
D. Restless legs syndrome
E. Nerve compression ("leg fell asleep")

16
Chapter * elopm ent, and Behavior
Growth, Dev Answers
SAW
the external genitals becomes c early
1.(0 By Wk 12 the gender
of
'

,x
cm

4. B)
.<
rr —
holding on to furniture.

5 (A) Infants at 2
native language.
mo of age

6.(C) A major milestone is the


can discriminate

achievemen
( constancy ), the understanding that objects
t
.
by

9 mo
« "cruises*' or walks

rhythmic patterns in native vs non -


, . .
of
.

object permanence
continue to exist, even when not
seen At 4*7 mo of age , infants look down for a yarn
ball that has been dropped
, infants persist
but quickly give up if it is not seen. With object constancy older
in searching. They will find objects hidden under a cloth or
behind the
examiner 's back.
7 (C) Gripe water ( containing herbal supplements ), and fennel extract may have
.
benefit, but the evidence is weak .
8.(C) 18 Months
Motor: Runs stiffly; sits on small chair; walks up stairs with 1 hand held;
explores drawers and wastebaskets
Adaptive. Makes tower of 4 cubes; imitates scribbling; imitates vertical stroke /

dumps raisin from bottle


; identifies 1 or more parts o<
Language: 10 words ( average ); names pictures
body
Social: Feeds self; seeks help when in
trouble; may complain when wet
soiled, kisses parent with pucker
9.(C) 24 Months

17
Motor : Runs well, walks up
and down stairs,
climbs on furniture; jumps 1 step at a time; opens
doors;
Adaptive: Makes tower of 7
cubes (6 at 21 mo ).
imitates horizontal stroke; folds scribbles in circular pattern;
paper once imita
anguage: Puts 3 words together tively
Social: Handles spoon well; often
( subject, verb ,
. ob ec )
tells about iimme
undress, listens to stories when . diate experiences; helps to
shown pictures
10.(D) 36 Months
Motor: Rides tricycle; stands
momentarily on 1 foot
Adaptive: Makes tower of 10
cubes; imitates construction of "
cubes; copies circle; imitates cross bridge" of 3
Language: Knows age and sex;
counts 3 objects correctly; repeats
a sentence of 6 syllables; most of speech 3 numbers or
intelligible to strangers
Social: Plays simple games ( in “parallel
" with other children ); helps in dressing
( unbuttons clothing and puts on shoes);
washes hands.
ll.(D) 48 Months
Motor: Hops on 1 foot; throws ball overhand; uses
scissors to cutout pictures;
climbs well
Adaptive: Copies bridge from model; imitates construction of
"gate" of 5 cubes;
copies cross and square; draws man with 2- 4 parts besides head;
identifies
longer of 2 lines
Language: Counts 4 pennies accurately; tells story
Social: Plays with several children, with beginning of social interaction and role -
playing; goes to toilet alone
12.( D ) 60 Months
Motor : Skips
Adaptive: Draws triangle from copy; names heavier of 2 weights
Language : Names 4 colors; repeats sentence of 10 syllables; counts 10 pennies
correctly
Social: Dresses and undresses; asks questions about meaning of words; engages

''children
^ ^ ^^
1
"


lB C By mo, are about half their ultimate adult height. Head

..— -—- *
growth slows slightly, with 85% of adult head circumference ac leve y age
yr, leaving only an additional 5 cm ( 2 in) gam overthe
wQrds the

£S£
age 2 yr, 3 by age 3 yr, and so on ).
£ «

18
„ should be referred for evaluation if it is severe #

, ,
dii de i'
,a,i
^ ° “^
syndroirie
hypogonadism or Marfan d when n0 teeth have erupted by
17.(D) Delayed
eruption ^^^^^+ 3 SD) common causes inc|ud’
approximately 13 n ^

hypopar
iod i he -i T "' xr
endocrine disorders (e.g., hypothyroidism,
, °
(crowding, gum fibrosis).
) + Paternal height ]/.2
18.(C) Boys: [( Maternal height + 5 inches
- 5 inches) J/
Girls: [Maternal height + (Paternal height
and below this value to
Furthermore, generally 4 inches ( 2 SD) is applied above
provide a genetic target height range . For example, if the mother
is 63 inches
tall and the father 70 inches tall, the daughter 's sex-adjusted midparental height
is 64 inches ± 4 inches, for a target height range of 60- 68 inches .
19.(D) The son of these parents would have a sex - adjusted midparental height
of 69 inches, with a range of 65- 73 inches.
20.(0) Sleep-related rhythmic movements, including head banging, body
rocking, and head rolling, are characterized by repetitive, stereotyped, and
rhythmic movements or behaviors that involve large muscle groups. These
behaviors typically occur with the transition to sleep at
bedtime, but also at nap
times and after nighttime arousals. Children typically
as a ,,
h n6 emselves
engage in these behaviors
commoenint°hpSr !
^
( r back t0 s|eep; these are much more
, f *° ° >
deve,op,n8 children
* and in the

majority of V
cases donot indicat»

^ ^*
problem. Usually, the 1 8 neurologic or Psychologic
rhythmic movements is
most important *'"
sPeCt ln management
of sleep-related
reassurance ,
common, benign, and self-
limited °* * arnily that this behavior normal,
is
21.(D) Poor linear growth
or
in the
gastrointestinal i
robust BMl

•(A)
gfOWth h none
“ Infamilial short
chronological age ), stature the bone
age
' ° "
whereas constitutenal is normal (
. comparable to
delay, .

endocrinology short stature,


19
and undernutrition may be
associatedwith delav
ay iin bone age
height age. comparable to the
23.( B ) Although widely accepted
as the
and overweight, BMI may not provide best clinical measure of underweight
an a urate ndex
does not differentiate lean tissue “
and bone from fat. ' of adiposity because it
In otherwise a healthy individual,
at lower percentiles BMI >80-85% "££
nonhnear relationship between BMI and IT ]
settj Took
,
adiposity. In the
Jrmi
BMI may reflec
V
reflectt increased ^* 0 BMI
" ' --
muscle mass. Measurement of
subscapular, and suprailiac skinfold
of
h eas in athletes, high
the triceps
thickness have been used to estimate
adiposity. Other methods of
measuring fat, such as hydrodensitometry
bioelectrical impedance, and total body water .
measurement, are used in
research, but not in clinical evaluation, but
whole body dual-energy x- ray
absorptiometry ( DXA ) is beginning to emerge
as a tool for measuring body fat
and lean body mass.
24.( D) Causes of early exfoliation include hypophosphatasia, histiocytosis
X,
cyclic neutropenia , leukemia, trauma, and idiopathic factors.
25.( D) Children approximately 9 yr and older do understand that death is
irreversible and that it may involve them or their families. These children tend
to experience more anxiety, overt symptoms of depression, and somatic
complaints than do younger children.
26.( C) Total sleep: 10- 19 hr per 24 hr ( average, 13- 14.5 hr ), may be higher in
premature babies
Bottle-fed babies generally sleep for longer periods ( 2- 5 hr bouts ) than
breastfed babies (l-3hr).
Sleep periods are separated by 1-2 hr awake.
No established nocturnal diurnal pattern in 1st few wk, sleep is Y
distributed throughout the day and night, averaging 8.5 hr at night and 5.75 hr
r
27.( A) There is substantial overlap between the clinical
with OSAS and the diagnostic criteria for ADHD, incu mg
concentration, and distractibility.
2 .(C) Because no combination of c hj and physica| findings can
*
accurately predict which children with sn g QSAS the g0|d standard for
noeram ( PSG)•
diagnosing OSAS remains an in-lab overnight polysomnogra
k

29.( E) In the majority of cases of pediatric OSAS,


adenotonsillectomy is the f ,
ar rst
line treatment in any child with significant adenotonsi ypertrophy ,
even
the presence of additional risk factors such as obesity . ^
30.( D) Nightmares, which are much more common
than partial ar 0
uSa|
parasomnias but are often confused with them , tend to be concentrated in the
last third of the night, when REM sleep is most prominent .
31.(D) Scheduled awakenings is a behavioral intervention that involves havi V |ng
the parent wake the child 15-30 min before the time of night that the \v
parasomnia episode occurs and is most likely to be successful in situations
where partial arousal episodes occur on a nightly basis.
32.(0) Diagnostic Criteria for Restless Legs Syndrome
A. An urge to move legs, usually accompanied by or in response to
uncomfortable and unpleasant sensations in the legs, characterized by the
following:
1. The urge to move the legs begins or worsens during periods of rest
or
inactivity.
2. The urge to move the legs is partially or totally relieved by movement.
3. The urge to move the legs is worse in the evening or at night than
during the
day, or occurs only in the evening or at night.
8. The symptoms in Criterion A occur at least three
times per week and have
persisted for at least 3 months.
C. The symptoms in Criterion A are
accompanied by significant
impairment in social, occupational, distress or
educational, academic, behavioral, or other
important areas of functioning.

i r “rher mmui“
discomfort, habitual foot tapping
). '°ra condition positional
E. The symptoms are not
abuse or medication (e g ,
attributable
akathisia )
G
. .
PhySio ,
ogical effects of a drug or
Chapter 3
Behavioral and Psychilatric
Disorders
AHMED
Questions
TAWFIQ
.
1 You are evaluating a

^ 7^ ^ quent regurgitation
of regurgitated material, archin , rechewing
fback inattentiveness, developmental
and failure to gain weight. Taking deep delay,
social history reveals baby separation
from biological mother because of
giver is the step mother. Baby was
divorce 3 months ago, and the
new care
completely well in the first 7 months
Of the following, the MOST likely diagnosis of life.
is
A. GERD
B. rumination disorder
C. Sandifer syndrome
D. pyloric stenosis
E. diencephalic tumor

2. A 4-year-old boy presents with history eating of


clay, paper, and soap. He is
quite intelligent boy with normal anthropometric
parameters, he is eating
balanced diet. During interview you noticed that the mother iis not
paying
attention to the child bad behaviors.
Of the following the MOST likely cause is
A. iron deficiency
B. autism
C. child neglect
D. family disorganization
E. zinc deficiency

3. Which of the following is representing a tic disorder in childhood?


A. thumb sucking
B. nail biting
C. trichotillomania
D. bruxism (teeth grinding)
E. head jerking

22
ic disorders
of childhood?
psychiatric
4. What are the MOST common
A. Anxiety disorders
B. Tic disorders
C. Major depressive disorders
D. Eating disorders
E. Psychotic disorders

5. A completely well and lacking


any
the last 2 days. She ts
Each episode lasts about 30 minutes.
Of the following the MOST likely cause is
A . posttraumatic stress disorder
B. acute phobic hallucinations
C. attention- deficit hyperactivity disorder
D. Tourette syndrome
E. psychosis

6. An 8- year- old boy referred to psychologist for evaluation because of frequent


running away from school.
The BEST way for avoidance of such behavior in future is to
A . transfer the child to new environment and
new school
B. teach him in special school
C. find a way to make father or mother
present during
D. address the pressure factors school time
and make a nia „
p an
staff parents and school
E. refer to pediatric psychiatrist

LA ssrsssr
.
wa ~ . J "
;!?
'
ml
" °. * ,
.
,b « «
ma gement
b
of anar .
“ *d‘ pllnarV team
? , . showed
hvsician " We eht f about
^
1.5 Kg/ week. She starts ) she
i
° eve op episodes / SaSed
confusion, and seizures
Of the following, the
of ta C ' ° ,
VCardia hypertension
MOST aPPropriate
- '
invest]gation that
A. thyroid fu
nction test helps i rnanagement
B . cranial CT
C. serum
phosphorus

2?
D. renal function test
E. random blood sugar

8. A 15-year-old girl referred from or

r„X" »
BMl 18
WOrk up were nor >al.
"
„„, jizzr .
Of the following, the MOST appropriate ni
P ofr mana8ement is
A . hospital admission with internk -
mrv management
=.- TO
C reassurance with follow up visits
D. primary health care management with interdisciplinary team
E. scheduled focused discussion visits to improve body image

9. What is the MOST common cause for complete suicide?


A. Family conflicts
B. Substance abuse
C. Hormonal changes
D. Social and media violence
E. Preexisting psychiatric illness

10. What is the antidepressant of choice in the treatment of depression in


preadolescent period?
A. Fluoxetine
B. Escitalopram
C. Sertraline
D. Paroxetine
E. Citalopram

11. Separation anxiety disorder SAD can vary in presentation according to the
affected age; those between the age of 13-16 years often have
A. excessive fear that harm will come to a parent
B. physical complaints
C. nightmares
D. reluctance to sleep alone
-
E persistent avoidance of being alone

24
.. . because of recurrent
12. An 8-year-old girl referred to cardiology Clin attacks
the interview you find
of chest pain which occurs almost
a well-dressed and quite intelligent
always at
girl
nig
«

^
describes the episode as
palpitations, sweating, shaking, shortness o ’
dizziness, chest pain, and
vmptoms and she is afraid
nausea She has concern about recurrence
, o
from impending death.
Of the following, the MOST likely diagnosis is
A. panic disorder
B. obsessive compulsive disorder
C. ischemic heart disease
D. separation anxiety disorder
E. hyperthyroidism

13. A 3-year -old girl with epilepsy on valproate 30mg/ kg/day. She is under
treatment for severe empyema with linezolid. She has responded very well
initially but on fifth day of antibiotic treatment, she develops severe attack of
hyperthermia, agitation, tachycardia, diaphoresis and tremor.
Of the following, the MOST likely diagnosis is
A. malignant hyperthermia
B. central hyperthermia
C. neuroleptic malignant syndrome
D. serotonin syndrome
E. evolving bacterial resistance

14. What is the MOST common dose-dependent


medications used in ADHD? si
side effect of stimulant
A. Irritability
B. Appetite suppression
C. Aggression
D. Social withdrawal
E. Visual hallucinations

IS. Which of the following


antidepre
inhibitor SSRI? ssants i» s a
A. Fluoxetine Elective Ser
B. Mirtazapine °tonin reuptake
C. Venlafaxine
D. Duloxetine
.
E Bupropion

16. Which of the following is class(f)


medication ? ^ as first feneration
antipsychotic
A. Risperidone
B Olanzapine
C. Haloperidol
D. Lithium
E. Lurasidone

26
Chapter 3
Behavioral and Psychiatric Disorders
Answers
AHMED TAWFIQ
l.( B) Risk factors for rumination
disturbed relationship with primary
stimulating environment, neglect, stressful life situations, learned behavior
reinforced by pleasurable sensations, distraction from negative emotions, and
inadvertent reinforcement ( attention) from primary caregivers. Treatment is
focused on resolving the social issue and behavioral treatment is to reinforce
correct eating behavior while minimizing attention to rumination. A and C can
have similar presentation but in absence of rechewing and developmental
delay. D can present earlier with completely different scenario. In E usually the
child mood is very cheerful.
2.(0) Pica involves the persistent eating of nonnutritive,
nonfood substances
(e.g., paper, soap, plaster, charcoal, clay, wool,
ashes, paint, earth) over a
period of at least 1 mo. The eating behavior
should be inappropriate to the
developmental level therefore minimum age
etiologies have been proposed but not
,
of 2 yr is suggested.
Numerous
proved ranging
causes to physical ones. They include #
from psychosocial
nutritional
and calcium), low socioeconomic
factors ( e g lead deficiencies (e.g., iron, zinc,
and neglect, family disorganization
(e g nonr c ,
Da
P exposure), child abuse
..
learned behavior, underlying (

^
1’ mental disorder,
* '°T" •
but
cultural and familial factors. undetermined
Regarding dktr„ , b chem cal
less likely because of balance ers*; niJtritional disorder, and
diet
normal growth and development child \
ect 1is a so deficiencies are
not in such scenario. Pira anc
* ^
* autism can be a
ess likely because of
co-morbidities but
3.(E) Habits involve an
action or
Habits are common in f beh»
behavior (e.g., thumbchildhood
suck ,
d
and "
ran«e° from vior that is repeated
-

trichotillomania, bruxism). no,


I bi i B) , usual! u
ben 8n ar>d
*re '
often.
tics, other examples: The
eve hi distra«er is to " ° ProblematicC
transient
extremities, all are fast
bZ movement
’Drief
ro
? ^ 8 s mple' motor"
Pre nting .
^ ,*. ,nvo shruKBinD
houkte a
(e g
6
fe ' o

*ving on “ "a ,fe SXte^' °n of the


S

27
* muscle groups.
Anxiety disorders are the most common psychiatric disorders of
4.( A )
, occurring in 5- 18% of all children and adolescents, prevalence rates
childhood
comparable to physical disorders such as asthma and diabetes.
5.(B) Acute
phobic hallucinations are benign and common and occur in
previously healthy preschool children. The hallucinations are often visual or
tactile, last 10- 60 min, and occur at any time but most often at night . The child
and might complain that bugs or snakes are crawling over
15 quite frightened
attempt to remove them. The cause is unknown. The differential
him or her and
diagnosis includes drug overdose or poisoning, high fever, encephalitis, and
psychosis. The child' s fear is not alleviated by reassurance by the parents or
physician, and the child is not amenable to reason. Physical and mental status
examination is otherwise normal. Symptoms can persist for 1- 3 days, slowly
abating over 1- 2 wk.
6.(D) Truancy is more common in older children and can be a function of
multiple factors, including but not limited to learning difficulties, social anxiety,
depression, traumatic exposure, bullying, peer pressure, and substance use. In
any of these cases, the child should be referred for further evaluation to assess
the barriers to returning to school. Best practices for dealing with truancy
resulting from school avoidance and anxiety include addressing the underlying
psychological symptoms causing the school avoidance and empowering parents,
children, and school staff to work on a consistent plan for a return to school.
7.(C) For patients with anorexia nervosa and low weight, the nutrition
prescription should work toward gradually increasing weight at the rate of
about 0.5-1 Ib/wk, by increasing energy intake by 100- 200 kcal increments every
few days, toward a target of approximately 90% of average body weight for sex,
height, and age. In addition, with extremely low weight, refeeding syndrome ( a
result of the rapid drop in serum phosphorus, magnesium, and potassium with
excessive reintroduction of calories, especially carbohydrates), is associated
with acute tachycardia and heart failure and neurologic symptoms.
8 (D) Classification of anorexia nervosa AN, Mild : BMI > 17 kg/m2, Moderate:
BMI 16-16.99 kg/m2, Severe: BMI 15-15.99 kg/m2, Extreme: BMI < 15 kg/m2.
indication for admission AN, Heart rate <50 beats/min, other cardiac rhythm
disturbances, blood pressure <80/50 mm Hg, postural hypotension resulting in
0 mm Hg decrease or >25 beats/min increase, hypokalemia, hypoglycemia
^
hypophosphatemia, dehydration, body temperature <36.1*C, <80% healthy
h dy weight, and plan,
° hepatic, cardiac, or renal compromise, suicidal intent
VerY P or motivation to recover (in family and patient ), preoccupation with ego-
°
Syntonic thoughts, coexisting psychiatric disorders, requires supervision after
7
treatment of a
,
failed dav
j
y treatme nt. The
restroom, jdeaNy provided by ar
meals and while using the
]

diagn osed wfth an ^ hea|th provjder)


child or adoles cent
interdisciplinary team (physician
„ ^ ^ ^
expertise treating pediatric
patients. , d suicide have a preexisting
9 .(E) Appro ximate ly 90% of youths who C
major
° Pjo n.
ep <
Among females, chronic
psychiatric illness, most often with suicide attempts and
anxiety, especially panic disorder
, also is ass .
substa nce use conveyV
disorder and
completion. Among males, conduct
.. .
increased risk. ors (SSRIs)v
, uoxetin e and
reupta ke inhibit
10. (A) Two selective serotonin
ressan ts appro ved ..
by the U S Food and Drug
escitalopram, are the only antidep
( FDA ) for the treatme nt of depres sion in youth, and fluoxetine
Administration
alone is approved for preadolescents.
11.(B) SAD is more common in prepubertal children
, with an average age of
onset of 7.5 yr. Girls are more frequently affected
than boys. SAD is
characterized by unrealistic and persistent worries about separation from the
home or a major attachment figure. Concerns include possible harm befalling
the affected child or the child's primary caregivers, reluctance to go to school or
to Sleep without being near the parents, persistent avoidance of being 8 alone
0ne
and nightma res involvin g themes of separatio n numernnc ,
mat c sV Ptoms,
'
and complaints of subjective distress. Symptoms varv ripPendmg
age: Children <8 yr often have associated school ref (
harm will come to a parent; childre 9-12 yr h3 Ve
n
7 ^ ^
on the child's
excessive fear that
separated from a parent; and those 13-15 yr excessive distress when
often have school
physical complaints. refusal and
12.( A) Panic disorder is a syndrome of recurr
0t
fear or discomfort in which patients exp - ' discrete episodes of marked
psychological symptoms called panjc enen ce abrupt onset of
‘ PhVsical
physical and
palpitations, sweating, shaking, and sho svmptoms can include
and nausea. Children can present with f
bfeath' dizziness,
fever, wheezing, or stridor, ruling
°
ol,ltaCUr®teanrespira
0 t chest pain,
rV distress but
associated psychological symptoms i ° »c
Caus es of without
of control

13.(0 )
, persisten

Patient s with
t concerns about
settings where attacks have occurred
e
sever serow
autonomic instability, and hyp©
^ havln
syndr
fear
6 futur
of dearK
*
h lfT
/

attack' 1
s,
the
Pend
and
avoidance of
distress. The
mg doom, loss

orne
rmia as require to
hydroxytryptamine (5-HT2A r t0ni
Serotonin syndrome is generally °selfnin ) well r
^ control agitation,
- ij antagonists
mited and
can
administration
of 5 -
29
resolve .^Proheptadine ).
-- -
after the

TJX
»
*
iszrssr
,
tremor
* *
serotonergic agents are discos

» «,z?z *»i"
*
rigidity, *
myoclonus
syndrome results from excessive
hx
3

^tT
bT °
' and
a
"“
r. -
neuromusr
S
f the CNS a d Pen
"
ia --
1

rr BP

'pheJ
6 10 1
#

nervous
2 ,
^
""
.
dl z ness,

-
system serotonergic receptors and be caused by a ranee of H 8 lncludin
that can cause seroto^ ' * 6
SSRIs, valproate, and lithium
int*

include SSRIs with linezolid lant


'^ monoamine oxidase inhibitor
«»«
T

aggression, social withdrawal, and ;;,r


Amphetamine preparations prescribed concurrently with serotonergic
antidepressants can be associated with the development of serotonin
--5 .

syndrome.
15.( A) Other distracters are atypical antidepressants (non SSRI). The non-SSRI
antidepressants include bupropion, duloxetine, venlafaxine, and mirtazapine.
These medications all lack rigorous evidence to support their effectiveness in
children and adolescents, and as such should not be considered first -line
options .
.
16 (C) Based on their mechanism of action, antipsychotic medications can be
divided into first-generation (typical antipsychotics) act by blocking dopamine
D2 receptors; may be used in psychosis, Tourette disorder, severe behaviora
disorders and agitation Second-generation antipsychotics (atypica
.
antipsychotics, A, B and E) which act as dopaminergic and serotonergi
antagonists can be used in addition to above in mania, schizophrenia an
,
b Polar and used mainly in mania an
disorders. Distracter D is mood stabilizer
biP lar disorders.
°
velo pme Disord
Chapter 4
Learning and De
Questions
ntal
^
USAMA A. AL JUMAILY
iis
- seen in pre -school children Wit
1. Which of the following manifestations ^
executive dys unc
neurodevelopmental and
A. Difficulty with learning colors
and shapes
B. Problems of syntactic and semantic
skills
C. Struggling with root words, prefixes, and
suffixes
D. Deficiency in reading comprehension
E. Difficulty in answering open-ended questions

2 . A 7-year- old boy acts before thinking, Interrupts olders during


and has poor behavioral and emotional control conversations
. The intervention that i
for this child is to IS helpful
A. repeat instructions as needed
B. break tasks into smaller,
manageable steps
C. ask child to verbalize plan
D. teach response-delay
before beginning
techniques work
E. place child with partner
formodeling and
cuin
3. Which of the following is
child with attention deficit classified as a Pr t>lerT
hyperactive, .
excessively ° ‘
related P
2°^ ' ** AOHS
.
A. Runs about
B. Overlooks or misses
in , n wh
(
°°r attention 1
» ,„
details ,„1 ch it lt '
C. Mb »1
* *
«
' ' Appropriate
D. Leaves seat in
classroom or in SUre actiVjti«o k
' * ,
is expected her s tuatj qu
E. Fidgets with hands or
feet or Squ ' 0n S
n
etly
,n ' whiCh remaining seated
4. Which of the following
hyperactivity/impulsivity in a child
js ^at
i
(ADHD)? *ith
re,a ed
A. Acts as if "driven by a motor ,cit hytT
^
ract» ^ to
3l vity
disorder
0. Does not seem to listen when spoken to directly
C. Fails to finish schoolwork, chores, or duties in the workplace
p. Dislikes engaging in tasks that require sustained mental effort
E . Loses things necessary for tasks or activities ( e . g ., pencils, books, tools )

boy has poor academic performance. His mother describes that


5 An 8- year- old
to listen when spoken to him directly; he has often fails to
he does not seem
finish homework and assigned chores; he always loses pencils, books, and
school tools . His teacher describes that he often leaves seat in his classroom,
and has difficulty playing or engaging in leisure activities quietly .
Of the following, the MRI finding that is MOST likely seen in this boy is
A. large volume prefrontal cortex
B. loss of the normal hemispheric asymmetry
C. small volume occipital cortex
D. large volume basal ganglia
E. loss of cerebellar vermis

commonly
6. Which of the following clinical manifestations of ADHD is MOST
seen in pre-school children ?
A. Disorganization
B. Distractibility
C. Inattention
D. Fidgeting
E. Disruptive behavior

7. Which of the following is the MOST appropriate initial drug for the treatment
of children with ADHD ?
A. Guanfacine
B. Clonidine
C. Atomoxetine
D. Ritalin
E . Lithium

8. Which of the following developmental impacts is usually encountered


school aged children with ADHD ?
A . Unintentional injuries
B. Academic impairment
C. Low self-esteem
32
and substa nee misuse
D. Alcohol difficulties
E. Occupational dyslexia in children?
comm0n cause of
followi ing is the MOST
9. Which of the weight
A. Low birth
B . Social stress
C. Familial
0. Birth injury
E . Substance abuse
by her parents complaining of failure to speak ,
is brought normally with her parents in the,,
10. A 7 -year -old girl meetings She speaks
school and during social
,

by the parents that she has excessive shyness


home. She is described The mother has a history of socia
dependency
withdrawal, and parental following the MOST likely diagnosis
is
anxiety during childhood . Of the #

A . autism
B . anxiety disorder
C. specific language impairment
0. selective mutism
E. Isolated expressive language disorder

11. A 3-year- old boy is brought to you by his parents because of poor soaa
communications . He communicates by reaching partner ' s hand and placing it or
the desired objects. Protests are demonstrated through pushing hands. He pla
functionally with toys when seated and used eye gaze appropriately during
^
cause- and-effect play, but otherwise eye gaze Is absent . He appears to be non
engaged and responds inconsistently to his name . He has repetitive finger
movements, body rocking and lunging, and echoing
words immediately aftef
parents are said. He has a pattern of
repeated turning light switches on and off
and opening and closing doors.
Of the following, the MOST likely
diagnosis is
A . social communication
disorders
B. language disorders
C. autism spectrum disorders
D hearing loss
E. social anxiety

12. At which of the


followi
owing ages onset of buttering js
typically occurring?
33
A. Below 2 yr
B. 2-4 yr
C. 5- 6 yr
D. 7- 8 yr
E. Above 12 yr

13. Which of the following syndromes is associated with mild intellectual


disability (IQ 50- 70)?
A . Down
B . Wolf- Hirschhorn
C. Fragile X
D. Prader- Willi
E . Noonan

14. Which of the following congenital infections is associated with progressive


encephalopathy and subsequent regression of intellectual disabilities that is
started after the age of 2 year ?
A . Cytomegalovirus
B . Toxoplasmosis
C. Rubella
D . Syphilis
E. Herpes

strong predictor
15. Which of the following features encountered in infancy is a
for risk of autism spectrum disorders (ASD ) ?
A . Reduced response to name
ns
B. Trouble picking up on the nuances of social interactio
C . Difficulty engaging in group interactions
D . Repetitive behaviors of body rocking and lunging
E. Repetitive speech such as echoing words

contributor to autism spectrum


15. Which of the following factors is a strong
disorders ( ASD)?
A. Younger maternal age
5 . Maternal underweight
C. Long interval from prior pregnancy
D. Post term baby
E . Prenatal cytomegalovirus infection
34
Chapter
Learning ^ and
Answers
Developmental OiSor
%
USAMA A. AUUMAILY
might present with delayed lang
^ ;

word finding, and rhyming.


They
° a|phabet; and days of the week,
rnir:

colors, shapes, letters, and numb *


' itjon impulsivity, so teach response
/
2.(0) This child mostly has d
techniques (e.g., counting to o bafore acting) is helpful. Choice " A" cam
applied when there is working memory Vcfimrtion' Choice "B" is helpful whe
* * *

rnere is a problem of initiation. Choice C and E are helpful neip in plannir;


dysfunction.
B.(B) A child with ADHD often fails to give close attention to details or mak:
careless mistakes in schoolwork, at work, or during
other activities (eg
overlooks or misses details, work is inaccurate ).
All other distracters are relate:
to problems of hyperactivity
/impulsivity.
.
4 (A) All other distracters
MB) Brain MRI studies in hUdren
of the normal he
volumes of specific str
mSHZUre
^'
^^ " ^
—- -
ith
asV imetry in
ADHCTW^, ^ attention-
'? Cate35a Auction
as
or even loss

**
-
5 as the
'
0 reduct|on in the

,
rjJS MR data !
88eSt deficits in
volumeo
dispersed
! vPrefro !^ andsustainedatt
Ual regio ADHD that include
lobe and '^
6’(tl The 1

^
hnruptive beh " maV
’ 5 < tem

ren to disorganized,
ADHD is ' wh ch '
lstract bii,ty ' arp
norms dun th s'
1\0)
The ..vm' R^r'od nn
oftp
difficu
^ inattenti are often
0
't to dl °
re P cal in older
^
^
in 'preschoolers
-
,

presynaptic widely. used „cu erGd developmental


dopaminergic medications
medications, \including agonists, for the
methylphenidate commonly
dexm , °f ADH0 the
ethVlph
enida drnnheitamine,
35
^
and various amphetamine and dextroamphetamine preparations. If a
methylphenidate compound is unsuccessful, the clinician should switch to an
amphetamine product. If satisfactory treatment results are not obtained with
the 2nd stimulant, clinicians may choose to prescribe atomoxetine, a
noradrenergic reuptake inhibitor that has been approved by the U.S. Food and
Drug Administration ( FDA ) for the treatment of ADHD in children, adolescents,
and adults. Long-acting guanfacine and donidine are also FDA approved for the
treatment of ADHD. These medications can also treat motor and vocal tics and
so may be a reasonable choice in a child with a comorbid tic disorder.
. .
8 (B) Unintentional injuries are seen in preschool children with ADHD Low self -
esteem and alcohol/drug abuse are encountered in adolescents with ADHD.
During adulthood, occupational difficulties are encountered.
9 (C) Dyslexia is familial, occurring in 50% of children who have a parent with
.
dyslexia, in 50% of the siblings of dyslexic persons, and in 50% of the parents of
dyslexic persons.
.
10 (D) Specific language impairment (SLI) is characterized by a significant
discrepancy between the child's overall cognitive level (typically nonverbal
measures of intelligence) and functional language level. These children also
follow an atypical pattern of language acquisition and use. Closer examination
of the child's skills might reveal deficits in understanding and use of word
meaning (semantics) and grammar ( syntax ). Often, children are delayed in
starting to talk. Children with isolated expressive language disorder ("late talker
syndrome") have age - appropriate receptive language and social ability. Once
they start talking, their speech is clear. There is no increased risk for language or
learning disability as they progress through school. A family history of other
males with a similar developmental pattern is often reported .
ll (C) Children with language disorders do not have associated restricted and
.
repetitive behavior or atypical use of language. The diagnosis of
social
of restrictive
communication disorder is also distinguished from ASD by the lack
some
and repetitive behaviors. Children with hearing loss may present with
"red flags" for ASD, such as poor response to name. However, they typically
have
develop nonverbal communication and play skills as expected and do not
social anxiety may
stereotyped or restricted behavior patterns. Children with
.
present with some symptoms suggestive of ASD Shy children
may have reduced
have preserved social
eVe contact and social initiation. However, they typically
stereotyped behaviors.
•Merest and insight and will not exhibit high levels of

36
——— ^
8
‘“
S. “” , *«

sirs ~
chromosomal syndromes (e.g., veio
syndromes). In children with severe >, a
- ~£ jr " - 8 ns c
and Noonan
cause (usually prenatal) can
Causes include chromosomal
be identified in about three fourths of all ca '
d mes) and other genetic
(e.g., Down. Wolf-Hirschhorn, andI deletion lp3
* .
and epigenetic disorders (e.g., fragile X, Rett, wige mo ,
and Prader - Willi
syndromes).
14.(D)
15.(A) Symptoms can present early in infancy, with reduced response to name
and unusual use of objects being strong predictors for risk of ASD. Alf other
features are seen in toddlers and older children.
16.( E) Older maternal or paternal age may increase the risk of ASD In
addition
factors influencing the intrauterine environment, such as
maternal obesitv nr
overweight, short interval from prior pregnancy, premature
prenatal infections ( e.g., rubella , cytomegalovirus ) are
birth *
Certaln
associat d with ASD.
Nutrition
Questions
AOEEL MAHDI
1 the following age groups have the highest energy and
Which of nutrient
relative to body size ?
requirements
A. Infancy
B . Early childhood
C. School age
D. Early adolescent
E. Late adolescent

2. Which of the following statements about successful breast feeding is


recommended by American Academy of Pediatrics ( AAP) rather than by WHO?
A. Help woman initiate breastfeeding withinlhour of birth
B. Give newborns no food or drink other than breast milk unless medically
indicated
C. Encourage breastfeeding on demand
D. Uses of pacifiers to breastfeeding infants
E. Age of starting complementary feeding

3. What is the indication of IV fluid in acute severe malnutrition?


A. Severe anorexia
B. Severe dehydration
C. Hypoglycemia
D- Hyponatremia
fc Shock

^ Which
A
of the following foods is very rich in vitamin A ?
Lean meat
B,
°rgan meat
c- Milk
^E. Cheese
Fruits
38
I
common micronutrient deficiency in the world?
5. What is the MOST
A. Iron
B. Magnesium
C. Selenium
0. Zinc
E. Copper

6. Which food stuff is considered as thiamine antagonist that may con


tribut
thiamine deficiency ?
A . Milk
B. Red meat
C. Oat
0. Coffee
E . Fish

7. what is the usual cause of death in patients with


thiamine deficiency ?
A. Cardiac
B. Renal
C. CNS
D. Infection
E. Hepatic

8. A 6- year-old girl lives in


refugee camp brought to
paresthesia of feet; she you with tingling and
has decreased deep tendon
sense, and hoarseness of reflexes, loss of vibration
voice, with ptosis of eye
cardiomegaly. They were eatin lids. Chest X -ray shows
g a polished rice- base
3 months. d monotonous diet for last
Of the following,
the MOST likely cause of
A. B12 deficiency these findings is
B. thiamine deficienc
y
C. Horner syndrome
D. degenerative
brain disease
E . Guillain-
Barre syndrome
9. Among B complex
cheilosis and glossitis is vitamins deficiency, the one that specifically causes
A. thiamine
B . niacin

39
C. riboflavin
D. pyridoxine
.
E cobalamin

10. Which of the following


vitamin B orr> plex
A. B1 is destroyed by
phototherapy ?
B. B 2
C. B3
D. B6
E. B12

11. Malnutrition is defined as acute


when its duration is below
A. 1month
B. 2 months
C. 3 months
D. 4 months
E. 5 months

12. A 10- year - old boy complains of headache for last 4 weeks
associated with
vomiting, anorexia, and dry itchy desquamating seborrheic
skin lesion with no
history of fever or trauma. The family mentions that he was well
previously and
only he took some tonics capsules for several weeks prior to this event.
Of the following, the MOST likely diagnosis is
A. thiamine toxicity
B. niacin toxicity
C. hypervitaminosis A
D. histiocytosis
E. riboflavin toxicity

13. A 14- year-old girl diagnosed with anorexia nervosa presents with glossitis
and erythematosus skin rash involving both hands and feet sharply demarcated
gloves and
from the surrounding healthy skin giving the appearance of
stockings. The girl has also diarrhea, anorexiaia, and depression with insomnia
and delirium.
Of the following, the MOST likely diagnosis is
A. pellagra
B. biotin-responsive basal ganglia disease
C. beriberi
40
D. riboflavinosis
E . contact dermatitis

of pyridoxrne ( B 6) deficiency is
14. Which of the following clinical features ^
usual finding in infants?
A . Irritability
B. Seizure
C. Peripheral neuritis
D. Microcytic anemia
E. Vomiting

attributed to oxalic acid bladder stones


15. Which vitamin deficiency may be
formation?
A. Niacin
B. Thiamine
C. Riboflavin
D. Pyridoxine
E. Cobalamin

16. Consumptions of large amount of raw egg whites over long periods results
in deficiency of which of the following vitamins ?
A. C
B. D
C. E
D. B 7
E. B 2

17. Which of the following vitamins is


i almost exclusively come from animal
food ?
A. Cobalamin
B. Vitamin D
C. Pyridoxine
D. Folic acid
E. Thiamine

18. A 3-Vear-old boy


diagnosed with
and bone pain. His
management
diet is
i devoid
for the last 3
months.
of $t
-
fru ts and
With difficultV in walking
yegetables as part of h
*
41
Which of the following vitamins deficiiency should
A. A be evaluated for ?
B. B
C. C
D. D
E. K

19. In a patient with biliary atresia; the expected _


age of onset of neurological
symptoms due to vitamin E deficiency is usually
after
A. 2 months
B. 4 months
C. 6 months
D. 8 months
E. 12 months

20. The typical age of onset of hemolysis


due to vitamin E deficiency in
premature babies is during
A . 1st month of life
B. 2nd month of life
C. 3rd month of life
th
D. 4 month of life
E . 5 th month of life

21. What is an initial neurological finding usually seen in child


with symptomatic
vitamin E deficiency?
A . Loss of deep tendon reflexes
B. Limb ataxia
C. Unsteady gait
D. Dysarthria
E- Ophthalmoplegia

22. Which of the following


vitamins has limited body store and the symptoms of
deficiency can develop within weeks of limited supply?
A. E
B. D
C. A
D. K
E. B

42
Vitami„
23 - What is the MOST common site of bleeding in ate onset
deficiency bleeding ( VKDB ) ? ' t

A. CNS
B. GIT
C Skin
0. Muscles
E - Genito-urinary

24. What is the FIRST clotting factor affected by vitamin K deficiency ?


A. VII
B. vim
C. IX
D. X
E. XIII

25. What is the MOST common cause of nipple pain in early breastfeeding
mothers ?
A . Nipple candidiasis
B. Breast engorgement
C. Lack of breast massage during pregnancy
D . Hormonal changes
E. Improper latch on

26. What is the MOST common reason for using


usi infant
formula ?
A . Parental preference
B. Medical condition affecting the baby
C. Medical condition affecting the mother
D. Inadequate weight gain
E . Commercial factors

27 . Micronutrients include vitamins and tr *ce eieme


nts . By
element is ii

% of the b dy weight
definition, a trace
a <0.01
' °
he body weight
<o o2% of ‘
0 03% of the body weight
<
’ of the body we,ght
<Q 04 %
05 % of the b dV WeiBht
°
E. <0
28. Which of the following micronutrients deficiency is associated with
cardiomyopathy ?
A. Chromium
B. Copper
C. Molybdenum
.
0 Selenium
.
E Iron

29. Which of the following micronutrients define


ency, ls associated with
microcyticanemia ?
A. Chromium
B. Copper
C. Molybdenum
D. Selenium
E. Iodine

30. At which age bottle weaning should begin?


A. 12-15 mo.
.
B 16- 18 mo .
C. 19- 21 mo .
D. 22-24 mo.
E. 25-27 mo .

31. Which of following micronutrients should not be given in the stabilization


phase of acute severe malnutrition?
A. Zinc
B. Vitamin A
C. Iron
D . Copper
E. Folic acid

^^^^
32. A 17-year-old girl suffering from
"
"

muWdisciplinary weight
obesity and BMI of 42 she had
management program. The girl is mg y about her weight and asks
you about NEXT step in management; your a v
A. behaviour changes with caloric reduction
B. behaviour changes and pharmacot
C. hormone replacement therapy
44
c anges wit Ca (or .
D. hormone replac ement therap y plus beha viour
reduction
E . bariatric surgery

ng geneti c cause s of ricket s is mainly found in Middle


33. Which of the followi
East ?
1A
A. Vitamin D-dependent rickets type
rickets
B. Autosomal dominant hypophosphatemic
rickets
C. Autosomal recessive hypophosphatemic
D. X-linked hypophosphatemic rickets
with hypercalciuria
E. Hereditary hypophosphatemic rickets

you regard ing feeding of her 3 month-old


34 . A breast feeding mother consu lt
recent ly diagno sed with primar y Herpes simplex infection.
baby . She has
?
Which of the following is the BEST advice for her
A. She can breastfeed her baby with no
contraindications
receiving treatment
B. She can start breastfeeding only after
C. Breast feeding is contraindicated
D . Breastfeeding is contraindicated with active herpet
ic lesions of the
breast
E . Breastfeeding is contraindicated only with genital herpes infection

35. Which of the following infections is absolute contraindication in developed


countries to breastfeeding ?
A. Hepatitis C infection
B. CMV infection
C. Hepatitis B infection
D. Hepatitis A
E . HIV infection

36. A nursing school teacher who works a hn


milk expression technique and keen ho s per daV asks you if she can use
refr eerator w en she is in duty-
This expressed breast milk
" '
can be used within ^
^B .
24 hours
C. 36 hours
D- 48 hours

45
E 60 hours

following is the recommended duration


3 7. Which of the of using the milk
removing the aluminium foil or plastic wrap ?
formula after
A. lwk
B. 2 wk
C. 3 wk
D. 4 wk
E. 5 wk

38. Which of the following is an indication for soy based formula in infancy ?
A . Infantile colic
B. Cow 's milk protein-induced enteropathy
C. Celiac disease
D. Acute gastroenteritis
E . Galactosemia

39. You are invited to a


nutritional medical conference where the lecturer starts
to speak about children who are vegetarian but occasionally eat meat .
What is the BEST term used to describe these children ?
A. Veganism
B . Ovovegetarianism
C. Lactovegetarianism
D. Flexitarian
E. Pescatarian

40 . Which of the following plants have a high concentration of iron ?


A. Leafy green vegetables
B. Whole grains
C. Lentils
D * Dark-green vegetables
E. bright- orange vegetables

Which of the following is the cutoff value to define anaemia in a 9 - month-


0|d
infant ?
A - 95 g/L

46
B. lOg/L
C. 105 g/L
D. 110 g/L
E. 115 g/L

calculate the fluid to be given to a severe y,


by a nurse to
42. You are called o Id boy with weight of 5 kg .
dehydrated malnourished 2 - year -
BEST treatment o ption ?
Which of the following is the
mLevery 30 min for 1st
2 hr
A. ReSoMal 25
hr
for 1st 2
B. ORS 25 mL every 30 min
for 1st 2 hr
C. ReSoMal 50mL every 30 min
1st 2 hr
D. ORS 50 mL every 30 min for
JV normal saline 100 mL over
30 min .
E.

describes the composition of ReSoMal?


43. Which of the following BEST 50 g, electrolyte/mineral
WHO ORS One sachet , Sucrose
A . Water 1 L,
solution 40 mL mineral
2 L, WHO ORS One sachet, Sucrose 50 g, electrolyte/
B. Water
solution 40 mL
2 L , WHO ORS One sachet, Sucrose 25g, electrolyte/mineral
C. Water
solution 40 mL
, electrolyte/mineraJ
D. Water 2 L, WHO ORS One sachet, Sucrose 50 g
solution 20 mL
E . Water 2 L, WHO ORS two sachet, Sucrose 50 g, electrolyte/
mineral
solution 40 mL
n
44 . You are asked to prescribe antibiotics for a 1- year- old malnourished boy >
the ward. His mother tells you that he has diarrhoea for the last 3 weeks, and
on examination he is hypothermic.
Which of the following is the BEST antibiotics regime ?
A . No regime is indicated
B. Amoxicillin oral for 5 days
C. Ampicillin IV or IM for 2 days, then
amoxicillin oral for 5 days
D . Gentamicin IV or IM once dailyfor 7 days and Ampicillin IV or IM f r
days oral for 5 days °
E. Gentamicin IV or IM once daily
for 7 days and Ampicillin IV or IM f r 2
days, then amoxicillin oral for 5
days plus
metronidazole oral for 7 daVs
°
47
45. Which of the following is considered the MOST
obesity ?
common genetic cause of
A. Down syndrome
0. Leptin or leptin receptor gene deficiency
C. Prader- Willi syndrome
D . Melanocortin 4 receptor gene mutation
E . Bardet-Biedl syndrome

46. which of the following vitamins used in preterm infants to improve


respiratory function and prevent development of chronic lung
disease ?
A. A
B. D
C E
D. C
E. K

47. Which of the following drugs is the only Food and Drug Administration
( FDA )-approved medication for obesity in children<16 ?
A . Orlistat
B. Phentermine
C. Liraglutide
D. Lorca serin
E . Phentermine/ topiramate

48. Which of the following is the recommended screen time by the American
Academy of Pediatrics ?
A . No more than 2 hr/day for children at any age .
B. No more than 2 hr /day for children > 2 yr old and lhr/day for children <2
yr old
<2 yr old
C- No more than 2 hr/day for children >2 yr old and that children
not watch television
children <2 yr old
D . No more than 3 hr/day for children >2 yr old a nd that
not watch television
2 yr o Id and that children <2 yr old
E. No more than 4 hr/day for children >

not watch television


following is considered
as an earlier clinical feature of Vi
49 , Which of the
A deficiency ?
A . Xerophthalmia
B. Bitot spots
C Delayed dark
adaptation
D. Keratomalacia
E, Corneal ulceration
the mainstay of initial treatment
50. Which of the following is t.
hypervitaminosis D ?
A. Eliminate the source of excess vitamin D
B Calcitonin
,

C, Bisphosphonates
D. Glucocorticoids
E . Normal saline

51. Which of the following vitamins B complex will be lost


when the ri
rice is
repeatedly washed and the cooking water is discarded ?
A . Thiamine ( Bl )
8 . Riboflavin ( B 2 )
C. Niacin ( B 3 )

——-
D. Pyridoxine ( B6)
E. Biotin

zzxzzszr*
A.
B.
A
D
cy ? screened for which of the

C K
D. E
E. C

$ 3 . Which
«ems has the hi
*
B Red
meat
6hest vitamin D content ?
C- Green
vegetable
D- Soya
milk
E Chicken
54. How much is the approximate content of vitam
in D in breast milk ?
A. 12-60 IU/L
B. 60-100 IU/L
C. 100- 200 IU/L
D. 200- 300 IU/L
E. 300-400 IU/L

* ***“?“* *E' ,„>


b mi0 0
A. Newborn
B. Infant
C. Toddler
D. Preschool
E. School age

56. Transplacental transport of vitamin D


can provide enough vitamin D for
first the
A . 1 month
B. 2 months
C. 3 months
D. 4 months
E. 6 months

57. Which of the following causes of vitamin


D deficiency is associated with a
high serum phosphate level?
A. Vitamin D-dependent rickets
.
B Nutritional vitamin D deficiency
C. Chronic kidney disease
D. Congenital rickets
E - Dent disease

58. An 8-year -old girl with precocious puberty had a history of multiple
pathological fractures. Examination shows hyperpigmented macules and her
investigations reveal hyperthyroidism, low levels of 1,25-D, and elevated ALP
levels.
0f the following
, the MOST likely diagnosis is
A. ovarian cyst
B. McCune Albright syndrome

50
r ovarian tumour
nev us syndrome
D. epid erm al
E . Raine syndrome
MO ST likely presentation of rickets 0,
is the
following is
59. which of the
prematurity ?
A Asymptomatic
,

B. Respiratory
distress
C Poor linear grow
th
and softening of the ribs
D . Frac tures
findings
E, Classic rachitic

is the MOST common cause of hypervitaminosis D?


60. Which of the following
A. Misuse of vitamin D suppleme
nts
B. Overfortification of milk
C. Contamination of table sugar
D. Use of vitamin D supplements as cooking oil
E. Over exposure to sunlight

61. What is the recommended daily upper limit for long- term vitamin D intake
for infants ?
A. 500 IU
B. 1000 IU
C. 1500 IU
D. 2000 IU
E. 2500 IU

D?
.
62. Wh ch of the following
laboratory finding will be normal in hypervitaminosis
A - Serum calcium
B. 25-D level
C 1,25 -D level
*

D - PTH level
-
E Phosphate level
Chapter 5
Nutrition
Answers
AQEEL MAHDI

1.(A) Growth during infancy is rapid, critical for neurocognitive development


and has the highest energy and nutrient requirements relative to
any other period of growth.
bodv size than
2.(0) WHO recommendation: Give no pacifiers or artifici
al nipples to
breastfeeding infants .
AAP recommendation: The American Academy of Pediatrics
endorsed the
UNfCEF- WHO Ten Steps to Successful Breastfeeding, but does
not support a
categorical ban on pacifiers because of their role in
reducing the risk of sudden
infant death syndro me and their analgesic benefit during
painful procedures
when breastfeeding cannot provide the analgesia.
3.( £) In acute severe malnutrition do not give IV fluids
except in shock.
4.(B) Organ meats (especially liver, kidney ) are very rich
in vitamin A, whereas
other meats, milk, and cheese contain moderate levels.
5.(A) Iron deficiency is the most common micronutrie
nts deficiency in the world
and is associated with iron deficiency anemia and neurocognitiv
e deficit in some
children.
6.( D) Coffee, tea, fermented fish are thiami
ne antagonist and may contribute to
thiamine deficiency.
7-(A) Death from
thiamine deficiency usually is secondary to cardiac
'revolvement. The initial signs are cyanosis and dyspnea, but tachycardia,
enlargement of the liver, loss of consc
iousness, and convulsions can develop
rapidly.
® *(B)
Thiamine deficiency is classically associated with a diet consisting largely of
Polished rice. It develops within 2- 3 months of deficient intake.
* ( c ) ft begins
with pallor at the angles of the mouth and progresses to thinning
??! maceration of the epithelium.
( B ) The .
side chain of the B 2 vitamin is photochemically destroyed unng
P
^ototherapy for hyperbilirubinemia,
^° ^ ^
chr n c w
a nLJtrition is
define d as acute when its durat ion is below 3 months and
' ^ en its equal or more than 3 months.
13.(A) Because
is
deficiency is usually made from the physical signs of glossitis q
diagnosis of

1JcjperipheralSSlESure
children, the other clinical features
of deficiency in adult but is not jisual
may occur in infants.
stones formation, hyperglycinemia- anc
1S.(D| Oxaluria, oxalic acid bladder
deficiency.
lymphopenia are associated with vitamin B6
antagonist. Signs of biotin
16.(D) Avidin found in raw egg whites acts as a biotin
deficiency have been demonstrated in persons who consume largeamounts of
raw egg whites over long periods . Deficiency also occurs with parenteral and
enteral nutritional formula that lack biotin.
17,(A) Dietary sources of vitamin B 12 are almost exclusively from animal foods
Organ meats, muscle meats, seafood ( mollusks, oysters, fish ), poultry, and egg
yolk are rich sources.
Infants and children on highly restrictive diets, devoid of most fruits and
vegetables, are at risk of acquiring severe vitamin C deficiency. Such diets are
occasionally promoted with unsubstantiated claims of benefit in autism and
other developmental disorders. High index of suspicion is required in children
on restrictive diets, particularly those with autism and other developmental
disorders, and they should be evaluated for scurvy whenever they present with
difficulty in walking or bone pains .
19.{E) Clinical manifestations do not appear until after 1- year of
age, even in
children with cholestasis since birth. Patients may have cerebellar
disease,
posterior column dysfunction, and retinal disease.
Loss of deep tendon reflexes
is usually the initial finding.

SKI H infar‘ '


hem VSis as a result f v amin
5

d m °nth ° * E deficiency
T " 8
21.(A) Loss of deep tendon
MAI , ° f Mfe - Edema V also be present.
reflexes is °usually the initial finriinr, c K
Subsequant
manifestations include limb ataxia (intention
"),
dVsd
,
A J
dochokinesia
phthalm* P eBia limited
truncal ataxia ( wide-based, unsteady gait)
decreased proorLent
Pt"
<
9
upward gaze ), nystagmus, '° °
decreased vibratory sensation, and dysarthria (p0sltlve Romberg test)

clotting factors have .


22.(0) Unlike other fat-soluble
K In addition, there is high
vitamins
,
turnover of

*
SZZ 7 Hmited•**
“ ** *
stores of vitamin
rn K-dependent
symptomatic vitamin
K deficiency

53
within weeks when there iis inadequate
deVeHal a bsofPti o n- supply because of low
2 A
r
TtlC ,° fliost common site of bleeding in late VKDB is
^,
cuta n 6 US
°
and 61 bleeding may be the initial
cause convulsions, permanent neurologic
intracranial, although
manifestation. Intracranial
b eed
»ngcan
Ct 0
,v| has the shortest half-life of
sequelae, or
the coagulation f death.
24 ,{A ^)
to be 3 ffected
by vitamin K deficiency,
but isolated factor
actors and is the
first partial thromboplastin time. VII deficiency
not ect the
^ ,° '
J 5 .( £) p0 r n 3 positioning and improper latch are the
beyond the mild discomfort felt most common reasons
palPfc
for o PP
'
e
persist and the infant
early in breastfeeding
refuses to feed, evaluation for . If the
pro

blem . _ -l. nipple candidiasis
is indicated
26 (A) Parental
preference is the most com ..
However, infant formula also indicated fr
is T? ,
for «ing infan
breaTmt '

« contraindicated
for infant factors (eg Z " h o s e intake of
of diabolism Z
.
eternal factors In addition, infant formula 5 "*
inadequate weight gain in breastfed infant ^"
3 ent to support

27.(A )
28.(0) Selenium is an enzyme cofactor (prevents oxidative damage), its
deficiency causes cardiomyopathy (Keshan disease), myopathy. Its available in
meat, seafood, whole grains, and garlic.
29.(B) Copper deficiency causes Microcytic anemia, osteoporosis, neutropenia ,
neurologic symptoms, and depigmentation of hair and skin.
30.(A) Around 12 mo. of age, the child learns to drink from a cup and may still
breastfeed or desire formula bottle feeding. Bottle weaning should begin
around 12 - 15mo, and bedtime bottles should be discouraged because of the
association with dental carries.
1-(C) iron is given in the maintenance phase of treatment .
that
American Paediatric Surgical Association guidelines recommends
^
surgery
$
be considered only in children with complete or near -
complete skeletal
obesity, after
maturity, a BMI £40, and a medical complication resulting from
management program.
ndVe foiled 6 mo. of a multidisciplinary weight
HJy 0ne re acement therapy is available for patients with
leptin deficiency.
33 / t| ^ hypophosphatemic rickets with hypercalciuria (HHRH) is a rare
disord , ,
S ma n y found in the Middle East
.
M infr *
&reastfceding has documented short - and long -term medical
an
of which is active
npi ,r .
,f erPe°deve ^ Pmental advantages and rare contraindications
c es°,ons of the breast.
*' *
54
>' r
. CMV infection may » causing svmptomatic illness in term ,
Transmission
through nu ^
uncommon_ routinely receives hepatitis B lmmunr
, infants
Hepatitisis 0 infection HBsAg positive. No delay in |nitl ,n. 0f)
vaccine if mother
^ is
and hepatitis 8
breastfeeding is required
.
milk is a common practice when the m0tf r r .
o1
-
f breast
36.(0) The pumping plastic containers should be used to coliCC { .
or
baby are separated. Glass
refrigerated and then used within 48 hr. Exp ,..,* -
/- 1
*
L
be
milk, and milk should for up to 6 mo. Milk should be tha J’
completely within 24* .
and used
breast milk can be frozen tepid water and used
running
rapidly by holding under
thawing . Milk should
never be microwaved.
after
be covered with the original plastic cap or aluminum foil ar.
37.(0) cans should 4 wk . Once prepared, all bottle-
can be used within
the powdered product within 24 hr , Formula should t-
of formula , should be used
regardless of type a feeding ha-
within 2 hr of removal from the refrigerator , and once
used
that formula should be used within 1 hr or be discarded. Prepares
started ,
stored in the refrigerator should be warmed by placing the containers
formula
for about 5 min . Formula should not be heated in a microwave
warm water
, despite appearing to be at the
because it can heat unevenly and result in burns
right temperature when tested.
galactosemia, preference for a
38.(E) Indications for soy formula include
vegetarian diet, and hereditary lactase
deficiency, because soy-based formulas
has no prover
are lactose free. The routine use of soy protein-based formula
value in the prevention or management of infantile colic, fussiness, or
atopic
cr
disease. Infants with documented cow' s milk protein-induced enteropathy
enterocolitis often are also sensitive to soy protein. They should be provides
formula derived from extensively hydrolyzed protein or synthetic amino acids.
39.(0) It is important to understand different variations in vegetarianism, &
follows:
Veganism: excludes all animal products. It may be part of a lar§ e
practice of abstaining from the use of animal products for any purpose
/ Ovovegetarianism; includes eggs but
not dairy products.
v Lactovegetarianism: includes dairy products but
excludes eggs.
Lactoovovegetarianism: includes eggs and dairy products.
55
Flexitanam a vegetanan who will
Pescatarian: consumes fish, but often
ption may be !
sell lab i
.
occasionally ea, m
•n Id iron absor inhibited by diet , * veget arian.
!f0und in
leafy green veget ables and whole graL*
*
such » PMate
‘r0n fortified
raisins, black
-
cerea
eyed
ls, blac
peas
k beans, cashews,
, soybeans, HJnT
kidney J"T ,
rhocolate. and tempe h . sun
Iron absorption
flow
can
er J '
seeds ' ,_
lckpeas
S oa meal
'
clas ses
-
be ’ ,
containing ascorbic acid (vitamin C) along with

-
food contain!! ^ "
s ea °
r0n' " ^

-* .“.“
AI D( ) Hemo globin cutof fs to define anemia are 8
- year, and 120 g/lfor lio fi/i Chk d, '
lor children 5 11 children 12- 14 ' n 6 59 mo 115
' '
yr - 8/1

E,« « »»« r« » r
" *
fluid unless the child is in shock. our' 0o not 8lve V"
43 (B) Water 2 L, WHO ORS One sachet,
Sucrose 50 g electrolvte /m ,
'
solution 40 ml. ReSoMal contains 37.5 mmol
WH0 ORS sachet contains 2.6 sodium
sodium and 40 mmol potassium
/l
dihydrate 1.5 potas
, g sium chlo ride, 2.9 g
chloride, and 13.5 g glucose.
44 (E) If no complications give Amoxicillin,
trisodium cite
25 mg/kg PO twice daily
complications (shock, hypoglycemia, for 5 days If
hypothermia, skin lesions,
urinary tract infections, or lethargy
/sickly give respiratory or
once daily for 7 days and Ampicillin, 50 mg kgGentamicin, 7.5 mg/kg IV or IM
/ IV or IM every 6 hr for 2 days,
then amoxicillin, 25-40 mg/kg PO every 8 hr
for 5 days. For persistent
or small bowel overgrowth, add diarrhea
metronidazole, 7.5 mg/kg PO every 8 hr for 7
days.
45.(D) Melanocortin 4 receptor gene
mutation causes early-onset severe
obesity, increased linear growth, hyperphagia
, hyperinsulinemia. Its
common known genetic cause of obesity. most
Homozyg ous worse than
heterozygous.
46.(A) An analysis of 9 randomized
controlled trials found that vitamin A
appears to be beneficial in reducing
death or oxygen requirement with no
difference in neurodevelopmental outcomes.
47-(A) Orlistat, which decreases absorption of fat, resulting in modest weight
loss. Complications include flatulence, oily stools, and spotting.
48 (C) TV
watching is often associated with eating, and many highly caloric food
products are
marketed directly to children during child-oriented television
Programs.

56
49,(C) An earlier symptom of vitamin A deficiency is delayed dark adapt,
a result of reduced resynthesis of rhodopsin; this may progress
blindness.
-
50 {E) The mainstay of the initial treatment is aggressive therapy with
saline, often in conjunction with a loop diuretic to further increase
^ d;

talq,
excretion; this is often adequate for treating mild or moderate hype
51.(A) Thiamine is water soluble and heat labile; most of the vitamin
when the rice is repeatedly washed and the cooking water is discarded.
52.(D) Ataxia with isolated vitamin E deficiency ( AVED), a rare
autosor :
recessive disorder, there are mutations in the gene for a-tocopheroi
transfe
^r
protein (TTPA). Because children with AVED do not have symptoms
0r
malabsorption, a correct diagnosis requires a high index of suspicion.
with this disorder are unable to incorporate vitamin E into lipoproteins
Patient'
their release from the liver, leading to reduced serum levels of vitamin E.
before
There
is no associated fat malabsorption, and absorption of vitamin E from
the
intestine occurs normally.
53.(A) There are few natural dietary sources of vitamin D. Fish liver oils have
a
high vitamin D content. Other good dietary sources include fatty fish and egg
yolks . Most children in industrialized countries receive vitamin D via
fortified
foods, especially formula and milk (both of which contain 400 IU/L ) and some
breakfast cereals and breads.
54.(A ) Breast milk has a low vitamin D content, approximately
12 -60 IU/L.
55.(B) Vitamin D deficiency most frequently occurs in infancy
because of a
combination of poor intake and inadequate cutaneous synthesis.
56.(B) Transplacental transport of vitamin D, mostly 25 -D
, typically provides
enough vitamin D for the 1st 2 mo of life unless there is
severe maternal vitamin
D deficiency.
.
57 (C) In chronic kidney disease, unlike the other
causes of vitamin D deficiency*
patients have hyperphosphatemia as a result of
.
58 (B) McCune Albright syndrome, an
decreased renal excretion .
entity that includes the triad of
polyostotic fibrous dysplasia, hyperpigmented
macules, and polyendocrino-
pathy. Affected patients have inappropriately
low levels of 1,25 -D and elevated

——
ALP levels.
.
59 { A) Most infants with rickets of

.
prematurity have no clinical manifestations,
and the diagnosis is based on radiographic and laboratory
nrpmaturitv orn rc i A findings. Rickets of

These infants have poor


linear growth.
si *
and poor ventilation.
There may be classic rachitic findings,
57
ylllh U
*•»
ankles
bO,( A ) Hypervitamiriosis
O Is
occur with long - term high i c sed by e ^
cases are secondary to fntake r ith cessive * **
‘.
°* "
i
Misuse a
'take
,
sri r
supplements,

*
of of
but
other Cas ? o t
°*
nverfortihcation of milk ' <s n .Mcan
npres nSestiQn
^ Z
0ntan
0 ' 7 / been
vitamin supplements
, natIOn of
8 0 7 Vita table sugar
^Ption m n °* t ,
to excessive exposure to
«- W The recommended
IU hr children < j j r
old
suni

,
^
Pper ^
ton ,
ft.'
rnin o ,
)m
gar

Nation
and
darV
-
n s, ,
neve
to «
cc
Gr ent
*
.
/der tai
use *
o
of
«W fhe classic finding
v
'
Z ,°°°
lu «oro/
d
0f e
' '" *
er > Vl
arni m 0,
Secondary

^
levels of 25 -0 ( > ( tamin D into > ta /ce
1
nZTd^ " are 1.000
^
Hyperphosphatemia is ng/ rpU, ts
nephrocalcinosis,
US -0 are usuaily
which
a sp

* * "'*«
c

*^ ^ ° a
raperca
-* '" Cemia csiti
SUpor «
*d

' ultrls° '


nor on refla rCak ed p
' Ur a
unknown.
s
° _ Und - Sur
can ead
^ times Present ,JrPnsingiy ve/s ofto
the ech
'^
* nism is

S8
Chapter 6 Diso rder s
Jid
^ and Elect rolyt e
Questions
KHALID ALAARJI
for surgery, should begin receiving maintr-.'
1A 6-month-old infant
in waiting narv1:.
%

IV fluids within
feeding
A. 4 hr of the last
feeding
B. 6 hr of the last
C. 8 hr of the last feeding
D. 10 hr of the last feeding
E. 12 hr of the last feeding

2. What is the percentile of body weight for height to


calculate the maintenance
IV fluid in an overweight child?
,h
A. 10
th
B. 25
,h
C. 50
D. TS*
th
E. 90

3. The upper limit of maintenance IV fluid in an overweight child (adult -sizee


patients) is
A. 1L/24 hr
B. 1.6L/24 hr
C. 2 L/24 hr
D. 2.4L/24 hr
E. 3L/24 hr

4. Calculating daily
maintenance fluid volume for children weighing 11-20 kg is
A. 100 mL/kg
B- 150 mL/kg
C. 1,000 mL +

E- 1,500 mL + 20 mL 6
/kg for each kg >10 kg

59
which of the following intravenous
solutions iis composed from Na * ( 77

s
eq/ L) Cl'
( 77 meq/ L ) ?
A . Normal saline (0.9% NaCI)
B VS normal saline ( 0.45% NaCI)
C . 0.2 normal saline (0.2% NaCI)
0 y2 Glucose saline
E. Ringer lactate

6. Administration
of 0.2 NS (osmolality=68) peripherally
as maintenance therapy
j5 contraindicated due to risk of
A . hemolysis
B. hyponatremia
C. hypernatremia
.
D brain edema
E . central pontine myelinolysis

7. As maintenance IV fluid, surgical patients during surgery and in the recovery


room for 6-8 hr postoperatively typically should receive
A. normal saline (0.9% NaCI)
B. '/2 normal saline (0.45% NaCI)
.
C 0.2 normal saline ( 0.2% NaCI)
D. /2 glucose saline
E. 5% dextrose

8. The BEST solution for replacement of ongoing losses in a 3-year -old child
presented with diarrhea and a limited ability to take oral fluid is
A. Ringer lactate
B. normal saline + 10 mEq/ L KCI
.
C 'A normal saline + 10 mEq/L KCI
D. NS + 30 mEq/ L sodium bicarbonate + 20 mEq/ L KCI
E. D5 /2 NS + 30 mEq/L sodium bicarbonate + 20 mEq/ L KCI
'
9. A 15-year-old girl admitted to the hospital for evaluation; investigation
( 82 mEq/ ,
reveals serum Na+ (131 mEq/ L), serum K+ ( 2.6 mEq/L), serum Cl
m
BUN ( 42 mg/dl), and serum creatinine (0.9 mg/dl), pH ( 7.52), HCO ^
pC02 ( 51 mm
Hg ) and urine for (Cl ) ( 25 mEq/ L)
Of the following, the MOST likely diagnosis is
A. frequent emesis
60
B. chloride-losing
diarrhea
C . low chloride formula
0 . Gitelman syndrome
E , cystic fibrosis

of the following is a feature of mild dehydration ?


10. Which
A. Thirst
B . Tachycardia
C . Lethargy
D . Sunken fontanel
E. Decreased tears

?
11. Which of the following is a clue to severe dehydration
A. Cold and mottled skin
B. Oliguria
C. Sunken eyes and fontanel
D. Dry mucous membranes
E. Irritability

12. An 18'hour - old newborn with normal vaginal delivery without complicate
admitted to NICU with repeated apneic attacks and hypotonia with history of
severe maternal preeclampsia treated with IV magnesium and hydralazine; 0/ f
floppy, decrease deep tendon reflexes, and hypotension. ECG shows prolonged
PR interval, QRS complex, and QT interval.
Of the following, the MOST likely diagnosis is
A . perinatal asphyxia
B . early onset sepsis
C. hypermagnesemia
D. hypocalcemia
E. Werdnig- Hoffmann disease

13. A 15- year - old girl presents to


emergency unit with breathlessness and chest
pain; ECG and chest radiograph
are normal; yesterday she was notified with
school failure,
0
fr " e; the MOST likely arterial blood gases in this condition is
A. pH (7.09), HC03- ( 28 mEq
/L), PC02 ( 68 mm Hg)
B- pH ( 7,12 ), HC03 ( 14 mEq
~

/L), PC02 (65 mm Hg)


C. pH (7,35 ), HC03 ( 20
"

mEq/L), PC02 ( 45 mm Hg )
D. pH (7.55), HC03- (16 mEq/L) prn , ,

. ] «8)
,4. An 11 month
-
^
E. pH (7.52 ) HC03 ( 40 rnEq/ PCQ

-old babv. who has small


bow »i ,
brought to emergency unit with seizure - Exam reSeC ion at of 2 month,
and steatorrhea; serum calcium (6.5 mg/d|) - 7' ' "kWi,
° h failure t0 thrive
lv
calcium gluconate 10 ml but still no respond' Err shows 'fusion 0f io%
wave and lengthening of the ST segment fattening of the T
Of the following, the MOST likely explanation is
A . incorrect dose
B. hypoparathyroidism
C. hypomagnesemia
D. hyperphosphatemia
E . hyponatremia

15 An 18-month-old child presents with history of vomiting, diarrhea , and


decreased urine output. Examination shows irritable child with capillary refill of
2 sec, dry mucous membranes, and tachycardia.
Of the following, the MOST likely percent of dehydration is
A. <3%
B . 3-6 %
C. 7-9%
D. 10-12%
E . >12%

vomiting, diarrhea, and no urine


16 . A 12 -month- old infant presents with
shows depressed consciousne ss with capillary refill of 4 sec
output. Examination
and very sunken eyes.
therapy is
Of the following, the BEST initial fluid
A. 0.9% NS
B. 'A NS
C. % N S
D 0.2 NS
E. yaGS
d vomiting ?
chiid with isolate
voided in a
17. Which of the following shou
A. Isotonic NS
B. Ringer lactate
62
C. KNS
D. Yi GS
E. 0.2 NS
care nit for management of Guillam-
girl admitted to intensive develops severe headachy
18. A 7- year-old 7
th
day , the patient
syndrome , at morning of afebrile , no neck stiffness, and blooa
Barre
vomiting; O/E well
hydrated ,
K (4.1 mEq/L), serurr
and repeated * (120 mEq / L ), serum
mm Hg; serum Na ), serum create ne
pressure 100 / 60
(98 mg / dl ), BUN 20 mg/ dl
glucose
chloride (75 mEq/L), blood (80 mEq/L).
(0.5 mg/dl), and urine
sodium
likely explanation of this condition is
Of the following, the MOST
A. meningitis
B. side effect of IVIG hormone secretion
antidiuretic
C. syndrome of inappropriate
D. cerebral salt wasting
E. acute tubular necrosis

resent with sudden onset of rapid deep breathing and


19. A 4-month-old girl p
Na + ( 145 mEq /L), serum K+ (6 mEq/L), serum Cl ( 90
mild dehydration; serum j
and PC02 ( 23 mm Hg)
mEq/L ), pH ( 7.11), HC 03 ( 10 mEq /L ), lactate 4 mmol /L
Of the following, the MOST likely cause is
A. diarrhea
B. distal ( type I) RTA
C. proximal (type II ) RTA
D. urinary tract diversion
E. isovelaric acidemia

20. Which of the following is a risk factor for hyponatremic dehydration n


children with diarrhea ?
'
A . Consumption of diluted formula
B. Inability to take oral fluid
C. Poor thirst mechanism
D. Intractable emesis
E . Hyperpyrexia

21. Hyponatremia is defined


as serum sodium less than
A. 125 mEq/L
B. 130 mEq/L
c. 135 mEq/L
D. 140 mEq/L
E. 145 mEq/L

22. In hyponatremic dehydration, the serum sodium SHOULD NOT be corrected


more than
A. 10 mEq/L/24 hr
.
B 12 mEq/L/24 hr
C. 14 mEq/L/24 hr
D. 16 mEq/L/ 24 hr
E. 18 mEq/L/ 24 hr

23. A bottle fed 18- month-old boy presents with history of vomiting and
diarrhea. Examination shows severe degree of dehydration; serum Na+ (126
mEq/L), serum K+ (4.5mEq/L), pH (7.25), HC03 (14 mEq/L), and PC02 30 mm
Hg; he received IV fluid therapy and after 12 hours, the patient became
confused and agitated. Serum electrolytes were done revealing serum Na+ (145
mEq/L), serum K+ ( 3.7mEq/L), serum Ca+ (8.7 mg/dl), and blood glucose ( 98
mg/dl).
Of the following, the MOST likely cause of his deterioration is
A. cerebral edema
B. central pontine myelinolysis
C. arrhythmias
D. brain herniation
E. intracranial hemorrhage

unit with
24. An 11-month-old bottle fed infant was brought to emergency
with excessive
tonic -clonic seizure. He complains from vomiting and diarrhea
water intake for the last 3 days. Examination+
shows moderate degree of
mEq/L), serum Ca+ ( 8.7
dehydration; serum Na ( 116 mEq/L), serum K ( 3.8
+

mg/dl), and blood glucose (98 mg/dl).


Of the following, the FIRST line of treatment is
A. 0.9% NS
B. 3% NS
C. Ringer lactate
D. % NS
E. 0.2 NS

64
25. A 10-month-old infant presents with frequent loose bowel motions
excessive thirst over the past 24 hours. Examination shows restlessness, su ’

eyes, and skin pinch goes back slowly. ^


Of the following, the MOST appropriate therapy is
A. IV normal saline (0.9% NaCI)
B. IV Yi normal saline + 10 mEq/L KCI
C. IV D5 % NS + 30 mEq/L sodium bicarbonate
D. Standard oral rehydration solution (ORS)
E. Low osmolarity ORS

^
received IV fluid therapy and after 12 hou * feveals; serum Na+ (l5$
with decerebrate posture, repeat serum
mg/dl ), and blood Sluc se (9g
°
mEq/L ), serum K+ ( 3.7 mEq/ L), serum Ca + (8.7
mg/dl). . j
his deteriora 10
Of the following, the MOST likely cause of
A . cerebral edema
B. central pontine myelinolysis
C. arrhythmia
D. renal failure
E. intracranial hemorrhage

vomiting and diarrhea for the


27. A 14-month- old bottle fed girl presents with
last 24 hours. Examination shows an irritable child with doughy skin
and
capillary refill of 3 seconds; serum Na ( 160 mEq/L), serum K * (4.1 mEq/L), BUN
*

( 110 mg/dl), and serum creatinine ( 1.3 mg/dl).


Of the following, the BEST initial fluid therapy is
A. Isotonic N S
B. 'A NS
c . y* NS
D. 0.2 NS
E. Ringer lactate

28. A 5-year-old child admitted to


polyuria, polydipsia, and dehydrat‘
emergency unit with rapid deep breathing,
ion for the last 2
mEq/L ) and serum
osmolality
320 mOsm/kg.
weeks; serum Na * (U
°
65
following, the MOST likely cause of hyponatremia is
Qf the
A. leukocytosis
B. increased
blood urea
C . acidosis
D. increased
I blood glucose
E. water
depletion

A 9- month-old girl
presents with vomiting and diarrhea for the last 24 hours.
^
Examination shows an irritable child with doughy skin and capillary refill of 3
Na’ ( 168 mEq L), serum Kf ( mg dl ), and
seconds; serum ( 1.3 mg dl); / patient 4.1 mEq/L , BUN 110 /
) (
serum creatinine mL / the shows improvement on two shots of
isotonic N S (each 20 kg / over 20 min) and has passed urine.
Of the following, the NEXT step in management is administration of
A . oral rehydration solution
B . 5% dextrose + A normal saline over 24 hr
C . 5% dextrose + A normal saline over 48 hr
0. 5% dextrose + A normal saline over 24 hr
E. 5% dextrose + A normal saline over 48 hr

30. A 9-month-old girl presents with rapid deep breathing and mild
dehydration; serum Na+ (132 mEq/L), serum K+ {3 mEq/ L), serum Cl (114
mEq/ L ), pH (7.05), HC03 ( 12 mEq /L), and PC02 ( 25 mm Hg).

Of the following, the MOST likely cause is


A. lactic acidosis
B . diabetic ketoacidosis
C . renal tubular acidosis
0. renal failure
E. proprionic acidemia

31. In acute respiratory acidosis; each 10 mm Hg increase in PC02 is


compensated by increase plasma bicarbonate of
A. 1
.
B 1.5
C. 2
D. 2.5
E. 3

66
, . failure to thrive, has past
his,
th-oid infant of repeated
is dehydration,%
vomiting,

^
bec
32. A hosp tal 3
dmissions stipation; abdominal ultrasound
frequent ' output wi / ,I-
urine
rS
^
increa
phroc
nt
... serum K+ ( 2mmoi of this patient js
^

-—
Of the
A . pH 0 09 |- .°
14 mEq/L) K 2
(6S mm 8

c. P %Z m
D. P
£. P ISSSSS^'
, , mon
of a 6 ontth.0id infant with failure to thrive reveals serum Nb

^
33. [ valua tion
c|. (g2 mEq/l), BUN (32 mg/dl), a„.
* ^
gas analysis showed pH (7.52), HCOr (3s
serum creatinine
( 0.6 mg/ dl) . Bloo d
mEq/l), and PC02 (46 mm Hg).
Of the following, the MOST helpful test for the diagnosis of this condition is
A. urine for Na *
'

B. urine for Cl
C. urine for K *
D. serum chloride
E. serum calcium

34. What is the FIRST electrocardiogram (ECG) finding


in hyperkalemia ?
A. Peaked T wave
B. ST-segment depression
C. Increased PR interval
D. P wave flattening
E - QRS complex widen
ing

* examination lt[
weakness
strength; ECG
interval .
^ shows peaked
hoCkle "
.Ce
- ^
d SeaSe presents
'
e ST segment
with generalized muscle
With deCreaSed
depression, and increasedF?
° the blowing,
A - dialysis
the FIRST Itne of ,
therapy is
1 insul
k
' |n and
3yexalate
glucose
1v
calcium
.
E albuterol nebulizer

36. What is the MOST like!y cause of


A. Emesis hypokalemi13 and
B . Diarrhea "
Mabolic acidosis ?
C . Aldosterone excess
D. Use of diuretics
E . Bartter syndrome

37. A 6- year-old girl presents to eme


dehydration; blood glucose (560
Of the following , the EXPECTED
me/ dT"^ With deep rapid
breathing and
arterial blood gas analysis
A. pH ( 7.05), HC03 (12
"
is
mEq/L), PC02 (25
B. pH ( 7.09), HC03 (28 mEq
~ mm Hg)
/L), PC02 (68 mm
Hg)
C. pH ( 7.12 ), HC03 (14
'

mEq/L), PC02 (65


D. pH ( 7.35), HC03 - (20 mEq mm Hg)
/ L), PC02 (45 mm Hg)
E. pH ( 7.45), HC03‘ (32 mEq
/L), PC02 (65 mm Hg)

68
Chapter 6
Fluid and Electrolyte Disorder s
Answers
KHALID ALAARJI
.
1.|C) A normal teenager
morning procedure does
,g
u iis
who .ven
not require
nothing
noth !
*
by mouth (NPO ) overnight f 0r a
^ , a

or 18 hr without o n contra a 6'


adolescent can easily tolerate 12
begin receivi r of
mo old child waiting for surgery should
become dehydrated more quickly than older patient s .
the last feeding. Infants
on weight do overestimate the
2.(C) Calculations of maintenance IV fluid based
overweight child, in whom it is better to ase t e ca culations
water needs of an
by using the 50 percentile of
on the lean body weight, which can be estimated
body weight for the child's height.
3.{ D)
4.{C)
5.( B >
6.( A) A normal plasma osmolality is 285- 295 mOsm/ kg. Infusing an IV solution
peripherally with a much lower osmolality can cause water to move into red
blood cells, leading to hemolysis. Thus, 0.2 NS ( osmolality - 68 ) should not be
administered peripherally, but D5 0.2NS ( osmolality = 346) or D5 'A NS + 20
mEq/L potassium chloride ( KCI ) with an osmolality of 472 can be administered .
7-(A) Surgical patients typically receive isotonic fluids ( NS, LR ) during surgery
and in the recovery room for 6-8 hr postoperatively; the rate Is typically
approximately two-thirds the calculated maintenance rate, with
dextrose added
if clinically indicated.
8.( E) Average composition of diarrhea;
sodium : 55 mEq/L, potassium: 25 mEq/ L
DTMbNSa 301
e q/
, caCh °nate + 20 mEq/of
LHApPrhO l reptacement
L kc,)- Ave
ongoing losses; Solution
:
( +
rsolmdrmb' ?°potassium: 10 mEq/ ^ c°mpmEq
mEa /|

Approach to replacement of '


°sition/L;
U chloride: 90
ongoing losses; Solution (
KCI ). normal saline + 10 mEq/ L
9 (D)
Chloride-Responsive
Cause5 0f Metabolic Alkalosis (
LegAL— urinary
chlorkl* <15
'
gastric losses ( Emesis or Nasogastric suction)
(loop or thiazide )
Diureticslosing diarrhea
Chloride-
formula
Low chloride
Cystic fibrosis
posthypercapnia
Causes of Chloride-Resistant Metabolic Alkalosis (
mJqA)
Normal Blood Pressure Hi9h Blood P
urinary chlorid >20 .
adenoma or hyperplasia leisure
Adrenal Gitelman syndrome
Glucocorticoid - remediable
Bartter syndrome
aldosteronism AD hypoparathyroidism
Renovascular disease EAST syndrome
Renin - secreting tumor Base administration
17a-Hydroxylase deficiency
Cushing syndrome
Liddle syndrome
~
im Mild dehydration (<5% in an infant; <3K i Told hild or adult)-
^ ^
Normal or increased pulse; decreased urine output; thirsty; normal physical
findings
11 ( A ) Severe dehydration (>10% in an infant; >6% in an older child or adult):
Peripheral pulses either rapid and weak or absent; decreased blood pressure;
no urine output; very sunken eyes and fontanel; no tears; parched mucous
membranes; delayed elasticity (poor skin turgor); very delayed capillary refill (>3
sec); cold and mottled; limp, depressed consciousness.
is almost always secondary to
are

receiving IV magnesium for preeclamps


not appear until the plasma magnesium >4 5 mg/dL. Hypermagnesemia
|ar junction, pr in
inhibits acetylcholine release at t e muscu
at high c nce
J*
^ ^
curs ’
hypotonia, hyporeflexia, and ^
^***«^*^££,*^-^**"^‘
jotia
The neuromuscular effects may " ated by
s
‘ poor suck.
Direct CNS depression causes sleepiness; infan use 0f
W
Elevated magnesium values are
vascular dilation, which also causes
70
13.(D) Respiratory alkalosis due to hyperventilation and washout of
C
Hyperventilation may be secondary to an underlying disease that cause , 0
P iri
stress, or anxiety. In psychogenic hyperventilation or in panic attacks, ^
no disease process accounting for the hyperventilation. This disorder mav rjr
there

^^
J
in a child who has had an emotionally stressful experience. Alternatively , lt
be part of a panic disorder, especially if there are repeated episode
hyperventilation.
14.(C) Hypomagnesemia causes secondary hypocalcemia by impairing
release of PTH by the parathyroid gland and through blunting of the
tissue
^
response to PTH. Thus, hypomagnesemia is part of the differential diagnosis
of
hypocalcemia . It usually occurs only at magnesium levels < 0.7 mg/dL .
The
dominant manifestations of hypomagnesemia are caused by hypocalcemia
tetany, presence of Chvostek and Trousseau signs, and seizures. However,
with
severe hypomagnesemia, these same signs and symptoms may be present
despite normocalcemia .
ECG changes with hypomagnesemia include flattening of
the T wave and
lengthening of the ST segment. Arrhythmias may occur,
almost always in the
setting of underlying heart disease. Gastrointestinal
and renal losses are the
major causes of hypomagnesemia . Diarrheal fluid
contains up to 200 mg/L of
magnesium; gastric contents have only
approximately 15 mg/L, but high losses
can cause depletion. Steatorrhea causes magnesium
loss as a result of the
formation of magnesium-lipid salts; restriction of
dietary fat can decrease
losses.
15.(B)
S Mild dehydration ( < 5% in an
infant; <3% iin an
Normal or increased pulse; decreased older child or adult):
urine output;
physical findings thirsty; normal
Moderate dehydration (5- 10% in j an
infant; 3-6% in an
adult ): Tachycardia; little or no urine
output;
older child of
eyes and fontanel; decreased tears * dry irritable/lethargic; sunken
mucous membranes; mild delaV
in elasticity (skin turgor ); delayed capillary
refill (>1.5 sec)
S Severe dehydration (>lo% jn an ; cool and pa e
adult):Peripheral
P'
,
pulses either rapid
infant
urine oulput; very
; >6% in
and weak or
an older child or
absent; decreased
'
plood Pr
^
mucous membranes, delayed sunken eyes
tears; P rc' and fontane|; no

^
very deiaYeu capiUarY refill (>3 sec); cold and
consciousness
elasticity (poor
skin turgor);
mottled; limp, depressed

71
— —
16.( A )
or LR); 20 mL/kg
over 20
Calculate 24 hr fluid needs m

» „„ ,
/
mEq/lKC1 * „
o
* 6 5% dex r se NS >
* °
.
jJ fB)
Replace ongoing losses as th
In a child with a known or
,
vomiting), LR or P|asm ut
, °° ,e Metabolic
.
nVbab
CCUr

Uld not be used


aM, ,
JU *«
* *
, «- '« „
1 e‘ < w b
isolated /l ° „, "*' »e or
worsen the alk i0
acetate wouldNS shoo, since » ,
» tacea
U
« cs *
ssxsssrsr
trauma,
fprtjon,
,
-
'
asthma, positive pressure ventilation) and malignant tumors
*
' 0ut
,u"g disease
(producing ADH
) are other potential causes.
^
Diagnostic Criteria for Syndrome of Inappropriate Antidiuretic Hormone
Secretion thyroid insufficiency - Heart failure,
V Absence of: Renal, adrenal, or
- Dehydration
nephrotic syndrome, or cirrhosis - Diuretic ingestion
kg (usually >plasma)
Urine osmolality >100 mOsm/
kg and serum sodium <135 mEq/L
/ Serum osmolality <280 mOsm/
/ Urine sodium >30 mEq/L
of Hyponatremia with

.
wasting " and correction
Reversal of "sodium
water restriction Gap
19.(E) Metabolic Acidosis with
Increased An on
/ Lactic Acidosis
, ia
Hypoxemia , Severe anemia)
• Tissue hypoxia ( Shock
• Liver failure
• Malignancy
• Intestinal bacterial overgrowth
• Inborn errors of metabolism
S Medications ( Metformin, Propo
jnezolid)
r

, ctarvation
.ketoacidosis, Alcoholic

Ketoacidosis ( Diabetic ketoacidosis


ketoacidosis ) paraldehyde )
s Salicylate,Toluene
20.(A)

JJ 2
[’
Even though central pontine myelinolysis (CPM) is rare in , ped att
avoid correcting the serum [Na+] by >10 mEq
to /i/ 24 h;
patients, it is advisable
or >18 mEq/L/48 hr.
23.(B) With all causes of
hyponatremia, it is important to avoid overly
correction, which may cause central pontine myelinolysis
(CPM). This syndrom^ e.
which occurs within several days of rapid correction of
hyponatremia , produces
neurologic symptoms, including confusion, agitation, flaccid or spastic
quadriparesis, and death. There are usually characteristic pathologic and
radiologic changes in the brain, especially in the pons, but extrapontine lesions
are quite common and may cause additional symptoms Despite severe .
.
symptoms, full recovery does occur in some patients Even though CPM is rare
in pediatric patients, it is advisable to avoid correcting the serum [Na+] by >IQ
mEq/ ty 24 hr or >18 mEq/L/ 48 hr.
24.(B) Patients with hyponatremia can have severe neurologic symptoms, such
as seizures and coma. The seizures associated with hyponatremia generally are
poorly responsive to anticonvulsants. The child with hyponatremia and severe
symptoms needs treatment that will quickly reduce cerebral edema. This goal is
best accomplished by increasing the extracellular osmolality so that water
moves down its osmolar gradient from the +ICS to the ECS Intravenous .
hypertonic saline rapidly increases serum [Na ], and the effect on serum
osmolality leads to a decrease in brain edema. Each mL/kg of 3% NaCI increases
the serum [ Na+] by approximately 1 mEq/L. A child with active symptoms often
improves after receiving 4-6 mL/kg of 3% NaCI .
25.(E) The World Health Organization defines some dehydration as the presence
of two or more of the following signs: restlessness/irritability, sunken
eyes,
drinks eagerly, thirsty, and skin pinch goes back slowly. Infants and children with
some dehydration need rehydration therapy with ORS: ORS, 50-100 mL/kg over
3-4 hr. Continue breast feeding. After 4 hr, give food every 3-4 hr for children
who normally receive solid foods. Item D is old ORS recommended in 1976 and
item E is new ORS recommended in July, 2001c,
26.( A) The goal is to decrease the serum [Na + ] by <10 mEq/L every 24 hr.
Idiogenic osmoles are generated within the brain during the development of
hypernatremia; they increase the osmolality within the cells of the brain,
providing protection against brain cell shrinkage caused by movement of watef
out of the cells and into the hypertonic ECF. Idiogenic osmoles dissipate slowly
during the correction of hypernatremia. With overly rapid lowering of he
*
'T*"*,"°*' * ECS osmotic
^
be created that causes hyf r
gradient may brai water e i3 3n
ells < the
"' Pr0ducin« «rebral edema ' r m
° edema can range from secures to brain
cerebral <
imt 0
the
«°death. resultant
h the
.(A) The initial resuscitation f h y p L 3

iteration of the
0
intravascular volume 20 S "
*20 dehydration
lactate should lsot S7
not be used
Ringer because it
is e hvo ,min mU
hypotonic than Ms °
S
c3Use to rapid a decrease in the serum [Na *i .
^
and may
are necessary . uses
28 01
( Hyperosmolality , as may occur with hynerei
moves ^ • Causes 3
because water down its osmotic gradient
diluting the ( Na ]. However, because the
' , lo (Na 'l
CS int0 the*
£CS,
manifesta , ,
result of the low plasma osmolality, patients with h^ ° P°natrem a are a
h
'
.T ,
hyperosmolality do not have symptoms of hyponarrlTI '3 resu tine ,
the et,0 0«V of
the hyperosmolality resolves, such as hyperglycemia in a
moves back into the cells, and the (Na *) rises to its ”tr
sucrose, a component of intravenous immune globulin
cause hyponatremia because of hyperosmolality.
(
^ ,
mel itus water

MGIS* - '
'
P paratlons may
r
°
29.( C ) Treatment of Hypernatremic Dehydration
/ Restore intravascular volume:
Normal saline: 20 mL/kg over 20 min
(repeat until intravascular volume restored)
/ Determine time for correction on basis of
initial sodium concentration:
•(Na) 145-157mEq/L:24 hr
•(Na) 158- 170 mEq / L: 48 hr
•(Na) 171- 183 mEq/L: 72 hr
•(NaJ 184- 196 mEq/L: 84 hr
Administer fluid at constant rate over time for correction: Typical fluid:
5% dextrose + half-normal saline ( with 20 mEq/L KCI unless
contraindicated )
v Typical rate: 1.25-1.5 times maintenance
30.(C) Metabolic Acidosis with Normal Anion Gap
Diarrhea
Renal tubular acidosis (RTA )
• Distal ( type I) RTA
• Proximal (type II) RTA
• Mixed (type III) RTA
• Hyperkalemic (type IV) RTA
• Urinary tract diversions
74
• posthypocapnia
• Ammonium chloride mt*
®
J Dlasma bicarbonate increases b y - r.
10 H
*32“.(*E) In"Bartte
" ‘r s . seve t top
here m .
(usurtlr <2.5
**"*
ja and increased urinary chloride with1 nOrrp
metabolic alkalosis, hypercalciuria
in a mino rity of patien
; ^
blood pressure; hypomagnesemia is seen ts but *
i

more common in Gitelman syndrome . ,. .


3J.(B) Measurement of the urine [CI- ] is the most helpf ul test in differentiating
among the causes of a metabolic alkalosis. The urine [C! ] is low in patients with
a metabolic alkalosis resulting from volume depletion, unless there is a defect m
renal handling of CI-. The urine [CI-] is superior to the urine [Na+] in assess
of volume status in patients with a metabolic alkalosis, because the norma ^
renal response to a metabolic alkalosis is to excrete bicarbonate Bec
ause .
bicarbonate is negatively charged, it can be excreted only with a cation,
usually
Na+ and K+. Therefore, a patient with a metabolic alkalosis may excre
te Na+ ir
the urine despite the presence of volume depletion, which normally
causes avic
. -
Na+ retention The urine [CI ] is usually a good indica
tor of volume status, and ft

differentiates Cl resistant and Cl- -responsive causes of a
metabolic alkalosis
34.(A)
3S.(D ) The goal of treatment of hyperkalemia
is to stabilize the heart to prevent


life-threatening arrhythmias with
calcium which stabilizes the cell
heart celMreventing arrhythmias membrane of
; it is given intravenously
and its action is almost over a few minutes
immediate. Bicarbonate causes potass
ium to move
1

ST ,
“ “ *r •"
n cellul
* * « v •««
with glue
* w hyp «
0giy d ja ' *
b,
r3
rV ^ must be "
8 giw
works within 30 min
Nebulized “
»ih ° natl
n of insulin and glucose
receptors
(
dialysis
, leads
and
to rapid
kayexaiate )
intra cellu I
lar ? ^ Stim lation of Pl-adrenergic
of K“ whi e other measures
|
. The combination are
M (B)
f hipokll T *
remove *
to K f '

dlarrtlea
.characteristic^d distal
0

RTf,
and proximal
A .
A
^
tabo lic
acidosis is characteristic
of emesis
or nas08£> -i« ' Concurrent metabolic alkalosis is

- ^ °
d nretics, and

f
'< >A , !
sul n-depende
"
Bamer and 6 elm
'°, C SSes aldo
SVndron 8 s- ' sterone excess, use of
hVPerglycemja and d bete
K
s lellitus, i
3 Cid C3
Uses
he metabolic
DKA t>
" inadequate insulin leads to
addons * °acetic acid and &-hydroxybutyr*c
'
‘ aCet
- J

75
ter 7
ChaP
Pediatric Drug Therapy
Questions
. N. ABOOD
Which of the following terms is BEST
1.
variations that give rise to diffJ
interindividual fences in
the study of
i response todrugs ?
genetic

A. Pharmacoproteomics
B. Pharmacokinetics
C. Pharmacogenetics
0 . Pharmacodynamic
.
E Pharmacogenomics

2.Which of the following drug responses is due to the effect of gene


polymorphism ?
A Hyperpyrexia following aspirin overdose
B. Prolonged paralysis after succinylcholine administration
C. Antipyretic effect of paracetamol
D. Bradycardia after administration of small dose of atropine
E . Hypoglycemia associated with insulin therapy

3. Drug biotransformation reactions are conveniently classified into 2 main


types, which occur sequentially and serve to terminate biologic activity and
enhance elimination, phase I and phase II reactions .
Which of the following is phase I reaction?
A. Glucuronidation reaction
B. Conjugation with glutathione
C. Sulphation reaction
-
D Oxidation reaction
.
E Conjugation with glycine

Which of the following is the major consequence of pharmacogenetic


^P lyniorphisms
° A- in drug metabolizing enzymes ?
Increased drug toxicity
increased drug metabolism
76

J
C. Increased drug clearance
D. Decreased drug efficacy
E. Decreased drug accumulation

5. A case of acute lymphoblastic leukemia , on maintenance treatment repirw


has recurrent attacks of severe myelosuppression that required stopp ,
chemotherapy for a long time. On screening, the patient was found genet ^^
C<

having low TPMT (Thiopurine S -methyltransferase) activity. TMPT js cyt


enzyme that catalyses the S- methylation of sulfur-containing drugs,
reducing their cytotoxic effects on bone marrow. For this reason, which
following anticancer drugs should be removed from the regime ?
hen '
of th
^'
C
°

A . Methotrexate
B. Vincristine
C. Prednisolone
D. L- asparaginase
E. 6-thioguanine

6. Polymorphisms of the |32- adrenergic receptor gene ( ADRB2)


are associated
with variable responses to bronchodilator drugs.
Of the following, the MOST likely claimed drug is
A. ipratropium
B . salbutamol
C. theophylline
D. aminophylline
E. tiotropium

7. Drug bioavailability represents the


fraction of the drugs that reach the
systemic circulation after
administration. It is always 100% if the drug is given bv
IV route.
Which age group has the LEAST bioavailabilit
y of drugs given by IM route?
A. Neonates
B. Infants
C . Toddlers
D. Preschoolchildren
E. School children

8. Aminoglycoside
antibiotics have a higher volume of distribution in neona
and infants (0.4- 0.7 l
/kg) than in adults (0.2-
0.3 L/kg) because
^
77
the percentage of total body water is lower
in neonates
0. adults have higher percentage of intracellular anti infants
the percentage of intracellular water is higher water
p adults have higher percentage of
in neonates ,an,i
E . the percentage of extracellular
extracellular wate
water is higher
? m
'
in neonates
and infants
h of the following age groups has

^ !
9 the HIGHEST drug absorption
ad stration through rectal route? after
A Neonates
B. Infants
Q Toddlers
P preschool children
E Schoolchildren

10. Which of the following is the primary organ responsible for


the elimination
of drugs and their metabolites in pediatric age group?
A. Liver
B. Kidney
C. GIT
D. Skin
E. Lung

11. Which of the following dosing interval is recommended for gentamicin in


young children who have serious infections ?
A. 4 hours interval
B. 6 hours interval
C. 8 hours interval
D. 12 hours interval
E. 24 hours interval
is used for
12 . In therapeutic drug monitoring ( TDM), plasma drug concentration
following rugs a
dose individualization for a given patient. Which of the
frequently requiring TDM?
A . NSAIDs like aspirin
B. Beta-blockers like atenolol
C. Anticonvulsants like phenytoin
D. Antibiotics like penicillins
E . Anticoagulants like warfarin
78
fo( lowing administration devices is MOST?convenient t 0
13. Which of the
adminis

ration of liquid drug formulations X
accurate peroral
A. Kitchen teaspoon
B. 5 ml syringe
C. Hypodermic syringe
D. 5 mL dosing spoon
E. Medication bottle cap

by inhalation, which of the following fact .


14. Regarding drug administration 0 (

drug absorption ?
has the LEAST effect on pulmonary
A. Drug particle size
B. Chemical stability of drug molecule in the lung
C. Internal airway diameter
D. Active versus passive drug delivery to tracheobronchial tree
E. Pulmonary surface area

15. A 24-month- old girl presents to the outpatient clinic with poor asthma
control, she received montelukast (4 mg chewable tablets) daily for the last 3
months without satisfactory control of symptoms, if you decided to star
inhaled corticosteroids for her, which of the following devices you will choose?
A. Metered-dose inhaler (MDI) alone
B. Ultrasonic nebulizer
C. MDI + spacer with mask
D. MDI + spacer without mask
E. Jet nebulizer

.
16 Drug-drug interactions can be
defined as alteration of pharmacokinetic
and/or pharmacodynamic properties
of drugs when >2 drugs are administered
to a patient at the same time.
Which of the following drug properties
A.
MOSTLY altered by drug interactions ?
Absorption
B. Distribution
C. Metabolism
D. Elimination
E. Receptor binding

17. Drug interactions due to physicochemical


when combined are ootpnti=,ii .V p incompatibility of 2 medicate4
angerous. In this
regard, which of the following
n
*7
) osp0rin
antibiotics should be avoided in
neonates receiving iv calcium-
c
A. cefixime
B ceftriaxone
c cefotaxime
D cefuroxime
E. Cefaclor

IS Which of the following drug-drug interactions 15i of antagonism type


?
A Fentanyl and midazolam
B . Penicillins and aminoglycosides
C Paracetamol and codeine
0. Naloxone and pethidine
E Penicillins and probenecid

19 Which of the following drugs can increase the hemorrhagic risk in


patients
taking warfarin by displacement of warfarin from plasma protein
binding?
A . Ibuprofen
B. Phenobarbital
C. Erythromycin
D. Cimetidine
E. Ketoconazole

20. Regarding the safety of dietary supplements, which of the following


agents
can exacerbate gastroesophageal reflux ?
A. Ginger
0- Magnesium
C. Peppermint
D. Chamomile
E. Zinc

Specific adverse drug reactions ( ADRs) occur at a much greater frequency in


Infants and children than in adults. In this regard, which of the following ADRs is
Specific for
infants <2 years of age?
A - Aspirin-associated Reye syndrome
Cefaclor-associated serum sickness-like reactions
C Lamotrigine
induced cutaneous toxicity.
Valproic acid-induced hepatotoxicity
rtrt
E. Tetracycline induced dental
discoloration

Which of the following is the BEST approach that can be used to \


22 .
pediatric patients of all age g tii
therapeutic compliance and adherence in
A . Using positive reinforcement
B. Improving drug formulation
quality coupled with positive reinf0rr
C. Ensuring that single caregiver is
single child
D. Repetitive education of caregivers and older children
E . Increasing caregiver vigilance and repetitive education with
^
responsible for the medicat ''*
° n C!

reinforcement

23. A 3- year-old boy presents with fever, mucopurulent nasal discharg,


productive cough, and wheezing for the last 48 hours without history of chro^ .
respiratory problem . The boy is a candidate for elective surgery to correct h -
left inguinal hernia and hydrocele next week.
What would be your decision regarding safety of general anesthesia ?
A . Treating chest infection and advice for using IV hydrocortisone befcr
-
induction of GA
B . Postpone the surgery for 6 wks
C. Adding prednisolone lmg/kg for 5 days to the treatment regime c;
chest infection
D. Postponed the surgery till symptoms subside
E. Treating chest infection and reevaluate the patient after 1 wk for fitness
to GA

24. A newborn presents with imperforated anus


36 hours after normal vagina
delivery at home, following stabilization
the pediatric surgeon was consulted
who prepared him for surgery.
Of the following, the MOST
critical preanesthetic medication is
A . vitamin K
B. prophylactic antibiotics
C. atropine
D. H2 blockers
*

E. benzodiazepines

25. A 4-year- old girl presents to the outpatient clinic with


temper tantrums, and
nighttiime crying as
oi
frequent enu(eS'
well as fear of strangers 3 day5 a e
^ ^
She was

rnad® - dv ,operations to
ral changes
W3 S
is the BEST
measure to be taken i,
n order
°
What changes In the next surgery ? to reduce Postoperative
ioral
Using IV midazolam as preanesthetic
A dedication
0 Parental presence during induction
c Preoperative psychological preparation program
Q Parental presence during induction and usi
USlnB «l midarolam
t.Preoperative psychological preparation]° Program and
midazolam using orai

preoperative fasting guidelines,


26. Regarding breast milk may be
up to gtven to
infants
A . 2 hours before surgery
5. 4 hours before surgery
C 6 hours before surgery
D. 8 hours before surgery
E , 10 hours before surgery

27. Benzodiazepines are sedative-anxiolytic agents which are very effective as


premedications, but they can severely depress respiration, especially when
administered with
A . propofol
B. barbiturates
C. opioids
D . chloral hydrate
£ ketamine
,

28. A 12-year-old boy presents to the E/D with severe abdominal pain mostly
localized over a sutured midline incision, he underwent laparotomy hefs°er ®®
o
hours for removal of liver hydatid cyst which necessitated the use
morp me
muscle relaxant. The boy was received the usual dose of
,
surgery which controlled his pain initially, in addition to a • .| extremities ,
also suffering from moderate headache and generalle P
^^
responsible for condition is
Of the following, the skeletal muscles relaxant
A . atracurium
82
1
B tubocurarine
Q pancuronium
D. succinylcholine
E vecuronium
the following general anesthetic agents represent
the
">Q Which of
standard for providing analgesia ? *

A. Nitrous oxide I
B. Midazolam
C. Ketamine
D. Thiopental
E. Fentanyl

swelling, and obvious deformity


30. A 5-year- old boy presents with severe pain,
in the left arm following fall on the ground while he is
playing. X - ray showed
supracondylar fracture of the humorous, he received opioid analgesia as a part
of his initial management , after 30 minutes, the boy developed swelling in his
lips, generalized urticarial rash, and inching all over the body.
Of the following, the MOST likely blamed opioid is
A. morphine
B. codeine
C. pethidine
D. methadone
E . tramadol

-- —
31. Malignant hyperthermia is critical
condition that may develop during
Krsssr '

1

'ZZSSZS •"** “*
A.
B.
Nitrous oxide plus vecuronium
Halothane plus vecuronium
- i w ti »

C. Halothane plus succinylcholine


D. Nitrous oxide plus succinylcholine
E. Sevoflurane plus vecuronium

32. Which of the following is the MOST commonly used


inhalational agent for
induction and maintenance of general anesthesia in children?
83
A. Nitrous oxide
Q. Isoflurane
C. Halothane
D . Sevoflurane
E . Desflurane

33. For children with vascularaccess, IV


general anesthesia. induction should be mutine
Of the following, the MOST commonly artm
A. Fentanyl
.nistered IV nduction
i
during

B. Propofol
' agent is

C . Etomidate
D. Ketamine
E . Sodium thiopental

34 A 5-year-old asthmatic b

Was silent Sp 02 65
.
breath, agitation, interrupted speech, and
% .
He was nebulized 5
salbutamol a d pratropiurn every 20 times at home with
alternating
prednisolone (2mg/kg) in addition to his minutes and received oral dose of
regular
medications.
f
°

kept on continues nebulized


salbutamol driven by oxygenIn the E/D he was
hydrocortisone, aminophylline and and received IV
magnesium sulfate
clinical improvement that necessitate without significant
ICU
Of the following, the MOST appropriate admission.
drug for sedating this patient
intubation is ,
before
A. diazepam
B. etomidate
C. ketamine
D. fentanyl
E. midazolam
35. Which of the
following is the MOST commonly used benzodiazepine
pediatric anesthesia in
?
A. Lorazepam
B. Clonazepam
C. Diazepam
D. Midazolam
.
E Oxazepam

84
anesthesia, opioid reversal with naiOX
recovery from general 0ne
36. During '4
increase the postoperative
A. pain
vomiting
B. nausea and
C. hypoventilation
D. hypotension
E. hypothermia
dietary supplements is recommended to prevent
37. Which of the following
antibiotic-associated diarrhea ?
A. Zinc
B. Probiotics
C. Peppermint
D. Chamomile
E. Omega- 3 fatty acids

38. Which of the following is the MOST likely complication of spinal anesthesia?
A. Infection
B. Epidural hematoma
C. Nausea and vomiting
D. Pruritus
E. Cephalad spread of blockade

39. Regarding house-hold products, ingestion


of which of the following cleaning
agents is considered as caustic ingestion
?
A. Soap solution
B. Electric dishwasher detergent
C. Single-use detergent sac
D. Powdered detergent
E. Glass cleaner

40. A 14-month- old girl frequently


recurrent chest infections, She
admitted to the hospital
always expresses
fears
because of
environment and resists physical examination from hospital
shows slightly enlarged cardiac shadow; since her first admissi
sion. Her CXR
cardiac assessment.
accordingly she was
referred for

85
0f the following, the MOST
for her is
convenient sedating
echocardiography agent to perform
A . chloral hydrate
g. morphine
C. methohexital
D. nitrous oxide
£ phenobarbital

41. In the presence of an anesthesioloeist


a V
which of the following drugs is the MOST
procedures like bone marrow aspiration in children?
" agentpediatrician
effectiv!TCsedating
' for painful
,

A. Midazolam
B. Propofol
C . Diazepam
D. Fentanyl
E. Dexmedetomidine

42 . Regarding pain categories , a sharp, pulsatile


, and well localized pain
characteristic for is
A. deep somatic pain
B. visceral pain
C. superficial somatic pain
D. dysfunctional neuropathic pain
E. inflammatory neuropathic pain

43. Which of the following pain measurement


tools is suitable for infants and
nonverbal children ?
A . Visual analog scale
B. Likert scale
C. Wong- Baker face scale
0. FLACC scale
E- FACES- R scale

• In the treatment of pediatric pain which of the following NSAIDs has


L #

extensive pediatric safety experience ?


Aspirin
8 - Ibuprofen
C- Diclofenac sodium
«6
D. Ketorolac
E. Naproxen

via pacifier or gloved finger bef


45 using sucrose solution ore s ar , ,
procedure; can provide significant
A. 2 minutes
analg esia that may last up to
' '
i

B. 4 minutes
C . 6 minutes
D . 8 minutes
E. 10 minutes

46. Which of the following NSAIDs is useful in treating pediatric patients


with
moderate to severe acute somatic pain ?
A. Aspirin
B. Ibuprofen
C . Diclofenac sodium
D. Naproxen
E. Ketorolac

47. Which of the following is the preferred


antidote for cyanide poisoning?
A. amyl nitrite
B. sodium nitrite
C. sodium thiosulfate
D. pyridoxine
E. hydroxocobalamin

48. A 6-year-old boy, known


case of sickle cell
severe intractable back pain for the anemia presents
last 6 hours. to the E/D with
Of the following, the drug of
CHOICE to control his oai
A. diclofenac sodium in is
B. ketorolac
C. morphine
D. naproxen
E. gabapentin

49. Which of the following drugs is useful in


treating pediatric
neuropathic pain ? patients with
A . Diclofenac sodium
87
B. Codeine
C. Acetaminophen
D . Ketorolac
E. Amitriptyline

of chronic severe
50. In the treatment Pain with opioid,
, troubling, but treatable side effect what is the most
commonNausea and vomiting ?
A.
B. Respiratory depression
C. Constipation
0. Pruritus
E. Addiction

51. A 5 -year-old boy, previously healthy, exposed to trauma while


football with his peers. On ER arrival, a 3 cm long lacerated wound is observed
playing

in this left arm.


of the following, the MOST appropriate measure to control his pain during
suturing is
A. IV Morphine
B . IV acetaminophen
C. topical lidocaine
D. topical phenylephrine-tetracaine
E. IM tramadol

52. The diagnosis of carbon monoxide poisoning is confirmed by


A. determination of HbCO level by CO-oximetry
B. determination of 02 saturation by pulse-oximetry
C. measuring partial pressure of 02 by blood gas analysis
D. measuring partial pressure of C02 by blood gas analysis
E. applying oxygen-hemoglobin dissociation curve

the last
53. A 3-year -old boy presents to the E/D with agitation and delirium for
brother in a
2 hours He was completely healthy before accompanying his older
. chest, dilated
forest trip. Physical examination shows plethoric face, clear The boy was
temp 38 C.
Pupils, and dry hot skin. HR 150 bpm, RR 24 bpm, and
witnessed holding a wild plant during the trip.
0f the following, the MOST likely plant poisoning is
A - foxglove
88
B. rosary pea
C . datura stramonium
D . rhododendron
E . conium maculatum

54, Pharmaceutical preparations account for approximately 60%o of


poisoning cases. Pediat r C
'
Of the following, the MOST commonly reported category Is
A . antibiotics
B. analgesics
C. anticonvulsants
D. antihypertensives
E. antidepressants

55 . In children <6 years old, the majority of poisoning exposure can be


managed
through
A. outpatient visit
B. emergency department visit
C. hospital admission
D. phone call to the poison control center
E. self - directed fist aids by caregivers

56. Which of the following agents is potentially


toxic to young children (<6
years ) in small doses ?
A . Aspirin
B. Soap solution
C. Diphenoxylate and atropine
D. Diazepam
E. Paracetamol and chlorpheniramine

57. Some poisons are associated with characteristic


odors that can be smelt in
exposed children.
Of the following, the agent that produces garlic odor is
A. cyanide
B. organophosphate
C. methyl salicylate
D. hydrogen sulfide
E. methanol
89
is the MOST appropriate action , potentially
5g. What hours after ingestion ? °' toxic plant ingestions
present within 1-2 care and symptomatic
A. Supportive ent
B. Decontamination
with activated
C. Decontamination with whole- baLr" *' ,nga lon

0. Gastric lavage and toxicologic anal


toxicotogTc
E. Identification of the plant and

59. A 3- year-old girl


presents to the t / n L
d 3rrhea vomiting, and
from the mouth.
Physical examination K ' ' drooling
wheezy chest, bradycardia,
excessive sweating , and miosis in both
The girlI was found alone in the
house garden few minutes before the rieve opment of symptoms and poisoning
is suspected. ° ^
Of the following, the MOST likely recoenbahi
aUe - s y n d r o m e (toxidrome) is
A. sympathomimetic
8. cholinergic
C. anticholinergic
D. sympatholytic
E. serotoninergic

60. Massive ingestion of which of the following drugs iis associated with high
anion gap metabolic acidosis ?
A. Acetazolamide
B. Ethanol
C. Salicylates
D. Barbiturates
E. Benzodiazepines

61. Regarding laboratory evaluation of poisoned children, liver transaminases,


as well as an international normalized ratio (INR ) should be measured for ever
/
patients has had overdose of
A. acetaminophen
B. aspirin
C. diazepam
D. ibuprofen
E - loratadine

90
1
toxicants quantitative blood concern .

^
following
62. For which of the the diagnosis and formulating treatment plar
a V
confirming
Organophosphate insecticide
integral to
A.
B. Diclofenac sodium
C. Kerosene
D. Desloratadine
E. Iron
for poisoning with
is effective antidote
63. Physostigmine
A. morphine
B. atropine
compounds
C. organophosphorous
D. sulfonylureas
E. heavy metals

- old girl found unconscious in her bedroom and brought to the


64. A 16- year
she is in deep coma with hypertonia, tachycardia and
hospital. On examination
ECG shows widening of
both pupils were slightly dilated. CT - scan is normal and
QRS complex (150 milliseconds).
Which of the following drug overdose is highly suspected?
A. Barbiturates
B. Opioids
C. Tricyclic antidepressants
D. Benzodiazepines
E. Ethanol

65. Glucagon is the preferred antidote for poisoning with


A. acetaminophen
B. nifedipine
C. diazepam
D. atropine
E. propranolol

66. A 15-year-old girl is brought to the emergency departm


ent by her
hours after ingestion of many ferrous sulphate tablets as family two
a suicidal
After stabilization, the MOST likely effective method of Gl de attempt.
this patient is contamination for
A. whole-bowel irrigation
01
0 induction of emesis by syrup of ipecac
C. administration of activated charcoal
D. administration of magnesium sulphate
g. gastric lavage
as cathartic agent

«7- ln
,
he management
° < Poisoning ,tr ,
strategies are most likely to be ?* °'
n esti l d<
effective in 6 flrst 0r "* “contamination
acute ingestion, but it may be consider m re than 2 second hour after an
A. cholinergic medications ° hours after ingestion of
B. massive amounts of liquid medjca
C. enteric-coated salicylates
0. sympathomimetic agents
E. lipid soluble B-blockers

68. Which of the following poisonings is effectively adsorbed by


activated
charcoal?
A. Ethanol
B. Cyanide
C. Lead
D. Carbamazepine
E. Lithium

69. Which of the following methods is no longer recommended in the


decontamination of most cases of pediatric poisoning?
A. Whole-bowel irrigation
B. Single-dose activated charcoal
C. Gastric lavage
D. Dermal decontamination
E. Ocular decontamination

?0. A 10-year -old boy underwent removal of skin lesion under local anesthesia
,
using subcutaneous bupivacaine without adrenaline. Few minutes after surgery
he develops severe postoperative headache, confusion and perioral
paresthesias. Physical examination shows hypotensioion and bradycardia, while
ECG demonstrates widening of QRS complex.
Of the following, the MOST likely treatment is
A. hemodialysis
B. intralipid emulsion therapy
92
dialysis
Q peritoneal

managing poisoning with


urine is MOST useful in
ion of the
71. Alkalinization
A . acetaminophen
B . ibuprofen
C. carbamazepine
D. methotrexate
E . amphetamine

agents is effectively removed by hemodialysis?


72 . Which of the following toxic
A . Valproic acid
B. Cyanide
C. Lead
D. Amitriptyline
E. Warfarin

73 . A 2 - year -old girl presents to the emergency department with severe


cyanosis 4 hours following ingestion of two tablets of dapsone 50 mg previously
prescribed for her dermatitis herpetiformis. The recommended dose for her was
25 mg once daily. Physical examination shows HR 140 bpm, RR 30 bpm, Sp02
85%. ECG and CXR are normal. She partially responded to
02 therapy, but
returned back to previous condition once 02 stopped .
Of the following, the MOST appropriate treatment is
A. continuous oxygen therapy and wait for spontaneous
drug clearance
B. multidose activated charcoal
C. hemodialysis
D. forced diuresis with alkalinization of
urine
E. exchange transfusion

74. Which of the following antidotes iS useful


benzodiazepinesoverdose?
.
in the diagnosis of
coma due to
A. Physostigmine
B. Dimercaprol
C. Flumazenil
D. Pyridoxine
E. Naloxone.
93
75‘ the general management of poisoning,
gastrointestinal dialysis is
describes which of the following the term
th3
tA whole-bowel irrigation
methods?
B Oral administration of magnesium sulphate
c Gastric lavage
'
Repeated administration of oral sorbitol
Multidose activated charcoal

76. Which of the following doses of acetaminophen can be


lowest toxic after single ingestion? considered as a
A. 60 mg/kg
B. 120 mg/kg
C. 160 mg/kg
D. 220 mg/kg
E. 260 mg/kg

77. A 2- year- old boy weighing 11 kg presents to the emergency department,


12
hours after ingestion of 50 ml of acetaminophen syrup 250mg 5
/ ml. The
expected clinical manifestations at this stage is
A. anorexia, vomiting, and malaise with normal hepatic transaminases
B . liver failure with peak transaminase elevation
C. right upper quadrant abdominal pain with normal hepatic transaminases
D. anorexia, vomiting, and malaise with elevated hepatic transaminases
E . right upper quadrant abdominal pain with elevated hepatic
transaminases

78. At which of the following durations after ingestion, a serum acetaminophen


level should be measured if a toxic ingestion is suspected?
A . 2 hours
B. 4 hours
C. 6 hours
D. 8 hours
E. 10 hours

79. A 15-year-old boy brought by his father to the


hours after ingestion of 2 sheets (8 tablets
suicidal attempt. On examination the boy < s healthy w th
Of the following, the MOST appropriate manag '° *
, ^ mg as a
vital signs,

94
A. measurement of acetaminophen level and app|yjng Rum
nomogram
B forced diuresis with alkalinization of urine %
C. emergency hemodialysis
D. starting empiric N- acetylcysteine
E. observation for 24 hours followed by assessment of liver f Ur»
cti0n

SO. A 3- year-old girl has had high grade fever associated with URTi f 0r
th
days. She received several doses of oral and rectal acetaminophen,
about SOmg/ kg/day for the last 2 days . You have been counseled
C0|(ePa^^
^
aboUt '
safety of the antipyretic regimen .
Of the following, the MOST appropriate advice is
^
A . measurement of acetaminophen level
B. no intervention is required
C. starting empiric N-acetylcysteine
D. requesting liver ultrasound
E. requesting AST, ALT , and INR

81 A 2-year- old boy presents to E D


/ with history of ingestion of large number of
unknown tablets before 3 hours . He vomits
twice at home. Physical examination
shows altered mental state, HR 140 bpm
, RR 40 bpm, temp 38 C \ Random
blood sugar 250 mg/dL and arterial
blood gas reveals a primary respiratory
alkalosis. During his stay in the E D,
/ the kid developed seizures twice that
necessitate diazepam administration.
Of the following, the MOST likely
culprit medication is
A. acetaminophen
B. carbamazepine
C. salicylate
D. tricyclic antidepressant
E. metformin

82. In acute salicylism, salicylate pills


( especially
occasionally form bezoars, which should be enteric - coated forms)
suspected if
A. the patient develops seizures
0. serum salicylate concentrations continue to
rise
C. the patient develops abdominal cramps
D. respiratory alkalosis shifted to high anion gap metabolic
acidosis
£. hyperglycemia shifted to hypoglycemia
95
g3 What is the primary mode of therapy for salicylate
A Multidose activated charcoal toxicity?
g Hemodialysis
Q Use of n- acetylcysteine
Q Urinary alkalinization
E Rehydration with ringer solution

g4 . A 2- year-old boy weighing 12 |~


_
hours after ingestion of about the h= M. , tS t 0 the
» bot„e
emereencv „
3
isZ
^ * " 100
,
A
^
„„ST
suspension (100 mg/5ml). He
0 ,« following,
ministration of activated chared
B. performing lavage
gastric
-
C. urinary alkalinization
D. measurement of ibuprofen level
E. supportive care with observation

85. A 3- year-old girl presents to the emergency department,


2 hours after
ingestion of one tablet of a medication prescribed for her
brother to control his
drug craving. She is in coma with pupils reacting to light
but moderately
constricted, HR 70 bpm, RR 8 bpm, temp 35 C\ Sp02
89 %.
Of the following, the MOST appropriate treatment is
A. Administration of activated charcoal
B. IV flumazenil and treating according to response
C. Starting IV naloxone therapy
D. Supportive care and request toxicological screen
E. Hemodialysis

86. Which of the following is considered to be the MOST toxic p-blocker in


overdose situations ?
A . Propranolol
B. Atenolol
C. Bisoprolol
D. Carvedilol
E. Metoprolol

96
87. A 2-year-old boy presents to E/D with history of ingestion of ia
unknown tablets before 6 hours. Physical examination shows a
'
itSB '"‘"V .

/
state, HR 60 bpm, Bp 65/45, RR 24 bpm, temp 37.2 c, Spo
auscultation reveals diffuse expiratory rhonchi. Random blood
mg/dL and blood gas is normal.
Of the following, the MOST likely culprit medication is
^
^
V

A. oral antidiabetic
B. tricyclic antidepressant
C. calcium channel blocker
D. 0- blocker
E. a- blocker

88. Which of the following is the MOST dangerous calcium


channel bl cker in
overdose situations ? °
A. Nimodipine
B. Diltiazem
C. Nifedipine
D. Amlodipine
£ . Nicardipine

89. Which of the following is


considered as the antidote of CHOICE in
treatment of calcium channel blockers the
overdose ?
A. Calcium salts
B. Dextrose
C. Adrenaline
D. Lipid emulsion therapy
E. Insulin

90. Which of the following is the


antidote of CHOICE for
ethylene glycol poisoning? both methanol and
A. Ethanol
B. Fomepizole
C. Sodium bicarbonate
D. Folic acid
E. Pyridoxine

91. A 4- year-old girl presents to the E/D with nausea and vomiting
f0r th
6
hours. She is a known case of heart failure on digoxin therapy. Before *
on d
Q7
the girl was treated with a macrollde
,,
antih

SSTi * onsnreveal sel|


-
"
'
* - “^ ^
0
- lnvestlgat ls r
> hi‘
SO hpm , With

Star * "
07
• serum v *
,
rj f

'r s
' :nrsTp ““ ~
complex ( 110
° '*d
.
8 serum drgoxin level
'” " > Outcome is

C. serum potassium
D. serum creatinine
E. heart rate

92. What is the expected time for deve ,°


P >ent of GIT
massive ingestion of iron in children ?
A. 5 -10 days
" strictures following
B. 2-3 weeks
C. 4-6 weeks
D. 7 -8 weeks
E. 9-10 weeks

93. A 2-year -old boy presents to the E


/D with altered mental
hours. His mother noticed that state for the last 6
her pills organizer is
days; she is a diabetic patient on empty for the next
oral antidiabetic two
tablets twice daily ). On examination therapy ( glibenclamide 5mg
the boy looks drowsy
sweating, HR 160 bpm, RBS 35 mg with profuse
/dL, and serum electrolytes are
patient was treated with
two bolus doses of IV dextros normal. The
sugar to 50 mg/dL and partially e that raises his blood
improved his mental state.
Of the following, the MOST
appropriate next step is
A. hemodialysis
B. continuous dextrose
infusion
C. administration of
glucagon
D. administration of
a third bolus dose of IV dextrose
E. administration of octreotide
94. What is the i
expected toxicity following large overdose of metformin?
A. hypoglycemia
B. nephrotoxicity
C. neurotoxicity

QR
D. lactic acidosis
E. hepatotoxicity
in his bedroom and brought
d
Uhe°
95. A 14- year-
'
^
t th
°
hospital. Before six months
, survived a suicidal attempt and ke * °
« 0r
( imipramine 25 mg at bed time ). The
antidepressant medications
found empty beside the victim. On examination he is jn
container was
ia
hypertonia , mydriasis, HR 130 bpm, Bp 70/50, temp 37.8 C - ' ^
ECG
coma with ) with multiple premature
complex ( 150 milliseconds
shows widening of QRS
ventricular contractions.
treatment is
Of the following, the MOST appropriate initial
A . esmolol
B. sodium bicarbonate
C. hypertonic saline ( 3%)
D. lipid emulsion therapy
E. amiodarone

96. A 10- year-old boy presents to the E/ D with visual disturbances and rapid
deep breathing 12 hours after accidental ingestion of a paint remover stored in
a carbonated beverages bottle. Visual disturbances include cloudy vision,
decreased acuity, and the " feeling of being in a snowstorm". Ophthalmic
examination reveals dilated pupils, retinal edema , and optic
disc hyperemia.
Of the following, the MOST likely ingested poison is
A. acetone
B. ethanol
C. methanol
D. ethylene glycol
E . benzene

97. A 2- year-old boy presents to the E D


/ one hour after ingestion of
quantity of sewage cleaner. The boy has unknown
excessive drooling and he
swallow anything. Examination of the chest is refuse to
normal.
Of the following, the MOST appropriate management
is
A. performing gastric lavage
B. administration of activated charcoal
C. nothing by mouth and proton pump inhibitor ther
apy
D. corticosteroids therapy and prophylactic antibiotics
E. immediate endoscopy
99
of the following pesticides is MOST
98 - Which commonly used ,n home ?
A Pyrethrins
0- Organophosphates
Q Carbamates
D. Organochlorines
£ Coumarins

drowsy with odor of kerosene clearly smelt in his breath, HR loo hn


»«
bpm, temp 37.5 »
C, 02 95% with clear chest. The
boy vomited twice at home
and his cloths were found contaminated with kerosene.
Of the following, the MOST appropriate management is
A. supportive care and observation for 24 hours
B. start prophylactic antibiotics
C. start systemic corticosteroids
0. immediate CXR and treat accordingly
E. supportive care and request CXR after 6 hours

100. A 2- year- old girl presents to the E/D one hour after accidental ingestion of
kerosene which was stored in a domestic container. She is asymptomatic with
odor of kerosene clearly smelt on her cloths, otherwise normal physical
examination.
Of the following, the MOST appropriate decontamination method is
A . Emesis
B. Gastric lavage
C. Activated charcoal
D. removing contaminated clothing
E . whole- bowel irrigation

difficulty in thing few


101. A 5- year-old boy presents to the E/D with
minutes after spraying an insecticide in the garden
by is a

^ ^ ^


and

- *—
, exce sweating
with oral frothy discharge, diffuse expiratory wheeze /

5SITUS, *•^’
1 1 1

A . activated charcoal
B. gastric lavage
C. whole- bowel irrigation
100
D. washing all exposed skin
with soap and water
E. induction of emesis

102. What is the MOST appropriate measure to establish a prima la1 n


8 0s
organophosphorous or carbamate poisoning? *3
A . Recommending history and physical exam findings
B. Monitoring signs and symptoms after administration of atroP ne
C. Qualitative assay of the pesticide in the blood '
D. Measuring pseudocholinesterase activity level
E. Measuring RBC cholinesterase activity level

103. A 14- year- old boy presents to the E/D with recurrent seizures
afie
ingesting unknown quantity of an insecticide as suicidal attempt . He is in
deep
coma and shallow breathing; physical examination shows drooling
from the
mouth, wheezy chest, excessive sweating, miosis in both eyes, generalized
fasciculation, HR 50 bpm, RR 12 bpm, and Sp02 80% with oxygen. After
suction
of oral secretions rapid sequence intubation is
followed.
Which of the following skeletal muscle
relaxant should be AVOIDED during
intubation?
A. Rocuronium
B. Tubocurarine
C. Pancuronium
D. Vecuronium
E. Succinylcholine

104. Which of the following


antidotes is SPECIFIC for
poisoning? organophosphates
A. Atropine
B. Activated charcoal
C. Pralidoxime
D. Clonidine
E. Glycopyrrolate

105. A 15-year-old boy presents to the E/D with


ingesting an insecticide available at home as suicidal recurrent seizures after
attempt.
with generalized choreoathetosis and tremor. Both reyes He is confused
are
examination, clear chest, HR 80 bpm, RR 18 bpm, Bp 100/60 and Spnormal on
02 95%. A
generalized urticarial rash with excessive pruritus is also observed.
101 i
following, the MOST likely culprit
Of the insecticides is
A. organophosphates
B. pyrethroids
C. carbamates
0 . organochlorines
£ ryanoids

106. Which of the following quantities of keros


toxic after aspiration?
. 15
considered as a lowest
A . lml
B. 2 mL
C. 3 mL
D. 4 mL
E. 5 mL
Chapter 7
Pediatric Drug Therapy
Answers
HAIDAR A. N. ABOOD
oonses can be influenced by the patient's genet<


this regard, pharmacogenetics
testing of genetic variations t g tQ interindividual
response to drug,
Pharmacogenomics defined a application of genome-wifc
disease processes that
2
represent

new Z2
.
£ S
adverse drug reaction
I
*
2.(B) Genetic polymorphism in butyrylcholinesterase enzyme will reduce the
ability of the body to hydrolyze succinylcholine which lead to prolonged
respiratory muscles paralysis after administration.
3.(D) Phase I reactions introduce or reveal ( through oxidation, reduction, or
hydrolysis ) a functional group within the substrate drug molecule that serves as
a site for a phase II conjugation reaction. Phase II reactions involve conjugation
with endogenous substrates, such as acetate, glucuronic acid, glutathione,
glycine, and sulfate. These reactions further increase the polarity of an
intermediate metabolite, make
i the compound more water soluble, and thereby
enhance its renal excretion.
.
4 ( A) The major consequence of pharmacogenetic
polymorphisms in drug
metabolizing enzymes is concentration-dependent toxicity
caused by impaired
drug clearance. In certain cases, reduced
conversion of prodrug to
therapeutically active compounds is also of clinical importance
.
5 (E) Thiopurine S-methyltransferase (TPMT) is a cytosolic enzyme
that catalyses
the S-methylation of aromatic and heterocyclic sulfur-
containing compounds,
such as 6-mercaptopurine (6MP), azathioprine, and 6- thioguamne
used in the
treatment of acute lymphoblastic leukemia ( ALL ). To exert its cytotoxic
effects,
6MP requires metabolism to thioguanine nucleotides by a multistep
rocess
TPMT prevents thioguanine nucleotide production by methylating 6
MP The
relatively few patients with low to absent TPMT activity are at increased
risk for
severe myelosuppression if treated with routine doses of thiopurines.
103
,
6.1B) Salbutamol acts as agonis ,druB8 on „
hronchodilator drugs are not
,^ adrene
rop urri and TRIC receptors. while
antagonist, while theophy||jne and tiotropium m are
other
aminophylline are muscarinic
inhibitors) phosphodiesterase
7. (A ) Redu ced musc ular bloo d f|ow

"^*
inefficiency of musc ular 11
contractions ( U m
an increased percentage of watPr
Ut
disPersmo
days 0f r e ,
'
the relative

^,
1
* !“ ,
muscle and
and extent of drugs given intramusculari » h* rate
increases into infancy, and thus the b a ne «e M(1*‘ he
°' °" *blood
route is comparable to that seen in labilitV of drugs B by fl *
' °
and descents he IM
8 (E ) Newb orns have a much higher ^
proP rtion of boriu
water (total body water ( TRVA/I °
^ '" ^ ,
0 13
*f 75 ^°f rm of


children. In addition, the percent % ) than old
(decreases) from the newborn staged ° eXtracel u r«r water (
(appr0)<jmately
|
?
?l 1 , ?
few1 chan8«
an<i

30%). In fact, the increase of TZ , ,n


the
45 % in 3 U thood
)
(20-

w,f,o„s,"
contrast , the percen tage ; donat e is attrih
0f
•»« »
b» drug, „ such are

9.(A) Mucosal permeability (rectal and h '" * * * , ,


* tha nadu ts ( -2-0 3 Uke )
"
*3
°
g3 n responsib e for the
and their metabolites. The development
,
elimination of drugs
of renal function begins
fetal development and is complete by during early
early childhood
ll.(B) Altered renal drug clearance
in the newborn and infants
result in the

.
different dosing recommendations seen
in pediatrics. The aminoglycoside
,
antibiotic gentamicin provides an illustrative
adults with norm values for 0
dosing interval for gentamicin is 8 hou
>130 mL/min
. V a OTR
,
/1.73 m2, a gentamicin dosing „
6 hours may be
necessary in selected patients wh ,nfections that require
maintaining steady-state peak and troug concentrations near the
f
upper boundary
of the recommen e
_ ’ range. m contrast, to
maintain “therapeutic" gentamicin plasma concentrations in neonates during
the 1“ few weeks of Qurs js required.
life, a dosing interval of
U-(C) Therapeutic measurement of drug
drug monitoring ITD ) y
, ^
concentrations in plasma at some point our dr(jg s dosing interval. These
104
a that are “desired" for a
red with thos giVen
2
^^
t the dOSe/dosing
reg,
' Ti
ieve s are
on published in
'° r tion are thera peuticallv
monitC
”f

*
^
drugs
approach isu sed p
^any
aminoglYc0S
antjbjoticSi vancomycin
,
settin g leg " . , tacrolimus, mycophenolate mofetii
clinical s
cVCiosponnnee
,
^
antiretroviral rtruR
* t S, and acv '' »
cl0 r-
q( ljquid med ca
jons for children can be asso
ci
js not appropriate
^J
^* theJ*
of a
,easpoon versus 5.0 ml
of hypodermic syrin
^
insure
prompted many physicians 30 P
^
medications in order to improve accuracy While this
approach would se
be associated with greater accuracy in i dosing, parents/caregivers ca
difficulty in reading the graduations on a syringe, and the plastic caps on th
plunger of syringes can produce a choking hazard for infants and yo *
children.
14.(E) The pulmonary surface area in pediatric patients of all ages
^
is a vo>
^
j

effective, easily traversable barrier for drug absorption. Rate-


limiting factors for
pulmonary drug absorption include physicochemical
drug and delivery system (e.g., particle
factors associated with the
size,diffusion coefficient, chemical
stability of drug molecule in the lung) and physical
intrapulmonary drug disposition (e. factors that influence
g., active versus
tracheobronchial tree, respiratory passive drug delivery to
minute volume ,i
many of which are developmental mternal airway diameter),
15.(C) For drugs formulated
developmental factors (e.g.,
^ dete rmin
for delivery
ed .

using a
inco
inability to follow instructions ordi nati on of device metered-dose inhaler
for clearing of
ai
actuation with inhalation,
actuation of delivery device) airwa y, passive
either inhalation with
children) or limit the bioavailability prevent their
of
instances, specific devices (e.g., space the dmg to bg ' !!' ' 1 n nfants and sm3
than 5 years or without mas
for older
r
cha mbe
, rs wifh
dniln stered. In these
‘ "
methods of delivery (e.g.,
can be used to improve the
16.(C) Drug-drug interactions largely
of
children n
efficient dTug deiT
theT "
occur at
bv ^ for children less
use) and
^ a d thus , 0 erm use)
dr , **
may occur at the level of drug eVel of 8 efficacY *
binding.
absorption ,
distrih„
* 0n drue ^
' eliminatjo etaholism but
^ and receptor

105
^ Sngor expected to receive IV calci'um' S
194A) Drug interactions
^ ^^
largely occur at the level
p

of drug
uaslueV "
^ are

metaWisro (eg
phenebar ans CYP 450 enzyme inducer that
^ increase the metabolism of
warfarin and increases thrombotic risk, while erythromycin, cimetidine and
ketoconazole are CYP450 enzyme inhibitors that decrease the metabolism of
warfarin and increase patient hemorrhagic risk ) but may occur at the level of
drug distribution (e.g., displacement of warfarin plasma protein
binding by
ibuprofen with consequent increased hemorrhagic risk ).
20.(C) Even when a dietary supplement is safe when used correctly, it can cause
mild or severe toxicity when used incorrectly. For example, although
peppermint is a commonly used and usually benign gastrointestinal spasmolytic
included in after- dinner mints, it can exacerbate gastroesophageal reflux.
21.(D) Valproic acid causes hepatotoxicity in infants <2 years of age. For other
drugs, ADRs can occur in older children.
22.(E) The only approach that can be used to facilitate therapeutic compliance
and adherence in the pediatric patient is the combination of vigilance (on behalf
of all caregivers) and repetitive education coupled with positive reinforcement.
23.(B) Respiratory illnesses associated with fever, mucopurulent nasal
discharge, productive cough, or lower respiratory symptoms ( wheezing, rales)

anesthesia be postponed 4-6 wk in this setting.


24.(A) In neonates, assurance of vitamin K prophylaxis and adequate
coagulation status is critical before any major surgery.
5.(E) Surgery and painful medical procedures are psychologically traumatic
*
events for children and families. Many children un ergoi Clir„prv have new -
onset inegative behavioral changes postoperatively. maladaptive
behavioral responses may include enuresis, separation anxiety emper
tantrums, fear of strangers, doctors, and
hospitals. behavioral changes include
is another .
risk factor. Preoperat ve
decrease the incidence of postoperative changes. Parental presence

106
surgery. Midaiotam an <% %
* is ar,
'
5
'
^ (B) Preoperative fasting
incidence of aspiration of
guidelines have been developed to redut.
gastric contents during anesthesia Clear, >h*
liquids (e.g., Pedialyte, 5% dextrose
in water) facilitate gastric emptying
hypoglycemia, and may be given up to 2 hours
before anesthesia.
/
Breast min
may be given to infants up to 4 hours before surgery . Infant formula , fruit
surgery . Solids
Juices"
and gelatin may be given up to 6 hours before should be avoid
for 6-8 hours before surgery.
27.(C) Benzodiazepines are effective premedication drugs with good safety
profile, but can depress respiration when administered with opioids ( known
potent respiratory depressant agents). Combination with barbiturates can also
increase the incidence of respiratory depression but to lesser extent than that
observed with opioids.
28.(D) Succinylcholine is a depolarizing neuromuscular blocking agent that
produces muscle fasciculations which may be associated with immediate
increases in intracranial and intraocular pressures as well as postoperative
muscle pain.
29.(E) Opioids represent the gold standard for providing analgesia .
30.( A) Among opioids, morphine and, to a lesser extent, hydromorphone may
cause histamine release after administration.
31.(C) Succinylcholine is a depolarizing skeletal muscle
relaxant agent that can
trigger malignant hyperthermia in susceptible
individuals, this risk is greatly
increased when succinylcholine combined with
volatile general anesthetic
agents like halothane.
32.(D) Sevoflurane is the most commonly used
and maintenance of general anesthesia in
inhalational agent for induction
children.
33.(B) Propofol is the most commonly
administered IV induction agent .
.
34 (C) Ketamine is a bronchodilator and is a
useful agent for sedating asthmatic
patients in the ICU. Etomidate is associated with
increased mortality when used
as a sedative in the ICU (for which it is now
contraindicated). Benzodiazepines
and opioids can depress respiration.
.
35 (0) Midazolam is the most commonly used
benzodiazepine
anesthesia. It is short acting and water soluble; it can be injected in pediatric
without
It is a potent hypnotic-anxiolytic- anticonvulsant and is
more potent than diazepam. Midazolam may be administered
approximately 4
orally, nasally
s time'
107
,


rectally, intravenously

^^ ^
*.
Qr

minimal effect on respir ^ *. J^ ,


u
^ 0 j5 rng/kg IV)

' * JJJ
Or has
r
midazolam OsTo
^^ * ^^
V provides

effects.
(

*° nZ
he : ' Z : ' °,
> ryTOlyi
« „
rup
reS 0 V depr
th rai
i
sa t
reversal with naloxone m
36 (A) Opioid
- be indl«ted in „
opioid effect is suspected u
" * Foil °, , "
excessive depressant and the
respiratory analgesic ,alp one
0 5 f opioi
wh
b h *h«
reversal,children may suppled ° *
experience increased pain
- " '°
' g na one
*
37.(B ) Some uses of dietary
as vitamin D supplements for ' ° , " probioticT ," ^
breastfed
6 C mnl0 ar d
nfan s a d
> recornm
*
antibiotic-associated diarrhea, whereas ot ' "
«7£ zz::z 'zr r*""“ "

— S sS E T? Ssscr
op,Old use necessitates antipruritic and antiem
ep,dura hematoma are extremely rare.

,
39.(B) Electric dishwasher detereent
highly alkaline (pH >13 , and
*
>L

11
^
- -'

***
and Vomiti - Neu axial
therppV Infection and

,
Significant
are

are

to the vocal cords and Gl tract. Single-use detereent


many children. When bitten into, a relative
detergent is expelled under pressure ontn th
cords- > can lean
«« are , ,
e !!*^ T^° Se
reSemb e «

**» » « «1
"*to
concentrated
.
in the neutral* . ^
cnna
products is tone
40 (A) Light sedation with chloral hydrate, benzodiazepines, and

,
dexmedetomidine is often sufficient for non-painful procedures.
.
41.(B For painful procedures (e.g , bone marrow aspiration) , the com a
of hypnosis and analgesia is required. The addition of »
regimens increases the risk of respiratory insufficiency She>
ej * * *^**
*
(e.g., propofol, methohexital, remifentaml
,
sedation but their use carries a higher like
)

general anesthesia. Use of these medications req


prow
^ . dural
jnduction 0f
hg presence of an
.
Qualified physicians,
experienced , and qua
anesthesiologist and/or specially trained,
42 (C)
Pain categories injury to or inflammation of tjSs
in resulting from
/ Somatic (pain ues)
Characteristics: structures : sharp, pulsatile, well |0
In skin and superficial
In deep somatic structures
Visceral (pain resulting from
Characteristics:
injury to or inflammation of Vj ^
: dull, aching, pulsatile, not we i ^Ca,ized
°
; poorly localized (e.g. , app
Aching and cramping; nonpulsatile
pain perceived around umbilicus or
) referred to distant locations
e&
angina perceived in shoulder)
J Neuropathic (pain resulting from injury to, inflammation 0f 0r
dysfunction of the peripheral or central nervous system)
Characteristics:
Spontaneous; burning; lancinating or shooting; dysesthesias (pins and
needles, electrical sensations); hyperalgesia (amplification of noxious
stimuli); hyperpathia ( widespread pain in response to a discrete noxious
stimulus); allodynia (pain in response to nonpainful stimulation); pain
may be perceived distal or proximal to site of injury, usually
corresponding to innervation pathways ( e.g., sciatica )
43.(D) FLACC (Face, Leg, Activity, Cry, and Consolability) pain scoring system
may be used in preverbal, mechanically ventilated, or cognitively impaired
patients. It is an acronym that includes five indicators, each scored as a 0, 1, or 2
that forms a ten-point composite scale with a range from 0" (no pain) to " 10"
"
(worst pain).
44.(B) Compared to other NSAlDs, ibuprofen has
extensive pediatric safety
experience .
45.(D) Using sucrose solution via pacifier or gloved
finger over 2 minutes before
starting procedure; can provide significant
analgesia that may last up to 8
minutes and the dose may be repeated once.
46.(E) Ketorolac, an IV NSAID, is useful in
treating moderate to severe acute
pain.
47.(E) The FDA has approved hydroxocobalamin for
use in known or suspected
cyanide poisoning. This antidote reacts with cyanide
to form the nontoxic
cyanocobalamin (vitamin B12), which is then excreted
in urine. Side effects of
hydroxocobalamin include red discoloration of the skin
and urine, transient
hypertension, and interference with colorimetric lab assays.
Hydroxocobalamin
has an overall safety profile that appears superior to that of
the cyanide

4r
\r\
antidote kit (amyl nitrite, sodium
nitrit and
preferred antidote for cyanide poison- ' sodium thi
osulfate) and thus
43.(0 Opioids are administered formL , is the

49.(E) Antidepressants like amitriptyij


?" Cance Pain
postoperative pain, sickle cell crisis pain°‘ a e and Severe
pain,
such as acute
useful in treating pediatric patients with “ antic°'nvU|sants Uke gabapentin
50.(C) The most common, troubling b, rneuropathic pain are
constipation . Patients who take nni treatable side Pffa *
P,oids USe
predictably develop tolerance to the
remains
term
,

opioid
but tolerance to £
Tver timea troublesome
administration.
£
Stool .
.J
£ *
and distressing proJ°
m
^
•* C X j j
m
eS ndt
°«ur-
Chronic

a most all
|
Pain 1 '
° °
, and
constipation
'*
periods
opioids

ulant laxativ" * T** '°


3nd stir n8'
administered most
to patients
Osmotic and bulk laxatives £rece " 2 *K*
* S £ ?T Pr°ducin« ore distention
°*
and discomfort. A peripherally acting *
effect ?V reVerSes
'" antagonist
^
methylnaltrexone promptly and
, P'receptor
constipation in patients with chronic in P id induced
pa who
^ reCe v,ng ° '° -
51.(D) Topical local anesthetic preparation ' °Pl0P'3dsindaily
,

circumstances: suturing of lacerations, placemen ? ^« in di


'
lumbar punctures, and accessing indwell
application of tetracaine, epinephrine and r
for suturing wounds but should not be
Combinations of tetracaine with phenylephrine and
^?!?
1 venous Ports. The

reSUltS g0Od anesth«ia


"
" iT“
membranes-

Z Z^*** "
^
1
tetracaine are equally as effective, eliminating the *
0 use a controlled
substance (cocaine).
52.(A) CO binds to hemoglobin with an affinity >200 times that of oxygen,
forming carboxyhemoglobin (HbCO). HbCO level is determined by CO-oximetry.
53.(C) Datura stramonium or jimson weed belong to belladonna alkaloids that
contain scopolamine and atropine. Ingestion results in typical anticholinergic
toxidrome.
54*(B) Approximately 40% of poisoning cases involve nondrug substances, such
dS cosmetics, personal
care items, cleaning solutions, plants, and foreign bodies.
Pharmaceutical preparations account for the remainder of exposures, and
analgesics, topical preparations, vitamins, and antihistamines are the most
commonly reported categories.
be
55- (D) The majority of poisoning exposures in children <6 years old can
beyond a call to the regional
managed without direct medical intervention
,
po son the product involved is not
control center (PCC). This is because
110
of the material is not sufficient to D
quantity
- ne^mlY toxi c or the . How eve r, a number of substances can b
Q „
Sretodledlernst smtypallically
toxic to
toxic effects
in doses .
implies 1 or 2 pills or
5 mL.
54(C)' Small
dose" odo r ), organophosphate (garlic ), meth Sal
almond '%t»
Ibit ter ( rotten eggs ), and methanol acet
Cya nide (
57.(8 ) sul fide
), hydrogen
(wmtergreen toxi c plant ingestions, consider decont
potentially
58.(8) For pati ents who present within 1-2 hr of inee ^
activated charcoal supportive and based on symptoms Th m 'st c rrin/
in
*
treatment is primtoxicarily
ity afte r plan t ° °
ingestion is Gl upset which Canbeman C'
manifestation of and electrolyte support *tec
with antiemetics and fluid
toxi dro me
.
is characterized bv Paras Pa h
l ,
br0^
. Cho liner gic
r hoft
59 (8)
brad c
manifestations (diarrhea, urination, miosis )
' *
txonchospasm, emesis, lacrimation, and salivation

60.(C) Any patient serum


chemistry panel) must have an anion gap calculated because of the
mor
specific differential diagnoses associated with an elevated anion gap
metabolic
*
acidosis which include ( methanol, metformin, uremia, diabetic
ketoacidosis
propylene glycol, isoniazid, iron, massive ibuprofen, lactic acid
osis, ethylene
glycol, salicylates, cellular asphyxiants (cyanide, carbon
monoxide, hydrogen
sulfide), alcoholic ketoacidosis, tylenol (clinical sign
ificance depends upon
presence or absence of liver injury ).
61.( A) Patients with a known ove
rdose of acetamin ophen should have liver

M- ( E ) For select of acetaminophen).


intoxications (e.g., salicylates,
some anticonvulsants
?
^ measured T "
80* met han ol
theophylline CO ead ’ '" * , ethanol, lithium, ethylene glycol

,
'alter
" management. All into
Powres, quantitative
xicant i °* S
8
,
* reatment Plan However, forrvafl
n0t readi Y ava
ilable and is not likelyt0
the historY - For
may
• . **
nontoxic
gnificant
example, a meth
, whereas
a ^VimUSt **20intemrPrdLeted in conjunction
simillr evel 24 hours8/afte1r hou
0
r after
with
ingestion

^ poisoning.
63-(B) Conside
r
Patiem iS 3 hreat t ^
phvso t
inge implies a
stion

* 0 self o

^^ he wlyuTe?no"* °f anticbolinergic poisoning if


1
conduction .
delays on ECG
i

>
ClCaf;' depressants
(
ic
1° and
,
^NS symptoms
ovasculaf andtoxicity . Symptoms
serious toxicity usually manifests within 6
.
dominate the ri,
typ cally* ' "°no'?
deveton W
" °' ’n^
Pre n »
h n 12
.
^
c
e5

*0
caan Mve an extremely rapid progression from m,ld
" d srhythmias. Patients often develop features of ,1 ' °"ll,e
including delirium, mydriasis, dry mucous membrane!"!!
urinary retention, and slow Gl motility . CNS iox,ri,'
"* 'coma, myoclonic jerks, and seizures. Sinus tachycardia_ i the
'
u
Qr!

W
^ ar jiovascular
nWidening 0
manifestation of toxicity; however ,
(
QRS complex.
patients
most
can also
is

^'” Polucagon is theoretically the preferred antidote of choice for docker


of the

<
65 tE1 Glucagon stimulates adenyl cyclase and increases levels of CAMP
to **
*1cpen ,ent 0f the p-receptor
1 . Glucagon is typically given as a bolus and , if this
l

followed by a continuous infusion.


‘S effeCQVseribed
methods of gastrointestinal decontamination include induced
^emes«s ith ipecac
.
, gastric lavage, cathartics, activated charcoal, and whole
gl) . Of these, only activated charcoal and WBI are of
*

b wel
°P tent,a
n
^
WBI involves instilling large volumes ( 35 mL/kg/hr in children
glycol electrolyte solution to " wash
°f 12 r jn a(jolescents) of a polyethylene
^
°< the tire Gl tract. This techniquewell may have some success for the ingestion

**
of SR Prep
, Orations
substances not adsorbed by charcoal (e.g., lithium,
natches foreign bodies (e.g., lead chips), and
drug packets.
decontamination strategies are most likely to be
67.(0 astr°. ' ^
effective in the 1
\ after an acute ingestion. Gl
absorption may be
G| motlljty (anticholinergic


.

—--
medications, l» ds) m * “ -
° °'
preparations, and agents t
form pharmacologic bezoars (e.g., entenc
may
her ingestion
coated salicylates). Gl decontaminate properties.

*68.(0) Substances poorly adsorbed


v Alcohols
yr
' xzsissz
<*

/ Caustics: alkalis and acids


/ Cyanide
/ Heavy metals (e.g., lead)
v Hydrocarbons
v Iron
/
Lithium

112
Although gastric lavage was used routinely for many years, obJP
69.(C)
document or support clinically relevant efficacy. This is partieddat, *
do not
in children, in whom
only small -bore tubes can be used. Lavage

induce bradycardia through a


^ fue
t r>1
consuming and painful and can vagal res ns '

tube placement. It can delay administration


of more definitive ° trP ^
(activated charcoal) and under the best
circumstances, only removes
scenarios, the use of gastric
aT ^
of gastric contents. Thus, in most clinical ®

no longer recommended .
70.(8) Intralipid emulsion (HE) therapy is a potentially
lifesaving intervene
^
HE therapy sequesters fat- soluble drugs, decreasing their impact at tar
organs. It also enhances cardiac function by supplying an alternative eneT
source to a depressed myocardium and acting on calcium channels in the heart,
increasing myocardial calcium and thus cardiac function. Intralipid is most
effective as a reversal agent for toxicity from inadvertent intravenous (IV )
injection of bupivacaine. Using the same 20% Intralipid used for total parenteral
nutrition (TPN), a bolus dose of 1.5 mL/kg is given over 3 min, followed by an
infusion of 0.25 mL/kg/min until recovery or until a total of 10 mL/kg has been
infused.
71.(D) Urinary alkalinization enhances the elimination of drugs that are weak
acids by forming charged molecules, which then become trapped in the renal
tubules. Charged molecules, being polar and hydrophilic, do not easily cross
cellular membranes, thus they remain in the renal tubules and are excreted.
Unnary alkalinization is accomplished by a continuous infusion of sodium
bicarbonate-containing IV fluids, with a goal urine pH of 7.5-8. Alkalinization of
the urrne rs most useful in managrng salicylate and methotrexate toxicity
72,(A) Few drugs or toxins are removed by dialvsis in => m„, 5 suff . Jclen
, .
justify the risks and difficulty of dialysis. Toxins ” ,
t 0 d,alys s have the
following properties: low volume of distribution ( <i I /L, /
water solubility, low molecular weight, and low deerl ' 3 h Bh degre

Hemodialysis may be useful for toxicity from meth ?? , PTOtein binding


ethy ene glvco1,
salicylates, theophylline, bromide, lithium, and valproic
severe electrolyte
T , '
H
Hemodia vsis 15
also used to correct disturbances and acid'b3SC
derangements resulting from the ingestion (e.g sever* metformin-
associated
lactic acidosis ).
73.(B) Whereas single -dose activated charcoal is used as a d f
jfth
decontamination, multidose activated
elimination of certain toxins, MDAC
charcoal ( MDAC ) can
is recommended in managing
help
^"^
to e ° 6 the
°
, 110 « o
" hv* .
* , |
-
line MDA
carbamazepine, daphne
C'is typically given as O S g phenobarbital.
0.2 mg IV over 30 __ /»Wg every 4 bh( (Mlor 4 d and
74"
fiumaten;il if response is j seconds can Averse the
odia «Pines inadequate, rePeat CNS s of
bin * eve ry \m
»n

by 2
proposed
nee and "Gl dialysis

Z' ZX'ZZSZSZi

"
the
Mucosa as a dialysis
: into the
they are adsorbed to the charcoal. inal
intralum
ce Where

jSmClasg/sically.children
kg in and >7.5-10 g ta
4 general stages of
»
'
T7.|A) acetaminophen toxicity have been
scribed:
^v Stage I: 0.5-24 hr after ingestion; characterized by (
anorexia, vomiting
malaise . Lab tests typic ally norm al, exce for acetaminophen level)
pt
Stage II: 24-48 hr after ingestion; characterized by ( resolution of
earlier
symptoms ; right uppe r quad rant abdominal pain and tenderness;
elevated hepatic transaminases (AST >AIT) and INR )
/ Stage III: 3 -5 days after ingestion; characterized by (peak transaminase
elevations; development of liver failure, multi organ-system failure,
death or recovery begins)
Stage IV: 4 days to 2 wk after ingestion; characterized by (resolution of
' liver function abnormalities, clinical recovery precedes histologic
recovery)
78.(6) If a toxic ingestion is suspected, a serum acetamin
ophen (APAP) level
stion. If the APAP level
should be measured 4 hr after the reported time of inge
g drawn, treatment
is known and the ingestion is within 24 hr of the level bein
Rumack-Matthew
decisions are based on where the level falls on the
onogram .
" ,
at ent Presenting clos e to or beyo nd 8 hours after an
Ketdlnj ^ ^
DenH;

(>90 m
^JPeated
L.
P en overdose should be
, ?°
ab ratory results.
°
emp

administration of acetami
f r consecutive days ) can
irica

nop
lly

hen
lead to
star

at
hepa
ted

sup
tic
on N -acetylcysteine

rath era peu tic doses


injury or failure
, poor nutrition
in
,
s fT)e h i
° ° ,»
ren esPec a y in the setting of
fever , deh ydra tion
3
°
'^'
dother conditions that serve to reduce glut athi one stores.

114
.(C) Early signs of acute salicylism include nausea, vomiting d,
81 ' aph
salicylate toxicity can manifest as tachypnea nd
tinnitus. Moderate Sr ,
tachycardia, and altered mental status . The tachycardia
marked insensible losses from vomiting, tachypnea, diaphoresis
of oxidative phosphorylation. Signs of severe salicylate toxicity
^J"^?
|arge

hyperthermia, coma, and seizures. Classically, laboratory values f ^ * ld


hySr .

fr orr

^
poisoned with salicylates reveal a primary respiratory alkalosis
elevated anion gap metabolic acidosis. Early in the course of acut
andV
respiratory alkalosis dominates and the patient is alkalemic. As theVe^ '^
stimulation diminishes, the patient will move toward acidemia . Hyperei ^
coagli
(early ) and hypoglycemia (late) have been described. Abnormal
studies and acute kidney injury may be seen but are not common.
82.(B) Salicylate pills occasionally form bezoars (especially enteric - coated
forms), which should be suspected if serum salicylate concentrations continue
^! !
to rise many hours after ingestion or are persistently elevated despiteI
appropriate management . I
83.(0) The primary mode of therapy for salicylate toxicity is urinary !
alkalinization. Urinary alkalinization enhances the elimination of salicylates by I
converting salicylate to its ionized form, "trapping" it in the renal tubules, thus!
enhancing elimination. In addition, maintaining an alkalemic serum ph !
decreases CNS penetration of salicylates because charged particles are less able
to cross the blood-brain barrier. Alkalinization is achieved by administration of a
sodium bicarbonate infusion at approximately 2 times maintenance fluid rates .
The goals of therapy include a urine pH of 7.5-8, a serum pH of 7.45 - 7.55, and
decreasing serum salicylate levels.
84.(E) Ibuprofen, the primary NSAID used in pediatrics, is well tolerated, even ini
overdose. In children, acute doses of <200 mg/kg rarely cause toxicity, but]
ingestions of >400 mg/kg can produce more serious effects, including altered]
mental status and metabolic acidosis. Supportive care, including use of
antiemetics and acid blockade as indicated, is the primary therapy for NSAID
toxicity. I
85.(C) Two specific oral opioids, buprenorphine and methadone, can cause life-
threatening toxicity in toddlers with ingestion of even 1 pill. Both agents are
used in managing opioid dependence. Ingestions can manifest with the classic
, sedation, and miosis. Signs of more
opioid toxidrome of respiratory depression
toxicity can include bradycardia, hypotension, and hypothermia. Patients
severe
significant respiratory depression or CNS depression should be treated
with patients who are not chronically
with the opioid antidote naloxone. In pediatric
115
taking opioids, the full reversal dose

13 doses contrast,
" , *° "" '
^^
response, avoiding abrupt 10.04-0.4
Induction 0 W thieve desiredthe
methadone and buprenorphine is murh . thdrawal , . Beri j
the
finical
can require multiple doses of naloxone These ha
infusion of
°T " 0
half -
\\\e ol
naloxone, patients
continuous naloxone Patients may benefit
86 (A) Because of its lipophilicitv ann , , from a
0f fa„
bl ckade
propranolol is considered to be the " °
thelT° ,
^
Channel5'
milder symptoms.
**
Overdoses of water-soluble P-blockeers i(e g., member of B bl cker
atenolol) class.
* aSSOCia ed with

! ° ,'
87 .(0) Toxicity of P -blockers ,
it may be delayed
tvpicallw
after ingestion
H Ve
It ' ps wi h n 6
.
preparer ' " ,
hr of in 1 0 al
' hou8h
these effects are manifested as hr * °
W '
release (SR )
hypotension anlTh ' "
' C iniCa V

-
Patients with reactive airways disease can , '
eart block -
experience bronchns
of blockade of p2-mediateri K
glycogenolysis and gluconeogen

sssxa;
mental status, coma, and
^^
~T "" ^ ^ "
"
'

the CNS
; t I
st toddlers
.
cause altered
,
ere

) ,

n after P b|ocker
include an ECG, frequent reasses ° ' overdose should

glucose. Serum levels of P-blockers


use and are not useful in management of the nT ' *
,
^and blood
M or routine clinical

Pi-
verapamil,

89 (El Lfr '


diltiazem,
^
in overdose because of
POphlllcltv and dlrect cafdiac suppressant effects.
Um SaltS administered through a peripheral IV line as calcium
'
gluconate or a central line as calcium chloride, help to overcome blocked
calcium channels. High-dose insulin eugWcemia therapy rnnsidered
c indications

antidote of choice for calcium


Of 1unit/kg of
regular insulin is o o
main mechanism of high-
dose »
infusion at 0.5-1unit/kg/hr. The
IS to
efficiency of a poisoned heart t a m need of carbo y

the metabolic
for energv
^ ^ ^
(instead
minima circu a » insuUn. Blood
of the usual free fatty s«6s)M
'
glucose levels
given to
should be closely mo
maintain euglyoemia, a ^^
rarety nece
^
116
may be
severeW
judicious IV f|uid b
Additional therapies) include
poisoned ' " inf very high doses . Lipid emulsion therapy js a °r s ,N
pat e 1. U
|)
vasopressors ^ especially for patients poisoned with the Potent Siijj
more
!SCCBsclassic
. verapamil and diltiazem.
antidote for methanol and ethylene glycol poisoning
90 (0) The v
ethanol, a preferential substrate for
alcohol dehydrogenase, thus prevent
the metabolism of parent
compounds to toxic metabolites. Fomepiz0|g
i
potent competitive inhibitor of alcohol dehydrogenase , has almost
entire
replaced ethanol because of its ease of administration , lack of CNS , !
metabolic effects, and overall excellent patient tolerability profile ciassi ,

indications for fomepizole include ethylene glycol or methanol level > 20 mg


( assuming no ethanol is present ), history of potentially toxic ingestion ( e g and ^
intentional overdose), or history of ingestion with evidence of acidosit
Adjunctive therapy includes folate ( methanol toxicity ), pyridoxine (ethylenj
glycol toxicity), and sodium bicarbonate infusion for both (if acidemic ). i

91.(C) In acute digoxin toxicity, initial assessment should include an ECG, serun
digoxin level, serum potassium, and kidney function tests. The serum digoxii
level should be assessed at least 6 hr after ingestion and carefully interpreted «
the setting of clinical symptoms, because the digoxin level alone does nt
entirely reflect the severity of intoxication. In acute ingestions, serum potassiui
is an independent marker of morbidity and mortality, with levels >5.5 mEq
predicting poor outcomes .
92.(C) Iron toxicity is described in 5 often-overlapping stages: 1
The 1st stage, 30 min to 6 hr after ingestion, consists of profuse
vomiting and diarrhea (often bloody ), abdominal pain, and significant
volume losses leading to potential hypovolemic shock. Patients who do
!
not develop Gl symptoms within 6 hr of ingestion are unlikely to develop
serious toxicity. |
The 2nd stage, 6- 24 hr after ingestion, is often referred to as the
"quiescent phase" since the Gl symptoms typically have resolved.
However, careful clinical examination can reveal subtle signs of
hypoperfusion, including tachycardia, pallor, and fatigue.
^ The 3rd stage, 12- 36 hr after ingestion, patients develop multisystem
organ failure, shock, hepatic and cardiac dysfunction, acute lung injury,
and profound metabolic acidosis. Death usually occurs during the 3rd
stage.
/ The 4th stage (hepatic) results
coa gulopathy about 2-5 days after n .
in fulminant liver failUre
gestion.
d

117
5th
stage, 4- 6 wk after ingestion, ,s m rked
/ rh* d signs of Gl obstruction. ^ bV for
eSan .
itr 'CLnts sulfonylurea overdose symptom

,3- pfOe! hpt,


|
pat
y treated with dextrose. In patients
. However
with
,h nT *sVmpt
' rr,sshould
«
be sufficient , patients Wj «
0

^7poglycemia should be treated with a ^Ymptorr* S Of


Of IV
infusions and repeated lv dextrose
s dextrose dextrose boluses should bf
, because this can stimulate further |
possible insulin
release and lead
'n and prolonged hypoglycemia. Instead the preferred
2 doses of IV dextrose ) antidote for
(j.e., requiring *
* , a somatostatin analog that inhibits insulin Suifony|Ufea .

^
SlVfl
-.
^ ravenously (IV) or subcutaneously (SC), typically in doses
de

or adults ) every 6 8 hr. -


release 0 J V s
'
2 Mg/k8

9
* D
^
m
' , Although

JCIFC acidosis
teens

is
metformin does not cause hypoglycemia, its assoc,at on ,
well documented ( metformin
w th
associated lactic acidosis MALA
-
. )
typically arises after a large overdose in which the agent interferes
A state
*
vn(tl the liver
's ability to clear lactic acid. Dangerously high serum lactate levels
to hemodynamic instability . Hemodialysis is usually the
can result, leading best
option for patients
with severe MALA .
95.I 0 J in the treatment
of tricyclic antidepressants ( TCAs ) overdose, initial
attention should be directed to supporting vital functions, including airway and
ventilation as needed. TCAs will block the fast Na + channels on the myocardial
eUs, slowing the upstroke of the QRS complex. Because the effect on Na +
channels is greatest within the 1st 6 hr, frequent ECGs (i. e., every 20- 30 min)
jnog this period are important. As the QRS approaches 160 msec , the risk of
“w patient developing monomorphic ventricular tachycardia rises to 30%.

usually in the form of sodium bicarbonate, is the antidote of choice ,

"( cations for sodium bicarbonate include a QRS duration >110 msec,
^
Ocular dysrhythmias, and hypotension. Multiple bolus doses of sodium
Carbonate, 1- 2 mEq/kg each, may be needed to narrow the QRS to <110 msec ,

, but
prefer then to place
the patient on an infusion of sodium bicarbonate
may not be necessary if the QRS is carefully monitored after the imtia
:- se
* and rePeat bolus dosing is provided as needed during the 1st 6-12isI hrin.
^' onic ( 3 % ) saline and or lipid emulsion therapy may bo benefit
/
, omiting develop early after
.°, ness
W5 v
mild inebriation, nausea, and metabolic
^
- -
n
< °anc ^ *
n est on.'
, including profound
The onset of serious effects
12- 24 hr as the parent
vistJal * disturbances, is often delayed up to
ndergoes metabolism to its toxic metabolites. This le
me th3n°
slowed if ethanol has also been ingested , since "
the
> ab0|,
^Trentially metabolite ethanol . Visual disturbances include bl
visionconstricted visual fields, decreased acuity, and the "feeling of b. -
'

snowstorm" and appear only after acidosis is well established these ^ ,

defects may be reversible if treated early, but untreated can lead to “ ' ‘u
blindness. On examination, dilated pupils, retinal edema, and Optic
dr
hyperemia may be noted. Initially, patients have an elevated osmolar
&> P, a
then develop an anion gap metabolic acidosis as the parent compound
metabolized to formic acid.
97,(C) Regarding treatment of children who ingest caustic materials,
emesis
lavage are contraindicated. Activated charcoal should not be used
does not bind these agents and can predispose the patient to
because !
vomiting 1, d
subsequent aspiration. Stridor or other signs of respiratory distress
the provider to the need for a thorough evaluation of the airway
should a J
for potentij
intubation or surgical airway management. Endoscopy can be performed
with!
12- 24 hr of ingestion for prognostic and diagnostic purposes in
symptoma
patients or those with suspected injury on the
basis of history and kno
characteristics of the ingested product. Endoscopy's role is purely diagnosti*
Whether the risks of the procedure are justified is
debatable . Expectai
management with a period of nothing by mouth (
NPO) and proton pu
inhibitor therapy is likely appropriate for the majority of patients
without airx
bums or signs of mediastinitis or peritonitis. Endoscopy
is contraindicated i
such patients, who instead require immediate
surgical consultatioi
Corticosteroids or prophylactic antibiotics are not
beneficial.
S8.{ A ) Pyrethrins are derived from the chrysanthemum
flower and along with
pyrethroids, synthetic derivatives, are the most
, . .. , commonly used pesticides in
the home, whde the most commonly used insecticides
in agriculture are
organophosphates and carbamates.
99.(D) The most important manifestation of hydrocarbon
toxicity is aspiration
pneumonitis through inactivation of the type II
pneumocytes and resulting in
surfactant deficiency. Aspiration usually occurs during coughing
and gagging at
the time of ingestion or vomiting after the attempted ingestion
of an aliphatic
hydrocarbon. Transient, mild CNS depression is common after
hydrocarbon
ingestion or inhalation. Aspiration is characterized by coughing,
which usually is
the 1 st clinical finding . Chest radiographs may initially be normal, but
they often
show abnormalities within 6 hr of exposure in patients who have aspirated
.
Respiratory symptoms can remain mild or progress rapidly to acute respiratory
119
syndrome (ARDS) and eSpiratory « ..
stress accompanying sien. .
common
necessarily imply bacterial
'
er,ts
suDe' rPat' hT
Ure- feVer

'
' “W
* Dn mo are
,
nfec < n. If
respiratory treatfn ' ° ^ <, „, Ce
not
develops Pneu
prop
ther
Wn anV c/ear K * ^'
'
nor
and lavage
ioo . charco k not
.(D)1 Emesis
3r

the ,
ACivated andalcan also i n d " bec se
^
Siting Th ° "
d es ot h ‘ aspiration
j( rK
of
hydrocarbonstoxic ost S , *
' “ * c fn0

irrigation
. Sinc

mm m theinsetrea
e
ity is asolran P eu
of kerosene kerosene odo
indicated of exposed areas is re
tment of
r J
^ -
" 6

'
,^
t

"
°
°
"
o,tis s

ded
the

Po sorw
H
'
°' ^hole bo**»oi w
aw «at
irriga
hs, removing all
tion*
cloth and ing
ion
is not

S ’ "*
nta > nationft **, * °' ean .

^
carbamates ctici des basjc 0phosPh S«« and
exposed skin *° *
" * d* mediate,
mb °*P and « be perf0
.
cashing all
exposed clothing. Acti Coa is unlikei to "pf
(
b " C * 3
lnclPdmg

are liquids that are rapidlv =l


Diagnosis
.
^ * use these

* W
anVlan T^
^
carb p sonine
on history
deadPselyudocholinesterase
and
eXam °'
such, As
these assessments are typically limited to farmworkers
undergoing ongoing
occupational surveillance. Clinical manifestations of
organophosphate and
carbamate toxic ity relate to ACh accumulation at peripheral
nicotinic and
muscarinic synapses and in the CNS. Symptoms of carbamate toxicity are
usually
less severe than those seen with organophosphates. A commonly
used
mnemonic for the symptoms of cholinergic excess at muscarinic receptors is
DUMBBELS: diarrhea/defecation, urination, miosis, bronchorrhea/
bronchospasm, bradycardia, emesis, lacrimation, and salivation. Nicotinic signs
and symptoms include muscle weakness, fasciculation, tremors, hypoventilation
( diaphragm
weakness), hypertension, tachycardia, and dysrhythmias. Severe
manifestations include coma, seizures, shock, arrhythmias, and respiratory
failure.
^ ( E) In the treatment of pediatric poisonings with organophosphates

carbamates, the use of succinylcholine for rapid sequence intubation


same cholinesterase enzymes tW «' *
avo ded because the
|

^ lize this neuromuscular blocking agent,


tabo JJ J
> «) Two on,Mo,ei „o UtoM to ««
leadin g to


» wo „Mm. Ml» «*» “ Z
** ” *
120
receptor
Often’
''infusion
continuous mrusion to v-
ui .
». « « symptoms. Atropine dosing
iu control wv rng is
^ ) S prj
ori *
to drying the respiratory secretions. Pralidoxime breaks the bond tlJ ^
organophosphate and the enzyme, reactivating AChE. P lidoxi * 1
^ ' '*'
effective if it is used before the bond ages and becomes ^ ' 5
Pralidoxime is not necessary for carbamate poisonings perman
because the
between the insecticide and the enzyme degrades spontaneously
105.(B ) Pyrethrin and pyrithroid exposures can lead to allergic
reactions
from dermatitis to urticaria to anaphylaxis. Acute exposure
can result
headache, nausea , dizziness, tremors, ataxia,
choreoathetosis, loss
consciousness, and seizures. The severity of the symptoms
depends on [
magnitude of the exposure.
;
106.( A) Compounds with low viscosity
and high volatility, such as mineral spirit
naphtha, kerosene, gasoline, and lamp
oil, spread rapidly across surfaces a
cover large areas of the lungs when aspirated
. Only small quantities (<1 ml)
such chemicals need be aspirated to produce
significant injury.

121
Urgency Medichve
r,
and Cri
tica| r are
Questions
ci MAHDI
following is the MOST c
ofthe
1. ^
yhich
fe
°
rnmori r ason
^ f0r a
Sicl> -Chi|d v
A .
6 \/
orniting *sit ?
c cough
n Diarrhea
nominal pain
l A 4V
ear old -child presents with stridor,
drooling and limitat
: mo«
ln
'

^Intension
He had history of upper
shows a febrile child with
of the neck.
respiratory infectio
toxic appearance
ton of neck
n 4 days
and limitatiion
ago.

hvpehof the following is the MOST likely diagnosis


?
of

Diphtheria
g^ foreign body aspiration
C Bacterial tracheitis
D. laryngotrachobronchitis
E. Retropharyngeal abscess

3. What is the normal respiratory rate range in premature


babies?
A. 20- 50
B. 25-55
C. 30-60
D. 35-65
£. 40-70

You re ln supermarket where a 2 - year- old girl suddenly develops cough


?
while she was eating
candy, she was conscious and mildly dyspneic.
w the following, .
the MOST appropriate initial ac i jj
(

A
- permit to cough
spontaneously
pen the
° airway
122
compression
C start chest
D. start back blows
and chest thrusts
E. start abdominal
thrusts

time to start intraosseous access


5. The recommended
cardiopulmonary arrest who
approximately
A.
B.
1 minute
2 minutes
failed to have intravenous j
^^ nt
H

C. 3 minutes
D. 4 minutes
E. 5 minutes

after road traffic accident


6 . A 13-year -old boy brought to emergency unit
Examination reveals muffled heart sounds, jugular venous distention, and
hypotension (with narrow pulse pressure ) .
Of the following, the MOST likely diagnosis is
A. tension pneumothorax
B. mediastinal emphysema
C. major flail chest
D. massive hemothorax
E. cardiac tamponade

7, Which of the following is the MOST


common type of shock in children
worldwide ?
A. Obstructive
B. Distributive
C. Hypovolemic
D. Cardiogenic
E. Anaphylactic

8. A 7-year-old girl brought to


emergency unit after road
neck veins, trachea traffic accident She
was dyspneic, cyanosed, distended
intercostal retractions, tympanic percussion of
the
deviated to the left,
breath sounds on the right side, and pulse right chest diminished
oximetry shows Po
of the following the MOST likely diagnosis is 2 84% at
room air.
A. tension pneumothorax
B. hydropneumothorax

123
chest
major flail
C hemothorax
massivetamponade
n

£. cardiac
sents with pam1u\
rpat,
< ** ^
af-o b
^° erythematous
ent had history of vesicular lesion In
travel to snowy the
« & of #
oST UkeW
resort lday
before
of the ^8"°»* 15

A. Pa
6.

D. chilblains
E. cryoglobulinemia
-old child presents with severe abdomi
w. A 12-yearreceiving
after an elbow blow while plavinm " °mitin
V
6 which
With his fr nds.
starts
Examination shows normal vital signs with I ! ' ,
tender 0 stended abdomen and lap *
belt mark across the abdomen.

*? >

“" * « « ,,
T W» ma> ,
B. Pancreas
C. Spleen
D. Bowel
E. Kidney

in shock.
Which of the following is the NEXT appropriate treatment .
A. Ringer lactate solution
B. Albumin
C. Fresh frozen plasma
D - Packed RBC
E. Ionotropic drugs

12. Which of the following


represents the mildest spinal
A. Spinal shock
B . Transient quadriparesis

124
V

.
r LOSS of sphincter
function
n <- pnsorv level
corresponding to the level of injury
° he (evel(Cl-C2)

13. Which of the


following arteries is MOST commonly
cannUiated ,Orfr
arterial sampling? %
A . Radial
6. Ulnar
C. Dorsalis pedis
D. Posterior tibial
E. Femoral
be closed ?
14 . Within what time high risk wounds should
A. 2 hours
B. 4 hours
C. 6 hours
D. 12 hours
E. 24 hours

15. Which of the following intracranial pressure ( ICP ) is considered significant ?


A. > 10 mmHg
B. >20 mmHg
C. > 30 mmHg
D. >40 mmHg
E . >50 mmHg

16. A 5-year-old boy brought to causality


, unconscious after road traffic accident,
he opens h s eyes in response to voice,
utters inappropriate words, he has
abnormal flexion to painful stimuli.
According to above data his Glasgow Coma
Scale (GCS) score is
A- 7
B. 8
C. 9
D. 10
E. 11

17. Which of the following drugs used in the treatment of


shock
vasoconstriction with no significant effect on cardiac contractility ? has potent
125
A
Dopamine
B
pobutamine
c Epinephrine
Q Norepinephrine
Ephedrine
^
A simple guide for pediatric blood pressure is that
18. the lower limit of systolic
A nressure for 1 10 year old should be
-
b
° A (A g e
x 2 ) +50 mm Hg
x 2 ) +60 mm Hg
B ( Age
c ( Age x 2 ) + 70 mm Hg
x ) 80 mm Hg
0 (Age 2 +
mm Hg
E . { Age x 2 ) + 90

Which of the following may be the only initial clinical


l9. manifestation of
shock ?
A . Tachycardia
g Delayed capillary refill
C. Decreased urine output
D . Alteration of mental status
E, Hypotension

20. Which of the following clinical findings is a late sign of shock?


A. Tachycardia
B. Tachypnea
C. Decreased urine output
D. Delayed capillary refill
.
E Low blood pressure

feeding. Examination
21. A 9-month -old infant presents with fever and poor
output less than 0.5
shows temperature 39°C, heart rate 180 beat/minute, urine positive
m|/kg/hour, blood gas analysis shows metabolic acidosis. Blood
culture
for gram negative bacteria.
Of the following, the MOST likely diagnosis is
A. systemic inflammatory response syndrome
8 . sepsis
-
C severe sepsis
-
0 septic shock
126
organ dysfunction syndrome
E. multiple-

the MOST common cause of syncope in


22. Which the following is the child
A . Vasovagal syncope
B. Panic attack
C. Exercise
D. Valsalva
E. Breath-holding spells

23. A 16- year- old boy presents with syncopal


attacks after head rotation
dUrifte
shaving his beard.
Which of the following is the MOST likely cause ?
A . Dysautonomia
B. Exercise
C. Stroke
D. Carotid sinus sensitivity
E . Panic attack

24. Which of the following investigations must be done in all patients


presenting with a first episode of syncope ?
A. Blood sugar
B. CBC
C. ECG
D. Echocardiography
E. Holter monitoring

25. Which of the following


medical conditions catachrestically has rapid and
shallow breathing ?
A . Pneumonia
B. Laryngotracheitis
C. Renal tubular acidosis
D. Encephalitis
E. Asthma

26. Which of the following diseases is


characterized by 8°runting ?
A . Mediastinal mass
8. Laryngomalacia
C. Diaphragmatic hernia

127
0 Foreign body aspiration
E. Bronchiolitis

27. What is the recommended ,ni


n lal
ni , ,
' dose of cardaversion
A. 2 joule/ kg in childrer<
B . 4 joule / kg with VT ?
C. 6 joule/kg
D. 8 joule/ kg


E . 10 joule/kg

S“:r:rA.
B.
Nonfatal drowning
Near drowning
!
- • *e denoted to describe survival of the
C. Secondary drowning
D. Silent drowning
E. Active drowning

29. Accessing venous line is an iimportant step in pediatric


.
Which of the following veins in the upper extremities is resuscitation,
the easiest to access?
A. Median antecubital vein
B. Veins on the dorsum of the hand
C. Cephalic vein
D. Median vein of the forearm
E. Basilic vein

30. Which of the following small 2nd degree burn is an indication for admission?
A. Upper chest
.
B Lower abdomen
C. Thigh
D. Forearm
E. Hand

ow,n8 Pediatric age groups has the


highest death rate from
31.
drown^n' 8^?
, ^
C

A - 1-4 year
B. 5-7 year
8-11 year
128
9
2.l V* **
MOsT common
place of dr 0
0 i 9 Wnj,
f £ *t * ,s the ns V
the
f
A
% °
vVh »ch
32
iof»ots
ool

8 w
'sid bucKet
r ho^se ditches
tion
D. » rn
^
f . River emergency department
hf to with
hi%
a sWjmming pool.
brougm

-
1
ZZZX
»'
* * "
SSSS * ** “"
dr0
nside red the anticonvulsan t drug of
choice?
A

B.

Midazolam
Phenobarbital
C. Diazepam
D. phenytoin
E. Valproic acid
raise suspicion of child
of the following hand
burns should abu ,
34. which
A. Splash pattern burns
*
B. Scald burns to the side
of the arm
C. Friction burns
D. Glove burns
E. Palm burns

35. Which of the following is the initial drug of choice in drowning victims w-
cardiopulmonary arrest ?
A. Dopamine
B. Dobutamine
Epinephrine
Norepinephrine
E. Phenylephrine

36. Which
managementn
g nt ,/ *
fol 0win6 is
16 ,
fa child with sey considered
,
A 2° ere
Hyperventilation
3r

Dra nnSeC
traumaticc brai | ° nd
^
injury ( TBI )
therapV in
*
?

129
0. Osmolar agents (mannitol)
mechanical ventilation
c Controlled
'

ad of bed elevation
^
Analgesia
I

positi n y u
' ° °
following, the MOST likely helpful ,es, J
and tunnelvision while
Chvcardita syndrome
3
U
pressure POTS.
A Blood measurement
0. Electrocardiogram
C Head up tilt test
*

p Holter monitoring
£ Echocardiogram

38- A 10-month old infant diagnosed with septic


-
boluses of normal saline up to 80ml/kg he stills
t .
seconds, and his urine output less than lml/kg/hour
tachvoneTcapillaryreceiving IV
refill time 4
Of the following, the MOST appropriate next step is
A . Blood transfusion
B. IVfrusemide
C. IV hydrocortisone
D. epinephrine
£. norepinephrine

39. A 10-month-old infant develops septic


shock; after receiving IV boluses of
normal saline up to 80ml/kg and dopamine IV
infusion he is still tachypneic with
capillary refill time of 4 seconds, and urine output
less than lml/kg/hour.
Of the following, the MOST appropriate
next step is
A. IV hydrocortisone
Blood transfusion
C- milrinone infusion
• start activated protein concentrate
£. ECMO

0Which of the following formulas estimates the proper internal diameter (ID)
0
epdotracheal tube ?
10 = (age ( year) 2 } + 2
/
' 10 = (age (year ) 3 } 3
/ +
130
C. ID = {age (year) / 4} + 4
*I
D. ID = {age (year) / 5} + 5
E. ID = {age (year) / 6) + 6

41. A 10- month- old infant presents with diarrhea and


Examination shows rapid deep breathing due to metabolic severe
dehVdr%
Which of the following formulas indicates a normal acidosis
respiratory
response ?
A . pH 7.15 PaC02 30 mm Hg, and [HC03- J 10 mEq/L
;
" "'
’ Kns*:
B. pH 7.20, PaC02 15 mm Hg, and [ HC03- ] 7.5 mEq/ L
C. pH 7.01, PaC02 10 mm Hg, and [HC03- J 10 mEq/L
D. pH 6.20, PaC02 10 mm Hg, and [HC03- ] 5 mEq/L
E. pH 7.27, PaC02 26 mm Hg, and [HC03- ] 12 mEq/L

42. Which of the following situations causes unreliable pulse oximetry


results
A. Deep sedation
B. Pulmonary edema
C. Mechanical ventilation
D. Poor perfusion
E. Large left-to- right shunts

43 . Which of the following symptoms is fundamental in the diagnosis of acute


mountain sickness AMS ?
A . Nausea
B. Vomiting
C. Headache
D. Dizziness
E. Light headedness

44. Which of the following is the leading cause of burn in


children <4 years old?
A. Scald
B. Flame
C. Steam inhalation
D. Chemical
E. Electrical

131
Chapter
**
Emergency Medicine and
Critical Care
Answers
AOEEL MAHD1

are
a sickchild visit Mos
-

However, pediatricians
— °f
need to be
aware of the age- dependent potential for serious bacterial infections, such as
urinary tract infection (UTI ), sepsis, meningitis , pneumonia , acute
infection, and osteoarticular infection.
abdominal

2.( E) in toxic-appearing children with stridor, the pediatrician should consider


epiglottitis, bacterial tracheitis, or a rapidly expanding retropharyngeal abscess.
The incidence of epiglottitis has greatly declined with the advent of the H.
influenzae type b ( Hib) vaccine, but it remains a possibility in the unimmunized
or partially immunized patient . Children with retropharyngeal abscesses may
also present with drooling and limitation of neck movement ( especially
hyperextension ) after a recent upper respiratory infection or penetrating mouth
injury .
<
3. E)
4.{ A) A conscious child suspected of having a partial foreign body obstruction
should be permitted to cough spontaneously until coughing is no longer
effective, respiratory distress and stridor increase, or the child becomes
unconscious .
5.(A) If venous access is not available within approximately ! min in a child with
cardiopulmonary arrest, an IO needle should be placed in the anterior proximal
tibia ( with care taken to avoid traversing the epiphyseal plate ).
6.(E) Cardiac tamponade, which is a form of obstructive shock, may be
suspected clinically or diagnosed by focused assessment with sonography in
trauma ( FAST) examination or echocardiography .
Beck ' s triad: decreased or muffled heart sounds, jugular venous distention, and
hypotension ( with narrow pulse pressure) are signs of cardiac tamponade.
Cardiac tamponade is best managed by thoracotomy or pericardial window ,
although pericardiocentesis may be necessary as a temporizing maneuver.
7- (C) Hypovolemic shock is the most common type of shock in c i
worldwide, usually related to fluid losses from severe diarrhea .
132
(p)Tension P
nPumofb * c when air accumulates
ra rurs
° °lung
" adjacent ^ under p

compacted, the
mediastiandnum
*
heart, great vessels,
>
I
°la are comprassed 0
..red
*****
% is a fjnjurv in which erythematous
rrn 0

^^ J^SS
• \
Chilblain
ulcerative «JP'° ccur The > .° ejn
c are presumed to
be of
,
va
vew
'
nS
, and cuu
r $
They are fte maY be painful
VaStngSand ° "
t often found on the ears,
the
scaP 8'
***° ^ ’ ^
f e6S'
^^*
so (Dl Specif sympt n and signs
ndthepotential Pfor lar
.njuries Pa
| °' *. '" £SKIft
tnic '.Tmioal
trauma- AlaP acr0SS the abdo raises concern for bowl
,
2££ £%
-
(and pelvic) CT s wjth v contrast med *
*" £
«
is the prefer
study in a
negative predictive value ( NPV) of 99.6%.
.
11 (D) if the patient remains in shock after boluses totaling 40- 60 mL/kg ol
**"* '
crystalloid, packed red blood cells should be transfused.
12.(B) The mildest injury to the spinal cord is transient quadriparesis evident fo'
seconds or minutes with complete recovery in 24hr. Significant spinal cord
injury in the cervical region is characterized by flaccid quadriparesis, loss of
sphincter function, and a sensory level corresponding to the level of injury. An
injury at the high cervical level ( C1-C2 ) can cause respiratory arrest and death in
the absence of ventilatory support.
13.(A) Arterial access is indicated when care providers
need frequent blood
sampling, particularly to assess adequacy of oxygenation
, ventilation, or acid-
,
base balance and/or continuous blood pressure
monitoring The radial arteiy,
the most commonly cannulated artery, lies on
the lateral side of the anterior
wrist, just medial to the styloid process of the
radius
14.(C) Although it is accepted that the time from
injury to repair should be as
bnef as
possible
to mmrmrze he nsk of infection,
accepted guideline as to what length of time is too Inna is no
there universally
A
closure. Also, this length of time varies for different types
oMacerations *
prudent recommendation is that higher -risk wounds should
be
hr at most after the injury but that some low-risk wounds e COSed Wlthin 6 f
lacerations ) may be closed as late as 12-24 hr. ' -
* - g , clean facial

133
!5.( B) Significantly raised ICP (>20mmHe»
can occur early
br 3in injury, but peak ICP generally js after severe traumatic
therapy rnay persist for longer than
seen at 48 - 72 hr . Need for ICP-
a week. directed
.
16 ( C )
EYE OPENING ( TOTAL POSSIBLE POINTS A )
Spontaneous ( 4 ) ‘
To voice ( 3 )
To pain ( 2 )
None (1)
VERBAL RESPONSE ( TOTAL POSSIBLE POINTS 5):
Older Children Infants and Young Children
Oriented ( 5) Appropriate words; smiles,
Confused ( 4) fixes, and follows (5)
Consolable crying ( 4 )
-

Inappropriate ( 3) Persistently irritable ( 3)


Incomprehensible ( 2 ) Restless, agitated 2
None 1 ( ) None (1)
MOTOR RESPONSE (TOTAL POSSIBLE POINTS 6):
Obeys (6)
Localizes pain ( 5 )
Withdraws ( 4 )
Flexion ( 3)
Extension ( 2 )
None (1)
.
17 (D)
Dopamine: 1s cardiac contractility, at high doses significant peripheral
vasoconstriction.
Epinephrine: T heart rate and T cardiac contractility, potent vasoconstrictor.
Dobutamine: T cardiac contractility peripheral vasodilator.

..
Norepinephrine: Potent vasoconstriction no significant effect on cardiac
contractility.

,,
Phenylephrine: Potent vasoconstriction
.
18 C A simple guide for pediatr c b ood
systolic blood pressure should be >60 mm g
limit of
Hg for 1

age ) for 1-1 V and >g0 mm Hg for any


mo l yr olds; >70 mm Hg + 2 ( «
child older than 10 yr
as only tachycardia, with or without
19-(A) Shock may initially manifest
output, poor peripheral
.
tachypnea Progression leads to e .
tion 0f mental status and low BP
Perfusion, respiratory distress or failure ai
,

1 DA
,n ici
A significant misconception is that shock occurs Only
°TlTbecause ,*
.
'
is often a late
finding and is not a criterion f0r K ith
on low to
preserve and BP
^*NXJ
of a complex set of compensatory mechanisms that
peripheral perfusion. Hypotension reflects an adva

a'd Ju* s
of

Qr
*
intravenous
.
decompensated shock.

« »** dysfun
» l organ
fluid
w
in 1
* ction, defmud
hr:
» Hypotension<5th percentile for age
1
as: Despu > 0 ml,, « ,,,„
or systolic blood pressure <2
SD
ls
s
N °
normal for age or
•Need for vasoactive drug to maintain blood pressure
or
Two of the following:
•Unexplained metabolic acidosis: base deficit >5 mEq/L
•Increased arterial lactate:> 2 times upper limit of normal 1

•Oliguria: urine output<0.5 mL/kg/hr


•Prolonged capillary refill:>5 sec
•Core-to-peripheral temperature gap:>3°C
22.( A) Vasovagal syncope is classically associated with a prodrome that
includes
diaphoresis, warmth, pallor, or feeling lightheaded and
is often triggered by a
specific event or situation such as pain,
medical procedures, or emotional
distress This type of syncope is characterized by
hypotension and bradycardia.
Approximately 30-50% of children will have
had a fainting episode before 18
yr of age.
23.(D) Carotid sinus: syncope with
head rotation or pressure on the carotid
sinus (as in tumors, shaving, tight
collars )
?04;CQTA;i,Pe
(

^ 7T
St electrocardi m obtained, looking primarily
lnrn °^
abnormalities, and conductTon aZSit'S ”"**"»*' T
* ~
breathinTircharaaer
^ ^^ ^. ^
pulmonary edema, 00
tidal volume) al y rap ,
pid J 7 PneUm0n 3
'
< an<f shallow ( decreased
26.(E) Grunting is an expiratory
sound caused bv suririo„
during expiration in an attempt to
prevent alveolar atelectasis, most 77 7 ^
maintain functional
commonly heard in dise
® f
°
C3pac,ty FRCand
FRC (e.g., pneumonia, pulmonary edema ) S decreased
and periDhprai
( e.g., bronchiolitis ). airway obstruction

135
Energy for Defibnlh
,
2J.
) Shock Shoc 4 i/kg
( A
«,definit *dr n>ng he pr< ess
Vn n
1< »e.
.. *
° ' '_
roir> sub" °*
}

of experiencing
respiratory /mo
IT * "' , !
,
, '
mmersion *
liquid." in

fa,a Of* non


drowning does not V th< in
' «»> or
l o ?
The term
outcome should be denoted
inology should improve
? ' J
COn
S
* #
0se of «
the
« a,n ' ><J r„ * h; the use •of *
term
confusing
descriptive terms such
* ^ wetF "*
" *ft
fep
ft
dry " "secondary," "silent,"
,


,'and "active " shou d bp
(
abahdoned
swe
>W Veins suitabie for ca ,
» variation fromp , ”*pat' aret.%
anatomic vein, focatedin*,he tecub*" l fossaP
, m s
10 1
°“ - there «
T
* ,he gesi1' eas «"* 1
med,a*n
Con

entecubital. Many veins on { d rsur** of ^


n r Amities
,he hand are'^also " *
to access ° °
'

^ "
Cd^ *
-
they are ofte large
cannulation isbecause * ”
elation
« « «affecting
generally
»« admissio,n
^ >1Kr ° «AeUrns
t erat6d
“ and their

* V >Borns
10-20% of 8 °* Sajn
(

V
v 3rd-degree burns ^ dult escent

Electrical burns caused by high-tension wires or lightning Chemical burns


/ inhalation injury, regardless of the amount of BSA burned
Inadequate home or social environment
Suspected child abuse or neglect
/ Burns to the face, hands, feet, perineum, genitals, or major joints
/ Burns in patients with preexisting medical conditions that may
complicate the acute recovery phase
s Associated injuries ( fractures)
^ Pregnancy
.
31 ( A) Drowning rates are consistently highest in 1- 4 yr old children, likely
because of their curious but unaware nature, coupled with the rapid
progression of their physical capabilities. The highest drowning death rates
were seen in children age 1-4 yr and 15 -19 yr ( crude rates of 2.56 and 1.2 per
100,000, respectively ).
^ (B) Most ( 71%) drowning deaths in children younger than 1 yr occur in the
bathtub, when an infant is left alone or with an older sibling. The next major risk
to children <1 yr is
the large ( 5 -gallon) household bucket, implicated in 16% of
lnfant drowning deaths.

136
or phenytoin ( loading dose
of 10-20
33.(D) Fosphen ytoin

^ ° •O*O'
dosing with 5-g of
by maintena nce
equivalents/kg, followed divided doses; levels should be 8 f
equivalents/ kg/day in 2-
3 mon
;it may have some
considered as an anticonvulsant
neuroprotectj
pulmonary edema. Benzodiazepines, bariv
6
ffects an<

may mitigate neurogenic


j

may also have some role in seizure therapV< 'tUratfs,


other anticonvulsants
shown improved neurologic outcome. altSni.
conclusive studies have
34.(0) Approximately 18
% of burns are the result of child abuse ( u
making it important to assess
the pattern and site of jn u ,
Vsca|ds)*
theif
consistency with the patient history. Glove
or stocking burns of th
-area deep burns on the trunk, buttocks , or back; and and ^
^
feet; single
thickness burns (e g., cigarette burns) in young children should ra,Se the ^
suspicion of child abuse .
35.(C) Epinephrine is usually the initial drug of choice in victims with
bradyasystolic cardiopulmonary arrest (IV dose is 0.01 mg/kg using the i:
10,000 (0.1 mg/mL] solution given every 3-5 min, as needed). Epinephrine can
be given intratracheally ( endotracheal tube dose is 0.1-0.2 mg/kg of 1: 1,000 (1
mg/mL] solution) if no IV access is available.
36.(A)
First Tier: intubation and controlled mechanical ventilation, head of bed
elevation, sedation and analgesia neuromuscular blockade, osmolar agents
(hypertonic saline, mannitol)
Second Tier: barbiturate infusion, decompressive
craniectomy, mild
hypothermia, hyperventilation, lumbar CSF drainage
37.(C) To diagnose POTS the patient needs
to undergo a head-up tilt test for at
least 10 min. It is important to have the patient
supine for at least 20 min before
the tilt test. POTS can also be assessed by a
standing test, measuring BP and HR
at 1, 3, 5, and 10 min standing, but
to have a reliable test
the patient needs to be supine for 1hr similar to the tilt test,
before standing.
Other tests may include
electrocardiogram, echocardiogram
monitor when there is concern of a , and Holter
primary cardiac cause
there is a need to determine if symptoms of tachycardia or if
correlate with tachycardia. Supine and
standing plasma catecholamines help
confirm the diagnosis
expects to see either the normal doubling of POTS as one
of norepinephrine
to standing, or a tripling with hyperadrenergic levels from supine
POTS. H
38.(D) Rapid fluid resuscitation totaling 60-80 mL
/kg or more is
improved survival without an increased incidence
of pulmonary
associated with
resuscitation in increments of 20 mL/kg should be edema Fluid
titrated to normaliz
HR
137
rHing to age'hased ^ Rs), urine outout / ,

,jfS sec and


'° /phrine
(
,kke
of volume
), mental
resuscitation '
status, if

, should instituted wfSe


shock
vasopressor
,
*

^'
«<6/ hr). capi||
refract0 rV
arv ref
follows
.
tnT
^ be th

4 X2
j resu **"« «? .
epin
pediatric goidehnes
Lnephrine .
for septic
shock unresponsive
or dopam ne whereas adult tofl -
3re"
sugRes*,
*
. *itation
guideline
“e
function is another rPr
Adrenal
3. ( A)
replacement may be
important
"°in«fpineph
drocortisonehave
may absolute or relative beneficial. up to
patients insufficiency insuffici
adrenal
include those with ent °
" ^ Shock and
f Crltic3 -
V 1«
adrenal
corticosteroids
of the hypothalamic-pituitary congenial
abnormalities including
( those
and inflammatory bowel disease) These .
tn
axis
with asthma, rheumarirV*
*"
risk
hVP°Plasia
therapV
mali ancies
^ 6031
1
at

8
,
with
am patients
dysfunction and should receive stress doses of hvdrn - h nSk for 3drenal ! " ,

may also be considered in patients with shock


resuscitation and catecholamines .
3
'
Vfh
rtlsone- C fticosteroids
° to
s unresPonsive


fluid
40 (C)


41.(E) Normal compensation does not completely
minimizes a change in pH that would
* « °r 1 m£ «' met*|HC03-].*Th,,*„„„ „ “Ln
" the adequacy offLrespiratory
analyze mmonl
bolic d .
compensation is
correct th
otherwise occur withe, ,t - PH bUt rather

Winter's formula
^^
u u

St

,,
PaC02= ((HC03-) x 1.5) +8 + 2 formula .-
A quick method is to look at the last 2
digits of pH (provided it is not
which should be within 2 mm Hg of PaC02 < 7.10) ,
.
42.(0) It is unreliable in patients
with poor perfusion and poor pulsatile
the extremities. flow to
43.|C) Diagnosis
AMS is easily identified in older children and adolescents
we Self-Report Lake using
, Louise AMS Scoring System. The criteria require that the
VI Ua
at |east ^ ,
n setting of a recent gain in altitude, be at the new altitude for
several hours, and report a headache plus
at least 1 of the following
° ^
mS astro , ,
ntGst nal (Gl) upset ( anorexia, nausea, or vomiting), general
we kn < ss or
44 ( A ,! dizziness or lightheadedness, or difficulty sleeping.
; cald burns account for 85% of total injuries and are most prevalent in
child <4
l

nfect°fn ^ °^ ^ ^
nS IS ano
‘ team halation used as a home remedy to treat respiratory
ernaj ^ er potential cause of burns. Flame burns account for 13%, the
nS are electrical and chemical burns.

A *5 0
Chapter
^ Human Genetics
Questions
ZUHAIR ALMUSAWI
following is an Indication for genetic counseling ?
1. Which of the
A. Paternal age 235 yr
B. Paternal age 40 yr
*
C. Paternal age £45 yr
D. Paternal age £ 50 yr
E. Paternal age 255 yr

2 . Genetic counseling has a major role in risk assessment for which of the
following cancers ?
A . colon
B. Prostate
C. Brain
D. Liver
E. Skin

3. Which of the following diseases is usually treated


with enzyme replacement
therapy ?
A. Niemann-Pick disease type B
B. Neuronal ceroid lipofuscinosis
C. Pompe disease
D. Mucopolysaccharidoses MPS IMA
E. Metachromatic leukodystrophy

4. Which of the following tests can


provide information about the copy
and location of a specific genomic region py number
?
A. Array-based copy number detect
ion assays
B. Chromosome analysis ( karyotype )
C. Fluorescent in situ hybridization (FISH)
D. Whole exome sequencing ( WES)
E. polymerase chain reaction

139
5 .
rue finding * e?
J inheritanc
o male to- male
Uansmission e
*sential(y confirms which of the
f0
"°A x linked dominant inheritance
.linked recessive inheritance
^autosomal dominant inheritance
C ’

tosomal recessive inheritance


^
t
v‘
1'
^^°
git,factorial inheritance

emission?
f
*
,° l 0Wing mher«
^nces ,
is
characterized by
trafl horizontal
A x -linked dominant inheritance
® X -iinked recessive inheritance
Autosomal dominant inheritance
1 Autosomal recessive inheritance
E. Multifactorial inheritance
of e
7. Which type inheritanc is expected if a woman with a genetic
offspring of either sex but an -\ disorder can
have affected affected father cannot pass on the
offspring?
d(Sease to his
A pseudogenetic Inheritance
3 Digenic inheritance
C Mitochondrial inheritance
D. Autosomal recessive inheritance
E . Autosomal dominant inheritance

8. When 2 chromosomes fail to separate during meiosis and


thus mi
migrate
together into one of the new cells, producing l cell with
2 copies of the
chromosome and another with no copy .
What is the name of the above mechanism of cell division ?
A. Nondisjunction
B. Anaphase lag
C. Genetic recombination
D. Crossing over
E. Dislocation

9. Which term is used to describe the presence of 2 or more cell lines in a single
individual?
A. Euploidy

140
£
B Mosaicisnn
C Polyploidy
0, TriploidY

10 Which
E. Aneuploidy

is the
chromosome abnormality
A . Eupfoidy
?
MOST common and
clinically significant
ty
*o . ,
t
\
8. Mosaicism
.
C Polyploidy
D. Trrploidy
E. Aneuploidy

U. A couple of 38-year -old husband and a 23- year-old wife with


syndrome consult you because the husband is planning to have a baby Dowr
he
you about the chance of having a baby with Down syndrome, rf
his UA
becomes pregnant.
Your proper answer, the chance of having Down syndrome is
A. 5%
8. 25%
C. 50%
D. 75%
E. 100%

12. Which of the following is a


Hall ' s criterion to aid in
syndrome? diagnosis of Down
A. Upward slanted palpebral
fissures
-
fi Epicanthal folds
C. Speckled irises (
Brushfield Spots )
-
D Three fontanels
E - Mild microcephaly

13 A couPle brought their


, 3- dav old
Mam nation you
find midline cleft L ft °"ate WlthW abnormal features, on
hoilar low set* 'I!
Vpotelorlsm 86 th b0513 31 Po|ydactyly;
bulbous nose -
' *'
holoprosencephaly;'microphthZ - ' malforrned ears; microcephaly;
Of and
the following, the
A. Trisomy 13 *
MOST likplw cause SCalp defect '
of these dysmorphic
features is
18
rfl5omV
8
rrisomyS
Trisomy 9
Trisomy
f
, is the
average time of walking
in Down sVndror e
J * j2 months ^ ?
j 6 months
0
2 0 0lonths
^
p 24 months
28 months
^
h of the following is the MOST
common congenital h
15
en *?
with Down syndrome ? eart
^sease ,n
*"
P
Ventricular septal defect
6. y\tri3l septsi defect
Patent ductus arteriosus
Endocardial Cushing defects
r Aberrant subclavian artery

j5 _ Which of the following tests is diagnostic


and considered
advance in prenatal diagnosis of Down syndrome as an important
?
A. Free fMiuman chorionic gonadotropin
B. Detection of cell free fetal DIMA iin 10’hCGj
-
C. Unconjugated estriol maternal plasma
D. a-fetoprotein
E. Fetal nuchal translucency (
NT) thickness

-
4-year-old boy with Down syndrome
poor weight gam, presents with
abdominal progressive

ir and P r 3ppetite for


°°
distension, loose offensive stool alternatingpallor

* 0p
, cfeatUres
OH e following,
t
w th normal WBC
the MOST helpful
the last 3 months.
'
and platelets count
His blood film Ihows
A- serum B investigation is
12
B. serum folic acid
C thyroid
D function test
tissue transglutami
serum ferritin nase antibodies

142
.
f

,
18. »Md hip
MK «
hOTI >«k aref°Ver 4«
“ which1««•*» ' ** N
A . Trisomy 13
B. Trisomy 18
C. Trisomy 8
0. Trisomy 9
E . Trisomy 21
following features; hypotonia, short
19.A 3 -week old neonate with the
shrill cry , microcephaly with protruding metopic '
characteristic suture
hypertelorism, bilateral epicanthic folds , high arched palate, and wide flat
nasj|
bridge *
Of the following, the MOST likely diagnosis is
A. Wolf- Hirschhorn syndrome
B. Cri du chat syndrome
C. DiGeorge type 2 syndrome
D. Jacobsen syndrome
E. Miller - Dieker syndrome

20. An 8- year-old boy brought by his father complaining from poor school
performance. On examination, you find a friendly boy with round face, small
upturned nose, long upper lip length, wide mouth, with full cheeks and lips. Lab
investigations reveal hypercalcemia , and echo shows
supravalvular aortic
stenosis.
Of the following, the MOST likely diagnosis is
A . DiGeorge syndrome
B. Williams syndrome
C. Kallmann syndrome
D. Angelman syndrome
E. Axenfeld- Rieger syndrome

21. An 18-month- old child presents


with following
ptosis, beaked nose with low-
lying philtrum
features; microcephaly,
intellectual disability. broad thumbs, large toes, and
Of the following, the MOST likely diagnosis is
A Rubinstein - Taybi syndrome
,

8. Williams syndrome

143
C. Kallmann syndrome
p. Smith- Magenls syndrome
g. Jacobsen syndrome

22 A 3- week-old small for gestational


age
1 e neCk, protruding ears, and non- newborn presents with
pitting swellings of the h webbing of
s bicuspid aortic valve with ands
coarctation of the aorta, and feet. Echo
nfthe following, the MOST likely diagnosis
° A Noonan syndrome
is
B. Trisomy X syndrome
C. Turner syndrome
D. Jacobsen syndrome
E. Kallmann syndrome

e Noonan syndrome
from Turner
A. Short stature
B . Low posterior hairline
C. shield chest
D. Right -sided cardiac lesions
E. Webbed neck

24. What is the MOST common sex chromosome aneuploidy in humans ?


A. Noonan syndrome
B. Turner syndrome
.
C Klinefelter syndrome
D. 47,XYY
E. Fragile X syndrome

25. An 18-month-old boy presents with long palpebral fissures, eversion of


On
lateral portion of lower eyelids, long eyelashes, and blue sclera .
, hypotonia with
examination, he has global developmental delay, microcephaly
finger nails.
Joint hyperextensibility, prominent fingertip pads, and hypoplastic
Of the following, the MOST likely diagnosis is
A. Bloom syndrome
B. Kabuki syndrome
C. Axenfeld-Rieger syndrome
D- Kallmann syndrome
144
E. Smith-Magenis syndrome

M adolesced
.
.
a l0ng <a« large cupped
“ rextensible finge
recurrent
ear
r joints, s, high *
« *
****•
^ eN
^TS***1*dr°me
NoonaJsvn
^
6
C
e

27. A preschool child has prominent full cheeks, abnormal ear lobes, sparse
in the temporal regions, pigmentary skin anomalies, supernumerary
hair
nipple
infrequent seizures, and profound intellectual disability. He also had
undergone
surgery for diaphragmatic hernia in neonatal period.
Of the following, the MOST likely diagnosis is
A. Jacobsen syndrome
B. Fragile X syndrome
C. Pallister-Killian syndrome
D. Kallmann syndrome
E. Smith -Magenis syndrome

28. A 1-year- old infant presents with


hypochromic microcytic
investigations found to have (3 anemia, after full
done for parents showing normal
thalassemia major, Hb electrophoresis were
father
What is the type of inheritance in this and mother with (3 thalassemia minor.
case ?
A. Chromosome breakage syndrome
B. Uniparental disomy
C. Autosomal dominant inheri
tance
D. Autosomal recessive inheritance
E. Multifactorial inheritance

29. Which of the following is a classic example


of rmprinting
A. Prader- Willi syndrome disorder?
B. Cartilage hair hypoplasia
C. Bloom syndrome

145
Kallmann syndrome
t• Smith-Ma8enis svndrome
of the following is a minor diagnostic criterion for Prader - Willi
,hlch
30. * Feeding
syndrome
?
problems and failure to thrive as an infant

a sleep aPnea 1- 6
Weight gain at vr
c- “ dysmorphic facial features
har 3Cteristic
°
r small genitalia

146
Chapter 9
Human Genetics
Answers
ZUHAIR ALMUSAWI
counseling
1 ( B) Indications for genetic
Advanced parental age
•Maternal age £35 yr
•Paternal age £40 yr
2.( A) Genetic counseling has a major role in risk assessment for cance
especially breast, ovarian, or colon cancer, for which well -defined risk mo ,
and genetic tests are available to assess risk to an individual. ^
3.(C) Enzyme replacement therapies are available for Gaucher disease and Fabr ,
disease, some mucopolysaccharidoses ( MPS I, II, IVA, VI), acid lipase deficiency
and Pompe disease, and are being tested for MPS IIIA, MPS VII, metachromatic
leukodystrophy, a-mannosidosis, Niemann -Pick disease type B, and neuronai
ceroid lipofuscinosis, late infantile (CLN 2).
4.(C) Fluorescent in situ hybridization ( FISH) can provide information about the
copy number and location of a specific genomic region. Array -based copy
number detection assays can be used to screen for chromosomal deletions
(large and small) and duplications across the genome, but do not provide
information about the orientation or location of genomic regions.
A chromosome analysis (karyotype ) can detect relatively large chromosomal
deletions and duplications and can also be useful in identifying inversions and
chromosomal rearrangements even when they are copy number neutral
changes that do not result in a deletion or duplication of genomic material.
5.(C)
.
6 (D) Autosomal recessive disorders
are characterized by horizontal
transmission , the observation of multiple affected members of a kindred in the
same generation, but no affected family
members in other generations.
.
7 (C) An individual's mitochondrial genome entirely derived from the mother
is
because sperm contain relatively few mitochondria, and these are degradated
after fertilization. It follows that mitochondrial inheritance is essentially
maternal inheritance.

147
g ( A ) Two common errors of ce l
and either can result in an ab _ divisi0n
)

°
, °'ChUr
nondisjunction, in which 2 rrnal dur|ng mej
Ch m nurnber o
^ som

^
rnoso^es The
migrate together into
thus °
e of the es a to
0
,
°° " n error 15
**
.°* ,
of the chromosome and n<'- w Cpll SeParaie durino
ariother w h no
which a chromatid or chrom «
quickly enough during ar> sorne 15 °
aphase
i <
ost durin 2nd error js h 2 c
*
°Pies “
daughter cells .
9.(
in a
B ) Mosaicism
single
is an abnor
individual .
pregnancy losses . Tripioid ^rS”r,r""
be
' e
^
as the
Corr> mon
^ b
^
Presence of 2
ause
Porated i n o 1 ,
‘of

r mor e
«• m
niove
the new
'° *
cells abnormality
( 3 n ) and are only viable
in a
are those
» th 3 °
seen in Ist - cell lines
do not survive long . Tripioid Mosaic f orm . ' '
haPloid sets of c trimester
sperm ( dispermy ). Failur
or sperm , esn slso result
y is
e of 1 of
often the
the
JriPloid
infams can he rom osomes
,
^
live b rn but
10.( E ) Abnormal cells
m triploidy Zat » on 0f a n
, egg b y 2 °
that do not resulting i n a diploid
chromosomes are termed c egg
11 ( C ) Most males
wifh
ntain a multiple
aneupioid cells . °
0 f haploid
been able to reproduce Down sVndrome number of
12 .( A ) Hall ' s criteria with a 50% are f
Hypotonia
Poor Moro reflex
Flat face
Upward
slanted
to aid
ai in d
agnoSis
* ooOWn7° °
f haVin

^^
^ndrome inc ,
yS02Teprfema' eS

^ ies.
bave

Palpebral fiSSu
Small dysplastic res
ears
Jomt hyperflexibility
ort neck ,
Sh rt 5th digit redundant skin
° with r

13 (
transversJtZ ^
S g /e
' dySpiasia Palmar crea* es
"
Pelvk
AJ
«

14 (C )

^Bstone c ,
«

rH oo vNs ,
“ “"* mo' ' " "°«' t
filing
Ro//iln8
Sittiihg
Cra / /ng
*
over
9
11
6 ^
r ge ( 1)

2 - 12
6 - 18
D
Range (mo)
15 -3
1
5
7
UNAFFECTED CHILDREN
Average ( mo ) Range (
1.5-3
2 - 10
mo )

7- 21 5 -9
8 6 - 11
148
8-25 10
Creeping
13
11
> 13

10 10- 32 8-16
Standing 12- 45 13
20 8- 18
Walking 9- 30 10
Talking, words
14
21
-
6 14
24 18-46 14- 3J
Talking, sentences
15.(0)

« °^
«» f p
•ed ,e'
“ “
hss s
c
^ :.
are common. Other abnormal
, , **»>
t es include megakaryob * *
hypothyroidism
leukemia , immune dysfunction, diabetes
hearing and vision.
^.
melhtus, seizures, alopecia, juven i
*
idiopathic arthritis, and problems with
18.(B)
19 (B)
20.( B)

22.(C) Older children and adults with Turner syndrome have


short stature and
exhibit variable dysmorphic features. Congenital heart defects ( 40%) and
structural renal anomalies ( 60%) are common. The most common heart defects
are bicuspid aortic valves, coarctation of the aorta, aortic stenosis, and mitral
valve prolapse. The gonads are generally streaks of fibrous tissue (gonadal
dysgenesis). There is primary amenorrhea and lack of secondary sex
characteristics. Most patients tend to be of normal intelligence, but intellectual
disability is seen in up to 6% of affected children.
23.(D) In contrast to Turner syndrome, Noonan syndrome affects both sexes
and has a different pattern of congenital heart disease, typically involving right -
sided lesions.
24.(C) Persons with Klinefelter syndrome are phenotypically male; this
syndrome is the most common cause of hypogonadism and infertility in males
and the most common sex chromosome aneuploidy in humans
25.(B) Clinical Manifestations of Kabuki Syndrome
Facial
Long palpebral tissues and eversions of lateral third of
lower eyelids
Y
S Ptosis
S Broad, arched eyebrows with sparse hair on lateral
third
S Long eyelashes
4 A n
sclerae
/ glue ears
/ protuberant
/ Short
nasal columella { depressed nasal tip)
developmental
N«ur lypotonia
j-
/
/ Developmental delay ( IQ about 60 ; >80 in 10%)
Low
birthweight
Z postnatal growth deficiency
Microcephaly
/ Seizures
/ Autism
1
Ertremitv/Ske e
'
**
, incurved 5th finger
Short
Brachydactyly
/
s Kyphosis
/ Joint hyperextensibility
/ Persistent fetal fingertip pads
Z Hypoplastic finger nails
Cardiovascular
Z Multiple forms of congenital heart disease
Other
Z Nonimmune hydrops
Z Hypothyroidism
Z Precocious puberty
/ Delayed puberty
z' Lymphatic malformations
s Feeding difficulties
26.(B) A clinically significant fragile site is on the distal long arm of chromosome
Xq 27.3 associated with the fragile X syndrome . Fragile X syndrome accounts for
3% of males with intellectual disability.
27 - ( C ) Pallister - Killian syndrome is caused by mosaicism for an isochromosome
12p. The presence of the isochromosome 12 p in cells gives 4 functional copies
for the short arm of chromosome 12 in the affected cells .
28 ( B ) Uniparental
disomy (UPD) occurs when both chromosomes of a pair or
areas foom one chromosome in any individual have been inherited from a single
Parent Examples of UPD are spinal muscular atrophy, cystic fibrosis, cartilage
a r hypoplasia
' , a- and -thalassemias, and Bloom syndrome .
^
150
f
29 ( A) A classic example of
imprinting disorder is seen in
very different clinical
Prader -u/ir ,
and Angelman syndrome, 2 conditions. Thes
are usually associated with deletion
of chromosome 15, A deletion
on
of the same region in the
the paternally derived
proxim
chromes
>'"^* .; V

, while a maternal deletion of the


^ ^*
Prader-Willi syndrome same reg
Prader-Willi syndrome causes Angelman
syndrome . 11

30,( B )
Consensus Diagnostic Criteria for Prader Willi
- Syndrome
__
71
^^^
MAJOR CRITERIA (1 point each ) MINOR CRITERIA /2
1 I Neonatal/infantile hypotonia Decreased fetal movement an(j~~'L
infantile lethargy
2 Feeding problems and failure to Typical behavior problems
thrive as an infant
3 Weight gain at 1- 6 yr; obesity; Sleep apnea
hyperphagia
4 Characteristic dysmorphic facial Short stature for family by 15 yr
features
5 Small genitalia; pubertal delay and Hypopigmentation for the family
insufficiency
6 Developmental delay/intellectual Small hands and feet for height
disability
7 Narrow hands, straight ulnar border
8 Esotropia, myopia
9 Thick, viscous saliva
10 Speech articulation defects
11 Skin picking

151
Metabolic Diisorders
Questions
A

Which of the following inb errors 0f


apnea? metabolism lis associated with
neonatalHyperphenylalaninemia
A.
B. Tyrosinemia
type jj

C. Urea cycle disorders


0. Hypermethioninemia
E. Citrullinemia type / /

following anticonvulsant dmB< m,


2 . Which of the ® v cause hyperammonemia ?
A. Valproic acid
B. Paraldehyde
C. Phenobarbltal
D. Clobazam
E. Topiramate

3. Which of the following glycogen storage diseases (GSD) is associated with


cardiomyopathy?
A. GSD type la (glucose- 6-phosphatase deficiency)
B. GSD type lb (impaired glucose- 6- phosphate exchanger )
C. GSD type III (glycogen debrancher enzyme deficiency )
D. GSD type VI ( liver glycogen phosphorylase deficiency)
E. GSD type IX (phosphorylase kinase deficiencies )

4. Macrocephaly is a pathognomonic clinical finding associated with inborn


errors of metabolism.
Which of the following is associated with macrocephaly ?
A . Mucopolysaccharidoses
B. Sphingolipidoses
C- Glutaric Acidemia Type 1
D- Wolman Disease
152
i

E. Farber Disease
with macroglossia, hypotonia, \
5. A 3-month -old infant presents mcreased %
creatinine kinase and
cardiomyopathy.
Of the following, the MOST likely
diagnosis is
A. glycogen storage disease type II
B. mucopolysaccharidoses
C. oligosaccharidoses
D . sphingolipidoses
E . galactosialidosis

6. A 6 - old -male infant presents with exaggerated startle response


ophthalmological examination revealed macular cherry -red spot.
H -
Of the followings, the MOST likely diagnosis is
A. GM1 gangliosidosis
B. Tay -Sachs disease
C. Farber disease
0. galactosialidosis
E. sialidosis

alopecia ? ""
A. Multiple carboxylase
b<5rn mors of metabolism iis characterized by
deficiency
B. Gaucher disease type 2
C. Steroid sulfatase deficiency
.
D Refsum disease
E. Serine deficiency
disorders
8. Which of the following
inborn errors of
sweaty feet urine odor ? metabolism is characterized by
A. Maple syrup urine
disease
B. Isovaleric academia
C. Phenylketonuria
D. Tyrosinemia type 1
E. Cystinuria
of the following
y‘ Which characterize urea
from other types
nyPerammonemia ? of inborn
error
cycle defects with
monemia of metabolism
hVPeramNormal serum pH
with

^ High bicarbonate value


c^ Lactic acidosis
p Ketosis
E. Hypoglyc
emia

10. A 6
- week- old Infant
,
presents
with recurr nt
hepatomegaly, ascites and jaundice. Blood ammoniatacks
.of hypoglycemia,
, pH, and
values are normal . bicarbonate
8nos,s is
a„
, the MOST likely dl3 „ .
of the followings
.
A propionic academia
B. classic galactosemia
Q Zellweger spectrum disorders
p long-chain fatty acid oxidation defects
l GLUD1-related hyperinsulinemic hypoglycemia

11. A concerned family had two children with classic phenylketonuria IPKIP
PKU
consult you about the main vital organ damaged by this disease '
Of the following, the BEST answer is the
A. brain
B. kidney
C. skin
D. eye
E. lung

12. What is the BEST time to assess phenylketonurea to avoid false negative
results?
A. 1st 24 hr of life
B. 24-48 hr of life
C. 48- 72 hr of life
D. 72-96 hr of life
E . 96-120 hr of life

13 - What is cy
the safe level of plasma phenylalanine during preg
<2 mg/dL
B. <4 mg/dL
C, <6 mg/ dL
D. <8 mg/dL
E. <10 mg/dL

already on treatm ent for phenylketonuria (PKU) show 6


14 . An infant
deterio ration
despite adequate control viN
of ne urological

Of the
following, the MOST likely cause is
deficiency
A . cofactor BH4
B . noni PKU hyperphenyla
laninemia
C . transient neonatal
hyperphenylalaninemia
hydrox ylase deficiency
D. severe phenyla lanine
hyperp henylal aninem ia secondary to liver disease
E .

15 . What is the MOST common renal manifistation of tyrosinemia type 1 ?


A. Renal cysts
B. Nephronophthiasis
C. Cortical necrosis
D. Fanconi - like syndrome
E. Renal stones

16. Increased level of a- fetoprotein in the cord blood of affected infants with
tyrosinemia indicate
A . liver damage
B. renal damage
C . neural tube defect
D. cardiac defect
E. hydrocephaly

17 Which of the following result in elevated levels of urinary


succinylacetone?
A . Galactosemia
B. Hereditary fructose intolerance
C. Neonatal iron storage disease
D. Citrullinemia type II
E. Tyrosinemia type I
, month-old infant after weaning from breastfeed,*6 and
jf Reding starts to Woo introduction of
including meet and fish;
, s°0d;Ketosis- failure to thrive severe metabolic
"
add *s,s anCj an unusual
, anemia,
mild hepatomegaly
odor . Investigations , renal
show elevated level
tubular
aCid° ' henylpyruvic acid in their urine. of 4-

^losinemia . likely diagnosis is


hyd towing the MOST
0fthe
' ryrosinehiia type I
A II
B
rvrosinemia type
type III

Cn
D'
hawkinsinuria
a|Kaptonuna
E
rned parents counsel you
about their infant , who had a black
i9 A conce with ochronosis ( dark spots on the sclera and ear cartilage)
-

‘"' /following
r

^
A'
, the MOST likely diagnosis is
tyrosinemia type I
tyrosinemia type II
type W
C. tyrosinemia
Q hawkinsinuria
t alkaptonuria
is reasonable drug use as treatment option in
20.Nitisinone
A. alkaptonuria
0, hawkinsinuria
r propionic acidemia
D tyrosine
E . transient
hydroxylase
tyrosinemia .
deficiency
of he ne»born

had gene
21. A 14-year-old female
postsurgical bleeding, an a un
epistaxis,
prolonged and platelet count is .
IS
likely di g
Ofthe following, the MOST
A . Hermansky Pudlak Syndrome
-
B. Chediak-Higashi Syndrome
C . Griscelli syndrome
D . Vici syndrome
E . Waardenburg Syndrome
boy presented to emergency unit ,
22. A iO-vear-old elongated limbs, with
tall with arachnodactyiy SCo| SH
examination, he W3S valgum , pes cavus, high arched palate, and' H
iosis ,
excavatum, genu
teeth. %
BEST treatment for this condition is high
Of the following, the
doses 0f v ta
A . B 1 ( thiamine
) '%
)
B. B2 (riboflavin
C. B 3 (niacin )
)
D. B5 (pantothenic acid
E. B6 ( pyridoxine )

episodes involving both


23 In classic homocystinuria, thromboembolic larfc
and small vessels, especially those of the
brain, are common and may 0CC1J
rai
any age.
Which of the following is recommended for preventing vascular events (n
patients unresponsive to vitamin B6 therapy ?
A . Folic acid
B. Betaine
C. Vitamin B 12
D. Riboflavin
E . Thiamine

24. A 6- week - old boy present with refusal to feed, vomiting, an exaggerated
startle reaction, severe intractable seizures , cortical atrophy with subcortical
multicystic lesions, and severe developmental delay, Opthalmological
examination reveals bilateral dislocation of ocular lenses .
Of the following, the MOST likely diagnosis is
A . sulfite oxidase deficiency
B. Hartnup disorder
C. maple syrup urine disease
D. isovaleric academia
E. biotinidase deficiency

25. Which of the following inborn errors of


metabolism, a high protein diet
results in a favorable response ?
A . Sulfite Oxidase Deficiency
B. Hartnup Disorder
C. Maple Syrup Urine Disease

157
1

D
isovaleric Academia
deficiency
Bl0tinidase
£•
followingvitamins in high dose$
yyhich of the ha
26c
'
rlroical
D cn
improvement
(thiamine )
in maple syrup urine
disease?
famatic clinical and

'°A )
82 (riboflavin
g3 (niacin )
C' acid )
BS (pantothenic)
B6 (pyridoxine
E
- old infant presents with severe vomiting
, A 3-month suggesting pylonc
Gnosis ; but
investigations show severe acidosis, hyperammonemia,
ste iwremia, hypocalcemia, and bone marrow suppression. He had
ihe
hVP °ecSstjC
odor of sweaty feet .
chara the following amino acid defects is blamed in this presentation

^hich "
Tl
°Tryptophan
A
B. Isoleucine
c Leucine
p. Valine
E. Methionine

A. Hartnup disorder
B Biotinidase deficiency
C Methylglutaconic acidurias
disease
0 Classic maple syrup urine
D syndrome
E. Hyperimmunoglobulinemia
error acrodermatitis
29. Which of the following inborn 0f

results in dermatological
manifestation
enteropathica ?
A. Hartnup disorder
B. Biotinidase deficiency
C . Methylglutaconic acidurias
D. Classic maple syrup urine d
S a ,* *
dronne
E. Hyperimmunoglobulinemi
inborn error of metabolism due
f
30. Which of the following to a
dermatological manifestation in form of m 0rbiU f , rml*'"h .
0

* ? **^
in
results 0
A. Hartnup disorder
B. Biotinidase deficiency
C. Methylglutaconic acidurias
D. Classic maple syrup urine disease
E. Hyperimmunoglobulinemia D syndrome

31. Which of the following inborn errors of metabolism due t0


,
acidopathy results in dermatological manifestation in form of jnt ,actam ,
seborrheic dermatitis ? ' ^
A . Hartnup disorder
B . Biotinidase deficiency
C. Methylglutaconic acidurias
D. Classic maple syrup urine disease
E. Hyperimmunoglobulinemia D syndrome

32 . A 10-day-old neonate present as a sepsis-like picture with clinical signs of


severe metabolic acidosis, with a high anion gap. Investigations revea
ketonuria , hypoglycemia, anemia , neutropenia, and thrombocytopenia. Bran
imaging shows cerebral atrophy, delayed myefination, and abnormalities in the
globus pallidus and other parts of the basal ganglia .
Of the following, the MOST likely diagnosis is
A. Propionic acidemia
B. Isolated homocystinuria
C. Methylmalonic acidemias
D. Combined malonic and methylmalonic aciduria
E . Combined methylmalonic aciduria and homocystinuria

33. A 10- hour- old neonate presents with poor feeding


, failure to suck, lethargy,
profound hypotonia, recurrent myoclonic seizures,
and hiccups. Blood pH is
normal with negative urine assay for organic acids .
Of the following, the MOST likely diagnosis is
A . sarcosinemia
B. primary trimethylaminuria
C. nonketotic hyperglycinemia
D. primary hyperoxaluria type I
E . creatine deficiency disorder
159
Howine ,b
»
1 err0r ,
the f faboi,.m 0ue to amino
?°"'
which of
°
,n body odor that mb es th 3 fisb? °^ acidopathy

.
rPt
* . Sarcosinemia
3
'
-
result
A
0‘
C'
trimethvl
Primary

p *""*“"
°marv
5S2 *
hyperoxaluria type 1

E ^
Creatine deficiency disorder
assess the pyridoxine responsiveness in patients
TO with primary
|uria Wpe the duration of treatment trial
oeroxamonth

should be at least
A 1
6 2 months
3 months
^
p 4 months
E 5 months

boy suffers from recurrent otitis media, sinusitis , scaly


36 A 3- year old
-

erythematous maculopapular rash, purpura, and telangiectasia. These skin


lesions proceed to recurrent, severe, and painful skin ulcers on hands and legs ,

investigation reveals a high level of urinary imidodipeptides.


Of the following, the MOST likely diagnosis is
A. prolidase deficiency
0, hyperprolinemia type I
C. hyperprolinemia type II
D. phosphoserine phosphatase
E. phosphoserine aminotransferase deficiency

37. in the 1st few days of life, a full term baby develops a constellation of
metabolic acidosis, jaundice, and mild to moderate hemolytic anemia. Urine
examination shows 5 -oxoprolinuria.
Of the following, the MOST likely diagnosis is
A- B-oxoprolinase deficiency
B. hyperprolinemia type l
C glutathione synthetase deficiency
V - glutamylcysteine synthetase deficiency
E. V glutamyl transpeptidase deficiency

the following drugs in patient with
38. Which of
limitation ?
renal fail
Urg
hypoglycemia due to substrate P

A. Trimethoprim sulfamethoxazole
- *4
8. Ampicillin
C. Ciprofloxacin
D. Ceftriaxone
E. Vancomycine

39. Which of the following is the MOST


,
common manifestat' 0n
hypo
in the newborn ?
A. episodes of cyanosis
B. jitteriness
C. seizures
D. lethargy
-
E intermittent apneic spells

40. Catabolism of amino acids results in the production of free ammonia


, wHich
in high concentration is toxic to the CNS.
In healthy term infants, normal levels can reach as high as
A. 50 pmol/L
B. 100 pmol/ L
C. 200 pmol/L
D. 300 pmol/L
.
E 400 pmol/L

41. Which value of blood glucose


concentration a in sick neonate should be
treated vigorously ?
A. <25 mg/dL
B. <35 mg/dl
C. <45 mg/dL
D. <55 mg/dL
E. <65 mg/dL

42. Which of the following counter -


regulatory hormones act to
glucose level by mobilizing amino acids from increase blood
muscle ?
A. Glucagon
B. Growth hormone
C. Cortisol
Epinephrine
Thyro in
E- *
of the following porphyrias
causing non -
43 whi ch erythr opoie tic blister|r>g photo
congenital
' porphyria sensitivity ?
A
porphyria cutanea tarda
8
hereditary coproporphyria
Cn° variegate porphyria
X-Linked ProtoPorPhVria
I
ing anti - epileptic dr $
irh of the follow
yyhic B Can elevate blood
44 phenobarbital ammonia ?
A
6. Midazolam
Phenytoin
c
D Valproic acid
E. Levetiracetam

45 .Which of the following is the MOST consistent laboratory finding in


progeria syndrome ?
Hutchinson-Gilford
A Low platelet
count
protein titer
B. High c reactive
-

C. Low serum leptin


D . High liver enzyme
E. High blood urea

progressive spastic diplegia with scissoring


46 A 15- month- old boy suffers from
movements, loss of already achieved
of the lower extremities, choreoathetotic
developmental milestones, and failure to thrive
follow ing inbo rn error of meta bolis m mimics the above
Which of the
presentation ?
A. Glycine encephalopathy
B . Urea cycle disorders
C. Organic acidemias
D. Fatty acid oxidation defects
E- Mitochondrial respiratory chain defects
r
47 . A 7 - year-old boy had night blindness, myopia , loss of peripheral vj j
posterior subcapsular cataract with atrophic lesions in the retina Kion
cerebral gyri.
Which of the following is the MOST likely diagnosis ?
s
'

A . Classic citrullinemia
B. Tyrosinemia type I
C. Hyperargininemia
D . Argininosuccinic aciduria
E. Hyperornithinemia

48. A 3- day-old baby presented with generalized seizures unresponsive t0


conventional anticonvulsant therapies . Mother reports abnormal intrauterine
fluttering movements. Laboratory findings show increased concentrations of Q.
aminoadipic semialdehyde and pipecolic acid in the CSF, plasma, and urine.
Which of the following is the BEST line of treatment ?
A. Magnesium
B. Vitamin B6
C. Calcium
D. General anesthesia
E. Paraldehyde

49. A 9- month- old boy with macrocephaly and normal developmental


milestones, after a minor Infection he suddenly lost head control followed by
seizures, generalized rigidity, dystonia, and choreoathetosis. Brain imaging
reveals increased extra axial fluid with stretched bridging veins, cortical atrophy,
and fibrosis .
Of the following, the MOST likely diagnosis is
A. glutaric aciduria type 1
B. Canavan disease
C. leukodystrophy
D . hyperprolinemia type II
E. Alexander disease

50. A 8-month-old boy presents with refusal to feed


nausea, vomiting, and
mild diarrhea after ingestion of a high-protein meal Physical
findings reveal
moderate hepatosplenomegaly, thin extremities withJ
Moderate centripetal
adiposity, and growth retardation. Laboratory findings show
hyperammonemia,
163
ced ferritin, hypercholesterolo ;
serum
ea

*' and hvper nglyceriderma


iocf
flowing 5 ed
.
urinary levels of lysine , arginine
* ri

^ sT
and
, the MOST likely d agn with

I
B-
Alexander disease
cystinurea
aciduria
c glutaric protein
D familial
intolerance
£
argininosuccinic aciduria

£Hi#.iklPs==
of the following, the MOST likely diagnosis is
A , glutaric aciduria type 1
, with

B. Canavan disease
C. leukodystrophy
0 . hyperprolinemia type II
E . Alexander disease

52. An adolescent boy after prolonged exercise presented


with muscle pain and
dark color urine, his serum levels of creatine kinase was
highly
normal glucose level, muscle biopsy showed increased deposition elevated with
of neutral fat .
Which of the following is the MOST likely diagnosis?
A . Primary carnitine deficiency
B. Plasma membrane carnitine transport
defect
C. Carnitine palmitoyltransferase - ll deficiency
r
D . CarnitineiAcylcarnitine translocase
deficiency
A . McArdle disease

53. Which of the


following peroxisomal disorders has normal VLCf A levels?
A . Bifunctional
enzyme deficiency
Zellweger spectrum disorder
C* Acyl-CoA oxidase deficiency
D *
Neonatal adrenoleukodystrophy
E.
Rhizomelic chondrodysplasia punctate

164
. , n0er lower mbs; r
54.
pro
ort ** 3
'UJL circumfere"nce beta ’n
exam
t3
lrt' 08cortna c eft * **^
*>
hfaoaf«bo^
W o\o6'c
vertebr * eS
° ,' ' '

M ST KVteW d'a6n 's?


CT
' plasia ° '
s tW 0S
Mbicb *
\ (Wowid6 0

A. Achondroplasia
B. Hypochondro
C. Kyphomelic dysplasia
D. Rhizomelic chondrodysplasia punctate
E. Spondylo epiphyseal dysplasia congenital

-
55. A previously well 7-year boy presents with hyperactivity, inattention
worsening school performance, followed by disturbance of vision
seizures, and strabismus. Within one year he progressed rapidly to ‘ Hcreasing
spasticity and paralysis, visual and hearing Joss, with loss of ability to sPeak
swallow . ^
Of the following, the MOST likely diagnosis is
A. multiple sclerosis
B. Alexander disease
C. Adrenoleukodystrophy
D. Refsum disease
-
E. Charcot- Marie Tooth disease

"
heart disease in both parents . Investieatinnc ch ^ i
Ch leSter01
ifkely fevel SIILVHT
level (800 mg/dL), normal triglyceride and 8 °
Of the following, the MOST diagnosis is ° **
H L

A . sitosterolemia
B. familial defective ApoB-100
C. polygenic hypercholesterolemia
D. homozygous familial hypercholesterolemia
E. heterozygous familial hypercholesterolemia

57. A 10-year-old boy presented to emergency room as severe acute abdominal


pain. Examination shows eruptive xanthomas on the arms, knees, and buttocks,

165
-
th
hepatosplenomegaly. A milky
serum was
observed during
folding the MOST likely diagnosis is
, sample

°i *
Tangier disease
hepatic lipase deficiency
f 3 fnilial chylomicronemia
familial hypertriglyceridemia
'

£ primary hypoalphalipoproteinemia

58 Secondary diagnos
before making a .
causes of transient hypertriglyceri
s of familial
demia should be
hypertriglyceridemia
Which, of the following medications is a recognized cause of
ruled out

hypertriglyceridemia ? secondary
A progesterone
8. Thiazides
t Anabolic steroids
0, Carbamazepine
E, Cyclosporine

59. Which of the following conditions may cause reduced high - density
lipoprotein ?
A. Sepsis
.
B Stress
.
C Hepatitis
0. Malnutrition
E . Renal failure

60. Treatment of hypertriglyceridemia in children rarely requires medications.


Dietary restriction of fats, sugars, and carbohydrates, accompanied by increased
physical activity is the main steps of management.
Which of the following levels is an indication for medical treatment desp
above mentioned measures ?
A - >200 mg/dL
B. >400 mg dl
/
C >600 mg/ dL
*

k >800 mg/ dL
E . >1,000 mg dL
/

166
r

61. A
'
with a stinging sensation f the
' ° lower I .
examination

Ofttefol!
A.
showsenlarged orange tonsils and hepatosp n

owmg > e MOST likely diagnosis


Tangier disease
is
^
^ # hj %
V

B. hepatic lipase deficiency


C. familial chylomicronemia
D. familial hypertriglyceridemia
E . primary hypoalphalipoproteinemia

is associated with very low


62 . Which of the following conditions |
^eis *
plasma cholesterol and triglycerides ?
A. Intestinal lymphangiectasia
B. Friedreich ataxia
C. Abetalipoproteinemia
D. Cerebrotendinous xanthomatosis
E . Familial dysbetalipoproteinemia

63. Fasting status is a pre-requisite in assessment of which of the following ?


A . Cholesterol
B. LDL- C
C. HDL-C
D. Triglycerides
E. VLDL

64. Which of the following drugs can be used


for the treatment of elevated TG
and LDL ?
A. Nicotinic acid
B. Ezetimibe
C. Gemfibrozil
0. Statins
E. Gemfibrozil

65. A 6-month-old boy presents with developmental


psychomotor retardation and tonic -clonic seizures V’ progressive
He has
^
T
^
bossing, depressed nasal bridge, abnormally
lone P trurn' and
hepatosplenomegaly. Ophthalmological examination shows m a , '
acular cherry - red

167
survey reveals anterior beaking
i
skeleta and thiCkening of the
of the
vertebrae. enlargement
a turCI J calvarium
se
" the MOST likely diagnosis is
. of

*8 ?candho
.sachs disease
aV
« disease
C
LuchergangVdisease
.osid s,s
E GMI °
n an was developmental normal till the age of S
£ 6 An **
AO« el Ped
dC reased eye contact, loss of motor skills
months, then he
and an exaggerated
dC °
rtlc
tarte
response nnise. Examination shows
to noise
^
macrocephaly, macular pallor
*retina horrv -red spots, and

'
^
°r mucoP
.sachs
Tav
J Niemanndisease
the MOST likely diagnosis is
CcharidoseS
°' disease
Pick disease
Vs3

-
D Gaucher
E GM1gangl»os»dos»s

67 Which of the following symptoms of Gaucher disease type 1 is NOT reversed


by enzyme replacement therapy ?
A. Organomegaly
B. Hematologic indices
C. Neurologic progression
0. Bone Pain
E. Pathologic fractures
in bathing
68. A preadolescent boy had telangiectatic skin lesions distributed
,
trunk area, hypohidrosis, corneal lenticular opacities
acroparesthesia .
Of the following, the MOST likely diagnosis is
A. Farber disease
B. Fabry disease
C. fucosidosis
0. Schindler disease
E. multiple-sulfatase deficiency

168
69. The finding of lipid inclusions
with characteristic
birefhrin8em
is highly
crosses" in urinary sediment examination suggestive of *
V
A . Tay-Sachs disease
B. Niemann- Pick disease
C. Gaucher disease
0. Fabry disease
E. GM1 gangliosidosis

70. A 1- year-old child presents with developmental delay and somatic f


similar to those of mucopolysaccharidoses. These features include
bossing, hepatosplenomegaly, coarse facial features, and rnacroglossia. nia '

Of the following, the MOST likely diagnosis is


A. Farber disease
B. Fabry disease
C. Fucosidosis
0. Schindler disease
E. Krabbe disease

71. A 1-year-old boy presents with myoclonic seizures, irritability, inability to


walk, and hyperextension of the knee causing genu recurvatum. Examination
reveals absent deep tendon reflexes, muscle wasting and hypotonia, with
nystagmus and optic atrophy.
Of the following, the MOST likely diagnosis is
A. Krabbe disease
B. metachromatic leukodystrophy
C. multiple -sulfatase deficiency
D. mucolipidoses
E. GM1 gangliosidosis

72. A 9-month-old female presents with failure to thrive, painful


joint swelling,
and nodule formation misdiagnosed as juvenile rheumatoid
arthritis.
Of the following, the MOST likely diagnosis is
A. Farber disease
B. Fabry disease
C. Fucosidosis
D. Schindler disease
E. Krabbe disease
3 - week -
old male presents with

.
relentless u
A

Otorrhea, and hepatosplenomegaly Investl “T'" «om, dlstenf


. 8' ab nai

£lenal
* nds * « "" ,
als calci XMm
n
, Z
**the
*
0f *
^
following
, , the MOST likely diagnosis
is
' ons cat 0f h


Wolman disease
Fabry disease
0.
£ Fucosidosis
0 Schindler disease
E Krabbe disease
the following laboratory findings iis
74. Which of seen iin type I glycogen
disease ? storage
Normal blood lactate
^B. Low uric acid concentrations
C. Elevated serum creatine kinase levels
D. Prolonged bleeding time
E. Marked elevation in liver transaminase levels

75. A 4-month-old boy presents with feeding difficulties, floppiness,


macroglossia, and hepatomegaly. Investigation reveals elevated level of serum
creatine kinase while chest x -ray shows massive cardiomegaly.
Of the following, the MOST likely diagnosis is
A. Danon disease
B. McArdle disease
C. Pompe disease
0. Tarui disease
E. Schindler disease

?6. A 10-year - old boy complains from exercise intolerance with muscle cramps
and pain which is relieved after a brief period of rest. He reports burgundy
colored urine after exercise. Lab findings show elevated levels of seru
re$t, which further increases after exercise,
the following, the MOST likely diagnosis is
A - Danon disease
B. McArdle disease
^ Pompe disease
Tarui disease
170
E. Schindler disease
1
77. Which of the following glycogen storage diseases can pres
non-spherocytic hemolytic anemia ?
A. Danon disease
B. McArdle disease
C. Pompe disease
D. Tarui disease
E. Andersen disease

78. A 2-year-old boy with history of recurrent stones formation presented


renal colic and hematuria . Abdominal US show multiple stones ( 6 to 8m With
) in
both kidneys while KUB is clear. Mother noticed brownish spots on his ditaper *
Urine examination revealed yellow -brown crystals.
Which of the following is the MOST likely diagnosis ?
A. Hyperoxaluria
B. Hypercystinuria
C. Hyperclaciuria
D. Hypocitrateuria
E. Dihydroxyadeninuria

79. A concerned parents bring their baby because of his self -injurious
behavior.
Examination reveals mutilated fingers, mouth, and buccal mucosa . Self- biting
is
so intense that lead to amputation of right little finger and
substantial loss of
tissue around the lips. He has dystonic movements and spasticity
with moderate
range of intellectual disability . Serum levels of uric acid
7mg /dL.
Which of the following is the MOST likely diagnosis ?
A . Athetoid cerebral palsy
B. Lesch- nyhan disease
C. Dopa -responsive dystonia
D. Myoclonus - dystonia
E. Familial dysautonomia

80. A 10- year-old boy presented to endocrinology department


as short stature
with normal mentality . Examination reveals short trunk
and neck upper to
lower segment (0.7/1), genu valgum, kyphosis, mild corneal
clouding small
teeth with abnormally thin enamel, and hepatomegaly.
Which of the following is the MOST likely diagnosis ?
171
syndrome
MpS- P: Hurler
,
^"
syndrome
A p5. |; Hunter
sanfilippo syndrome
'!
C'
'
MPS 1 Morquio syndrome
MPS |V
ps - V / : Maroteaux Lamy syndrome
P -

t ^
0f the following is the SOLE
VVhich? manifestation of
Kalactok mase
,*
8
.ficiency
^ jaundice
Cataract
l Hepatomegaly
13
Hyp glVcerri
p °
E. Ataxia

Which of the following types of mucopolysaccharidoses usually presents


82 - of mild physical features ?
late becauSe
A MPS-I: Hurler syndrome
Hunter syndrome
6 MPS-H:
c MPS-III: Sanfilippo syndrome
p MPS -IV: Morquio syndrome
E . MPS- VI: Maroteaux
- Lamy syndrome

is an early marker of Hunter syndrome in African and


83. Which of the following
Asran patients ?
A. extensive mongolian spots
B. coarse facial features
C. dysostosis multiplex
D. joint stiffness
E. intellectual disability
, and
Amalnourished infant presents with feeding, swallowing problems
lesions
developmental delay together with radiologic evidence of symmetric
effecting the basal ganglia, brainstem, and subthalamic nuclei.
0f the following, the
MOST likely diagnosis is
^ Leigh Disease
B, Pearson syndrome
C. forth syndrome
D.
mitochondrial DNA Depletion Syndromes
11
'CV
chain detects
E. nuclear DNA respiratory

mslc presents with seizures, global develop


85 . A 1- year - old
and cherry -red maculae. Skeletal^
visceromegaly, corneal clouding, \
dysostosis multiplex
Of the following, the MOST
..
likely diagnos s s
S
A. sialidosis
B. a- mannosidosis
C. mucopolysacharradiosis
D. aspartylglucosaminuria
E. cerebro- ocular dysplasia-muscular dystrophy

86 . A 14-month - old female presents with feeding difficulties and speech delay
Examination shows a characteristic facial features ( short nose, long phittr
and large ears), inverted nipples, and abnormal fat pads distribution with ^
alternating strabismus, nystagmus, hypotonia, and decreased reflexes.
Of the following, the MOST likely diagnosis is
A . phosphomannomutase- 2 deficiency
B. mannosephosphoisomerase deficiency
C . glucosyltransferase-1 deficiency
D . cerebro- ocular dysplasia- muscular dystrophy
E . muscle- eye- brain disease

87. Which of the following vitamins are used for the treatment of mitochondrial
disease?
A . ( B1, B 2 , C )
B. ( B1, B12 , C )
C. ( B1 , B 1 2 ,K )
D. ( B1 , B 2 , K )
E. (B1 , B2 ,A )

88. Which of the following manifestations is


i a unique finding in all types c
mucopolysaccharidoses ?
A . Intellectual disability
B . Coarse facial features
C . Visceromegaly
D . Short stature
E. Dysostosis multiplex
Metabolic Disorders
Answers
$
apnea is associated with
J- <ClNtlYCi eenCephal0PathV
" synthetase deficiency
' 3Sparagine
' organic
Ires acidemias
ri
/
disorders
cycle

metabolism
/ disorders of pyruvatedefects
fatty acid oxidation
respiratory chain defects
/ m itochondrial
ions causes hyperammonemia
.
2 (A) Medicatio
/ Valproic Acid
/ Cyclophosphamide
/ S-Pentanoic Acid
Asparaginase
Glycogen storage diseases (GSD ) associated with cardiomyopathy
IfC)
)
GSD type II (Pompe disease
/ GSD type III (glycogen debrancher
enzyme deficiency)
/ related disorders (includes lethal congenital glycogen storage
PRKAG2
disease of heart)
Macrocephaly seen m
in glutaric acidemia type 1 and Canavan disease.
4.(CJ
5.(A)
m multiple carboxylase deficiency (
holocarboxylase
7,(A) Alopecia is seen in are causes
deficiency) while other choices
synthetase deficiency and biotinidase
of ichthyosis.
(B) tend
*'(A) Infants with elevated blood
ammonia levels from
urea cycle defects
easurement of
* ; without m
> have normal serum pH and bicarbonate valuessuccumb to t <- r

N od ammonia, they may remain undiagnosed and '
When blood .Jo*. . ond Unborn
« U>
m * should he
*aminoacidopathies hyperglycinemia or
(e.g-
pH
galactosemi )

174
. Galac tosem ic infants may also manifest cataracts,
ic
considered hePat0
ascites and jaund ice .
organ damag ed by PKU , but
Nit
main the exact
11 ( A ) The brain is the
of injury remains elusiv e. Both toxic levels of phenylalanine
and " lech«1»

tyrosine may play a role.


12.( B) It is recommended
that the blood for screeni ng be ^
obtained in the
S
g protein to reduc e the
possibility of fa|Se lst 2
<
48 hr of life after feedin
forms of the condition.
results, especially in the milder
.
1B (C) Pregnant women with
phenylalanine hydroxylase ( PAH)deficiency
are not on a phenylalanine restricted
-

lity,
diet have a very high risk of
micro cepha ly, growth
hav ( ^
offspring with intellectual disabi retardate
e
congenital malformations, and congenital heart diseas These complicatjonj
.
phenylalanine levels durj
are directly correlated with elevated maternal blood %
pregnancy.
.
14 ( A) Infants with cofactor BH4 deficiency are identified during screening
programs for PKU because of evidence of hyperphenylalaninemia . p|a$ma
phenylalanine levels may be as high as those in classic PKU or may be in the
milder range. However, the clinical manifestations of the neurotransmitter
disorders differ greatly from those of PKU.
15.(D) Renal involvementis manifested as a Fanconi-like syndrome with
hyperphosphaturia, hypophosphatemia, normal-anion gap metabolic acidosis,
and vitamin D-resistant rickets . Nephromegaly and nephrocalcinosis may be
present on ultrasound examination. Glomerular failure may occur in
adolescents and older patients.
16.(A) In untreated patients, the blood level of a- fetoprotein is increased, often
greatly, and liver- synthesized coagulation factors are decreased in most
patients. Increased levels of a- fetoprotein are present in the cord blood of
affected infants, indicating intrauterine liver damage.
17.(E) Neonatal screening for hypertyrosinemia using tyrosine alone detects
only a fraction of patients with tyrosinemia type I. Succinylacetone, which is
assayed by many neonatal screening programs, has higher sensitivity and
specificity than tyrosine and is the preferred metabolite for screening.
18.(0} Symptomatic infants and asymptomatic affected children and adults
excrete hawkinsin, 4-hydroxyphenylpyruvic acid, and its metabolites in their
urine.
19.(E) The diagnosis is confirmed by finding massive excretion of homogentisic
acid on urine organic acid testing. Tyrosine levels are normal. The enzyme is
expressed only in the liver and kidneys.
7q
, efficient/,. red
< A
i inone
| Nit I f Presy
. "*
ma ,' c ,nri ?
af ** ac1 Drop,
" >
ana .' e , «
-
al-J kaptonuri, combined
0
a
pilismone ,although no eJ
reasonabl e -
with
*Peri nce is**
*ome ,e
> with
seems

^^
H igashl ,
)inChediak
III* Class,homocy B6 U00- S00
>
*
*therapy
Betaine <trimethyigiyci 1
«
6 /24 hr 0r ,
are responsive to this

»2oo Je WM ,
.
' *has"ty* ^ *** ^>°on b " *, ' 0r
23(8) ) lowers homocY ev< in
children
*^
to methi0nmp r Sult in lev
homocysteinelevels. This treat
methmnme vascular events! «e„ ent
* °««
' "Pr ed c n cal
p asma

(preventing
therapy.
" *P ts wf>0 are iln

is confirmed by measurement of sulfite oxidase and


rarri n 86
'
Diagnosis
24.(A) t
cofactor in fibroblasts and liver biopsies, respectively or by
molybdenum DNA
studies. tment with nicotinic acid or nicotinamide (50- 300 mg/ 24 hr ) and a
25.( &) Trea
in diet results in a favorable response in symptomatic patients with
high Protein
Hartnup disorder .
with mild or intermediate forms of maple syrup urine
26 (A) Some children
. are treated with high doses of thiamine have dramatic
disease MSUD who
. Although some respond to treatment
clinical and biochemical improvementmay require as much as 100 mg hr for
with thiamine at 10 mg/ 24 hr
, others /24
response is observed.
at least 3 wk before a favorable
27.(B) In isovaleric acidemia, acute
episodes of metabolic decompensation may
dehydration, surgery, or high-
occur during a catabolic state, such as infection,
protein intake. Acute episodes may be mistaken for diabetic
ketoacidosis. Some
patients may experience acute and recurrent episodes of
pancreatitis.
patient is
28 ( A) The major clinical manifestation in the rare symptomatic
red after moderate
cutaneous photosensitivity. The skin becomes rough and
-like rash may
e Posure to the sun, and with greater exposure, a pellagra
* may develop. The skin
evel P- The rash may be pruritic, and a chronic eczema
°
changes have been reported in affected infants as young as 10
days of age.
enteropathies occurs in
(D) A clinical condition resembling acrodermatitis
become very low; addition of
3 ®cted
infants whose plasma isoleucine or valine recovery of skin
will hasten the
eucine or valine, respectively, to the diet
rash .

176
have perio dic bouts of fever associated *
30.(E) These patients , arthralgia, arthritis,
pain, vomiti ng, diarr hea
and mor billif orm rash (even petechiae and
lymphadenopathy PUrPUra) ' i,
l y r o 3g
usually start before
mQSt|y confjned to skin
^^
-
d the
f > myoc'lonic
^
31 C Ca cand idiasi
,c or seborrheic dermatitis , s, alope cia', ata
*» .^
SuX ), hypo tonia , devel opme ntal delay optic nerv
*
rs

dermatitis resolved with biotin therapy, have been reported; these chj
were otherwise asymptomatic.
32.( A) Clinical findings of propionic acidemia are not specific to this
only. In the severe form, patients develop symptoms in the 1st few days of
disorder
iife
Poor feeding, vomiting, hypotonia, lethargy, dehydration, a sepsis-like pictur
e
and clinical signs of severe ketoacidosis progress rapidly to coma and
Neuroimaging shows that these abnormalities, which often occur
death
after an
episode of metabolic decompensation, are the result of damage to
the basal
ganglia, especially to the globus pallidus. This phenomenon has
been referred to
as metabolic stroke.
33.(C( Diagnosis of nonketotic hyperglycinemia (
NKH) can be suspected based
on the findings of elevated glycine
in plasma or CSF and the abnormal
thX
^ wH^rel ate d '
6
o genes *
confirmed usi g molecular analysis
"
.
34 (B) Treatment with oral
. .. . activated charcoal, short cour
cours5es 0ofT oral

==
metronidazo le, neomycin, or lactulose camp
\
5.5) can also help control the odor
. ° ,
r’ Top cal use
of acidic soaps (pH
r
35.(C) Administration of large
urinary excretion of oxalate.
doses of nvririnx ,
ne reduces plasma
level and
36.(A) Treatment of prolidase
can be fatal and warrant close
deficiency is su
and proactive
,^
n ect us complications
37.(C) Three forms of this rare
condi
enzyme deficiency causes glutathione
tion have b ° t C mana
8ement .

patients present with hemolytic deficie, nt ?r 0rted
- »n the mild form,
demonstrate high residual activity of «hou
anemia wi 1°"^ ^ in hrocytes. These
glutath, 6
^^
0 me
testing. A moderate form has also
been ob
' ‘ abolic acidosis and
synthetase
SerVed on enzymatic
m
177 which the hemolytic
associated with variable degrees

.
t
of
Its severe form distinguished
* bv metabolic ackJos
<of>r
0
d bv severe aados s, massive 5
presence of hemol * *nd y

>;epiV
«V )
- oxoprolinuria, ,andc anemia
"eurologlc
t' nroximate order of frequency, symptoms

^patch . .'"Jjj
3 (6
)
,
ln
ren ors
apa jhy episodes of cyanosis, seizures,
or high-pitched cry, limpness
lachVP06300’ and eye rolling. Episodes of
Idelude- ii
intermittent
or lethargy,
J tterin ss or
aPneit
^ '
spells
difficulty feeding
sweating, sudden pallor
oc

' *Norn newborn


and cardl3
d’
C rest and failure may also occur, , hypcrth
ermiar

as h
, values are higher
13 than those of the
lQ0 imol/l. can occur in healthy
* a btood
older child or adult .
ieyels
u5Ually ^301* ammonia level >150 pmol/L.
term infants, An ill

e hypoglycemia should be considered a


infant

1(0 M
Convulsions cause of an initial
*
episode
or level of
or a sudden deterioration iin
consciousness.
psych behavioral
functioning
42 (C)
43.(E)
, ,
44.(0) in patients with CPS1 OTC or ASS
deficiency ar

attacks may be precipitated by valproate administration


nypera
K .
,„
T nriortem a ,
'
,
45.(C) The most consistent laboratory
,findings are u
, Sis ot
detectable levels (>90% and insulin resistance (60% . P
. ^
moderately high Otherwise, lipid panels, high-sensitivity C- reactive prote n
Wood chemistnes, liver and kidney function tests, endocrine test and
coagulation tests are generally normal .
46.|B)

Ilf:" " '",


48 fBj
-yPerammonemia-
° ^ ^ ne n ox (vitamin
,
86 6 d SOrders feature hyperornithinemia

B6)-dependent
: gyrate atrophy of the
hyperornithinemia homocitrullinemia syndrome

treatrnent with vitamin B6 ( 50-100 mg/ day ) usually results


the E £G abnormalities
,

epilepsylhypophosphatasia
in a dramatic
. High doses of
)

^ ^ Vement seizures and


°
pyripoxine can
result in peripheral neuropathy and doses >500 mg/
day should
long.
the therapy are life-
Pyridoxine dependency and thus encephalopathic
.. by acute
condition is usually heralded
4g I
^ ° * loss of normal developmental milestones (head
6 nSe control, rolling
findingS SUch
' choreoathetos ,
, opisthotonos are prone to
over, or siS tting)
|
seizures, generalized rigidity
;

dystonia caused by acute striatal injury . Affected infants

178
and retinal hemorrha8 foil
development of subdufd ^ „,t e misdiagnosed as child abuse. °
Wi*

rS"
transport
intestine and
th
kid ¥
Defjcjencv 0f the
transporter protein ( yH
causes multisystem manifestations,
arhiryj
^^V
"
•s
transporter aj / g|) symptoms.

STI B T Def i c i e nTo f aspartoacylase leads to Caravan disease, a


- ,,
Z“ dystrophy charac terized by excess
degeneration of the white matter
ive excretion of N Acetylasp
of the brain. *
S2 C Th
( )
^ first
adulthood. Attacks
episode
are
aching muscle pain and
usually does not occur until late childhood 0r eark
frequen tly precipi
myoglobinuria that
tated by prolonged exercise. There
may be severe enough to cause
kinase are elevated to 5,000-100,005
is

renal failure. Serum levels of creatine


has not been describ ed, but fasting may contribute to
units/L. Hypogly cemia
attacks of myoglobinuria .
53.(E ) It must be emphasized that although
plasma VLCFA levels are elevated in
always the case. The most
many patients with peroxisomal disorders, this is not
, in which VLCFA
important exception is rhizomelic chondrodysplasia punctata
levels are normal, but plasma phytanic acid levels are increased and red blood
cell (RBC ) plasmalogen levels are reduced.
54.(D ) Other findings are vertebral bodies have a coronal cleft filled by cartilage
that is a result of an embryonic arrest. Skin changes such as those observed in
ichthyosiform erythroderma are present in approximately 25 % of patients. The
most decisive laboratory test is the demonstration of abnormally low
plasmalogen levels in RBCs and an alteration in PEX7.
55.( C)
56.(0} The diagnosis may be confirmed genetically or by measuring LDL-B
activity in cultured skin fibroblasts.
57.(0 The disease usually presents during childhood with acute pancreatitis.
58.(C) Other options causes secondary hypercholesterolemia .
59.(D) Causes of reduced high- density lipoprotein:
S Smoking
S Obesity
S Type 2 diabetes
S Malnutrition
Drugs: p-blockers, anabolic steroids
.
60 {E) In such patients, the aim is to prevent episodes of pancreatitis.
Tlnjs autosomal co dominant disease ,s a
(l (W jj caused by mutations in ABCA1 a a *'** *'»h HDi r , <s
,
« ce iculoeodotbelial system,
manifested L actum ‘
*
n<
color and C 2 *
Jj
n in

diS maV
"
fr m Ch
'
** ° I recessive
nna«hy autosoma
ester001
'
sccumulad !
In m
, *" «nt |

< This ra«


fSing microsoma tnglyceride transfer
ann
disease is caUSM b " ns*«in*•
* '
sn
Hs to nascent

"*; ' cholesterol


chylomicrons in the small
protein
intestine nrt ^' *' "*., gerw
,? °r he « »
« of
.
^
P sence of chylomicrons, VLDL IDL,
> ts ^in
and triglycerides .
and poB
and
Itar. This
very i0w )eve|
|p| Datadetecting
^
suggest that obtaining a non -fasting
iipld profile r
severe genetic dyslipidemias
as a fasting hpid be just a*
tsn
j ip
be used as first-line screening in children . Fasting lipid
1 Profile ,
and
thus can Profiles may a|so
used depending on parental, child, and clinician
t#? Preference, especially jf
there is concern for hypertriglyceridemia, si
since triglycerides
are affected more
by fasting
status .
61(A)
manifestations of the infantile form of
65.(E) Tbe clinical GMl gangliosidosis
may be evident in the newborn as hepatosplenomegaly, edema, and skin
eruptions (angiokeratoma ).
61(B) The GM2 gangliosidoses include Tay * Sachs disease and Sandhoff disease;
each results from deficiency of p -hexosaminidase activity and lysosomal
accumulation of GM 2 gangiiosides, particularly in the CNS,
67.(C)
63.(B) This classic phenotype is caused by the absent activity of the a -
galactosidase A and has an estimated prevalence of approximately 1 in 40,000
males. The angiokeratomas usually occur in childhood and may lead to early
diagnosis. They increase in size and number with age and range from barely
visible to several millimeters in diameter. The lesions are punctate, dark red to
blue-black, and flat or slightly raised. They do not blanch with pressure, and the
^ger ones may show a slight hyperkeratosis.
:

.
fi5 (D) Early in
the course of the classic phenotype, casts, RBCs, and
lipid
ioclusions with characteristic birefringent "Maltese crosses appear in the
binary sediment,
by the deficient
70- (C )
Fucosidosis iis a rare autosomaI recessive disorder caused -contaming
actiVI< v of a- f fucose
fucosidase and the accumulation 0
, «**.*or • “ » »** and
*« .. Ysos
**
sgsr sncsvr ^iN
® ' nSf
^ i
tkpr
\

— ——
sulf
glycosphingolipids-
gussss-
tissues, especiallythejo
- -
rpce

nts.
' ^^
-
,sive disorder results from the defir,,
•»
,ysosoma
—*
| storage diseases result
deficiency of hW I « 1» °f chote'«
characteristics of type I GSD are hypoglycemia,
74 (D) The biochemical
acidosis , hyperuricemia, and hyperlipidemia . Despite marked hepatomegaly
usually normal or only slightly elevated.
^
liver transaminase levels are Eas,
bruising and epistaxis are common
and are associated
with a
prolonged
bleeding time as a result of impaired platelet
aggregation and adhesion.
Electrocardiographic findings include a high-voltage QRS complex, Wolff .
75.(C)
Parkinson- White ( WPW) syndrome, and
a shortened PR interval
Echocardiography reveals thickening of ventricles and/or the intraventricular
septum and/or left ventricular outflow tract obstruction
. Muscle biopsy show
glycogen ; acid phosphatase
*
the presence of vacuoles that stain positively for is
increased, presumably from a compensatory increase of lysosomal enzymes.
76.( B) The standard diagnosis for GSD V includes a muscle biopsy to measure
glycogen content as well as enzyme and sequencing of PYGM . An ischemK
exercise test offers a rapid diagnostic screening for patients with a metabolic
myopathy lack of an increase in blood lactate levels and exaggerated blood
,

ammonia elevations indicate muscle glycogenosis and suggest a defect in the


conversion of muscle glycogen or glucose to lactate
77 (D)
78.(E) The renal calculi, composed of 2,8-dihydroxyadenine, are radiolucent,
soft, and easily crushed. These stones are not distinguishable from uric acid
stones by routine tests but require high-performance liquid chromatography
(HPLC), ultraviolet ( UV) light, infrared light, mass
spectrometry, x -ray
crystallography, or capillary electrophoresis for diagnosis
, particularly to
distinguish from stones in HPRT deficiency
79.{B)
SO,ID) Both types of MorQuio
, a
dwarfism skeletal rOrr ,
/ nucopolysaccharidoses, and nr
S1.IB) The affected infant -
es0 rva ,i
ton
junction of galactose . ther ise of
oth

gl (C) Sanfilippo
syndrome *
AS C/>
degeneration with mild som 3
- racfer fed
.'
^Cds
at age 2-6 yr in a child ' ease b
who '
»(*> Coarse facial features "T'°Usly aPPeOnset
and intellectual n
disability man < Mature ared
/ 4.IA ) Leigh disease (subjf 5 faefM'ee0‘ >'Sost0S s
' / )
heterogeneous
neurolog,c te
ecr
necrosis, relative neuron ,
r0 na- ! ° ISease rh "
regions,
SS. / A ) Sialidosis is
an aut
faring a ^

te
Capilli
°
Son)al feces , -
deficiency of neural
« ««
Sl.( A*
)
, r ‘ e of
««
nautat
87 (A)
>
Mitochondriai c
tsmins (Bl , B2
"modifiers CL anti
cktails of
° t

U- ° xidants frC
.
(creatine and SuPple
Ql 0
mi ° ' ^Po/ acid rpents Viably
carnitine' L Arginine
/ lc
,
c acid )Warni
;
, folin
arid "Wiide
mg
' .
labour

182
Chapter 11
The Fetus and the Neonatal |nfaht
Questions
AHMED TAWFIQ
mother of 5 - day - old precious baby boy, pr
1. A concerned
vaginal delivery describes a very brief
uneventful normal
of lower limbs occurs d,
rhythmic movement of one
Examination was normal .
What is the MOST appropriate plan
of management ?
workup for seizure
A. Order initial investigation
B. Admit the baby to neonatal care unit for video monitof ,n
*

C. Send for EEG record ^


D. Start empirical anticonvulsant therapy
E. Reassurance

2. What is the major cause of mortality during infancy ?


A. Unsafe sleep practices
B. Congenital malformations
C. Complications of prematurity
D. Accidents
E. Non-accidental injuries and mal-treatment

3. Which of the following organs


is affected more often by
anomalies? congenita,
A. Gl tract
B. CNS
C. Heart
D. Kideny
E. Eye

4. Presence of
bilateral red eye reflex iin a
A. cataract newborn suggest
B.
chorioretinitis
C. retinopathy
of prematurity

183
hyperplastic primary
D- p
efsiste nt Vit reous
of intraocular pathology
l Absence
,hat is the MOST prominent sign of neuromuscular j .
5- Arthrog ryposis < sease at
7 Microcephaly ' birth?
r
c Hydrocephaly
Q Dislocated hipstestes
I
yndescended

is the recommend time for delayed


6- what cord damping ,
? n a v gorou
term infant ' $ full -
15 seconds
^B 30 -60 seconds
Q 1- 2 minutes
Q 2 -4 minutes
l Until umbilical pulsations stops

7. You are preparing your tool - set for neonatal resuscitation program ( NRP );
which tool would be used for oral suction?
A. Mucous extractor
B. Soft cloth
C. Bulb syringe
D. Large bore catheter
E. No tool is required as suction is not needed

8. Which of the following drugs is contraindicated in a nursing mother ?


A. Thiouracil
B - Aspirin
C. Amiodarone
D - Domperidone
E- Phenobarbitone
indication for
an absolute contram
^ bich of the following medical conditions is
breast feeding? days)
A - Active TB ( mother on treatment since 21

^ Genital herpes infection


Cl HumanT-cell lymphotropic virus type
184
r
infection
D. Hepatitis C virus
infection
E . Varicella zoster
, .,„
,ch the a leading cause of
is53 prematurity?
° 2Station '
following
1 wh
°A Multiple gei *
g
B Cervical shorten
C. Genital infection
D. Presence of feta ^
. .
jn cen/ic0vag nal secretions

E premature rupture

11. Polyhyramnios is
A. 4 cm
diagnosed
membranes
, .
amn o fluid index is more than
a iff amniotic

B. 14 cm
C. 24 cm
D. 34 cm
E. 44 cm

12. Which of the following conditions is associated with polyhyramnios?


A. Twin-twin transfusion ( donor)
B. Cystic adenomatoid lung malformation
C. Fetal akinesia syndrome
D. Prune -belly syndrome
E. Angiotensin-converting enzyme inhibitors

13. Fetal bradycardia is defined as fetal heart


beats less than
A. 50 bpm
B. 70 bpm
C. 90 bpm
D. 110 bpm
E. 130 bpm

14. Which of the


following is the MOST
A. Cytomegalovirus common form of congenital infections
-
B Rubella
.
C Herpes
virus
D - Syphilis
E. Toxoplasma

185
av0ong ladV exposed to diagnostic radiation of
1
1& - th wk of gestation; she
develops a about o i RAD ,
.

) 3 t 0 St X -
raV t0local health authority .
she
uropinion?
WhaA Approve the case
pjsmiss the case
0
c postpone the case meanwhile taking opinion
of radinir,
D Assessment of mother radiation exposure bv bind
send mother for hematologic screen if negative dismiss
JBV expert

' metry
E the case

l6 you
are attending a labor of 23 ~ year - old primigravida;
the cardiotocography
slowing of fetal heart rate, beat-to-
shows beat variability declining, and
appearance of late deceleration pattern .
pf the following, the MOST appropriate action is to
A. start infusion of tocolytic medications
B. start oxytocin infusion
C. counsel parents about possibility of hypoxic- ischemic encephalopathy
0. administer high 02 concentration to the mother
E. prepare for emergency cesarean section

17 . Which of these parameters is a poor predicator in hypoxic ischemic -

encephalopathy injury ?
A. Low ( 4- 6 ) 10 min Apgar score
B. Need for CPR in the delivery room
C. Delayed onset of spontaneous breathing > 5 min
D. Seizures onset > 48 hr
E. Absence of EEG findings of burst suppression pattern

18. According to Sarnat classification of hypoxic -ischemic encephalopathy, a


term-infant who appears to be hyperactive with mydriatic pupil , exaggerated
Moro reflex and myoclonus has
A - ongoing subtle seizure activity
B. HIE stage I
HIE stage II
D - HIE stage 111
E . unclassified

186
a 3-day-old term neonate
19. You are assessing with
encephalopathy.
Of the following, the MOST
..
sens t ve test that helps ln
management is
A. brain MRI
B. brain CT scan
C. brain ultrasound
D. brain and bowel ultrasound
E. brain Doppler study

20. What is the targeted core rectal temperature in therapeutic hyPoth


hypoxic-ischemic encephalopathy ?
err%
.
A . 32 °C
B. 32.5 °C
C. 33 °C
D. 33.5 °C
E. 34 °C

21. What is the ideal timing of initiation of therapeutic hypothermia in hy


ischemic encephalopathy ?
A. As early as first 30 minutes of life
B. First 6 hours of life
C. First 12 hours of life
D . First 24 hours of life
E. First 48 hours of life

22. What is the duration of therapeutic hypothermia in hypoxic - ischemic


encephalopathy ?
A. 12 hours
B. 24 hours
C. 48 hours
D. 72 hours
E. 96 hours

23. A useful adjunct to


therapeutic hypothermia in hypoxic -ischemia
encephalopathy which shows promising
short term motor outcome is results in reducing brain injury an ^
A. erythropoietin

187
phenytoin
nigh dose
(3
0barbital
coma
r Phen infusion
lldocaine infusion
f amiodarone
baby boy suffered from severe
birth a 5 phvwa
hypoxic -
l
*- *
ische
. encephalopathy
17110
.
with recurrent seizures
not responding to
3 tbrtone
Phenob appropriate next choice is to use
1 MOST
loading dose phenytoin
A
B levetiraoetam
.

pyridoxins
^
p nnidazolarn
£ lidocaine
is the MOST important radiological finding in MRI correlating
2 $. What with
poor prognosis in hypoxic - ischemic encephalopathy ?

A Lossof cerebral cortical gray white differentiation


-

B. Parasagittal cerebral cortex lesion


C. Basal ganglia /thalamic lesions
D. Occipital lobe lesions
E. Cerebellar Lesions

26. The brain involvement of periventricular area in process of hypoxic - ischemic


encephalopathy can lead to future development of
A . cognitive delay
B . visual and auditory processing difficulty
C. dystonia
D - seizure disorder
E spastic diplegia

4 ? The brain involvement of parasagittal area in process of hypoxic - ischemic


-
^ephalopathy can lead to future development of
A. spastic
quadriparesis
dystonia
seizure disorder
ataxia
E.
Pseudobulbar palsy
188
f
priority in treatment
28. What
?
is the initial *
0 hyp
encephalopathytherapeutic hypothermia %

A. Starting base status


of acid
B. Correction
ic stabilization
C. Hemodynamic
situ

D . Control of
seizures
E . Starting
parentral nutrition
common acquired neuronal injury in newborns?
29. What is the MOST
A. Phrenic nerve
B. Brachial plexus
C. Facial nerve
D. Spinal cord transection
E. Horner syndrome

who is depressed at birth you


30. You are resuscitating a full term neonate
need to stat bag and mask resuscitation. The fraction of inspired 02 that IS
needed should be
A. 21%
B. 40%
C. 60%
D. 80%
E. 100%

31. An ex -28- week-preterm baby, at 3 weeks of life is recovering from RD$,


extubated since 2 wks, he is developing frequent apneic attacks with
bradycardia and desaturation. The baby general
condition is good with good
capillary refill; he is not pale and appears
protocol TPN had stopped
.
alert He iis on increasing feeding
since 4 days, increasing weight, his acid base balance
-
is normal with euglycemia.
Of the following, the
MOST appropriate action is to
A. do septic screen
and start empirical
B. reintubate the antibiotics
baby and put on low
C. start nasal ventilatory settings ,
continuous positive airway pressure
D. start heated (nasal CPAP, 3- 5 ^
E. start caffeine
humidified high-flow nasal
-
cannula (HFNC,1 4 l/min)
# ell
,
preterm baby recovered
from ,
}
lJ
f 3i ° '
, . omepra0 es PW
following
>
,
ref ux <GER - Vou are
the MOST
>
appropriate
anticipat '
Ption f 0r manag
^evelops
0
tA zole ° ementi15 ePisodes.
to start
g ranitidine
of
c combination omeprazole and ranitidine
p domperidone
E.
monitoring

baby boy delivered to


3 A near
-term diabetic mother with
was
from motherdevelops
being poor attenda nt to antenatal fac or,
2 g kg. He
tachypne a and desaturation
few hn
‘ «
, , u V birth wei h<
his
respiratory distress , heart rate 170 bpm
pressure 55/35 mmHg. He has hepatomegaly and no cardiac
.
respirator ! " birth with

ray shows complete white - out. Blood gas analysis


metabolic acidosis with hypoxia. The baby admitted to NICU
therapy with minimal response, he is euglycemic
shows

now .
m ^
lT 1

received
"
blood
~

Ches
~

and
^
INSURE
'
**
Of the following , the MOST appropriate action
nd is to
A. give 2 dose of survanta
B. intubate and start mechanical ventilation
C. collect septic screen and start empirical antibiotics
D. order an Echocardiography
E. consult metabolic / endocrine team

34. What is the BEST type of steroid that should be used antenatal
ly to decrease
incidence of RDS of prematurity ?
A. Dexamethasone
B. Betamethasone
C. Methylprednisolone
D. Hydrocortisone
E. Clobetasone

35. A 2- day -old full -


term baby boy with difficult labor has unilateral scalp
sweHing which is firm, tense with palpable rims that does not extend to
,
contralateral side, and it has central depression. The baby is hemodynamically
s able
and neurologically normal; he has jaundice over the face and upp
trunk .
0f the
following, the MOST appropriate action is to
190
1
A. closely follow the results
of PCV and TSB
give appointment follow Up
B. reassure the family and

C. reassure the family about the benign nature which may 1 We'^ ^
dlSaPpear
2weeks-3months %
D. order radiological assessm ent
E. refer to pediatric neurologist

, A full-
6
. .'
term babv & rfeiivered bY vatruunvass
, istcd delivery,
attention
attendant that n t an increasing scalp
*,*n?
overtime V.° ,
the scalp and s subcutaneous
sutures
Q
T appropna ,
n s t0
accordingly
B. keep the baby beside the mother and informed staff nurse to
monitor
vitals and the lesion size
C. admit the baby to NICU
D . reassure the parents about benign nature of lesion
E. anticipate the expected hyperbilirubinemia and anemia by starting siingle
phototherapy and serial PCV

37. You are counseling a mother of large for date baby with difficult
labor who
was admitted to NICU for complication of meconium aspiration syndrome
; now
he is improved and the team is willing to discharge the baby
; mother is raising a
point about the result of head MRI which shows a
small subdural hemorrhage in
posterior fossa; she asks about the prognosis.
Which one of the following represents the MOST
appropriate answer ?
A. We will arrange an appointment in pediatric
neurology clinic for follow -
up
B. We will assess neurological status of
the baby in next visit and we will
order radiological assessment according
ly
C. Serial of MRI will decide the need
for evacuation of the lesion
D. Such lesions will calcify leaving
asymptomatic cyst which needs no
intervention
E. These lesions causes no residual neurological
problem

38. All preterm babies should be subjected to cranial


ultrasound for detection of
intraventricular hemorrhage during the first 3 -7 days of
life if delivered
A. 24 weeks of gestation below

191
weeks of gestati0n
8. &
weeks of gestation
C. 28
of gestation
0 . 30 weeks
32 weeks of gestation
E .

28- week gestational


- age preterm baby
„ A Wlth a . .
3
Pived prophylactic surfactant at birth,
.
ventilated
2 his Pcv
on SIMV mode with moderate
sh0WS df 0 p
eventual!
from 40 t0 28. and
^
settings
he
,
TPIT
Wei8h of

*
del ' ,S
' ° be
toon

, aned day i. 0 n
bolic acidosis. He received fluid resuscitation and pack bypo ens n and
!!! ,
meta
d blood gas shows persistence of metabolic aril '° ,on
ri»Kfus
RePeatefollowing, the MOST likely explanation
Of the is
of early
A development
onset sepsis
inborn error of metabolism
B underlying
Q development
of anemia of prematurity
Q development
of intracranial hemorrhage
complication
E. TPN related

baby with 1200 grams birth weight, developed


40. A 32- week preterm
-

respiratory distress and received rescue INSURE therapy since 4 hours. Now he
is on nCPAP with pressure
of 6 cm of water and FiOr of 60, his saturation
ranging between 85 - 88%; other hemodynamic parameters are accepted.
Of thefollowing, the MOST appropriate next step in the management is to
to avoid 02 toxicity
A . keep the nCPAP parameters the same
B. increase nCPAP parameters
C. consider the second dose of surfactant
D. start mechanical ventilation
E. use inhaled nitric oxide
) can increase the risk
41. Assisted reproductive technology ( in vitro fertilization
of
A . Beckwith- Wiedemann syndrome
B. neural tube defect
C. Cornela de- lange syndrome
D - major chromosomal disorders
E - Peirr- robin sequence

192
way to increase the 02 saturation iIn dentil
42. The MOST effective atecj ,
neonate is by increasing
the
X
A. inspiratory time
8. ventilator rate
positive end-expiratory pressure (PEEP)
C.
D. peak inspiratory
pressure (PIPJ
E . fi02

oxygen saturation targets in1 ,


43 The currently recommended Venti|ated
neonates rs
A. 81-85%
B . 86- 90%
C. 91-95%
D. constantly above 95%
E. 100%

44. You are preparing to extubate a preterm baby.


Which premedication can enhance the success of extubation?
A. Methylxanthine
B. Dexamethasone
C. Betamethasone
D. IV atropine
E. Endotracheal atropine

45. Which of the following medications proved to have a role in decreasing the
risk of development of bronchopulmonary dysplasia in preterm babies?
A. Inhaled corticosteroids therapy
B . Diuretics
C. Vitamin A
D. Low dose systemic dexamethasone
E. Caffeine

46. In delivery room, you


received a 28 weeks gestational age preterm baby
with a birth

^
6
A 'U Iant
ve p r o p h y l a c ^ **
t herapy
m a n a g e m e n t i s t0

193
,
f 3l
for the signs
of severe respiratorv o
^
tstress then og\Ve ,
6 * 3Ctant
surf rescoe dose of
.
sal CPAP
Q start nasdix
on low f/0 asal 02
0 continue
maintaintoedneonatal care unit h ^ ^ as 02
saturation is
E- transfer °
C 08f0r and
decision should
be ma

1-

^ Icrease urine output


« »»
jndn pulse and hepatomegaly.
won
Oftl.*
(jW

fo||owing, the
--
* < Iml/kg/hr. By examination Z hiZ
’ ve
MOST appropriate action is to
order an echocardiography
o"
notlced

» order renal function and monitor urine output


C, a dminister diuretics
0. administer ibuprofen
E . refer to pediatric cardiology department

43. You are explaining the effectiveness of supportive measures in spontaneous


closure of PDA in extremely preterm infants to a junior colleague.
Which of the followings represent the MOST appropriate answer ?
A. Most of the infants eventually will needs cyclooxygenase inhibitors
B. About half of them will be cured
C. Most of them will be cured
D. Surgical intervention is high in those who treated by this way
E. It is no longer considered as an option for treatment

49 What is the MOST common etiology of tachypnea in newborns ?


A - Transient tachypnea of the newborn
B. Hyaline membrane disease
C - Meconium aspiration syndrome
Amniotic fluid aspiration
I Early onset sepsis

What the MOST important effective step in prevention of meconium


Nation syndrome ?
4 A A
S'pa nd SsSoSS ofdSessed
B

fr
^ 3 meconium stained
babv

51. A full
^dn
-term hahv
stained arnniotic
boy
OH
cesareansections
delivered by cesarean
, ' developed
flu d
day 4 discovered
because of m
section
respiratory distress and required
to have systolic mum, , >
*
ventilation ciftimatelvbetwee 82% and 96%. He had frequent weaning fai| ”
saturation is ranging
V

.
from ventilation
Of the following*
A. associated
the MOST
cyanotic
likely
.
etiology of cardiac murmur is
congen jtal heart disease
arteriosus
6. patent ductus
C. pulmonary hypertensio
n
cardiomyopathy
0. asphyxiated
E. functional mitral
regurgitation

by uneventful simple vaginal delivery, tad


52 A full-term baby boy delivered
home. At the age of 20 hours developed
good APGAR score and discharge
refusal to feed, rapid breathing, and bluish discoloratio
n of the face. He hadm
dysmorphic features, HR 185 beat/min, RR 70 breath/min, 02
saturation 6 «
with
which sometimes increases to 90%, blood gas analysis shows hypoxemia
COj retention. Mother was G4 P3, attending antenatal care regularly but had
used ibuprofen frequently for headache without medical consultation
Investigations shows RBS 55mg/dl; Hb 20 gm/dl; WBC 24,000 mm mainly
neutrophils; echocardiography shows no congenital heart disease; blood culture
and metabolic screen is pending.
Of the following, the MOST likely diagnosis is
A. bacterial sepsis
B. concealed congenital heart
disease (distal aortic arch anomaly)
inborn error of metabolism
D- persistent pulmonary
hypertension
b Pulmonary hypoplasia

53 A post -term
baby girl
hypertensi°n 1

suffering from hypotensionventilated for persistent pulmonary


metabolic needs, volume which is persistent even after adjustment of
P acement and vasopressor
agents.
,,<h,following, the MOST appropriate step in ma„

0
J
01 use another bolus of volume repiacement * *gement is to
use of hydrocortisone 1
therapy
_
use of desmopressin
send for serum cortisone
'

c* order urgent brain ultrasound

arereceiving a newborn in delivery


S4- VoU room with prenatal
matjC hernia. He delivered with good diagnosis

^
APGAR
score, vitals are of
normal 02 saturation. normal
^ 1he following, the MOST appropriate action is to
01
a(jminister 02 through face mask
0 pass a wide bore nasogastric tube to deflate the
r intubate and gently ventilate the baby with stomach
hypercapnia standards of permissive
D. contact with pediatric surgeon
E. transfer to neonatal care unit

55. What is the best time for operation in ventilated infant with
congenital
diaphragmatic hernia ?
A. As early as possible
B. First 48 hours
C. After 48 hours
D. After neonatal period
E. After total weaning from ventilation

*
„ Mdta .w - m
admitted for recurrent lobar infiltration o rig
elevated right dome of diaphragm and paradoxical mo

"“ .
otoscopy
Of the following,
.
movement during

the MOST likely cause current lobar infiltration is


A eventration of right
dome of diaphragm
B. lobar sequestration
0' lung hypoplasia
0. phrenic
nerve palsy
E. spinal
muscular atrophy

196
s7' A 48-ho - d
,, , term babV b°:V , distension
, weigh l-8kg d dn
his
* '
,
passt
57 ^an°' ncreeasmg abd°
U
iriWl
dd and

^
occasional vp js
inos
' chows
*' alc
.
arge bowl and PR
|
cpwere
,
exam ^, ng !

preedapmsia and
un
magnesium sulphate
following, the MOST likely diagnosis is
Of the
A. meconium plug
B. meconium ileus
C. necrotizing enterocolitis
D. hirschsprung disease
E. cystic fibrosis

58. What is the characteristic X - ray finding in abdominal contrast study d0 >
IQT
a neonate with meconium ileus ? *
A. Coil spring sign
8. Dilated distal bowl
C. Microcolon
D. Whirl sign
E. Double bubble sign

59. The usual onset of necrotizing enterocolitis in VLBW preterm babies is at


A. first few hours of life
.
B first 48 hours of life
C. first week of life
D. 2 nd and 3rd weeks of life
,
E . 4 h week of life

60. What is the MOST effective preventive


strategy for necrotizing
enterocolitis?
A. Human milk
B. Probiotics
C. Synbiotics
D. Pretreatment with H
2 receptors antagonist
E. Good antibiotic
coverage for bacteria that colonize intensive care
^
61. A well active full
-term baby boy his
year -primigravida blood group is B+ve; delivered to
mother, her blood group
3 na had . is O + ve with good antenatal
&
perinatal problems
. His complete blood count shows, HB 13
1Q 7
36. The blood film,
92 fL, MCHC Coomb's test, and
tfC* biliruh;, r> are
n6
Pe ftd *the avaliable data, what is the MOST probable
'

v dilagnosis?
ABO incompatibility
A. trait
g. a-thalassemia
iron deficient
c. Maternal
ditary spherocytosis
D - Here
I values for age
E . Norma

pregnancy,
fesarean section had been done . Her baby was
resuscitation including volume replacement.
pale and |imp emergency
and required full

What laboratory test should be done for precise diagnosis?


A Urgent baby complete blood
b
B. Kleihauer- Betke test using maternal °' °d smear
C. Reticulocyte count, MCV and MCHC of baby sample
D. Baby and mother Coomb ' s test
E. Flow cytometry using maternal blood

63. A 3 - day-old full-term baby girl presents with history


of increasing neonatal
jaundice . Examination shows active, pale
baby, with splenomegaly. Her
investigations show HB XI gm/dl, MCV 102,
MCHC 37, direct antiglobuline test -
ve, indirect bilirubin 18 mg/dl, direct bilirubin
0.4mg/ dl, blood group B ve;
hematologist reported increased reticulocyte count
and bizarre RBCs
morphology featuring acanthocytes. The baby developed
a stormy course,
treated aggressively ,
with freQuent blood transfusions
consanguineous and are free from
p
01the following
, the MOST likely
A. hereditary
."
spherocytosis with complete P
"°"
penetrance
e'

B. pyruvate
C. G6PD
kinase deficiency
D. ABO
deficiency
blood
E. congenital groups Incompatibility
abetalipoproteinmia
64.
Whatis the suggested PCV transfusion threshold for a 5-day-old VLBW baby
no respiratory
support?
198
A. 40
B. 35
C. 30
D. 25
E. 20

factors reduce mother sensitizati0n


65 Which of the following in
disease of fetus and newborn HDFN resulting from Rh
hN
incompatiblyHty ?
'
.

A . ABO incompatibility
B. Increasing order of pregnancies
C. Type E Rh antigen
D. IgG type maternal antibody response
£ . Larger volume of fetomaternal blood transfusion

66. Which of the following is a recognized complication of hemolytic disea


fetus and newborn HDFN ? ^ of
A. Hypocalcemia
B. Hypoglycemia
C. Hyperkalemia
D. Sepsis
E. Hyperthermia

67. A G5P4 mother who had previous 2 affected babies with hemolytic disease
of fetus and newborn ( due to Rh incompatility ) is asking about how know
to
whether her coming baby is going to be sffected or not.
Which of the following tests you would use inresponse
to her question ?
A. Amniocentesis
B. Umbilical blood sampling


C. Fetal DNA from maternal
serum
D. Chorionic villous sampling
E. Rising RhD antibody
titers

“oA.”Z°oteis,m'a ” ""
,
',T T“* '
!
» r p 3 BhD imMl
-

C.
Order umbilical b l o o d
Order fetal blood
transfusion
''
r
of Rh-,mmuno„
, ,
n
o Adminiitration , '
Doppler study of the m ddle erebra< artery
l order
« ,r,
o9
'
pthophvs "
What «5 -ologv
,
" ,^
the Rger point of

Severe anaemia
4
B
Extramedullary hem*t ^ Cj
that cauS<?S
«
Poalh
h
“ "*"* < .
hePat eads ,
0ce) U ar
.

dysfunctron
that leads to rlehf,S dedheart ha Ur .
C. Anaemia ?* '
secondary „ etal renai *
0.

C.
Renal failure
hemolysis
GeneraHzed
^ °*
tissue hyP0J( ja
** ^^ osis
*evere

team leader expecting delivery of a baby with hydrops due to


are a
70. You baby is delivered and initial steps of resuscitation were done
. The
HDFN
successfully* He has pallor
, upper and lower limbs edpm

tachypnea and tachycardia. You started 02 and other suooortJ ""'’


'rh

Which of the following is the MOST appropriate step in the m


ma eement?
^
A. Whole fresh compatible blood transfusion
B. Small packed RBCs transfusion
C. Small dose of diuretics
D. 20% albumin transfusion
£. Assisted ventilation
- old full term neonate with hemolytic disease of
71 You are treating a 6 - hour
incompatibility; he received small packed
fetus and newborn HDFN due to Rh
. His serum bilirubin 8mg/ dl
RBCs transfusion, and he is stable right now
(136mmol/l) and hemoglobin 12 g/ dl.
management is to
Of the following, the MOST appropriate next step in the
A . proceed for immediate exchange transfusion
exchange transfusion standard
B. plot the serial results of total bilirubin on
graph and manage accordingly
C. start intensive phototherapy
D. administer IV immunoglobulin
E. administer 20% albumin
exchange transfusion is an
72. In which of the following scenarios an immediate
)
cation ? ( lmg of bilirubin= 17mmol/l of bilirubin
'^
200
„ ^ith 1jaundice, fever, lethargy and hVperw
. , davs full-term baby n
negative mother, bilirubin 13 m k ^
8/d|.
HE
12em/dl „ baby with bilirubin of 15 mg/dl
c A 48 hours 35 wks
high rkkkb
" ,
biljrubjn of 15 mg/d

^^
aspbvx
0. A 48 hours baby with bilirubin 20 mg/d|
r
A 72 hours standar
. MOST common cause for ne
following is the
73. Which of the
? °natai
polycythemia in term infant for the recipient
A. Twin-twin transfusion
B. Maternal- fetal transfusio
ns
C. Post-maturity
age
D. Small- for-gestational
E. Delayed clamping of the umb
ilical cord

74. A 3-day-old low birth weight neon


ate brought to emergency room with
feeding disturbance and lethargy . Examinatio
n shows alert baby with mild
tachypnea, cyanosis, and jaundice affecting the
face and trunk. His RBS is low
history suggest sepsis.
and capillary hematocrit is 68%. No risk factors in the
Of the following, the MOST appropriate step in management is to
A. measure central hematocrit
B. start partial exchange transfusion
C. start empirical antibiotics
D. start hydration with normal saline
E. order echocardiography

75. At which age vitamin K-dependent coagulation factors reach adult ranges?
A. End of first week of life
B. End of neonatal period
C. At 4 months of life
D. At 6 months of life
E. At end of first year of life

76. A well and active 3-day-old


full term breast fed baby delivered at hom
ts
presen to OP clinic
with fresh blood in stool, and
Of the following, the MOST some nasal bleed.
likely diagnosis is
A. early form of
hemorrhagic disease of the newborn
B. classic form of
hemorrhagicaei disease of the
newborn
201
deficiency
c ' factor VIIIdeficiency
p factor IX
E neonatal

77- A
lowering
-edicetioPS -
thrombocytopenia

pregnant mother with familial Wo


medication

' t is Your BEST


consults

advice for
f
you about

her ?
• anY
..
es e o
'
,,
Possible ^
Qn
cholesterol
risk of tak ,ng
such

A. Such medications
are safe and causes
no possible he
h t0 coming baby
B. We should such
stop medications right now
,
C. A plan is needed for you and your baby 0 control hype
D. A plan is needed for you and your baby to rcholesterolemia
control possible
disorder bleeding
E . A lipid profile is needed for baby after birth

78. What is the MOST common presentation for late onset hemorrhagic disease
of the newborn ?
A. Cutaneous bleeding
B. Bleeding from injection site
C. Ear -nose -throat and mucosal bleeding
D. Gastrointestinal bleeding
E. Intracranial bleeding

79. Which of the following causes of non -immune hydrops can be treated
prenatally ?
A. Supraventricular tachycardia
B. Gaucher disease
C. Acardiac twin
D. Fetal akinesia syndromes
E . a- thalassemia

-mesenter ic duct OMD ?


80. What is the MOST common remnant of the omphalo
A . Umbilical polyp
B . Umbilical granuloma
C. Umbilical sinus
D- Umbilical fistula
E - Meckel diverticulum

202
nt of umbilical granuloma ?
81. What is the BEST treatment
A. Topical use of silver
nitrite
pads
B. Topical use of alcohol
-iodine
C. Topical use of povidone
D. Ligation of the base and
excision
E. Frequent soap and water
toilet

82 . A 5 -day -old newborn presents


with lethargy, mild umbilical qj
. IScharge
redness of peri- umbilical tissue
Of the following, the MOST appropriate antimicrobial treatment is
A . Oral amoxcillin
B. Vancomycin pulse amikacin
C. Ampicillin plus gentamicin
D . Vancomycin
E. Cefuroxime

83 . A young parents counseled you about the possible risk of antidepressant


medication ( fluoxetine ) during pregnancy .
What is the MOST appropriate answer ?
A. It ' s safe to use such medication
B. It ' s acceptable to be used as so far has no recorded risk registered
C . We can use it with caution, if its benefits outweigh its risk
D. We can use it only in life threatening emergencies
E . We should not use it

84. You are taking first on-call duty in postnatal ward when staff nurse cafleo
you for an 8-hour-old baby who has fever, weak suckling, irritability, tremor
and weak cry . Mother is giving history of taking fluoxetine started in 1
trimester for major depression.
Of the following, the MOST appropriate action
/advice is to
A . advice the mother about proper positionimg
and attachment on r
B. explain this is not related to side
effects of medication
C. order RBS and advice for small frequent
breast feedings
D. stop breast feeding and start
bottle feeding
E. admit the baby to NICU

85. Which of the following is


differentiating primordial dwarfism from Pituitarv
'
dwarfism?

203
It had in utero growth failure
hyperbilirubinemia
B it has
* it has hypoglycemia
micropenis at birth
Q it has
B It has
associated birth defects

colleague is asking about the


86 A
genital hypothyroidi sm and
maximum tilme limit
by which ,,
t0 S ar
r««men ,
dev elopment
7
. Your
days
answer
of life
would be
We can „
still '
achieve normal
A. first
B. first 14 days of life
C. first 21 days of life
Q. first 30 days of life
E. first 2 months of life

MnstiSZ^
vomiting, increased tone, and
A fi m
appreciated on the trunk. All laboratory work up shows
from hypercalcemia.
Of the following, the MOST likely diagnosis is
^^ ^ rpte ZLle''' '
6
can be
normal results apart

A. parathyroid hyperplasia
B. subcutaneous fat necrosis
C. idiopathic hypercalcemia
D. maternal hypoparathyroidism
E. transient hyperparathyroidism

88. What is the confirmative diagnostic test for salt wasting congenital adrenal
hVPerplasia ?
A . Decreased serum glucose
B. Elevated serum potassium
C- Reduced serum cortisone
D - Elevated 17- hydroxyprogesterone
E Reduced serum
.
aldosterone
a, hypoplasia of the

^
A emale infant with webbing of the neck, lv phe
** , (oW.set ears, high
-
nipP s, cutis
‘* laxa, and low hairline at the nape ofthe
arch d Palate, and
cubitus valgus should be
suspected
eck
J
A - Noonan
syndrome
204
ndrome
B.

M uhMC in pregesuvi
‘ %a

a
f **
<5
<6%
0.
t. <7%*? of normal vaginal delivery to a

^^ 7
C
91. A full-term es develops

is on 02, supportive
withpulmo

^
symptomatic.
Of the following, the MOST
appropriate action is to start
A. digitalization
B. dopamin
C . propranolol
D. captopril
E . amlodipine

92. A fullterm macrosomic baby boy delivered by difficult and assisted normal
vaginal delivery . History reveals, mother had pregestational diabetes with fair
control and the baby had passed motion only once since birth. On 36 hours of
life, he develops decreased activity and feeding, and red color urine.
Examination shows fair hydration, HR 140 b/min, BP 100/75 mmHg, and left
sided flank mass. Investigations; RBS 44 mg/dl, TSB 12.9 mg/dl, blood gases
normal, HB 20 gm/dl, WBCs 22000/mm3, and platelets 55000/mm3.
Of the following, the MOST likely diagnosis is
A . small left colon syndrome
B. Hirschsprung disease
C. congenital infantile polycystic kidney
disease
205
I dysplastic kidney
o- ilateran thrombosis
^reoalvein
°l
rs who are elected for cesarean section the BEST
,rtStoPera
3.1" jve breast
feeding delay problem is to
enatally expressed breast milk
solution to solve

rA - US
nge for spinal anesthesia
B- arra | dextrose gel
C- use ora milk with spoon
e formula
?£ use
'

breast milk bank

u/hat is the preceding target plasma glucose concentratilon in


94. of diabetic mothers ?
first 48 hrs
l in infants
fel
of
' A. ^30 /
mg dI
mg dl
B. 35 /
*
C. >40 mg/dl
D. >45 mg/dl
l > 50 mg/dL

glucose level at which IV glucose is indicated in neonatal


95. What is the plasma
hypoglycemia in first 4 hours of life ?
A. <20mg/dL
B. <25mg/dL
C. < 30mg/dL
D. <35mg/dL
E. < 40mg/dL

96. Screening for asymptomatic hypoglycemia during first day of life among at -
risk infants is indicated for
A. post -term infants
B. infants with congenital infection
-
C infants of obese mothers
0. infants with birth asphyxia
E - infant with
mothers on beta blockers

Which identify neonates with


of the following markers have ability to
'°Us bacterial infection?
Erythrocyte sedimentation rate
206
neutrophil count
B. immature-to- total
C-reactive protein
C. Serum
D. procalcitonin
E Thrombocytopenia

empiirical antibiotic therapy for early onset neonatal s


98. What is the ePsis?
A. Ampicillin
6 Ampicillin + Aminoglycoside
Aminoglycoside
C . Vancomycin +
0. Cefotaxime + Aminoglycoside
E. Cefotaxime

99 you are explaining the


non- antimicrobial treatment
strategies and th
- CSF and IVIG in treatment
of neonatal sepsis to
of recombinant GM aj
colleague.
sentence .
Which of the following is the MOST appropriate
A . Recent studies shows encouraging results
of their use
B . Recent studies has established their role
C. Studies shows a lot of adverse effects during their use
D. There is insufficient evidence to support their clinical use
E. There is no role in treatment of neonatal sepsis

100. Which of the following is true regarding the dose of empiric antibiotic
regime in early onset sepsis ?
A. Ampicillin 50 mg /kg/dose BID, gentamicin 5 mg/kg / dose BID
B . Ampicillin 50 mg/ kg/dose BID, gentamicin 5 mg/kg / dose OD
C . Ampicillin 100 mg/kg/dose BID, gentamicin 4 mg/kg/dose 0D
D . Ampicillin 150 mg/kg/dose BID, gentamicin 4mg/ kg/dose OD
E. Ampicillin 50 mg/kg/dose BID, gentamicin lOmg/kg/dose OD

101. You are reviewing the clinical data for a G 2 PI mother who has elected for
induction of labor tomorrow. Past obstetrical history reveals GBS infection «f
her previous baby during early neonatal
period.
Of the following, the MOST
appropriate plan is to
A. order for GBS screening
vaginal culture
B. give GBS vaccine to the
mother
C. start combination f
n
broad spectrum antibiotics to the mother
°
D- give intrapartum penicillin
chemoprophylaxis
culture for the
blood
col |eCt neonate after
l dingly birth and
r tre« him
acc °
ek.gestation, G1P0 mother presents with
A I ** sits and hef 6BS StatUS S
6 *i labor, she... had
' "

-
taicafd 10hrane unknown
' . She has neither Poor

^^', 0f m
*" .
fever nor
*

ing lh M0ST
U appropriate action is to
ie f0 llo '.
* '
0ft orde rapld intrapartum nucleic acid amplification test
A '
cortibina
6 start rapa
tior1 of broad spectrum antibiotics
to
NAAT for GBS
the mother
, ^ . chemopr 0 phylaxis
iP
0|lect
°
- CCOrdingW
‘'
C a ve b d ru|ture for the neonate after
-D *
° delivery and
*reat him
3 tjc therapv to the
I start err P** neonate

of the following congenital viral infections causes


103. Which ?
persistent

postnatal infection
A. Mumps
B
B. Coxsackievirus
C. Rubeola
D. Smallpox
E. Hepatitis B virus

congenital viral infections can cause congenital


104 Which of the following
glaucoma ?
A. Cytomegalovirus
B. Rubella virus
C. Treponema pallidum
D. Toxoplasma gondii
E. Human parvovirus B19
causes mild
1Q5- What is the viral infection if acquired during P
an( j cerebellar
mfection to mother but causes microcephaly, issen
hypoplasia to the baby ?
A - Rubella
B. vzv
C. Coxsackievirus B
D - HIV
E. Zika

208
MOST common health care acquired infecti
106. What is the
th ne
%° ! Moated
Cent al
associated
bloodstream
pneumonia
infection

I ventilator
-

r Surgical site infection


urinary tract infection
n catheter-associated
dialysis catheter infection
'
E. Peritoneal
microorganism causing the largest
107. What is the predominant fractn
>i $
acquired infection in the NICU ?
health care
A . Coagulase -negative staphylococcus
0. Staphylococcus aureus
C. Enterococcus
D. Escherichia coli
E. Klebsiella pneumonia

108. What is the MOST common reported microorganism in ventilation


associated pneumonia ?
A. Enterococcus
B. Gram-negative rod
C. Coagulase -negative staphylococcus
0. Staphylococcus aureus
E. Escherichia coli

109. What is the MOST effective measure in reducing fungal colonization and
invasive infection in NICU for babies < 1000 gm birth weight ?
A. Limit the use of broad-spectrum antimicrobials
B. Limitation of postnatal corticosteroid
use in VL 8 W
C. Early enteral feeding with human milk feeding
.
D Prophylactic administration of
fluconazole
E. Early extubation

209
f

and the Neonatal


' Infant
Answers
TAWFIQ
my0c/o „ , Sei l e u 3
Benign neonatal "
, *
m “' 2°
neon cessation aro W With a $ent concomitant EEG findings
°*
.1(0 3 al onse> mvoclomr er < dUri non- rapid eye movementcharacle
disorder nze
sleep .and
» »..

consistent in a well - baby requires no investigations .


. Such

mm While prematurity is the main cause in early neonatal period


( fi rst 7 days
1(A) is takes the lead in late neonatal life .
. ( life )
the sepsi
no , d str3Cters use white reflex wh
, always
HIIother “
ribs and clubfoot are recogniZ ,
also
P«holo lcal
*
5.W Thm
6. (81 Full -term , vigorous infants may
delivery, during which
^ , *^
be Pieced on K ,
after Yed imping 0
ibdomen sec) is recommended to * circuitT'
-
cord W 60
crease
DC )
neonatal red blood cell (RBC) volume
^ ******
,
Clearing the mouth of secretions with gent e Suct on with a h,,IK
'
, and

indicated if there is an „
soft catheter is

Spon
to clear their airway .
8 ( A ) Famous drugs that is contraindicated in a nursing mothers are
amphetamines, antineoplastic agents, bromocriptine, and chloramphenicol
while other distracters can be used with caution.
9.{ C) For TB, treatment for 14 days is usually suffice; for all types of hepatitis
breast feeding is not contraindicated, while in varicella covering of the lesion is
enough.
10 ( E)
Q The ultrasound criteria for these diagnoses are based oni either the
is
iniotic fluid index (AFI ) or a deepest vertical pocket (DVP). The AFI

^ ermined by measuring the vertical dimension of amniotic fluid pockets


rants ar|d reporting the sum of these values. An index >24

210
in 4
cm suggests
ios, where as
an index <5cm suggests
0hvd olig
polyhyrammos pocket of fluid identified Wfth 3 arnn os
method reports the deepest '1?! K

considered normal. ^ S
.hypo
*' ' P'aCenU'UanS,W
t .
3

a(vesthe

«^
a

w
St
bloc

«
« °
*
tSS
^"sv^
\tne
«
^
' o,W> oUl
l
J '
^^
ei
S* '
nte6nan
*
s

a prtmarV
S
perlnaW lran
-^s.*ss t ?« 5i
’ sequetae occur
neonates « .
signs and symptoms at birth, and as many as 25% can develop |g haw
with
«*
^

8
complications. The most common complication is congenital hearin *Srrn
Severely affected infants have an associated 30% mortality, and 65-
survivors develop severe neurologic morbidity.
in
of
15.(B) Before implantation ( 0 * 2 wk postconception), radiation doses of 5-
I0 ra<j
may result in miscarriage. At 2- 8 wk gestation, doses in excess of 20
rad have
been associated with congenital anomalies and fetal growth
restriction.
Severe
intellectual disabilities can occur with exposures of 225 rad
before 25 wk
gestation .
16.{D) Start with 0 then E.
17.(B)
18.(6)
19.(A) MRI is the most sensitive i
imaging modality for detecting hypoxic
injury in neonates. brain
20.(D)
21-(B)
22.(0)
23.(A) High dose erythropoietin
.
24.(B) For treatment of
refractory seizures in hypoxic
levetiracetam is preferable now days -ischemic
first or second line, encephalopathy
as ,
commonly used include; third-line agents
should also be attempted, midazolam, topiramate, and
abnormal EEG background. particularly in lidocaine Pyridoxine
ongoing refractory
25.(C) seizures with highly
26.(E)
27.(A)
28.(0 All are iimportant
but in order of sequence of
C, B A, D, E.
'
211
,
j|/ < ” *
0ve«n- H f
' reSUSO ati0 with

,
" C ^^tUtlor, Kad

-
U JS W' bl f ^ '°7
!t^pln adlSste
0 f nd 8enera ion Of
oxygen
shouldThen bTt J" *"" WtaTS
IS « w«h

^
ed in

><<t "
e ** * s s;a
saturations in a term iinfant after
range by minute of life.
<
#^
thywnthines are good choice for apnea of prematurity
„rnl baby wth good recovery and had no comnlir .7 S'
tor MKe,wise

"Cat apnea of prematurity can happen late even after wee n


^
in

7 oped RDS wWle


il 3PPearS ear ¥ in the first 7
’ days iin those**“** who
RDS free

jf
£updated
Jw "
) Gastroesophageal

« SE«
with apnea
and apne
gastric add production
sepsis,
. reflux ( GER ) is common in
anecdotally
c events . In preterm
neonates, but despite beine
data do not support a
causal
.
relationship
infants, medications that inh bd
have potentially harmful side
necrotizing enterocolitis,
effects (increased
incidence of death) and may actually iincrease
M incidence of
apnea and bradycardia. Therefore the| routine
use of
medications that inhibit gastric acid synthesis or promote gastrointestinal
motility to reduce the frequency of apnea in preterm infants should be
discouraged.
11(D) This baby had TAPVR ( mostly obstructed type) who is diagnosed by
echocardiography. Although infants of diabetic mother can present in term by
RDS they can also presented with congenital heart disease. The key here was
quite tachypnea without respiratory distress, hypotension, minimal response to
surfactant and hepatomegaly. The clinical and radiological picture of TAPVR and
.
RDS are much similar Congenital pneumonia is a possibility and collecting septic
screen and starting empirical antibiotics should have been done initially.
Antenatal corticosteroids do not increase the risk of maternal death,
^orioamnionitis, or puerperal sepsis. Betamethasone and dexamethasone have
toth been used antenatally. Betamethasone may reduce neonatal death to a
TOer extent than
(D) The
dexamethasone . benign
.
lesion k
L

description is for cephalohematoma which is


appears usually after 2weeks 3months, but a sensation of centra ep
-
tetmg but not indicative of an underlying fracture or bony

212
f
usually linear and not depressed
.
^ skuu
fracture,
. The %r
underlying jated
scenario
in
is
10 -25% of
indicative
cases
of severe subga|ea (Sut) , %

S
Ld
.
hemorrhage, baby should

loss; secondary
coagulopathy (hemophilia
be admitted

causes
to NICU, monitored f0r
, and consumptive coaguloPathy
and hyperbilirubinemia in such severe bleeding
3s
) should rolled-out. less severe cases S(
*
1>,
observation of progression of the lesion. b
admrtted also for close
subdural hemorrhages following labor should
37.(f) Most asymptomatic
r

by 4 wk of age.
that premature infants <32 wk of gestati0
.
38 E( ) Its recommended
time cranial ultrasonography (USj through **
with routine real -
evaluated
fontanel to screen for IVH . Infants <1,000 g are at highest risk ^
anterior
should undergo cranial US within the 1
st 3-7 days of age, when approxim# ?
75% of lesions will be detectable. All at-
risk infants should undergo follow -up
adequately for PVL, as cystic changes
at 36-40 wk postmenstruai age to evaluate
mo.
related to perinatal injury may not be visible for up to 1
39 (0) Classic presentation of ICH in a preterm baby.
.
-
40.(B) Applying nCPAP at 5-10 cm H20 with at FI02 of >40 70% is indicated if
there is significant respiratory distress (severe retractions and expiratory
grunting) or if Sa02 cannot be kept >90% and usually produces a rapid
improvement in oxygenation . Warm, humidified oxygen should be provided at a
concentration sufficient to keep Pa02 between SO and 70 mm Hg (91 95% -
SpO;) to maintain normal tissue oxygenation while minimizing the risk of 02
toxicity. Second dose of surfactant usually considered after 6-12 hours after the
first dose.
41.(A) By altered imprinting assisted reproductive technology can lead also to
Silver-Russel, Angelman syndromes.
42.(0) During mechanical ventilation, oxygenation is improved by increasing
.
either Fi02 or the mean airway pressure The mean airway pressure can be
increased by raising the peak inspiratory pressure (PIP), inspiratory time,
ventilator rate, or positive end-expiratory pressure (
pressure is usually most effective (
.
PEEP) Adjustment in
but we should not forget its case dependent )
oTdelvery^
output an
^
43.(C) Hyperoxia contributes to lung
lower target range of oxygenation (85-
95 %) increases mortality and does not
retUm thereby reducing
'
injury in preterm infants. However, a
89%) compared with higher range (91 -
alter rates of BPD, BPD/death, blindness,
213
w
lopme 31 imPairmen - Therefore h
"' * ,
CUfrentiy
^ targets is 91-9596.

^
> )
0f
en saturation
.dance of mechanical ventilation ,
with he ear|
, raJy , ° -
u$e <
recom

l *£L |f) surfactan t replacement therapy Wi


, JpP ‘ *
h e
0Ap
# J i^ s
1
of BPO.
£|ated
Modestly Caffetne therapy or
with a decreased risk of BPD.
"'•
ub tion d
«cfei4t
maturity ha,
" 0
past, intratracheal surfactant replacement ,
*"C"
0 infants immediately after birth (prophylactic) 0 rV n


P* (early rescue
) showed reduced air leak an
! "* teRt'•*
i
" . substan tial evidence supports the feasibility
no , ' , nCPAP as the primary means of respiratorv ,, *ffitKV *ol
prop*
J>:,r» K:
fer,r»r
£“ 3 All the above
eloped pulmonary edema deterioratio
cardiac
n
-of renal
-ssr-:
mentioned actions are true, but evidently the baby
and
function
has

volume overload. The first action is to


tZ
undiagnosed PDA and consequent
and order fluid restriction,
minister diuretics renal function results. meanwhile waiting for an
^jchocardiography and Starting cyclooxygenase (COX)
or ibuprofen) is a later on decision depending on
inhibitors (indomethacin
progress and case response to supportive treatment.
discharge in the majority of extremely preterm infants
48.(0 BV the time of
(>90%), the PDA will close spontaneousl
y. Spontaneous ductal closure may be
facilitated by general supportive measures, including early (<7 days of age}
avoidance of excessive fluid administration and judicious use of diuretics to
manage pulmonary edema. Pharmacologic and surgical ductal closure may be
indicated in the premature infant with a moderate to large, hemodynamically
significant PDA when there is a delay in clinical improvement or deterioration.
W) Transient tachypnea of the newborn is a clinical syndrome of self -limited the
tachypnea associated with delayed clearance of fetal lung fluid. Although
tual incidence is likely underreported, it is estimated at 3 *6 per 1,000
term
* of tachypnea in the
'^ant births, making
the most
TIN common etiology
^wborn.
, by rapid identification
, )
D| r sk of
meconium aspiration may be decreased
of late fetal
0
etal dlstress
and initiation of prompt delivery in the Presenc
.
*initial
^ art rate
deceleration or poor beat-to-beat FHR variability

214
— -
enthusiasm r
%
delivery,

«« «0through «»• •.
* " » born
aspiration of aep
» * "- stained f
meconium a
uld ! ° | * r

ef, ,iVe
decreased respjfaor MAS or other major adverse outcomes and " ^
(
eC
recomme IdedwithoTneonatal
resuscitation.
severe MAS developed severe
PHT which causes
funct onai
51.(C) Babies mimicking congenital heart disease. '
'IW
H PeRistent

^
feta circulation
^pulmona
occurs ^
in
hypertension of the newborn ( PPHN ) or persistent
term and postterm infants most often. Predisposing
factors include birth asphyxia
, MAS, early-onset sepsis, RDS, hypoglycemia ,
oolycythemia, maternal use of nonsteroidal
antiinflammatory drugs within

utero constriction of the ductus


arteriosus, maternal late trimester use of
pulmonary hypoplasia caused by
selective serotonin reuptake inhibitors, and
or pleural effusions
diaphragmatic hernia, amniotic fluid leak, oligohydramnios,
PPHN is often idiopathic. Hypoxemia is labile ( changing
) and out of proportion
to the findings on chest radiographs .
53.(B) Inotropic therapy is frequently needed to support blood pressure and
perfusion. Whereas dopamine is frequently used as a first -line agent, other
agents, such as dobutamine, epinephrine, and milrinone, may be helpful when
myocardial contractility is poor . Some of the sickest newborns with PPHN
demonstrate hypotension refractory to vasopressor administration. This results
from desensitization of the cardiovascular system to catecholamines by
overwhelming illness and relative adrenal insufficiency. Hydrocortisone rapidly

—--
upregulates cardiovascular adrenergic receptor expression
and serves as a
hormone substitute in cases of adrenal insufficiency.

zzr;;r.r“rrri
54.(E) Infant with diaphragmatic hernia CDH at
. .. . A
birth who are stable at birth
«
the management of CDH. In the delivery
center with good experience in
room, those with respiratory distress
*
should be rapidly stabilized with
endotracheal intubation. Prolonged mask
ventilation in the delivery
^ sTbe ' V
^
and thus makes oxygenation more
difficult, av ded
d and a naso- or
orogastric tube placed immediately for
decompression. Gentle
permissive hypercapnia reduces lung ini ventilation with
njury, need for
ECMO, and mortality.

215
contribute to pulmonary hypert
t
*nvP
ofS ' should
tha ,
3
be avoided.
ension ( hypoma
,
acvdosis,
° ‘ ' al time 48hr after diaphragmatic defect is under
ther^
The dfi
to repair the
ss-icl virait' 3 t least relative indicators
stabilization and resolution debate. Most
of the
pulmonary
G d of stability
^Otional °°'
^
ensio0 are the requirement
hyPe r ve t ation only , a low peak inspiratory pressure, and F for
e js indicated to rule out 102 <S0 An
co ^ associate congenital heart

*^
cardioSr P QnarY and systemic pressure
3
ecb sS the Pu disease and
se ° * .
th wen grown child with normal
to **
(fl this
cen
the same lobe should rise the
development and the
rrent 1
lobe
sequestration ^ as n '°" possibility of
most of the eventrations is asymptomatic associated
and need no

"?Li Meconium plugs are associated with small left colon


mothers, CF ( 40 ) Hirschsprung disease ( 40 . maternal
% %)
syndrome in infants
diabetic sulfate opiate ose
° nesium refers to
therapy for preeclampsia , and tocolysis.
intestinal obstruction , usually in
Mecomum p|u«
the distal colon, rectum,
SVvndrome , caused by meconium plugs . Initial treatment may include
and anal canalof a glycerin suppository or rectal irrigation
with isotonic saline.
administration -ray for
58.(C) The characteristic
X meconium ileus shows dilated small bowl due
rneconium in terminal ileum which eventually will cause
to inspissation
, B non-specific and occurs in distal bowl
microcolon. A in intussusception
obstruction, D in midgut volvulus, and E in duodenal atresia.
is usually in the 2 nd or 3rd week of life but can be as
59 (D) The onset of NEC
late as 3 mo in VLBW infants.
go ( A ) The most effective preventive strategy for NEC is the use of human milk .
exclusively breastfed have a reduced risk
it is well documented that newborns
the use of
of NEC. While extensive data and meta - analyses would support
, most
probiotics to prevent NEC, there is no clear consensus on the safest
therapy. Other
effective formulation, timing of administration, or length of
preventive strategies using prebiotics and synbiotics have also
been studied,
(H 2 receptor
with variable outcomes. Inhibitors of gastric acid secretion
blockers, proton pump inhibitors ) or prolonged empirical antibiotics in the eary
NEC an
neonatal period have been associated with increased risk of
avoided. underlying a -
61 (B) MCV <100 fL at birth should prompt consideration of
-

f
° “-thalassemia syndrome should
unlikely if mother

be considered in the
216
.. .
different ! d agnos
. m0Vernent is the most
common Sl
*
aSS0C
'athas Pa or' Ttest,
" •W
* f ' the go d standard bUt
dassic
resistaot *
elotion, « ' ' * »•
S abor
*
^ #» rf Ora tori e s *
to
on the
the
*
^ .
advance d ah offer a
depeo . « more
^*w
highW
« ' PJT - of ca ta$
P d or
testusingfl
0
^^
ometry od
pical p ctufe
,W W»
Q qUant«
' ***« ^ lC cirtulation.
disease of the fetus and ne ,
d •
W
uggesting enzyrnopathv whrch is

**
wSth reticulocyte

sgszs=&.
morphology
' nsfusion thr
Absence of RS
Presence of RS
11.5 (35%) 10.0 (30%)
Week1
10.0 (30%) 8.5 (25%)
Week 2
Week 3 8.5 ( 25%) 7.5 (23%)
In addition to these factors, transfusion should be considered
for infants with
acute blood loss ( >20%) or significant hemolysis, as well as before surgery
. With
no similar evidence-based guidelines for term infants, transfusion should be
based on hemodynamic stability, respiratory status, overall clinical condition,
and laboratory values.
65.(A) Factors that affect the outcome of antigen-positive fetuses include
differential immunogenicity of blood group antigens (RhD antigen being the
most immunogenic), a threshold effect of fetomaternal transfusions (a certain
amount of the immunizing blood cell antigen is required to induce the maternal
immune response), the type of antibody response (
IgG antibodies are more
efficiently transferred across the placenta to the fetus)
mother and fetus are also ABO incompatible, the Rh negative
.
Notably, when the
mother is partially
protected against sensitization due to rapid removal of
the fetal Rh-positive
cells by maternal isohemagglutinins (preexisting IgM anti A
that do not cross the placenta). The severity of Rh illness
- or anti-B antibodies
typically worsens with
successive pregnancies because of repeated immune
n
stimulatio
217

Vcem a occurs i |„fanfs
, C*
W
661 1 H
* "T
^ ^
5U
and hypertroPhy o
dlstracters are more relate(J
0 * (exchange transfusion).
th
condition
«
Severe HDfN anrf
« ,7, " *
he C0*hplicat
and
* n S in these Infantj°
|

" tr »tment of '* ^ '


genotyping pr the
1C) fetal o HDFN in sensitised m ,
an accuratP predi
jlfi/elopment thers «»on
isolating feta / cells or fetaJ DNA ol^
( ° - feta| DL
fr m the Status is awa»|aK, the
. "? °
" ' , C ‘“ ^
mat*
replacing the more invasive and ri t C rcula
«on, w .
«
villus sampling
and chorionic Wto an methods * 1^
establish
ri
, " amni 0cYt<? test

*, "' esis
,
orM
6«.(EJ
*^
11 an -negative mother is " ''
fn
ol tlc process r , ated dr
han
i bodv »
'
(15 lU/ mL in Europe ) at any time! '
durin° have fih
'
0
° antib ^' , ..
dy
H
nion
of a
,
fe al anemia should be * ** **
,
"
monitoredd bY D0pple ,.
*
« p
P 8.nancy
n
the sevP „,
6

middle cerebral
nw'*01«««»' wC2"
artery ( MCAI U
«£ 7 ,
* the
t a„JJ? * »M
>w
for
*anemia, * t r n t a o c« R8Cs) »»e J, " Mical
"£fetalis also "effective
” * although intrauti
15 f
' "
prefer
3r (l

of
vein)
t
nsfusion ‘hoice
****fetal^ anamia into * fetal Peritoneal
cavity .

indication for umbilical vein


m Extramedullary hem m*
nydr
transf in « with , .
^ *< «
/hp

**haP«ic congestVimmaturi
n
*!' )
V-

^
' stasis with „
5 and
intrahepatic vessels and prod COmpre
the
*
eCreased albumin JnT , tension
hepatocellular dysfunction and a
typically present when feta ) h g ,ob, HVdr
*HydroPs e s- 0ps
,
js
seen with a fetal Jmoglobin *
n evef js <
frequently ° 5
level < /df
7 L is also
7„ and in «on» cases
between 7 and 9 /dL g
70.{B) If clinical signs of severe
hemolytic anemia {pallor,
edema, petechiae, ascites) are evident hepatosplenomegaly,
at birth, immediate
supportive therapy, temperature resuscitation and
stabilization and
, monitoring before
proceeding with exchange transfusion may
save severely affected infants. Such
therapy should include a small
anemia; volume
<0.
transfusion of compatible PRBCs to correct
correction of acidosis with 1-2 mEq i
/ <6 0 s0
ventilation for respirator 1 HOfN shoul(i t>e closely
monitored with frequent hemoglobin
need for phototherapy
a “
bin testing to determine their
n.(D) Because of its , simple transfusion, ore hanee transfusion.
ability to interfere wit > e(jiated clearance of
antibody sensitized RBCs,
early administra ,0 may be an effective
therapeutic intervention for
HDFN. WIG can p immune hemolysis, lower
218
f

levels, shorten the duration of Phot


peak serum bilirubin
.
both length
does not
of hospitalization
effectively
RBC destruction and
are
prevent
and need for exchange
anemia
inadequate
, which results from
erythropoiesis .
usually needed as an adjunct to IVIG therapy
both^^
trarJ' ^VS V» .
*

t >
transfusions
0.5-1 g/kg is typically
used, but optimal dosing has not be,
groups A or B should be
''
V G
V
Treated infants with blood -B antibodies
monitored f 0r
in IVIG . °' * '“-
hemolysis caused by anti-A or anti
present
for immediate exchange transfusion is
72.(A) The only 2 indication
encephalopathy (jaundice, fever, lethargy «
signs of acute bilirubin
arching, retrocollis, high pitch
cry) and a total bilirubin is more than
the accepted level for age in hours (the standard graph of hyperbilirubjn !'^ 1
*
73.(E)
always be confirmed with a venous
74.(A) A capillary Hct >65% should
and dehydration should be treated. All polycythemic infants should be cl
^
monitored for intake and output, and blood glucose and bilirubin levels shouji
be closely followed. Asymptomatic infants whose central Hct is 60-70% can be
monitored closely and hydrated with adequate enteral intake or administration
of intravenous (IV) fluids. Treatment of symptomatic polycythemic newborns is
not well defined. A partial exchange transfusion ( with normal saline) can be
used in infants with severe polycythemia and symptoms of hyperviscosity and
should be considered if the Hct is 70-75% and symptoms worsen despite
aggressive IV hydration.
*
75.(D) Plasma levels of the vitamin K-dependent coagulation factors (II, VII, IX,
X, protein C, protein S) and antithrombin are low at birth and do not reach adult
ranges until approximately 6 mo of age.
76.(B) This classic form of hemorrhagic disease of the newborn, which is
responsive to (and entirely prevented by) exogenous vitamin K therapy, is
characterized by hemorrhage that is most frequently gastrointestinal, nasal,
subgaleal, intracranial, or post-circumcision. Prodromal or warning signs (mild
bleeding) may occur before serious intracranial hemorrhage. Classically, vitamin
K deficiency bleeding occurs early in the newborn period, typically between day
2 and 7 of life, and most often in exclusively breastfeeding infants who did not
receive vitamin K prophylaxis at birth. Severe vitamin K deficiency is also more
common in premature infants. This baby had home delivery and he didn' t
received vitamin K prophylaxis.
77.(D) Early-onset vitamin K deficiency bleeding (after birth but in 1st 24 hr)
occurs if the mother has been treated chronically with certain drugs (e.g-,
219
,
nt warfarin
^^
lsant
icoagulalowering
afif - medi n tf>at
. ftJjS**
chol e st e rol . ° ,
^
of high r
paction. Avoidance o

50on
«/ promptly
corrert response, if a
delayed
by Wf

^ ^
rnotl ^ istJ^ ^
bleJ
can haye t

>llbe*neasurl?,***'£J*^
' *», •
*
* i, ? "
kn0VVf) be ** . ^
«
ratl0

V °°f or
'^ to

g 5
*
tawn an infant Pr sh(
11- mg of
* °
imme r m the Presence D Significant f s toaf/,
^ ^ ^ ,'
' ***
c d6 and * * >


improve, , ** 1
, ' ', ***
shou/dbe adm S er d.
frozenp/asma onset vn
If n
» hage
,

^^^ fed

P «e ate
* *
7 ( E)
intracra mal bleeding , * * ^ usually present with

uter0 treatment has been successful for fetal


serious

79 .( A ) m supraventricular

^ hycardiafSVT ') twin twin transfusio


- n syndrome , nonimmune fetal
anemias
and some surgically treatable fetal conditions.
gO.(E) The most common remnant of the OMD k a M„ L , „ rticulum, where
, ^^ ,
..
as abnormalities that would become symotom3 C the neona al Period
, ,
include a sinus or fistula that would drain m r„, °f -'nteS nal conten s through
.
the umbilicus. An umbilical polyp i5 0ne of the i
and represents exposed 6 mucosa at the umbfa
,
'
” ^T" °
polyp is bright red, firm, and has a mucoid secretion Th6rapy r al1 0MD
°
* f the

remnants is surgical excision of the anomaly.

-
gl.(A) Persistence of granulation tissue at the base of thp nmKiii

y “‘ T M0 mm in
pink, and may have a seropurulent discharge. Granulation tissue is treated by
cauterization with silver nitrate, repeated at intervals of several days until the

--s t-r
the peritoneum, the umbilical or porta vess ,
omphalitis includes prompt antibiotic therapy wl
Staphylococcus aureus and Escherichiacoh suc ^ ^
effective against
antistaphylococcal
,
:
penicillin or vancomycin in combmatio
formation has occurred, surgical incision an
, ^
may be required
f abscess

re t0 these medications
,

83.(C) All psychotropic agents cross the P acen a


,
in utero may lead to a higher risk of congenial ma forrriations
poor neonatal
(

220
el and persistent pulmonary hyPen
(pNA )'
adaptation undrodefeme anencephaly, atrial septal
cllude > '*
reported ^ '" ob
|on, omphalocele, and
gastr % .
Some
ventricular outflo w tract ' djcation during pregnancy is gradedS \

-
of safety of anV
g when animal studies shows
classification qoestiofS A 15
as oUr scenario
the clinica
n0 n >

~
X. As given
the l findings
n studies, C '
u d risk VS benefits should
i be

.~~zzrrsf-ast^
>

sgris
.
autonomic, resp ratory. and
gas
and hvpotonia, hyperthermia.
k refle*, irritability, tre
* °
absent cry, sleep disturbance
;esT- p
VP0g|ycemia, respiratory problems, von* *,
and diarrhea, and Mos V
sejtures
are mihJ> and severe symptoms a *„
,
rare No deaths have been rep^
measures, and most cases arti m ,
frequent, on -demand feedings,
.
Treatment consists of support ,
durati0n, and self-limiting. Small
^
and skin-to-skin contact
,

beneficial to support DroceSs. *


Breastfeeding is protective
against developing
antidepressant medications are safe with breastfeeding.
85.(A) Pituitary dwarfism (growth hormone deficiency) is not usually apparent
at birth, although male infants with panhypopituitarism may have neonatal
hypoglycemia, hyperbilirubinemia, and micropenis. Conversely, primordial
dwarfism manifests as in utero growth failure that continues postnatally, with
length and weight suggestive of prematurity when born after a normal
gestational period; otherwise, physical appearance is normal.
86.(D) Congenital hypothyroidism is one of the
most common preventable
causes of developmental disability. Congenital
screening followed by thyroid
hormone replacement treatment started
within 30 days after birth can
normalize cognitive development in
children with congenital hypothyroidism.
87.(B) Subcutaneous fat necrosi
s can cause hypercalcemia and can occur after a
traumatic birth. On examination,
firm purple nodules can be appreciated on the
mn r extrem ties - An infant
°
.. ' *j with hypercalcemia presents with irritability,
hunlrrl? ' mcrease tone poor we 8ht gain, and constipation. Other causes of
#

acemia in t e newborn period ‘


are iatrogenic ( excess calcium or vitamin

221
hyp<>ParathVroldism' Williams syndrome
aternal ,
parathyroid hVPerplasia
.
and* All are true but D is confirmative .
),
o idiopatNc ,

8.101
/ )
,
.
/9(8 The risk of diabetic e ,^^
Pathv eur l t , „. <%ec s
°
.
90(01 regression syndrome nd sPontan s ab
> *’
^***althe7 '
(

caudal diabetes * ' ntr


*
pregesta.tioThere ^
nal is a ? 'ncrease C° f 0i

trimesterwith pregestationx
mothers
9UC
) A and
B are contcaTn
'
Adrenergic resolves inri
theconditionvein
^ ^*
*
°*,
0Wtf
, .
,
d » it i Cr ases «,
blockers haye be n 5h *n , *
t h *r o 55m ?-b . o* b," *
6e
6s
^
' n tn>an

bs ti0n fi
.
c /0n° '"'
'
,*

thro* *
Kenal
9W
hematuria, and
2 * ' * t/ie a f ank

^
mass, nT **
anoT^ "* '
third trimester
becaJsTof“ 3 fare

glucose intestinal

° hWomen
93 ( A)
can begin to express breast milk before the birth of the baby
age); this will provide an immediate supply of milk
(>36 wk gestational to
event hypoglycemia . Others are still true alternative options.
feedings within 1hr after birth. A screening glucose
94 ( C) Infants should initiate
test should be performed
within 30 min of the first feed. Transient
hypoglycemia is common during the 1st 1-3 hr after birth and may be part of
normal adaptation to extrauterine life. The target plasma glucose concentration
is >40 mg/dl before feeds in the 1st 48 hr of life. Clinicians need to assess the
overall metabolic and physiologic status, considering these in the management
of hypoglycemia. Treatment is indicated if the plasma glucose is <47 mg/dL.
Feeding is the initial treatment for asymptomatic hypoglycemia. Oral or gavage
feeding with breast milk or formula can be given. An alternative is prophylactic
use of dextrose gel, although early feedings may be equally effective.
feedings or IV
95.(B) Recurrent hypoglycemia can be treated with repeat
th®r pv
glucose as needed. Infants with persistent (and unresponsive to oral *
birth and <35 mg/ a
glucose levels <25 mg/dL during the 1st 4 hr after
sVmp ®
hr after birth should be treated with IV glucose, especially >
) shokl be
> »11 bolus of 200 mg/kg of dextrose (2 mL/kg of ION dextrose small
;
"
administered to infants with plasma glucose below these Nmrts. The

222
should be <° aboUt an fan
queSti
°naf '
V infusion at
"* «*V4.g mg/kg/min should be given
. NeuX
* >
31 with
pehphef giycenua mus be treated IV glucose . Bolus ,
^*
symptoms 0 VP ^8iucose should because
^ they may •<
*
hvpert n e febound hypoglycemia.
°' tent a
mia V
' * " ° ' ,Ivooglyce
nen a d P
hypehnsull b during first day of life
hYPJ „
.
infants, those who are among
tomatiC
9640
flf'
^
-ndicated for at
* on, and infants of obese or
small

.°° .
-
lucose concentrations
have
bl g)uc()Se accordlngly. other
to do measu g
^
, :.«*
distracters it may be true
, | various serum biomarkers
, h jnvestjgated for their ability

t s,
t0

srr
97 B
(SBI). An immature-to-
tou
phagocyte
indices for predicting neonatal sepsis. After
01
*•”*•
the newborn period, serum C-
reactive protein (CRP) and procalcitonin have demonstrated reasonable
sensitivity and specificity for SBI.
98.(B) Option C for late onset sepsis, option E should be added in case of
meningitis in early onset sepsis.
99.(D)
100.(D) This has long been a standard regimen for early-onset sepsis and
provides coverage for the most prevalent organisms, predominantly GBS and
gram-negative ones. Ampicillin plus cefotaxime ( if available ) or cefepime may
be substituted if the patient presents with infection after discharge from the
nursery, or when infection with ampicillin-resistant
E. coli is suspected
L ,

Ceftriaxone may be substituted if premature infants are £


41 wk postconception
age; it may be used in term infants if they
are not receiving intravenous calcium
or do not have hyperbilirubinemia.
101.(0) Empiric antibiotic therapy for
iintrapartum GBS prophylaxis
previous infant with iinvasive indicated in;
GBS disease; GBS
of the current pregnancy; and positive bacteriuria during any trimester
GBS screening culture during current
gestation
102.(C)
"Unknown Pregnant W
°men
** SCreened for GBS 3 35-37 wks of

GBS status at the onset
incomplete, or results unknown) and of labor (culture
any of the not done,
weeks ' gestation, amniotic membrane rupture following: delivery at <37
£18 hr i
£38.0 c (100.4 f), intrapartum NAAT intrapartum temperature
positive for GBS

223
r

causes
.
v, Rubella, VZV, HSV, 88v
coxsackie v rus
premaurity.
causes ’ Hlv Snd
ab Causes
develoPrne persistent
^*'
5 P
l®4' 6
isually acquired by
anomalies; Rubeola
postnatal
and
l®5
' ’
f Ventilator-associated pneumonia
by surgical
mosquito bite.

site infection
and
(
VAP is the)
next most
catheter -associated
infeCtTlt for about 50
account
% of
common
urinary tract
5taP ccus aureus about 10
The most commonly
% and bloodstream
others < 5% eachinfections in the NICU,
, reported
organisms
.
negat ve
.1 ^ (including
Pseudomonas associated
source o fectjng organisms is generally), 5. aureus , and with VAP are gram-
Enterococcus . The

.
ntaminatecj thought
respiratory equipment to be the infant’s oropharynx
are occasionally implicated. and tracheal ,
suction catheters
re.ducesfungako onization
g Neonatal practices that
)
and
may L v L i J e E l B W,i an"'incluT
limited use of broad-spectrum reduce the risks of invasive cand
diesis
antimicrobials, use of an aminoglycoside
of a cephalosporin for empirical
iinstead
resistance is not suspected, and therapy when meningitis or
VLBW infants, early enteral limitation of postnatal antimicrobial
feeding, and corticosteroid use in
microbiome with human milk feeding establishment of the neonatal gut
.
'/

Chapter 12
Adolescent Medicine
Questions

puberty in an adolescent girl?


1 What is the hallmark of
A. Axillary hair
B. Breast enlargement
C. Menarche
D. Pubic hair
E. Growth spurt

2. What is the hallmark of puberty in an adolescent boy ?


A. Voice change
B. Axillary hair
C. Pubic hair
D. Testicular enlargement
E. Growth spurt

3. In a girl who lacks any pubertal signs; what is the age at which
prompt
evaluation for pubertal delay should be started ?
A. 10 years
B. 11 years
C. 12 years
D. 13 years
E. 14 years

work up include testing


A. LH
B. FSH
C. Thyroxin
D. free and total testosterone
E. 17-hydroxyprogesterone

225
able causeiniurV
recognizthalamic of high F S H j rT1en
°rrhea ,s
5\
- hyp
° injury
pituitary
'

0 failure
C ovarianathlete
D- female ovary
£ •
polycystic
arlY postmenarcheal years, the MOST common
6 |n
the
Uterine
bleeding in adolescents is cause of irregular and

^ ^
A novulatorV
B. eating
disorder
C. polycystic
cYcle
ovary disease
ic inflammatory disease
D. pelvic
causes
l hematologic
is the MOST common cause of secondary dysmenorrhea in
.
7 What
?
adolescents
A Mullerian anomalies
B. Endometriosis
inflammatory disease
C. Pelvic
D. Uterine fibroid
E. Ovarian cyst

226
chapter 12
Adolescent Medicine
Answers

l.(B)
2.(D)
the absence of menstruation, generally reqU|rK
(
3 D ) Amenorrhea ,
^
*' a 15 yr, or if there has been no menstruation within 3 yr of 2*
the V
puberty [primary amenorrhea
), or if there has been no menstruation ,
patient ( secondary
length of 3 previous cycles in a postmenarchal amenorrhea
However, the following caveats exist: lack
of any pubertal signs by age I3 yrln
girl should prompt evaluation for pubertal delay.
4.( B) According to FSH level, 2 groups of amenorrhea will be classify, those w
low and those with high FSH ,
5.{ C) Other distracters causing amenorrhea with low FSH.
6.( A) In the early postmenarcheal years, the most common cause of abnorm
uterine bleeding in adolescents is anovulation caused by immaturity of the
hypothalamic -pituitary ovarian axis . Other distracters are true but less common
causes .
7.(B) Secondary dysmenorrhea results from underlying pathology, such as
anatomic abnormality , or infection, such as pelvic inflammatory disease
However, the most common cause of secondary dysmenorrhea rn adolescents is
endometriosis, a condition in which implants of endometriaf tissue are found
outside the uterus, usually near the fallopian tubes and ovaries. Often, othe
family members have endometriosis . Although characteristically there is severe
pain at menses,
adolescents can present with noncyclic pain as well.
Questions

-old child has humoral tyne imm


* 2 - year
‘ |"
,
leameS S M0S ke V manifested in
th ^l d W h i C h o f the
f Recurrent bacterial pneumonias
G Chronic mucocutaneous candidiasis
H. Absence o f lymph nodes and tonsils
I. Recurrent infections with
J. Disseminated atypical mycobacterial 'nfection^ 5

2. Which of the following types of immunodefici


'
ciency is suggestive
develops paralysis after vaccination with live •attenuated when a child
poliovirus ?
A. B-cell defect
B. T-cell defect
C. Complement deficiency
0. Macrophage dysfunction
E. Neutrophils dysfunction

3. A 1-year-old boy has T-cell defect immunodeficiency. His complete


blood
count MOST likely
shows
A. neutropenia
B. neutrophilia
C- lymphopenia
0. eosinophilia
E. monocytosis

of the following immunoglobulin isotypes has a MAJOR role in host

^ense against parasites ?


A - IgG
8 IgM
c IgA
228
D. IgE
E . IgD

5 A medical student asks you


about postnatal immunoglob
ulin d
The statement that should be included in this discussion is evelo
A . serum IgM rises sharply at about 6 month after birth
B . serum IgM reaches adult levels by approximately 6yr of age
C. serum IgA remains low throughout infancy
st
D. maternal serum IgG disappears after 1 yr of life
E . total IgG are reached and maintained by 1 yr of age

6. A 1- year-old boy develops recurrent pneumonias and otitis H* (IfMU


Examination reveals absence of tonsils, and no palpable lymph nodes. Lab
tests
reveal very low level (< 2 standard deviations below the age -appropriate |eve
of IgG, IgM, IgA, and IgE. Peripheral blood flow cytometry
demonstrates
absence of circulating B cells .
Of the following, the MOST likely diagnosis is
A . common variable immunodeficiency (CVID )
B. transient hypogammaglobulinemia of infancy
C. hyper IgM syndrome
D . Bruton agammaglobulinemia
E . selective IgA deficiency

7. A 2- year-old boy develops pneumonia due to pseudomonas infection . He has


been diagnosed with X-linked agammaglobulinemia ( XLA ) at the age of 1 year.
Of the following, the WBC parameter that is typically seen with such infection is
A . neutropenia
B. neutrophilia
C. lymphopenia
D . eosinopenia
E. eosinophilia

8. Common variable
immunodeficiency ( CVJD ) patients may appear similar
c mically to those with X -
linked agammaglobulinemia ( XLA) in the types of
infections experienced .
Which of the following infections, is
MORE likely encountered in XLA than in
A. Hemophilus influenzae
meningitis
229
Streptococcus sinusitis
pneumoniae
B- meningoencephalitis
C Enterovirus
pneumonia
Myc0plasma
5taphVlococcus aureus septicemia
E
ldg I has recurrent pneumonias and

^'
.
A 7 ye meningitis with
'

9. Sh aS normal sized tonsils, enlarged


encapsulated
b3 teria
- lymph nodes,
, ^ "
immunoglobulin level assay
en0 eealV atjons ,below the age -adjusted
reveals very low serum and

>
tgG (<2
ard dev at ° norms), with low IgA and
ieVe
This
!s
>* *-

girl is particularly
<

,
at increased risk fnr ^ eve opment
of
igM

A. leukemia
B. lymphoma
C. neuroblastoma
D. nephroblastoma
£. medulloblastoma
boy has recurrent attacks of gastroenteritis
10. A 4-year -old caused by Giardia
lambiia.
Which of the following diseases should be suspected in this boy?
A . common variable immunodeficiency (CVID)
0, hyper IgM syndrome
C. Bruton agammaglobulinemia
D. selective IgA deficiency
E. transient hypogammaglobulinemia of infancy

- -
11. A 7 year old child with selective IgA deficiency develops car accident
requiring blood transfusion. The type of blood transfusion reaction that is MOST
likely encountered in this child is
A. acute hemolytic transfusion reaction
B. bacterial contamination
C. acute circulatory overload
D. acute anaphylactic reaction
E . delayed hemolytic transfusion reaction

12 Chi|dren with markedly susceptible to



X - Linked hyper -IgM syndrome are
Hemophilus influenzae
B . Streptococcus pneumoniae

230
*
c. Mycoplasma pneumoniae
DL staphylococcus aureus
£. Pneumocystis jiroveci

13 A 5-year-old girl
develops fever, leucopenia, and thrombocy ,
0pe
months after allogenic bone marrow
transplantation for recurrent Alt. 11 ,
f
CMV infection is positive . '
j

for this girl is



Of the following, the l line of therapy
A . acyclovir
B. ganciclovir
C. foscarnet
0 . oseftamivir
E. peramivir

14 . Which of the following is the treatment of choice in X - Linked hyper - j


syndrome ? ^
A . Monthly infusion of intravenous immunoglobulin IVIG
8. Granulocyte colony - stimulating factor
C. Trimethoprim prophylaxis
D. Acyclovir prophylaxis
E. HLA - identical hematopoietic stem cell transplant

15 . Which of the following viruses, is often fatal if acquired in children with X -

linked lymphoprotiferative disorder ?


A . Cytomegalovirus ( CMV )
B. Epstein- fiarr virus (EBV )
C . Coxsackievirus
D. Herpes simplex virus
E. Influenza virus

16 . A 9- month- oJd Infant develops disseminated


tuberculosis after BCG
vaccination He has previous attacks of diarrhea , pneumonia, and sepsts -
.
Complete blood count reveals low lymphocytes
count according to his age.
Radiological studies show very small sized thymus and absence
of lymph nodes.
Of the following, the MOST likely diagnosis is
A . severe combined immunodeficiency ( SCIDJ
B . Bruton agammaglobulinemia
C. common variable immunodeficiency fCVID)
231
IJ .
--
D. a
E-

tofV of
;
2
.
rnune polyendocrinopathy -candidiasis
hyp0gammaglobulmemia ot nfancy
'

nth-old boy develops wide spread


-ectoderma\ dysplasia

odes of lobar pneumonia caused byeczematous rash, He has a


ePldeSQf
sepsis due to
streptococcus pneumoniae

^LedTscat,tered° ^*
Hemophilus
\ ep s influenzae ,

^ cl nng
'° ne0natal period. Examination shows
. There is a
history of
e\e& ,
ur c rash. Lab tests reveal platelets eczematous rash as well

^
count of 20000/mm
evated IgM, low IgA and IgE, and normal IgG \ small
s P
atelet „® 1the treatment of choice for this boy level.
Of "’
the follow '
- ' latelets transfusion
A P
, . is

B
'
nropriate Pr Phv actic antlt 0t CS
iunoglobuim° replacement "'
C'
0- aggmissive management of eczema
tic stem cell transplantation (HSCT)

18. A 5'year *old


boy develops dilated tiny blood vessels involving his eyes, lips,
jpper neck, and both legs. He has a history of progressive difficulty in walking
and unsteady gait since the age of 2r dyear. He also has progressive delay
developmental milestone. Parents are 2 ' degree relatives. Examination reveals
spider -like blood vessels involving the sclera, mucous membrane of the mouth,
and the skin.
Of the following, the MOST frequently encountered infection is
A. chronic mucocutaneous candidiasis
B. recurrent sinopulmonary disease
C. Recurrent skin abscesses
D. disseminated atypical mycobacterial infection
E. recurrent meningitis


19. A 7- year-old girl develops staphylococcal skin abscesses, atopic
recurrent shedding primary
delay in
dermatitis, and pneumatoceles. He has a history of asVmmetrv pr
7;"
^
... . . ~
teeth and recurrent fractures. Examination shows facial Complete '|o
.
behead, wide-spaced eyes, and wide fleshy nasal tip
K A. IgA
» «I “
B. lgD
c IgE
232
D. IgG
E. IgM

- - . °
. Knu
o Y develops extensive cutaneous molluscum
20. An 8 year old
involving the whole mi a historY 0f frequent viral COfnacrKr
;
,
infectUs
CMV and EBV. He
medical history reveals ^ g
severe eczema
of jnfection with cryptosporid
since infancy
Of the following, the MOST likely diagnosis is
,
1Um
J
A . ataxia-telangiectasias
B. cartilage-Hair hypoplasia
C. Job syndrome
D. Wiskott-Aldrich syndrome
E . DOCK 8 deficiency

21. A 3-year-old girl develops extensive skin infection of the left leg. She has
a
history recurrent gingivitis and periodontitis with subsequent loss of
some
primary teeth. She has a history of delayed separation of umbilical cord
and
omphalitis during the neonatal period. Examination reveals ulcerative
skin
lesions with limited signs of inflammation and no pus formation.
Of the following, the MOST likely lab finding in this girl is
A. neutrophilia
B. neutropenia
C. monocytosis
D. eosinophilia
E. lymphopenia

22. Which of the following leukocyte


adhesion deficiency (LAD) syndromes is
associated with neutropenia rather than
neutrophilia ?
A. LAD-1
B. LAD- 2
C. LAD-3
D. E- selectin deficiency
E. Rac2 deficiency

23. Which of the following leukocyte


adhesion deficiency (LAD ) syndromes is
associated with bleeding tendency ?
A . LAD-1
B. LAD- 2
,
v'
LAD-3 deficiency
E.selectin
2 deficiency
£ RaC

3
-
*-*
. 4-year -old boy has recurrent attacks of inW
Jprrato V tract . He has light skin and silvery * involving
,
,
and nu ler 0 granular inclusions of all
*Of the*following
US
, the complication that is
overwhelm infections
ing
haT
WBC
MOST likely fatal
'
Lab

a in this
skin and
findings reveal

disease is
e bleeding
.

Q neuropathy
0. cardiomyopathy CMP )
(
E. hemophagocytic lymphohistiocytosis ( HLH )

25. The only curative therapy 0f Chediak - Higashi syndrome


hematopoietic stem cell transplantation (HSCT) The is allogenic
.
complication that is
mevitable even after HSCT is
A. recurrent infections
B. bleeding diathesis
C. neuropathy
D. hemophagocytic lymphohistiocytosis
£. growth retardation

26 A 7-year-old boy has persistent fever in spite of 5 days of


administration of
broad spectrum antibiotics. He has been underwent hematopoietic allogenic
bone marrow transplantation for AML IS days ago. Lab finding reveals
neutropenia. CT scan of the chest shows multiple nodules surrounded by a halo
of ground-glass attenuation.
Of the following, the MOST likely fungal infection is
A. Candida
B. Aspergillus
C. Mucor
0. Fusarium
E . Scedosporium

27 • A 5-year -
old boy develops fever, productive cou h. nd shortness of breath
consistent with pneumonia. He has been diagnos
^
jh chronlc granulomatous
(CGD) since infancy. You start empirical antibiotic .

234
Of the following, the
treatment is
A. chest x-ray
MOST useful investigation to follow the

B. CT scan of the chest


C. sputum culture
D . Erythrocyte sedimentation rate (ESR )
-
' X, .
)
E. C-reactive protein (CRP

28. A 5 -year -old boy develops jaundice


, hepatomegaly, right upper
quadrant
tenderness, progressive weight gain, and
ascites. He has been
underwent
hematopoietic bone marrow transplantation for neuroblastoma 20 days ago.
has been treated initially for neuroblastoma by chemotherapy and abdominal ^
irradiation.
Of the following, the MOST effective treatment for this condition is
A. ursodeoxycholic acid
B. heparin
C. phenobarbital
D. defibrotide
.
E vitamin K analogue

29. A 4- year - old boy develops high grade fever, productive cough, and shortness
of breath. He has been diagnosed with chronic granulomatous
disease ( CGD)
.
since infancy He has a history of recurrent pneumonias
since infancy . The
mother describes this illness as " unusually tough" .
Of the following, the offending organism that
should be considered and
empirically covered is
A. staphylococcus aureus
.
B serratia marcescens
C. borckolderia cepacia
D. Candida albicans
.
E aspergillus fumigatus

30. Transient neutropenia associated


viral infections is
in childhood. frequently encountered
Which of the following viruses is accompanied
by chronic
than acute one ? neutropenia rather
A. Influenza A
B. Cytomegalovirus

235
C- Adenovirus
herpesvirus 6
p Human
E. Measles
of «
the followingi nes
.
' s ass°ciated *' h minlmal rtA of infectifons?
jl- Which neutropenia
A.
Cyclic
neutropenia
"
8. Congenital
neutropeni
Autoimmunefollowing
.
C,
Neutropenia c dfugs
0.
f. Neutropenia secondary to ° '
*
Acute graft-versus-host disease (GVHDI 311 devel Ps 2-8
hJ !*' ’'
32.

Which of
6VHD?
the
,
transplantation manifested skin, Gl or
following is
as
the characteristic rm ^*** '
° wk after

cutaneous manifestation of acute

A. Maculopapular rash
B. Lichen planus
.
C Scleroderma
D, Hyperpigmentation
E. Hypopigmentation

33. The term " leukemoid reaction" is applied when WBC counts exceeding
50,000/pL with relatively small proportions of immature myeloid cells.
Which of the following cells constitutes the major component of these
immature myeloid cells?
A. Bands
B . Metamyelocytes
C. Myelocytes
D- Promyelocytes
E. Blasts

34. Which of the following conditions of neutrop is associated with almost


zero leukocyte alkaline phosphatase .
A. Chronic infections
B. Chronic blood loss
C. Vasculitis
D. Postsplenectomy states
E.
Chronic myelogenous leukemia
236
malignancy has been received ch
35. A 7-year- old boy with
is typically indicates P3 *
ago. The lab test finding that tien rer Py V !
myeloosuppressive therapy is
A. neutrophilia

B. lymphocytosis
C. monocytosis
D. eosinophilia
E. basophilia

36. What is the lab test finding that is MOST likely present iIn
infections?
A. neutrophilia
B. lymphocytosis
C. monocytosis
D. eosinophilia
E. basophilia

37. Which of the following


conditions is MOST likely
lymphocytosis? associated with
A. Addison disease
B. Polyarteritis nodosa
C. Dermatitis
D. Systemic mastocytosis
E. Asthma

38. A 7 year old girl


,
develops swelling of the
t0 V0 Ve the right hand after minor trauma,

ZILHTZ ^ *
orearm‘ There is no itching,
PreVi0US attack of redness, or urticaria. She
neck swelling after dental procedure that
rhte ?
oaternal
0St0
respiratorY support for few days.
orv o recurrent There is a positive
conditions. attacks of swelling after trauma or stress
Of the followi
condition is -
8 the MOST
likely complement
deficiency describing this
A. Cl inhibitor
B. factorB
C. properdin
D. MASP-1
E. C8b
,
i 2 Year .

*1.
f cosmet
. '
f the
-
. .
.old ado escent female with herMittary an8 oederriA .S
irc nasal surgerV -
'

following s drug of choice preonerat


0
histamines
Perat Ve v?
'
,
'
' going to

^Epinephrine
A -
Anti
6‘
Corticos teroids
£
Q, Ecallantide
E Eculizumab
autologous hematopoiet ,
JO
or
Allogeneic and nonmalignant disorH*
malignant ,
^ Cel1 have h
* ^^ to'**
t>0th following diseases is treated hyautologoUs er than allogeneic
c 0f the
ich
# rrow transplantation?
'
'
bone ^
A. Leukemia
lymphoma
B Relapsed
. syndrome
C. Myelodysplastic
major
D. 8-thalassemia immunodeficiency
£. Severe combined

238
Chapter 13
Immunology
Answers
SAMAA.A

2.( A)

type because of high a


' " ’ ( M at an increased rate soon after birth. ,

——
5.(C) Neonates begin to V »
about 6 davs after birth h concentration of IgM rises sharply. This
by approximateiy 1 yr of age. Sen .^
*

follows that of IgA .


:=e

6.(D) Flow cytometry is an important test to demonstrate the absence of


circulating B cells ;
«
point at about 3 -4 mo of postnatal life. The rate of development of IgE generally

which will distinguish X- linked agammaglobulinemia from


-
most types the hyper- IgM syndrome, and transient

typica v seen t diagnosis when infected, can be associated


,
W th p doZ ’
" aistr t>ution in CVID is almost equal, the age *
3

** 3

' and infections' may be less severe. Enterovirus


Wen,n«oenr» lhalitis
1

8 rare in
assT^
,
-
CVlD
9 (B) CV/O h

* 0 ,
ith ut nodu! r a
! °
yn,
Sbeen
Phoid
ated
K
with „
oftJPrUe -like enteropathy with
ntest,ne Other autoimmune
or

leases inchiri ' -


ynphoid
ac £' - 3 OP

lntJydna
ia hern
andPern,C 0Us
' Vtic
anem ' °an
/a
,
« m a

'
thromt >ocytopenia ,gastric

‘ '1
Ce WPphomas
lal Pneurn0ni ln
'
,^
, Voidosi
tinai

noncaseabnn
/roma
dise«e,pseudo /ymp of the
-
Ungs sp/epn Slt
' n
- *
and Ver a( CCUr
( )
saond S Sarcoid' lige granulomas
- There /$ an creased risk
° ^of lymphomas .
patients may be asymptomatic
or rmaY
especially Giardia ) ' develop sinopulm nary or
&
'^ 1 (GO
intestine
serum
infections (
antibodies to IgA are reported in a . °
five
“'f
se
ctions.
IgA deficiency These antibodies can m
Washed erythrocytes
^ ( frozen blood ' ^
would
Z
L^
individualsTho
,**,**
rans u>
'°"
^
products from other IgA - deficient e «W «M
D ood IgA deficiency. h 0uld be "
*f % tients with
. in addition to opportunistic infections such as P' J„roveci
administered

ic an increased incidence of extensive pneumonia,


-
»
there sporidium enteritis, subsequent liver verruca vulgaris lesions,
CrYPt0 disease, and an increased
risk of
malignancy-
0 ( 8) First -
line therapy is usually ganciclovir, with foscarnet
as an alternative
for resistant
strains or ganciclovir intolerance.
.
J4 (E)
lymphoproliferative diseases have susceptibility to EBV 3 H ,K
1S'(B) linked
development of hemophagocytic lymphohistiocytosis (HLH) There are 31
clinical phenotypes: (1) fulminant, often fatal, infectious mononucleosis (50%
cases); (2) lymphomas, predominantly involving B-lineage cells (25%l'- and iv31 ,’^
acquired hypogammaglobulinemia (25%).
.
16 (A) A combination of opportunistic infections and a persistently low
lymphocyte count is an indication to test for SCID.
syndrome which is an X -linked
.
17 (E) The most likely diagnosis is Wiskott -Aldrich
, thrombocytopenic
recessive disorder characterized by atopic dermatitis
defective platelets,
purpura with normal-appearing megakaryocytes but small
modality of treatment is HSCT.
and susceptibility to infection. The curative
characterized by progressive cerebellar ataxia,
18.(B) Ataxia- Telangiectasia is
telangiectasias , chronic sinopulmonary disease, a high
oculocutaneous immunodeficiency.
variable humoral and cellular
incidence of malignancy, and begin to walk and
soon after these children
Ataxia typically becomes evident usually by age 10 12 yr
The
a wheelchair ,
progresses until they are confined to
telangiectasias begin to develop at 3-6 yr
. The most
absence of IgA, w ic
immunologic abnormality is the selective m ec i
jn

8W& of these patients. Recurrent sinopulmonary


approximately 80% of these patients. syndrome b syn(jrome
) is
19 (C) Autosomal dominant hyper -lgE an elevated
- exceptionally high serum \ g ^^^ IgA, andlgM.
characterized by an
concen ra
Serum IgD concentration; usually normal

240
EBV and Cryptosporidia are
common. Yw
%
neutrophilic leukocytosis, often >250000/mmJ, ,
a (A) Significant adhesion
sa
feature of severe forms of
leukocyte deficiency (Ud)
delayed separation of the umbilical
children may have a history of
with associated infection of the cord stump . The presence
COrd
of Sl.“
^
>
^
omphalitis is an important feature that distinguishe s these rare patie
the 10% of healthy infants who can have cord separation at age 3 wk 0r
Infected areas characteristically have very little neutrophilic infiltration,
inflammation, and no pus formation.
22.(D)
23,( C) LAD- 3 is characterized by a GJanzmann thrombasthenia- like bleedi»
^ ng
disorder.
24.(E) The most life-threatening complication of Chediak-Higashi syndrome
rs
the development of an accelerated phase characterized by pancytopenia, high
fever, and lymphohistiocytic infiltration of liver, spleen, and lymph nodes. The
onset of hemophagocytic lymphohistiocytosis can occur at any age. This occurs
in 85% of patients and usually results in death.
25.{ C) HSCT corrects hematopoietic and immunologic function, the NK cell
deficiency, and prevent conversion to the accelerated phase, but cannot correct
or prevent the neuropathy .
26,{B) Disseminated candidiasis presents frequently as
a central venous
catheter associated infection. The most common presentation of invasive
-
aspergillosis is pulmonary aspergillosis. As a
nodule enlarges, the dense central
core of infarcted tissue may become surrounded
by edema or hemorrhage,
forming a hazy rim known as the halo
sign . When bone marrow function
recovers, the infarcted central core may
cavitate, creating the crescent sign.
27.(D) Management of infection in
CGD is dramatically different than in normal
children. CGD patients are always at risk
for deep-seated, indolent bacterial
infections, so determination of the appropriate
treatment can be difficult. The
erythrocyte sedimentation rate ( ESR )
can be quite helpful. If the child does not
have a deep-seated infection, the ESR
will be normal or will normalize within
several days with standard management.
Cultures should be obtained, but are
usually negative and not helpful, so
many support an "antibiotic sensitivity by
sedimentation rate response" approach to
treatment.

241
, Onset of venoocclusive <
* »4 5
,

* ^ , »
C * J
3

——
of
n afi ns, neUfob 0r hep3t(c

' ,, ursode
miii*
3
hemophagocytic
l
oxycho
JZonstrated some l i c „
'
,
°
°
/ymph

«"
v
- pJJ^?
acid and occaiinna
°m
*- °»
yt0sis

heParin ^ ,'
*
,

* eo
,
has ad t|on3//yr

°'' 4 *’ > y de

^
efficacy f pre t n and tre
porcine oiigode
nation of
^
2 ,
elude influenza
resP rat rv syncytia
' ° "°
enter V’iruses human
s A and B,

"*,
ad<fnovirus'rubella, and varicella rh '0 neutropeL herpesvirus 6,
measles, with Epstein-Barr virus cvtnJ
L a
8 virW,or HIV‘
0
often accompanies

**
infect infections tend to be less
^
in aotoimm e
31.10with the corresponding deem* neutroPen« frorr) C3USes pr ( "
than
because remainde r of the immune s
wtrophiis is greater than that
production. The
neutropenia accom
„ £
. °
°miH° W" t a *O^
C

,
"
°
'4

l J !'
tact an
ns result
> '
^
de very 0f

,
»» » Wilfrw "" ! *venous
B * *
t|y* *
“ Promised
" J* * ' .
secondary to r al
JZ °
, "° * win cancer
immunky and
hamer compromise lines muc 5it s therebyV
much ^
patients to a °
V ^° " ”
o
< T
Action f
predisposing
thJ ’

distractJ.
32.M Maculopapular rash is h ?
°" recognbeii
°' acute
. Alt
GVHD other
mi Unlike leukemia
CUtaneous
** "^ tationsofchmnkGv
leukemoid reactions show relatively small proportions
of immature myeloid cells, consisting largely of band forms, occasional
metamyelocytes, and progressively rarer myelocytes, promyelocytes, and
blasts. The presence of a left shift, defined as having >5% immature neutrophils
in the peripheral blood, is consistent with marrow stress.
34.(E) The leukocyte alkaline phosphatase score of circulating neutrophils
can
uniformly
differentiate chronic myelogenous leukemia, in which the level is
almost zero, from reactive or secondary neutrophilia, which features normal
to

elevated levels.
recovering from
15.(C) Typically, monocytosis occurs in patients
return of the
myeloosuppressive chemotherapy a harbinger of the
neutrophil count to normal. additional
16.(0) Hypereosinophilic syndrome a ic mastocytosis are
Systemic
important causes of an elevated eosin count.

242
afe endocrine
# » > disorders
**
« !
>
*A*«e®lW,pW^ ton>e %» m c'
'HVP rtantcause'' oi> ^eW V,AV" ^'*°* “'° '”,' "
C syi

UAdomi
10 1C
n ant
i5

‘JljjHef* " “ isnrrriaia


" dieaw
persons

°otte
a
onah
,
'
«ected
**,**,thoutseVe ^^
sVnthcs
et0
Part " gres' ”rap dW .
oan. **e * < the
'
'
ieeve

^ '.
^
\
funcW S
llrticaria

^
",
1C1
t
itching,
"
estinai wall, however, * ;
eS
«
or '.*< ,
^
ie**
red
0

-
^*JLas «°L
0
! ^
cra<nP -
\
nc mitant u utaneous
can
edema » ^
e abdoih „ C
* 0
' *' ha'e^urf ne abd! surgerv or wite
h
**
-
0

^ -^
srsr r
esta »sh®>d. Laryneeai edema
can *
daw* .
oma, especiaiiy *

-
aft
* *' aiSotia(ed

,W
i5
curreoi Poeu
’*in(ertio s-
|
° ^
39.(D) An inhibitor of kallikrein ( ecallantide ) that blocks bradykinin production
and an antagonist of the bradykinin receptor ( icatibant ) are approved in the
and

United States for use in adolescents and adults for long-term prophylaxis,
preparation for surgery or dental procedures, or treatment of acute attacks,
Eculizumab, a humanized monoclonal antibody to C5, prevents generation of
the membrane-attack complex C5b9 and is an effective treatment for
paroxysmal nocturnal hemoglobinuria and atypical hemolytic uremic syndrome.
Antihistamines, epinephrine, and corticosteroids have no effect.
.
40 (B) Autologous (from the same individual) transplantation employed as a
is
rescue strategy after delivering otherwise lethal doses of chemotherapy with or
without radiotherapy in children with hematologic
malignancies such as
relapsed lymphoma or selected solid
tumors (e.g., neuroblastoma, brain
tumors). Allogeneic ( from a donor) transplantation
genetic diseases of
is used to treat children with
blood cells, such as hemoglobinopathies, primary
immuno eficiency diseases,
various inherited metabolic diseases, and bone
syndmmes '*
Ure flematolog c
' malignancies (leukemia ), and myelodysplastic
'
V

Allergic Disorders
Questions
ALMUSAWI
Mowing signs is often seen in allergic children rubbing their nose
.
- o nalm of their hands ?
cluck
g Allergic saliJte
shiners
c Allergic morgan folds
D oennie-
linearity
E Hyper

2. prominent
infraorbital skin folds that extend in an arc from the inner canthu
lid margin is called
beneath and parallel to the lower
A. allergic shiners
8. chemosis
C. keratosis pilaris
D. Dennie lines
£. allergic salute

3. What is the MOST common skin abnormality of allergic children ?


A. Xerosis
B. Keratosis pilaris
C. Hyperlinearity
D. Atopic dermatitis
E. Urticaria
'

4. Which of the following conditions is characterized by peripheral eostnoph


.
A Sensitized patients after exposure to grass
Systemic corticosteroids use
C. Eosinophilic pneumonias
D. Eosinophilic esophagitis
E Drug reaction

244
1
.
associated with decreased risk
5. Which of the
following is of ,
asthn a?
A Delay introduction
of egg
diversity of the
. . microb
intestinal
. ..
ota duri
rin8
B. Reduced
early in childhood infancy
dogs in
C. Exposure to
D Bottle feeding
E. Delivery by cesarean section

when ordering epicutaneous


6 To avoid false-negative results skin tests u
patient to withhold montelukast f0r
should give instruction for the
A. 1day
B. 3 days
C. 5 days
D. 7 days
E. 14 days

7. Which of the following is a MAJOR risk factor for development of persistent


asthma in childhood?
A. Allergic rhinitis
B. >4% peripheral blood eosinophils
C. Food allergen sensitization
D. Eczema
E. Wheezing apart from colds

8 . Which of the following is characteristic to consider acute asthma attack as


moderate ?
A. Peak expiratory flow 50% of predicted
B. Pulse rate 150 beat /min
C. Loud Wheeze throughout
inhalation and exhalation
D. Respiratory rate >30breaths
/min
E. Confusion

coughing, throat tightness,


provisional diagnosis of asthma
anTaudiWeTr' !
treated ; muwheezin
f shortness of breath
° 8 and stridor with
'

^ '
asthma medications with poor response tlple d fferent classes of
Hi GP d Ct 0r '
clinic for reevaluation. Spirometry was
done ° referred him to asthma
! •
^
inconsistent inspiratory and expiratory reveals truncated and
flow- voii oops while flexible
245
f
scopy reveals paradoxical vocal cord movements w th
' anatomically
rf^ voca' «**,he MOST likely diagnosis is
A' ,
fiEBO 3,acia.
hronct>om dysfunction
.
vocal cord
C' iminersensitivity pneumonitis
P
£. sSe
persistent asthma

Whic h of the following lung function parameters is MOST consistent with


10.
xthnia ?
A FEV1 /FVC
ratio <0.60
>12% after exercise challenge
B Worsening in FEV1
C . Daily peak expiratory flow (PEF) AM -to-PM variation il 5%
0. Increase in FEV1>12% after bronchodilator administration
[ High FEV1 ( relative to percentage of predicted norms )
,

H which of the following statements regarding FeNO ( Fractional Exhaled Nitric


Oxide ) is TRUE ?
A. It is an invasive measure of allergic airways inflammation
8. It is a biomarker of allergic eosinophilic airways Inflammation
C Children as young as 3 years can perform this test
D. Cannot predict future asthma exacerbations
E, It is a significant biomarker of cystic fibrosis

12. A 6-year -old asthmatic patient on medium dose budesonide as MDI since 4
weeks with no improvement in his condition.
Of the following, the NEXT step in his management is to
A. add monteleukast
B. reevaluate the inhaler device technique
C. continue same treatment and reevaluate after 2 months
0. increase budesonide dose
E , consider other diagnoses

13. Which of the following is the main component of asthma action plan ?
A. Describing regular asthma medication use
B Standardize assessment of asthma control
C . Understanding of the pathogenesis of asthma

246
Monitoring lung function
0. conditions
E Determine comorbid
criteria in early childhood is a risk
14. Which of the following factor
of persistent asthma ?
development
A. Sibling asthma
B. High birth
weight
C. Female gender
0. Breastfeeding
E. Severe pneumonia

15 . Which of the following is the most


common chronic symptom of asthma?
A. Nasal congestion
B. Non focal chest pain
.
C Expiratory wheezing
D. Productive coughing
E. Exercise induced wheeze

16 . Which of the following is TRUE regarding the use of salmetrol in the


treatment of asthma ?
A. Used as monotherapy
B. Combined with monteleukast
.
C Combined with inhaled corticosteroid
D . Used as reliever therapy
E. Onset of effect 5-10 min after
administration
17. A 7 - year-old asthnr\ atic bo
cou£h, shortness of breath

^
with inability to sleep last night e ^ n ^ '^ ^
W

.
hour with no improvement in svmntnm^
,
O the <o lowing. the M0ST app
A . continue same treatment
B. use short course oral ^ aUn **
inhaler 3 times over one
PEF <80% Personal.
next 5teP is to

steroids
C . add montelukast
D. add Inhaled ipratropium
E . use intravenous aminophylline

18. What is the hallmark of atopic


dermatitis?
A . Severe dry skin
247
f
aris
%
Kef tosis '
era
3
P
'
£
0
£
HjJ scalV erythematous lesion
following sites is mainly involved in infantile atopic dermatitis?

:I Napkin
r
area
sho^lder
p. Hands
Scalp
£
with irritability, erythematous scaly lesion over
,0. A onth -boy presents allergy in her infant.
gives a history of egg
cheeks* <he mother
following is the first line therapy ?
Which o f the
limus cream 1%
^B. Pimecro
Tar preparation
Q Diphenhydramine
0 . Moisturing cream
E. Topical corticosteroid

atopic
21- Which of the following vitamins deficiency is associated with severe
dermatitis ?
A. E
B. C
C. B 1
D. B 12
E. D

^A is the best tolerated sport for a boy with atopic dermatitis ?


Football
B.
Swimmiing
Running
D-
E, ^ seball
Basketball

248
1
develops small , punctate pruritic wheal
23. A 10-year
-boy
flare after taking a hot shower,' onCe theStJr otJndea
prominent erythematousin 30 -60 min.
down the rash
subsides
the MOST likely diagnosis is
Pat
^^ *

Of the following
,

A. cholinergic urticaria
B. pressure urticaria
syndrome
C. contact urticarial
0. acute papular urticaria
E. dermatographism
, yellow- tan to reddish brown macula on
24. A 6-year-old boy develops small
scratching his arms, what is the name of this
sign?
A. Nikolsky
B. Darier
C. Candle
D. Auspitz
E. Buttonhole

25. Which of the following antibodies are found in patients with chronic
urticaria more than general population ?
A. Antihepatitis A
B. Antidouble strand
C . Antithyroid
0. Antinuclear
E. Antitransglutaminase

26. What is the main stay of treatment of chronic


urticaria ?
A. Steroid
B. HI antihistamine
C. Monoclonal antibody
D. Montelukast
E. Plasmapheresis

.
27 An 8 year -old- well developed
girl presented to outpatient clinic w h «
s ortness of breath and
increasing progressive swallowing difficulty, there is no
history of cough or flue like
illness, just a visit to dentist for tooth extraction
before one hour .
Of the following, the MOST likely
cause of her condition is
249
'f

hereditatyangioedern
croup
A spasmodic
B.
C
epiglotitis
covid19
0 . hemangiorng
Weil
,"-vcar
l

0 -boy develops a sudden attack of vomiting, diarrhea, and


th after attending a birthday party when he ate a
cake
shortness
fye 3
. containing some
butter
pi$ t * chio , the MOST appropriate first line measure is
e following
oft
*
A ORS solution
g. diphenhydramine
Q gentamycin
Q IM adrenaline
l ranitidine

iZZSX
least
A. 2 hour
'
^^
be kept forobse at on before being discharged for a .
B. 4 hour
C. 8 hour

.
~ -
D. 12 hour
E. 24 hour

«
can include
,» M dn « *
A. bradycardia
B hypotension
c- headache
0. flushing
E. dry
skin

sickness is a classic example of


^ Wpe I hypersensitivity reaction
type II hypersensitivity reaction
^D- type
type III hypersensitivity reaction
IV hypersensitivity reaction

250
, .'
neAse0a$e
t auX
°'^
»
e et
»*0-
, .* ! *.'>
*\>
^
a
^
tooS

1
-
Ae

^
a dva ®3
«.
e

«
. ne6

•!
%
"!
%
f and >te hroat

A
' '*
**
ec e P nVc\ttvo, aHe
*
Led
* * *
"' ^^•
*
3 35
*
*
, dtefP*
N
4 44

^ * *
0

^> uo
^
w P e
ov. ^ * " ***** 'W
and' 3
"* ^
t
rtmb0 - .. theN
* *** ^*
*

d3ftno
C

^
*.
X'C****
C
*
vc
'
#
* **
33. Adverse food reaction can be divided into food intolerance and food allergy
which of the following is an example of food intolerance ?
A . Food protein- induced enterocolitis
B. Enteropathy syndromes
C. Galactosemia
D. Oral allergy syndrome
E. Allergic eosinophilic esophagitis

34. A 7- month- old baby boy typically


manifest as irritability, intermittent
vomiting that occur 1- 4 hour after feeding,
with bloody diarrhea , anemia, and
failure to thrive, the baby is bottle fed
with complementary feeding.
Investigations show anemia and hypoalbuminemi
a with normal serum IgE.
Of the following, the MOST likely diagnosis
is
A. food protein - induced
enterocolitis syndrome
B. proctocolitis
C. enteropathy
D. eosinophilic gasteroenteropathies
E. celiac disease

35. A 2-month- baby girl presented


by her anxious
blood-streak in her daughter stool, she mother when she found
is exclusively
looking baby, her investigations are
normal apart from
breast fed healthy well
Of the following, the MOST likely diagnosis is mild anemia .
A. gastroenteritis

251
r

prot ein-induced allergic proctocolitis


B- food
nterOpa Y ^
C' osinophil c gasteroenteropathies
'
^^ f
0- - induced enterocolitis syndrome
d protein
E*

of the following vaccines is contraindicated iln


hich children with egg
36.
yellow fever
B. Measles
C. influenza
0. Rabies
E. MMR
family with a history of food allergy seeks your advice to
educated
37. An expected daughter.
prevent allergy in their
Your proper advic
e is to
during pregnancy
A. avoid allergenic foods
to prevent allergic diseases
B. use soy -based formulas
C delay introduction of potentially
highly allergenic foods
D. exclusively breast feed for first 8 mont
hs
) after 4- 6 mo
E. introduce solid (complementary foods

ions can be divid ed into predictable and unpredictable


38. Adverse drug react
reactions.
drug reactions?
Which of the following is an example of predictable
A. Drug toxicity
B- Allergic (hypersensitivity ) reactions
C. Pseudoallergic reactions
D- Idiosyncratic reactions
E. Dose independent reactions
by drug specific T
When the adverse drug reactions are
mediated

^ ,rnmune
ymPhocytes, this interaction is classified as
complex reactions
Delayed- type hypersensitivity reactions
Cytotoxic antibody reactions
D- Immediate hypersensitivity reactions
^ Serum sickness
252
, maculopapular rash, facjai
boy develops fever
40. A 6'year old
-
Investigations reveal increment a ,-
generalized lymphadenopathy
.
in f

SGOT with eosinophilia


months ago
What is the
*

MOST
i likely
. He had history of

diagnosis ?
epilepsy

^
treated with valp

A . Kawasaki disease
g Hodgkins lymphoma
.

C . DRESS syndrome
0 . Brucellosis
E . Typhoid fever
apil 14
' Allergic Disorders
Answers
ALMUSAWI

because of nasal pruritus and rhinorrhea. children with allergic rhiw,


1-(B) allergic salute by rubbing
often perform the
This repeated maneuver may
their nose upward with
give
"
the pd of
3nd . rise to the nasal crease
the f h
|
over the bridge of the , a
I wrinkle nose.
Allergic shiners (blue -gray to purple discolorations beneath their
2 (0) child' s
eyelids , which can be attributed to venous stasis found in
lower up to 60% of
allergic patients and almost 40% of patients without allergic disease ) Dennie-
Morgan folds ( Dennie lines) are a feature of atopic dermatitis.
3.(A ) Xerosis or dry skin , is the most common skin abnormality of allergic
(

children.
4.( A ) In certain pathologic conditions, such as drug reactions, eosinophilic
pneumonias, and eosinophilic esophagitis, significantly increased numbers of
eosinophils may be present in the target organ in the absence of peripheral
blood eosinophilia .
5 ( C) There is also a decreased risk of asthma, AR , and atopic sensitization with
early introduction to wheat, rye, oats, barley, fish, and eggs.
6 IA ) Montelukast should be withheld for 1 day, most sedating antihistamine
preparations for 3-4 days, and nonsedating antihistamines for 5- 7 days
7-(D) Early childhood
risk factors for persistent asthma have been identified and
have been described as major ( parent asthma, eczema, inhalant allergen
sen$itiiation)and
minor ( allergic rhinitis, wheezing apart from colds, £4 %
Peripheral blood
3 TA )
eosinophils, food allergen sensitization) risk factors.
Approx . 40-69% or
response lasts < 2 hr .
Vocal cord dysfunction ( VCD ) can manifest as intermittent daytime
wheezjng-
The vocal cords involuntarily close inappropriately during inspiration
and SOrnet
rnes exhalation, producing shortness of breath, coughing, throat
tiPh^ ' cases o
VrnneSS * ' anc
,audible laryngeal wheezing and /or stridor . In most
;"
i«S0
Pir0metric un function testing reveals truncated and mconsisten
8
0ny
ant exP ratory flow - volume loops, a pattern that i ers ro
reDrort *
Producible pattern Qf a|rflQW | jmitatjon in asthma that improves w,th
'
254
V
,rn can c e , Hvpercarb«
and tet
w
0 wndWaW^' ° *'.
n s/CO. f e \e » 00
w
'" ' \ &\ cord
erehVDO,
' *' '
fomdO
'
ran revea parad % ca , 0
' J w° e d'aonos^.parents withMCQ
' movement H
D

* J *
iZk
10.(0}
&ZZZ * *
'
nt classes of

!uTlm relative
( to percentage of predicted norms)
ratio
•FEV1/FVC <0.80
Bronchodilator response (to inhaled (3-agonist ) assesses reversibility of airflow
limitation.
Reversibility is determined by an increase in either FEV1 >12% or predicted
FEV1>10% after inhalation of a short-acting P- agonist (SABA)
Exercise challenge:
•Worsening in FEV1£15%
Daily peak expiratory flow (PEF) or FEV1 monitoring: day-to-day and/or AM -
to-PM variation >20%*
1MBJ
12 (B)
13.(A) Two main components:
(1) a daily "routine' management plan
describing regular asthma medication
use and other measures to keep asthma
under good control and
(2) an action plan to manage ^
worsening asthma, describing indicators of
impending exacerbations, identifying
what medications to take, and specifying
2
^ ° *° **
W
me alc e Phvsician and/or obtain urgent/emergency
14 (E)
Early Childhood Risk Factors for
Persistent Asthma
Parental asthma *
Allergy: (Atopic dermatitis (
.
Severe lower respiratory tract
eczema) ,
allergen sensitization* Food allergen * Allergic rhinitk Food allergy, Inhalant
sensitization) '
infection: (pnpt
hospitalization) monia, Bronchiolitis require
Wheezing apart from colds
Male gender
Low birthweight
tflV r
cental tobacco smoke exposure
'
RedU ced lung function at birth
|a feeding rather
than breastfeeding

feSJET'"
chro
a
«•«common
16 ( 1
^«)
17.1
18.(AI
19.IA)
20.(0) Moisturizers are first-line therapy for treatment of atopic dermatitis
21.(E) Vitamin D deficiency often accompanies severe atopic dermatitis.
22.(8) A sport such as swimming may be better tolerated than others
that
involve intense perspiration, physical contact, or heavy clothing and equipme
nt
Rinsing off chlorine immediately and lubricating the
skin after swimming are
important.
23 (A)
24.(B)
25.(C) The incidence of elevated thyroid antibodies in patients
with chronic
urticaria is approximately 12%, compared with 3-6% in the general populat
ion.
Although some patients show clinical reduction of the urticaria with thyroid
replacement therapy, others do not. The role of thyroid autoantibodies
in
chronic urticaria is uncertain; their presence may reflect a tendency of the
patient to develop autoantibodies, but they may not play a direct role in chronic
urticaria .
26.(B)
27.(A) Dental work with the injection of procaine hydrochloride ( Novocain) into
the gums is a common precipitant for laryngeal edema in patient with
hereditary angioedema.
28.(D) Epinephrine is the most important medication, and there should be no
delay in its administration in case of anaphylaxis. Epinephrine should be given
bY the IM route to the lateral thigh ( 1 : 1000 dilution, 0.01 mg/kg; maximum 0.5
i

29.(8) More than 90% of biphasic responses occur within 4 hr, so patients
should be observed for at least 4 hr before being discharged from the
emergency department .
3<MC)
3Mc)

256
generally begin 7 -12 days after injection
32. sickne ss f th
( E ) Serum
materialfdrugs or protein s from other species). ° %
33.( C )
34.(A)
35.(B)
36.(A)
37.( E ) Prevention of
Food Allergy
-Breastfeed exclusiv
ely for 4-6 mo.
-Introduce solid (
complementary) foods after 4 -6 mo of exclusive breastf
entary foods 1 at a time . eedinf
-Introduce low risk
- complem
-Introduce potentially highly
allergenic foods (fish, eggs, peanut, milk
or delay ).
soon after the lower-risk foods (no need to avoid
-Do not avoid allergenic foods during pregnancy
or nursing.
-Soy- based formulas do not prevent allergic disease .

38.( A ) Predictable drug reactions , including drug toxicity, drug interactions,


and adverse effects, are dose dependent, can be related to known
pharmacologic actions of the drug, and occur in patients without any unique
susceptibility. Unpredictable drug reactions are dose independent, often are not
related to the pharmacologic actions of the drug, and occur in patients who are
genetically predisposed. These include idiosyncratic reactions, allergic
( hypersensitivity ) reactions, and pseudoallergic reactions.
39 ( B )
40.{C)

257
15
S^ RjJjumatic Diseases of Childhood
Questions
0$3&BE SAW

If a rthralgia is accompanied with a history of dry skin, hair loss, fatigue,


1- ii * j
, or cold intolerance
* , testing for which of the following
gro disturbance is
mefited?
A. Infection
B. Thyroid disease
C. Malignancy
D. Orthopedic conditions
E. Pain syndromes

2. An evanescent macular rash associated with fever is part of the diagnostic


criteria for which of the following?
A. Henoch-Schonlein purpura
B . Juvenile dermatomyositis
C. Systemic-onset arthritis
D. Systemic lupus erythematosus
E. Mixed connective tissue disease

3. Which of the following antibody tests has a high specificity for SLE; associated
with lupus nephritis?
A . Double- stranded DNA ( dsDNA )
B. Antinuclear antibody ( ANA )
C. Ribonuclease protein (RNP )
D. Antineutrophil cytoplasmic antibodies ( ANCAs )
E. Smith ( Sm )

A. How much is the sensitivity of rheumatoid factor ( RF ) as a diagnostic tool in


children with Juvenile idiopathic arthritis?
A . <10%
.
B <20%
C. <30%
!

D. <40%
E. <50%
adverse effect of NSAIDs in children?
5 . What is the
MOST frequent
A. Gastritis
8. Nausea
change
C. Mood
D. Irritability
E. Anemia
may cause aseptic meningitis when used
6. Which of the
following NSAID ^
treat patients with lupus
?
A. Naproxen
B. Celecoxib
C. Ibuprofen
D. Indomethacin
E. Meloxicam

7 . Which of the following is given as an adjunct to minimize


adverse effects at
methotrexate?
A . Zinc
B. Vitamin B12
C. Vitamin C
0. Folic acid
E . Vitamin D

8. What is the MOST significant potential


adverse effect of Hydroxychloroquine
A. Skin discoloration
B. Retinal toxicity
C. Gastric irritation
D. Bone marrow suppression
E. Myositis

9. What is the intravenous corticosteroid


of choice
manifestations of systemic rheumatic diseases ? to treat severe, acute
A . Hydrocortisone
B. Methylprednisolone
C . Dexamethasone
259
p Betamethasone
E. Mometasone
long arthritis must be present to make a diagnosis
10. How subtype? of any Juvenile
idiopatbic arthritis
A 22 weeks
g 24 weeks
C. 26 weeks
D. 28 weeks
E. 212 weeks

11. A 4- year- old girl presents with limitation in range of motion, tenderness, and
warm swollen right knee joint with left ankle joint for the last 8 weeks. She also
complains from easy fatigability and poor sleep quality. ANA is positive .
Of the following, the MOST appropriate measure to be done is
A. periodic slit-lamp examination
8. genetic study
C. rheumatoid factor study
D. echo study
E. MRI of involved joints

12. Which of the following Juvenile idiopathic arthritis JIA subtype is associated
with a worse prognosis?
A. Persistent oligoarticular JIA
B. Extended oligoarticular JIA
C . Enthesitis related arthritis
D - Polyarticular JIA
E . Systemic JIA

, spiking fever . with


13 . An 18- month- old boy presents with arthritis
temperatures > 39“C for the last 3 weeks .
evening and is frequently accompanied by a
macular rash distributed over, the trunk end
proximal extremities. On
examination, the baby has hepatosplenomegalv .
lymphadenopathy, and
pericarditis documented by echo .
Of the following, the MOST likely diagnosis is
A . oligoarticular JIA
B . rheumatic fever
260
*.

C. serum sickness
D. systemic JIA
E. Kawasaki disease
diagnosed as systemic juvenile M
-old child
14. An 18-month °
P3ttl
treatment, today
with pres
is in remission
one month ago and
lymphadenopathy , hepatosplenomegaly witl>^
spiking fever,
encephalopathy . Laboratory evaluation shows thrombocytopen ,aUrpUra
' nc
*
with elevated liver enzymes, lactate deh ° Penia
hypofibrinogenemia
and
ferritin, and triglycerides. A bone marrow aspiration shows •
/
na$
e of
0

hemophagocytosis.
Of the following, the MOST likely diagnosis is
A. immune-mediated thrombocytopenia
B. macrophage activation syndrome
C. autoimmune hepatitis
D . familial hypertriglyceridemia
E . visceral leishmaniasis

15. Which of the following laboratory features is useful in distinguishing


macrophage activation syndrome ( MAS) from a flare of systemic disease ?
A. Cytopenias
B. Abnormal liver function tests
C. Coagulopathy
D. Decreased erythrocyte
sedimentation rate
E. Hypertriglyceridemia

16. Which of the following viral


diseases iis a recognized cause of arthritis?
A. Measles
B. Mumps
C. Chickenpox
D. Roseola infantum
E. Hepatitis A

17. If a patient with oligoarticular


JIA shows
injections, what is the next step no response and
treatment ? to NSAIDs
A. Etanercept
B. Abatacept
C. Methotrexate

261
1
0. Rituximab
e Anakinra

•ssssErr ssr&ssz? -r
s srr- -
JSES
^
Of the following, the MOST likely diagnosis is
A. juvenile ankylosing spondylitis
B. juvenile psoriatic arthritis
C. inflammatory bowel disease
D. reactive arthritis
E. persistent oligoarticular JIA

19. What is the gold standard imaging for early visualization of


sacroiliitis ?
A. MRI with gadolinium
B. Short-Tl inversion recovery ( STIR ) MRI
C . Conventional radiographs
D. CT scan
E . Bone scan

20. Which of the following if used initially and continually may slow the
progression of structural damage spondyloarthritis ( SpA ) ?
A. Methotrexate
B. Etanercept
C. Naproxen
D . Rituximab
E. Anakinra

21. A 5- year-old boy presents with acute onset of severe pain in the right groin,
had history of an upper
referred to the thigh for the last week, the patient
cell count are
respiratory tract infection 2 weeks ago. ESR and white blood
widening of the joint space;
normal, while ultrasound examination confirm
a piration of joint fluid was normal and results in dramatic clinical improvement
*
Of the following, the MOST likely diagnosis is
A. toxic synovitis
B. poststreptococcal arthritis
0. septic arthritis
262
D reactive arthritis
tic fevef
c acute rheuma

- Old bov
22. A S-year the face , extre Laboratory investigations reveals E$R
nodules over
localized positive C-reactivetendteej
pro n
and and leukocytes/mm3 with
te $ t WBC 20000
muscular pain

50 mm/hr, 3
neutrophils/ mm diagnosis is
75% likely
following, the MOST
Of the
A. Sweet syndrome et
acuta
varioliformis
B. pityriasis lichenoides
fibromyalgia syndrome
C. juvenile primary
granulomatosis with polyangiitis
D.
- syndrome
E. Churg Strauss
ior criterion for relapsing
major
following is a suggested
23 . Which of the
polychondritis?
A . Eye inflammation
B . Hearing loss
C. Vestibular dysfunction
D. Seronegative inflammatory
arthritis
nose cartilage
E. Typical inflammatory episodes of

24 . ANA is very sensitive for SLE ( 95-99%), but its


specificity is
A. 10%
B. 30%
C. 50%
D. 70%
E . 90%

25. Which of the following is MORE common in drug- induced lupus compared
with SLE ?
A. High percent of positive dsDNA antibodies
B. Hypocomplementemia
C. Significant renal disease
D. Hepatitis
E . Significant neurologic disease
he
following medications has definite association with drug -
h ?*
Whiclupus
of

;nduAcedIsoniazid
g
phenytoin
C.
Carbamazepine
n Nitrofurantoin
\ ^blockers
the followin g tests correlate well with SLE disease activity ?
Which of
A. ANA titers
B. Anti-dsDNA
antibody
C. Anti-Smith
protein (CRP)
0. c-reactive
E. Antihistone
antibodies

for the treatment of lupus


28. Which of the following is MOST appropriate
nephritis?
A. Hydroxychloroquine
B. Cyclophosphamide
C. Corticosteroids
D. Methotrexate
E. Rituximab

29. A 5-week -old infant presents with a characteristic macular rash affecting the
periorbital area, trunk, and scalp. The neonate also has thrombocytopenia,
leukopenia, and elevated liver enzymes; ECG and echo reveals complete heart
block, while maternal serum demonstrates anti- Ro and anti La antibodies.
-

Of the following, the MOST appropriate management of this infant is


A. IVIG
B.
corticosteroids
C. cardiac pacing
0. cardiac transplantation
E. hydroxychloroquine
30 - A 7-year -
old girl presents with generalized erythema in sun- exposed areas
°sver the chest, neck, knees and elbows with a heliotrope rash of the eyelids that
associated with periorbital edema . The girl also has fever, dysphagia, arthritis,
V
, with
fatigue muscle weakness and tenderness difficulty iln climhi
108
combing hair.
likely diagnosis is
**
of the following the MOST
A. juvenile polymyositis
B. juvenile dermatomyositis
C. SLE
D . juvenile psoriatic arthritis
E . enthesitis related arthritis

31. Which of the following is usually elevated on initial presentation of jUVe


nile
dermatomyositis ?
A. CK
B. ESR
C. Rheumatoid factor
D. ALT
E. CRP

32 . A 10- year- old child diagnosed as juvenile dermatomyositis for the last 2
years with poor compliance and under treatment, develops painful ulceration of
the skin of right lower calf area near ankle joint with extrusion of crystals.
Of the following, the MOST likely cause is
A . cellulitis
B. osteomyelitis
C. lipodystrophy
D. calcinosis
E. trophic ulcer

iol;rr?yrers
3 6 with indurati n f the
oulders, face, trunk, and extending to ° ° ^
the arms. The child looks well with no **
constitutional symptoms apart from a
febrile streptococcal pharyngitis 10 days
Of the following, the MOST likely diagnosis
is
A . pseudoscleroderma
B. juvenile systemic sclerosis
C. subcutaneous morphea
D. scleredema
E. En coup de sabre

265
gave history
of edema and ervthBm, < , ,.
vear -old boy rated ,
g.
° 6 exte'nding from
wer limb,
hypopigmented atrophic lesions °
*"° Resents
w
ie thigh
with indu
through knee joint to above ankle resulting in knee
contracture
-

skin breakdown of scar on the lateral knee .


"Tchronic
follo wing, the MOST likely diagnosis is
w
° 'A linea.r scleroderma
generalized morphea
B
Q subcutaneous morphea
0 . En coup
de sabre
E eosinophilic
fasciitis

frequent initial symptom in pediatric systemic sclerosis ?


35. What is the MOST
A . Dyspepsia
B . Raynaud phenomenon
C. Dyspnea at rest
D. Dysphagia
E. Proximal muscle weakness

painful oral ulcers


36. A 12-year - old boy presents with recurrent , multiple,
ut scarring. He also
ranging from 2- 10 mm, which last 3- 10 days and heal witho
bia, together with
has blurred vision, redness, periorbital pain, and photopho
, folliculitis, purpura, and
papulopustular acneiform lesions, erythema nodosum
ulcers.
h ing signi fican tly impro ves oral ulcers and skin features of this
Whic of the follow
disease?
A. Cyclophosphamide
B. Colchicine
C. Corticosteroids
D. Azathioprine
E. Sucralfate

37. A 10- year-old girl presents with recurrent paro


tid
mouth, halitosis, widespread dental canes,
dry V • ,
serologjc3 studies
and arthralgia . CBC shows leukopenia and
tact
^
, elevated serum
amylase,
reveal high- titer ANA, positive rheumatoid
anti- (3 - fodrin aut
hypergam mag lobu linem ia , and posit ive
Of the following, the MOST likely diagnosis is
A . juvenile recurrent parotitis
266
B. Sjogren syndrome
C. HIV
D. polycystic parotid disease
E. sarcoidosis

38. A 6-year-old boy presents with intermittent unilateral


lasting few days associated with fever. Parotid biopsy showsParotid
a
lymphocytic infiltrates. *
Of the following, the MOST likely diagnosis is
A. juvenile recurrent parotitis
B. Sjogren syndrome
C. eating disorders
D. mumps
E. trauma to the buccal mucosa

39. A 13-year -old Syrian boy presents with recurrent, short -lived (1- 3 days), self-
limited episodes of fever, unilateral pleuritic chest pain, severe generalized
abdominal pain, arthritis of knee, and an erysipeloid erythematous rash that
overlies the dorsum of the foot. Other clinical findings include scrotal pain
exertional leg pain. Between flares, the boy is generally symptom free but has
persistent elevation of ESR.
Of the following, the MOST appropriate treatment for this boy is
A. rilonacept
B. anakinra
C. colchicine
D. corticosteroids
E . azathioprine

fS WO*
40 . Which of the following is a diagnostic Indicator of Tumor Necrosis
Receptor- Associated Periodic Syndrome ( TRAPS) ?
A . Recurrent episodes of inflammatory symptoms spanning >6 mo duration
B . Episodes last >10 days on average
C. Responsive to colchicine
D . Affects family members in autosomal recessive pattern
E. More in Caucasians
osteomyel 1
41. A 5 - year old child presents with recurrent fevers, sterile
-

congenital dyserythropoietic anemia ( CDA ), neutrophilic dermatosis


, failure ^
267
hepatomegaly. The parents are cousins and genetic study reveals
3 d 2 gene.
jn the LPIN
likely diagnosis is
ing, the MOST
f
0f »he ° acrophage activation syndrome
a Majeed syndrome
b of interleukin- 36 receptor antagonist
c Hcficiencv of adenosine deaminase 2
deficiency
syndrome
E . Blau

year- old child presents with recurring episodes of fever , malaise,


42. A ^ -apPear,n6 tonsillitis, cervical lymphadenopathy, oral aphthae,
exudative pain, and arthralgia. The episodes last 4 - 6 days and often
headache , abdomi nal
3 - 6 weeks cycles . Laboratory findings reveal mild
leukocytosis,
recur every
throat cultures.
elevated acute- phase reactants, and negative response in this child ?
Which of the following drugs shows dramatic
A. Azathioprine
B. Cimetidine
C. Anakinra
D. Prednisone
E. Colchicine

amyloid A ( AA ) amyloidosis ?
43. Which of the following is MOSTLY affected by
A. Testes
B. Adrenals
C. Heart
0 . Skin
E . Kidneys

44 . What is the MOST common chest radiograph finding in sarcoido


sis ?
A . Parenchymal infiltrates
B . Bilateral hilar adenopathy
C. Miliary nodules
D- Pleural effusion
Multiple pneumatoceles
arthritis involving
45 . A 3 -year -old child presents with symmetric polyamcula
rge snd small joints
hlst
followed by photoph obia
the form of red-brown to purple
p
#
^
°^ of skin rash starting in infancy in
maculopapular lesions <1 cm on
the face, upper back, and
eosinophilia, and high ESR while
X fen
%s.
,
reveals anemia , leukopenia,
serolooS;tauCfif
and hypercalcemia. '
shows hypergammaglobulinemia
likely diagnosis is
Of the following, the MOST
A. juvenile idiopathic
arthritis
B. earfy-onset sarcoidosis
C Wegener granulomatosis
D . chronic berylliosis
E . chronic granulomatous disease

46. Which of the following is a predictor of poor outcome in Kawasaki disease ?


A . Female gender
B. Neutropenia
C . Young age
D . Thrombocytosis
E . Hypernatremia

47 . Which of the following clinical features is consistent with Kawasaki disease?


A. Exudative conjunctivitis
B. Erythema and cracking of lips
C. Generalized lymphadenopathy
D. Splenomegaly
E. Vesicular rash

48. Patients with acute Kawasaki disease should be treated with IVIG within
how many days of disease onset ?
A. 5
B. 7
C. 10
D . 14
E. 21

^
on lower xtremities 'extensoraspecTofVhe
with edema of hands and feet , periorbital area
ofle
PUrPUra and
' /
^
, 1
extremit es ar d buttoC

5

^
SCrotunr1, be a s0
right knee and left ankle swellings with bouts '
*
melena . CBC is normal with mild elevation of ESR an < j CRp a om * na Pa n an
6
^^
Of the following, the MOST appropriate treatment now
^ '
is
269
4.
ibuprofen
B- corticosteroids
immune eiohl
„.
bu ln
c intrav enous
D.
azathioprine
£. cyclosporine
mont h - old well-looking child presents with fever;
50. An 18 -
’ tender edema of
, and feet ; with large ecchymotic
... face, hands
.the .
g the trunk CBC and urinaly rec
sis .iJ^ the face and
extremitie s sparin
, the MOST likely diagnosis is
n rmal'
°
Of the following
A. Churgstrauss syndrome
B. hypocomplementemic urticarial vasculitis
C. hypersensitivity vasculitis
D. infantile acute hemorrhagic edema
E. Henoch Schonlein purpura
, headache ,
51. A 15 -year - old boy presents with fever, malaise, weight loss
reveals
myalgia , arthralgia, dizziness, and abdominal pain; examination
extremities, blood
decreased peripheral radial pulse with claudication of
90. Magnetic resonance
pressure in right arm 160/ 105 while in left arm 140/
the aorta .
angiography reveals angiographic abnormalities of
Of the following, the MOST likely diagnosis is
A . Takayasu arteritis
B. temporal arteritis
C. fibromuscular dysplasia
D. coarctation of aorta
E. polyarteritis nodosa
abdominal pain,
S2 A 10- year -old boy presents with weight*. loss
u spvere
i«c , <

- u|cer5 tender
an ^
hematuria, arthralgia, myalgia with superficiaspin hemorrhages He gave a
and 3 months ago
. His
subcutaneous nodules, digital necrosis, for ew
history of transient loss of consciousness recj blood cell

blood pressure is 140/ 100; urinalysis shows


p *
rena
[° '
aneurYSms.
casts . Abdominal aortogram shows bilateral
boy -
What is the mainstay of therapy for this
A. Oral cyclophosphamide
B. Plasma exchange
C. Methotrexate
270
4 .
i

D. Oral prednisone
E . IVIG

53. A 14-year-old girl presents with fever, malaise, weight


cough, wheezing, dyspnea, and hemoptysis. Examination arth
raW
purpura, nasal ulceration, epistaxis, saddle nose, and hear ^ ' S
108 loss
°
ophthalmic examination reveals conjunctivitis, scleritis and
OMU UV 0|Jjt lj
9 %
‘ M r
pressure 140/100, urinalysis shows proteinuria with hematuria *
Of the following, the MOST likely diagnosis is
A . Henoch- Schonlein purpura
B . granulomatosis with polyangiitis
C. Churg- Strauss syndrome
D. microscopic polyangiitis
E. Goodpasture syndrome

54. Which of the following differentiate Churg-Strauss syndrome from


granulomatosis with polyangiitis ?
A. Small vessel vasculitis
B. Circulating antineutrophil cytoplasmic antibodies
C. Granulomatous inflammation
D. Necrotizing vasculitis
E. Eosinophilia

affecting the anterior


,
and bi ateral cramping pain,
mostJy in ate afternoon or evening.
*
Pain may wake the child from sleep
resolves quickly with massage A
following morning. Physical finding
^
"
W
8 feVV m nutes t0 hours, bi/f
'
Y note pafn s never present the
'
and gait is not impaired.
S S ls
A. growing pain
B. restless leg syndrome
C. idiopathic musculoskeletal pain syndrome
svndro
D. fibromyalgia
E. complex regional pain syndrome

S6. A 15 -year- old boy comes to you complaining from


sensations, worsening with periods of rest and at night
uncomfortable painful M
and is relieved by
movement. His mother gives history of same complaint during adolescence
272
, the following options may benefit this boy ?
* *A* Reassurance
W1
hygiene
Health sleep
'

5 during the episode


C
,
D - r0
n supplementation
agents
E NSAIDs
-

272
Chapter 15
Rheumatic Diseases of ChildhOOcj
Answers
ZUHAIR ALMUSAWI
l.( B) Arthralgias are common in childhood and are a frequent
referral to pediatric rheumatologists. Arthralgias without physical
arthritis suggest infection, malignancy, orthopedic conditions
hT*fcr

syndromes, or pain syndromes such as fibromyalgia . Although beni


diseases may manifest as arthralgias, arthritis is a stronger predicto atic
rheum
r of the
presence of rheumatic disease and a reason for referral to a
rheumatologist. Pediatric
2.(C)
.
3 (A)
4.( A) Rheumatoid factor ( RF) is present in <10% of children with JJA
and thus
has poor sensitivity as a diagnostic tool; RF may be elevated by infections
such
as endocarditis, tuberculosis, syphilis, and viruses (parvovirus 819, hepatitis
B
and C, mycoplasma), as well as primary biliary cirrhosis and malignancies. In a
child with chronic arthritis, RF serves as a prognostic
indicator.
5.( B) The most frequent adverse effects of
NSAIDs in children are nausea,
decreased appetite, and abdominal pain.
Gastritis or ulceration occurs less
frequently in children. Less common
adverse effects (<5% of children
undergoing long-term NSAIO therapy),
include mood change, concentration
difficulty that can simulate attention
deficit disorder, sleepiness, irritability,
headache, tinnitus, alopecia, anemia,
elevated liver enzyme values, proteinuria,
and hematuria.
.
6 (C) Aseptic meningitis has been
patients with lupus. Naproxen is
,
associated with ibuprofen, primarily in

^^ ° *^ '
unique skin reaction called pseudop NSA S
7.(0)
.
8 {B) The most significant potential
adverse effect is retinal toxicity which
occurs rarely but results in irreversible color
blindness or loss of central v sion.
9 (B) Methylprednisolone, 10- 30 mg/kg/dose up
toTJ° ieiven
over 1 hr daily for 1- 5 days, is the IV Preparation of
lo.(C)
choicT
e'

273
.“" 71 , » »« > » anterioi
*" ySisT wJ*"*! tel “*
^ s

Ss
ir
is s.
"is
, £=..
6 mo changes
the classification 0 extended oligoarticular
. JIA
with a worse prognosis Isolated involvement of the hip is almost
associated ting sign and suggests enthesitis related arthritis ERA or a
never a presen .
nonrheumatic causerash is
.
13 (D) The classic
nonpruritic and migratory with lesions lasting <1 hr.
Koebner phenomenon, a cutaneous hypersensitivity in which classic lesions are
brought on by superficial trauma, is often present.
14.(B) Macrophage activation syndrome (MAS) is a rare but potentially fatal
complication of sJIA that can occur at any time (onset, medication change,
active or remission) during the disease course. It is also referred to as secondary
hemophagocytic syndrome or hemophagocytic lymphohistiocytosis (HLH),
.
15 (D) The erythrocyte sedimentation rate ( ESR ) falls because of
hypofibrinogenemia and hepatic dysfunction, a feature useful in distinguishing
MAS from a flare of systemic disease.
.
16 (B) Viral illness causing arthritis or extremity pain
(parvovirus, rubella,

mumps, Epstein -Barr, hepatitis B, chikungunya )


ritis show no response to
.
17 (C) A substantial fraction of patients with oligoarth
t with disease modifying
NSAIDs and injections, and therefore require treatmen
), including methotr exate , and, if no response,
antirheumatic drugs (DMARDs
if they have arthritis and pOTMSB or
Children have psoriatic arthritis
^
Jollow
^
arthritis and at least 2 of the of psoriasis
onycholysis, and (3) psoriasis in a
aids in diagnosis but is not required. sacroili itis is evidence of bone
of
.
19 (B) The gold standard for early visualization fluid sensitive sequences sucb
MRI wiith to the
marrow edema adjacent to the joint on not add value
as short-Tl inversion recovery (STIR). vea | abnormalities
before
is used. MRI win
study of the sacroiliac joints if STIR initially
frequently used
desmophyte formation
and may slow the progression
of structural damage
continually.
and growth) if used
274
21.(A)
Diagnos tic Criteria for Classic Sweet Syndrome
22.(A)
Major Criteria
onset of painful erythematous plaques or nodules
Abrupt
evidence of dense neutrophilic infiltrate with0ut
Histopathologic
leukocytodastic vasculitis
Minor Criteria
Pyrexia >38"C
hematologic or visceral malignancy, inflam
Association with unae y e
disease or pregnancy, or preceaeu *
infection or vaccination
an upper respiratory or gastrointe
%,a .
or potassium iodide
Excellent response to systemic corticosteroids
)
Abnormal laboratory values at presentation (3 of 4 :
Erythrocyte sedimentation rate > 20 mm/hr
Positive C-reactive protein test result
>8,000 leukocytes/mm3
>70% neutrophils/mm3
The diagnosis is established by the presence of 2 major criteria plus 2 of the 4
minor criteria .
23.( E) Suggested criteria for relapsing polychondritis
MAJOR
Typical inflammatory episodes of ear cartilage
V Typical inflammatory episodes of nose cartilage
V Typical inflammatory episodes of laryngotracheal cartilage
MINOR
^ Eye inflammation ( conjunctivitis, keratitis, episcleritis, uveitis)
V Hearing loss
Vestibular dysfunction
Seronegative inflammatory arthritis
Th
The diagnosis is established by the ore« 2 major or 1 major and 2 minor
criteria. Histologic examination
presentation is atypjca|
J afferTJ^
affected cartilage is required when the
24.(C) Of note, although a positive antinucle,
negS i*" * > * *
®0 ( ANA « result is not
required for the diagnosis of SIE, AMA-
, „Tt Z ^ " rare. ANA is
< '
extre V
( -
very sensitive for SIE 95 99% , but i
)
50%)- The ANA msy
e positive many years before a diagnosis of
, 5 6 However, most
ANA' P Sitive P3t ems
° ^ *’" '’other
not have su 0r * autoimmune
eTse
^
di
'
275
lementemia, and significant renal or neurol
,
an lbodies
*dlsease >°
in bypocomP associated with drug-induced lupus
dsDtfdAssociatio
icationS"
procainamide, hydralazine, isoniazid,
penicillamin
*0nCVcline’ ethyldopa, chlorpromazme , etanercept, diltiazem,
infliximab, adalimumab.
* Association , carbamazepine, sulfasalazine,
hle
0
tfP 01 gthosuximide amiodarone * quinidine,
enyt *
nitrofurantoin, p-blockers, lithium, captopril,
P
” P' interferon-y,
i
^
v''
°•a'
valproate
CyC|
,
C

^ rothiazide,
eriseofulvin
&
glyburide, docetaxel, penicillin,
, gemfibrozil, propylthiouracil.

ophosphamide,
tetracycline, statins, gold,

MMF, and azathioprine are appropriate for the


!8 |
' tfrient luPus
nephritis, whereas MMF and rituximab are often used for
hematologic manifestations, including severe leukopenia, hemolytic
gmficant
, or thrombocytopenia.
inemiaSignificant conduction abnormalities after birth are treated with
system
29 ( C)
and corticosteroids, whereas severe
cardiac pacing and occasionally IVIG defect is
cardiomyopathy may require cardiac transplantation, if the conduction
intolerance,
addressed, affected children are at risk for exercise
arrhythmias, and death With cardiac pacing, children
,
with conduction system
prognosis.
disease in the absence of cardiomyopathy have an excellent
during disease course
30.(8) Clinical features of juvenile dermatomyositis
%
FEATURE
/ Muscle weakness 90-100
/ 13-40
Dysphagia or dysphonia 10
/
Muscle atrophy 30-75
^ Muscle pain and tenderness 85 - 100
/
Skin lesions 66-95
^ Heliotrope rash of eyelids 57 - 95
Gottron papules 42 -100
Erythematous rash of malar / facial area 80-90
^ Periungual ( nail fold) capillary changes 5-42
^
/ Photosensitive rash 22-30
v derations 12-30
^Icinosis
^ E
11-14
'podystrophy
276
f

Raynaud phenomenon 2-15


/ Arthritis and arthralgia 22 - SS
/ Joint contractures 26-27
/ Fever 16- 65
/ Gastrointestinal signs and symptoms 8-37
Restrictive pulmonary disease 4 32
/ interstitial lung disease 1- 7
/ Cardiac involvement 0-3
-derived enzymes (creatine kilna
31,(D) Elevated serum levels of muscle tcxi
aldolase, aspartate transaminase , alanine transaminase [ALT ), *
.
dehydrogenase) reflect muscle inflammation Not all enzyme levels rise
inflammation in a specific individual; ALT is usually elevated on intiy
presentation, whereas CK level may be normal. The erythrocyte sedimentat
^
**
rate (ESR ) is often normal, and the rheumatoid factor (RF) test result istyptcaih,
negative.
32.(0) Lipodystrophy and calcinosis are thought to be associated with kwg
standing or undertreated disease. Dystrophic deposition of calcium phosphate
hydroxyapatite, or fluoroapatite crystals occurs in subcutaneous plaques ew
nodules, resulting in painful ulceration of the skin with extrusion of crystals or
calcific liquid .
33.(0) Scleredema is a transient, self-limited disease of both children and adults
that has sudden onset after a febrile illness ( especially streptococcal infections].
34.( A) Linear Scleroderma
Linear lesions can extend through the dermis, subcutaneous tissue, and imis
to underlying bone; more likely unilateral. *
35.(B) Raynaud phenomenon ( RP) is the most frequent initial symptom in
pediatric systemic scierosis, present in 70% of
affected children months to ye
5
before other manifestations. RP refers to the *
''
o
classic triphasic sequence
blanching, cyanosis, and erythema of the digits
induced by cold exposure and/o
.
36|B) in patients without major organ
involvement, colchicine significantly
improves oral and genital ulcers, skin
features, and disease activity of
disease.
.
37 (8) Proposed Criteria for Pediatric Sjogren Syndrome
Diagnosis requires > 4 criteria
l CLINICAL SYMPTOMS
.
1 Oral; recurrent parotitis or enlargement of parotid
gland, dry
( xerostomia)
277
2.

;,
P3
.II IMMUNOIOGIC

0
SSSSlSSEr'
mucosal: recurrent vaginitis
systemic: fever, noninflammatory arthra|gias, hvpoka]emic
lhdOtntnal
ABNORMALITIES
— without obvious

paralysis, and

presence of at least 1 of the following antibodies : anti -SSA, anti-SSB,


antibody, rheumatoid factor high - titer
ntinuclear
h OTHER ABNORMALITIES OR INVESTIGATIONS
: elevated serum amylase
1 Biochemical
leukopenia , high erythrocyte sedimentation
2 . Hematologic: rate
3. immunologic : polyclonal hyperimmunoglobulinemia
4. Renal: renal tubular acidosis
5. Histologic proof of lymphocytic infiltration of salivary glands or other organs
(i.e„ liver )
6. Objective documentation of ocular dryness ( rose bengal staining or Schirmer
test )
7. Positive findings of parotid gland scintigraphy
IV. Exclusion of all other autoimmune diseases
.
38 ( A) Juvenile recurrent parotitis is characterized by intermittent unilateral
parotid swelling typically lasting only a few days. It is frequently associated with
fever and may undergo remission with puberty. Unlike in Sjogren syndrome,
there is a male predominance, juvenile recurrent parotitis is seen in the younger
children ( 3- 6 yr ), and there is a lack of focal lymphocytic infiltrates on biopsy.
.
39 (C) Prophylactic daily oral colchicine decreases the frequency, duration, and
intensity of FMF flares. This regimen also prevents the development of systemic
M amyloidosis.
« ( A)
Diagnostic Indicators of Tumor Necrosis Factor Receptor- Associated Periodic
Syndrome (TRAPS)
6 mo duration
1- Recurrent episodes of inflammatory symptoms spanning >
( several symptoms generally occur simultaneously )
a. Fever
b. Abdominal pain
c . Myalgia ( migratory)
myalgia )
d. Rash (erythematous macular rash occurs with
e . Conjunctivitis or periorbital edema
f. Chest pain
278
g

in aUtOS ma d°minant Pattefn alt ugh m


always be present
)
° ' ^ **°
Any ethnicity
may be affected
5.
Aphthous Stomatitis, Pharyngitis, and Aden, , ,
X periodic Fever,
^
SI ? SIFSS
me interval
?'doesVorprevent
Cimetidine
between flares.
recurrence

preventing recurrences in approximately


shown that anakinra may be effective
and may act

^
at 20- 40 mg/kg/day is effective
one third of cases. Small series ha
during a flare, but because corticosteroids
S„

are effective, this may not be a cost-effective
approach. Colchicine may enteric
the time between flares. Complete resolution has
been reported after
tonsillectomy, although medical management should be the first approach,
43.( E ) Although organ involvement may vary, AA amyloidosis most frequently
affects the kidneys; 90% of patients have some degree of renal involvement
Unexplained proteinuria may be the presenting feature in some patients
Nephrotic syndrome and renal failure may develop if the underlying
inflammatory condition is not controlled or if diagnosis is delayed.
44.(B) In adults and older children, pulmonary involvement is most frequent,
with infiltration of the thoracic lymph nodes and lung parenchyma . Isolated
bilateral hilar adenopathy on chest radiograph is the
most common finding.
45.(B) There appear to be 2 distinct, age dependent
patterns of disease among
children with sarcoidosis. The clinical
features in older children are simitar to
featured ^ with frequent systemic
lymph

^ vTntntS
<4 yr of age is characterized by
46.(0 several risk stratification
which patients with Kawasaki
thetLTrash
model .
disease (KDl
ha t
,
manifeSting
UWeWS and po yarthrit S
'been '
constructed to
,^ ,
determine
,
abnormalities CAA . Predictors of poor
ou
^ at hi8 flest for coronary artery
young age, male gender, persistent ^ tc 0 fr| e across several studies include
fever response to IVIG, and laboratory
abnormalities, including neutrophilia
hyponatremia , hypoalbuminemia,
elevated ^mb cytopenia, transaminiti
° ^
natriuretic protein and elevated C- of N'termj'nal'brain
reactive Protein
Pacific Islander race and Hispanic ethnicity (CRp) levels . Asian and
are also ri
nsk factors
for CAA .
279
rn„

=
features that are not

=5
Clinical
47.1») ctivitis
with KD include exudative
conju
0

m1C )
All
,"
ral esionS •=£
Patients with acute
usually administere

and ideally
d over 10- 12 *
be treated with 2 g/kg of IVIG
KD should
hr
infusion, as soon as possible after diagnosis.
lymphadenopathy, discrete
splenomegaly, and bullous,

as a single
within 10 days of disease onset ,

such as oral prednisone (1- 2 mg/kg/day), or in


Glucocorticoids severe
49,(B) (IV ) methylpredni solone for 1- 2 wk , followed by taper ,
cases, intravenousand joint pain but do not alter overall prognosis of Henoch
reduce abdominal
Schonlein purpura . age, the nature of the lesions, absence of other organ
SO.(D) The younger
distinguish infantile acute hemorrhagic
involvement, and a biopsy may help
purpura .
edema from Henoch Schonlein
51.(A)
) or IV pulse methylprednisolone 30
(
52.{D) Oral prednisone ( 1- 2 mg / kg / day
mainstay of therapy of polyarteritis nodosa . Oral or IV
mg/kg/day) are the
as adjunctive therapy, and plasma exchange
cyclophosphamide are often used
threatening disease .
may be warranted for life-
53.(B) causes inflammation of the upper
frequently
54.(E) Churg- Strauss syndrome CSS is rare . CSS may
initially
cartilage destruction
and lower respiratory tracts, but sinusitis , nasal polyposis, non- fixed
demonstrate chronic or recurrent
rhinitis/ Eosinophiiia (>10% of
- to-treat asthma
.
pulmonary lesions and difficult may precede a vascuhtic phase.
leukocytes) with pulmonary infiltrates termed benign norturna iw
ly
55.(A) Growing pains, more appropriate % of children, wit pe
^

childhood , growing pains affect 10-20 ^ Qf


^
occur dur ngf physical
between age 4 and 12 yr. Pain does not
a day wit
' e or other
at growth sites . Pain often follows not impa ^
normal , and gait is seen more
activities. Physical findings are that
^^ ^ disturbanc e
"
(RLS nsohrnotor
56.(0) Restless legs syndrome *
, is a - to- control
and adults a difficult
frequently among adolescents farm nig t and is
relieved
pains . Often at ^
may be confused with growing with RLS.
urge to move the leg that is exacerbamay
by movement . Iron supplementation
duringf
^
ted bene i ediatric patients

280
Chapter 16
Infectious Diseases
Questions
USAMA A. ALJUMAILY
1. A 5 - year-old boy with leukemia develops fever and chills; you
infection with gram- negative bacilli.
Of the following, the best lab media that support growth of such organisms JS
A. sheep blood agar
8 , chocolate agar
C . Sabouraud dextrose agar
0 Mactonkey agar
,

E Inhibitory mold agar


,

2. A 6-year - old boy develops fever and decrease level of consciousness; hel»
been diagnosed with posterior fossa low grade glioma since the age of lye
that was treated successfully by VP shunt insertion and tumor resection. He a
*
fully vaccinated. You suspect shunt infection.
Of the following, the MOST common organism isolated from shunt infection is
A, Haemophilus influenzae type b
-
B Streptococcus pneumoniae
C. Proptonibacterium
D. Mycobacterium tuberculosis
E . Cryptococcus

3. Parasitic infections are usually detected by microscopic examination


clinical specimens like blood, bone marrow, or fecal smears.
Of the following, the parasite that is detected by serologic tests rather tten
clinical specimens is
A. Plasmodium
B. Trichinella
C. Babesia
D. Leishmania
£ Giardia lamblia

281
i£E2 5Kss
of the
saar *
following, the micro- organism that i S Strongly

he has been

related with disease


severity is
aeruginosa
A pseudomonas
B. Haemophilus influenzae
C. Staphylococcus aureus
D. Mycoplasma pneumoniae
E. Mycobacterium tuberculosis

5. Which of the following is the MOST effective measure to reduce the


likelihood of recurrent Clostridium difficile-associated diarrhea (CDAD) ?
A. Metronidazole
B. Fecal transplantation
C. Vancomycin
D. Avoid using antibiotics
E. Amoxicillin - clavulanate

6. A 5-year- old boy is going to receive immunoglobulin against hepatitis A


before travelling with his parents to a country endemic with hepatitis A. The
mother is concerned about adverse reactions of immunoglobulin
administration.
Of the following, the MOST common adverse reaction to immunoglobulin
administration is
A . flushing
B . headache
C. chills
D . pain at the injection site
E. nausea

has
7. A 4- year - old boy is exposed to his relative with varicella infection . He
since infancy.
been diagnosed with primary immune deficiency disorder
Varicella- zoster immunoglobulin is not available.
is
Of the following, the BEST alternative treatment
)
A . intravenous immunoglobulin (IGIV
( IGIM)
B . intramuscular immunoglobulin
(IGSC)
C. subcutaneous immunoglobulin
D. palivizumab
282
E . prophylactic acyclovir

8. Which of the following vaccines consists of whole j


Inactiv
microorganisms ?
A . Acellular pertussis
B. Hepatitis A
C. Hepatitis B
D. Varicella
E. Rotavirus

9. A 7- year -old boy developed AIDS due to HIV infection . Lab test shows that
hiS
CD4 * T-lymphocytes count is 250 cells/ mm **
Of the following, the live attenuated vaccine that can be given to this child is
A . oral polio
B. small pox
C. yellow fever
D. BCG
E. varicella

10. Children with primary immunodeficiency may not receive vaccines, whether
live-attenuated or killed ones. In which of the following diseases, all vaccines
can be administered?
A . Chronic granulomatous disease
B. DiGeorge syndrome
C. Properdin deficiency
0. Severe combined immunodeficiency (
SCID )
E. Common variable immunodeficiency

11* 5- year -old girl is going to receive


^
diagnosed with immune thrombocytopenia
her scheduled BCG vaccine . She has been
3 weeks ago and has been treated
with prednisolone 2 mg/kg day,
/ since that time. What is your recommendation
regarding vaccine administration ?
A. She can receive her
vaccine immediately
B. She should not receive
vaccine any more
C . Stop corticosteroids for
2 weeks before resuming her vaccination
D. Stop corticosteroids
for at least 1 month before resuming
vaccination
E . She
can receive her .
Varrc ne
discontinued ' Irnmediate y once c 1
|

° icostefoids are
l2 A
preterm baby is born to a HBsAg-
?
1 .
*
your proper management for this baby
A. give birth dose of hepatitis B
vaccine
B. defer hepatitis B vaccine until chr
^ bother . u,„:
birth weight
is 1.9

C. give hepatitis B immunoglobulin (


HBIG ,
onological
8 f 1 month

.. . _
D. give hepatitis B vaccine within 12
Ec. g ve hepatitis D
hr of birth
.
B immunoglobulin
hr of
withjn
birth
(HBIG ) and
anH u
*
r of birth

hepatitis B vacci
ne within 12

13. Which of the following vaccines


should be deferred for at least 3 months
after immunoglobulin administration
?
A. Pertussis
B. Measles
C. Diphtheria
D. Rotavirus
E. Live attenuated influenza
vaccine ( LAIV )

14. Which of the following is the MOST common


route of nosocomial infection
during hospitalization?
A. Hands
8. Medical equipment
C. Toys
D. Hospital furnishings
8. Phones

15. A 5-year- old boy is admitted to ICU because of


cei v eTi n tensi v e
been diagnosed with acute myeloblastic leu
chemotherapy with subsequent febrile neutropenia .
of antibiotics since 3 weeks. .
^
prolonged course

Of the following, the MOST common type of infectio


A. rota virus
B. respiratory viruses
C. fungi
D. staphylococci
E. gram - negative bacilli
284
16 . Which of the following
pathogens requires droplet ,
isolation ra h
contact isolation ? * »hsn
A.Enterohemorrhagic Escherichia coli 0157:H 7
B. Hepatitis A virus
C. Shigella species
D. Neisseria meningitidis
E. Clostridium difficile

17. A 6- year-old boy is going to receive hepatitis vaccine before travelling to


A
hepatitis A- endemic country. The parents ask about the time needed for th
vaccine to provide adequate protection for safe travel.
*
Protective immunity after receiving hepatitis A vaccine will develop within
A . 1 week
B. 2 weeks
C . 4 weeks
D . 8 weeks
E . 12 weeks

18. Which of the following is the MOST common pathogen that may cause
traveler ' s diarrhea ?
A. Enterotoxigenic Escherichia coli
B. Enteroaggregative Escherichia coli
C. Campylobacter
D . Entamoeba histolytica
E . Shigella

19 . A 7- month- old infant develops diarrhea during a


travel with his parents to a
country in the Southeast Asia.
Of the following, the MOST appropriate
antibiotic for this infant is
A . amoxicillin
B . trimethoprim- sulfamethoxazole
C . ciprofloxacin
D . azithromycin
E . erythromycin

20
Aw
^:r i*
B . Azithromycin
uowi
"e a'"e> ^ ^
hotericin B
r

P'
A
"* ,n
l TeicoPlan
following disorders is characterized by periodic fever ?
, Which of the
2
» Cyclic neutropenia
nean fever
B. Familial Mediterra
-associated periodic syndrome
TNF receptor
P Hyper
-lgD syndrome
- Wells syndrome
E Muckle

the following conditions is MOST likely associated with relative


22 Which of
tachycardia rather than bradycardia ?
A. Brucellosis
B. Diphtheria
C. Chlamydia pneumonia
D. Legionnaire s disease
E. Leptospirosis

23 . Which of the following conditions is classically associated with


double
quotidian fever (fever that peaks twice in 24 hr ) ?
A . Familial Mediterranean fever
B. Cyclic neutropenia
C. Behcet disease
D . Inflammatory arthritis
E. Crohn' s disease

24. Which of the following CSF parameters is not affected by prior antibiotic
administration in bacterial meningitis ?
A . PCR
B. Culture
C Cell
-

D - Glucose
E Protein

25 . A 3- year -old boy drooling of the mouth, difficulty in


develops fever ,
swallowing , noisy breathing, and neck pain . Examination is unremarkable
.

0f t|n|e following,
the MOST appropriate diagnostic study
is

286
A. lumbar puncture
0. MRI of the brain
C. MRl of the spinal cord
D. CT scan of the neck
E. lateral X-ray of the head and neck

26. An 18 -month-old girl develops prolonged fever of 3 weeks durat On It h


been preceded by a flue like illness * The mother describes that the fever
' *
been subsided after 5 days followed by recurrent spikes every 2 - 4 days Other
medical history is insignificant . Examination is unremarkable.
Of the following, the MOST likely diagnosis is
A , nephrogenic diabetes insipidus
B . familial dysautonomia
C. pseudo- FUO
D familial Mediterranean fever ( FMF )
E. hyper igD syndrome

27 Which of the following conditions is accompanied by fever in the absence of


sweating ?
A . Hodgkin lymphoma
B Tuberculosis
.

C . Hyperthyroidism
0 Hypoglycemia
,

E . Familial dysautonomia

28. Which of the following is the MOST likely


diagnosis fn a 1- year old fflste-

child with prolonged fever associated with


bulbar conjunctivitis ?
A . Measles
B . Lymphogranuloma venereum
C. Kawasaki disease
D . Coxsackievirus infection
E . Tuberculosis

29 . Fever, lack of tears , absent corneal


of fungiform papillae is highly suggestive
A Familial dysautonomia
.
J whlch of,mthe°°thfoltonSue
S
|
* ' th abse
owing diseases ?
0 Ectodermal dysplasia
C Kawasaki disease

2S7
P
Dermatomyositis
£ scleroderma
boy with chronic granulomatous H
-old
30- A 3 -vear tachycardia consistent with. ?*
ease <CGD) develops fever,
,cchi„lls, toxicity, and MOST .
of
is
,
A.
he following, the

Staphylococcus aureus
^
common mlcro-ureanism
on causing sepsis in this child

B. Aspergillus
C. Chromobacterium violaceum
D. Streptococcus pneumoniae
E. Candida albicans

31. In which of the following primary immunodeficiency diseases, Pneumocystis


jiroveci pneumonia ( PCP) infection is more likely to occur ?
A . X-tinked agammaglobulinemia ( XLA )
B. Common variable immunodeficiency
C. Selective IgA deficiency
D. igG 2 subclass deficiency
E. Hyper IgM syndrome

oral mucositis; he has been

intensive chemotherapy 1 week before 3


P
His absolute neutrophil count ( ANC) is 100/mm . to this
shou|d be given
Of the following, the MOST appropriate antibiol
boy is
A . cefepime
B. vancomycin
c- Piperacillin
D. ceftriaxone
E. amphotericin
mucositis 1 week after
and oral
33 . A 12- year -old boy with AMLdevelops fever
. He has been kept on
high dose cytarabine chemotherapy administration
chemotherapy . His ANC is
of
ciprofloxacin prophylaxis after completion
300/mm3. in this boy is
d pathogen
Of the following, the MOST commonly encountere
2 &S
A. Candida albicans
B. Mucor mycosis
C . Aspergillus fumigatus
D. Staphylococcus aureus
E . Streptococcus viridans

34. Which of the following is the MOST commonly used intravenous 3


st
as 1 line monotherapy in children with fever and neutropenia ?
ntibtotr
A . Ceftazidime
B. Cefepime
C. Vancomycin
D . Meropenem
E. Gentamicin

35 . A 9- year- old boy develops lymphoma few years after liver transplantation
Which of the following viral infections that may lead to such complication ?
A . Cytomegalovirus ( CMVJ
B . Varicella zoster virus ( VZV )
-

C . Hepatitis B virus ( HBV )


D. Hepatitis C virus (HCV )
E . Epstein Barr virus (EBVJ
-

36 . A 5 year-old boy has greenish discharge


at the exit site of central venous Ime
-

( CVC ) .
Of the following, the MOST likely
offending causative micro- organism is
A . Staphylococcus aureus
B . Coagulase - negative staphylococci
C Streptococcus pneumoniae
D . Candida albicans
E . Mycobacteria

37. A 7- year - old boy has persistent


candidemra after removal of central l ne
venous catheter . He is on daily liposomal
amphotericin B for the last 2 weeks -'
Of the following, the MOST appropriate man3
gement in this setting
A . stop antifungal treatment is to
B. add itraconazole
C. add fluconazole
D . continue amphotericin for 6 weeks
289
by fluconazole
(
place amphotericin
r0
id boy develops fever . Examination
H 3 Vear 3
- '
° reveals ill child With
history is negative upon parental
, nation
tfnes*- VaCC’ the MOST common
5 ° ,,
refusal,
causative organism is
neck

the

J
- H ae Ph
^
USinf UenZaetVPeb
E5chericWa coh
C. Streptococcus
0. Neisseria
pneumoniae
meningitidis

E . Staphylococcus aureus

39 A 5- year- old boy develops fever . Blood culture reveals Streptococcus


pneumoniae pathogens.
Of the following, the MOST appropriate antibiotic effective against this
pathogen is
A. ampicillin
B. ampicillin- sulbactam
C. amoxicillin-clavulanate
0. ticarcillin- clavulanic acid
E. vancomycin

40. Which of the following cephalosporins is highly effective against


pseudomonas aeruginosa ?
A- Cefazolin
Cefuroxiime
C- Ceftriaxone
0. Cefotaxime
Ceftazidime
41. A of the right
year- old girl develops erythema, swe mg
hand. ^
)
Swab culture reveals methicillin-resistant stap ccus aureus (MRSA .
against this pathogen
^is the following, the cephalosporin that is MO

A. cefepime
B. ceftazidime
C. ceftaroiine
D. cefazolin
E.
cefuroxime
290
42 . Which of the following
pathogens is resistant to meropenem?
A. Streptococcus pneumoniae
B. Pseudomonas aeruginosa
C. Stenotrophomonas maltophilia
D. Klebsiella
E. Acinetobacter
is particularly
43 . Which of the following aminoglycosides effects6 tyiV
'
Francisella tularensis infection?
A . Streptomycin
B. Kanamycin
C. Gentamicin
D. Tobramycin
E. Netilmicin

44. Which of the following is the MOST effective antibiotic against extend
spectrum beta lactamase ( ESBL ) organisms ?
A . Meropenem
B . Ceftazidime
C. Amikacin
D . Colistin
E . Ertapenem

45. Staphylococcus aureus pathogens can affect the host by direct invasion or
through toxin elaboration . Which of the following diseases is caused by direct
invasion of the staphylococcus aureus rather than by toxin- mediated process ^
A. Food poisoning
B. Pericarditis
C. Scarlet fever
D . Scalded skin syndrome
E. Toxic shock syndrome

46. Skin lesions caused by S. aureus may be indistinguishable from those «us*<
by group A streptococci .
Which of the following manifestations is MORE specificfor the former ?
A. slow expansion
B. less erythema
C. more tenderness
291
more swelling
b'
less fluctuation

,, which of the following organisms may mimics S


pneumonia?
* aureus infecti,on
javitary causing
streptococcus
A group A
"

B Mycoplasma pneumoniae
C. streptococcus pneumoniae
0. Klebsiella pneumoniae
E. pseudomonas aeruginosa

XZZ2SSEZS tzzssT' ,
01 he the
Of the following, the MOST appropriate initial
antibiotic treatment is
A. ampicillin
8. cefazolin
C. amoxicillin
D. azithromycin
E. ciprofloxacin

49. Toxic shock syndrome ( TSS ) closely resembles


Kawasaki disease clinically.
However, some clinical manifestations are more common in TSS.
Of the following, the clinical manifestation
that is MORE specific for TSS is
A. fever
8- hypotension
0. erythematous rash
0. conjunctival hyperemia
E . skin desquamation

^0- Which of the following strains of coagulase negative staphylococci iCoNS) is


-

commonly causing urinary tract infections (UTI ) ?


A. Staphylococcus epidermidis
B. Staphylococcus hominis
C. Staphylococcus haemolyticus
D. Staphylococcus saprophyticus
E. Staphylococcus lugdunensis
to methicillin.
51. Most of coagulase - negative staphylococci ( CoNS ) are resistant
292
methrcif lin susceptible ?
Which of the following strains is
A . Staphylococcus epidermidts
B. Staphylococcus hominis
haemolyticus
C. Staphylococcus
D . Staphylococcus saprophyticus
E. Staphylococcus lugdunensis

52. Treatment of 5. epidermidis infections often requires effective antib«


orKi
with removal of indwelling medical devices.
Of the following, the indwelling medical device infection that is treated
antibiotics without its removal is ^
A. central venous line catheter
B. prosthetic heart valve
C. peritoneal dialysis catheter
0. CSF shunt
E . prosthetic bone

S 3. A 2 - year-old girl develops fever, cough, and shortness of breath secondary


to right middle lobe empyema . She is not vaccinated upon parental refusal. You
suspect invasive pneumococcal infection .
Of the following, the MOST appropriate initial treatment is
A. amoxicillin
B. ampicillin
C . penicillin G
D . ceftriaxone
E . vancomycin

S4. A 5 year old boy develops otitis media


due to pneumococcal infection. He
has a history of non type I allergic reaction to penicillin
-

Of the following, the BEST alternative


treatment is
A . cephalexin
B. clindamycin
C. levofloxacin
D. trimethoprim - suiphamethaxazoJe
E . azithromycin

55. A 4 - year- old boy develop bacterial meningitis due


infection .
t0 invasive pneumococcal
293
ST commonly encountered
0ftHefo
°
Homing, the M adverse neurological sequel
s hearing loss
A
B- epilepsy
,
C intellectual
n
- paralysis
deficits
Q
E, blindness

Which of the following is the MOST common infection


56 caused by
Streptococcus pyogenes?
A. pharyngitis
B. Perianal cellulitis
C. Pneumonia
0. Osteomyelitis
E. Suppurative arthritis

57. Erysipelas is an acute infection involving the deeper layers of the skin and
the underlying connective tissue.
What is the MOST characteristic finding of this infection?
A. Cutaneous swelling
B. Skin erythema
C. Very tender affected skin
D. Sharply defined, slightly elevated border
t e
E . Reddish streaks of lymphangitis projecting out from the margins of
lesion

SB. A 5-year - old boy develops nonbullous impetigo involving the;


extremities. The skin lesions have been resolved after antibiotics
a
pre
_
Of the following, the MOST reliable test that confirms evidence
§roup A streptococcal infection in this child is
A . anti-streptolysin O
B. anti-DNase B
C. streptozyme test
D. Streptococcal rapid antigen detection
Isothermal loop amplification
secondary to group A
59. A 5 -year - old boy develops oral pharyngit starting therapy with oral
.
streptococcal (GAS ) infection You recommend
294
penicillin. To achieve maximal pharyngeal eradication of GAS
and pr ®
rheumatic fever, the oral penicillin must be administered for a venti
of
fnaxi s
A. five days
B. seven days
C. ten days
0. fourteen days
E. twenty-one days

60. A 4- year-old girl develops oral pharyngitis due to group A streptococci


(GAS) infection; she has had a history of immediate (anaphylactiic -
hypersensitivity to penicillin.
type .
Of the following, The MOST appropriate treatment for this girl is oral
A . cephalosporins
B . clindamycin
C. sulphonamides
D. tetracyclines
E penicillin

61. A S- year- old boy develops acute arthritis following an episode of group A
streptococcal ( GAS ) pharyngitis. The characteristic distinct feature that
differentiate post streptococcal reactive arthritis ( PSRA ) from arthritis due to
rheumatic fever ( RF ) is
A . involvement of large joints
B. nonmigratory arthritis
C. long latent period between the antecedent episode of GAS pharyngitis
and arthritis
D . dramatic response to aspirin therapy
E , concurrent high grade fever

62 . Which of the following Jones criteria is required as the only sole criterion to
diagnose acute rheumatic fever ( RF ) ?
A . Acute carditis
B. Polyarthritis
C . Erythema marginatum
D . Subcutaneous nodules
E . Chorea

295
.
A j year-old boy develops acute rheumatic fever manifested
by fever and

^^° arthritis-
,hE following, the LEAST commonly involved joint is
A. hip
8. knee
c. ankle
D. elbow
E, wrist

64 . Characteristic feature of arthritis due to acute rheumatic fever


that
distinguishes it from arthritis due to other causes is
A. hotness
B. non tender
C. dramatic response to small dose of salicylates
D. polyarticular
E. deforming

65. Carditis, whether clinical or subclinical, is a major criterion for diagnosis


of
acute rheumatic fever . The MOST commonly involved cardiac valve is
A. mitral
B. aortic
C. pulmonary
D. tricuspid
E. both aortic and pulmonary

66. A 15- year -old male adolescent has been diagnosed with acute rheumatic
fever at the age of 5 year. His initial manifestations were clinical carditis and
migratory arthritis.
Of the following, the MOST common echocardiographic finding at this age is
A . mitral regurgitation
B. mitral stenosis
C. tricuspid regurgitation
D . tricuspid stenosis
E . pulmonary stenosis

67 . A 5 year - old boy develops acute rheumatic fever . The parents are concerned
-

about future development of valvular heart disease.

296
the feature that is highly correlated with significant rhe
Of the following ^at <
heart disease is joints involvement
A. number of initial *
than 38 C
B. fever more
C . chorea
D. erythema
marginatum
E. subcutaneous nodules
develops acute rheumatic fever manifested by fe e, ,
68. A 6-year -ofd girl
arthritis and carditis . Chest radiography shows cardiomegaly with pulmonan ,
treatment.
edema. She is on oral amoxicillin
following, the MOST appropriate initial treatment for this girl is
Of the
A. prophylactic benzathine penicillin
B. oral salicylate
C. oral acetaminophen
D. oral prednisolone
E. NSAIDs
. Risk of
69. You are discussing acute rheumatic fever with medical students
recurrent carditis is raised during the discussion.
Of the following, the statement that should be included in the discussion is
A. the more severe the initial cardiac involvement, the lesser the risk
for
residual heart disease
B. patients without carditis during the initial episode are less likely to have
carditis with recurrent attack
C. there is a stepwise decrease in cardiac involvement as the number of
episodes increases
D. patients with initial episode of carditis never have carditis with
recurrences
E. the risk for permanent heart damage decreases
with each recurrence.
70. A 2-day -old neonate develops recurr ttac s of aPnea and respiratory
distress; he is lethargic, with bradycardia ^* ^
r feed n He is a full term
baby, a product of normal vaginal deliver rh ? °° '*
prolonged rupture of membranes 24 hours ,'
< mother has a history of
de Very -
Of the following, the MOST likely causative pathoSen is
A. Staphylococcus aureus °
B. group A streptococci
297
Q Streptococcus pneumoniae
0 group B streptococcus
£ Haemophilus influenzae
,


n- »»* ®roup 8 streptococci (G8S) disease in children Kbeyond earV fancy
" . The MOST common infection assnrbt
is uncommon '"
associateda with childhood GBS
A. bacteremia is
B. meningitis
C. pneumonia
D. endocarditis
E. arthritis

72. A 10-day - old neonate develops apnea, bradycardia , and


respiratory distress.
He is a ex - preterm baby by normal vaginal delivery at a
gestational age of 28.
He has been admitted to NCU and an intravascular catheter has been
inserted.
Of the following, the MOST likely causative pathogen is
A. group A Streptococcus
B. group B Streptococcus
C. Staphylococcus aureus
D. Enterococcus
E. Listeria monocytogenes

73. A 3 -year - old boy develops sepsis. Blood culture revealed Enterococcus with
susceptibility to penicillin. The child is allergic to penicillin.
Of the following, the MOST appropriate treatment for this boy is
A. cephalexin
B. azithromycin and gentamicin
C. vancomycin
D. cephalexin and gentamicin
E. vancomycin and gentamicin

74. A 6 -year - old child develops fever, toxicity, tachycardia, and borderline blood
pressure. He has been diagnosed with acute myeloid leukemia 3 months ago
and received intensive chemotherapy . Blood culture revealed vancomycin -
resistant Enterococcus faecalis.
Of the following, the MOST appropriate initial treatment is
A. linezolid
B. daptomycin
298
C. quinupristirvdalfopristin
D. tigecycline
£. cephalexin

75. The major virulence of the Corynebacterium diphtheria organism f


ability to produce a potent exotoxin . The earfy local manifestation of * tT *
ini to
* *
is *
A. paralysis of the palate and hypopharynx
B. kidney tubule necrosis
C. thrombocytopenia
Ot cardiomyopathy
E demyelination of nerves
,

76 . Pharyngeal diphtheria may be misdiagnosed with exudative pharyngitrs


caused by streptococcus pyogenes or EpsteirvBarr virus .
Which of the following features is characteristic of pharyngeal diphtheria rather
than exudative pharyngitis ?
A . Easily detached non adherent membrane
B. Extension of the membrane beyond the faucial area
C . Absence of dysphagia
D. Presence of high grade fever
E . Hoarseness of voice

77 . What is the MOST common cardiac problem associated with respiratory


diphtheria?
A . Myocarditis
B . Dilated cardiomyopathy
C. Hypertrophic cardiomyopathy
D . Cardiac dysrhythmias
E . Heart failure

78. What are the MOST frequent ECG changes detected in children with cardiac
toxicity due to respiratory diphtheria ?
A first degree heart block
.

B. second degree heart block


Q third degree heart block
D sinus tachycardia
E ventricular tachycardia
299
mainstaV of PmanagementIPthaththerishould
,a with
r
t
tensive Membrane
be
is
P<>ssible started as early as
A . crystalline penicillin
B. erythromycin
C. equine antitoxin
D. droplet isolation
E. contact isolation

80. A 1-day - old preterm


ulaTrash^^ °°
^
P r g consistent
with sepsis associated with diffuse pus The mother has a history of
flu-like illness during the 2^ trimester .
What is MOST likely recovered organism by blood
culture?
A. Staphylococcus aureus
B. Escherichia coli
C. Group B streptococcus
D. Listeria monocytogenes
E. Chlamydia trachomatis

81. Listeriosis is indistinguishable clinically from neonatal sepsis and meningitis


caused by other organisms.
Of the following, the lab finding that suggest Listeria infection is
A . increased peripheral blood monocytes count
B . decreased peripheral blood lymphocytes count
C. increased peripheral blood neutrophils count
D. increased platelets count
E . increased C- reactive protein

82 . Listeria monocytogenes CNS infection is usual


Of the following, the CNS site that is c haracteristically
beyond the
affected s
neon
.
atal period.

A. cerebrum
B . cerebellum
C. brainstem
D . hypothalamus
E . spinal cord
inst Listeria ?
agams
is ineffective
83 . which of the following antibiotic
'
A. Ampici hn
300
B. Cefotaxime
C. Vancomycin
D . Trimethoprim - sulfamethoxazole
E . Erythromycin

84. A 10-year-old boy develops right submandibular tender mass 3


dental procedure, associated with pain, fever, and draining fistula after
with
sulfur granules. He has been developed type I diabetes meflitus since Velio*
the of
5 year that is poorly controlled.
Of the following, the MOST likely causative pathogen is
A. Staphylococcus aureus
B. Streptococcus pneumoniae
C. Actinomyces
D. Nocardia
E . Candida albicans

85. A 6-year - old boy develops fever, cough, and dyspnea, followed by
headache,
altered mental status, and speech impairment few days later He has. been
diagnosed with acute lymphoblastic leukemia 1 year
ago and has been on
chemotherapy since that time. CT scan of the chest reveals multiple
pulmonary
cavitary lesions. MRI of the brain shows
multiple-ring enhancing lesions
suggestive of multiple abscesses.
Of the following, the MOST likely causative organism
is
A. Staphylococcus aureus
B. Aspergillus fumigatus
C. Mycoplasma pneumonia
0. Actinomyces
E. Nocardia

86. Which of the following is the


MOST common form of meningococcal
infection?
A. Nasopharyngeal asymptomatic
carrier
B . Meningitis
C. Septicemia
D . Pneumonia
E. Chronic meningococcemia

301
T
following is the MOST common complication of
f the meningococcal
hiC
f ^ ?
>ceZren*'
hemorrhage

i endophthalmitis
Endocarditis
: infarction
0 Focal skin
E- Arthritis
ich of the following factors indicates the POORER prognosis for invasive
$& . Whic
?
mefiingococcal disease
A Hypothermia
0 Hypotension
C . Purpura fulminans
D , Seizures
E. Absence of meningitis

89 . You are hospitalizing a 5 - year- old boy who develops meningococcal


septicemia ; the guardian in close contact with him is his mother who is now
st
pregnant at 1 trimester .
Of the following, the drug of choice for prophylaxis of the mother is
A. oral rifampin
B. oral ciprofloxacin
C. intramuscular ceftriaxone
0. intramuscular ampicillin
E . oral azithromycin

90- Which of the following is the MOST common type of gonococcal infection in
prepubertal girls ?
A- Vulvovaginitis
B . Salpingitis
C Endometritis
D- Perihepatitis
E. Peritonitis
Hematogenous disseminated gonococcal infection may begin 7 - 30 days

* er primary
infection.
the blowing, the MOST is
common initial manifestation
pustular acral skin lesions
ia with fever
B polyarthralgia
C. tenosynovitis
D. suppurative arthritis
E. osteomyelitis
, develo ps ophthalmia neonatorum due to ,
, , av^(d neonate
,
92. A 5-day
a
mother. He is an ex preterm neonate n»
£ n»is ,« .
ZZ XZ
«,,llwlns. ««
«h Bvpe * « *
*“
h
A. cefotaxime
W<
**“
B. ceftriaxone
C cefexime
D. azithromycin
E. tetracycline

year -old boy develop s septic arthritis of the left knee joint. He is a case
93 A 6-
«
hemodialysis. Gram stain of the
renal failure and underg oes frequen t
of chronic with tapered
fluid aspirate reveals pairs of gram -negativ e coeco bacilli
synovial
ends.
Of the following, the MOST likely infected organism is
A. Neisseria gonorrhoea
B. Neisseria meningitides
C. Haemophilus influenzae
D. Kingefla kingae
E. Moraxella catarrhalis

94. Although septic arthritis is the most common clinical presentation °


Kingelia kingae infection, other manifestations may occur with a
relative frequency.
Which of the following is an exceptional clinical
manifestation ?
A. Osteomyelitis
B. Bacteremia
C. Endocarditis
D. Spondylodiscitis
E. Pneumonia

95 . A 3-year - old girl develops osteomyelitis of ieft


clavicle . Of the following, the
pathogen that is frequently infect such site is
303
aureus
$tapMoCOCCUS
o
^ KinBe a kingae
* salmonella
'
typhi
Haemophilus influenzae
'

E Neisseria gonorrhoea
-old boy develops right femur
g6 A 5-year
tingella kingae bacterium. Which of the osteomyelitis . Lab tests confirm
following antibiotics
» this pathogen?
against i
ineffective
A. Vancomycin
B. Ampicillin
C. Ceftriaxone
D . Cefuroxime
E. Oxacillin

! UA ! °'
? Ve r d eiM develops meningitis caused by Hemophilus
"

B. Her mother has a concern regarding hearing impairment influenzae type


sequelae.
Of the following, the drug of choice that
decreases the incidence of this
complication is
A. phenobarbital
B. phenytoin
C. paracetamol
D. dexamethasone
E. rifampicin

98. A 2-year- old boy develops preseptal orbital cellulitis. The characteristic
feature that distinguishes this type of infection caused by H. influenzae from
that caused by 5, aureus and group A streptococcus is
A. fever
B. lid edema
C. lid tenderness
D. warmth of the lid
E. purple discoloration
of
mainly caused by nontypeable strains
99. Which of the following infections is
H . influenzae rather than by H . influenzae type b ^-
A. Meningitis
B . Pericarditis
304
C. Suppurative arthritis
D . Acute epiglottitis
E . Otitis media

100 . The distinction between preseptal


and orbital cellulitis
may bg
clinically . The feature that is more in favor of orbital cellulitis is %
A. fever
B. lid edema
C. tenderness
D. warmth of lid
E. limitation of the extraocular movements

101. Which of the following is MORE common in neonates with pertussis?


A. High grade fever
B . Prominent cough early in the disease
C. Frequent whooping
D. Apnea
E . Rhinorrhea and sneezing

102. Which of the following is a characteristic feature of pertussis in


adolescents ?
A. Fever
B. Posttussive vomiting
C. Sore throat and hoarseness
D. Malaise or myalgia
E. Wheezing and rales with tachypnea

103. A 5 -year - old boy develops rhinorrhea, sneezing, ilacrimation, and


.
conjunctival suffusion Blood count reveals WBC count of 18,000 celfs/pl - You
suspect pertussis. Of the following, the blood smear finding that support you
diagnosis is '
A. absolute neutrophilia
B. high number of band cells
C. absolute lymphocytosis with small normal ceils
D. absolute lymphocytosis with large atypical cells
E . absolute eosinophilia

following lab findings predict poor


104. Which of the outcome in pertussis ?
305
leucopenia
SeVere
g polycythemia
c Anemia thrombocytosis
p Extreme
thrombocytopenia
I Severe

. A 5-year-old boy develops pertussis. He has a history of frequent fainting


105 diagnosis of long QT syndrome has been established
attacks 2 years ago; the
lives with her mother in a single house.
since then. He
Of the following, the MOST appropriate antimicrobial regimen is
A. azithromycin for the child only
B. azithromycin for the child and mother
C. trimethoprim-sulphamethaxazole ( TMP-SMX ) for the child and
azithromycin for the mother
D . azithromycin for the child and clarithromycin for the mother
E. TMP -SMX for the child and mother

106 . Which of the following is the usual complication that may lead to death in
pertussis?
A. Pneumothorax
B. Epistaxis
C. CNS hemorrhage
D . Pulmonary hypertension
E . Bronchiectasis

vomiting, and periumbilical


107. A 5- year- old healthy boy develops nausea,
products, followed by
crampy abdominal pain 1-day after ingestion poultry Typhimurium.
nella
severe watery diarrhea Stool culture revealed Salmo
ment is
Of the following, the MOST appropriate manage
electrolyte disturbances only
A. correction of dehydration and
B . start oral tetracycline for 7 days
days
C . start oral cefexime for 10
for 14 days
D . start intravenous cefotaxime
medications
E . anti emetics and anti diarrheal

following subsets of children are at increased risk for


108 , Which of the
?
salmonella osteomyelitis
infection
A . Children with HIV
306
*
B. Children with leukemia
C. Children with sickle cell disease
D. Children with schistosomaisis
E. Children with chronic kidney disease

109. A 5- year-old boy develops high grade fever, myalgia, abdomi11 31 ,


Mr> M
hepatosplenomegaly. He is on oral antibiotics for 1 week requested
physician. You suspect typhoid fever. by i
Of the following, the lab test that increases the likelihood of bact eri
confirmation in this boy is %
A. blood culture
B. urine culture
C. stool culture
D. bone marrow culture
E. Widal test

110. Complications pertinent to Salmonella typhi infection may differ in


frequency in children comparing to adults.
Which of the following complications is more common in infants ?
A . Intestinal hemorrhage
B. Intestinal perforation
C . Acute cerebellar ataxia
0. Deafness
E . Disseminated intravascular coagulopathy

111. Symptoms due to shigellosis and enterohemorrhagic E. cofi ( EHEC;


infection may be similar.
Of the following, the typical symptom with shigellosis that distinguishes it from
EHEC is
A. high fever
B. bloody diarrhea
C . abdominal pain
D. vomiting
E. anorexia

112. Which of the following is the MOST common complication of shigellosis ?


A , Dehydration
B . Seizures
307
C.
meningitis
ss
0. &
r
t*
'
Encephalopathy
boy develops high fever,
A 3-year-old abdominal rra ps,
SwaterV diarrhea of 1 day duration.
Thereis an " anorexia, emesis,
associated
painful defecation. You suspect bacillary dysentery. Waiting the urgency and
that support your diagnosis stool culture
resuIt, the lab test is
A absence of stool leukocytes
g absence of fecal blood
C. marked anemia
0. marked leucopenia
E. leukocytosis with presence of bands cells

114. A 9-year-old girl develops high fever, bloody diarrhea, and


colicky
abdominal pain. You suspect shigellosis. Of the following, the empirical drug of
choice is
A. ampicillin
B. ciprofloxacin
C. TMP-SMX
D. cephalexin
E. cefuroxime

ns. » ,
bo develops « .* " •
microangiopathic hemolytic anemia a er a pe
»
Of the following, the infectious pathogen that le complication is
A. enterotoxigenic E. coli (ETEC)
B. enteroinvasive E. coli (ElEC)
C enteropathogenic E. coli lEPEC)
D. Shiga toxin-producing E. coli (STE )
E. enteroaggregative E. coli (EAEC)
diarrhea that
116. A 2-year-old boy develops
abdomiiAccompanying
becomes bloody few days later, t e r e i
fever. Microbiology
(0157.- H7 strain
, ^^ STEc
test confirms den C*
2 Appropriate
Of the following, the MO i app w*
step is to

B. SSCSetbopr, -sulphametho«atole TMP


m
-S

308
C . give ciprofloxacin
D. start with oral azithromycin
E . avoid antimicrobial treatment

117 . A 2 - year-old boy develops sudden onset of


profuse
associated with vomiting. There is no crying when waterV dia
His m%
passing stool
describes the stool as " rice- water " with a fishy smell, but U%
Of the following, the MOST likely causative pathogen is
no blood or °
A. rota virus
" 1UCUS
.

B . Vibrio cholerae
C. enterotoxigenic E . coli
D. enteropathogenic E. coli
E . Salmonella enteritidis

118. Campylobacter infections can be both


food-borne and water-borne.
main source of this bacterial pathogen Th
is
A . chicken
B. fish
C. oysters
D. red meat
E . swine

119. Which of the following species


of Campylobacter is MOST likely causing
endocarditis rather than gastroenteritis?
A . coli
B. fetus
C. rectus
D . C. hypointestinalis
E . upsaliensis

12a A 4-year- old girl develops acute


lower limbs flaccid paralysjs
acute diarrheal illness characterized by fever few days after
*
abdom|naJ cramps, and watery
diarrhea .
Of the following, the MOST likely
causative organism i
A. Vibrio cholerae
B . Enterotoxigenic E . coli
C. Salmonella enteritidis
D . Campylobacter jejuni

309
Shigella flexneri
E.
-year- old
boy develops watery
A 2 diarrhea associate with
\ll
3bdom
lnal
'
pa >n- He is a «se of thalassemia major diaBn0 JH fever and
,
h age of 6
, on regular
blood transfusion regimen since
months blood transfusion. Examination of oral tim that U*^*
3 hist0ry
of recent cavity r prominent

,
haryng 5-
0 the
^'
following, the MOST likely offending micro- organism is
jejuni
A . Campylobacter
B. Salmonella typhimurium
C Yersinia enterocolitica
,

D, Shigella dysenteriae
E, enterotoxigenic E. coli

122. Adolescents may be infected by Y . enterocolitica; the MOST common


presentation in such age is
A. enteritis
B. mesenteric lymphadenitis
C. septic arthritis
D. pneumonia
E. meningitis
for
123 . Which of the following drugs is recommended as empirical treatment
children with Yersinia enterocolitis?
A. Trimethoprim- sulfamethoxazole ( TMP- SMX )
B. Ampicillin
C. Cephalexin
D. Amoxicillin- davulanate
E. Cefadroxil
i u Aon irk with remarkable
124. A 7- year -old boy develops inguinal lymp . . how £ insect bites. As
®

tenderness, fever , chills, and weakness. The overlym y pestis .


he is living in area endemic with plague, you suspect in
' ^
es aSpjrate and
Waiting bacteriologic confirmation by culture
o V ^
is
blood, the MOST appropriate initial management of the index case
, and

A . droplet isolation and streptomycin


treatm^
doxycycline prophylaxis of close contacts t of the inde* case only
treatm ^
B . droplet isolation and streptomycin
110
case only
C. droplet isolation of the index
the index case, and doxycy r,
D. streptomycin treatment of ne pr
of close contacts °%
E. close observation waiting
results of bacteriological tests

125. Pseudomonas aeruginosa infection


s are opportunistic infecti
ns th
affect various organs including skin . The characteristic skin |eSj0ns
l that
*
pathogen is °
A . erythema nodosum
B. ecthyma gangrenosum
C. erythema multiforme
D. cellulitis
E. folliculitis

126. P. aeruginosa is an occasional cause of nosocomial bacteremia in


newborns . The focus that is frequently preceding bacteremia is
A . ecthyma-like skin lesions
B. perianal abscess
C. nasal discharge
D. conjunctivitis
E . omphalitis

duration He has
diagnosed with acute leukemia 3 ' 77^ °f 2S dayS '
0 intensive chemotherapy
’ !f° ^ ' "
since that time administered by imolant6 lntravenou Examination
revealed borderline blood press ,
s catheter.
vascu itic skin lesions. Lab tests
revealed: WBC, 500/mm5 - absni„ fDUre ^
'

B. S. epidermidis
C. coli
D. Klebsiella pneumoniae
E. P. aeruginosa

128. A 6-year - old boy has been diagnosed with cystic


fibrosis since early
childhood. He develops chronic lung disease and P .
aeruginosa infection with
frequent pulmonary exacerbations.
Of the following, the drug of choice to decrease pulmona nr
exacerbations is
311
A ceftazidinie
B
cefepime
c pjperacillin-tazobactam
ceftazidime and aminoglycoside
D - combined
E. azithromyc
in

children. Wh ch
A. Fever
.
129. Brucello.sis
. x ^u f i
of the .
is a systemic illness that can be
following r
verV difficult to diagnose
features occur less frequently in children ?
in

0, Sweating
C. Arthralgia
D. Myalgia
E. Back pain

130. A 6- year - old boy develops fever, arthralgia , myalgia, and back pain of 2
weeks duration. He is living in a village . He has a history of consuming raw dairy
products. Examination shows hepatic and splenic enlargement.
Of the following, the MOST widely used test to confirm the diagnosis is
A. blood culture
B . bone marrow culture
C . enzyme immunoassay
D. serum agglutination test
E . polymerase chain reaction
pain, and
131. A girl develops fever, arthralgia , back
5- year -old
a history of consuming raw milk 3 weeks ago.
hepatospl enomegaly . She has
positive.
Serum agglutination test for brucellosis is strongly
Of the following, the MOST appropriate treatment is
A. doxycline
B. doxycline and rifampin
C. doxycycline and gentamicin
( TMP-SMX )
D. trimethoprim- sulfamethoxazole
E. TMP - SMX and rifampin
according to disease
for brucellosis is variable
132. Duration of
treatment ^ ^ of
^ following complications due to
is recommended ?

A . Meningitis
312
B. Endocarditis
C. Arthritis
D. Osteomyelitis
E. Spondylitis

133. Many risk factors may predispose to legionnaire ’s diseas (


Leci
pneumonia ) . The factor that poses the highest risk for infection is *
A. asthma
B. bronchitis
C. chronic corticosteroid therapy
D . diabetes
E . renal failure

134 . A 3- year- old boy develops malaise, anorexia, and right axillary
lymphadenopathy . The lymph nodes are tender with overlying erythema. There
are multiple red papules at the base of right middle finger. He has a history of
contact with kitten 3 weeks ago.
Of the following, the MOST likely infectious pathogen is
A . group B streptococcus
B. Bartonella henselae
C. Toxoplasma gondii
D. Mycobacterium avium
E . Epstein - Barr virus

135. What is the MOST common presentation of cat scratch disease caused by
Bartonella henselae ?
A. Fever
B. Chronic regional lymphadenitis
C. Red papules
D. Headache and malaise
E. Erythema annulare

136. Chronic regional lymphadenitis is the hallmark of cat -scratch disease


affecting the 1st or 2 nd set of nodes draining the entry site .
Of the following, the MOST commonly affected lymph nodes
A . femoral
B. inguinal
C. epitrochlear
313
D axiMai7
an dibular
t subff>

137 -

body weight . Examination reveals expressionless face, bilateral ptosis, and


his
hypotonia ia . His vaccination is integral up to his age.
following, the MOST likely diagnosis is
0f the
A . sepsis
g, myasthenia gravis
C . Guillain- Barre syndrome
D . botulism
E . poliomyelitis

138. Which of the following is a recognized feature of botulism ?


A . Asymmetric flaccidity
B. Ascending paralysis
C. Clear sensorium
D . Fever
E . Paresthesia

by demonstratio
139 . The diagnosis of botulism is unequivocally established
following ancillary investiga
toxin in serum or organisms . Which of the
helpful in the diagnosis?
A . Electromyography ( EMG )
B. Motor nerve conduction study
C . Sensory nerve function
D . Muscle biopsy
E . Nerve biopsy
; it is indicated
140 , Antibiotic therapy is not part of the treatment of botulism
only for the treatment of secondary infections . avoided is
Of the following, the class of antibiotics that shout
A . penicillins
B . cephalosporins
C . macrolides
D. aminoglycosides

314
E. quinolones

141. Which of the following types of botulism requires


with antibiotics in addition to antitoxin ?
a6gressive ,r
A. Infant
B. Foodborne
C. Wound
D . Inhalational
E. Iatrogenic

142. Which of the following is a favorable prognostic factor in tetanus


?
A. Long incubation period
B. Presence of fever
C. Generalized disease
D . Onset of trismus < 7 days after injury
E. Cephalic tetanus

143 . A 10- year- old boy develops crushed injury during biking. The wound is
contaminated with dirt . His immunization status is uncertain.
After immediate and thorough cleansing of the wound, the NEXT step of
management is to
A . administer tetanus toxoid ( TD )
B . administer tetanus immunoglobulin ( TIG )
C. administer both TD and TIG
D . no TD nor TIG is required
E . administer intravenous immunoglobulin

144 . A 7- year - old girl develops diarrhea , fever, and abdominal cramps. The stoo
contains blood and mucus. She has been diagnosed with non-Hodgk |n
lymphoma 6 months ago; she is on prophylactic antibiotics after receiving
intensive chemotherapy.
Of the following, the valuable investigation to confirm the diagnosis is
A . ultrasonography
B . colonoscopy
C . stool analysis
D. stool culture
E. stool toxin test

31 «;
- year- ld boy develops right foot pain aft»
.A lO
°
\ A5 - waling in the grass .

^
j nn

ground
Of the
. Examination reveak r
sanguineous discharge with a sweet odor, an
the wound; crep tations are felt by palpation
following, the MOST likely causative pathogen
is
*
??dental
^
wound injury
swelling,
6
-
edema,
necrotic tissue

A. Staphylococcus aureus
B. Streptococcus pyogenes
C. Klebsiella pneumoniae
D. Clostridium perfringens
E. Pseudomonas aeruginosa

146. Ethambutol is an effective drug against mycobacterial infections. The major


adverse effect that renders this drug reserved for use in older children is
A. headache
B. hyperuricemia
C. optic neuritis
D. peripheral neuropathy
E. hepatotoxicity

147. Trimethoprim-sulphamethaxazole is a commonly used drug in some non -


tuberculous mycobacterial infections. The MOST common adverse effect with
prolonged use of this drug is
A. myelosuppression
B. renal abnormalities
C. aseptic meningitis
D. pancreatitis
E. diarrhea
feature of tuberculosis
148. A medical student asks you about the distinctive
infection. ( TBI ) is
Of the following, the hallmark of tuberculosis is infection
A . fever and night sweating
B . hemoptysis
C . positive tuberculin skin test ( TST )
D . radiological evidence of lung cavitations
lung mfH r
E . radiological evidence of diffuse
149. The time between initial infection and clinically apparent
Tfc di
variable.
Of the following, the organ that has the latest manifestation after TB
infec ,
A. meninges
B. lymph nodes * ^
C. kidney
D. bones
E. joints

150. A 12-year- old boy develops fever, productive cough, hemoptysis


, nL
sweating, and significant weight loss. He has a history of contact
with h,
grandmother who has have TB disease. Physical examination is insignificant
.
Of the following, the MOST common radiological findings in this boy is
A. small nodules evenly distributed on the chest
B. unilateral pneumothorax
C. calcified hilar lymphadenopathy alveolar consolidation
D. extensive infiltrates with thick- walled
cavities in the upper lobes
E. unilateral pleural effusion

151. Skeletal tuberculosis is a f


late complication of tuberculosis and is more
likely to occur in children than
inJ adults. Which of the following bones is MOST
likely involved?
A. Skull
B. Vertebrae
C. Humerus
D. Pelvis
E . Femur

152. A 13-year-old girl


undergoes t „hn .
.
skin test (TST) as a prerequisite m

entering a secondary school
. The t«r
of application. She has
been receiven D ~ WS ^ induration after 12 houR
birth -
/
CG vaccine at the age of 1 month aW
Of the following, your MOST
appropriate d
A. to repeat the test after 1
week
ecision is
B. the test should be
considered
C. to consider the test negative Positive
D. the girl is considered to be infected
with non
E. the test is considered a reaction due TB mycobacterium
r
to previous BCG
*7
-* -sitssKCK
' fliose born in high
,n skin test (TST )
with a

^
- prevalence regions of
tuberculosis
*
B. Those
in close contact with known contagious People
with tuberculosis
disease
with Hodgkin disease
C. Those
with diabetes mellitus
D. Those
E Those
with chronic renal failure

medical student asks Vou about the advantage of using Interferon-y


U. A
*
please Assay (IGRA
) in tuberculosis. Which one of the following
statements
should be included in your answer?
A. It can differentiate between TB infection and TB disease
B. It has cross - reaction with most other mycobacteria
C . It can be performed well in malnourished infants
D . It can be performed well in children with disseminated TB disease
E. it is preferred in children > 2 yr of age who have received BCG vaccine

155. A 4- year- old boy has recent exposure with his grandfather who
has active
tuberculosis disease . His tuberculin skin test ( TST) is negative.
Of the following, your NEXT step in the management is to
A. start isoniazid ( INH) for 10 weeks then repeat
the TST
B. start INH for 9 months
infection
C. observe for early development of tuberculosis
D. repeat TST after 2 weeks
E. start rifampin for 6 months
pregnant woman in i
156. A gynecologist asks for your opinion regarding to
trimester with active TB disease . Your c linical
decision is
A . terminate the pregnancy
B. defer treatment until after delivery
C . start treatment with isoniazid
D. start treatment with
,
aminoglyco
rifampin
sides ,
,
and
an
. .
\ ^ ^thambutol

eth o nam jd
d

— .
ui
E . do PCR for tuberculosis of the ammotic

1S 7. A 5-year- old boy develops ui “tysrr srs


,‘" P'
S
nodes. He has a history of playing m . IWW no
non erythematou
tender , freely movable, and
318
evaluation reveals well capsulated lymph node, about 1.8 cm jo
preservation of hilum. Fine needle aspiration PCR and
cult
^
mycobacterium avium. Which of the following is the preferred initial
A. Wait- and- see approach
B. Complete surgical excision
C. Rifampicin for 3 months
^
D . Ethambutol for 3 months
E. Combination of clarithromycin and rifabutin for 3 months

158. Which of the following drugs is the only effective treatment for
congenital
syphilis ?
A. Oral penicillin V
B. Oral ampicillin
C. Oral doxycydine
D. Parenteral crystalline penicillin G
E. Parenteral procaine penicillin

159. A 4- year - old girl develops recurrent attacks of high fever and flu-like illness
lasting about 4-7 days. At the end of each febrile episode, a diffuse macular rash
appears over the trunk and shoulders lasting for 1- 2 days. She has poor socio-
economic status, living in mud hut . She has been developed pediculosis capitis 2
weeks ago .
Of the following, the MOST likely diagnosis is
A . primary syphilis
B. leptospirosis
C . cat- scratch disease
D. relapsing fever
E. toxoplasmosis

160. Which of the following is the drug of


choice for a 9- year- old boy with louse-
borne relapsing fever ?
A. Ampicillin
B. Chloramphenicol
C. Doxycydine
D. Azithromycin
E. Ceftriaxone
of the following is the 1«
161 Which«e? characteristic cutaneous
Lym,e disea
manifestation of
A Erythema nodosum
B. Erythema migrans
C Erythema annulare
D Erythema induratum
l Erythema toxicum

162 . Which of the following cranial nerves is MOST commonly affected by Lyme
disease ?
A. Olfactory
8. Oculomotor
C. Abducent
D . Facial
E. Vagus

163 . A school- aged boy developed non - productive cough; he has a preceding
headache, malaise , fever, and sore throat few days before . Examination is not
significant . Chest radiograph shows bronchopneumonic changes in the right
lower lobe.
Of the following, the MOST likely causative pathogen is
A . staphylococcus aureus
B. streptococcus pneumoniae
C. mycoplasma pneumoniae
D. parainfluenza virus
E. respiratory syncytial virus

164 . 6- year - old girl develops mycoplasma pneumoniaia . The first -line therapy
^
for this girl is
A . azithromycin
B. doxycycline
C. levofloxacin
D . trimethoprim
E . linezolid
MOST likely
16S. Which of the following electrolytes distu
therap
encountered with amphotericin B deoxycholate
A . Hyponatremia
320
I

B. Hypokalemia
C. Hypochloremia
D. Hypocalcemia
E. Hypophosphatemia

166 . A 7- year-old boy with acute lymphoblastic leukemia

*rv.
develops ,nv
pulmonary aspergillosis after prolonged neutropenia.
Of the following, the MOST effective antifungal drug is
A . 5- fluorocytosine ( 5- FC)
B. fluconazole
C. itraconazole
D. voriconazole
E. amphotericin B deoxycholate

167 . What is the MOST effective azole antifungal treatm


ent for mucormycosis?
A . Fluconazole
B. Itraconazole
C. Voriconazole
D . Posaconazoie
E . Isavuconazole

168. A 7-year- old boy who underwent


allogenic bone marrow transplantation
develops refractory pulmonary asperg
illosis in spite of prolonged therapy with
voriconazole.
Of the following, the 2 nd line of
treatment is
A. fluconazole
B. itraconazole
C. 5- fluorocytosine
D. amphotericin deoxycholate
E. caspofungin

169. Which of the following is the


MOST common manifestation of invasive
candidiasis in neonatal period ?
A . Meningoencephalitis
B. Endocarditis
C. Osteomyelitis
D . Arthritis
E . Endophthalmitis
321
,70 Which ofinfants
.
the following hematologic
with invasive , ,S
premature candid s COmmonlV encountered
A. Neutrophilia
0. Neutropenia
C. Thrombocytopenia
D. Thrombocytosis
E. Anemia

in. A 10-day-old infant develops


feeding. He is a breast fed baby since dell'T* ° -
' fussiness and decreased
m her has been received
oral antibiotics after episiotomy procedure f i
presence of extensive pearly white CUrd
curdkh
'
1 Week ' Examination reveals
T
Sh mate on the tongue, palate,
m

buccal mucosa. ' '


Of the following, the MOST appropriate treatment Is
' and

A. Topical nystatin to the infant


B. Intravenous amphotericin to the infant and mother
C. Topical nystatin to the infant and mother
D. Baking soda to the infant and mother
.
E No treatment is required

172 . A 3- month-old infant develops diaper dermatitis manifested as a confluent


erythematous rash with satellite pustules.
Of the following, the MOST appropriate treatment is topical
A. fucidic acid
B. clotrimazole
C. betamethasone
D. zinc oxide
E . vaseline
.provnized manifestation of cryptococcosis?
173. What is the MOST comm V
A. Pneumonia
B. Cutaneous lesions
C. Sepsis
D. Meningitis
E. Arthritis
is ithe typical finding
seen in MRI of a child with
following
174. Which of the
invasive pulmonary
aspergillosis ?
322
A . Hafosign
sign
B. Air crescent
C. Target sign
D. Lobar emphysema
E. Segmental bronchiectasis

of the following is the diagnostic test of choice for ,j


Which
175 .
children with cancer and hematopoietic stemnVJsrvt
aspergillosis iin ceil
transplantation ?
A . Blood culture
B . Histological specimen
C . Eosinophil count
D . Galactomannan test
E. Beta-glucan assay

176. A 5- year- old boy develops fever, headache, chest pain, cough,
arxj

myalgias. He is living in a rural area; he has a history of prolonged exposure tc


chicken coops present nearby his house . Examination reveals
hepatosplenomegaly. Chest X -ray shows right side patchy bronchopneumonia
and perihilar lymphadenopathy.
Of the following, the MOST likely fungal infection is
A . Candida
B. Aspergillus
C. Histoplasma
D. Blastomyces
E . Coccidioides

177. Which of the following radiographic findings is MORE likely suggestive d


pulmonary mucormycosis rather than other
pulmonary invasive
fungal
infections ?
A . Solitary pulmonary nodule
B . Segmental/lobar consolidation
C. Cavitary changes
D. Bronchopneumonic changes
E . Pleural effusions

178 . Which of the following is the primary


mucormycosis ?
mainstay of treatment for
323
Liposomal amphotericin B
g Fluconazole
C Voriconazole
p posaconazole
£ Caspofungin

-old boy develops


0. A 9- year cough, abrupt onset of dyspnea, tachypnea
fever, and
productive with decreasing oxygen saturation i

non i i . i
. Elimination is
not
has been diagnosed
conclusive. He with acute lymphoblastic leukemia
9 months
ago and he is now on maintenance chemotherapy. Lab tests show decreased
WBC count , chest radiograph reveals bilateral diffuse interstitial ground glass
infiltrates.
Of the following, the organism that is MOST likely detected by bronchoalveolar
lavage is
A. Staphylococcus aureus
B. Streptococcus pneumoniae
C. Aspergillus fumigatus
D . Pneumocystis jirovecii
E . Cytomegalovirus

180 , Which of the following is the alternative prophylactic treatment for


Pneumocystis jirovecii in children who are allergic to TMP -5MX ?
A . Benzathine penicillin
B. Oapsone
C. Fluconazole
0 . Itraconazole
E . Chloroquine
boy who has recently develops
181. You are meeting a mother of 3 -year old
-

the infectivity period to avoid risk of disease


measles. She is asking you about
transmission to other siblings.

"?
before the onset of rash
-
1
D. only three days
days after the onset of rash
E. only four

324
following is a pathognomonic feature of
182. Which of the measles?
A. High fever
B. Photophobia
C. Conjunctivitis
D. Koplik spots enanthem
E. Maculopapular exanthem

of measles is the longest to last?


183. Which of the following features
A. Fever
B. Cough
C. Conjunctivitis
D . Photophobia
E. Rash

184. A 14- year- old boy develops repetitive attacks of myoclonic jerks not
associated with loss of consciousness. He has been experienced poor academic
experience, irritability, reduced attention span, and temper outbursts few
months ago. He has a history of measles infection at the age of 4 year. EEG
shows suppression-burst episodes.
Of the following, his cerebrospinal fluid ( CSF ) MOST likely reveals
A. high WBC count
B. low sugar
C. positive measles culture
D. elevated measles antibody
E. positive measles antigen

185 . A 3- year-old girl has


been exposed to an index case
nursenr home 4 days ago. She doesn't with measles in the
receive any vaccine yet.
°,echj° s
8 th £ M ST appropriate
' ° management to prevent or modify
A. vaccine administration
B. immunoglobulin
administratiion
C. vitamin A supplementation
D . ribavirin treatment
E . no treatment is required

186. Which of the following is the MOST


congenital rubella syndrome ? common finding among infants
325
A. Deafness
B. Blindness
C. Congenita!heart defects
D. Growth
retardation
delay
E. Psychomotor

following is the MOST common


IS7 . Which of the ocular abnormality in
cong enita l rube lla synd rome ? children
tfith
A. Cataract
B. Microphthalmia
C. Iridocyclitis
D. Glaucoma
E . Retinopathy

188. Which of the following is the MOST frequently


encountered cardiac defect
in children with congenital rubella syndrome
?
A. Ventricular septal defect
B . Patent ductus arteriosus
C. Pulmonary artery stenosis
D . Pulmonary valve atresia
E. Aortic valve stenosis

A pre nant worr> an in the 1st trimester is exposed to an index case with
r he la .
| ^
Three subsequent blood samples for IgG are collected; the 1st one has
n obtained at

^ A If the Is bloo
'
once, the 2 nd one after 3 weeks, and the 3rd one after 6 weeks.
>ch of the following
termination of pregnancy?
indicate recent infection of the mother that necessitates

d sample is positive
nd rd
^sample
1 blood sample is negative with positivity of either the 2 or 3

C - If the l and nd
2 samples are positive and the 3 one is negative
rd

If all 3 samples are positive


If all 3 samples are nega
tive
190. A pregnant wom
an in the Is' trimester is exposed to an index case of
rubella. Lab tests confirms recen infection. She refused to termmate
t
Pregnancy .
0f the
following, the MOST appropriate management is
A. dose follow up
6. MMR vaccine
C. immunoglobulin
administration
D. antiviral therapy
of pregnancy
E obligatory termination
.
191. Which of the following is the MOST common CAK cornp,^%
mumps?
A. Meningoencephalitis
B. Transverse myelitis
(ADEM)
C. Acute disseminated encephalomyelitis
D. Facial palsy
E. Sensorineural hearing loss

192. A 13 -year -old male adolescent develops fever and bilateral parotja
swellings consistent with mumps. He has been never received MMR vaccine
during childhood.
Of the following, the MOST commonly encountered complication is
A. meningoencephalitis
B. orchitis
C. pancreatitis
D. myocarditis
E. thyroiditis

193. Which of the fo / iowing is a feature of poliomyelitis rather than Guiflain-


Barre syndrome?
A. Asymmetric paralysis
B. Absence of fever
.
C Sensory changes
D. High CSF protein
E. Normal CSF cells

194. A 4 - year -old boy develops painful cutaneous


, lesions involving lips and bot*
hands
assorted w th low grade fever.
,
Examination shows tender vesicles on
the tongue, buccal mucosa hps, and dorsa surface and paims
of the bend
;
8
erythema. ° '
W esions surround ^
Of the following, the MOST likely causative pathogen is
327
Herpes simplex virus
g parechovirus
c Coxsackie A virus
D coxsackie B virus
l Adenovirus
of the following is the MOST
l95. Which common ocular manif stations
by enteroviruses
? ^ caused
A. Chorioretinitis
B. Uveitis
C. Acute hemorrhagic conjunctivitis
D. Optic neuritis
E. Acute maculopathy .

196. A 6-year -old girl develops low grade fever, headache, and flu -like illness.
She then develops bilateral cheeks erythema followed by diffuse macular rash
over the trunk and extensors of extremities, with central clearing in some areas
giving a reticulate appearance . Soles and palms are spared.
Of the following, the MOST likely diagnosis is
A. erythema nodosum
B. erythema multiforme
C. erythema marginatum
D. erythema infectiosum
E. erythema migrans
feature of Parvo B19 virus infection
distinctive with erythema

infectiosum only ?
A . Well -looking appearance
B. Profound anemia
C. Absence of fever
D. Extensive rash
E. Long incubation period herpes labialis in
inical feature of
is the EARLIEST dim
198. Which of the following
older children?
A. Fever
B. Vesicular lesions
328
C. Burning sensation
D. Sore throat
E . Local lymphadenopathy

- -
,
.. develops feve • headache, vomiting, nuchal hr,
’.
ear d h
D0V
199 . « 7¥ 0 6
seizures and 3 n leve| of
conscious.
tf>s and 20% neutrophils
);
^
^
«
of the fo owne .
ce
x intravenous ceftriaxone
B. intravenous vancomycin
C. intravenous amphotericin
D. intravenous ganciclovir
E. intravenous acyclovir

who develops varicella 3 days


200 , A full term neonate is born to a mother
of cutaneous lesions.
before delivery . Examination is normal with no evidence
is
Of the following, the MOST appropriate management
A . close observation for the development of rash
B. to give intravenous immunoglobulin (IVIG )
C. to give varicella -zoster immunoglobulin ( VZIG)
D . to start acyclovir prophylaxis
E. to give varicella vaccine

201. Which of the following is a characteristic feature of congenital varied


syndrome ?
A . Microcephaly
B. Microphthalmia
C. Cataracts
D . Low birth weight
E . Cicatricial skin scarring

202 . Which of the following sites of lymphadenopathyy is < cu /arfy suggest


particularly
15 Part sugge>
of infectious mononucleosis ?
A . Anterior cervical
B. Posterior cervical
C. Axillary
329
D Epitrochlear
E inguinal

-
Which of the following malignancies is associated with Epstein - Barr virus
203
infections ?
£ Retinoblastoma
g Medulloblastoma
C Nasopharyngeal carcinoma
D . Nephroblastoma
E . Neuroblastoma

204. An 8 - month- old boy develops high grade fever associated with fussiness
and irritability . Three days later, there is abrupt defervescence with the
appearance of rose- colored, nonpruritic , 2 3 mm morbilliform rash over the
-

trunk . He is a breast fed infant, and his vaccination history is complete up to his
age .
Of the following, the MOST likely diagnosis is
A . measles
B . rubella
C. erythema infectiosum
D. erythema toxicum
E . exanthem subitum

of the sixth
205 . Which of the following is the MOST common complication
disease ?
A . Seizure
B . Acute disseminated demyelination
C. Arthritis
D. Hepatitis
E . Myocarditis

following is the 1* sign of respirator syncytial virus infection


206. Which of the
in infants ?
A. Fever
B . Rhinorrhea
C . Cough
D. Audible wheezing
E . peripheral
cyanosis
330
of the following features differentiate Rsv b
207 Whi ch ror
less thanH*
.
is interstitial pne umo nia in infa nts of
Chlamydia trachomatis 4
age ?
A. Cough
"X
B. Fever
C. Nasal discharge
0. inspiratory crackles
E. Expiratory wheezing

208 Which of the following is the MOST common manifestation of h


uman
adenovirus infection in children ?
A. Meningoencephalitis
B. Myocarditis
C. Conjunctivitis
D. Hepatitis
E. Hemorrhagic cystitis

209. A 10- month- old healthy male infant develops watery


diarrhea of 2 days
duration . There has been a preceding mild fever
and vomiting that are stoppee
1 day ago . He is a breast fed infant with
introduction of solid food after the age
of 6 month. Examination shows
moderate dehydration . Stool analysis reveals
absence of blood or white blood cells.
Of the following, the MOST important
step of management is
A . correction of dehydration
B. stopping breast feeding
C. treatment with TMP-
SMX
D. ondansetron antie
metic
E . probiotic therapy

210. A 7- year- old girl iIS


exPosed to a bite by a dog
observation for 10 days . . The dog is kept iof
Of the following, the
MOST
A . give immediately
6. give immediately
C give immediately both ^ ^,
killed Tvlccin
rabies immunoglobulin
vaccine and RIG
8^6
"' “ *
(RIG )
°
D. give immediately both
£ . euthanize the dog
vaccine and RIG i
mediately and test
.
the dog shows 1* sign of rab es
the brain tissue for rabies

331
of the °"ow n8 microscopical s
nl whichhistolytica
'rather'than * ool findings
‘nt 3moeba
£
Entamoeba dispar ? is specific
for
A
presence of leucocytes
B
presence of fat droplets
presence of phagocytosed erythrocytes
^0 Presence of trophozoites
,

E. Presence of cysts

212. Which of the following immunodeficiencies is particularly


chronic Giardia infection? associated with
A. Chronic granulomatous disease
B. X-linked hypogammaglobulinemia
C. Leukocyte adhesion deficiency
D. DiGeorge syndrome
E. Chronic mucocutaneous candidiasis

213. Which of the following has the highest


diagnostic sensitivity for detection
ofLeishmania donovani infection ( kala -azar ) ?
A. Indirect fluorescence assay
B. K 39 immunochromatographic strip test
C. Direct agglutination test
D. Bone marrow aspiration
E . Splenic aspiration

214. You are discussing malaria infection in children with a group of medical
students.
An important statement that should be included in the discussion is
A . incubation period is the longest with P . falciparum species
B. children with malaria often lack the typical paroxysms seen in adults
C. periodicity of paroxysms is more apparent with P. falciparum
D . complications are the highest with P . knowlesi species infection
E. most common serious complication is bleeding diathesis

215 . Which of the following species of malaria causes nephrotic syndrome as a


complication?
A . P . vivax
B . P ovale,

C . P malariae
.

132
D . P. falciparum
E. P. knowlesi

216 . Which of the following antimalariaJ


drugs is used to
Preven r ,
vivax and P. ovale infections
A. Primaquine
?
^ lnp

B. Chioroquine
C. Quinine sulphate
D . Quinidine gluconate
E. Mefloquine

217. A 9- year-old boy is going to travel with his parents to chloroquine-


resistant
P. falciparum endemic country . The plan of the parents is to stay there fori
month. He has medical problems, including seizure disorder and G6PD
deficiency that are diagnosed since early childhood.
Of the following, the MOST appropriate chemoprophylaxis is
A. atovaquone-proguanil
B. mefloquine
C. primaquine
0. doxycycline
E . sulfadoxine - pyrimethamine

218. Which of the following is the MOST


common ocular manifestation of
toxoplasmosis ?
A. Strabismus
B. Microphthalmia
C. Cataracts
D . Nystagmus
E. Chorioretinitis

219. Which of the following sites has a


calcifications secondary to congenital propensity to be involved by
toxoplasmosis ?
A . Frontal cerebral cortex
B . Cerebellum
C . Caudate nucleus
D . Corpus callosum
E . Hypothalamus
of the following is the drug of
ijO. which by Ascaris lumbricoides choice for i•ntestinal
ion caused or biliary
infestations ?
^ p
Q
p
Albendazole
Mebendazole
Lvermectin
piperazine citrate
l Nitazoxanide
the following worms ' infestation can
221 - Which of cause iron deficiency
anemia ?
A . Tapeworms
B. Threadworms
C. Roundworms
D. Hookworms
E, Pinworms

222 . Which of the following worms ' infestation can cause nocturnal perianal
pruritis with subsequent sleep disturbances ?
A. Tapeworms
B. Threadworms
C. Roundworms
D . Hookworms
E . Pinworms

nt for schistosomiasis ?
223. Which of the following is the recommended treatme
A. Piperazine citrate
B. Praziquantel
C . Albendazole
D. Mebendazole
E . Lvermectin
megaloblastic
infestatio n can cause
224 . Which of the following parasitic
anemia ?
A . Diphyllobothrium latum
8. Hymenolepis nana
C Ancylostoma duodenale
,

D Taenia saginata
,

E, Ascaris lumbricoides
334
described as cortmuous ^
^*?S£i yg iSkX
< >ont^°
rJ
deve
^
ifi -
**£^
and <
**s ** ' # »
^
\ b
**
eo*>y
ates
a
tbe
*

Most ViV
ca
.eW ^
se *s

n(?»
in\\0 ^j\
0 ^ *k -*^ aorew ^
£ coV' y,omats

a\ virv
S •
)
tf , cVt \
C C- ofS
,esp'tat
E. a

33S
Chap ter 16
Infectious Diseases
Answers

MacConkey agar supports growth of gram - negative rods while suppressing


J.(0)
gram-positive organisms. Nutrient-rich media such as sheep blood agar and
chocolate agar are used to aid in the recovery of fastidious pathogens . Special
media, such as Sabouraud dextrose agar and inhibitory mold agart are used to
recover fungi,
2.(t) Propionibacterium and coagulase-negative staphylococci are the
organisms most frequently isolated from shunt infections .
3.IB ) Plasmodium and Babesia can be detected in stained blood smears,
Leishmania tan be detected in stained bone marrow smears. Serologic tests are
important in documenting exposure to certain parasites that are not typically
found in stool or blood, and thus are difficult to demonstrate in clinical
specimens, such as Trichinella .
MA) An age-dependent change in lower airway colonization occurs among CF
patients, which starts in early childhood with S. aureus and H . influenzae and
progressively shifts toward more intrinsically multidrug - resistant organisms,
including the notoriously persistent and treatment -refractory bacteria
Pseudomonas aeruginosa and Burkholderia cepacia complex which is strongly
correlated with disease severity,
MB) Fecal transplantation, or administration of feces from healthy donors to
COAD recipients, is cost- effective treatment and superior to antibiotics in
educing the likelihood of recurrent disease .
MD)
MA) IG1V may be used for varicella after exposure when varicella - zoster
Imrnune globulin is not available . 1GIM is mainly used for Hepatitis A meas
es,
and rubella prophylaxis . IGSC is used for treatment of patients wit P ry
antibody is use or pr
mrnunodeficiencies . Palivizumab monoclonal
gainst respiratory syncytial virus .
( e.g. , P
Vaccines consist of whole inactivated microorganisms *

.
A) parts of the organism ( e . g acellular pertussis, HPV , hepatitis B),
polysaccharide
Polysaccharide capsules ( e .g . , pneumococcal and meningococcal
. . coW>6ate t0 protein
f ae C3PS °
c
^'' ° carnet

-
vaccines), polysa
varicella. « «^^
con

^**^£
rca\ vaccinesV
nio
'°L
6

„ -
pneumococcal, and
microorganisms
attenuated:
0«« 5
i
e. . r e s ' ^ ps, rube
'
( g 0 3 eS
AVS

: ^ \ eWnos, pWVve* *
L vacdne*ft c0 'iaV
'
.mv
*
=
« TAw
cells/mm
^3
VW
nercentageis c
for
^ . —
those
n>.
X
w

yr .
iw
old MMR
provided the child is asymptomatic
immunosuppression.
10.(C) Children with complement
«
*"
>
*,*lN
» «" * 35S;
1 may be given to
*0a
or symptomatic without evidence of

deficiency disorders may receive all vacc-



including live-attenuated vaccines. In
disorders may receive both inactivated and
not live-attenuated bacterial vaccines.
contrast, children with phagoc
live-attenuated viral vaccines

11.(D) Children receiving corticosteroids (£2 mg/kg/


day or £20 mg/day
^^
prednisone or equivalent ) for >14 days should not receive live vaccines unt„
^
therapy has been discontinued for at least 1 mo. Children on the same dose
levels but for <2 wk may receive live virus vaccines as soon as therapy is
discontinued.
12.(E) Infants born to HBsAg -negative mother, with a weight £ 2 kg and who are
stable should receive a birth dose of hepatitis B vaccine within the 1st 24 hr of
life. However, hepatitis B vaccine should be deferred in infants weighing <2 kg
at birth until chronological age of 1 mo. All preterm, low birth weight infants
-
bom to HBsAg-positive mothers should receive hepatitis B immunoglobulin
( HBIG ) and hepatitis B vaccine ( at separate
anatomic sites) within 12 hr of birth.
13.(B) Immunoglobulin does not
interfere with inactivated vaccines. However
immunoglobulin can interfere with the
and by inference to varicella
immune response to measles vaccine
vaccine. In general, immunoglobulin, i
should be administered at least
the dose of immunoglobulin
I 2 wk after the
measles vaccine. Depen
*,
Vn
received, MMR should be
11 mo. Immunoglobulin is not
expected to
deferred for as long ^ ^
interfere with the immune resp
to LAIV or rotavirus vaccines
14.(A) Hand washing is an i
.c .
*

°^
important
control. method for infection prevention and
15.(C) Fungi and multidrug-
resistant organisms
in immunocompromised children are common causes of infection
as well aS
prolonged hospitalization. those requiring intensive care
16 (0) Transmission by droplets
involves
particles over a short distance (<3 feetj . the IPropulsion of
infectious large
337
Position on another's mucous
gSS^S:Srr ^“
^
(

18
A
ectrfjn-

.
‘ istent
than bacterial
- ryptosPor ,c
protective

pathogens

^'
immunity develops within 2

; Giardia lamblia is the most


diarrhea.
Less
urin sPPw histolytica
^
likely
common travel-associated
Provide adequate
wk after the initial

«7&*,» enteric protozoa are a much less common cause of traveler s


'
protozoal cau

, and Cyclospora.
vaccine

protozoa
include ^ o'

^
(

jO) Azithromycin is
highly effective against most
bacterial pathogens that
'
C 30& traveler_
s diarrhea and is the preferred antibiotic among many travel
. . ,
experts. Ciprofloxacin is an alternative for children >1 yr old but should not be
prescribed for travelers to the Indian subcontinent or Southeast Asia , where
fluoroquinolone resistance is common.
20.(C) Drugs known to cause fever include vancomycin, amphotericin B, and
allopurinol.
21-lA) Other disorders are characterized by recurrent episodes of fever that do
not follow a strictly periodic pattern.
22.(B) Relative bradycardia can accompany typhoid fever , brucellosis,
leptospirosis, or drug fever . It may also be a result of a conduction
defect
resulting from cardiac involvement with acute rheumatic fever, Lyme disease,
viral myocarditis, or infective endocarditis.
23.(D)
24 -( A)
Z5.{D) The presence of drooling and
neck or throat pain in an infant or toddler
maY be suggestive of a retropharyngeal abscess, which is usually confirmed by
irnaBing that may include a lateral radiograph of the soft tissue of the neck or
computed tomography (CT ) if clinical suspicion is high. CT scan is the definitive
diagnostic imaging test. X-ray of the neck often (80% of the time ) shows
swelling of the retropharyngeal space in affected individuals.
Pseudo-FUO is defined as successive episodes of benign, self-limited
infections with fever that the parents perceive as 1 prolonged fever episode. It
usually starts with a well- defined
infection (frequently viral) that resolves but is
followed by other febriie viral illnesses that may be less well defined.
The continuing absence of sweat in the presence of an elevated or
changing body temperature suggests dehydration caused by vomiting, diarrhea,
or central or nephrogenic
diabetes insipidus. It also should suggest anhidrotic
ectodermal dysplasia , familial dysautonomia , or exposure to atropine.

338
28.(C) Palpebral conjunctivitis
coxsackievirus infection,
is in a febrile patient may be
tuberculosis,
,

infectious to-
lymphogranuloma venereum, or cat scratch disease. \n contrast,
-

conjunctivitis in a child with FUO suggests Kawasaki disease or \


29.(A)
BO.(C) Sepsis can occur but is
more common with certain gram
^
eptosp\ros\s.

organisms such as C. violaceum and F. philomiragia.


nefotiv?
31.(E) Unhke a r P tjbody defect, besides being important in
T cel4
important in the interaction between T
cell interactions
^ ^^
and macrophages//mo
.
mnnnrvtes
.
v ' influencing opportunistic infections such
Pneumocystis jirovec pneumon a l cp) and Cryptosporidium
infection in such disease.
cells

intestinal*
. jn children with febrile
(
32 B) Indications for using: neutropenia
prophylaxis, colonization
include severe mucositis, VP ^' qujno|ones
with methicillin resistant staphylococcus aureus, and obviOUs catheter related
infections.
33.(E) Viridans streptococci are particularly important potential pathogens in
patients with the oral mucositis that is often associated with use of cytarabine
and in patients who experience selective pressure from treatment with
certain antibiotics such as quinolones. Infection caused by this group of
organisms can present as acute septic shock syndrome.
34.(B} The Most commonly used antibiotic monotherapy is cefepime or
piperaeillin-tazobactam. Ceftazidime should not be used as monotherapy if

concern exists for gram-positive organisms or resistant gram negative bacteria.
Carbapenems such as imipenem/cilastm and meropenem should not be first
line, aiming to prevent pressure on carbapenem-resistant Enterobacteriaceae
The addition of a 2nd anti gram-negative bacterial agent (e.g., aminoglycoside)
for empirical therapy can be considered in patients who are clinically unstable
when multidrug- resistant organisms are suspected.
35.(E) Post transplant lymphoproliferative disorder is more common amonS
children than adults, because primary EBV infection in the immunosuppres
*
host is more likely to lead to uncontrolled proliferative disorders, including post
^
transplant lymphoma .
36.(E) Green discharge is strongly suggestive of mycobacterial infection, and
appropriate stains and culture should be performed.
37.(0) Prolonged therapy (4-6 wk) is indicated for persistent bacteremia f °
fungemia despite catheter removal, since this may represent unrecognized
infective endocarditis or thrombophlebitis.
339
the advent of conjugate vaccines against
wM l 1 aticatty- However, outbreaks still occur, Hib, iinvasive disease has
* *
dr and have heen
Ctined ( q parenta
{ l refusal of vaccines. observed
< £ and streptococcus pneumoniae
mediate resistance
hanisms other than fl lactamase
59-1 Mec
throu'Bh. nt of little value for the management
* 5
- production, rendering
other
to penicillins

of these infections.
antib' 0
tThe third generation cephalosporins (cefotaxime ,ceftriaxone, and
^' '
ceft3Z d m
highW ac '
® are typically used for serious pediatric infections. Ceftazidime is
agairist most strains of P. aeruginosa. A fourth-generation
cefepime) has activity against P. aeruginosa and retains good
tephatosporjn ^
st metWdmn.susceptjb|e staphylococcal infections.
first-generation cephalosporins are effective against susceptible
tt- C
'
strains of aureus and
c
, group A streptococcus. The second, third, and fourth
generation ^ ha osporins have better activity against gram-negative bacterial
al activity
infections. Ceftaroline (a fifth generation cephalosporin has bactericid
)
, and common gram -
against resistant gram-positive organisms, including MRSA
negative pathogens .
Stenotrophomonas maltophilia,
42.(C) Meropenem is not active against
rendering their use for cystic fibrosis patients
who are! infected with this
E . faecium are not
susceptible to
organism problema tic. MRSA and
is poorly active against P . aeruginosa and
carbapenems. Ertapenem
should be avoided when these pathogens are
Acinetobacter species and
encountered. Mycobacterium tuberculosis
managem ent of
43.(A) It is also important in the
resistant gram negative
beta lactamase
organisms (Enterobacteriaceae
nd carbapen emase producin g strains). suppurat i y j a arthritis,
ive
* result in osteomyelitis ,
, pericard itis,
MB Direct invasion
) can a, endocarditis
nia, empyem
PVomyositis, deep abscesses, pneumo rapidiY
and rarely meningitis. spread more
are prone to
46.(A) Group A streptococci skin infectio
and can be very aggressive . includes M . tuberc ulosis ,

by MSSA
47.(D) Other etiology of cavitary Pne umonia thought to be caused
infection s or a first -generation
4S.(B) Initial treatment for serious
.gnafcil lin )
>90% of all
n ( e because
should include semisynthetic penicilli
,
, are not appropriate
cephalosporin . Penicillin and amp
cl
staphylococci isolated are resistant to these agents. Quinofone
unpredictable activity against MSSA and no activity against MRSA 'Qtics s
.
49 ( B) Other features commonly seen in TSS include diffuse mya[gia
. * NB
abdominal pain, diarrhea, azotemia, ARDS, and shock. X *
50.(D)
51.(0) Unlike most CoNS, S. saprophyticus is usually methicillin suscept ,b
UTl can typically be treated with a first generation cephalosporin ( a cephauj
amoxicillin—clavulanic acid, or TMP-SMX . %
52.(C) Peritonitis caused by S. epidermidis in patients on continuous ambuiat
peritoneal dialysis is an infection that may be treated with IV or intrape
antibiotics without removing the dialysis catheter.
53.(D) Empirical therapy with a parenteral third generation cephalosponn
should be prescribed for hospitalized infants and children who are not fully
^
immunized, in regions where local epidemiology of invasive pneumococcal
strains documents widespread penicillin resistance, or for infants and children
with life-threatening infection, including those with empyema. Non-fJ- lactam
agents ( vancomycin ) have not been shown to be more effective than thiru
generation cephalosporins in the treatment of pneumococcal pneumonia.
54.( A) For individuals with a non-type I allergic reaction to penicillin ,
cephalosporins ( standard dosing ) can be used. For type I allergic reactions
( immediate, anaphylactic) to 3-lactam antibiotics, clindamycin and levofloxacin
are preferred alternatives.
55.( A) Pneumococcal meningitis results in sensorineural hearing loss in 20-30
%
of patients.
56.( A ) Streptococcus pyogenes is a common cause of infections of the upper
respiratory tract ( pharyngitis) and the skin ( impetigo, pyoderma ) in children
.
Less frequently, it causes perianal cellulitis, vaginitis, septicemia, pneumonia
,
,
endocarditis, pericarditis, osteomyelitis, suppurative arthritis, myositis, cellulite
omphalitis, and other infections.
57.(D)
58.(B ) The anti-streptolysin 0 response can be feeble after group
streptococcal skin infection. In contrast, the anti-DNase B responses are
generally present after either skin or throat infections. Slide agglutinate
( Streptozyme ) test for the detection of antibodies to several streptococci
antigens is much less well standardized and less reproducible than other
antibody tests, and it should not be used as a test for evidence of a preceding
GAS infection . Streptococcal
rapid antigen detection and isothermal 1
°^
°
341
are used for identification of recent GAS
jieP
infection diirectiy from
15
*
itir®
rt.(Q
6O-( 0) Some
, and
-allergic
penicillin
these
persons ( up to lrwa
agents should be avoided ,n °
,
a so allergic
to
a losporins with immediate
ceP^ -type ) hypersensitivity to penicillin
(anaP
hylactic clindamycin iis an
agent for treating penicillin-allergic patien
appropriate A

(erythromycin or danthromycin) or azalide (azithromycin


allergic to penicillins
V ) is
lsals
'
£° macrolide
an appropriate
agent for patients
6t(B) PSRA
usually involves the large joints similar to the arthrit , ,
but it may also involve small peripheral „,
joints as « 11 , l ,
rtls of acu e RF,

^
131 Ske eton- The
latent period between the antecedent episode of ell.uPharvngitis
^
may be considerably shorter (usually <10 davo L a d PSRA
"
acute
with
RF (
aspirin
usually 14 - 21 days) PSRA do
or other nonsteroidal anti ti •
* typical|V
resP nd dramatically to therapy
°
with

< NSAIDs) ln addition


fewer patients with PSRA than with acute
Very small proportions of patients with PSRA h
RFhaveT
! T!f

'
>38°C < 100- 4*F)-
H b6en reported to develop
valvular heart disease and carditis.

Sciterilaen° SiS f 3CUte RF n bC made Without strict adherence to the


° “
in B circumstances: (1) when
manifestation of acute RF, (2) when indolent c a r t
patients who first come to
medica a en, mont.hs after the apparent
onset of acute RF, and (3) in a limited number of pa patients with recurrence o
acute RF in particularly high-risk populations.
6MA) Arthritis involves larger joints,
in acute rheumatic fever ty
*
particularly the knees, ankles, wrists, and elbows,
small joints of the
&4.1Q Rh
being
pi ,« « »
* - ^
hands and feet, or hips is uncommon.
nt of the spine,

the friction of bedclothes

nature, (K
dramatic response
characteristic feature of the arthritis, a absence o
suggest an
tt
alternative diagnosis.
-(A)
.
Valvular insufficiency is ... j acute and convalescent
aracteristic of both
char usually
stagesof scute RF, whereas mitral and /or aortic ic valvular stenosis
appears years or
67 even decades after the acute illness.
SSSSSESi
70.(D)
the 1
:
congestive heart
69.(B)
Early
st
6
complications
-onset
days
failure should receive corticosteroids.

and premature labor.


71.(A)
neonatal
of life
, including chorioamnionitis

72.(D) Late-onset enterococcus infection


, prolonged rupture of
^
group B streptococci ( GBS) disease pres
and is often associated with matem

(>7 days old) is associated


*^
**

'"*"«»*
*
*
'

factors such as extreme prematurity, presence of an intravascular cattef^


, 0r
necrotizing enterocolitis ( NEC), or it follows an intraabdominal
procedure.
7B.(E) Cephalosporins should not be relied on in situations where Enterococ
is known or suspected to be involved. Vancomycin can be substituted for the
penicillins in allergic patients but should be used with an aminoglyco
because vancomycin alone is not bactericidal.
**
74.(A) Linezolid is a bacteriostatic antibiotic against most E. faecium and l
faecalis isolates, including VRE isolates. Response rates are generally >90V
including cases of bacteremia and sepsis, and this antibiotic has become the
preferred agent in treatment of VRE infections in many institutions. Experience
with daptomycin in children is limited, particularly in the setting of Enterococcus
infections. Quinupristin-dalfopristin is inactive against E. faecalis. Tigecycline is
an expended-spectrum derivative of the tetracycline family. Its efficacy in VRE
infections has not yet been demonstrated in clinical trials. Tigecycline
cause discoloration of the teeth, and its use in children <8 yr old should
generally be avoided.
75 .( A) All other manifestations are systemic due
to toxin absorption.
76.( B) The characteristic adherent
membrane, extension beyond the ^auC ‘,3 '
area, dysphagia, and relative lack of fever help differentiate diphtheria ff 0 T1
exudative pharyngitis.
77.(A) Subtle signs of myocarditis can be detected in
most patients.
78.(D) Tachycardia disproportionate to fever is
common and may be evidence
of cardiac toxicity or autonomic nervous system dysfunction
. A prolonged
interval and changes in the ST-T wave on an electrocardiographic tracing are
relatively frequent findings. Cardiac dysrhythmias can
occur. Ventricular
tachycardia is also described.
343
antitoxin is the mainstay of therapy and
, $PeC’ ' diagnosis . Because
should be administered
fl finical
^
°nishes
it neutralizes only free
b3
^ elaPsed time after the onset
of
toxin antitoxin
,

e ms- ie r
WV dir" mucocutaneous
le
0 of antimicrobial therapy is to halt toxin production
pt° and prevent transmission of the treat ,
^Waed '°rf,eConset
' ^ neonatal listeriosis (<5 days organism
of birth)
to contacts .

JQJD) E Vfor
3 °rm characterized by premature
, is predominantly
delivery or still birth obstetric
0
gpticetf' ' . multiorgan involvement including rash but without
,
cationS'
c0(np
"
localization *
CNS
elevated peripheral blood monocytes suggests listeriosis.
*^ ^ ^
cytosis or lymphocytosis may be modest or striking .
82.1C)

ESESHrHiHrS
possible L. monocytogenes infection .
• '
MOT * hallmark of artmomycoss co
tissue or fascial planes, wi

^^
abscesses and sinus tracts. The presence o su
of the diagnosis. Risk factors for actmomy
dental caries, debilitation , and poorly control e 1
oranules is highly suggestive
.
children include trauma ,
eWitus >

Mocardia organisms are primarily pathogens infecting


immunocompromised persons. Nocardiosis ^ chrontc
suppurative infections. Dissemination after prima infection is common ,
^
The central nervous system ICNSf Is the most concerning an( j most common
secondary site of infection.

W.fD) It typically affects the lower limbs


u ano.1 can lead to substantial scarring
and require skin grafting
.
Poor prognostic factors on presentatio . rlllde hypothermia or extreme
hyperpyrexia , hypotension or shock, purpura u. inanSi seizures, leutopenia ,
thrombocytopenia (including D1C), acidosis, a circulating levels o
endotoxin and TNF -a. The presence of petec iae 2 br before admission ,
absence of meningitis, and low or norma ^ rapid , fuiminan
Progression and poorer
prognosis.
8S.(C) _
Ceftriaxone ;and ciprofloxacin are the most effective
agents for
women. R ampin is
prophylaxis. Ciprofloxacin in pregnant
is not recommended
most w dely used colonization in 15% of cases.
‘ but faj|s tQ eradicate
^AA
routinely. Neither penicillin
n is
Azithromycin
; not rec ommended nor drrirv
nasopharyngeal carriage and should not be
treatment eradicates
for prophylaxis. vaginal epithelium and alkaline pH of the vagi
ified
90.( A) Thin noncorn infection of the lower genital tract. ^ UClr
age group
predispose this 0SteOmyelitis are rare manifestations.
^
SlSCefotaxime should be substituted for ceftriaxone in neonates
*
(particularly those who are

^
premature) and in
hyperbilirubinemia those
Says old if receiving calcium-the
containing intravenous fluids.
most common etiology of septic arthrn,
93.(0) Kingella kingae is
osteomyelitis and spondylodiscitis in young
children. Older children and
with K. kingae infections often
have underlying chronic diseases **
immunosuppressing conditions, malignancy, or cardiac valve pathology.
.
94.(E) Pneumonia is an exceptional clinical presentation
95.(B) K. kingae osteomyelitis usually involves the long bones
of the extremities
The calcaneus, talus, sternum, and clavicle are also frequently affected (and are
rarely infected by other bacterial pathogens ).
96.( A ) K. kingae is always resistant to glycopeptide antibiotics, and the majority
of isolates are also resistant to clindamycin.
97.(D) Dexamethasone, particularly when given shortly before or concurrent
with the initiation of antimicrobial therapy, decreases the incidence of hearing
loss .
98.( A) Absence of fever and interruption of the integument (by insect bite or
trauma ) are more likely to occur with the latter 2 pathogens.
99.(E) Most H. influenzae isolates causing otitis
media are nontypeable.
100.(E ) Orbital cellulitis may manifest as lid
edema but is distinguished by the
presence of proptosis, chemosis, impaired
vision, limitation of the extraocular
movements, decreased mobility of
the globe, or pain on movement of the

SequenthApnae7 ndbc%Pno rcTnfoHowCial V


0

as the
occur
a
only symptom (
^
without couahl
102.(B) Pertussis should be susDectPri ,
cough of 14 days' duration, with ° ^
. ,
, ^
3 C Ugh ng paroxysm or apnM
° '
lndiv dual who has paroxysms o
#

* wh in

^ °T posttussive
the absence of fever, malaise or my
ale vomiting, especially
hoarseness, tachypnea, wheezes, and r enanthem, sore throat
103. ( C) Leukocytosis ( 15,000- 100,
rales ° /

000 cells/pL )
lymphocytosis is characteristic in the caused by absolute
catarrhal stage . i
- Vmphocytes are norma
345 *
. rather than the large, atypical lymphocytes
,, rt|ls
Lhilia suggests secondary bacterial infection ith Viral ,1

^JJi- cytosis
'Ration
ajo) severe
of pertussis.
course and death are correlated with
(medianpeak white blood cell count
ranirf
Eosinophils nfections
Is not

extreme
.
a

in fatal
pL, respectively ) and thrombocytosis

^^ 000 vs 18,000/
nt in fatal vs
nonfatal cases, 782,000 vs 556,
( JIT peak
000/m, re5Peetively),
cases,
platelet
both of
^
1

tiicti are findings with progressive pulmonary hypertension


Azithromycin is the drug of choice in all age
10S {CJ
- -groups, for treatment Or
postexposure prophylaxis. However , there is a risk of fatal heart rhythm
with
use of azithromycin in patients already at risk for cardiovascular events
especially those with prolongation of the QT interval. TMP-SMX is an alternative
to azithromycin for infants >2 mo old and children unable to receive
airthromycin-
106.(0) Progressive pulmonary hypertension in very young infants and
secondary bacterial pneumonia are severe complications of pertussis and are
the usual causes of death.
107.( A) Antibiotics are not generally recommended for the treatment of
isolated uncomplicated Salmonella gastroenteritis in immunocompetent child,
because they may create a chronic carrier state , However, they are
recommended in young infants ( < 3 mo old), and in immune compromised
children.
1®.(C) Children with HIV infection may develop florid disseminated disease
Children with schistosomaisls are also at a greater risk for development ^
o

chronic salmonellosis. result of


105 (D) The mainstay of the diagnosis of typhoid fever is a positive seen
culture . Results of blood cultures are positive in AO-60% t * may render
early in the course of the disease . Widespread libera an i i may
cultures
bacteriologic confirmation even more difficult. Bone
increase the likelihood of bacteriologic confirmation of ^ intravascular
H0.(E) Diarrhea, toxicity, and complications such as fatality
hjgher case
coagulation ( DIC) are more common in infancy, resu jn admits are
rare , n
rates . However, Gl and neurologic implications ^^
children,
111.(A) frequently causes rectal
1 infection
also
112.{A) S. dysenteriae serotype
prolapse, and hemolytic-uremic syndrome-

346
... <ri Presumptive data supporting a diagnosis of bacillary flrsef
“ !L n of
leukocytes (usually >50 or 100 PMNs
the presence of colitis
}, fecal blood. and de
^^** S
%

£ of feukocytosis with a dramatic left shift (often w*, ^


segmented neutrophils). *
(B) Ciprofloxacin (20-30 mg/kg/24 hr divided into 2 doses; f
114
choice recommended by WHO for all patients with bloody darrt *. *
is recommended
of age. Concurrent zinc supplementation
therapy.
enterohemorrhagic E. coli ( EHECJ or
115.(0) It is also known as
producing E, coli ( VTEC).
116 (E ) The STEC strains represent a particularly difficult therapeutic
many antibiotics can induce bacterial stress, toxin production, and pnagt
mediated bacterial lysis with toxin release. Antibiotics should not be giver
STEC infection because they can increase the risk of HUS.
^
117 (B )
118.( A ) Ingestion of contaminated poultry and raw milk are mainly commo-
sources of this pathogen.
119.(B) C. fetus shows a predilection for vascular endothelium, leading it
endocarditis, pericarditis, thrombophlebitis, and mycotic aneurysms. C
hyointestinalis has been associated with proctitis, C. upsaliensis with bras:
abscesses, and C. rectus with periodontitis.
120.(D) C. jejuni has been identified as the trigger in up to 40% of patients with
Guillain-Barre syndrome (GBS ). When associated with Campylobacter, G6S is
more likely to be the axonal form and has a
worse prognosis with slower
recovery and more neurologic disability.
121.(C) Y. enterocolitica is transmitted to
humans through food, water, animal
contact and contaminated blood products
. Conditions associated with iron
' 0f
kf “
ma v be seen m 2n
from other causes of g°
122.(B)
^ astraSerititsPreSentati°
<
this P ogen. Prominent pharyngi
^
n Whidl
' h lp diStingUiSh
^ *
*
"
resi.staLe to penicillins - andPfi?sdt generarCtamaSeS' Wh'Ch are responsible for
124 (B) Patients with suspected plague
^ * ^
^^
until pneumonia is ruled out, sputum P acet on droPlet isoJat <ofl
cul
^ 5 ne® ative and antibiotic
treatment has been administered for 4g
h Th& ^ '
plague has been streptomycin for lo davc o treatrnent of choice for bubonic
V 0Stexp
sure ° prophylaxis be
should
347
7
c ntacts of patients Wlth Pneumonic piagUf>
Ited
.
°
close

^
f
bubonic plague do not require proph - ^
s of cases of
^
<, , P
| begins as pink macules and progress
With eCchvmotic
'
to hemorrhagic
nodules
^I*ation.
uallV t0 ulCer by an intense and gangrenous centers with and
$

surrounded red areola. eschar

aw>)Ch,ldren With ,eukemia or thelgnanc


other malio es . ,
,,
, parti, ,
'
**
0
receiving immunosuppressive
intravascular catheters
Who are
, are extrem SUSCeptib e to sept/
i , neuJ
' Wa
cem CSUSed
°
V

"
hose wt>
*‘ *
f aeruginosa . Signs of sepsis are often acc0mpanied by a general,'rt
necrotic lesions mav , vascuit« , , *.
jnd hemorrhagic
hp f
f Und n 1 or«ans.

^^ ° '
3
) Macrolide therapy decreases Pulmonary exacerbationc •
MIC
chronic lung disease and P
killing.
129.(B) Fatigue, sweats, chills, anorexia, headache, weight loss, and malaise, are
reported in the majority of adult cases but are less frequent in children.
cases. Bone
130.(D) Cultures are insensitive and positive only in a minority of
evaluating patients
marrow cultures may be superior to blood cultures when
enzyme immunoassay
who have received previous antimicrobial therapy. The
serum agglutinatio
should only be used for suspected cases with negative
c r C
or for the evaluation of complicated cases, suspected ° c|jnjca|
assays are not avai a
reinfection. Polymerase chain reaction
laboratories. because of the risk of
131.(E) Combination therapy is generally recommended more than
is recommended for children
relapse with monotherapy . Doxycline
in severe cases of
°
132.(A) Treatment may need to be continued
for up to 1 yrin
with
central nervous system (CNS ) disease. infection by interfering
risk for with exposure
133.(C) Corticosteroid therapy poses a high combined
risk factor
T cell and macrophage function. it is a major
papules develop
to contaminated potable water. more red
-12 days , 1 or ^ cat scratch
.
134.(B) After an incubation period of 7 a linear
reflecting in -Barr virus
,
inoculation , often wk . Epstein
at the site of cutaneous
within 1-4
usually Lcause
more
lymphadenopathy is generally evident
Toxop as gondii infections
cytomegalovirus , and
generalized lymphadenopathy .

348

occurs in

, ,s
-=
^
1 'r?
apP

137 D ingestion of honey


relative blood flow and
£
anorexia
symptoms, including
malaise,

ndibuTafpreauricular
sras*;
, fatigue, and

, epitrochlear, femoral,
is an identified risk factor of infant botulism.
innervations
botulism manifest neurologically
beginning with the cranial nerve
are greatest in the bulbar
as a symmetric, descending, flaccid par
musculature and progressing over
^
headache, affect N
k*

BJJ
rnuscuiatUf
%l
hour;
.


days,
onset of a symmetric flacctc
138 (f) The classic triad of botulism is the acute
descending paralysis with clear sensorium, no fever, and no paresthesias.
139.(A) EMG can sometimes distinguish between causes of acute fiacc *
paralysis. The distinctive EMG finding in botulism is facilitation (potentiation) oi
the evoked muscle action potential at high- frequency (50 Hz ) stimulation. Nerve
conduction velocity and sensory nerve function are normal in botulism.
140.( D) This class of antibiotics can potentiate the action of botulinum toxin a!
the neuromuscular junction.
141.(C) Wound botulism requires aggressive treatment with antibiotics and
antitoxin in a manner analogous to that for tetanus and may require wound
debridement to remove the source of the toxin.
142.( A Other favorable prognostic factors are absence of fever, and localized
>
disease. An unfavorable prognosis is also associated with onset of generalized
tetanic spasms <3 days after onset of trismus. Cephalic tetanus has an especially
poor prognosis because of breathing and feeding difficulties .
143.(C) Non- minor wounds require human TIG fully
except those in a
immunized patient (i.e., >3 doses of adsorbed
tetanus toxoid). A tetanus toxoid
booster ( preferably Tdap ) is administered to all
persons with any wound if
tetanus immunization status is
unknown or incomplete. A booster is
administered to injured persons who have completed
senes if ( 1) the wound is dean and minor
the primary immunization

booster.
,
booster or ( 2 ) the wound s more serious
but no yr have passed since the
and « *
yr have passed since the last

,
U4.(E) Clostridium difficile infection ( CDI) s
diagnosed by the detection of a C
or nudeic acid
spectrum antibiotics, hospitalization, Gl surgery
Ude the use of broad'
surppru,
inflammatory bowel disease
349
.iBOl
( ..

Sissr-sa;
—— -*, or H2 -

. «
/
^ Rhambuto should ge
(Cl
have visual
J47.(A)
^ ^ " bTrel
y
acuity and color discrimination
eTfor cS'
^ "86"5 Sp0re$'
reliably monitored^
.
old enough to
"

148.(0 The hallmark of TBI is a positive TST or interferon-y release assav nro „
result, in this stage the child has no signs or symptoms a
examination, and the chest radiograph is either normall or
granuloma or calcifications in the lung parenchyma.
. normal physical
reveals only’
.
149 (0 Disseminated and meningeal tuberculosis are early
manifestations
often occurring within 2-6 mo of acquisition. Significant lymph
node or
endobronchial tuberculosis usually appears within 3 9 mo. Lesions
-
and joints take several years to develop, whereas renal
of the bones
lesions become evident
decades after infection .
150.(D) Reactivation tuberculosis is the most likely diagnosis in
this case. All
other radiological findings can be seen with primary TB, military TB,
and TB
.
pleural effusions
151.(B) The classic manifestation of tuberculous spondylitis is progression to
Pott disease, in which destruction of the vertebral bodies leads to gibbus
deformity and kyphosis.
152.(C) The amount of induration in response to the test should be measured
by a trained person 48 -72 hr after administration. Immediate hypersensitivity
reactions to tuberculin or other constituents of the preparation are short -lived
(<24 hr ) and not considered a positive result. Older children and adults who
receive a BCG vaccine are more likely to develop tuberculin reactivity, but most
lose the reactivity by 5-10 yr after vaccination.
153.(B) In other groups TST are considered positive when induration is 10 mm.
154 ( E) IGRA has superior specificity (95%) in BCG -immunized, low-risk children.
.
Most experts support the use of IGRAs for the evaluation of TB infection in
young children at low risk of infection, especially in those who have received a

. a negative TST or
'
1S5 ( AMNH should be given to children <5 yr old who have
have a known recent exposure to an
adult with potentially
IGRA result out who
result but w
tice is often referred to as window prophylaxis .
ByTheTimVdelayed hypersensitivity develops (2-3 months), an untreated child
350
ss rs SS S iS Ss
S
l
^ ^ ^'sirii
WBecause
fetus and represents

TB infection ( not
?
ssj
?
:
a
rtrjrjsr
pulmonary

disease
great

) is often
^
tuberculosis is harmful to both the mother
danger to the infant after
pregnant women always should be treated. The most common regime V
delivery

drug-susceptible tuberculosis is isoniazid, rifampin, and ethambutol


aminoglycosides and ethionamide should be avoided because of
teratogenic effect , the treatment of pregnant women who have asymptomal
deferred until after delivery .
,
and
tuberc '
^
{ ^
\

157.(B) The preferred treatment of NTM lymphadenitis is complete surg


excision as surgery is more effective than antibiotic treatment. Nodes should ^be
removed while still firm and encapsulated. If surgery of NTM lymphadenitis
cannot be performed for some reason, or removal of infected tissue s
incomplete, or recurrence or chronic drainage develops, a 3 mo trial of
combination chemotherapy is warranted . M. avium complex bacteria are ofte
resistant to rifampin or ethambutol alone , although spontaneous resolution can
*
take several months, a wait - and-see approach can be chosen in selected patient
which is not the case in this scenario as the lymph node is enlarging.
158.(D) Parenteral penicillin G is the only documented
effective treatment fix
congenital syphilis, syphilis during pregnancy ,
and neurosyphilis. Aqueous
crystalline penicillin G is preferred over procaine
penicillin, because it better
achieves and sustains the minimum concentration of f 003
0.018
units/ mL ) needed for 7- 10 days to achieve
the prolonged treponemicidaf levels
required for the long dividing time of T. pallidum
. In penicillin allergic patient
desensitization followed by standard penicillin therapy
is the most reliable
strategy.
159.(D) Louse-borne relapsing fever is
caused by Borrelia recurrentis and »
ransmrtted from person to person by
Pediculus humanus the human body
louse. Human infection occurs as a result of
crushing lice during scratching
Diagnosis depends on demonstration of
in thin or thick blood smears stained with
spirochetes by darkfield
.
microscopy °
dmc <
,
Giemca ^ .
culture. During afebrile remissions, spirochetes
160.(C) Oral or parenteral tetracycline or
are ^*
Und n tbe blood'
doxycycline ,js° the '
louse - borne and tick- borne relapsing fever . (n drug of choice fof
children V unger
° than 8 yr of age»
351
„,ycin (50 mg/kg/dav divided every 6 hr P0) for a total f 10 days
r0
e
^ -The
jnded.
rash is generally either uniformly erythema ,.
°
. is

^ (hence the nlm^


clearing. The rash gradually
‘Javerage
central
diameter of
,
15 cm and
expands
typically remains
present
eSi n

for -
'
l 2 J2T *°
0

62 (0) All of the cran al nerves e


* <
*pt the olfactory have been relTa . .
ivdved
l6J (C) The
the sudden
Lyme
with
gradual
onset of
disease
,
onset
lobar
in .
but
ch
pneumonia
the most common are VI
ldren with atypical
pneumonia is in
and especial

. Coryza and gastrointestinal contras to


,
complaints
are unusual and usually suggest a viral etiology. Examination may
be
unrevealing. Pneumonia is usually described as
interstitial or
bronchopneumonia and involvement is most common in the lower lobes
164.( A) Mycoplasmas lack a cell wall, so they are resistant to 3 -lactam agents
that act by inhibiting the cell wall synthesis. Other drug classes, such as
trimethoprim, rifampin, or linezolid are inactive against M. pneumoniae
.
Typically, M. pneumoniae is sensitive to macrolides, tetracyclines, and
quinolones. Macrolides are preferred in children younger than 8 yr of age.
Tetracyclines may be used for children older than 8 yr of age. Fluoroquinolones
are effective and bactericidal but have higher minimum inhibitory
concentrations (MIC) compared with macrolides and currently are not
recommended as a first-line therapy in children.
165.(8) Nephrotoxicity, hypokalemia, and hypomagnesemia are common.
166.(D) Voriconazole is guideline-recommended as the preferred primary
therapy against invasive aspergillosis. Antifungal resistance develops quickly to
S-FC monotherapy . Fluconazole is ineffective against Aspergillus and other
molds. Itraconazole has the benefit of antifungal activity against Aspergillus
species but has erratic oral absorption in high-risk patients and significant drug
interactions, and it is no longer recommended for primary therapy of invasive
asPergillosis. Amphotericin B was once the preferred treatment for most
invasive fungal infections. .
.
167.(D) Posaconazole is active against Zygomycetes such as muc° mvc°s '
voriconazole is not active against these particular mold ^ . .
nfect ons.
Isavuconazole is not as potent against Zygomycetes as
, ~
^^ ^ ^ |"
0 V
fluconazole plays an important role in the rea emic mycoses
Itraconazole has a role in treating less-serious infectio
( histoplasmosis, coccidioidomycosis, and blastomycosis).
168-( E) Caspofungin was
approved for refractory aspergj|f0sjs 0r
other therapies and for candidemia and various other sites of tol
* '"
invasrv ;
infections. *C u*
\
involvement is common and is
169.( A ) Central nervous system
described as meningoencephalitis. Candida infections involving the '
nervous system often result in abscesses
. Renal involvement
complicates neonatal invasive candidiasis . Renal involvement maybeK
i
:
candiduria or can manifest with diffuse infiltration of Candida throughout
renal parenchyma or the presence of Candida and debris within the collect
system.
!^!
X70.(C) Hematologic parameters are sensitive but not specific
Thrombocytopenia occurs in more than 80% of premature infants withinvas
candidiasis, but also occurs in 75% of premature infants with Gram-negatrve
^
bacterial sepsis and nearly 50% of Infants with Gram positive bacterial sepsis ,
171.JC) Treatment of mild cases might not be necessary. When treatment B
warranted, the most commonly prescribed antifungal agent is topical nystatin
In breastfed infants, simultaneous treatment of infant and mother with topica
nystatin or oral fluconazole may be indicated.
172.(B) Candida diaper dermatitis is treated with topical antifungal therapy such
as nystatin, clotrimazole, or miconazole . If significant inflammation is present,
the addition of hydrocortisone 1% may be useful for the 1st 1-2 days, but
topical corticosteroids should be used cautiously in infants because the
relatively potent topical corticosteroid can lead to adverse effects.
173.(D) CNS disease is the most commonly recognized manifestation of
cryptococcosis. Affected patients may develop intracerebral niasse5
( cryptococcomas) and increased ICP. Pneumonia is the 2 nd most commonly
recognized form of cryptococcosis.
174.(C) Target sign, a nodule with lower central signal compared to the rifl *

enhancing periphery, is typically seen in MRI. Halo and air crescent signs are
usually seen in CT scan.
175.( D ) ELISA - based test for galactomannan, one of the components of th^
Aspergillus cell wall, is the molecular biomarker of choice for the diagnosis of I
*
in serum, BAL fluid, and CSF. It is the most sensitive test in detecting disease in
cancer patients or hematopoietic stem cell transplant recipients. The bet *
glucan assay, is a nonspecific fungal assay that detects the major component of
the fungal cell wall but it does not discriminate which fungal infection <s
infecting the patient . Isolation of Aspergillus from blood cultures is uncommon ,

likely because fungemia is low - level and intermittent .


353
Acute
, icl space mocula in
* s
0 n contaminated
*arnP^ °time is 14 days.
soil, chopping decayed exposure ( e .g.,
wood). The median

0» bation presence of multiple nodules ( £ 10), pleural


* |E) The
l77. are more suggesti ve of mucormy cosis
effusions, or the
Pjgd halo sign .

151 A ) Voriconazole
is not active against mucormycosis.
many of the Mucoral es but is
Posaconazole s active
presently not recommended for primar
,
!
against
may be used for salvage therapy or step down from an amphotericin
Iberapv; it
leased regimen. Echinocandins lack significant activity against the Mucorales
0*

of P. jirovecii pneumonia occurring in children with


179.1DI In cases due to immunosuppressive
immunodeficiency
medications, the onset of
hypoxia and symptoms is often more abrupt. Rales are usually not detected on
physical examination. Ground glass appearance in both lungs is the usual
radiographic finding.
e and pentamidine,
180.(B) Other alternative prophylactic drugs are atovaquon
but all of these agents are inferior to TMP - SMX .
lSl.(C)
182.(D) Koplik spots represent the enanthem and
are the pathognomonic sign
of measles, appearing 1- 4 days prior to the onset
of the rash . They first appear
bluish white spots in the center on the inner aspects
as discrete red lesions with

Tine desquamation of skin in its wake . cells but elevated


i SSPE reveals normal
184.(D) Cerebrospinal fluid analysis in
igG and IgM antibody titers in dilutions >1:8. of measles if given
modific ation
185.(B) The vaccine is effective in prevention or to 6 days after
may be given up
within 72 hr of exposure . Immunoglobulin
exposure to prevent or modify infection. finding among
infants with
186.( A) Nerve deafness is the single most commo
CRS. ocular abnormality
mos are the
187 .(E) Salt- and-pepper retinopathy is the teral cataract s
but have little early effect
most serious eye finding,
on vision
occurring in about a th
re
.
. Unilatera ^ ^ ^
i
|nfants -
^ reported cardiac defect,
188.(B) Patent ductus arteriosus is the most 3 disease ,
vajvU|ar
followed by lesions of the pulmonary arteries ^
354
"Til ^
immune. If an
s
"•*“ "S Ttha
specimen
recent infection.^ - M
positive test result in

****" te un 1in the



I90.(C| The routine use of immunoglobulin for susceptible pregnant
exposed to rubella is not recommended and is considered only if te
1
- 1
either th OCcurr,

rrrn
pregnancy is not an option because of maternal preferences und nation L
that prophylaxis may reduce the risk for clinically apparent infectio **Ur ri *
not guarantee prevention of fetal infection.
n >ut d
* * ;
191.( A All other complications are less common. Sensorineural h
)
earing loss «.
rare.
192.(B) in adolescent and adult males, orchitis is 2nd only to parotitk
as d
common finding in mumps.
» 3.(A) In Guillain-Barre syndrome , which is the most
difficult to distinn-
from poliomyelitis, the paralysis is characteristically symmetric,
and seL
poliomyT
66
h “? Phyramida'traCt SignS are COmmon trasting with
' “"
m
*•®
1 d,

^ romeHsaisdmStrpd;m°ThV Caused ne of the


SeaSe
sy ' ° distinctive rash
1 T
'
19S iciar , ,
by ««aclcieviruS A16 .

and coxsackievhr
h
- ' XaTvTrtem ’
contagiousness, with ^
VTa rilV
spread mainly via eye
bV enter ViruS D7°
|
hand fo >
W
0

, **
196.(D) The most common
manlfestSL i
infectiosum ( fifth disease ) . The initial
a Stage
f Parvo
V tra 0a
* “
Vlrus 819 is erythema
( slapped-cheek appearance)
. The r ' ls an erythematous facial flushing
rapid or concurrently to the
trunk and proximal
extremities
stage. Central clearing of
macular le
3 USe macu ar
'
ns 0ccurs promptly ^ ^
erYthema in the second
reticulated appearance . ° , giving the rash a lacy.
197.( B) In contrast to children
with ervth
aplastic crisis are ill with fever
, and h , nfectlosum only, patients with
'
^ S enS anc symPtoms of profound
anemia . Rash is rarely present . The
is shorter than that for erythema
coincident with the viremia .
incub

198.(C) Older children experience burning


inf * * * ^!?
1 Per od
'
*
for transient aplastic crisis
ntectiosum because the crisis occurs
, tinglin •
as long as 24 48 hr ) before the development
- tching, or pain 3-6 hr (rarely
'
of the h
erPes lesion.
355
19«

;
H

>
.f

al
nuinber
)

acyclovir
or
of
.
typically
slightly
Shows a

decreased
moderate number of
glucose
m
protein
concentration, and often
erythrocytes. Patients should be
"
r
-
CerebrosP
concentration, a
promptly treated with i
ra moderate
mtravenous
ntal

*
l fluid
cells and

200.1CI infants whose mothers demonstrate varicella in the period from s „ $


prior to delivery to 2 days afterward are at high risk for severe varice
mother has not yet developed a significant antibody response Z1 V

^
yZIG as soon as possible after birth. If VZ G .
tash usually occurs toward the end of the 1st wk . Those infants should
is not available,
immunoglobulin (1VIG ) may provide some protection. Acyclovir
when lesions develop. Vaccine is given to healthy children ( not
The in
reco ^
il
intravenoul
should be used
neonates) within
3-5 days after exposure.
201.(E) The characteristic cutaneous lesion has been called a cicatrix, a
zigzag
scarring, in a dermatomal distribution (zoster-like distribution ), often
associated
with atrophy of the affected limb.
202.(D) Generalized lymphadenopathy is present in 90% of cases; it occurs
most commonly in the anterior and posterior cervical nodes and the
submandibular lymph nodes and less commonly in the axillary and inguinal
lymph nodes. Epitrochlear lymphadenopathy is particularly suggestive of
infectious mononucleosis.
203.(C) EBV is associated with lymphoid malignancies, such as Burkitt
lymphoma, Hodgkin lymphoma, aggressive NK cell leukemia, T- and NK cell
lymphoproliterative disorder, and epithelial cell malignancies such as
nasopharyngeal carcinoma and gastric carcinoma.
204.(E) Roseola infantum (exanthem subitum, or sixth disease ) is an acute, self
limited disease of infancy and early childhood. It is characterized by the abrupt
usua y
onset of high fever, which may be accompanied by fussiness. The fever
over a day ysis
resolves acutely after 72 hr ( crisis) but may gradually fade
, nonpruri ic ,
coincident with the appearance of a faint pink or rose - colored
mm morbilliform rash on the trunk.
,
206.(B) Typically, the first sign of infection in in an

356
“ iS
^ IsTbl
pneumonia , there may be a history of conju
. With C. trachomatis
207 61
(
tends to be or v
mav'be prominent illness associated with the ^
js usually absent.
wheezin8
; ^ is

-
« a
*
208 '
conjun ctivitis
.ruses Other comm on jfestations
^
are upper and lower resp,
rit

r s Hemorrhagic occurs cystitis


d '. ,
frequen W k voung
in males
^* ^, ^
and w
memngoenc
or
r, r e e
*
occurs “frequently
^
less
jtjs
! !

mj|d t0 moderate fever

,

as m

^ ^sis? - ^
Rotavirus infection
Vomiting , followed by the 0 frequent, watery stools. Vorrud
of illness, but d
arjea oftw
srjuiii" * «* »
secondary goal is maintenance --r^ — - - —.
Antibiotics are of no beneflt. The Pj
probiotic organisms such as

^ ^
Lactobacillus species has been
dehydrating disease. Antiemetics ^, on|y in mild cases and not
he|p a |eviate vomiting but in this case ’
was stopped early in the course of disease.
210.(D) During the 10- day observation period, at the first sign of rabies in the
biting dog, cat, or ferret, treatment of the exposed person with RIG (human)
and vaccine should be initiated. The animal should be euthanized immediately
and tested.
211.(C) Microscopy cannot differentiate between E. histolytica and E. dispar
unless phagocytosed erythrocytes ( specific for E. histolytica ) are seen. The
shape and size of trophozoites and cysts of both are identical and difficult to
differentiate .
212.( B) Giardia is a particularly significant pathogen ini children with
malnutrition and certain immuno
i deficiencies ( IgA deficiency, common variable
immunodeficiency, X -linked hypogammaglobulinemia )
213.(E) In patients with visceral leishmaniasis,
smears or cultures of material
from splemc bone marrow, or lymph node aspirations
c usually
are u:> ud y u diagnostic. In
experienced hands, splenic aspiration hae , u - . . ,"
8 ^ diagnost c
immunochromatographic strip test usinp
diagnostic sensitivity and specificity for VL of
214.( B ) Incubation period is the longest with D "
ant gen ( K 39 h3$ 3

'

3 d 95% resPective y‘ *, .
days)-
Periodicity of paroxysms is less aooarpne malariae species ( 18- 40
aMM«irent with p
. falciparum. Plasmodium
knowlesi malaria is most often uncomplicated
but Can lead to severe malaria

357
^fjV^'*
,S
Nephrotic syndrome is a rare complication of P

nst 6r0 S '


w Mter h

uine ISused t0
*n^>«*,. » *
,
eradica e 3nY hypnozoites
malaria, , , .
woriy ism;
tlAl Prin a<1

£ ’ ^ e and thus prevent relapses,


that may remain
,
Sion * '
ant in in P. vivax and P
ven once daily for 14 days. Testing for glucose ova e
>- It is B -6-phospha e
iydrogenase deficiency must be performed before
initiation of primaquine
“Lause it «n cause hemolVtic anemia in such patients. ’

.(0) in areas where chloroquine-resistant P. falciparum exists, atovaquone-


217
is generally recommended for shorter trips ( up to 2 wk ) because it
Proguanil
jnust be taken daily . For longer trips, mefloquine is preferred, since it is given
pflly once a week, but it should not be given to children if they have a known
hypersensitivity to mefloquine, are receiving cardiotropic drugs, have a history
of convulsive or certain psychiatric disorders, or travel to an area where
mefloquine resistance exists . Doxycycline is an alternative for children >8 yr old.
primaquine is a daily prophylaxis option for children who cannot tolerate any of
the other options but it is contraindicated in children with 66PD deficiency,
Sulfadoxine-pyrimethamine is given to infants.
218.(E) T. gondii is estimated to cause 35% of cases of chorioretinitis.
for
219.(C) Calcifications occur throughout the brain, but there is a propensity
ganglia, choroid
development of calcifications in the caudate nucleus and basal
plexus, and subependyma .
of the parasite and
220.(D) Piperazine citrate causes neuromuscular paralysis
choice for intestinal or
rapid expulsion of the worms; it is the treatment of
through a nasogastric tu .
biliary obstruction and is administered as syrup
and
221.(D) Chronically infected children with moderate
in iron e i
infections suffer from intestinal blood loss that results
lead to anemia as well as protein malnutrition.
222.(E)
.
223 (B)
224 .( A) - .
Hktineuish C. trachomatis
wheezing he P ®
225.(0 The absence of fever and d stinctive laboratory
10
" "
pneumonia from respiratory syncytial
^
virus P
eosmop ce||s/(lL), The most
finding is the presence of peripheral accompanied by
hyperinflation
consistent finding on chest radiograph
is

minimal interstitial or alveolar infiltrates.


35 B
Chapter 17
The Digestive System
Questions
KHALID ALAARJI
cause of premature loss of primary teeth ?
1 what is the MOST common
A , Malpositioned teeth
6, Supernumerary teeth
C. Hypothyroidism
D. Cleidocranial dysplasia
£. Premature eruption of the permanent teeth

2. What is the expected time of eruption of first mandibular permanent molar


teeth?
A. 4- 5 yr
8. 6- 7 yr
C. 8- 9 yr
D. 10-11 yr
E* 12- 13 yr

3. Which of following is a recognized cause of severe brownish discoloration of


primary teeth ?
A. Hyperbilirubinemia
B. Tetracycline
C Osteogenesis imperfecta
D. Porphyria
E . Fluoride

4. A healthy full-term boy found to have cleft palate . Your advice about the
preferred time of surgical closure is before age of
A , 6 months
8. 12 months
C . 18 months
D . 24 months
E . 30 months
359
»

following is the MOST cariogenic


of the sugar associated
5- with dental
car'eS ? Sucrose
^0.
Glucose
c Fructose
0. Lactose
E . Galactose

6 . A 4 - year -
old child presents to out - patient clinic with
recurrent painful oral
lesions. Examinati on shows a well - circumscribed ulcerative lesion in buccal
mucosa with a white base surrounded by a red halo.
Of the following, the BEST palliative therapy is topical application of
A. nystatin
8. steroid
C. acyclovir
0. tetracycline
E. gention violent

.
7 A 3-year - old
child presents with history of fever, malaise, and difficult oral
intake. Examination shows cervical lymphadenopathy with clusters of
erythematous small vesicles involving oral mucosa and perioral skin.
Of the following, The MOST beneficial therapy is
A. topical steroid
B. oral acyclovir
C. oral nystatin
0. oral co- amoxoclav
E. topical lidocaine

8 What is the BEST position of plain radiograph for a newborn with suspected
tracheoesophageal fistula ?
A. Prone
B. Supine
C. Left lateral
D. Right lateral
E. Semi- sitting

360
newb orn presents with respiratory distress -
9. A 24-hour -term
frothy secretion from the
rornin mout
emvu .
.
h., 02 saturation 83% at room air ni • ^
" esopha'gea>l
.-
,v6raph
radiog
lau r.. show
- airless abdomen
' ; you
MOST likely type
are
is
suspe cting

es« fty
Of the following, the
A. Htype
to the distal and proximal ends
B. EA with fistula connected
C. pureEA
D. blind proximal end and fistula
connected to the distal end
to the proximal end
E. blind distal end and fistula connected

10 . A 3-year-old boy presents with low grade fever


and large volume bloody
diarrhea , he ate a sandwich of hamburger before 4 days.
Of the following, the MOST causative pathogen is
A. Salmonella
B. Shigella
C. Campylobacter
D. Antamoeba histolytica
E. Enterohemorrhagic E. coli

11. A 9-year- old boy presents with regurgitation, difficulty in swallowing and
weight loss; barium fluoroscopy show bird's beak sign
Of the following, the MOST likely diagnosis is
A . repaired oesophegeal fistula
B. GERO
C. acquired pyloric stenosis
D. eosinophilic esophigitis
E . achalasia

12. A 10-year-old boy, a kno

,
-^ rrnc
f' ^.
0f he following, the Most ,
sophagea r
°
eCldyj
es ph t *
c- hiatal hernia ° * '
C
,
rh,s otogiCai studv

^
nosisjs
V‘
8E' £ndosc<W shows food
'

H Vpe esoPf ogeal atresia


'
achalasia ’
361
** Ce of pain. Examination
« BEST «» ha
*
^7 observesteroid
start
in hospital
.
shows excessive drooling
and oralb *
^
,
dlff CUltV

6
c discharge and follow up after 1 week
swallow
0 do barium
upper endoscopy
E arrange for

14. A 14-month old child


- presents to emergency unit with history of choking
,
coughing, excessive salivation, and refusal to drink . The AP view of chest
radiograph shows a coin facing you and the lateral view shows the edge of coin.
Of the following, the MOST likely place of coin impaction is
A. larynx
0. trachea
t. esophagus
D . carina
E . right bronchus

15. A 5- year-old boy presents with sudden onset of severe nausea, vomiting,
and abdominal cramps within 3 hours of ingestion of unrefrigerated meats and
potato, followed by watery diarrhea .
Of the following, the MOST likely causative pathogen is
A . Salmonella
B. Shigella
C . Staphylococcus aureus
0 , Campylobacter
E - Shiga toxin producing E . coli
-

16. Accidentally esophageal ingestion of button batteries, must


removed within
A . 2 hr
B. 4 hr
C. 6 hr
D. 8 hr
E. 12 hr

362
month - old infant evaluated for failure to thrive; history revea(
17 . A 9 -

vomiting and arching his back


during feeding with recurrent hospfc
.
, lsn
wheezing and shortness of breath
following , the MOST likely diagnosis is
Of the
A. intuscessption
B. gastroesophageal reflux disease
C. asthma
D. H-type esophageal fistula
E. achalasia
risk of pyloric stenosis if
18. Which of the following has the highest ft

neonatal period?
A. Cefexime
B. Erythromycin
C. Ampicillin
D. Co-trimoxazole
E. Co- amoxoclav

positive
19. Asymptomatic high risk child for celiac disease (CD) who has
CD is
serologic screening tests and severe mucosal damage consistent with
regarded as
A . celiac disease
B. silent celiac disease
C. latent celiac disease
D. potential celiac disease
E. non-celiac gluten sensitivity

20. A 5 -year - old child under evaluation for short stature; examination shows
pallor, abnormal teething, and wasting of thigh and buttock - CBC reveals |Ht>
7gm/dl), serum ferritin ( 9 mg/dl) and celiac screen reveals anti -TG2 (90 U/mil
but EMA and HLA DQ8/DQ2 not available .
Of the following, the MOST appropriate step of management is to
A . search for other diagnosis
B. continue normal diet with foJ / ow up serological testing
C. start gluten- free diet
D. do duodenal biopsies
E. regard as non- celiac gluten sensitivity

363
child of first - degree
ASV^Pitomatic relative ceViac
;
sitive
.HLAD
11 A
Q 2 and DQ 8 and TG 2 Titer
W following, the MOST appropriate step of
( 350 u/mL) disease patient with
of the normal diet with
management is
further serological testing to
A continue
g continue normal diet till became symptomatic
c send for EMA
0. do esophagogastroduodenoscopy and biopsies
E . start gluten-free diet

22. Which of the following is the MOST common extraintestinal


of cel iac disease?
manifestation
A. Iron-deficiency anemia
B. Short stature
C. Enamel hypoplasia
D. Aphthous stomatitis
E. Peripheral neuropathies

23. Which of the following diets should be avoided in child with celiac disease?
A. Soybean
B. Rye
C. Corn
D. Buckwheat
E. Maize

24. What is MOST likely causative pathogen of diarrhea after consumption of


undercooked eggs ?
A. Shiga toxin-producing E. coli
B. Bacillus cereus
C. Staphylococcus aureus
0 - Listeria
Salmonella
25. The Codex Alimentarius Guidelin food, are those
es define glute
containing gluten less than
A - 20 ppm
B . 30 ppm
C. 40 ppm
.
D SO ppm

364
E. 60 ppm

for Health and Care


26. According to the National Institute
Guidelines, for which of the following conditions celiac testing
recommended ?
A. Reduced bone mineral density
B. Unexplained iron deficiency
C . Dental enamel defects
D . Down syndrome
E. Unexplained peripheral neuropathy

27. A 14- year-old child under evaluation for melena; gave history of 2 months of
intermittent epigastric dull pain, sometimes awaking him at night.
Of the following, the MOST appropriate test for diagnosis is
A . urea breath test
B. stool antigen test for H . pylori
C. upper endoscopy
D. serology for H. pylori
E . abdominal ultrasound

28. What is the BEST time to repeat stool


test for H. pylori antigen after
complete eradication therapy ?
A . Immediately after stopping the
therapy
B. 1wk after stopping therapy
C. 2 wk after stopping therapy
D . 4 wk after stopping therapy
E . No need for the test

29- A 13- year-old boy present

palpab|e mass . examination shows


nosuni - '
Of the following, the Most .. and a a< examination
"
,

gnosis
A - infectious colfe
B - ulcerative colitis
*
C. Crohn' s disease
D . Henoch-Schonlein purpu
E. Behcet disease
365
30-
de
vClopment*
T and 'weight below 3" centile small testicular
'
JJi ^^
1
,
d Wn d
from anus.
unjient drainage
, the MOST likely diagnostic tool is
fthe following
^ sweat test
^0 growth hormone assay
Q colonoscopy with biopsy
0 . sex hormone study
£, TTGAbs

31. Which of the following is the MOST sensitive and specific marker for
diagnosis of inflammatory bowel disease?
A. al- Antitripsin
B . Calprotectin
C. Steatocrit
D . Elastase
E . Chymotrypsin

32. A 15 - year - old girl who' s known case of ulcerative colitis presents with high
grade fever and hematocheiia for last 7 days; examination shows pale, thin
girl
with leg edema . Investigations reveal; Hb (8gm/ dl), WBC count ( 23000/
mm ) .
platelate count (550000/mm 3 }, and serum albumin ( 2.5gm /dl ).
Of the following, the first line of treatment is
A. probiotics
B. oral sulfasalazine
C . oral azathioprine
D . IV corticosteroid
E . colectomy

tend to occur MORE


33 . which of the following extraintestinal manifestations
.
commonly with ulcerative colitis than with Crohn disease
A . Oral aphthous ulcers
B . Chronic active hepatitis
C Peripheral arthritis
D Erythema nodosum
,

E . Digital clubbing
366
of the following is used as a conservative management ,n
34 . Which surgical candidates ? PVfer
who are not good *
stenosis in patients
A . Ondansetron
B . Atropine sulfate
C. Hyoscine bromide
D . Domperidone
E. Omeprazole

montfv old infant presen ts to emergency department with recurrent


35. A 12 - .
colicky abdom inal pain.
attacks of bilious vomiting and
is
Of the following, the imaging test of choice
A . CT abdomen
B. abdominal ultrasound
C. abdominal plain film
0 . upper gastrointestinal series
E . barium enema

36. The hallmark of duodenal atresia is bilious vomiting and


A. history of polyhydramnios
B. Jaundice
C. no abdominal distention
D. visible peristalsis
E. bile stained amniotic fluid

37. Which of the following should be avoided in the child with acute diarrhea ?
A. Yogurt
B. Wheat
C. Rice
D. Fruit juice
E. Potatoes

38. An 18 - month- old female presents with recurrent attacks of painless rectal
bleeding; she had previous history of intussusception at age of 9 month treated
by pneumatic reduction; clotting profile and blood film are normal apart from
mild anemia .
Of the following, the MOST likely diagnosis is
A . Meckel diverticulum
B. intussusception
367
dysentery
C amoebic
0 fissure in ano
C.
E rectal polyp
Wtiat is the typical abdominal radiograph findiing of
39
, meconium ileus?
A. Gasless abdomen
B. Double-bubble
C. Multiple air-fluid levels
D. Large, dilated stomach
E. Ground glass appearance

40. Malrotation is incomplete rotation of the intestine during fetal


development, which of the following is the MOST commonly affected part ?
A. Duodenum
B. Jejunum
C. Ileum
D. Colon
E. Cecum

41. Which of the following metabolic abnormalities may cause paralytic ileus?
A. Hyponatremia
B. Hypocalcemia
C. Hypomagnesemia
D. Uremia
E. Alkalosis

, chemotheraPV drugs may cause paralytic


ileus ?
A. Vincristine *
0 0Wing

Methotrexate
!;D 6C rnercaPtopurine
• ’
splatin
'
'
E - cVclophosphamide
, failure
43. AQ jg d child under evaluatation for
abdominal distention
to medical
to thrive 0 °
"

*
COnstipation starting in infancy with a poor response
with explosive passage
rectum
management Rectal examination shows empty
of stool
tool is
ic .
owing, the MOST appropriate diagnostic
368
A . abdominal ultrasound
B . plain radiograph of abdomen
C anorectal manometry
D . unprepared contrast enema
E. CT of the abdomen

44 . Constipation is defined as a delay or difficulty in defecation that


distress to the patient with duration for more than cauve*,
A. 2 weeks
B . 4 weeks
C . 6 weeks
D. 8 weeks
E . 12 weeks

45 . A mother of a 10 month- old infan


-
t , who recently underwent hydrostatic
reduction of intussusception, asks you
if there is a risk of recurrence. You
informed them that the recurrence rate
is approximately 10% and mostly within
A. 3 days of reduction
B. 7 days of reduction
C. 14 days of reduction
D . 21 days of reduction
E . 28 days of reduction

46. A 16- month - old child

01
rsnrs"sr “ "“"
B. upper endoscope
l
MS
C. CT of abdomen
D. ultrasound of abdomen
E. barium enema

47. A 3- year - old child


received treatment
presented with severe with clindamycin
watery diarrhea for osteomyelitis /

abdominal pain. containing mucus,


fever, and crampY
Of the following, the FIRST line
therapy is
A . IV metronidazole
B. oral fidaxomicin

369
c probio tic
0ral
vancomycin
D
E. IV bezlotoxum
ab

48. Which
of the following antiemetic medications is more suitable to be used in
children ? zine
A . Prochlorpera
B. Cyclizine
C . Metoclopramide
D . Domperidone
E . Ondansetron

49 . A 4- year-old child presents to emergency unit


with seizure, high grade fever ,
and bloody diarrhea .
What is the FIRST line antibiotics therapy ?
A. Ciprofloxacin
B. Cefixime
C. Trimethoprim-sulfamethoxazole
D. Metronidazole
E. Ampicillin
is infectious in small inoculums ?
50. Which of the following enteropathogens
A. Cholera
B. Shigella
C . Nontyphoidal Salmonella
D. Enterotoxogenic E. coli
E. Campylobacter
in children ?
viral caus e of acute gastroenter
51. What is the MOST com mon
A. Rotavirus
B. Norovirus
C. Sapovirus
D. Adenovirus
E. Astrovirus vomiting
en
Qf
ons t severe nausea ,and
followed by
- old boy prese nts with sudd cooked rice
52. A 6 - year
within 2 hours of ingestion of
improperly re nger
water y diarr hea .
370
causative pathogen is
of the following, the MOST
A . Salmonella
B. Shigella
C. Campylobacter
D. Bacillus cereus
E . Shiga toxin- producing E. coli

cancer syndromes has a


S3 . Which of the following inherited colorectal HIGHER
risk of colon cancer ?
A . Juvenile polyposis syndrome
B . Peutz -Jeghers syndrome
C. Familial adenomatous polyposis
D. Lynch syndrome
E . Cowden syndrome

54. A 6- year - old boy is under evaluation of recurrent attacks of painless rectal
bleeding mainly after defecation with mild abdominal pain . Examination shows
dark brown macular lesions around the lips and oral mucosa; endoscopy reveals
multiple polyps primarily found in jejunum and ileum.
Of the following, the MOST likely diagnosis is
A . Juvenile polyposis syndrome
B. Peutz - Jeghers syndrome
C . Familial adenomatous polyposis
D. Gardner syndrome
E . Cowden syndrome

55 . A 10-year - old boy with hepatic


failure presents to emergency unit witt1

Of the following, the MOST likely stage


of hepatiIC encephalopathy is
A.
B . II
C. III
D . IVa
E . IVb
56
^ °Ciated With the hVP°plasia or
aS5

abse.nceCystic fibrosis
B sickle cell anemia
Q Gilbert disease
Q Kawasaki disease
E Henoch- Schonlein purpura
diarrhea is defined when the diarrhea lasting more than
57. Persistent
A. 2 wk
B. 4 wk
C. 6 wk
D. 8 wk
12 wk

58. Which of the following is characteristic of toddler's diarrhea ?


A. Painless passage of stool
B. Failure to thrive
C. Perturbed appearance
D. 2 or more wk duration
E. Nighttime defecation is usually present

cause of chronic
59. Which of the following pathogens is the MOST common
infectious diarrhea in developing countries ?
A. Shigella
B. Rotavirus
C. Enteroadherent E . coli
D. Campylobacter
E. Salmonella
ia for abdom inal migraine ?
60. Which of the following is ONE of diagnostic criteria
A. Usually less than 1 hr in duration
B. Mainly epigastric
C. May be associated with photophobia
D. Episodes are separated by days
E. Pain does not interfere with normal activiti
es

372
-old girl presents with sudden onset of repeatM
61. A -7 year
for the last 48 hr with no
, and pho toph obia
vomiting, pallo r
; she has had 5 same attack • SlBnifir
.
of othe r syste ms
abnormality in review
completely normal between attacks; her ther otw
months and she is
° ‘
from migraine.
of the follow ing is the BEST abortive medication of this condition?
Which
A. Ondansetron
B. Triptans
C. Lorazepam
D. Diphenhydramine
E. Chlorpromazine

. Whic h of the follow ing radio grap hic positions is the BEST to determine tl*
62
type of imperforated anus ?
A. Upside - down x -ray
B. Prone cross table lateral plain x -ray
C. Erect x -ray
D. Lateral decubitus x - ray
E. Supine plain x - ray

malformations
63 . Which of the following are the MOST commonly associated
with imperforated anus ?
A . Tetralogy of Fallot
B. Vesicoureteric reflux
C. Tethered cord
D. Tracheoesophageal fistula
E . Spina bifida

64 . What is the liver span of a 12 - year - old girl ?


A . 4.0- 4.5 cm
B. 5.0- 5.5 cm
C. 6.0 6.5 cm
-

D . 7.0 7.5 cm
-

E. 8.0- 8.5 cm

65 . Which of the following is TRUE regarding pruritus of chronic liver dise^5 ^


A . Common in trunk
ing
B. Usually worse at morn
3 73
*
C Exacerbated with stress
D. Unrelated to the temperature
' the MAIN area of spider a„
P er an
66 What is
A. face '°mas distribution?
8. Abdomen
C. Back
D. Thigh
E. Hand

67. Which of the following is the MOST


valuable procedure in the evaluation of
neonatal hepatobiliary diseases?
A. Abdominal ultrasound
B. Hepatobiliary scintigraphy with
technetium-labeled iminodiacetic acid
derivatives
C. Endoscopic retrograde cholangiopancreatography
D. Magnetic resonance cholangiopancreatography ( MRCP )
E. Liver biopsy

68. A 2-year-old child under evaluation of failure to thrive, recurrent episodes o


cholestasis with elevation of serum aminotransferase, lka 0S

^ ^
and bile acid levels associated with lymphedema of the lower limbs, between
these episodes, the patient is asymptomatic with normal biochemical indices.
Of the following, the MOST likely diagnosis is
^
A. Zellweger syndrome
B. Alagille syndrome
C. Neonatal hemochromatosis
D. Aagenaes syndrome
E. Byler disease
„ht their 2- month-old infant because of
69. A first cousin parents brough
prolonged jaundice , generalize w '
Examinatjon rcveals high forehead
and epicanthal folds; m
lace
^
',
flattened , planting pa P
tomega|y; LFT shows conjugated
hypotonia, cataracts,
a u trasound reveals renal co ica
hyperbilirubinemia of the patella and grea er
,
/

calcifications
radiograph shows *
Of the following, t e
tp
,
qsT ike|y diagnosis is
374
A. Zellweger syndrome
B. Aagenaes syndrome
C. Byler disease
0. neonatal hemochromatosis
E. familial hypercholanemia

70. A 3- week - old infant presents with prolonged neonatal jaundiCfi


recurrent attacks of hypoglycemia; O/ E hepatomegaly with no dys
features. Laboratory finding shows blood sugar ( 42 mg/dLJ . TSB ( ig mg/
^^ ^
13 mg/dL is direct ), ALT ( 140 U/ L, AST ( 122 U / L ), serum albumin ( 2gm/ t )

^^
ferritin ( 650 mg / dL ), PT ( 28 ), PTT (58 ), and JNR ( 3.5 ). Family history of {
died with same presentation at the neonatal period.
Of the following, the BEST method to confirm diagnosis is
A. ultrasonography
B. neonatal screening test
C. buccaJ mucosal biopsy
0. liver biopsy
E. al- Antitrypsin phenotype

71. A 6- week - old infant under evaluation of cholestasis. Examination reveal


triangular face; broad forehead, deep- set eyes, prominent ears, straight notf
with bulbous tip, and pointed chin. Echo study shows peripheral pulmon
artery stenosis. Radiograph of the spine shows "Butterfly ” vertebrae. Lab
^
findings; TSB ( 13 mg/dL with 10 mg/dL is direct ) and cholesterol level
mg/dL ) .
Of the following, the MOST likely diagnosis is
A. Zellweger syndrome
B . AlagiJIe syndrome
C . Neonatal hemochromatosis
D. Aagenaes syndrome
E . Byler disease

f
72 . In biliary atresia, the success rate for Kasai operation is much higher (90 )
performed before
*'
A . 4 wk of life
6 wk of life
C . 8 wk of life
D . 12 wk of life
375
l J6
wit oflife

-old child with chronic cholestasis and


A 3-year

A hepatic
encephalopathy
g Friedreich's ataxia
C. vitamine E deficiency
D. zinc deficiency
E. vitamine A deficiency

74. Which of the following is the MOST common presenting


manifestation of
Wilson disease in children?
A. Jaundice
B. Kayser - Fieischer ring
C. Intention tremor
D. Hemolytic anemia
E. Depression

75. A 6-year -old boy presents to emergency unit with severe epistaxis and
abdominal distension for 2 days. There was no history of trauma or bleeding
Irom other site and normal bowel motion. Two previous siblings had died at 6
and 8 years of age with jaundice and abdominal distension of unknown cause.
0 / E stuporous, severe icterus, ascitis, hepatomegaly 3 cm, and splenomegaly 3
cm. Lab finding reveals Hb 10,8 gm/ dl, WBC and platelate counts within normal
limits, TSB 18.0 mg/di mainly direct, SGOT 112 IU/ Lr SGPT 22 IU/ L, ALP 28.4 IU/ L,
PT 1 min, PTT 2 min, INR 3, and hepatitis viral study was negative .
Of the following, the MOST appropriate initial test is
A. serum ceruloplasmin level
B. serum free copper level
C. urinary copper excretion
D . liver MRl
E. liver biopsy

for diagnosis
76. Which of the following is the method of choice
disease?
A . Decreased serum ceruloplasmin level
376
c. i«SXur
Increas
, m free copper level
na
^ Wperr «ior
loyser- fleischer rings
Demonstra
D. hepatic parenchymal copper concentration
E. Measurem ^
77. Which relatives
asymptomatic first-degree
A . Liver function tests
8. Serum ceruloplasmin
level
C. Serum copper level
D . Liver biopsy
E. Genetic study

of the following diets should be AVOIDED in patients with Wife


78. Which
disease ?
A. Rice
B. Chocolate
C. Tea
D . Eggs
E . Fresh tomatoes
therapy
79, Which of the following agents is MOST likely used as adjuvant
patients with Wilson disease ?
A . Vitamin E
B. L-carnitine
C Zinc
D . Medium chain triglyceride
-

E , FoJic acid

80. Which of the following therapies is MOST likely improving the outcome of
patients with neonatal hemochromatosis ?
A. Intensive phototherapy
B. Exchange transfusion
C . Disferal
D . Vitamine E
E . L carnitine
-

377
recovery <of acute viral hepatitis A infection, the liver
th morphology
W
1
*retu 's to normal within infection
the acute

^
c
9
'

mo
3 mo of
4 mo of
the acute infection
6 mo of the
of
acute infection
the acute infection
D infection
£ 12 mo of the acute
mother asks you about the time of return nr u
8 2 - A concerned
her
child with acute
hepatitis A viral infection
Ct n tto
° , V -
0 schoo> the BEST time is

approximately
A . 3 days after the onset of jaundice
B. 7 days after the onset of jaundice
C. 10 days after the onset of jaundice
D. 14 days after the onset of jaundice
E. 21 days after the onset of jaundice

83. Which of the following is a marker of infectivity ?


A. HBsAg
B * Anti-HBc
C. Anti-HBs
D . HBeAg
E . anti-HBe

84 , The serological autoantibody profile defines 2 main types of autoimmune


hepatitis ( AIH ) ; type 1 and 2 . Which of the following autoantibodies is positive in
AIM type 2 ?
A , Anti-nuclear antibodies {ANA}
Anti-smooth muscle antibody { SMA )
C Antineutrophil cytoplasmic antibodies
D . Anti-liver kidney microsomal type 1 antibody ( anti - LKM l)’

E . Antiactin antibody

85 . A 14- year - old girl under evaluation of Insidious onset of fatigue, anorexia ,
malaise, joints pain, rash with secondary amenorrhea and behaviora c
lor the last 4 months . Examination shows jaundice, palmar erythema, sig. tV
enlarged tender liver with enlarged spleen. Serum aminotransferase 1 f ,
TSB 3mg/ dL, normal ALP and GGT, elevated serum y-globulin levels, PT is
378
££ == ”
*25^ *
£££ r
A. Prednisone
B . Azathioprine
C . D- penciliamin
0 . Budesonide
E. Cyclosporine

___ hepatic failure experience serious infeaw-


86 At |east 50% of patients with
likely cause ?
which of the following pathogens is the MOST
A . Staphylococcus aureus
B. E . coli
C. Pseudomonas aeroginosa
D . Streptococcus pneumonia
E . Salmonella

87 . Liver failure in the perinatal period can be associated with prenatal


INF

injury and even cirrhosis . Which of the following conditions is MOST ccmnmor
cause of acute liver failure in the neonate ?
A. Gestational alloimmune liver disease
B. Galactosemia
C . Tyrosinemia
D . Familial hemophagocytic lymphohistiocytosis
E . Congenital herpes simplex infection

88. Which of the following electrolytes when decreased is a reflection of W


regeneration ?
A . Sodium
B . Potassium
C . Magnesium
D. Phosphorus
E . Calcium
CMpt£ LiZ The Digestive
System
Answers
KHAUD ALAARJI
UE) While the delayed eruption of the 20 primarv
h
indicate systemic or nutritional disturbances ?Lh “ be familial or

.
u

f
^° ^ arise
local causes such as malpositioned tee h supernumerary teeth, cysts
, or
retained primary teeth.
m
Permanent Dentition Eruption
Maxillary Mandibular
Central incisor 7 -8 yr Central incisor 6- 7 yr
Lateral incisor 8-9 yr Lateral incisor 7-8 yr
Canine 11-12 yr Canine 9-10 yr
First premolar 10-11 yr First premolar 10-12 yr
Second premolar 10-12 yr Second premolar 11-12 yr
First molar 6-7 yr First molar 6- 7 yr
Second molar^
12-13 yr Second molar 11-13 yr
Third molar 17-21 yr Third molar 17-21 yr
3*(E) Anomalies of Color: Discolored teeth can result from incorporation of
foreign substances into developing enamel. Neonatal hyperbilirubinemia can
Produce blue to black discoloration of the primary teeth. Porphyria produces a
red-brown discoloration. Tetracyclines are extensively incorporated into bones
and teeth and, if administered during the period of formation of enamel, can
result in brown-yellow discoloration and hypoplasia of the enamel. Sue
fluoresce under ultraviolet light. The period at risk extends from approxima e y
4 mo of gestation to 7 yr of life. Repeated or prolonged therapy wi
tetracycline carries the highest risk. Excessive fluoride during ename °r a °
affects ameloblastic function, resulting in inconspicuous white, acy pa c
the enamel to severe brownish discoloration and hypoplasia.
.
4 (B) Surgical closure of a cleft lip is usually performed by 3 mo o age
, w en
ory, or
infant has shown satisfactory weight gain and is free of any ora , respira
380
healthy child, closure of
systemic infection. In an otherwise the Q
done before 1 yr of age
most
to enhance
cariogenic
normal
sugar
speech
because
develop
one
,,
** ^ *
5 (A ) Sucrose Is the of its by-
Pf
bacterial metabolism is glucan , a polymer that enables bacteria to «kts V.
readily to tooth structures. Dietary behaviors, such as consuming . *
^
beverages in a nursing bottle or
frequently consuming sticky
cand
the cariogenic potential of foods because of the long retention of * sJN
mouth.
6.(B) The aphthous ulcer lesions generally last 10 14 days and heal
-

scarring. Nonprescription palliative therapies, such as berrzocaine and top


lidocaine, are effective, as are topical steroids. ^
7.(8) The symptoms of herpetic gingivostomatitis (herpes simplex virus} usua s
regress within 2 wk without scarring. Fluids should be encouraged because
'
child may become dehydrated. Analgesics and anesthetic rinses can make
^
child more comfortable . Oral acyclovir, if taken within the first 3 days of
symptoms in immunocompetent patients, is beneficial in shortening the
duration of symptoms. Caution should be exercised to prevent autoinoculaticr
especially of the eyes.
.
8 ( A) Prone positioning minimizes movement of gastric secretions into a distal
fistula, and esophageal suctioning minimizes aspiration from a blind pouch.
.
9 { C) Plain radiography in the evaluation of respiratory distress might reveal a
coiled feeding tube in the esophageal pouch and/or an air -distended stomach
indicating the presence of a coexisting TEF. Conversely, pure EA can manifest a:
an airless scaphoid abdomen In isolated TEF (H type), an esophagogram with
.
contrast medium injected under pressure can demonstrate the defect .
10 (E) Enterohemorrhagic Escherichia coli ( EHEC) including E coli0157:H 7 ana
. .
.
other Shiga toxin-producing E. coli ( STEC) Incubation period; 1- 9 days (usu

^
3-4 days). Clinical presentations are watery diarrhea that becomes bloody in 1

^
days in ~40% of infections; in contrast to dysentery, bloody stools are
volume and fever/toxicity are minimal. More common in children <4 yr old.
factors are food and water contaminated with feces from ruminants; infeCteb
people and animals ( fecal-oral); predominantly high-resource country
undercooked beef especially hamburger, unpasteurized milk and juice,
fruits and petting zoos, recreational swimming, daycare. Antimotility agents n
*
antibiotics increase risk of hemolytic uremic syndrome.
1ME) Achalasia manifests with regurgitation and dysphagia for solids and
liquids and may be accompanied by undernutrition or chronic cough; retained
esophageal food can produce esophagitis. The mean age in children is ft ® ' ^
381
- -
.

-
dura tion of 5yfT1
a mea n mo; it is uncommon

..
with

—- J'<sz«••*»s?
m 3 dilated

-
lower

s r^,*t o “»££sxs
•f
loss of primary

“-
poor emptying are
*
often
test and helps
h
differen
diagnosis of eosinophilic esophagitis should be considered in the
J2.(B) The
of esophageal dysfunction, associated with esophageal
clinical presentation of
epithelial infiltration at least 15 eosinophils per high power field. Many
( a ive y history) and associated
patients have other atopic diseases or posit famil
peripheral eosinophilia and
food allergies; laboratory abnormalities can include
elevated immunoglobulin E (IgE) levels.
hageal strictures. Absence of
13.(E) Twenty percent of patients develop esop
ficant
oropharyngeal lesions does not exclude the possibility of signi
esophagogastric injury, which can lead to perforation or stricture
. The absence
al lesions; hematemesis,
of symptoms is usually associated with no or minim
predicts severe lesions .
respiratory distress, or presence of at least 3 symptoms
most efficient means of rapid
An upper endoscopy is recommended as the
undertaken in all symptomatic
identification of tissue damage and must be
as acute treatment, but
children. Dilution by water or milk is recommended
lavage are contraindicated.
neutralization, induced emesis, and gastric
n bodi es lodge at the level of the cricopharyngeus
14.(C) Most esophage al foreig
sphin cter ), the aorti c arch , or just superior to the diaphragm
(upper esophageal
junct ion (lowe r esop hage al sphincter ). Evaluation of
at the gastroesophageal plain
histor y of foreig n body ingestion starts with
the child with a with lateral
radio grap hs of the neck , chest, and abdomen, along
anteroposterior in the esophagus is seen
The flat surfa ce of a coin
views of the neck and chest. later al view. The reverse is
and the edge on the
0n the anteroposterior view steriorly
trach ea ; here , the edge is seen anteropo
true for coins lodged in the
laterally.
and the flat side is seen (prefo rmed ente rotoxin). Incubation
aureu s
15 (C) Etiology; Staphylococcus
. sudd en onse t of severe nausea and
presentations are
period; 1-6 hr. Clinical and fever may be present Risk
cramps; diarr hea
vomiting and abdominal improperly refrigerated meats, potato and egg salad
s,
or
factors; Unrefrigerated
cream pastries. ple magn ets or single magnet with a
esoph agus , multi
obje cts in the respiratory symptoms
16. A( ) Sharp bodie s asso ciate d with
or foreign
metallic object , 382
mandate urgent removal within 12 hr of presentation. Button h
particular, must be emergency removed within 2 hr of presentation ^
* * ' *
of the timing of patient ' s last oral intake because they can induce mu
in as little as 1 hr of contact time and involve all esophageal layers
/^
17,(B) Most of the common clinical manifestations of esophageal H
^
signify the presence of GERD and are generally thought to be mediaw
^
pathogenesis involving acid GER . Infantile reflux manifests more
regurgitation ( especially postprandially ), signs of esophagitis (
often
irritat/
^
arching, choking, gagging, feeding aversion ), and resulting failure to
th ^
^
symptoms resolve spontaneously in the majority of infants by 12- 24
mo Q ,
children can have regurgitation during the preschool years; this
comply
diminishes somewhat as children age, and complaints of abdominal
andch
pain supervene in later childhood and adolescence. Occasional children preser ^
with food refusal or neck contortions (arching, turning of head) designates
Sandifer syndrome.
18.(B) An association has been found with the use of erythromycin in
neonates
with highest risk if the medication is given within the first 2 wk of
life . There
have also been reports of higher incidence of pyloric stenosis among mostly
female infants of mothers treated with macrolide antibiotics during pregnancy
and breastfeeding.
19 (B)
Symptomatic Frank malabsorption symptoms and signs (e.g., chronic
diarrhea, failure to thrive, weight loss )
. Extraintestinal symptoms and
signs ( e .g ., anemia, fatigue, hypertransam
inasemia, neurologic disorder
short stature, dental enamel defects
, arthralgia, aphthous stomatitis)
i ent No apparent symptoms in
spite of histologic evidence
villous atrophy, in most cases
groups
identified by serologic screening in at - risk
^ Latent — Subjects who
have
.
other time have shown a ei ,rBn
Potential Suhio
intestinal histolo8y' but at 5
ten dependent enteropathy

"S'
evidence of altered intestin
symptoms and signs of
dependent enteropathy
disease
later
Non-celiac gluten sensitivity
^
histol f^
P
d may or
and

(Nrrc
tS
maV
"
not
y r “
° ^ .In
develop a gllJ

„tn0i
-
'
condition. Diagnosis is suspect,
wheat allergy, and yet show Gl *" *
~ iS a p00rly
Patients w “ < Tn
and n n ^°
do not have
f
gluten- or wheat-containing foodn °sVmptoms upon ingest 00 0
'
383
. vvhen antiresult
-TG 2 <10 x normal should
of biopsy
do Phagogas ,
sy; jf the is roduode^oscopy
/ —
0-1 Unclear case and consider: false
positive
negative biopsy or potential CO and extended serology or false
serology/biopsies evaluation of HU/;

/ Marsh 2 or 3 diagnosed as a celiac disease and start B|uren r
gluten sensitivity (NCGS)
NttKeliacis suspected in patients who do -
is a poorly
iet
understood condition.
gnosis not have CD or wheat allergy and uo,
-GI symptoms upon ingestion of gluten-
show Gl and non or wheat ccntainmg
food.
2L(D) Asymptomatic person at genetic risk for CD, HLA positive DQ2 and/or
DQ8, TG 2 Titer > 3 x normal should do EGD & biopsies from Buibus & 4 x
pars descendens, proper histological work up ( Marsh 0 or 1 or Marsh 2 or 3).
22.(A) One of the most common extra intestinal manifestation of CD is iron -
deficiency anemia, which is usually unresponsive to iron therapy . Osteoporosis
may be present; in contrast to adults, it can be reversed by a gluten- free diet ,
with restoration of normal peak bone densitometric values. Other
ertraintestinal manifestations include short stature, delayed puberty, arthritis
and arthralgia , epilepsy with bilateral occipital calcifications, peripheral
neuropathies, isolated hypertransaminasemia , dental enamel hypoplasia, and
aphthous stomatitis ,
.(B)
*^ Avoid all foods containing wheat, rye^ anc
barley gluten { p
*
usually safe ).

=--
c rn

x.
Avoid malt unless clearly labeled as derived from ° h quino3
_

' ££
nut flours. used
t ctarch should only be
Wheat starch and products containing w free. "
if they contain < 20 ppm gluten and are fo0 ds .
o pro additives,
^ Bead all labels and study ingredients
su
. merits, food
* Beware of gluten in medications,
jS evidence
of lactose
emulsifiers, or stabilizers.
if there
^ Limit milk and milk products initially
gluten free ) -
intolerance. ( unless labe
led
Avoid alt beers, lagers, ales, and stouts
Salmonella and Shigella (egg salad ).
384
. ,
.» C.o< c V(ncntaf oS 6ofoeVneS ®tine duten-free food
' '
^'°
15-
aO **
ied0 3 6
Metho
20
"'d*' «’? *
fo& 6 m \ kg o< ternt0,(

'^
c0ota i PP
or g\uten dete • n have ProduttV,ta ,
^ *
ca
naWt . reached a , *
V" « ' '°
1„ M' more n r „ s need d on the da\N gvten
CO an ,
The data 3 '
sotar seemto * *
**
that the thre
^ * oW
a unWe«a
«««
' 1thoughindividual
van ,
**
26. ( B )
f Celiac testing recommended
' Celiac testing should be
considered
Persistent unexplained abdominal or Metabolic bone disordersTreduced
gastrointestinal symptoms
Faltering growth
mineral density or osteomalacia )
Unexplained neurological tynptor
^
:

Prolonged fatigue (particularly peripheral neuropathy 3


Unexpected weight loss ataxia )
Severe or persistent mouth ulcers Unexplained subfertility or recurred
Unexplained iron, vitamin B 12, or folate miscarriage
deficiency Persistently increased concentrations
Type 1 diabetes liver enzymes with unknown cause
Autoimmune thyroid disease Dental enamel defects
Irritable bowel syndrome Down syndrome
First degree relatives of people with celiac Turner syndrome
disease William syndrome
Dermatitis herpetiformis
,
° lcante'Z? ^ ^
~~~~ ~ ~^~ Si?Pcv
^ ^
of £ c uiC ^
enosc
,
pyirTh ^ ~

^
J0

tric %£ ? * ** *" *

222 S [
pedi9

peT, ^
lesions. Biopsy >ec niens
1 dnd duodm
PY
ident
a ows the direct
fymg the specific
kf must dtair ed from tu 'esopk a’8 , stomach,ad
K
iogic as essm °
) ) U$
duodenun ) far
PVlori infection ' °
End
St
ak ^
$
* , as to °
S for the presence
SCreen
of H.

therapy includin ° ° Sc Py
Pr v des the
nd the *
° * for oPport ^
28 (C) Patients shCl'
Ppin

^ coagu/abon.
& injectio

as neZ
y *SUltS
^ p *
P
^ p 2 wk Phor K

H PWori eraSZ?
testing
d ,> ^ of
rePre Senth
^ eraW
ne atives- Success^
, ^ and veryW
risk of relapse . Theref
these patients 4-6 Wk af '
^^ ?^ ° fn nitorfn
8
W K CUre
Penti
°
Access “
cer disease
f theraPy is mandatoryir
5t pPine
° testl °
cs and at east 2 wk after stoppiM
or stool
-
PPI theraPV Eradication
antigen
be
test ^
^' *
^ th the 23C~Urea breath (13C-UBT) tesf

385
* * '**« >•to ,, ..
*^ ~

Sfjti w • »< «"»<««. •»«S ! ZZ g
2 !S
-
»
* and bloody stools it is * ** can
.
- - -* - ^
, as abdominal
^S£ pain PUrpura
. , not usuaiiv
h most difficult distinction is from Crohn d s
* b T with
«
*
Llopment of small bowel disease eventually leads to the correct diagnosis-
a, can occur
years after the initial presentation. Behcet disease can
distinguished by its typical features
signs and symptoms are more common in Crohn disease than in
30,(C) Systemic
ulcerative colitis. Fever, malaise, and easy fatigability are common. Growth
failure with delayed bone maturation and delayed sexual development can
precede other symptoms by 1 or 2 yr and is at least twice as likely to occur with
Crohn disease as with ulcerative colitis. Children can present with growth failure
in
as the only manifestation of Crohn disease. Decreased height velocity occurs
often
about 88% of prepubertal patients diagnosed with Crohn disease, and this
precedes Gl symptoms.
31|B)
/ Fecal calprotectin and lactoferrin are increasingly being
used as more
of bowel inflammatio n as compared to
sensitive and specific markers
.
serologic parameters, and these are often elevated
protein loss
/ al-Antitripsin -— Increased intestinal permeability/

^ Steatocrit — Fat malabsorption


^ Elastase —- Pancreatic function
Chymotrypsin — Pancreatic function
*
&.(D) Fever, severe anemia , hypoalbuminemia ia , leukocytosi s , and more than 5
.
bloody stools per day for 5 days define fulminant colitis remission inn for ulcerative
ce of
Probiotics are effective in adults for maintenan
• • „ rfurina an active flare . The first
jS an aminosalicylate
colitis, although they do not induce remissio -
drug class to be used with mild or mild- to- moderate ^ ^osteroj<j therapy **re
c
Many children with disease requiring frequent l g da or 6
started on immunomodulators such as
azathiopr n
0- 2
^
, 5 mg/ < /
^
an effective
.

are
mercaptopurine (l-l.Smg/kg/ day ). Steroids priate maintenanece
are
"
medication for acute flares, but they

for intractable disease, complications of


side
medications because of loss of effect andtherapy,
e
Colectomy is P
fulminant
^
disease
°
^
t a

unresponsive to medical management .


33*{B)
Extraintestinal manifestations that tend to occur
more
ulcerative colitis than with Crohn disease i
gangrenosum, sclerosing cholangitis, chronic active
ankylosing spondylitis .
S Extraintestinal manifestations occur more commonly with Crohn j
than with ulcerative colitis; those that are especially associated " '
Crohn disease include oral aphthous ulcers, peripheral
erythema nodosum, digital clubbing, episcleritis, renal stones furic
oxalate), and gallstones. ^
34.( B) Conservative management with nasoduodenal feedings is advisable T
patients who are not good surgical candidates. Oral and intravenous atropir
*
sulfate (pyloric muscle relaxant) has also been described when surgical
expertise is not available with 80% success rate described in some studies.
.
35 (D) The diagnosis of malrotatron may be suggested by ultrasound butt
confirmed by contrast radiographic studies. The abdominal plain film is usuai
nonspecific but might demonstrate a gasiess abdomen or evidence of duodenal ^
-
obstruction with a double bubble sign. Upper gastrointestinal series is the
imaging test of choice and the gold standard in the evaluation and diagnosis of
malrotation and volvulus.
36*(C) The hallmark of duodenal obstruction is bilious vomiting without
abdominal distention, which is usually noted on the first day of life. Peristal^
waves may be visualized early in the disease process. A history of
polyhydramnios is present in half the pregnancies and is caused by inadequate
absorption of amniotic fluid in the distal intestine. This fluid may be bile stained
because of intrauterine vomiting, jaundice is present in one-third of the infants
.
37 (D) Soda beverages, fruit juices, tea, and
other home fluids are not suitable
for rehydration or maintenance therapy
glucose concentration and
because they have inappropriately M
osmolalities and low sodium concentrations. F00
*
,
w th complex carbohydrates (rice, *
wheat, potatoes, bread, and cereals),
fruits, ear meats, yogurt, and vegetables
should be reintroduced while ORS
tf 1

rT fTOm emesis or stools and for maintenance


aSded Simple
^ rs (juices, carbonated sodas) shouW
*
38.(A) Symptoms of a Meckel diverticulum
usually arise in the first or second f
of life ( average: 2.5 .
vr but initial symptoms can
)
in the first decade. ^
majority of symptomatic Meckel diverticula are ,
occur
ined by ar> ectopie m
***
induding an acid- secreting mucosa that

387
causes intermittent Unless '
recta
or a
Ltrafced
^
bicarbonate. The stool is typical
by pancreatic

. jelly colored. Bleeding


colored or currant because
usua IV self limited
-
sign
can cause
of contraction of the
et
splanchnic veils
*
'

anemia but is
become hypovolemic Less often, a Meckel diverticulum is as Las?!
,
* 3cidas is1 not
brick
n
ial or complete bowel obstruction. The
^
obstruction occurs when the diverticulum
common
most
as the lead nZ
acts
v orT an t
intussusception.
».(£) In patients with obstruction caused by jejunoileal atresia or long segment
Hirschsprung disease, plain radiographs
typically demonstrate multiple air
levels proximal to the obstruction in the upright or -fluid
lateral decubitus positions
These levels may be absent in patients with
meconium ileus because the
viscosity of the secretions in the proximal
bowel prevents layering. Instead, a
typical hazy or ground glass appearance may
be appreciated in the right lower
quadrant. This haziness is caused by
small bubbles of gas that become trapped
in inspissated meconium in the terminal
ileal region.
40.( E) The most common type of
malrotation involves failure of the cecum to
move into the right lower quadrant.
41.(0} Ileus is the failure of intestinal peristalsi
s caused by loss of coordinated
gut motility without
evidence of mechanical obstruction. In children, it is most
often associated with abdominal surgery or infection ( gastroenteritis,
pneumonia, peritonitis). Ileus also
accompanies metabolic abnormalities (e .g.,
uremia , hypokalemia, hypercalcemia , hypermagnesemia, and acidosis) or
administration of certain drugs, such as opiates, vincristine, and antimotility
agents such as loperami
de when used during gastroenteritis.
42.(A)
43-(D) An unprepared
contrast enema is most likely to aid in the diagnosis in
children older than 1 mo of age because the proximal ganglionic segment might
not be significantly
dilated in the first few wk of life. Classic findings are based
n the presence
° of an abrupt narrow transition zone between the norma
dilated proximal colon and a smaller -caliber obstructed distal aganghomc
segment In the e
absence of this finding, it is imperative to compare the * *
of the rectum to
that of the sigmoid colon, because a rectal diameter
same as or smaller than the sigmoid colon suggests Hirschspru H
that
^is

ng disease
Radiologic evaluation should be performed without prior preparation (i.e.,
the
,

388
unprepared contrast enema study) to prevent
transient dil
aganglionic segment.
44.(8)
n
^ %
45.( A ) The recurrence rate after hydrostatic reduction of i
- lntussusce
approximately 10%, and after surgical reduction it is 2 5%; n ne
° ^
after surgical resection. Most recurrences occur within 72 hr 0f r rr
hac
^

^
Corticosteroids may reduce the frequency of recurrent intussusception
rarely used for this purpose.
46.(D) The diagnostic findings of intussusception on ultrasound i
appeal
tubular mass in longitudinal views and a doughnut or target
.
transverse images Ultrasound has a sensitivity of approximately 98
specificity of approximately 98% in diagnosing intussusception.
.
47 (D) Management of pseudomembranous colitis; Oral vancomycin
^
100% andi
^

ar
'
*

metronidazole for 7-14 days ( first line therapy) displayed equivalent efficacy tri*
prospective randomized trial .
48.( E) Ondansetron (oral mucosal absorption preparation) reduces
tit
incidence of emesis, thus permitting more effective oral rehydration and is well
established in emergency management of AGE in high resource settings,
reducing intravenous fluid requirements and hospitalization. Because persistent
vomiting can limit ORS, a single sublingual dose of an oral dissolvable tablet of
ondansetron ( 4 mg for children 4-11 yr old and 8 mg for children older than 11
yr [generally 0.2 mg/kg|) may be given. Antiemetic
agents, such as the
phenothiazines ( such as prochlorperazine ), are of
little value and are associate
with potentially serious side effects (
lethargy, dystonia, malignant
hyperpyrexia ).
49.(A)
First line :
Ciprofloxacin 15mg/kg/day PO
bid 3 days - OR
Ceftriaxone 50- 100mg/kg/day IV *
or IM, qd x 3 days for severe illness
requiring parenteral therapy;
OR
' Azithromycin 12mg/kg once
days 2 through 4 ( total
on 1st day then 6 mg/kg once daily n
course: 4 days) °
Second line :
Cefixime 8 mg/kg once daily
for 3 days; OR
v' Trimethoprim- sulfametho
SMX twice a day for 5 days '£ ,
V f suscePt bi , ° f Pa d2
" °
likely based 0
tV known or
local data )
'
389
properties of the organism help
intrinsic
** **JZ Sr “
,( 9)
50 incubation *
mission and (Shigella , period. m mode of
are infectious in
inocula 5 TEC, norovirus,
sinaM difficile, EH histolytica )
rotavirus,
, intestinal,
mptosporidiumsppcontactr
are read , V
" to- person via the fecal-oral transmitted by
person doses, such as cholera, NTS, route Pathogens with larger
ETEC, and Campylobacter
, generally
ire food or water vehicles.
is the most common cause of AGE
SUA) Rotavirus other among children throughout
0 world. Several viruses occur less frequently.
*
52.(0) Etiology; Bacillus cereus ( preformed emetic toxin). Incubation period 1-6
hr. clinical presentations are sudden onset of severe nausea and vomiting;
diarrhea may be present 24 hr Soil and water Risk factors; improperly
,

refrigerated cooked or fried rice, meats.


53.(0 Risk of colon cancer; Juvenile polyposis (10-50%), Peutz-Jeghers
syndrome (47-93%.), Familial adenomatous polyposis ( 100%), Lynch syndrome
(not increased), and Cowden syndrome ( 30%).
54.(B) Peutz-Jeghers syndrome ( PJS ) is a rare autosomal dominant disorder
(incidence: ^1:120,000 total population ) characterized by mucocutaneous
pigmentation and extensive Gl hamartomatous polyposis. Macular pigmented
lesions may be dark brown to dark blue and are found primarily around the lips
and oral mucosa, although these lesions may also be found on the hands, feet,
or perineum. Lesions can fade by puberty or adulthood.

* MC)
56.(A) The gallbladder is congenitally absent in approximately 0.1% of the
population. Hypoplasia or absence of the gallbladder can be associated with
extrahepatic biliary atresia or cystic fibrosis. Kawasaki disease and Henoch-
Schonlein purpura are associated with hydrops of the gallbladder. Sickle cell
anemia and Gilbert disease are associated with cholelithiasis.
day in
57.(A) Chronic diarrhea is defined as stool volume of more than 10 g/ kg/
lasts for 4 wk
toddlers/infants and greater than 200 g/day in older children that
14 days. In
or more. Persistent diarrhea began acutely but lasts longer than
practice, this usually means having loose or watery stools
more than 3 times a

ianheaf
58.{ A) The most benign and common etiology of
diarrhea that encompasses functional diarrhea (
children younger than 4 yr of age and irritable b
age and older . It is the leading cause of chronic diam
,
^
an otherwise well
in
those 5 yr of

pa $sage of 4
child. Toddler's diarrhea is defined by the daily P
390
or larger unformed stools, for 4 or more wk, with onset in infa
is usually absent. The child appears JX,
vears Nighttime defecation
the diarrhea, there is no evidence of failure to thrive, and the
sympl0 X '

spontaneously by school age.


, enteroadherent Escherichia colj
59,(C) In developing countries and
implicated in chronic diarrhea , whereas, jn
lamblta have been
countries, chronic infectious diarrhea
usually runs a more benign
coursedX
etiology is often viral, with a major role of rotavirus and norovirus.
60.(C) Diagnostic criteria for abdominal migraine must include an f
following occurring at least twice: ° tv
1. Paroxysmal episodes of intense, acute periumbilical, midline, or diffu
abdominal pain lasting 1 hr or more ( should be the most severe and distress *
symptom) ^
2. Episodes are separated by weeks to months
3. The pain is incapacitating and interferes with normal activities
4. Stereotypical pattern and symptoms in the individual patient
5 . The pain is associated with 2 or more of the following: a . Anorexia b. Nausea
c. Vomiting d. Headache e. Photophobia f. Pallor
6. After appropriate evaluation, the symptoms cannot be fully explained
another medical condition.
61.( B) Supportive Care and Abortive Treatment Approaches in Cyclic Vomiting
Syndrome
1- Supportive care
^ Fluid and electr lyte management
° —- Dextrose containing fluid (Dl®
and normal saline
^ Nutrition - Resume enteral

nutrition as soon as possible and if unab^
to tolerate enteral nutrition 1
and meets criteria, start parent ^
nutrition after 3-4 days.
Medications
* Antiemetics — - Ondansetron
and
• Sedatives — - Diphenhydramine alternative: Granisetron
Chlorpromazine+diphenhyd Lorazepam ,
Analgesics — Ketorolac
ramine IV
*
Treatment of specific signs
and symptoms
W s
Epigastric pain — H 2RA or
PP|
S Diarrhea — - Antidiarrheals
Hypertension - — captopril
s Treatment of specific comp| jcatj
ons
391
. -

care Antimigraine (triptans), Sumatriptan
and refeeding •
J- °Theinvertogram or upside-down x-ray

tabte
^'
i teral P ain
' x"ra at
^ br
^ e
'
is of little value,
a ow lme f r
" ' °
bowel
ed air) with a radiopaque marker on the perineum
but a prone cross
distention from
can demonstrate a
W d o' n
5
leS by showing the rectal gas bubble <1 cm
from the perineal skin.
*
k>
ray 0f the entire sacrum , including both iliac wings, A
^'"
ld v.n fy
"
sacral
e
anomalies
are many
and the adequacy of the sacrum.
~ -

anomalies associated with anorectal


is important to

6J B

"“ ' common are anomalies of the kidneys and urinary tract malformations. The
abnormalities of the sacrum. This complex is often referred to as
in conjunction
caudal
regression syndrome.
u Id The liver span increases linearly with body weight and age
railJ in b th «•
from approximately 4.5-5.0 cm at 1wk of age to approximate°y **cm
boys and 6.0-6.5 cm in girls by 12 yr of age.
65.(E) Pruritus; intense generalized itchi g patients with chronic
liver disease often in association with cholestasis (conjugated ^
hyperbilirubinemia). Symptoms can b g „onorali,ed or localized (commonly to
palms and soles), are usually worse at g exacerbated with stress and
>
heat, and are relieved by cool temperatures. P ur tus is unrelated to the degree
of hyperbilirubinemia; deeply undtced pa re
^
66.(A) Spider angiomas; vascularspidersltelange
^ characterized by
central pulsating arterioles from which sm , may seen
in patients with chronic liver disease, ese ^ i
^ ^
m0St prominent in the
superior vena cava an(J chest). Their size va e*
distribution area (on
between 1 and 10 mm and they exhibit cent clearingg with pressure. * "
presumably reflect altered estrogen metabolism presence of hepatic
dysfunction.
67.(E) Abdominal
ultrasound is a helpful
,

^
* ..
agnostic

.
tool in evaluating ne o
cholestasis because it identifies choledochohth as s, pe tion of the bitea duct

^^
or other structural as a cM
abnormalities of the biliary tre
Hepatobiliary scintigraphy .with .
_m technetium-labeled
| of ERCP
biliary atresia. The ro e
Mi

derivatives is a sensitive but not speci


in the diagnostic
process of biliary a r
*v

rernains indetermina
not
- jlar|V the
establisheci.
(

value of MRCP in the diagnosis of b 1 atresia has


Percutaneous liver biopsy is the m
neonatal hepatobiliary diseases and Pro
"
, proCedure in tn eva|uation of
the most re
discrjminatorY

evidence . ^
relationship between liver disease
and lymphedema is not
understoodcN
be attributable to decreased
.
hypoplasia Affected patients
hepatic lymph flow or hepatic
usually present with episodic
choiJJS
*
, alkaline phosphatase, and bile , ^
elevation of serum aminotransferase acid
, the patients are usually asymptomatic
Between episodes and
_ improve. Compared to other types
of hereditary neonatal choi %
indices ^ s -

patients with Aagenaes syndrome have a relatively


(

good prognosis. The | ^J


for Aagenaes syndrome is mapped to a 6.6 cM
interval on chromosomel5
q
.
69 ( A) Zellweger (cerebrohepatorenal) syndrome is a rare autosomal recess
genetic disorder marked by progressive degeneration of the liver and kidney
the disease is usually fatal by 6-12 mo of age. Affected infants have sever?
generalized hypotonia and markedly impaired neurologic function with
psychomotor retardation*
.
70 (C) Neonatal iron storage disease ( neonatal hemochromatosis, gestational
alloimmune liver disease) is a rapidly progressive disease characterized by
increased iron deposition in the liver, heart, and endocrine organs without
increased iron stores in the reticuloendothelial system. Patients have
multiorgan failure and shortened survival. Familial cases are reported, and
repeated affected neonates in the same family are common. This is an
alloimmune disorder with maternal antibodies directed against the fetal liver
Neonatal hemochromatosis (or fetal loss) seems to be a gestational alloimmune
disease, and reoccurrence of severe neonatal hemochromatosis in at -risk
pregnancies may be reduced by maternal treatment with weekly high-dose
intravenous immunoglobulin (1 g/kg) beginning gestational age 18 wk The .
diagnosis is usually confirmed by buccal
mucosal biopsy or MR1 demonstrating
extrahepatic siderosis.
The prognosis is poor; however, liver transplantation
.
71 (B) Alagille syndrome (arteriohepatic
can be curative .
dysplasia) is the most common

^?
'aTo * *!.
* ** Edition to others clinic *
8 dUCt paucitV-
manifestations as ocular
abnormalities (posterior embryotoxon, mtoWW '
optic disk drusen, s allow anterior
cardiovascular
abnormal
*
£2zsrs
chamber ),

fuserT
vertebral defects ( butterfly vertebrae *
^ 3 defect' aortic
-- - -- coarctation!.
, rib
nephroDatherte
^
r 8 e Spma b flda
anomalies), and tubulointerstitial
iyJ^
|
'
'

^
f ndlngS Sudl
stature, pancreatic insufficiency, vascufopathyy
' syndrome,^stroke
(Moyamoya
^
393
fjents ,
. defective Aspermatogenesis
agille svndrome
can reflect or prod
likely to have
,,
nutritional deficiency.
Pruritus, xanthomas
dt¥ elevated serum cholesterol levels and with
TaS" E deficiency if untreated.
* ici if flow is not rapidly established in the fire - .
eurologic complication s of

!
^
cirrhosis ensue. If microscopic
obliterationareandfound channel 76
' ressive
, postoperative
diameter for establishing establishment of bill f ' °
pa
V m in
1
success rate) if performed good bile flow after the Kasai peration is much
before 8 wk of life.
The
higher (90%
.
7J (C) A degenerative neuromuscular syndrome is found in patients

.
Tronic cholestasis, caused by vitamin E deficiency; affected children
progressive areflex a, cerebellar ataxia, ophthalmoplegia, and decreased
vibratory sensation.
expert! ^

74v(A) L ver disease is the most common disease manifestation in children and
'
can precede neurologic symptoms by as long as 10 yr. Females are 3 times more
likely than males to present with acute hepatic failure . When Wilson disease
presents after age 20, neurologic symptoms are the most common
manifestation. Kayser-Fleischer rings are absent in young patients with hepatic
Wilson disease up to 50% of the time but are present in 95% of patients with
neurologic symptoms. Coombs-negative hemolytic anemia may be an initial
manifestation, possibly related to the release of large amounts of copper from
damaged hepatocytes.
75.(A) Most patients with Wilson disease have decreased serum ceruloplasmin
levels. The serum free copper level may be elevated in early Wilson disease, and
urinary copper excretion (normally <40 pg/day ) is increased to > 100 pg/day and
often up to 1,000 pg or more per day, in equivocal cases, the response of
urinary copper output to chelation may be of diagnostic help.
76.(E) Liver biopsy can determine the extent and severity of liver disease and for
weight ) but is
Measuring the hepatic copper content ( normaily < 10 pg/g dry
only required if clinical signs and noninvasive tests do not allow a fina
lagnos

or < f another liver disorder is suspected. Hepatic copper accumu a ion


copper
hallmark of Wilson disease and measurement of hepatic parenchymal
concentration is the method of choice for diagnosis.
of the 1 nitanlasmin
«
77.(8) This screening should include determination
abnormal or
level and 24 - hr urinary copper excretion. If these results
equivocal, liver biopsy should be carried out
hepatic copper content. Genetic
DNA mutation analysis is possible
screening by
Jage ^
ana|ysis or direct

TO A
78.(6) For patients with Wilson disease, the dietary intake 0f co
.
restricted to <1 mg/day High copper content foods such as liver
and chocolate should be avoided. If the copper content of the drnn* •S
exceeds 0.1 mg/L, it may be necessary to demineralize the water.
79.(C) Zinc has been used as adjuvant therapy, maintenance therapy
^",^
t no ,
therapy in presymptomatic patients, owing to its unique ability to
gastrointestinal absorption of copper. Because penicillamine
antimetabolite of vitamin B6, additional amounts of this vitamin are neces^ *
80.(B) Immune therapy with exchange transfusion and intrav
immunoglobulin has been reported to improve the outcome and reduce
need for liver transplantation in patients with neonatal hemochromatosis.
81.( A) With recovery, the liver morphology returns to normal within 3 mo of
^^
^
acute infection. If chronic hepatitis develops, the inflammatory infiltrate settles
in the periportal areas and often leads to progressive scarring. Both of these
hallmarks of chronic hepatitis are seen in cases of HBV and HCV.
82.(B) Patients infected with HAV are contagious for 2 wk before m
approximately 7 days after the onset of jaundice and should be excluded from
.
school, childcare, or work during this period Careful hand -washing is necessary,
particularly after changing diapers and before preparing or serving food. In
hospital settings, contact and standard precautions are recommended for 1 wt
after onset of symptoms.
83.(0)
S4.(D >
Characteristic autoantibodies in type 1AIH
S Antinuclear antibody
S Smooth-muscle antibody
S
• Antiactin antibody
Autoantibodies against soluble liver antigen and liver pancreas antigen
-
S Atypical perinuclear antineutrophil cytoplasmic antibody
Characteristic autoantibodies in type 1 AIH
S Antibody against liver -kidney microsome type 1
Antibody against liver cytosol type 1
S Antibody against liver- kidney microsomal type 3
s
85 .( A ) Prednisone , with or without azathioprine or 6 - mercaptopurine ,
the clinical, biochemical, and histologic features in most patients ^
seve .
autoimmune hepatitis and prolongs survival in most patients with
disease The goal is
*
to suppress or eliminate hepatic inflammation with min ^
side effects.
395
S6.IA ) Patients should
pneumonia , peritonitis,
be monitors
and urinarv ,
Cl se
° V for
re5
;nf

injury ; gestational ,
alloimmunp actose ia can be a«
^
8at Ve 3nd
'
neonlT
of acute liver failure in the
.(D)
d Sease (
' GA 1-0) js°t? * ith prenatal li
'most rcommon
ed

. Hypophosphatemia, problf
88 <
fver
cause
phosphorus
administration
fa lure, whereas
, 3° ' on of liver regeneration,
V a ref ecti

^^
^ witha a better prognosis in and early
hyperphosphate predicts
failure of spontaneous
acute liver
recovery.
Chapter 18
The Respiratory System
Questions
7IJHAIR ALMUSAWI
following hematological diseases may develop digital
1 Which of the
. clubb
A. Thalassemia
B. Sickle cell anemia
C. Iron deficiency anemia
D. Fanconi anemia
E. Congenital spherocytosis

2 . The following arterial blood gas readings


A . depressed respiratory center
PCO 2,
^ PO 2 ) are seen in

B . mild intrapulmonary airway obstruction


C. Ineffective neuromuscular function
D. central ( above the carina ) airway obstruction
E . alveolar-interstitial pathology

3. The utility of MRI of the chest is largely limited to the analysis of


A. vascular anatomy
B. early metastases
C. pleural lesions
D. cystic parenchymal lesions
E . bronchiectasis

4 . What is the MOST


persistent cough
10
common cause of recurrent or
children ?
A . Asthma
B. Chronic rhinitis
C. Cystic fibrosis
D . Habit cough
E . Foreign- body aspiration
day with no
,5 A 7-Vear 8ht
rtctitotiona
c n following, the MOST likely diagnosis is
°
0ftbe croup
A
6 - trac
heomalacia
Q tracheitis
body
D . foreign
cough
E . habit

An 8- year - old boy brought by his father because of throat clearing sound all
6
the day for the last three months, the boy looks healthy with normal ENT
examination. likely cause of his problem is
of the following, the MOST
A. gastroesophageal reflux
B. postnasal drip
C. vocal tic
D . sinusitis
E. reactive airway disease

7. A 5 -year - old child developed


bronchopneumonia 6 weeks ago, treated
. Since that time till now he
properly with clinical and radiological improvement
chest x -ray, complete blood
has persistent cough, for this reinvestigated with revealed mostly
normal . Sputum examination
count, and ESR which were
macrophages. . .
coug is
.
persistent
Of the following, the MOST likely cause of his
A . asthma
B. cystic fibrosis
cough receptors
C. postinfectious hypersensitivity of
D. repeated aspiration
E. plastic bronchitis
stridor and wheezing which
8. A 7-year -old child presents with intermittent no response to asthma
as asthma with
accompanies physical activity, diagnosed
therapies. is
the MOST likely cause of his/her problem
Of the following,
A. allergic ( spasmodic) croup

C. bilateral abductor paralysis of the cords


398
>

paradoxical vocal cord dysfunction


D.
ired
acquir subglottic tracheal stenosis
E.

9. Which of the
following is a risk factor associated with sudden ,
info
syndrome ?
A . Age ( peak 6-9
mo )
^
B. Female gender
C . Pacifier ( dummy )
D. Postmaturity
)
E Recent febrile illness ( mild infections

10. A young mother brought her 6 -month- old boy for evaluation, he ,
completely normal now but she gave a history of sudden attack of cyanos!S
irregular breathing, hypotonia, and poor responsiveness which lasted about 3o
seconds. This event occurred last night and he returned normal after blowing on
his face .
Of the following, the MOST likely explanation for this scenario is
.
A aborted crib deaths
B. breath holding spell
C. apnea of infancy
D. brief resolved unexplained event
E. prolonged QT syndrome

11. What is the MOST common presenting clinical symptom


of foreign body in
the nose of a child ?
A . Mucopurulent nasal discharge
B . Foul nasal odor
C. Epistaxis
D. Nasal obstruction
E. History of insertion of
foreign bodies
12. What is the MOST
common childhood cause of nasal polyposis?
A. Chronic sinusitis
B. Allergic rhinitis
C. Cystic fibrosis
D- Low vitamin D levels
E- Samter triad
. ,hange m the color or co stency of the nasal
13 A f £0mnion cold illness may indicate secretions during the
sinusitis
superinfection
g bacterial
of polymorphonuclear cells
c accumulation
p vasomotor rhinitis
E. allergic rhinitis

the following is no more effective than placebo


14. Which of for the treatment
of cold symptoms ?
A. Zinc
B. Pseudoephedrine
C. Phenylephrine
D. Vitamin C
E. Saline nose drops

IS. Which of the following is a minor symptom of sinusitis ?


.
A Purulent anterior nasal discharge
B. Nasal congestion or obstruction
C. facial congestion or fullness
D. Hyposmia or anosmia
E. Dental pain

16. What is the preferred antimicrobial agent used for treatment of group A
streptococcal pharyngitis in children ?
A. Penicillin
B. Amoxicillin
C. Benzathine penicillin
D - Benzathineprocaine penicillin G combination
E. Azithromycin

17 . A 3-year - old child ! intake , and


presents with fever , irritability, decreased ora
.
drooling Physical examination revealed torticollis, refusal to move the neck
with muffled voice and stridor .
Of the following , the MOST likely diagnosis is
A . acute epiglottitis
B foreign body aspiration
.

C . meningitis

400
retropbarycg eal abscess


0.
os teomyelitis -
vertebral
'

-
E.

-
rhinorrhea, mild cough, and low -grade fever
- year - oW boyy
oio has . , and
fot
inspiratory sWd„ "*n

.
18. A 2 hoarseness
« I »
?,’? ».« ”» "£
tea 110
S
saturation 95%
34
" "" «
the MOST likely diagnosis is
Of the following
A. laryngotracheobroncnrtis
B* acute epiglottitis
C acute infectious laryngitis
,

D spasmodic croup
,

E , bacterial tracheitis

19. A 3-year -old healthy child suddenly develops a sore throat and fever. Within
a matter of hours, the patient appears toxic, swallowing is difficult with
drooling, and labored breathing. The neck is hyperextended and the child
assumes the tripod position.
Of the following, the MOST likely diagnosis is
A . laryngotracheobronchitrs
B. acute epiglottitis
C . acute infectious laryngitis
D. spasmodic croup
E. bacterial tracheitis

20, A 3 -year - old child


awakens with 3 characteristic
inspiration, respiratory barking cough, noisy
distress, and
He was afebrile and
W
„ frightened appearance .
Wem C sleep He bad similar attack 3
months ago which resolve
with - °
Of the following, the
MOST li
is
A-
'
is
acvngotracheobronchitis
B. acute epiglottitis
c acute infectious laryngitis
U. spasmodic croup
f bacterial tracheitis
! of oral con
,r effectiventheesslEAST effectse° <on, *W croup
*
A
e

B-
* *
° . *Intramus ,ar dexamethas nTne
nele dose
cu
of oral

budesonide
dex
0
^ Pt s
is we\\
established.

C- nebulized one
sing le dose of oral prednisol
D. 2 mg/kg/day for 3 days
£ pre dni sol one
- year-old
boy looks toxic with high f
XL A 5

--- *'
• *
=
“, “l' ' „ '1 W

x
croup

,
epinephrine) is ineffective. His t was update.

“r'
h 0ST

is

B.
^S.pneumoniae
'
C. Moraxella catarrhalis
D. nontweable H. influenzae
Parainfluenza virus
idered in a child with
23. Which of the following conditions should be cons
recurrent or persistent croup -like symptoms?
A. Laryngomalacia
B. Congenital subglottic stenosis
C. Vocal cord paralysis
D. Congenital laryngeal web
E. Subglottic hemangioma
with
24. A 6-month-old infant with congenital hydrocep
respiratory distress, high -pitched inspiratory strid
halus Pr
or, dysp om “*"** an(j

inspiratory weak cry .


Of the following, the MOST likely diagnosis is
A . bilateral vocal cord paralysis
B, congenital laryngeal web
C. subglottic hemangioma
.
D laryngocele
E. congenital subglottic stenosis

402
of subglottic hemangiomas ?
25. What is the -
first line treatment
A. Oral prednisone
B . Oral propranolol
injection
C. intralesional steroid
D. KTP laser
endoscopic excision
E, Tracheostomy

26. Acute bronchiolitis in a previously healthy infant presenting with


a first
episode of wheezing ( following a period of upper respiratory symptoms)
diagnosis ? needs
which of the following for the
A . Chest radiography
B. White blood cell and differential counts
C. Polymerase chain reaction
D. Rapid immunofluorescence
E. Good clinical examination

27. Which of the following is of value in the management of children with viral
bronchiolitis ?
A. Chest physiotherapy
B . Corticosteroids
C. Hypertonic saline
D. Ribavirin
E . Frequent suctioning of nasal
and oral secretions
28. What is the BEST treatment
for cough in an infant
A. Cough suppressants with acute bronchitis ?
B. Antihistamines
C. Expectorants
D - Cold medicines
.
E Frequent shifts in posj ,
ion
29. Which of the following
A. Pulmonary lymphatic conditions iS p oved to
B . Toxic inhalation anomalies ' cause plastic bronchitis ?
C. Sickle cell acute
chest Syndrome
D. Cystic fibrosis
.
E Bronchiectasis
healthy child presented with 3 neck mass noticed while
- old
A 3- r hing.
veacoug
30 r
&
ininK °
rra following, the MOST likely diagnosis is
. a

Ofthe
'
cystic hygroma
extrapulmonary sequestration
Q
bronchogenic cyst
p laryngocele
hernia
E . Sibson

is the earliest clinical sign of pulmonary edema ?


31 What
A. Tachypnea
B. Fine crackles and wheezing
C. Gallop rhythm
0. Peripheral edema
E. Jugular venous distention

sign of pulmonary edema ?


32. What is the early chest radiographic
A . Diffuse streakiness
B. Diffuse patchy densities
C. Cardiomegaly
D. Peribronchial cuffing
E . Pleural effusion

33 . What is the MOST common underlying probl


em associated with recurrent
pneumonias in hospitalized children ?
A . Oropharyngeal incoordination
B. Micrognathia
C. Macroglossia
D . Achalasia
E . Nasal feeding tube

for the diagnosis of recurrent


34. What is the MOST appropriate test
microaspirations?
A Plain chest radiograph
.

B . High- resolution CT
study
C . Videofluoroscopic swallowing
D . Barium esophagram
E . Esophageal pH monitoring
404
. r


,
the clinician in supporting the
MOST helpM test
0

-
dta

^ssss -
35. w b the

^ scs-f oScomt tteritedtomog a


^
> sssr"
E. Elev

36. What is the ideal method of treatment


hypersensitivity pneumonia ?
A. Removal of the
offending antigen
-
and prevention of
recurrences

B. 0.5 mg/kg/day of prednisolone


C. High dose pulse intravenous
methylprednisolone
D . Cyclosporine
E. Azathioprine

peripheral
37 . A 2- year - old boy presents with cough, dyspnea, migratory
pulmonary infiltrates, and blood eosinophilia . He had a history of pica, chronic
abdominal pain, and vomiting.
Of the following, the MOST likely diagnosis is
A . acute eosinophilic pneumonia
B . Loffler syndrome
C. chronic eosinophilic pneumonia
D . Churg- Strauss Syndrome
E . allergic bronchopulmonary aspergillosis

3S. A 5 - year-old boy presents with snoring during sleep, behavioral


wtT,sIeM 7 diffiCUlt eS and

^
What s the MOST , '
Tllikely cause '
A . Habitual snoring
B. Obstructive sleep apnea
of this problem ?
daytime sleepiness.

C. Allergic rhinitis
D. Adenotonsillar h
Vpertrophy
E . Bilateral nasal polyps

39 . Which strains of the following


organi lv
5 be associated with particu|ar >
rapid pulmonary deterioration and
death n cyst c, ^
fibrosis ?
A. Staphylococcus aureus

A C\ r
influenza
Haem op hilu s aeruginosa
. Pseudomonaascepacia
g
C
P ffurkholdericoli
Escherichia
£

mo nth-old i „fant StorV of rpn


P<?afed cha
chronic
- uSht to h„


6 to thrive hr »
40- A
a gave
dorrheand , and failure
historv of
„ °°
°< * parents
Svve« test
are

^
'
cousin not mailable . The resid
ent asked you what other investigati
you
«told him to do arterial blood gases.
8 on s

helpful and
the following findings aid your diagnosis ?
Which of hypochloremic metabolic alkalosis
A . Hyp erna trem ic
Hyp ona trem ic hypochloremic metabolic alkalosis
B .
Hyp erna trem ic hype rchloremic metabolic alkalosis
C .
D. Hyponatremic
hypochloremic metabolic acidosis
E. Hypernatremic hyperchlo
remic metabolic acidosis

Whic h of the follo wing cond ition s associated with false -negative sweat test
41 .

results in patients with cystic fibrosis?


A. Eczema ( atopic dermatitis )
B. Edema
C. Congenital adrenal hyperplasia
D. Hypothyroidism
E. Nephrogenic diabetes insipidus

panc ic
reatic malabsorption in cystic
What is the BEST diagnostic test for
fibrosis?
a 72 -hr stool
Quantification of fat malabsorption with
-1 activi ty in a fresh stool sample
Quantification of elastase
Oral glucose tolerance testing (OGTT)
j
*

Spot testing of blood and urine glucose level


s
Glycosylated hemoglobin levels
has been
shown to
43. Which of the following pulmonary function studies
ate in cysti c fibrosis .
Prel MOST closely with morta lity
A - Residual volume
functional residual capacity
C - Forced expiratory volume in 1 sec

406
forced vital capajW
3C ,tv
E Total lung caP
broadly effective oral antibiotic f0f
of the following is a Pse Wo
44. Which X
in ic fibrosis ?
cystic
infection
A. Dicloxacillin
B. Linezolid
C. Cephalexin
p Ciprofloxacin
E Amoxicillin -clavulanate
indication for aerosolized antibiotic therapy is to eradicate ;
45. An important
aeruginosa in the airways after initial
detection in patients with cystic fibrosis
commonly used for this purpose ?
Which of the following antibiotics is
A . Ticarcillin
B. Piperacillin
C. Tobramycin
D. Meropenem
E . Ceftazidime

46 . Which of the following anti -inflammatory drugs are commonly used in cystic
fibrosis ?
A . Macrofide antibiotics
B. Systemic corticosteroids
C. Inhaled corticosteroids
D . Ibuprofen
E . Cephalosporin antibiotics

*
infiltrates. Total serum irnmun ^ I th
6 aPV CheS
' radioSraPh shows new, foc<i

^
|
E
of eosinophils in rust -
colored sp um
Of the following, the MOST ^
appropriate initial
W 3 S 1100 With
^****
A. oral corticosteroids . treatment is
B. oral antifungals
C. omalizumab
D . montelukast

407
E. mer0penem

what vitamins supplementation is recommended in


Patients with cystic
A. A , C, Et, K
0. A, 0, E, K
r A, 0, B, K
0. .
A, 0 E, B
E. C, D, E, K

49. What is the MOST common type of inheritance Of primary ciliary dyskinesia
A. Autosomal recessive ?
B. X -linked recessive inheritance
C. Autosomal dominant
D. X -linked dominant inheritance
E. Multifactorial

-
50. A 48-hour term newborn developed increased
work of breathing,
tachypnea, and upper and middle lobe atelectasis with
dextrocardia on chest
radiograph.
Of the following, the MOST likely diagnosis is
A. transient tachypnea of the newborn
B. pneumonia
C. hyaline membrane disease
D. primary ciliary dyskinesia
E. meconium aspiration pneumonia

51. Which of the following is a rare clinical manifestation of primary ciliary


dyskinesia ( PCD) ?
A. Unexplained respiratory distress in term neonate
B. Daily, nonseasonal rhinosinusitis since early infancy
C. Atypical asthma unresponsive to therapy
0. Hydrocephalus
E. Male infertility

hemosiderosis is associated with


52. Which of the following pulmonary
Pulmonary capillaritis ?
A . Idiopathic pulmonary
.
hemos derosis
408
r
B. Heiner syndrome
C. Goodpasture syndrome
D. Hemolytic uremic syndromes
E. Pulmonary capillary hemangiomatosis

53 . Which of the following laboratory findings is classicallv


pulmonary hemosiderosis ? associatec)
A . Microcytic hypochromic anemia '•'r
B. Increased total iron- binding capacity
C . Decreased ferritin levels
D. Low erythrocyte sedimentation rate
E . Reticulocytopenia

54 . What is the first-line treatment of idiopathic pulmonar


y hemosiderosis(
A . Plasmapheresis IPH)?
B. Methotrexate
C. Systemic corticosteroids
D. Intravenous Immunoglobulin
E . Extracorporeal membrane oxygenation
( ECM )
O
55. What is the MOST common iden
tified risk factor of venous thromboembolic
disease ( VTE ) in children ?
A. Obesity
B . Immobility
C. Chronic obstructive pulmonar
y disease
D . Infection
E. Diabetes mellitus

56 What is the diagnostic


test of choice to detect a pulmonary embolism
children?
A . Ventilation-perfusion
( V - Q ) radionuclide scan
B . Spiral CT with an
intravenous contrast agent
C. Pulmonary angio
graphy
D . MRI
E . Doppler ultrasonsgraphy

57. What is the MAIN


side effect of heparin ?
A . Thrombocytopenia
6.
Leukopenia
c Renal impairment
p pruritic skin rash
E. Hemolytic
anemia

58.
w “°Sbe“ “ mm n
° * l""l'"d « »1«
““* ,?
R Left lower lobe
C. Right upper lobe
D. Right middle lobe
E. Right lower lobe

59. Migrating atelectasis in the newborn infant ,


a rare and unique presentation
may be secondary to which of the follo ,
wing ?
A. Neuromuscular disease
B. Bronchopneumonia
C. Meconium- aspiration pneumonia
D. Bronchopulmonary dysplasia
E. Cystic fibrosis

60. What is the MOST common form


of pulmonary malignancy in children ?
A. Bronchial carcinoid
B. Adenoid cystic carcinoma
C. Mucoepidermoid carcinomas
D . Metastatic lesions
E . Pulmonary blastoma

61- Hydrothorax is usua


lly bilateral, but it can be limited to the right side in
which of the following ?
A . Cardiac disease
B. Severe nutritional edem
a
C . Superior vena cava
obstruction
D - Ventriculoperitoneal
shunt
E. Peritoneal dialysis

62 . What is the MOST comm symp


on tom of pectus excavatum?
A . Fatigue
B . Chest pain

410
c palpitations
D'
E. Decreased exercise tolerance
is the MOST common cause of death in asphyxiati
lnS tho
63. What
dystrophy?
A. Renal failure
8. Hepatic failure
depression
C. Central nervous system
D. Pancreatic insufficiency
E . Respiratoryfailure

64 . What are the MOST common mucolytic used in ventilated patients?


A. Dornasealfa
B. N- acetylcysteine
C. Hypertonic saline nebulizer
D. Adequate hydration
E. Glycopyrrolate

65 . Ventilated patients may have nutritional needs that are equal to, greater, or
lesser than those of comparably aged well children. Excessive growth is as
harmful as inadequate growth, and excess calories may lead to
A . increased oxygen consumption
B . increased carbon dioxide ( C02 ) productio
n
C. overhydration
D. obesity
E. metabolic acidosis

66. A 24 -hour-fullterm

develnnlT* , Weight for gestational age


product of
newborn, ,

uneventful pre8nancy,
frequent resnirat st )a low respiration with
Gnosis and resni
Paco, of 95 mm
^ ,
*
,
* PdUSes which progress
a ure - Capi// J* ° , eXt fSW days t0 app3rent
arv blood g3S ana s s during sleep reveals
Of the following, thp M decl ,
ne to normal /P
^ ^ th<? infants
*
* COr)genital
^ dia
^ is

r:r
C

c "
A11
£, hypothalamic dys regulati
0n hypo
67. A 9-month-old infant With
wi
Elation
and sleep disordered breathing
history of epai'
red'
presented w«h aphonia, hyPovenl . inn <he f 0 rrn Vyelo menin '
MOSTlikeiS
Of the following, the
' .
at n
'° and
of
obstr
bran t °
dyarrh uctive ' ep
- h
Ce e big
V dla n
Vthmias . ^ aPnea:
S
A. hypothalamic dysfunrt ^° sis is

c. Arnold -Chiari tyPe


D. central

nervous SVstp °rrriation
£. Dandy
mZfn' format<
Walker ma CNS
i0 n
> hemorrhage

•112
Chapter 18
SeRespiratorySyste ^
ZUHAIR ALMUSAWI
Answers ^^
Disease s Associated With Clubbing
l.( A ) Nonpulmonary
CARDIAC
Cyanotic congenital heart disease
Bacterial endocarditis
Chronic heart failure
HEMATOLOGIC
Thalassemia
Congenital methemoglobinemia ( rare )
GASTROINTESTINAL
Crohn disease
Ulcerative colitis
Celiac disease
Chronic dysentery, sprue
Polyposis coli
Severe gastrointestinal hemorrhage
Small bowel lymphoma
Liver cirrhosis ( including al - antitrypsin deficiency )
Chronic active hepatitis
OTHER
Thyroid deficiency ( thyroid acropachy )
Thyrotoxicosis
Chronic pyelonephritis ( rare )
Toxic ( e.g., arsenic, mercury, beryllium)
Lymphomatoid granulomatosis
Fabry disease
Raynaud disease, scleroderma
Hodgkin disease
Familial
UNILATERAL CLUBBING
jjijordBi' s ( S B - subclavian arterial aneurysm
J

V3SC
tilar , brachial arteriovenous

-
)
fistufa f shoulder

SSE « *
locaI trauma

small lesions (e.g.,


cystic parenchymal
early metastases, mediastinal and pleural
lesions, pulmonary embolism, and bronchiectasis).
lesions solid 2
4.(A) The most common cause of recurrent or persistent cough in children is
airway reactivity ( asthma ). Because cough receptors also reside in the pharynx ,
paranasal sinuses, stomach, and external auditory canal, the source of a
persistent cough may need to be sought beyond the lungs. Specific lower
respiratory stimuli include excessive secretions, aspirated foreign material,
inhaled dust particles or noxious gases, cold or dry air, and an inflammatory
response to infectious agents or allergic processes.
5.(E) Cough all day, never during sleep is habit cough . Causes of barking, brassy
cough are croup, habit cough, tracheomalacia, tracheitis, and epiglottitis.
6 (C)
7,(C) Only sputum specimens containing alveolar macrophages should be
interpreted as reflecting lower respiratory tract processes. Sputum eosinophilia
suggests asthma, asthmatic bronchitis, or hypersensitivity reactions
of the lung,
in
but a polymorphonuclear cell response suggests infection; If most of the cells
sputum are macrophages, postinfectious hypersensitivity of
cough recepto
hemosi
should be suspected . Sputum macrophages can be stained for
, or or ipi
content, which is diagnostic of pulmonary hemosiderosis
repea e
which in large amounts suggests, but is not specific for,
Rarely, children may expectorate partial casts of the
airway, ^ ^
characterized in investigating causes of plastic bronchitis
.
accompanies
8.(0) In a child with intermittent stridor { with w eei1
paradoxical vocal
Physical activity and is not responsive to asthma ^ corcj dysfunction
cord dysfunction may be of consideration. Paradoxic aryngoscopy during an
|
maV he highly supported by history and confirme
y
Yetjc
|te(j speech
therapy
exercise challenge test if symptoms are success
u
and behavior modification may be therapeutic
9 ( E)
414
ANTE RISK FACTORS
MATERNAL AND NATA L a-fetoprotein
ter serum
Elevated 2nd trimes
Smoking
Alcohol use , heroin )
Drug use (cocaine
Nutritional deficiencycare
Inadequate prenatal
Low socioeconomic status
Younger age
Lower education
Single marital status
Shorter interpregnancy interval
Intrauterine hypoxia
Fetal growth restriction
INFANT RISK FACTORS
Age ( peak 1-4 mo)
Male gender
(AfnCan AmenCan
'
,
Indian, Alaska Native, othe

Growth failure
No breastfeeding
No pacifier (dummy)
Prematurity

£emfn '
^
eS eep P05
*
sleeping surf ace Toft^bedd^
Bed sharing with parent "®


C8ntral heating
l0 (D) Brief Re

;Bt;aUE (6RUf)

^
IPr nounced
°
£? - «
«i

^ rrin
~ ^, ^
a
sr j r
^
. bSent ^'
fo lowing:
Pallor
now-resolved

' leased. or irregular


*
epISod

breathing
de ibed by the observer as a
lnvo v
^ at least one of the

4.1 K
change in tone, either hyper hyP tonia
. marked
level of responsiveness °
. altefed
of BRUE applies only to , who Were =,syfT>Ptomatic nr,-
The
diagnosis
and during evaluation a,J ‘hen no e*nia
event “'


the event is
appropriate histnn
foun d through
co« «>d for o Periods
they initially produce few1
* "' (
are difficult t vis ««ze Fir.
of time
because include unilateral obstructand snee ng reht , !*
0
* t
symptoms
rarely, pain. Presenting din
, * ' *' of Action of
and ,

, disch ge (24%) f 7 ‘°
)
epistaxis (6% , nasal obstruction ( 3%
^
mouth eathmo
^ nl ^ ' od <9% )
°' ,

-
8

12.(C) Cystic fibrosis is the most rn


J
'
COrnmon childhood caul J '

poVposis

Therifnf , ,
and up to 50% of CF patients Penence obstructing
^
nasal
teTinT <
rare in non-CF children
12 Vr old with nasapo B I e
digestive symptoms.
13 (C)
""
'
Sh0U d be

^ ^
ISSK SSS
14.(D) Vitamin C, guaifenesin, and inhalation of warm, humidified air are no
2
more effective than placebo for the treatment of cold symptoms.
15.(E)
MAJOR SYMPTOMS
•Purulent anterior nasal discharge
. Purulent or discolored posterior nasal discharge
•Nasal congestion or obstruction
•Facial congestion or fullness
•Facial pain or pressure
•Hyposmia or anosmia
•Fever (for acute sinusitis only )
MINOR SYMPTOMS
•Headache
•Ear pain, pressure, or fullness
•Halitosis
•Dental pain
•Cough
.
•Fever (for subacute or chronic sinusitis )
•Fatigue
of activity, and has few
16.(B) Penicillin is inexpensive, has a narrow spectrum because of taste,
often pre for children
adverse effects. Amoxicillin is
416
of
-
I

.. .
intramu
combination is effective and ensures
compliance .
The use of macrolides and
related ant b ot cs should be restnc
tN
receiv e a 0-lactam drug for GAS pharyn
who cannot safely gfe
fluoroquinolones, or sulfonamides should not be used to treat GAS ph
17.(D) Clinical manifestations of retropharyngeal abscess are nonspe. v
indude fever, irritability, decreased oral intake, and drooling. Neck
torticollis, and refusal to move the neck may also be present . The vem
'
^stifr— l
*

might complain of sore throat and neck pain. Other signs can induo * ^
voice, stridor, respiratory distress, or even obstructive sleep apnea*
mjv

examination can reveal bulging of the posterior pharyngeal wall, although


present in <50% of infants with retropharyngeal abscess
Cenr,
lymphadenopathy may also be present .
Lateral pharyngeal abscess commonly presents as fever, dysphagia
ax
prominent bulge of the lateral pharyngeal wall, sometimes
with
*
displacement of the tonsil .
18 ( A)
19.(B) A brief period of air hunger with restles
sness may be followed by rap:
increasing cyanosis and coma. Stridor is a late
finding and suggests ne*
complete airway obstruction. Complete
obstruction of the airway and deatticj *

ensue unless adequate treatment is provided


. The barking cough typtc o’
croup is rare. Usually no other family ^
members are ill w ;th acute respirat:'
symptoms. '
20.(D) Spasmodic croup occurs
most often in children 1- 3 yr of age anc s
cynically similar to acute laryngotracheobronchitis, except that the history of a
prodrome and fever
in the patient and family
0016 Cases kut a er
are often absent . The cau*1
21 im r 0 ' ^ '6 C and other factors may also contribute
rtlcostero s decrease the
,

their a '^
ammatory action. Oral
edema in the laryngeal mucosa throws
steroids are beneficial , even in mild
as mpat
reducehT J reduced hosP talization, shorter
‘ duration of hospitalization, * *
Most stur
H
°rfSu dseq uent interve
, ntions such as admi nistra tion
stiIntramle uscul
dose of o f
ar d ZX T
he e «cy of oral
" '
epineph rine
dexamethasone
**
1

;
^ ^^
clinical effect ora , dosing of '
dexametha one ^^
T * ^
^
administration. A single dose of oral as effective as intramusO
ls
prednisolone is less '
effective; 1 random * ^
1 1

417
trial found no difference in the effectiven prednisolone 2

“ aCUste Epper airway that


yST , ^ethidIin-Te?stammT
' 'L
^umoniae
** iso ated
, S . pyogenes,
^
eports of
Moraxella catarrhalis

anaerobic is a slight male predominance .


7 yr There
, nontypeable H.
influenzae-
organisms have also been implicated. The mean age is between S and' ^
stenosis is a narrowing of the subglottic larynx and manifests
23.{B) Subglottic
jn the infant with
respiratory distress and biphasic or primarily inspiratory
stridor. It may be congenital or acquired.
24.( A) Vocal cord paralysis is the third most common congenital laryngeal
anomaly that produces stridor in infants and children. Congenital central
nervous system lesions such as Chiari malformation, myelomeningocele, and
hydrocephalus or birth trauma may be associated with bilateral paralysis.
25.(B) Propranolol was introduced in 2008 and rapidly became the first -line
treatment of infantile and subglottic hemangiomas, including in a recent
randomized clinical trial comparing it with systemic steroids. The mechanism is
thought to be through VEGF or adrenergic vasoconstriction pathways and can
involute the lesion in a few days. Typically, treatment is with 1-3 mg/kg/day of
propranolol for 4- 12 mo, based on clinical monitoring as noted in a 2011
consensus guideline.
26 - ( EJ The diagnosis of acute bronchiolitis is clinical, particularly in a previously
healthy infant presenting with a first episode of wheezing following a period of
upper respiratory symptoms. Chest radiography is not routinely indicated in
children with suspected bronchiolitis. Areas of atelectasis associated with
bronchiolitis are often observed on chest radiographs and may beradiograp difficult to
y
distinguish from bacterial pneumonia; as a result, obtaining chest
* n 3 patient whose clinical course and exam are consistent with bronc
rnaY er, courage unnecessary antibiotic use. Laboratory testing is ..
routinely indicated ; the white blood cell and differential counts
and are not predictive of bacterial superinfecfon
. Viral te g
formal 0
(polymerase
chain reaction or rapid immunofluorescence
is
"
recommended in the diagnosis of bronchiolitis but may be he Pf
Prevents more invasive evaluations. Concurrent serious bacterial ' “ Action
«°n .
infection
sepsis, pneumonia
, meningitis ) is unlikely, althoug . febrile
i Y
ronchiolitis may obviate the need for a sepsis evaluation
infant
, with viral bronchiolitis is su
oof children
treatment rece|ve supp|emental oxygen
27.lt) The
management -
Hypoxernic
consensus
^
n
djf)g target oxygen
.
saturations; nati ^
a developing a threshold 0f 90%.
guidelines inthC
Some children
U
"f Loire
o support
or
with supplemental hydration. F|uid
enterally via nasogastric tube, with Sonit
mt«
be administered w an association between better outcw .
preference given
to ^
entera| nutrition.
*
and continued provis ,

ofa secretj0ns often
provides relief of distress
Frequent suctioning o
t0 feed. Chest physiotherapy has

srsx
bronchiolitis. Pharmacologic
•*
agents have largely proven ineffective in the
with

management of bronchiolitis.
hypertonicic saline in children with bronchiolitis.
There is debate over the use of
improve length of stay or clinical
Racemic epinephrine has not been found to
. Ribavirin is also not currently
outcomes among inpatients with bronchiolitis
recommended, because of minimal impact on disease outcomes, and
because it
is costly, difficulty to administer, and associated with important toxicities.
28.(E ) Frequent shifts in position can facilitate pulmonary drainage in infants.
Older children are sometimes more comfortable with humidity, but this does
not shorten the disease course. Cough suppressants can relieve symptoms but
can also increase the risk of suppuration and inspissated secretions and
therefore should be used judiciously. Antihistamines dry secretions and are not
helpful; expectorants are likewise not indicated. Nonprescription cough and
cold medicines should not be used in children younger
than 4 yr of age, and
their use is cautioned in children age 4- 11 yr
.
29.|A) Conditions Associated With
•PROVEN CONDITIONS Plastic Bronchitis
Congenital heart disease with
Fontan physioloev
8V
Pulmonary lymphatic anomalies
Influenza A pulmonary infection
•POSSIBLE CONDITIONS
Toxic inhalation
Sickle cell acute chest syndrome
Hypersecretory and near- fatal
•UNLIKELY AND UNPROVEN COND
Cystic fibrosis
Chronic obstructive pulmonary
. ot?'
asthma
T , n°PhiliC casts >
disease
410
Bro nch i
ggcterial
e ctM
pneum S onia
mass only when a chest film js taken for anoth ** *
.
nM
asVmPtomatic and
Findin6s on
?
.
I
detectedexamination are normal except durine Val«h Maneuver, when a soft
physicJ a
noticed in the neck.
bulge imay be
3t<A) The •*»» Wua » « SM =,
dyspnea. As fluid accumulates in the alveolar space,
breathing, tachypnea, and
reveals fine crackles and wheezing, especially in dependent lung
auscultation pulmonary edema, a gallop may be present, as well as
fields. In cardiog enic
peripheral edema and jugular venous distention.
. provide useful ancillary data, although findings of
32 (0) Chest radiographs can
phic signs that represent
initial radiographs may be normal. Early radiogra
and perivascular
accumulation of interstitial edema include peribronchial
cuffing. Diffuse streakiness reflects interlobular edema
and distended
butterfly pattern,
pulmonary lymphatics. Diffuse, patchy densities, the so-called
represent bilateral interstitial or alveolar infiltrates
and are a late sign.
edema. Heart
Cardiomegaly is often seen with cardiogenic causes of pulmonary
size is usually normal in noncardiogenic pulmonary edema.
found to have
.
33 (A) In 2 reports, between 26% and 48% of such children were
dysphagia with aspiration as the underlying problem.
.
34 (C) A modified barium swallow study with
videofluoroscopy
standard
(videofluoroscopic swallowing study) is generally considered the gold
for evaluating the swallowing mechanism .
and very
.
35 (D) Bronchoalveolar Lavage is one of the most sensitive tests
helpful to the clinician in supporting the diagnosis of hypersensitivity
pneumonia HP. Lymphocytosis frequently exceeding 50% of the
recovered cells
is seen on the BAL and should alert the clinician to the possibility of HP.
is a key
36-(A) The control of environmental exposure to the offending antigen
to curing hypersensitivity pneumonia HP and remains the ideal
method of
and pathologic
treatment and prevention of recurrence. The clinical
with removal of the
manifestations of acute and subacute HP are reversible

transient
3
is
.
37 (flNThp
B) The trans pulmonary infiltrates with eosinophilia syndrome that
syndrome) is
most often seen ^ (formerly known as Loffler
inBtrat with peripheral blood
characterized *
420
helminthic infections. A . lumbricoides
eosinophilia caused by the causing this disease. orr r
most common parasite
is the
°un jX
(

38.( B)
with certain strains of B. cepacia and other multidru
39.(D) Infection
with particularly rapid pulmonarv
organisms may be associated
and death.
) These children may present with hypochloremic alkalosis. HyPOn
40 .(B atr
is a risk particularly in warm
climates . ^ia
41. (B)
WITH FALSE-POSITIVE RESULTS
dermatitis)
^ Eczema (atopic
Ectodermal dysplasia
/ Malnutrition/failure to thrive/deprivation
/ Anorexia nervosa

^ Congenital adrenal hyperplasia


/ Adrenal insufficiency
Glucose-6-phosphatase deficiency

^ Mauriac syndrome
v Fucosidosis
Familial hypoparathyroidism

^
V
Hypothyroidism
,
ephro6en c diabetes insipidus
v
^^dohypoaldosteronism
/ Klinefelter syndrome


,
famillal cholestasis
5tas s sVndrome
'
°Stt
,
rnic dysfunction
8landin E in USion5
M,

Malnutrition
v Edema
insufficient sweat quantity
Hyponatremia
Cystic fibrosis transmemh
, e COndUC anCe
" '
re6 ator mutahons
“'
with
«-(B)
;"
notification of elastase- actTj
UnC
°
^^^
8

^ 1 y ln a ' Ption can


fresh stool
.
3n be made bV the
mPle by an enzyme-linked
421
assay specific for human elastase. The quantification of fat
nsorbentwith a 72-hr stool collection is rarely necessary in the clinical

^
***
^
'J’
iio

Ts
4
eptjon

tandard
yr of age
pulmonary function studies are usually obtained starting at
and are routinelV done by age 6. Forced expiratory volume in 1
ab° ) is the mea sure men t that has been shown to correlate most closely
(FEVl
Sh WS 3 gradual decline averaging 2-3% per year throughout

^
S
rtali tv and °
h m°
d- quinolones are the only broadly effective oral antibiotics for
childhooThe
44.(0) , but resistance against these agents may emerge.
pseudomonas infection ginosa, such as
may reduce the virulence properties of . aeru
P
Macrolides , and cont ribu te antiinflammatory effects . Long - term therapy
biofilm prod ucti on
3 times a week improves lung function in patients with
with azithromycin
.
chronic P. aeruginosa infection
( C) Aero soliz ed tobra mycin inhalation solution or powder, or aztreonam
45 ,
ive therapy ( on 1 mo, off 1 mo), may
inhalation solution used as a suppress
function, and decrease the occurrence
reduce symptoms, improve pulmonary
of pulmonary exacerbations.
antiinflammatory effect, and 3 days/ wk
46,(A) Macrolide antibiotics have an
show n to redu ce the likeli hood of development of
azithromycin has been
nts with chronic Pseudomonas
pulmonary exacerbations, especially in patie
therapy .
airway infection, so this is a commonly used
chop ulmo nary aspe rgillo sis occu rs in 5- 10% of patients with
47 ( A Aller
, ) gic bron
recovery of Aspergillus organisms
CF. The presence of rust -colored sputum, the of
a posi tive skin test for A . fumigatus , the demonstration
from the sputu m , of
specific IgE and IgG antibodies against A .
fumigatus, or the presence
diagnosis .
eosinophils in a fresh sputum sample supports the
inflammatory reaction with oral
Treatment is directed at controlling the
ls are usua lly rese rved for patients who relapse
corticosteroids. Oral antifunga
For refrac tory case s, omalizumab, humanized
after initial steroid treatment .
monoclonal anti-lgE, has been effective.
malabsorption of fat- soluble
48.(B) Because pancreatic insufficiency results in
vitamins ( A , D, E , K ) , vitamin supplementation is recommend ed.
sive patte rns of inheritance, although
49.( A) PCD typically has auto soma l reces
X-linked inheritance has been reported.
clinical features .
50.( D) Primary ciliary dyskinesia PCD has several characteristic most affected
feature, and
Neonatal respiratory distress ( NRD ) is a common

7
1ZL of the brain ventricles are lined by ciliated epithelium
J a,
Q

T forfinding
cerebrospinal fluid
t
flow through the ventricles and aquedu
when li„ked jj
?

ST e of enlarged brain ventricles on sonograms ,

ii nversus
PCD.
proposed as a prenatal diagnostic
totalis, has been marker f

5Z. C)
< classically associated with a microcytic
53.(A) Pulmonary hemorrhage is
ja . Reductions of serum iron levels , decreased or normal
hypochromic anemia ..
...
to increased
.
ferritin
.
levels may be
total iron-binding capacity, and normal found
with chronic disease . An elevated
erythrocyte sedimentation rate is 3
nonspecific finding. The reticulocyte count is frequently elevated.
54.(C) In IPH, systemic corticosteroids are frequently
utilized as first - line
treatment and are expected to be of particular benefit In the setting of immune-
mediated disease. Steroids modulate neutrophil influx and the inflammation
associated with hemorrhage; consequently, they may decrease progression
toward fibrotic disease.
55.(0) The most common identified risk factors in children include infection,
congenital heart disease, and the presence of an indwelling central venous
catheter .
56.{B ) Although a ventilation-perfusion ( V - 0) radionuclide scan is a
noninvasive and potentially sensitive method of pulmonary embolus detection,
the interpretation of V - 0 scans can be problematic . Helical or spiral CT with an
intravenous contrast agent is valuable and the diagnostic test of choice to
detect a PE .
57.( A) Side effects include heparin-induced
thrombocytopenia as well as
bleeding and osteoporosis.
58.(C) In contrast to atelectasis in adult patients
, in whom the lower lobes and,
in parfcular the left lower
lobe are most often involved, 90% of cases in
children Involve the upper lobes and 63%
involve the right upper lobe.
59.(A)
60.(D) Metastatic lesions are the most
in children; primary processes include
common forms of pulmonary malignancy
i WiJms tumor, osteogenic sarcoma, and
hepatoblastoma .
61.( A) Hydrothorax is usually bilateral buf in ^
,
limited to the right side or greater on the right ^ ^ epat C disease ] t C 3n

423
th
^
than on the left side .
^
pain,
0iy be
is P
^ nt at 0r k
did not associated with ar y portly
rhe deformitY )

, recurrent resnir * ^
palpations Decreased exerc
present .
^
symptoms. patients with this diSOrH
^
. ^
0rr,s at th^ -

^
^^ ****
, *s'«
*^
*ercise to erance
°
t ' °nQ-thirL
n
*
*'* *
»**
_
«.*
f the m0st
<»«

«mm0n

--
Most SOrder die sh
" ^ bir >
^
dll )
failure.
yen°nic s /ne is the most
^
.. ,, „
: ^ '"""
6410
SEXS ::r 0 nc e SMMrbo ’ d exc«s cal0ries

66.( A) ° iJ production.

67.(C) Arnold-Chiari type II malformation is associated with myelomeningocele,


hydrocephalus, and herniation of the cerebellar tonsils, caudal brainstem, and
the 4th ventricle through the foramen magnum . Sleep disordered breathing,
including OSA and hypoventilation, has been reported. Direct pressure on the
respiratory centers or brainstem nuclei or increased intracranial pressure
because of the hydrocephalus may be responsible. Vocal cord paralysis, apnea,
hypoventilation, and bradyarrhythmias have also been reported. Patients with
Arnold-Chiari type II malformation have blunted responses to hypercapnia, and,
to a lesser degree, hypoxia.
Chapter 19
The Cardiovascu lar System
Questions
^
^
AHMED TAWFIQ
, the umbilical venous partial pressure of O2 is about
1. During fetal life
A. 20-25 mmhg
B. 30- 35 mmhg
C. 50- 55 mmhg
D. 60-65 mmhg
E. 70-75 mmhg

ventricular output that enters to the lung


2. What is the percentage of right
during fetal life ?
A. 5%
B. 10%
C. 15%
D. 20%
E. 35%
you had
.
3 A 10-day - old full- term boy has been diagnosed with large VSD;
informed parents that close follow -up is essential Heart failure would
.
expected at the time of
A. starting decrease pulmonary vascular resistance
B. nadir of normal physiological anemia of infancy
C. pulmonary hypertension
D. any coincident upper respiratory tract infection
E. at onsent of lower respiratory tract infections

4. A 7-year -old boy with inoperable complex cyanotic heart disease


complaining of headache and occasional chest pain
for the last one month
. 1
current CBC shows HB of 21.5 gm/ dl and PCV of 66, his serum ferritin 85mg/dL
Of the following, the NEXT step in the management would
be
A. follow up visit after 2 weeks for another
workup
B. advice about appropriate hydration and stop
diuretics medicat > n
425
°
exchange transfusion
C. Partial
therapy
p. iron
l reassurance
are preparing a child with inoperable cvannt u
5. You partial

B. 1/2 glucose saline


C. normal saline
0. fresh whole blood with lower PCV
E . fresh frozen plasma

6, Which of the following is an indication for screening fetal echocardiography ?


A. All pregnant women
B. Gestational diabetes on oral hypoglycemic agent
C. Suspecting fetal chromosomal abnormality
0. History of exposure to teratogens during late pregnancy
nd
E. 2 degree relative with congenital heart disease

7. A 7 -year-old child with familial hypertrophic cardiomyopathy is being


evaluated with cardiac exercise testing.
Which of the following results is considered as abnormal ?
A. ST segment elevation > 2 mm for at least 0.2 second
B. Q wave > 4 mm
C. Appearance of U wave
D. ST segment depression of > 2 mm for at least 0.06 second
E. An increase in blood pressure before maximal exercise is reached

^
8. A 10- day - old boy with a recent diagnosis of
and intact interventricular septum develop
.
cyanosis In order to
( surgjca|
decrease cyanosis and improve the general condition
o
correction; the appropriate action at this moment is
A. start prostaglandin Ei . ._ C K
B. do stent procedure to reopen ductus a e |

C. do atrial septostomy
D. start high flow nasal 02
E. arrange for home nasal CPAP
9 A 7-year-old
boy with fully corrected tetralogy of Fallot (TOF), 0
complaint f

increasing of orthopnea
clinic, he had and s
to car diology
Echocardiography assessment shows significant pulmonary in >«
- -
( pulmonary regurgitatioj jate management is to

°Tf
OP EE5
give antifailure medications
V with valve replacement
as long term therapy
B
distal branch artery stenosis
C offer balloon dilatation of
advice reassurance and follow -
up
D.
E. use melody valve

10. What is the MOST common


congenital heart disease ?
A. Tetralogy of Fallot
B. Transposition of great arteries
C. Atrial septal defect
D. Ventricular septal defect
E. Pulmonary stenosis

.
11 Which of the following congenital heart diseases can cause
fetal
hemodynamic disturbance and hydrops ?
A. Ebstein anomaly
B . Pulmonary atresia
C. D-transposition of great arteries
D. Severe type of hypoplastic left heart syndrome
E. Patent ductus arteriosus

12 . A young parents are feeling guilty and want to know the cause of their bab\
heart problem ( ventricular septal defect ).
Of the following, the MOST reasonable answer is
A. Most cases of congenital heart disease are multifactorial
B. It is related to racial factors and chromosomal disorders
.
C It runs in certain families with unknown
1
'
inheritance pattern
D It follow classical mendelian
inheritance
E. The cause of most heart defect is
still unknown

13 . A 3 day old baby boy admitted


with poor feeding and recurrent seizures,
had receding chin, cleft palate, heart murmur, n
chest x - ray.
and absent thymic shadow °
n the MOST helpful diagnostic
the follow 6'
'

0
A total and '
ionized calcium test is
g blood gas analysis
,
chromosomal karyotyping
D fluorescence in situ hybridization analysis
E - echocardiography

l4 Which of the following congenital heart defects may be


CATCH 22 syndromes? associated with
A. Perimemranous VSD
B. Tetralogy of Fallot
C. Double inlet single ventricle
D. D- transposition of great arteries
E. Pulmonary atresia

15. You are assessing a full-term boy in the postnatal


ward with features of
DiGeorge syndrome, father ( who is a pharmacist ) is
highly concerned as he had
previously similar affected baby who died because of recurrent infections.
Currently the baby is stable*
Of the following, the MOST appropriate way for genetic
counseling should be
done by
A. sending the baby for FISH analysis
B. sending the baby for chromosomal karyotyping
0. sending the baby and parents for FISH analysis
D. reassurance of the family as the course of illness may be mild
E . sending for metabolic screen

16, You received a


call from staff nurse about the result of routine screening
Pulse oximetry saturation for asymptomatic 24 -hour-old full-term baby in
Postnatal ward. The staff nurse reported that the result of oximetry as follow ;
eht hand 92%, left foot 90% in 2 occasions. You have evaluated the baby and

Und oo murmur, pulses and pressure between upper limbs and lower limbs
°
ave marginal difference.
f the following
, the MOST appropriate next step is to
A - discharge the baby home with reassurance
3 days
discharge the baby home with follow -up appointment after
.
C send for erect chest X - ray
D - send for an ECG
rdiography
an echoca
E. send for
congenital heart lesions is ductal de
the following Pendent ?
17. Which of
A. Tetralogy of Fallot
B. Pulmonary atresia with VSD
of grea t arter ies
C. D-transposition
- trans posi tion of great arteries
D. L
stenosis
E. ASD with pulmonary

the follow ing congenital heart diseases can cause pressure


18. Which of
overload?
A. Dilated cardiomyopathy
B. VSD
C. Ebstein anomaly
D. Atrioventricular septal defects
E. Coarctation of the aorta
>
follow ing state ment s is true regar ding patent foramen ovale
19. Which of the
A . It is a recognized type of atrial septal defec
ts
-right shunt
B. It usually causes significant hemodynamic left
C. It can persist in up to 50% of adults
D. Device closure is needed in case of thromboembolic strok
e
E. Left-right shunt is common beyond neonatal period

20. A 4 - year- old girl with asymptomatic small ASD secundum,


echocardiography
t the timing of
shows no dilatation of cardiac cham bers , Fami ly conc erned abou
closure of the defect .
What is the MOST appropriate answer ?
A . The best time for closure is now
B . We need to watch the girl closely anticipating the complications
C. We need not to worry as it is a small defect and closure is not indica * ^
D. The closure is needed before school entry even it is small to Pre
Eisenmenger syndrome u

E. We can wait till adolescent period when her weight will be good en u °^
to bear operation

21. A 3- year - old girl diagnosed as a case of large ASD secundum, not fit f °
transcatheter closure and need surgery. Echocardiography shows right 3 tr ^ '
429
d right
ventricular dilatation. Family shows rn.. much i
thev see the r Ch ld has no much sympt° ms;
' '
Jaduittiood - ° they¥ want TorePa'rin the
6
to Postponed
it
BEST advice for the family ?
"VVtia y°isurbetter
t is
tobe done as early as
possible
A It
is good chance for spontaneous closure;
B There so i would agree
till adulthood to wait

C. We can wait as
long as she is well and having
normal growth parameters
0 We can do appointment echocardiography every 3 months for
dose
follow-up
E. The best period for closure is just before adolescence

22. Which of the following structures remains normal in size in patients with
large ASD?
A. Pulmonary artery
B . Right atrium
C. Left atrium
D . Right ventricle
E . Left ventricle

23. Which of the following is the characteristic finding in a child with moderate
large size ASD secundum ?
A. Right ventricular systolic lift at left sternal border
B. Grade 3 / 6 systolic ejection murmur at pulmonary area
C. Wide and fixed splitting of S2
D. Wide splitting of S 2 during inspiration
E. pulmonic ejection click

connect ion between one of


24 . Scimitar syndrome represents the anomalous
the pulmonary veins and
A. superior vena cava
B . inferior vena cava
C. right atrium
D. left atrium
E. pulmonary artery
,al
partia anomalous pulmonary
25 . Which type of ASDs is MOSTLY associate with
venous return ?
430
A . sinus
venosus
l SmoTcanal
D Primum
( atrioventricular )
foram en ovale
E Pate nt
MOST mmo
com n card iac defect associated with ostium pr m , urn?
26. What is the deformity of mitral valve
A. Gooseneck
B. Cleft
tricuspid valve
C. Cleft mitral valve
of VSD
D. Supracristal type
E. Sinus venosus type
of ASD

time of surgi cal closure of an atrioventricular septal defect


27. What is the ideal
in Down syndrome ?
A . Neonatal period
B. 3-6 months of life
C. Late infancy
D . Preschool age
E . During second decad

28. In babies affected by atrioventricular septal defects; the


associated Gerbode
shunt can occur between
A. left atrium to right atrium
B. right atrium to left atrium
C. left atrium to right ventricle
D. left ventricle to right atrium
E. left ventricle to right ventricle

29. The VSD type that is MOST commonly associated with aortic regurgitate 1
A . supracristal
B. membranous
C. muscular
D- Swiss cheese
E. inlet

30. In which type of ventricular septal defects, an aneurysm can form | jrr) itir> 8
left - to- right shunt ?
431
supracristal
^B. membranous
c muscular
p swiss cheese
E. inlet

explaining the role of PDA in a case Of


31. You are pulmonary atresia to your
junior colleague .

The MOST appropriate statement is that


A. absence of PDA is such lesion reduces the risk of surgery
B. it represents the only source of pulmonary blood flow
C. pharmacological trial for it is closure is indicated
D. the smaller the size of PDA; the better course of disease
E. heart failure is usually secondary to persistence of such lesion

32. Which of the following conditions simulate clinically patent ductus


arteriosus?
A . Coronary artery fistula
B. Large ASD
C. Severe pulmonary stenosis
D . Pulmonary stenosis with ASD
E . Congenital mitral stenosis

33. A 3- year - old boy with mild developmental delay referred to cardiology clinic
with heart murmur . He has hypertelorism, down- slanting eyes, webbed neck,
short stature, and chest deformity .
What is the MOST likely cardiac lesion associated with this condition ?
A. Supravalvular aortic stenosis
B. ASD
C. VSD
D . Coaractation of aorta
E. Pulmonary stenosis

hmijrmur
34 . A 12 - day - old baby boy who had uneventful perinatal h!
delivery, presents with increasing feeding difficulties, cyanosi
and hepatomegaly. Chest X - ray shows cardiac
and right atrium with decrease mtrapulmonary vascuia
Y*
^^^^
,
!
ventric ) e
peaked R

waves in right chest leads, with tall spiked P waves


.

432
MOST appropriate treatment for this
of the following the conditionis
flow nasal 02
A . t0 S tart high
CPAP
B to start nasal
infusion
C to start PG El
medications
D. tostart antifailure
E. to arrange for
balloon valvuloplasty

35. What is the MOST


common cardiac anomaly that is associated Witt
coaractation of aorta ?
A. Bicuspid aortic valve
B. Parachute mitral valve
C. Subaortic stenosis
D. Anomalous right subcalvian artery
E. VSD

36. A 4 -year -old boy underwent elective surgical correction for coaractation of
aorta, as he recovers from anesthesia; he develops severe abdominal pain
vomiting and hematochezia, his blood pressure 130/95 mmhg, HB 14 gm/d<
and WBC 24,000/mm .
3

What is the MOST likely cause ?


A. Malignant hypertensive crises
B. Acute peritonitis
C. Mesenteric arteritis
D. Fibrinous adhesions
E. Thrombo embolic complication

37. A 5-year -old boy, a known case of coaractation of aorta, developed sudden
very severe headache, nausea, vomiting, blurred vision, photophobia, wlt'
gradualdeterioration in the level of consciousness which ended with
Examination shows stiff neck, B.P 126/86.
Of the following, the MOST likely diagnosis is
A . malignant hypertensive crises
B. ruptured cranial aneurysm
C. thrombotic cerebrovascular accident
D. acute meningoencephalitis
E . brain abscess

433
- month old
- boy had history of recurrent
18 wheezing, frequent hospital
An
Emissions , and failure to gam weight. Examination reveals ,n J ° !!
<J 3 nd SPht S2 ' LCft 3 tnal and right ventricular enlargements
tachyPnea l0Ungostio with
pulmonary ^ , the MOST likely diagnosis is .
n are seen m chest X -ray and the ECG shows bifid P
wave
Lthe following
°A . large
ASD secundum type
B. moderate primum ASD
C coaractation of aorta
p interrupted aortic arch
E. mitral stenosis

39. A 12 -week- old baby boy, a known case of congenital heart disease had
history of wheezing and recurrent lung collapses. Examination shows
underweight baby, O2 saturation 72%, tachypnea and tachycardia, and diastolic
murmur at left upper sternal border.
Of the following, the MOST likely diagnosis is
A . critical pulmonary stenosis
8 . Ebstein anomaly
C. tricuspid atresia
D. absence of pulmonary valve
E. atrioventricular septal defect

left sided chest pain and


40. An 8- year - old girl presents with occasional
with apical late systolic murmur
palpitation, examination shows a healthy girl
* an ' er, wi
^
sitting p ltl0 C
preceded by a click, more prominent in
leads a
° " |
normal and ECG shows biphasic T waves
in
.
unifocal premature ventricular contractions
is
Of the following, the MOST likely diagnosis
A . mitral valve stenosis
B. ventricular ectopics
C. mitral valve prolapse
D . tricuspid valve regurgitation
E. small ventricular septal defect
as a risk factor for graft dysfunction in
41- What that is consid ered
is the finding
cardiac transplantation ?
A . Tricuspid regurgitation
B Mitral regurgitation
434
c. Aortic regurgitation
D . Pulmonary regurgitation
E . Thrombo- embolic phenomena

42 . An 18-month- old boy who underwent open heart surgery f0r


tetralogy of Fallot; developed increased needs for postoperative
flare
^*
^
is associated with sweating and irritability . Lab studies shows incre thil
ECG shows ST elevation in left chest leads , and echocardiography
pending.
assets***^** *
Of the following, the MOST likely cause is
A . transient heart block
B. postcardiomyotomy syndrome
C. interruption of an aberrant coronary artery
D. significant residual VSO with left- to-right shunt
E . right ventricular failure

43. You received a call from a colleague in remote area describing a 5 -day-old
neonate who is having cardiac murmur, cyanosis, cardiomegaly, and an ECG
shows left axis deviation with bifid P waves. No availability of echocardiograpny
in that remote hospital.
Of the following, the MOST likely diagnosis is
A. tricuspid atresia
B. pulmonary atresia
C . atrioventricular septal defects
0, mitral atresia
E . transposition of great arteries

44 . You are counseling parents of a 21-


day-old boy who underwent pulmona V '
artery banding for tricuspid atresia
and large VSD; he is stable now, showingnC
signs of heart failure, and start to gain
weight
They are asking about the ideal time
of next operate for their baby.
Of the following, the BEST response will
be
A . during first 2 -6 months of life
to av id LV dysfunction
B. as late as 10- °
nnonths to gam adequate
weight to stand operation
C . during second year of life
D . 3 - 4 years is the ideal time to do total
correction
E. no time limit is important as are keeping close follow -up for the

43S
heart diseases may have
45-
Congenita
ination. ' ,
characteristic shape on radiographic
ih° on°a‘ string
Win8
fthef
' 0 S USU 3
" V S 6en in Ebste|n anomaly
^
' A0 Boot
.
Egg
shape
appearance
t snow man
Dt Box shape
l Gooseneck shape

45. Vou are explaining the reason behind early surgical correction in D - T6A to a
.
junior colleague Which of the following is the BEST statement describing that
reason?
A . Intervention should be started before pulmonary hypertension become
evident
B. For early correction of associated coronary artery anomalies
C. Left ventricular mass may become less with time
D. Early intervention results in better future growth parameters
E. Better neurological prognosis is observed with early intervention

47. What is the procedure of choice for correction of D -transposition of great


arteries ?
A. Arterial switch
B. Atrial switch
C. Ventricular switch
D. Sano
E. Norwood

^8. The clinical course of patients with double outlet right ventricle without
-

Pulmonary stenosis is usually similar to


A. VSD with a large left - to right shunt
-

B. pulmonary atresia
C. hypoplastic left heart syndrome
D tricuspid atresia
E . D-transposition of great arteries

49 . What is the MOST common type of total anomalous pulmonary venous


return that is associated with obstruction ?
A . Supracardiac type
436
B. Coronary sinus type
C . Mixed type
D. Infracardiac type
E. Right atrial type

50. A full- term baby boy, product of uneventful normal vaginal d


eliv
after birth developed increasing cyanosis and respiratory difficulty, ChefY, * 0n
°
shows a very dramatic perihilar pattern of pulmonary edema and a
blood gas analysis shows hypoxemia and hypercapnia. Mother
^
end
antenatal period regularly with no history of maternal illnesses.
Of the following, the MOST likely diagnosis is
A . persistent pulmonary hypertension
B. respiratory distress syndrome
C . total anomalous pulmonary venous return with obstruction
D . congenital group B streptococcal pneumonia
E. hypoplastic left heart syndrome

51. A full- term baby boy, product of uneventful normal vaginal delivery soon
after birth develops increasing cyanosis and respiratory difficulty, chest X -ray
shows a very dramatic perihilar pattern of pulmonary edema and a small heart,
blood gas analysis shows hypoxemia and hypercapnia . Mother attended
antenatal period regularly with no history of maternal illnesses.
Of the following, the MOST appropriate action is to
A . start prostaglandin El infusion
B. escalate FI02 accordingly
C. start antifailure treatment
D . refer for urgent surgical
intervention
E . start mechanical ventilation

52'
uAr 2 r nth
0
? ' 0ld ,
mf 3nt has historv of mi d
' tachypnea and excessive sweats
wit eedmg since birth increasing with
time, he presents with moderate - severe
respiratory distress, and cyanosis .
Examination shows gallop rhythm with
systolic murmur along the left sternal #
w th penh ar edema, blood gas analysis
color Doppler shows pulmonary veins are
border, chest X -ray shows car
***"
shows hypoxemia and hypercaP '
draining awayV from the heart.
13

Of the following, the MOST likely diagnosis is


A. truncus arteriosus
B . large VSD

437
venous return
total anomalous
n single
ventricle
hypoPlastic left heart sYndrorne
E-

3-davold
neonate' a Pr d»JCt of normal vaginal delivery
5 A °
change in color and poor feeding. Examination shows
presents with
dden webbed neck
* peripheral pulses, cold extremities, palpable right parasternal lift, and

^ patomegaly. ECG shows right ventricular dominance with peaked P wave.


Chest X-ray
attendant to
shows plethoric lung with cardiomegaly. Mother was
the primary health care .
Of the following, the MOST likely diagnosis is
poor

A. tricuspid atresia
B. total anomalous pulmonary venous return
C. L-transposition of great arteries
D. D-transposition of great arteries
E. hypoplastic left heart syndrome

S4. Which of the following congenital heart diseases shows encouraging


antenatal interventional results ?
A. Tricuspid atresia
B. Hypoplastic left heart syndrome
C. Total anomalous pulmonary venous return
D. Severe coaractation of aorta
E. D -transposition of great arteries

55. A 7-month- old boy has history of intermittent vomiting, recurrent wheezing
Parted soon after birth that is exacerbated by crying, feeding, or flexion of the
n ck. The mother informs you that extension of the neck tends to relieve the
*
noisy respiration. Past medical history reveals mild atopy and history of hospital
admission twice because of pneumonia.
0 f the following
, the MOST likely diagnosis is
A. 6ERD
B. Sandifier syndrome
C. severe atopic asthma
D- vascular ring
E. lobe sequestration

438
. .

^ confrer babN* ' ^


#* no ^(ronuecunentbout
* *& £
.

^
** » *r5ss "*rj ri

*'*"+ !

C . O^
«
6
%
& „

< s
S * «/
" r
d
*
=^
'‘“"*w"
e < tto 06’
'* 'mito*chon
'
ost #
A. coaractation of
8. drial
*
aorta
cardiomyopathy
C. dilated cardiomyo
pathy
D. large VSD
origin of left coronary artery from pulmonary artery
E. anom alous
cardiology clinic from asthma clinic for furtho
57. A 6-year-old girl referred to
^

evaluation because of "treatment failure "


. She had history of recurrent
pain, and syncope for the last year
symptoms of dyspnea, fatigability, chest
During examination you find left parasternal heave
with loud S2, increase
X -ray, and tricuspid
jugular venous pressure, cardiomegaly on chest
insufficiency with increase right ventricular pressure on echoc
ardiography.
pressure and
Cardiac catheterization reveals elevated both pulmonary wedge
left ventricular end-diastolic pressure.
Of the following the MOST likely cause is
A. primary pulmonary hypertension
B. pulmonary venous stenosis
C. mitral stenosis
D. restrictive cardiomyopathy
E. cor triatriatum

58. Which of the following can develop early pulmonary vascular disease
( Eisenmenger) ?
A. Atrioventricular septal defect with Down syndrome
B. Moderate VSD with Edward syndrome
C. ASD with Holtoram syndrome
D. ASD with Noonan syndrome
E. Very large VSD with Moderate pulmonary
stenosis
59. A 6- year -old intelligent child with small ASD brought for followup
cardiology clinic; mother claiming he is not eating
439
well and he is not interest
^
o coronary
in p aV
|
:
3| disease
ia
arterl following, the BEST advice/ plan is to
0f the
'
child physical exercise and encourage intake
A restrict
8 refer the child to cardiac center for further evaluation
the nature of the child ' s congenital heart disease
Q emphasize
D call for
whole family counseling session
c make strict rules for the family to protect the child

50. The mother


of a 2 - month- oid infant, who is a known case of cyanotic heart
vaccination is recommended or not .
disease, asks whether
What is your BEST advice ?
A. Routine immunization may lead to infective endocarditis
B. We will postpone the immunization till after surgical correction
C. Herd immunity will be the best protection for your child
D . Routine immunization plus influenza vaccine are needed
E. Selected schedule for your baby should be implemented

61. A young parents of a 4-month -old baby boy with


tetralogy of Fallot, has
months, they have plan
scheduled operation for total correction at the age of 8
to visit mountainous area, they are asking about any
expected precautions to be
taken.
What is your BEST advice ?
A. It is safe journey as all airplanes are pressurized
baby on antibi otic proph ylaxis durmg the journey
B. It is better to put the
time
as there is still 4 months to surgery
C. It is the time to travel now
D. Travel but take care of environmental preca
utions
surgery
E . Postpone the travel till after corrective

ter exercise; with


62. A 13-year-old girl presents with chest P
completely normal physical examination
^ ECG , chest X - ray and echocardiography

accidental finding of tiny PDA .


^
, assessed y
; all shows
who order
rt
normal results apart
r

Of the following, the BEST advice is to


A . go forward for Holter monitoring assessment
of PDA
B. refer to pediatric cardiologist for closure
C start decongestive cardiac treatment
.
440
analgesia and reassurance about cardiac lesinn
n Scribe
simple
-up for cardiac
lesion on 6-monthly ba °
l
^ follow $ es
nsure regula r
to cardiac clinic because of che
-old referr
boy ed '0 1His
63. A year aortic valve W M n , Efe
'
7- shows bicuspi d
and echoc ardio graph y asking about the nrn»' °rr >ai
normal is worried and W gn 5iS - %r
cardiac structures
father
,

answer?
°
What is your BEST finding and " nothing to worry about"
A . This is normal
p is recommended
B. Annual checku
is usually affected
C. Life expectancy
is the most frequent complication
D. Bacterial endocarditis
He may have some degree of stenos
is during adulthood
E.

64. A 4-year - old girl with history of PDA


closure by catheter intervention 2 years
ago; referred from dentist asking for antibiotic
prophylaxis of bacterial
endocarditis before proceeding to a filling of premolar tooth.
What is the MOST appropriate answer ?
A . This is a low risk prophylaxis need only oral amoxicillin
B. This is a high risk prophylaxis and need parental ampicillin and
gentamicin
C . Dental prophylaxis is no longer recommended for all congenital heart
diseases
D. Prophylaxis is needed right now, but it ' s not recommended after 5 years
from catheter intervention
E. No prophylaxis is needed any more for her

65 . Which of the following is the MOST common form of cardiomyopathy ?


A . Restrictive
B. Hypertrophic obstructive
C. Hypertrophic non obstructive
-

D. Dilated
E Non-compaction

66 . What is the MOST


common etiology of dilated cardiomyopathy ?
A . Idiopathic
B. Genetic
C. Metabolic
D . Viral

441
E. Ischemic
-old girl presents
A 2 month
- with 2 weeks historv ,

- - ss sitaa^sa
c7 nf fo
Acuities with excessive sweating. Examination
shows tach
8 respiratory

£Utine r
R0
t
investigational workup shows hypoglycemia
. anH
transaminases.
Of the following, the MOST likely etiology is
A . genetic
B. metabolic
C. viral
D . mitochondrial
E. ischemic

68. A 14-month-old boy presents to emergency room with mild respiratory


distress. Examination shows RR 65/min, HR 170/ min, BP 50/30, poor peripheral
pulses, gallop rhythm, and hepatomegaly. Cardiomegaly on chest X -ray and
dilated LV with poor ejection fraction on echocardiography. Supportive
measures started soon in emergency room.
Of the following, the MOST appropriate step is to start
A. valsartan
B. captopril
C. carvedilol
D. dopamin
E . digoxin

^9. Which of the following is the drug of choice for hypertrophic


cardiomyopathy ?
A. Diuretics
S. p -Adrenergic blocking agents
C . Digoxin
D . Angiotensin-converting enzyme ( ACE ) inhibitors
E . Angiotensin receptor blockers

?0. A 7 year - old child referred by school doctor comp airnng ro . .


episodes of chest pain, palpitation, and occasional syncope
, ec
well in school and quite intelligent . During examination you in

442
precordial impulse, systolic ejection murmur the aortic region not as
in
apical blowing murmur. Referring
^
with an ejection dick, and an $ocj

is family history of sudden ^


in the referring letter that there death
during football play .
is
Of the following, the MOST likely diagnosis
A . severe aortic stenosis
B. severe pulmonic stenosis
C. coaractation of aorta
0 . dilated cardiomyopathy
E. hypertrophic obstructive cardiomyopathy

71. A 6- year - old girl had history of recurrent episodes of cough, mild
shortness
of breath, chest pain, and occasional syncope in the last 12 months, all episode
were treated symptomatically . Examination shows gallop rhythm
hepatomegaly, and mild pitting pedal edema . Mild cardiomegaly on chest X -
ray ,
with dilated atria on echocardiography.
Of the following, the MOST likely diagnosis is
A . dilated cardiomyopathy
B. hypertrophic obstructive cardiomyopathy
C severe aortic stenosis
.

D pulmonary hypertension
.

E . restrictive cardiomyopathy

72. What are the MOST common pediatric cardiac


tumors ?
A . Rhabdomyomas
B . Fibromas
C . Myxomas
D Sarcomas
-

E . Metastatic tumors

73. Which of the following


an
myocarditis ?
IS
°minous complication in patients with acute
A . Shock
B . Arrhythmia
C . Thrombo - embofism
D Congenstive heart fail ure
E . Dilated cardiomyopathy

443
Y0U are starting digitalization for a baby with heart failure
an ECG strip were done before Basic rmetabolic
.
66
r0|yte, and the first digitalizati ,
8 hours you are
willing to give the 2 d nd ion dose .
No* ' after vitalizing dose
following, the MOST appropriate next step is to
0f the echocardiography
A repeat

digoxin level
B measure
c repeat ECG strip
0. check renal function tests
£ repeat serum potassium

75 . Which of the following electrolyte disturbances can increase digoxin


toxicity ?
A. Hypokalemia
B. Hypocalcemia
C. Hyponatremia
D . Hyperkalemia
E . Hypernatremia

76. You are managing a 2 - year - old boy who develops heart failure after an open
cardiac surgery for tetralogy of Fallot . He has only minor response to dopamine .
Of the following, the MOST appropriate next step is to add
A . epinephrine
B. isoproterenol
C. milrinone
D . digoxin
E. furosemide

77. Which of the following inotropic medications used for cardiogenic shock can
increase urine output when given at low doses ?
A. Epinephrine
B. Isoproterenol
C. Milrinone
D. Dopamin
E. Dobutamine
is controlled
78. A 6- year - old boy emergency room with SVT which
presents to
,, a ECG revea s s o
successfully by 2 dose of adenosine . Post episodic

444
upstroke of the QRS . His echocardiograp V
interval and slow reveals syncope.
historyr
c rne
^ V ^
Antecedent MOST appropriate next action is to
«

, the
Of the following therapy
A. start digoxin
prophylaxis
B. start amiodarone
C. start propranolol
D. order Hotter monitor study
E . refer for electrophysiological

to emergency room with history 0f jrrit »


79. A 6- month- old girl presents 1!
and off - color appearance . Her pulses is intact and
excessive crying, ^ Pillar,
monitor shows narrow complex tachycardi
refill is <2 seconds, cardiac 13 WITT
the rhythm with a
heart rate of 220/min. you are unsuccessful to convert
an ice pack over the face.
Of the following, the MOST appropriate next step is
^
A . carotid massage
B. adenosine
C. verapamil
D . propranolol
E. synchronized DC cardioversion

80. A 3- year- old boy who presents to emergency room with SVT successful ,
converted to sinus rhythm after 3 doses of adenosine. His post episodic ECGanc
echocardiography are normal.
Of the following, the MOST appropriate next step is to
A. send for Holter monitoring
B. send for electrophysiological study
C. start digoxin
D . start amiodarone
E . start propranolol

81. A 5 - year-old boy who is


a known case of WPW syndrome on amiodsro
treatment for the last 3 months came
for follow - up.
Of the following, the
MOST likely indicated investigation is
A. thyroid function
test
B. renal function
test
C. HbAlc
D. echocardiography
445
E. ECG

on duty in PICU when you notice


yoU arG
82- rhythm
0ld infan
with acute myocarditis . You noticed an conversion iin a 9-month-
tachvcardia. Vital signs are; HR 180 emergence of regular wide
beat/min, B.P 80/60 mmHg
w th
|
following, the MOST appropriate management
°T start CPR
f is to

give DC defibrillation
give synchronized DC cardioversion
0, start amiodarone
£ start dysopyramide

83. Which of the following antibiotics can cause prolong QT interval?


A. Cefuroxime
B. Trimethoprim/ sulfamethoxazole
C. Amoxicillin- clavulanic acid
D . Ceftriaxone
E. Tobramycin

34. All the following medications can cause prolonged QT interval EXCEPT
A . fluconazole
B. imipramine
C. domperidone
D. ondansetron
E. metocloporamide

85. A 5- year- old boy brought to emergency room after he experienced syncopal
episode after exercise. The patient had been diagnosed as seizure disorder
and
he is on valproate for previous 2 similar episodes. His older sib who was
shows a heart
described as smart swimmer died as a result of drowning. His ECG
rate-corrected QT interval of 0.5 sec.
Of the following, the MOST likely diagnosis is
A. LQTS type 1 ( LQT1)
B. LQTS type 2 (LQT2 )
C. LQTS type 3 (LQT3 )
2 degree heart block
nd
D.
E. complete heart block
446
ith tetralogy of Fallot and B- T shunt had d
86. A 4 - year -old
boy
'
wi
high grade fever
enta\proc.
ikes of , rigors, ,
1 week ago. INow
he
h0WS moderate size vegetation attached
and sptenor L
' * to
^<
%
«

Echocardiography puW,Or,;.
valve.
following, the MOST likely causative organism is
Of the
A. Staphylococcus aureus
B. Streptococcus viridans
C. group D enterococci
D. Streptococcus pneumonia
E. Coagulase-negative Staphylococci

87. First episode of acute rheumatic carditis often results in


A. complete healing of the valvular lesions
B. chronic rheumatic heart disease
C. subacute prolonged carditis
D. involvement of almost all valves of the heart
E. bacterial endocarditis
apter 19
^ The Cardiovascular System
Answers
Q

that
,
.J SiSE SE -

entersThe
n3 al period Approximately 50% of the umbilical venous blood
nepatic circulat on, whereas the rest bypasses the liver and joins
'
the inferior

^
vena Va

U S Venosus where il
' Partially mixes with poorly
oxygenated IVC blood derived from the lower part of the fetal body .
2.( A) Approximately 5% of right ventricular ( RV ) outflow enters
the lungs The
major portion of this blood bypasses the lungs and flows right -
to- left through
the ductus arteriosus into the descending aorta to perfuse the
lower part of the
fetal body, including providing flow to the placenta via the 2 umbilical
arteries.
.
J|B) In acyanotic infants with large left - to-right shunts, the onset of heart
failure often coincides with the nadir of the normal physiologic anemia of
infancy. Decreasing pulmonary vascular resistance is a dynamic and continuous
process starting after birth and continued till few weeks later.
4 (C) In cyanotic patients with inoperable conditions, partial exchange
transfusion may be required to treat symptomatic ( most often headache or
chest pain) individuals whose hematocrit has risen to the 65-70% level.
polycythemic patients with concomitant iron deficiency are at even greater risk
for cerebrovascular accidents, and needs treatment with iron but in this
scenario serum ferritin is normal.
This procedure is not without risk, especially in patients with an extreme
Ovation in peripheral vascular resistance. Because these patients do not
tolerate wide fluctuations in circulating blood volume, blood should be replaced
,
w th fresh frozen
® * ( C)
pr ious
0f
^
plasma or albumin.
Screening fetal echocardiogram is recommended for women w
.,
child or first -degree relative with CHD, for those who are at h
gher ns
aving a child with cardiac disease ( e.g., insulin -dependent dia et C pa
°
^
w men exposed
,n whom a to teratogenic drugs during early pregnancy
chromosomal abnormality is suspected or confirme
), an
. ^ ..
'

40
ges
*
Ucta !
0r

dependent les transport of an unstable infant. ^Oid,<ng


^
the require

f rind
?
^
segment ) in conjunction
js considered abnormal if the ST segment deprest

with a horizontal
after the i point ( onset of
extends for at least 0.06 sec , upward, or
downward
^J
Lent A decrease in blood
pressure before maximal exercise is reached , .
2 indicator in patients with
8 (C) Balloon atrial septostomy
hypertrophic cardiomyopathy
(Rashkind procedure ) is the procedure
0‘
good in first few days as the ductus arteriosus is
choice. Option A is opened
( pulmonary atresia ) . Options
Option B is good for lesions with RVOT obstruction
D and E are for respiratory rather than cardiac problem.
9.(E ) One of the greatest advances in interventional catheterization over the
past decade has been transcatheter valve implantation , Typically , a porcine
valve is sewn into an expandable stent , which is then collapsed around a balloon
catheter . At this time, the most common application in children is replacement
of the pulmonary valve ( Melody Valve ) In patients who have had prior surgery
for tetralogy of Fallot ( usually because of residual pulmonary insufficiency ) and
antifailure medications can be used as short -term therapy meanwhile arrange
the procedure.
10.( D) Ventricular septal defect accounts to about 30 - 35% of congenital heart
diseases; atrial septal defect 6- 8%; tetralogy of Fallot and pulmonary stenosis 5*
7 % and lastly transposition of great arteries - %
35 .
ll.( A) Most congenital defects are well of the
tolerated in the fetus because
parallel nature of the fetal
circulation. Even the most severe cardiac defect 5 '
such as hypoplastic left heart
syndrome, can usually be well compensated for by
he fetal circulation. One

anomalvTAV ? ^
the parallel fPt
notable exception is the case of severe regurgitant
* 1^ °f the tricuspid vs1ve - In these lesions, such as Ebste n
v 3 lve or severe r ght
‘ ventricular outflow obstruction
, -
The
S'Sss >« w
"
in m 510
QU f 3ch

^
t0
the Parents is that, the cause
congenital heart defpr , °
problem. Many unknown and parents are not responsiblefort
of senetic
cases of run mult | factorial and results from a comb ns ® , *
predisposition
stimulus. A small percentae
/l ^ as Vet - t0-be-determined environR en
'

cbromosomal abnormal 0t. C°neenital heart lesions are related to W V


,
"
^°ther Beetle
A
factors may have a role in 0
t *
w
"
, ' '“
WDi s ,p s
" “* « ^
-Irheandinvestigations choiceot for management
is RRS ,
echocardiography but ; the investigation
that ie\ V and ioni2ed
c3
nGeorge syndrome
* Lrative
is fluorescence in situ
hybrid, ? *'°" °
V U t0 diagnosis
lF SH) analvsis.
C
genome hybridization has in many ca
'
P C6d routine
karY
;:
otvPingin
(6) Question
'
the cliniCa W°rkup 0f newbor s with CHD
"
is asking indirectly about conotruncal
cardiac M .. „
**esions, associated
with 22 qll .2 deletions are most often seen in ?
'

with ejther the OiGeorge syndrome or the Shprintzen (


velocaS?
ome. The acronym CATCH 22 has been used to summarize the
components of these syndromes: cardiac defects, abnormal facies thymk
2
aplasia, deft palate, and hypocalcemia. The specific cardiac anomalies are
conotruncal defects (tetralogy of Fallot, truncus arteriosus, double- outlet right
ventricle, subarterial VSD ) and branchial arch defects (coarctation of the aorta,
interrupted aortic arch, right aortic arch ). Congenital airway anomalies such as
tracheomalacia and bronchomalacia are sometimes present.
15.(C) Although the risk of recurrence is extremely low in the absence of a
parental 22qll. 2 deletion, it is 50% if 1 parent carries the deletion. In this
scenario there is high possibility of one parent is carrier .
16.(E) Screening for duct - dependent cyanotic congenital heart disease is
performed between 24 and 48 hr of life and before discharge in asymptomatic
newborns. A pulse oximetry saturation of 90-94% in the right hand or either
loot requires urgent echocardiography. A pulse oximetry saturation <95% in
either location or a saturation difference >3% between the right hand and
either foot is considered a positive test and should be repeated in an hour; if
Positive again, it should be repeated in another hour. If it remains positive,
is indicated. In addition, a careful reexamination of the pulses
Kl ocardiography
'
ind blood pressure as cardiac
in the upper and lower extremities as well
auscultation are indicated in children with an initial positive screen. oxrmetry
181 Multiple studies demonstrate the benefit of routine pubf:
cyanotic CHD; leswn
** " '
ni « or all
newborns to detect unsuspected critical t e t r a o g y o f F a o .
2?? « hyp c left heart syndrome, pulmonary atresia
ano »lous pulmonary venous return, transposition of < e '
f

" .
> TKh
d «resia

%) e hYPOplasia
truncus arteriosus,
/ atresia -
neonatal coarctation of the aorta,

volume load are those


% Cau most common lesions resulting in
increased septal
defect ( A 5 D) ventricular
* left - to-right shunting : atrial septal
Acn
.
defect (VSO), atrioventricular
includes regurgitant
lesions
septal defects and patent ductus
and the
of
dilated
lesions
artery
cardiomyopaK s ,
^
pathophysiology common denomi nator resulting jn • '
to normal blood flow .The most fre
pressure load is an obstruction
obstructions to ventricular
stenosis and coarctation
outflow: valvular pulmonic stenosis, valvula
of the aorta. Less
,
common
..
are obstru
r
Cti
^obstruc'
0ns
'<
^
"* ,**
ventricularinflow: tricuspid or mitral stenosis cor tr atr atum, and
of the pulmonary veins.
t
foramen ovale (PFO) is a
19.iD } An isolated valve -incompetent patent commQr
echocardiographic finding during infancy. It is usually of no hemodynarnjc
significance and is not considered an ASD; a PFO may play an important role if
other structural heart defects are present. PFO may be present in 15-30% 0f
adults. An isolated PFO does not require surgical treatment, although it may be
a risk for paradoxical ( right to left } systemic embolization. Device closure ot
these defects is one treatment option considered in adults with a history of
thromboembolic stroke.
20.(C) In patients with small secundum ASDs and minimal left -to -right
shunts
without RV enlargement, the consensus is that closure is not required. Infants
with small to moderate- sized ASDs can be watched closely, since these
defects
will often grow smaller in the 1st yr of life.
Transcatheter device or surgical
closure is advised for all symptomatic patients,
as well as for asymptomatic
patients with Qp:Qs ratio of at
least 2 : 1 and those with RV enlargement. The
timing for elective closure is usually
after the 1st yr of life and before entry into
school.

,
term risk of arrhythmia causer h .
,
slgn flcantlV greater
in adulthood; the long
V C r mC atrial
repair in adults. ° dilation is also greater after ASD

'* ^
^ * hf
enlargemern o the r *
ht atrium
the r Bht side of the heart results in
'
artery. The left atrium
may also .Ventricle and Nation of the pulmonary
flow returns to the left
si e -
*
atrium
' the , 3 S the
incfeased pulmonary blood
eft ventricle and
aorta are normal in
23.( C) All except D are
the characteristic finding
true, but C is Cha
is that the sero ' ‘
,
St C n most
Patents with an ASD
fixed in its splitting during all sound (S2 ) is widely split and
phases

451
.„1 Occas onallv' m PAPVR' an anomalous vein dra
24 radi'ography as a crescentic shadow
Jest - ,
the Vc s
° card iac silho uett
° der ofhenan assoc ated ASD scim e (
of -'•
visible

.
i
the itar syndrome). alon6 »e right
jS,(A) ^ '
secundum.
is Present, it is
generally
V of
pe but can be 'Qt the s
nus venosus
rtlC) In m05t cases a cleft m the anterior leaflet of thQ
po,td . The tricuspid valve is usually functionally normaT
3na omic
abno rmal ity of the septal leaflet is prese
nt. The
ventric^ *
l °
also
S me
septum is
intact -
27 (B) The risk for development of pulmonary vascular ,
disease s pr«,
patients with Down syndrome, and therefore surgical
considered early in these patients, within the 1st 3 -6 mo corre ction lZl„
of life, while inno
syndromic babies the timing of operation is between
6-12 months
28.(0) In complete AV septal defects, left-to-right
shunting occurs at both the
atrial and the ventr icular level. Additional shunting
may occur directly from the
left ventricle to the right atrium (known
as a Gerbode shunt) because of
absence of the AV septum.
».(A) A supracristal (subarterial) VSD can be complicated by prolapse
aortic valve into the defect and aortic insuf of the
ficiency, which may eventually
develop in 50-90% of these patients. Altho
ugh supracristal VSD accounts for
approximately 5% of all patients with VSD, the
incidence is higher in Asian
children and in males. Although most common
with supracristal VSDs, aortic
insufficiency is occasionally associated with VSDs
located in the membranous
septum
In defects of the membranous septum , a thin membrane (
called a
ventricular septal aneurysm but consisting of abnormal tricuspid valve tissue)
can partially cover the
defect and limit the volume of the left -to-right shunt ,
PDA is seen in 10% of patients with other congenital heart lesions and
often plays a critical role in provi
ding a source of pulmonary blood flow when
the right ventricula
r outflow tract is stenotic or atretic or in providing systemic
blood flow in the presence of aortic coarc
tation or interruption. It has beneficial
effects and works as palliation.
32- ( A) A congenital
fistula may exist between a coronary artery and an atrium,
ventricle (especially the right ), or pulmo
nary artery. Regardless of the recipient
chamber , the clinical signs are similar to those of PDA, although the machinery -
like murmur may be
more diffuse.
50% of cases .Surger
stenosis, urgent treatment asia.
34.(0 a neonate
balloon

^
valvuloplasty
ca present in similar
^
jth crjtjca| pu monic
or surgical
way is
is
|
valvotomy is warranted. The
persistent
unlikely.
pulmonary hypertension
£
,
condition wf)Kh
bo SrT>
Oott)
make this possibility
Coarctation „
i

SX in males as in females.
flrv

anomaly occurs twice as often

“ la
e may be a feature
aortic valve in > 70% of
of Turner syndrome and » associated with a
patients . Mitral valve abnormalities ( a
bicUSN
supravalvoi
mitral ring or parachute mitral valve
) and subaortic stenosis are potential *
left sided obstructive lesions
associated lesions. When this group of occurs
together, they are referred to as the Shone complex .
36.{ C) Postcoarctectomy Syndrome; Postoperative
mesenteric arteritis may be
associated with acute hypertension and abdominal pain in the immediate
postoperative period. The pain varies in severity and may occur in conjunction
with anorexia, nausea, vomiting, leukocytosis, intestinal hemorrhage, bowel
necrosis, and small bowel obstruction. Relief is usually obtained with
antihypertensive drugs { e.g.,nitroprusside, esmolol, captopril) and intestinal
decompression; surgical exploration is rarely required for bowel obstruction or
infarction.
37.|B ) Severe neurologic damage or even death may rarely occur from
associated cerebrovascular disease. Subarachnoid or intracerebral hemorrhage
may result from rupture of congenital aneurysms
in the circle of Willis, rupture
of other vessels with defective elastic and
medial tissue, or rupture of normal

^
heSe aCddents are secondary to hypertension
«
S
/
38.[ E ) If congenital mitral
.
annpar u ithin
/

'
i
,
stenosis is moderate to severe, symptoms usually

^^
degrees of dyspnea and haVe fai ure t0 thrive and
allo)r lp some patients,
symptom, and a misdiagnosi wheezing may be a dominant
have been made. Heart
right ventricle and left
enlargememT'^ '"' 0 5
airW3 V diSe3Se
ecause of dilation and hypertrophy of the
^
atrium i
diastolic murmurs, but the
loud and split . An opening
^ f
ost Pat > ents have rumbling epics
ndmgs
'
snap of ' maV be relatively obscure . S 2 *
reveals RVH and may show *
bifid nr valve may be present . The ECG
enlargement. Radiographs
*'
dually sho
pulmonary congestion in a HDerihibr
p
waves indicative of left atrial
,
r al anc RV enlargement anu
innar or
venous Pattern . *
453
Sabl e
^hyphemia
. Florid pulmonary valvular incompetence may are
not be we tolerated
1death may occur from a combination of bronchial

,
compression ,
3nd heart
fa
40.(C) M tral
|lure
,
valve Pro aPse r su ts rom an abnormal mitral
* ^
Jat '
causes billowing of 1 or both mitral
valve mechanism
leaflets, especially the posterior
toward the end of systole . The abnormality CUSP
into the left atrium is predominantly
congenital but may not be recognized until adolescence or adulthood. Mitral
valve prolapse is usually sporadic, is more common in girls, and may be
inherited as an autosomal dominant trait with variable expression. This lesion is
not progressive in childhood, and specific therapy is not indicated . Antibiotic
prophylaxis is no longer recommended during surgery and dental procedures.
41.( A ) Tricuspid regurgitation is seen in up to 30% of children after heart
transplantation, which can be a risk factor for graft dysfunction ,
42.(C) Uncommon immediate postoperative problems include RV failure ,
transient heart block, residual VSD with left - to- right shunting, and myocardial
infarction from interruption of an aberrant coronary artery . Scenario shows
classical features of myocardial infarction results from interruption of an
aberrant coronary artery, the anomaly that is present in about 5 - 10% of TOF
cases.
43.( A) The key for diagnosis in this scenario is association of cyanosis with
superior axis or left axis and the age of presentation which fits tricuspid atresia ,
AV canal have the same ECG finding but neither the age nor the associated
gnosis is fitting.
«.(A) The next stage of palliation for patients with tricuspid atresia involves the
creation of an anastomosis between the superior vena cava and the pulmonary
arteries (bidirectional Glenn shunt ). This procedure is performed usua y
,
between 2 and 6 mo of age.
5 - ( D) The that are most
cardiovascular imaging signs of congenital anomalies transposition
often seen in radiologic practice
f the great
Include
, egg on a string
( side ) in
venous return
° arteries, snowman in total anomalous pulmonary
scimitar in partial anomalous pulmonary venous return, o
endocardial cushion defect , figure of three and reverse figure o

454
shaped heart in tetralogy of Fallot, bo* -shaped
coarctation* boot - K i (T|

SESsssSSSSs
Cdeles
^ ,
as PVR declines after
pu monary va u ar b
birth, pressure in the left ventricle (connected *
. This pressure drop results in a
declines deCr
cho<c

' '
n P Ven
operation is
'
attempted^ ^
after
left ventricle will be unable
LV
^^
pressure
to generate
.
few weeks of m

adequate pressure to
|f

pump
^
arteria| switch
(and mass) has declined too farr,
b|
the
ood t0
the high pressure systemic circulation
and Norwood procedures are used for hypoplastrc left heart
47.(A ) Sano
syndrome.
48. A) In double-outlet right ventricle
without pulmonary stenosis, both the
aorta and the pulmonary artery arise from
the right ventricle. The only outlet
from the left ventricle is through a VSD. In the absence
of obstruction to
pulmonary blood flow, clinical manifestations are similar to those of an
uncomplicated VSD with a large left -to-right shunt. Systemic desaturation may
result from mixing of oxygenated and deoxygenated blood in the right ventricle.
49.(D) Infracardiac type associated with obstruction in 95 -100% of cases.
50.(C) In neonates with marked pulmonary venous obstruction, the chest
radiograph demonstrates a very dramatic perihilar pattern of pulmonary edema
and a small heart. In option A the presentation is usually similar but the cardiac
shadow is normal or increased with decreased to normal perihilar shadow. In
option B the patient is usually preterm baby. In C it is typical for TAPVR with
obstruction. In D neither antenatal history nor CXR finding are suggestive. In E it
is ductal dependent so the presentation will be later (
after few days).
51.( D) Obstructed TAPVR is a pediatric
cardiac surgical emergency because
prostaglandin therapy is usually not
effective. Option B C, and E all a c
important supportive measures
but delay in decision taken in such situations '
may lead to death .

.. .
. .
52,(C) This is the classical presentation
, of TAPVR without obstruction In
cases without obstruction, the heart k „„i, . ,
pulmonary arterv
.
an
.
, „ ht
ventricle are prominent, and pulmonarv u
lncreased'
of any vein with Doppler flow away from *
° ,
the
Option A, B, and D have similar course nf hMrt S Path gnomonic f TAP
clinical findings.
' °
events with some differences in h
15 is classical presentation of
^ LHHS

^* inance
do 1
absent

Stnutular
Jrifestomeationands
Rubinstein- Taybi syndrome.
,
of the syndrome may be
In these
evident and ,,
Ll? *
' Holt - rani
tanCes’ noncardi
« °
ma*11
outcomes -
Occasionally it is familial and
inherited as an
06
'Lllio"
CNniCal
trait - recessive

reduces flow through the left ventricle during development, resulting in


gradual
ventricular chamber hypoplasia. The potential for preventing this hypoplasia has
been demonstrated by performing in utero aortic balloon valvuloplasty
in
midgestation fetuses. Early results are encouraging, although even if
the aortic
valve is successfully opened, adequate ventricular growth occurs in only about
30% of patients. At present, this procedure is regarded as experimental.
55.(D) If the vascular ring produces compression of the trachea and esophagus,
symptoms are frequently present during infancy. Chronic wheezing is
exacerbated by crying, feeding, and flexion of the neck. Extension of the neck
tends to relieve the noisy respiration. Vomiting may also be a component.
Affected infants may have a brassy cough, pneumonia, or rarely, sudden death
from aspiration. Echocardiography in combination with either MRI or CT used
for diagnosis.
56.(E) In anomalous origin of the left coronary artery from the pulmonary artery
( AlCAPA ), the blood supply to the left ventricular ( LV ) myocardium is severely
compromised. Soon after birth, as pulmonary artery pressure falls, perfusion
pressure to the left coronary artery ( LCA ) becomes inadequate, myocardial
lschemia, infarction, and fibrosis result. In some cases, interarterial collatera
anastomoses develop between the right coronary artery ( RCA ) and I- •

-
flow ir> the LCA is then reversed, and it empties into the P a
“' "
ss2= it2=222?« ^r;:
Sequent complication secondary to a dilated valve ring or
Papillary muscle.
< with only minor
” D> This description
is fit for all
anges. The echoca
the ment ioned distra
rdiography and cardiac catheteriza
cted
can differentiate

456
between different types of
left-sided obstructive lesions (pU|mo
stenosis mitral stenosis, restrictive cardiomyopathy) that result i°
nafV ve „
venous hypertension. The presence of
pulmonary arterial hypert
ens %
,.
'
wedge pressure ion lpAH
with a normal pulmonary capillary is diagnostic
of PM. |
v,fth
wedge pressure is elevated and left ventricular end- diastolic pressure vn
*
at the level of the pulmonary veins , left
normal obstruction atrium or
valve should be suspected. If LVEDP is also elevated, the diagnosis of
cardiomyopathy should be entertained. The risks associated with
restnctw
card *,
catheterization are increased in severely ill patients with primary PH. **
58.(A) Pulmonary vascular disease occurs more rapidly in patients with trinsofty
21 who have left-to-right shunts.
.
59 (C) Emphasizing the nature of CHD will reduce anxiety of the parents and
also reduces the reflected anxiety on the child himself. Calling for whole family
counseling session is not indicated and may cause horror. Most patients who
have mild congenital heart disease (CHD) do not require treatment The parents
and child should be made aware that a normal life is expected, and that no
restriction of the child 's activities is necessary. Overprotective parents may use
the presence of a mild congenital heart lesion or even a functional heart
murmur as a means to exert excessive control over their child's activities.

zrbsSEtSS °
unexpressed fear of sudden death and th
*^
-" SEKE
Sease The familV may have an
' f th S manifestation should
'

'
be emphasized in discussions directed at provinS their understanding of the
child s congenital heart defect . Thp H H etween CHD and degenerative
coronary disease in adults should he , ^-
as zed Ger>eralhealth maintenance,
V '
including a well-balanced "heart-health " let aerot
' >ic exercise, and avoidance
o smoking, should be encouraged
-
60 (D) Routine immunizations should h g
- ,*
ven » h the inclusion of influence
vaccine during the appr0prjate
, ’
r0phy axis against the respiratory
syncytial virus (RSV) s recommended d "
significant JT* 1® seas young infants with
unrepaired CHD and °"
m dvnamic abnormalities . Careful
consideration of the timing of administrat° ,
Ve'virus vaccination is required

° '' °
, fTh
in patients who are potential candidates
heart r beart-lung transplantation,
and these patients cannot receive ive v vaccines after they have received
their transplant.
61 (C) In cyanotic patients high altitudes ana sudden
changes in the therm9'
environment should also be avoided, as b0th can
result in either respir3 t fV °
457
in
,.ction
n
*hich
6
COuld be severe or Precipitation
eneral cvanotic patients should
of TET ,«P'
1,S „
dUe to high

Jydration‘
> ide also avn H

^
may occur, which leads to increased viscosity
and increase "!

important
Diuretics may need to be decreased or temporarily

in
concentration
hem0globin . r M
despite polycythemia .
dis o
Ling epi es of acute gastroenteritis. Treatment of iron deficiency anemia
^ cyanotic patients, who may have a low
mean corpuscular^
ued
is
'

62.(0) The pam follow strenuous exercise in a healthy adolescent is usually


in origin. A silent PDA is a tiny defect that cannot be heard by
muscular
auscultationand is only detected by other means such as echocardiography. Life
expectancy is always normal in this population and the risk for endocarditis is
extremely low .
63.{ E ) Bicuspid aortic valve is the most common congenital heart disease that
manifest during adult life . By the age of 45 yr, approximately 50% of bicuspid
aortic valves will have some degree of stenosis . Usually have normal life
expectancy .
64.(E) Only patients with cardiac conditions associated with the highest risk for
adverse outcomes should continue antibiotic prophylaxis before surgery:
previous endocarditis; unrepaired cyanotic CHD, including palliative shunts and
conduits; completely repaired congenital heart defects with prosthetic material
or device, surgically placed or by catheter intervention, during the 1st 6 mo
after the procedure; and repaired CHD with residual defects at or adjacent to
the site of a prosthetic
[ patch or prosthetic device ( which inhibits
prophylaxis is
endothelialization ). Except for the conditions just listed, antibiotic
no longer recommended for other forms of CHD .
cardiomyopathy in
65.{ D) Dilated cardiomyopathy, the most common form of
well as the most
children, is the cause of significant morbidity and mortality as
common indication for cardiac transplantation.
idiopathic, its likely
66.( A) Although the most common etiology of DCM remains .
myocarditis p r e 0
that undiagnosed familial/genetic conditions and
rare in c i ®
Unlike adult patients with DCM, ischemic etiologies are
18 Yr is
annual incidence of DCM in children younger than
100,000 per year . Incidence is higher in males, blacks, and
67.( 6 ) in dilated cardiomyopathies, the presence of
in ants
hypog ycern
suggests an inborn error
'
.V

^
aodos,.
hypotonia , or signs of liver dysfunction
Neurologic or skeletal muscle deficits are
disorders or muscular dystrophies.

458
* care"measures
hea
«
* » 10 Pa
* i n tensi v e /;
required
^
6;
circulatory
collan
iapse

°^ 5* •
(J diuretics, mechanic ventilatory supp
intravenous .
chanical circulatory support
occasion, me
competitive sports and strenuous physical
69.(0) Restriction from
hiehlv recommended, and
additional recreational exercise activities
each individual based on their overall clinical status P 'A d
tailored to
blocking agents (propranolol
( verapamil) may
, atenolol , metoprolol) or
calcium channel
be useful in diminishing LVOT obstruction, modify
and improving ventricular filling. They also confer
LV
bi
'


hypertrophy,
antiarrhythmic benefit and may reduce symptoms. Digoxin is contraindicated *
70.(E) Presence of family history of sudden death and absence of ejectsn click
makes diagnosis of severe aortic stenosis is unlikely. Many patients with
hypertrophic cardiomyopathies are asymptomatic, and 50% of cases present
with a heart murmur or during screening when another family member has
been diagnosed with HCM. Symptoms of HCM may include palpitations, chest
pain, easy fatigability, dyspnea, dizziness, and syncope. Sudden death is a well -
recognized but uncommon manifestation that occurs during physical exertion
Characteristic physical examination findings include an overactive precordial
impulse with a lift or heave, a systolic ejection murmur in the aortic region not
associated with an ejection click, and an apical blowing murmur of mitral
insufficiency. In critical PS cyanosis is predominate.
71.( E) Dramatic atrial dilation can result from the abnormal ventricular
myocardial compliance and high ventricular diastolic pressure. Abnormal
ventricular filling, sometimes referred to as diastolic heart failure, is manifest in
the systemic venous circulation with edema, hepatomegaly, or ascites.
72.(A) The vast majority of tumors originating from the heart are benign
Rhabdomyomas are the most common pediatric cardiac tumors and are
associated with tuberous sclerosis in 70-95% of cases.
73 ( B ) AI 1 types of arrhythmia
*
can presents in viral myocarditis, the most
dangerous and can lead to death is
ventricular tachycardia .
74.( C) The ECG must be closely
monitored, and rhythm strips obtained before
each of the 3 digitalizing doses. Digoxin
should be discontinued if a new rhythm
isturbance is noted. Prolongation of the P - R interval is not necessarily an
indication to withhold digitalis, but a delay in administering
the next dose or
3
reduction in the dosage should be considered, depending 's
on the patient
clinical status. Minor ST segment or T- wave changes
are frequently noted with
digitalis administration and should not affect the
digitalization regimen.
4 S9
.oaseline serum electrolyte levels should be measure B
75 ( ALation - Hypokalemia and hypercalcemia exacerbate
*
dST' .toxlc***,tV'-
'
W13 hypokalemia is relatively common in patients rer 8 a s

potss '
^Lcarditis
po
' ^
ssium
ting
levels should be monitored closely in those
iuretic in
. .b
HC7 'ntati°
receiving
n with d gl iis. In patients
' ^ ^
vL
, some card olog sts recommend avoiding digitalis altogether
digitalis should be started at half the
um
active
and if
used, maintenance normal dose without
because of the increased risk of arrhythmia in these patients
italization is useful in treating patients .
^
76.(C) Milrinone
with low cardiac output who are
refractory to standard therapy . It has been shown to be highly
effective in
the low - output state present in children after
managing open heart surgery.
Milrinone has both positive inotropic effects on the heart and peripheral
vasodilator effects and has generally been used as an adjunct to dopamine or
dobutamine therapy in the intensive care unit.
77.(D)
78.(E) Patients with WPW syndrome have a small but real risk of sudden death.
Risk stratification, including 24 hr Holter monitoring and exercise study, may
help differentiate patients at higher risk for sudden death from WPW . Syncope
is an ominous symptom in WPW, and any patient with syncope and WPW
syndrome should have an electrophysiology study ( EPS ) and likely catheter
ablation. Although most often presenting in patients with a normal heart, WPW
syndrome may also be associated with Ebstein anomaly of the tricuspid valve,
hypertrophic cardiomyopathy or L-TGA .
79.(B) Do not loss a lot of time in vagal stimulation, adenosine is the drug of
choice. Verapamil can cause cardiac arrest and severe hypotension in children
less than one year and contraindicated in patients with WPW. Synchronized
cardioversion reserved for patients who shows signs of severe heart failure.
80*(E) Once the patient has been converted to sinus rhythm, a longer acting
antegrade
aSent is selected for maintenance therapy . In patients without an
accessory pathway (non- WPW ), the 3- adrenergic blockers are the mainstay o
drug therapy . Digoxin is also popular and may be effective in infants, ut ess so
in older children. In children with WPW, digoxin or calcium channel bloc ers
through the bypass
maY increase the rate of antegrade conduction of impulses
tract, with the possibility of ventricular fibrillation, and are therefore
contraindicated.
® MA ) Hypothyroidism or hyperthyroidism, elevated triglycerides, hepatic
toxicity , and pulmonary fibrosis all are possible side effects of amiodarone.
Btric0\ar »chVC3' ’a VT who are hemodynaft1ic
„ yyith venU
Vldocaioe. or Procaioamide is the MM 4, ,
s "wei>:; oV
« •
w search for and con** «
**d*,
0r

^
underlying
w
^Sersioo

Heto reserved r pat,erts 'enng
atel ,
. opt
'° * *
from Vr .

r r“
84.(E)
85.(A) LQTS type 1 (
~LQT1)
in clarithromycin, azithromycin,

events
occur
are
at rest,
. „ . .
especia lly during
,
sleep
telithromYcrn

usually induced by stress or exertion


. LQT2 events
whereas events in LQTB often
occurring in the postpartum period or with
have an intermediate pattern, often
auditory triggers . LQTB has the highest probability for
sudden death, followed
by LQT 2 and then LQT1.
86.(B) No relationship exists between the infecting organism and the type of
congenital defect , duration of illness, or age of the child. Staphylococcal
endocarditis is more common in patients with no underlying heart disease
Viridans group streptococcal infection is more common after dental procedures,
group D enterococci are seen more often after lower bowel or genitourinary
manipulation.
87.( A) A single episode of acute rheumatic carditis often results in complete
healing of the valvular lesions, while repeated episodes, especially involving

.
p eviously affected valves, result in
wh ch is the rationale for secondary
chronic rheumatic heart disease ( RHD )
prophylaxis.
>

461
rhapterjg
Diseases of the Blood
Questions
UASANEIN H. GHALI
in the fetus are larger than in adults, and at 22 - 23 wk gestation
1. Erythrocytes
,t,e mean corpuscu
lar volume can be as high as
A . 105 femtoiiters ( fL )
B. 115 femtoiiters ( fL)
C. 125 femtoiiters ( fL)
D. 135 femtoiiters ( fL)
E. 145 femtoiiters ( fL)

2. How much is the normal ratio of HbA to HbA 2 throughout life


?
A . 15 : 1
B. 20 : 1
C. 2 5 : 1
D. 3 0 : 1
E. 3 5 : 1

follow ing cond itions is asso ciate d with increased Levels of


3 Which of the
,

HbA 2 >3.4%?
A. Megaloblastic anemia
B . Iron- deficiency
C . a- thalassemia
D . Hemolytic anemias
E . Diamond Blackfan anemia
?
films of whic h of the following conditions
4 . Microcytes are seen in blood
A . Aplastic anemia
B . Liver disease
C . Anemia of chronic disease
D. Down syndrome
E . Hypothyroidism

462
of which of the following
tippling is seen in blood films co

0. Aplastic anemia
C . Liver disease
0 . Thalassemia
E . Xerocytosis

6. A 4 -month-old infant presents


with progressive pallor,microcephaly ,
, S(llj
nose, high-arched palate, hvpert elorism, low -set ears, and triphal angeal thump
Peripheral blood smear shows macrocytic RBCs with low reticulocyte count.
Of the following, the MOST likely diagnosis is
A. Diamond-Blackfan anemia
B. Fanconi anemia
C. Shwachman -Diamond syndrome
D . Pearson marrow - pancreas syndrome
E. Aase syndrome

7 . Which of the
following is the mainstay therapy of Diamond - Blackfan anemia 7
A. Chronic red cell transfusions
B. Corticosteroids
C. Hematopoietic stem cell transpla
ntation
D. Hydroxyurea
E. spenectomy

,
^
f
° he following
childhood from iron - deficiency differentiate transient erythroblastopenia of
A. MCV
anemia ?
B . Reticulocytopenia
C. Neutrophil count
D. Platelet numbe
r
F- Age of onset

Which of the follow ,ng ,


%
of chrome disease (
is he BEST
•agnostic test
ACD ) from iron - used to distinguish anen^ 5
A. Serum iron defiCl
cencY anemi a ( IDA ) ?
B. Serum transferrin
C. Serum ferritin
D. Soluble transferring receptor

463
E. t/icv

A 7-year
-old child presents with pallor, jaundice, frontal bossin
, , supernumerary toes, and gall stones revealed by US *
patosplenomegaly
iratorY
ani50P
" '
0 d
.. .
findings revealed hemoglobin 8 g/dl MCV no a, nomo

^
|ocVtos s and basoph I c stippling of RBCs. The bone marrow aspirate
erythroid hyperplasia, megaloblastosis, with incompletely divided cells
shoWS bridges between nuclei of pairs of erythrocytes. Electron
with thin chromatin
erythroblasts with a characteristic "Swiss cheese"
microscopy revealed
pattern.
heterochromatin
following, the MOST likely diagnosis is
Of the
A . congenital
dyserythropoietic anemia
B Diamond- Blackfan
anemia
C. fanconi anemia
D . hereditary spherocytosis
E. hereditary elliptocytosis

brought by his mother because of pallor, he has been


baby is
11. A 10- week- old weight of 3.6 Kg, now he looks well
but his
normally at 38 weeks with
delivered
Hb is 10 mg/dl. management is
the MOST appropriate
Of the following,
A. iron therapy
B. folic acid therapy
C. vitamin E therapy
D. reassurance
therapy
E. erythropoietin
is folate deficient ?
12. Which of the following milk
A. Human breast milk
B. Infant formula
C. Pasteurized cow' s milk
D. Goat' s milk
E. Camel milk acid
can impair folic
drugs
13. Which of the following anticonvulsant
absorption?
A. Phenobarbital
B. Oxcarbazepine
464
C Ethosuximrde
D. Lamotrigine
E . Levetiracetam

14. A 7- month-old boy presents with progressive pallor, j tabi ,


diarrhea, and poor weight gain. He exclusively breast fed till the ltY, CK
months, then shifted to boiled goat milk because of mother j||nes5
MCV 105 fl, reticulocyte count 1%, WBCs and platelets normal.
ao
b
of t
7 *
^
Of the following, the MOST important investigation is
^
A. serum B 12
B . serum folic acid
C. serum ferritin
D . tissue trasglutaminase IgA Ab
E . serum zinc

15 . A 7- month- old boy presents with progressive pallor, irritability, chron -


diarrhea, and poor weight gain. He was exclusively breast fed till the age of two
months, then shifted to boiled goat milk because of mother illness. Hb 7 gm/ci
MCV 105 fl, reticulocyte count 1%, WBCs and platelets were normal.
Of the following, the MOST appropriate treatment for this boy is
A. folic acid therapy 1.0 mg/day for lwk
B. folic acid therapy 5.0 mg/day for 2 wk
C . folic acid therapy 1.0 mg/day for 4 wk
D . folic acid therapy 1.0 mg/day for 8 wk
E. multivitamin dropper ( containing 0.2 mg of folate )

16 . A 1- year - old boy presents with progressive pallor, weakness, lethargy


feeding difficulties, failure to thrive, and irritability. On examination he has a
tinge of jaundice, glossitis, hypotonia, and developmental delay. His periphcr
blood shows macro - ovalocytosis of the RBCs with neutropenia an^ °
thrombocytopenia . His mother has peptic ulcer disease and on proton Pu
inhibitors .
Of the following, the MOST likely cause is
A . Bii deficiency
B . folate deficiency
C. hereditary ovalocytosis
0 . iron deficiency
£ . Imerslund- Grasbeck syndrome
46 S
17 - nia, developmental
mo gs revealedJ megaloblastic delay, and
. The parents are l degree

proteinuria relatives and the child h anemia and
sibling5 * as two normal
Of the following, the MOST likely diagnosis is
A. B12 deficiency
B. folate deficiency
C. Imerslund -Grasbeck syndrome
D. Rogers syndrome
E. Oroticaciduria

18. A 3 -year -old child presents with megaloblastic anemia, sensorineural


deafness, and diabetes mellitus. He has visual problems, congenital heart
,d
disease, and his height is below 3 percentile. The bone marrow revealed
megaloblastic changes with ringed sideroblasts .
Of the following, the MOST likely diagnosis is
A. B 12 deficiency
B. Transcobalamin deficiency
C . Imerslund -Grasbeck syndrome
D. Rogers syndrome
E. Oroticaciduria

to maintain positive
19. How much is the necessary daily dietary intake of iron
iron balance in childhood ?
A. 10 mg
B. 20 mg
C. 30 mg
D. 40 mg
E. 50 mg
murmurs are often heard
,

, lethargy, and systolic flow


20. Irritability, anorexia
when the hemoglobin level falls below
.
A 8 g/dl.
B. 7 g/dL
C. 6 g/dL
D. 5 g/dL
E. 4 g/dL

466
I
I

provides very useful and early


21. Which
deficiency?
of the following
^ 0rs
ot
%

A. Serum ferritin
B . Serum iron
C. Serum transferrin
D. Free erythrocyte
protoporphyrins
hemoglobin concentration
E. Reticulocyte

22. Iron deficiency is best


prevented to avoid both its systemic m
be encouraged, with the
and anemia. Breastfeeding should
H

supplemental iron at the age of


^
A. 2 months
B. 4 months
C. 6 months
D. 8 months
E. 10 months

23. Iron therapy in developing countries may increase the virulence of certain
A . viruses
B. gram negative bacteria
C. gram positive bacteria
D. fungi
E. atypical bacteria

24. Blood film of a 2 - year - old child shows poikilocytosis, microcyte


fragmented erythrocytes, and elliptocytes.
Of the following, the MOST likely
diagnosis is
A . sickle cell disease
B. liver disease
C. hereditary spherocytosis
D. hereditary elliptocytosis
E. hereditary
pyropoikilocytosis
2S, Which of the
following RBC indices is MOST helpful in the diagn 5is
hereditary spherocytosis? °
A . RBC count
B . MCV
C . MCH

467
0. MCHC
E. RDW

Which of thefollowing closely mimics


25 hereditary spherocytosis in
period ? the
neonatal incompatibility
ABO
^
B Autoimmune hemolytic anemia
C. Clostridial sepsis
D. Wilson disease
E . Congenital dyserythropoietic anemia type II

27. When splenectomy is indicated for hereditary spherocytosis, it should be


performed after age of
A. 3 years
B. 4 years
C. 5 years
D. 6 years
E. 7 years

28. What is the MOST important lab finding that differentiate hereditary
pyropoikilocytosis from hereditary elliptocytosis ?
A . Extreme microcytosis
B. Extreme anisocytosis
C. Extreme macrocytosis
0. Extreme elliptocytosis
E. Extreme ovalocytosis

29. A 2- year- old child presents with mild compensated macrocytic hemolytic
anemia with splenomegaly 2 cm below costal margin with intermittent jaundice.
CBC shows elevated MCHC, MCV, decreased erythrocyte osmotic fragility, and
decreased K + concentration, blood film reveals small numbers of stomatocytes,
target cells, and contracted RBCs .
Of the following, the MOST likely diagnosis is
A. Hereditary hydrocytosis
B. Hereditary xerocytosis
C. Hereditary cryohydrocytosis
D. Tangier disease
E. Sitosterolemia
468
PreSC with large orange tonsils, hepatospie ,
30. A 10-yea
old
r- git
'° op,c and peripheral neuropathy

lymphadenopathy' Cl UdV moderate stomatocytic hemolytic Her


.
ion

blood smear rreveals afiemia


thrombocytopenia. likely diagnosis is
MOST
Of the following, the
A. Phytosterolemia
Syndrome
B. Rh Deficiency
C. Cryohydrocytosis
D. Tangier disease
E . Sitosterolemia

ia with bone marrow infiltration is seen in


31. Pancytopenia
A. B12 deficiency
B. CMV infection
C. systematic lupus erythematosus
D. osteopetrosis
E. aplastic anemia

32 . What is the MOST common inherited pancytopenia syndrome ?


A. Dyskeratosis congenita
B. Fanconi anemia
C. Shwachman-Diamond syndrome
D . Diamond- Blackfan anemia
E. Congenital amegakaryocytic
thrombocytopenia
33. Which of the following
pancytopenia is transmitted through autosomal
dominant manner ?
A. Congenital amegakaryocytic
B. Amegakaryocytic thrombocytopenia
thrombocytopenia with radioulnar synostosis
C. Nijmegen breakage
syndrome
D. Seckel syndrome
E . Dubowiz syndrome

34 . Which of the
following pancytopenia
inheritance ? is transmitted through
mitochond
A . Diamond- Blackfan
anemia
B . Amegakaryocytic
thrombocytopenia
C . Pearson marrow - with radioulnar synostosis
Pancreas syndrome

469
Schimke syndrome
° Et
?
Cartilaeehair hVPOplasia

MOST common congenital anomaly in


Wha * is the Fanconi anemia ?
A Cardiac
B Gastrointestinal
c. £ ve
p. Renal
6 Skeletal
latest hematological abnormality evolved in fanconi
36 What is the
,
anemia ?
A . Anemia
B . Thrombocytopenia
C. Neutropenia
D. Macrocytosis
E . Increased hemoglobin F

37 . What is the MOST common available test for diagnosis of Fanconi anemia ?
A . Complete blood count
B. Bone marrow aspiration and biopsy
C. Chromosomal breakage test
D . Mutation screening
E. Next- generation sequencing

hematological
38. Which of the following options provides curative outcome for
abnormalities in patients with Fanconi anemia ?
A . Androgen therapy
B . Hematopoietic stem cell transplantation
C. Granulocyte colony stimulating factor
D. Erythropoietin
E . Blood / blood products transfusions

considered as a complication of
39 . Which of the following malignancies is
Fanconi anemia ?
A. Acute lymphoblastic leukemia
B. Acute myeloid leukemia
C. Osteosarcoma
D. Rhabdomyosarcoma
470
K

E. Retinoblastoma
androgen therapy in Fanconi anemia is her
40. Initial response of aided b y
A rise in hemoglobin
increase in neutrophil
count
B
c. increase in lymphocyte count
D. reticulocytosis
E. increase in platelet counts

41. What is the MOST commonly used


androgenic drug in Fanconi
anemia >
A. Testosterone
B. Oxymetholone
C. Methenolone acetate
D. Nandrolone decanoate
E. Boldenone undecenoate

42. Which of the following is TRUE regarding Pros and cons of androgen therap ,
in Fanconi ?
A . They can produce a response in about 30% of patients
B. Treatment is curative in some patients
C. Dose modification is always required according to hematologic
response
D. Once response is achieved, it usually
continues for many years
E. Liver tumors are reported as possible
consequences of androgen use
43. Granulocyte colony - stimulating
factor ( G -CSF) in Fanconi anemia can usually
induce an increase iin the neutrophil
count ; however, a notable limitation tc
starting therapy is
A . severe bone pain
B. local bleeding at site of
C. risk of expansion of
subcutaneous ini
injection
bone marrow cells with clonal cytogen<
abnormality ^ '

D. the rise of counts


involves only white blood
E. possible loss rfJSponse cells
° after
^
w doses due to disease progression

471
.ear-old male develops recurrent epistaxis and easy bruising The
44 . ^ the first child of unrelated healthy parents born after 41
nt vvas
'
weeks of
p3
lancv
‘ mesophalangia on bilateral 5th fingers
Physica ! examination of the patient
reveal short stature, clinodactyly
, multiple cafe -au lait
brachv - spots
W
thighs and right buttock. He has no other abnormalities His initial
'

n both
° 0|ete blood
cell count results are as follows: white blood cell, 5.0 10VL
*
9.5 g / dL; platelets 80 x 109/ L.
hemoglobin
, the MOST likely diagnosis is
f the following
anemia
A megaloblastic
B. Pearson syndrome
C. congenital Amegakaryocytic Thrombocytopenia
D. Fanconi anemia
E. Diamond- Blackfan anemia

45. What is the MOST common hematological abnormality in Shwachman -


Diamond syndrome ?
A . Anemia
B. Neutropenia
C . Lymphopenia
D. Thrombocytopenia
E. Pancytopenia

46. What is the MOST common non -hematological abnormality in Shwachman -


Diamond syndrome ?
A . Pancreatic insufficiency
B. Elevated transaminase
C. Skeletal abnormality
D. Short stature
E. Rib cage abnormality

47. A 24 - month - old girl was admitted to the hospital because of steatorrhea
and short stature. At the age of three months, she presented with convulsion
and neutropenia . Since then she has had frequent purulent otitis media and
pneumonias She has angular stomatitis, and many dental caries. The results of
.
the laboratory studies were as follows: hemoglobin 11.3 g/dL, leukocytes
3,000/ pL ( neutrophils 17.3%, lymphocytes 59.5%, monocytes 22.9%, basophils
0.3%, and eosinophils 0%), platelets 152,000/pL.
Of the following, the MOST likely diagnosis is

472
*B. 2
^

-
* *“
dtarnond syndrome
c Shwachman
antitrypsin deficency
D. alpha-one
E . Pearson syndrome

following feature are more specific of Fanconi


4 g Which of the anemi \
- diamond syndrome
? *
Shwachman
A. Metaphyseal dysplasia
secondary ossification centers
B. Delayed appearance of
C . Triphalangeal thumb
D . Short or flared ribs
E . Thoracic dystrophy

49 . What is the MOST common reported chromosomal abnormality r

Shwachman- diamond syndrome ?


A . Isochromosome 7q
B. Monosomy 7
C. Isochromosome 7q combined with monosomy 7
D . Deletions of 7q
E. Deletions of 20q

50. What is the old diagnostic triad for dyskeratosis congenita ?


A . Nail dysplasia, reticular pigmentation of the chest, and oral leukoplakia
B. Bone marrow failure, nail dysplasia, and oral leukoplakia
C. Reticular pigmentation of the chest, bone marrow failure, aid «
dysplasia
D. Avascular necrosis of hip, nail dysplasia
, and bone marrow failure
E. Optic atrophy, nail dysplasia
.and oral leukoplakia
51

^ ^
051 common clinical manifestation of dyskeratosis conge 1'13
LeuVopl '
8. Epiphora
C - Blepharitis
0. Skin pigmentation
Ei Phimosis
syndrome has many of the features of
52 - Revesz dyskeratosis congenital DC
in early childhood in addition to
and Presents
A blepharitis
B conjunctivitis
Q exudative
retinopathy
D , cataract
E optic atrophy

the following inherited bone marrow failure syndromes has the


53. Which of
lowe st survival and
highest mortality ?
A. Fanconi anemia
B. Shwachman- Diamond syndrome
C. Dyskeratosis congenita
D. Congenital amegakaryocytic thrombocytopenia
E. Diamond- Blackfan Anemia

54. Congenital amegakaryocytic thrombocytopenia patients usually have normal


phenotype . Yet, about 10% of cases involve physical anomalies.
Of the following, the MOST common anomalies are
A . eye anomalies
B. kidney malformations
C. skeletal malformations
D. cardiac anomalies
E . dysmorphic features

, and painful throat


55 , An 8 - year- old boy presents with fever, headache, chills
of his finger an
for five days. Leukoplakia is noted on his tongue and some
with pigmentation n°
nails are markedly dystrophic. His skin seem spotted
anterior chest and neck. Vitally, his blood pressure
106/ 74 m m g P.
temperature -
106/min, respiratory rate 28b/ min, and body and Hb 9.2 g/di.
s platelets 21.000
count was 0.300/mm with 37% neutrophil ,
3

Of the following, the MOST likely diagnosis is


A. congenital amegakaryocytic thrombocytopenia
B. dyskeratosis congenita
C. reticular dysgenesis
D. Fanconi anemia
E. cartilage - hair syndrome

M -=
js implicated as a possible cause 0f
_ f the following viruses
56. Which 0 anemia ?
prolonged aplastic
virus
A. Rubella
virus
B. Measles
C. cytomegalovirus
D. Herpesvirus
B19
E Parvovirus
terial agents is implicated as a possible c
57 . Which of the following antibac **
of acquired aplastic
anemia ?
A. Ampicillin
B. Trimethoprim- sulfamethoxazole
C. Cefotaxime
D. Gentamycin
E. Azithromycin

of severe aplastic anemia ?


58. Which of the following findings is a criterion
A . Absolute neutrophil count 600/ mm
3
B . Platelet count 18,000/mm
C Reticulocyte count 1.5 %
D. Bone marrow biopsy cellularity 33%
E . Hemoglobin level 6.0 gm /dl

59. Cases of Aplastic anemia should be assessed for paroxysmal nocturna '
hemoglobinuria. The MOST sensitive test is
A. direct antiglobulin test
B . hemoglobin F level
C. erythrocyte CD55 and CD59
D . haptoglobin
E. urine for hemosiderin

60. What is the MOST common


. chromosomal abnormality found within the
marrow of patients w th myelodysplastic
A . Monosomy 7
syndrome ?

B. Monosomy 8
C. Trisomy 11
D. Trisomy 13
E . Trisomy 21
h of the following lab findings suggest polycythemia due to heart
61' Whic or
lung’ mass with low plasma volume
Normal red cell
Elevated red cell mass with high carboxyhemoglobin
g|evated red cell mass with low arterial oxygen

D Normal red cell mass with abnormal hemoglobin study


E Elevated red cell mass with low erythropoietin
following is a major WHO criterion for diagnosis of
62. Which of the
Polycythemia Vera ?
A Red cell mass 20% above mean normal predicted value
B . Bone marrow trilineage myeloproliferation
C. Subnormal serum erythropoietin level
D. Endogenous erythroid colony growth
E . Presence of JAK 2 or similar mutation

63 . What is the usual physical finding in polycythemia ?


A. Hepatosplenomegaly
B. Skin ecchymosis
C. Pruritus marks
0. Hypertension
E. lymphadenopathy

64 . What is the MOST convenient initial treatment of polycythemia ?


A. Phlebotomy
B. Aspirin
C . Hydroxyurea
D. Interferon -gamma
E. Iron

65. Among infants less than 4 months of age, which of the following is
'ndication of blood transfusion ?
A . Hemoglobin less than 14.0 gm/ dl in patient with severe pulmonary
disease
B. Hemoglobin less than 10.0 g/ dl in patient with moderate pulmonary
disease
C . Hemoglobin less than 14.0 g/ dl in patients with severe cardiac disease
D Hemoglobin less than 11.0 g/ dl preoperatively and during major surgery

476
in less than 10.0 g/dl postoperatively
E. Hemoglobin
hemoglobin values 0f
reason that the nadir


66. What is the
u
V
infants?
preH,
of term
infants are lower among premature infants
"a«» »infants
*
c

STJtam
Dimi nished * *
plasma
. .
in p nmmu e

erythropoietin
is greater than term
in response to anemia
0. among premature infants
rep tacement is slow
E. Fetal hemoglobin

of pediatric platelet transfusion, patients With


67 According to the guidelines
factors (infection, organ failure,
marrow failure and hemorrhagic risk a
^
count maintained above
clotting abnormalities ) should have their platelets
3
A. 80,000/mm
3
B. 60,000/mm
3
C. 40,000/mm
3
D. 20,000/mm
E.
3
10,000/ mm

68. A 1-month - old infant with sepsis on aggressive antibiotic therapy, while he is
clinically unstable requires a platelet count to be maintained above
A. 100,000/mm 3
B. 70,000/mm 3
C. 50,000/ mm 3
D. 30,000/ mm 3
E. 10,000/ mm 3

SicsTn:x?rhet forders '


, ,
p ate et transfusions are

B . no
c. life threatening bleeding
is selHrsr^
0
E. repeated
transfusionS lead t 0 SorT
refractoriness
" ers

70. Granulocyte
transfusion for a 3 day
threshold of neutrophil count old -
-

newborn is indicated with a


below ?
A. 1,000/mm 3

477
mm *
B. l OO/
c 2, ^
000/ mm
3
3
p 2, 500/mm 3
E. 3,000/ mm

n. Which of the following factors reduces the level of von willebrand factor ?
A . Stress
B, infections
Q Hypothyroidism
D, Exercise
E. Pregnancy

72 . Which of the following types of von Willebrand disease is misdiagnosed as


hemophilia due to low factor VIII level ?
A, Type 1C
B, Type 2 A
C , Type 2B
0 , Type 2M
E. Type 2N

73 . Which type of von Willebrand disease presents with thrombocytopenia ?


A. Type 1C
B. Type 2 A
C. Type 2 B
D. Type 2M
E- Type 2N

?4 . Which of the following subtypes of von Willebrand disease is likely to benefit


from Desmopressin ?
A. Type 1
B. Type 1C
C. Type 2 A
D. Type 2 B
E. Type 3
thrombocytopenia through
75 Which of the following conditions causes
impaired platelets production ?
A . Kasabach Merritt syndrome
-

478
B.

,Osteopetrosis
tr
£
S
-
Hyperspten'srn
tbrombocvtopenla

T edacZ
SN
skin
boywith
lesions .
normal vitals and no abnormality in generalexam
His heart exam is normal, chest note is norma ,
from these .
exam falls to Identify organ enlargement
abdominal .
likely diagnosis is
Of the following, the MOST
A. aplasticanemia
6. immune thrombocytopenia
C acute leukemia
D. Glanzmann thrombasthenia
E. von Wiflebrand disease

77. Which of the following Jab tests results is worrrsome in immune


thrombocytopenia and should raise the suspicion of another diagnosis ?
3
A. A platelet count of 1.000 /mm
3
B. A WBCs count of 2.500 / mm
C A hemoglobin count of 8.0 gm / dl
D. A bone marrow examination showing increase number of
megakaryocytes
E. A negative Coombs test

78. Autoimmune screen, in the context of immune thrombocytopenia, 15


required in which of the following clinical
conditions ?
A . Five year old girl with long history of
skin and mucosal bleeding lasting
- -
more than one year
B. Acute history of severe profound
epistaxis, heralding possible need f <
blood transfusion °
C. Acute presentation of
intracrania / hemorrhage in a background of tw °
weeks skin bleeding that wasn't


without a prior problem

-
D. Periorbital bleeding suggesting
basal skull fracture after road traffic
~ bocytopenia with

4 79
r

girl presented with


E. An adolescent
background of few weeks
counseling for social reasons
skinSeVere menorrhagia and
ecchymosis with pallor
reluctance for
, with a
medical

-old girl presents with recurrent skin


79. A 6-year ecchu
abdomen and lower
over the negligible apart limbs for the last few ? rash
history is from frequent neonatal exrhl? ^ medicalor ,
severe Rh incompatibility. Her regional exam shows a dear rh& normal
heart sounds, and soft palpable spleen of 5 cm BCM with no h
no palpable masses . His CBC film shows isolated t h r o m b o
indices and normal film .
pen a normal
^ '
"I
other
Of the following, the MOST likely diagnosis is
A . acute lymphoblastic leukemia
B. portal vein thrombosis
C. immune thrombocytopenia
D . chronic liver disease
E . aplastic anemia

80. Autoimmune thrombocytopenia may be the initial manifestation of which of


the following malignancies ?
A. Neuroblastoma
B. Lymphoma
C. Wilms' tumor
0 . Rhabdomyosarcoma
E . Ewing' s sarcoma


{ ecchymosis and petechial
81. A 3- year- old boy presents with skin bleeding of hlppdine and

:rs:
thrombocytopenia ( 33.000/mm ). ^ * ““
Of the following, the MOST appropriate initial
treatment is
A. observation
B. steroid
C. IVIG
D. rituximab
E. thrombopoietin receptor agonist

480
82. What is the BEST treatment
.
option or a newly
thrombocytopenia in a 6-year - old naughty ch ld, diagnosed as ADHD
diaenoSect
.
in JS
childhood?
A. Observation
B. Splenectomy
C. IVIG
D. Rituximab
E . Thrombopoietin receptor agonist

83. What is the BEST curative option for a chronic immune thrombocytopenia
a 10-year- old male child, with recurrent epistaxis and gum bleeding that impair-
his daily active life style?
A. Observation
B. Splenectomy
C. IVIG
D. Rituximab
E. Thrombopoietin receptor agonist

84 . Which of the following is a TRUE fact regarding therapy of idiopathic


immune thrombocytopenia?
A . The majority of patient have moderate to severe symptoms of bleeding
B . Early therapy does not prevent intracranial hemorrhage
C. Initiating early therapy affect the outcome of the disease
D. Antiplatelets antibodies bind to autologous rather than transfused
platelets
E . Aggressive way of management is preferable so as to control bleeding

85. Splenectomy in Immune thrombocytopenia (


ITP) is considered in which of
the following conditions?
A . A two - year old boy with newly diagnosed
ITP whose bleeding symptom
failed to be ameliorated with
steroids and IVIG
B. A ten - year old girl with chronic
ITP with recurrent symptoms of skin
bleeding that is diffuse, not
responding to steroids, rituximab aod
medical therapy.
C . A six - year old boy with acute ITP
developed intracranial hemorrhage
from a road traffic accident who is
hemodynamically stable and
bleeding is controlled with IVIG and
steroids

4Rl
p A thirteen-year old girl with chronic ITP
and
necessitated blood transfusion and faited
7eTe
Vaeina b eedi ,
rituximab and contraceptive therapy
A five-year old boy with acute ITP on
re '
° 5pond »0 steroid;"«* that
to
IVIG,
E,
initial therapy with
presented with severe epistaxis and
consequent anemia steroid and
need for blood transfusion declared the

86. Passing into the chronic phase of the disease a


*
thrombocytopenia (ITP) should be carefully evaluated'to
A. aplastic anemia
exclude
ae ‘
immune

8. acute leukemia
C. HIV infection
D. non-hodgkin lymphoma
E. medical child abuse

87. Thrombotic thrombocytopenic purpura is a rare thrombotic


microangiopathy characterized by the pentad of
A. Fever, microangiopathic hemolytic anemia, thrombocytopenia .
abnormal renal function, and central nervous system changes
B. Pneumonia, microangiopathic hemolytic anemia, thrombocytosis ,
abnormal renal function, and central nervous system changes
C. Fever, autoimmune hemolytic anemia, thrombocytopenia, abnormal
liver function, and central nervous system changes
renal
D. Fever, autoimmune hemolytic anemia, thrombocytosis, abnormal
function, and central nervous system changes
thrombocytosis, abnormal
E. Pneumonia, autoimmune hemolytic anemia,
liver function, and central nervous system changes

88. A 14-year -old boy presents with 3 weeks - ,


vomiting and irritability, followed
progressed to pallor and diffuse
by wea
petechial ras o
^
^ '
^ imbs then
trunk and lower
infection j h injectable

^
limbs. He was treated initially as nonspecific
antibiotics, but seems fruitless. Regrettably
,
* genera| examination
episodes of abnormal movement and disturbe essjve behavior , having
boy w
showed a semiconscious blind adolescent | mp nodes
enlargement.
pallor, diffuse skin bleeding, without any .
edema or ^ and abdomen
exam ElVt, chest
His meningeal signs were ^ limb weakness of
were normal. His neurological exam
482
reflexes. His CBC showed abr)0r 31
grade 3 with diminished
schistocytes, spherocytes, helmet cells, and an elevated retie ?
* R8Cs.
"" ^
association with thrombocytopenia . Blood urea nitrogen and
1
*
slightly elevated .
likely diagnosis is
Of the followings, the MOST
A . hemolytic uremic syndrome
B. thrombotic thrombocytopenic
purpura
C. acute leukemia
D. aplastic anemia
E . meningococcemia

89 . What is the MOST appropriate effective initial management of acquire


thrombotic thrombocytopenic purpura ?
A . Steroids
B . Splenectomy
C. Rituximab
D . IVIG
E . Plasmapheresis

90. A 1- month- old female presented with diffuse skin ecchymosis and petechia ,

rash since early days of life, she was noticed to have occasional blood spits frorr
the mouth. No significant antenatal, neonatal, and family history. Initial CBC
shows thrombocytopenia with normal other indices. Other workup tests were
normal. BMA shows normal hematopoietic elements with frank absence of
megakaryocytes .
Of the following, the MOST likely diagnosis is
A. Fanconi anemia
B . congenital dyserythropoietic anemia
C. congenital cytomegalovirus infection
D . congenital leukemia
E . congenital amegakaryocytic
thrombocytopenia
91. Which of the following conditions has
been linked with Thrombocytopenld
absent radius ( TAR } ?
A . Cow' s milk intolerance
B . Diabetes Meilitus
C . Congestive heart failure
D. Chronic renal failure

483
E. Bronchiectasis
mother of a 3-month-old girl
9l A consulted the pediatric
clinic for a con
%£ anernTTheTniel' " cer n of
*

» be
^
Physiological
bleeding in her daug hter, furth er workup failed to rev
a
?
i
"' "
Pa r which
°’
sV Woms of
later
abnor,TlalitV in
girl, physical examination was completely normal nn fnv , the
mother who has sens orin eura l deafness since early life
bad mild to moderate thrombocytopenia during earlv rhna^
' nquir> of the
! that she
t0 adulthood with no significant bleeding Medical
, Lh i ’,
hat persisted
S
coun o« 25.000 - 50 TO .
neutrophil inclusion bodies, with normal other
Of the following, the MOST likely diagnosis is
A. thrombocytopenia -absent radius
8. MYH9-related thrombocytopenia

^
indices
“"
S accompanied
by

C. congenital aplastic anemia


D . Wiskott - Aldrich syndrome
E . congenital leukemia

93 . Which of the following is considered as diagnostic of splenic hypo


function?
A. Target cells
B. Decreased osmotic fragility
C. Heinz bodies
D. Poikilocytosis
E. Pitted erythrocytes

94. Banti syndrome (congestive splenomegaly ) is seen


A. acute myeloid leukemia
B. amyloidosis
C. juvenile idiopathic arthritis
D. alphal- antitrypsin deficiency
E. Niemann- Pick disease

95 . Which of the following abnormalities constitute to


aspjenja and
A . Congenital heart defects, renal anomalies,
levocardia
B. Congenital heart defects, dextrocar lateral trilobed lungs, asplenia
and heterotopic abdominal organs
484
c. cyanotic heart defect, skeletal anomalies, retinal detach
^, ,b
%
gastroschises
D'
and
congenital renal anomalies , Inzer cysts, bowel atresia, ompha( *
. genital abnormalities
.con
of the lower limbs
, multiple brain cysts, imperforate
E . Congenital renal anomalies
cysts
prune belly syndrome and lung

RBC changes in peripheral blood Sf ear


96. What are the characteristic
patients with splenic hypofunction ?
*
A . Howell-Jolly bodies
B. Basophilic stippling
C. Pappenheimer bodies
D. Cabot rings
E . Hemoglobin crystals

97. What is the MOST common pathogen implicated in the etiology of


lymphangitis ?
A . Group A streptococci
B . Escherichia coli
C. Pseudomonas aeruginosa
D. Clostridium difficile
£ . Proteus species

98. Regardless of the presence or absence of systemic symptoms or otbe


abnormal physical findings, which of the following properties should always
raise the question of malignancy in a lymph node ?
A . Axillary in site
B. Firm and fixed
C . Matted
D . Red overlying skin
E . Tender

99 . Which of the following features signifies


adenopathy caused by bacte^
rather than other infectious agents ?
A . Atypical anatomic areas
B A draining sinus
C , Lack of prior pyogenic infection
D , A short course

485
E. Unusual clues in the history
of the following is an unusual site of tuber
100. Which culous
A. Hilar
B. Supraclavicular
'ymphadenopathy ?

C. Abdominal
D. Cervical
E. Inguinal

101. which of the following viruses has been implicated as an association with
Castleman disease?
A. Measles virus
.
B Human herpes virus 8
C. Epstein-Barr virus
D. Parvovirus B19
E. Cytomegalovirus

102. TAFRO syndrome stands for

issrsrttsrzzZ'
Organomegaly
C. Thrombocytopenia, Alopecia, Facial
media
D. Thrombocytopenia, Anasarca,
Organomegaly
v
- — .
Rhinitis and otitis

ever, Reticulum fibrosis, and


Otalgia
-
E. Tetany, Allergy, Fluid retention, Fever, Reticulum fibrosis, and

103. POEMS syndrome stands for


onHocrinopathy , M-proteins, and skin
A. Polyneuropathy, organomegaly, eno
lesions . ns
. , and skin les^
Polyuria, organomegaly, endocardi ' Mvasthenia
.
B.
..
1S
. oathy, M -pmteins and Serosi
C. Parotid swelling, otitis externa , endoc
D. , Myasthenia , and Seros t s
Polyneuropathy, orchitis, endOCr
E. Polyuria , organomegaly, edema, "°
^
0teins and skin
lesions
Chapter 20
Diseases of the Blood
Answers
HASANEIN H. GHALI
are larger than in adults, and at 22 - 23
1.(D) Erythrocytes in the fetus
gestationthe mean corpuscular volume can be
as high as 135
femtoliters (ft^)
Similarly, the mean corpuscular hemoglobin is
very high at 22 - 23wk and faijs
relatively linearly with advancing gestation. In contrast, the mean corpuscular
hemoglobin concentration is constantthroughout gestation at 34 ±1g/dL.
2.(D) The minor adult hemoglobincomponent, HbA2 , contains delta ( 6) chains
and has the structure a2 62 . Atbirth, <1.0% of HbA 2 is detected, but by 12 mo
of age the normal level is 2.0-3.4%. Throughout life, the normal ratio of HbA to
HbA 2 is about 3 0 : 1.
3.{ A ) The normal adult level of HbA 2 ( 2.0-3.4%) is seldom altered. Levels of
HbA2>3.4% are found in most persons with the (3 -thalassemia trait and in those
with megaloblastic anemias secondary to vitamin B12 and folic acid deficiency.
Decreased HbA 2 levels are found in those with iron -deficiency anemia and a -
thalassemia.
.
4 (C) Microcytes are seen in Iron deficiency, thalassemias, Lead toxicity, and
anemia of chronic disease. Macrocytes are seen in newborns, vitamin B12 or
folate deficiency, Diamond- Blackfan anemia, Fanconi anemia, aplastic anemia ,
liver disease, Down syndrome, and hypothyroidism.
.
5 (D) Basophil stippling are seen in thalassemia, lead intoxication, and
myelodysplasia.
6.( A ) Diamond- Blackfan anemia (DBA ) is a rare, congenital bone marrow failure
syndrome that usually becomes symptomatic in early infancy . More than 90% ^
cases are recognized in the 1st yr of life. The disorder is characterized by
a
anemia, usually normochromic and macrocytic
; reticulocytopenia;
insufficient or absent red blood cell (RBC) precursors in an otherwise
°
affected individuals have
normallycellular bone marrow . Up to 50% of
additional, extrahematopoietic anomalies
.<
'

7 B) Corticosteroids are a mainstay of therapy, and approximately 80%

.
patients mitiaNy respond. Because corticosteroids impair linear growth as el1
asphysical and neurocogmt ve development, many hematologists mainWn *
transfusion therapy and
jnfaotson chronic delay the start of
steroids until after
' ce 1 Vf -
* ,A The
)
temporary suppression of erythropoiesis

and moderate to severe normocytic anemia.


in up to 20% of cases. Platelet numbers
occursSoluble transferring receptor ( sTfR ) is a
9 (0) diagnostic test used
iron-deficiency anemia ( IDA ); sTfR levels are to distinguish
« CDfrom high in IDA and
normal in
ACD.
dyserythropoietic anemia type I
10.(A) Congenital CDA I may be diagnosed at
any age , most cases are recognized during childhood or
adolescence CDA i is
rareiy diagnosed in utero. In addition to anemia related symptoms
, other
findings often include splenomegaly, jaundice, and hepatomegaly.
Hemoglobin
concentrations generally range between 7 and 11 g/dl. The anemia is usually
macrocytic ( mean corpuscular volume: 100- 120 fL ), but normocytic
indices may
be seen during childhood . Anisopoikilocytosis is appreciated on the peripheral
blood smear . In some cases, normoblasts and basophilic stippling of RBCs may
be seen. The bone marrow aspirate shows binucleated and, more rarely,
multinucleated polychromatophilic erythroblasts are also appreciated.
Electron microscopy is the gold standard for diagnosis, revealing erythroblasts
with a characteristic "Swiss cheese" heterochromatin pattern.
ll.(D) In the full-term infant, physiologic anemia requires no therapy beyond
ensuring that the infant' s diet contains essential nutrients for normal
hematopoiesis.
12.(D) Human breast milk, infant formulas, and pasteurized cow ' s milk provide
adequate amounts of folic acid , Goat ' s milk is folate deficient, and
supplementation must be given when it is the child's main food.
13.( A) Certain anticonvulsant drugs ( e .g., phenytoin, primidone, phenobarbital )
can impair folic acid absorption, and many patients treatedwith these drugs
have low serum levels. Frank megaloblastic anemia is rare and readily respon s
t0 folic acid offending drug is
therapy , even when administrationof the
continued .
l4 ( B) Normal deficiency, levels are
serum folic acid levels are 5 - 20 ng/ mL; with
<3 ng/ mL
«.|q Folic acid therapy (0.S- 1.0 mg/ day ) should be continued for 3-4l wk until a
definite hematologic response has occurred. Maintenance therapy with
.
Multivitamin (containing 0.2 mgof folate ) is adequate.
n nutritional, resulting from
, 'lA ) Vitamin B ] 2
l0w Cbl
deficiency in infants is most
°
levels in the breast milk of Bi;-deficient ^
mothers. Associated
appears during the 1st yr of |jfe .
a often

infec tion, celiac disease, Crohn "


srf « , .
P' 10
'"
previo
i
ry dise a
us gastric bypass surge , treatment With * > ^
ic insufficiency
pancreatic
pump inhibitors

JK
die

! SE HX S*
, or inade quate

In selec
intak

tive
e

.
vitam
deficiency. It
from a strict unsupplemented Ve

svBdfoP* Is a rare’ recessivelon inhe


in B„ malabsorpti ,n fe
usuall y becom es clinica
,
lly
8« ,

rited r
appar ent
_
to.
*

Wltbr
consequent vitamin Bu
° 1s t 6 vr of life, in addition to megaloblastic pmentaldelay,
anemia , the patient
a ,
(e.g., hypotonia, develo brain atr%
have neuro logic defec ts
disor ders , deme ntia ) and/or proteinuria.
movement anemia ( Rogers syndrome) is a ver,
respo nsive mega loblas tic
18.(D) Thiam ine -
by megaloblastic anemia
reces sive disor der chara cteriz ed
rare autosomal
deafn ess , and diabe tes mellitus. Congenital heart defects
senso rineu ral ge myelodysplasia, an!
, trilinea
arrhythmias, visual problems, short stature
sive megaloblastic anemia usual,y
strokes are also described. Thiamine respon
-

presents in infancy but may occasionally


developin childhood and adolescent
. The bone marrow s
and occurs in several ethnically distinctpopulations
characterized not only by megaloblastic changes but also by
ringed sideroblasts
Continuous thiamine supplementation usually reverses the
anemia and
diabetes, but not existinghearing defects.
19.( A) It is therefore necessary to absorb approximately 1 mg daily to mainta
in
usually
positive iron balance in childhood. Because <10% of dietary iron is
irGr
absorbed, a dietary intake of 8- 10 mg of iron daily is necessary to maintain
levels.
20 ( D ) When the hemoglobin level falls to <5 g/dL, irritability, anorexia,
r
lethargy develop, and systolic flow murmurs are often heard. If the hemoglob '
continue 5 to fall, tachycardia and high output
cardiac failure can occur.
21. E) Detection of increased soluble
|
transferrin receptor and decrease *
reticulocyte hemoglobin concentration provides
very useful and early indicator *
of iron deficiency, but availability of these
tests is more limited.

23.{ B ) Iron therapy may increase the


virulence of malaria and certain
negative bacteria , particularly jn se 15
developing countries. Iron over d
associated with Yersinia infection . °
24.( E ) Sickle cells: sickle cell disease
Target cells: hemoglobinopathies ( HbC, HbS,
thalassemia ), liver disease
489
-
poikifoc
^*
5SS Sm
-Bite.ce |os: Gs 6PrnlCrO
Cigar"

' '
yropoikilocytosis
|S0 deficiency
CVtOS
5
'
*"

'
,
l s/ K
"^
fragmented
erythrocytes elliptocytes
emia

emolytic

:hereditary
25.(0) ^e mean
corpuscular volume (MCV)
of HS erythrocytes
decre ,
ased and the mean corpuscular > low normal
5
even slightly hemoglobin or
( MCHC) is usually increased (>35 g/ dL). An MCHC >35 4 g • concentration
/ dL
red cell distribution width (RDW) <14% has been suggested combined with a
as a screening test
for HS.
.
26 (A) Isoimmune hemolytic disease of the newborn,
particularly when a result
of ABO incompatibility, closely mimic s
.
. . * x
antibody on an infant
_ . the appearance of HS. The
s RBCs using a direct antiglobulin
nri detectionof
(Coombs)test should
establish the diagnosis of immune hemolysis.
.
27 (D) Most experts recommend splenectomy for
patients with severe HS and
believe it should be strongly considered for patients
with moderate HS and
frequent hypoplastic or aplastic crises, poor
growth, or cardiomegaly.
It is generally not recommended for
patients with mild HS. When splenectomy
is indicated, it should be performed after
age 6 yr.
28.( A ) HPP is characterized by extrem
e microcytosis (mean corpuscular volume,
50-65 fL/cell), extraordinary
variation in cell size and shape, and
microspherocytosis with occasional elliptocytosis.
29.( B) Hereditary xerocytosis ( )
HX , the most common type of the hereditary
stomatocytosis syndromes, is a dominant disorder of erythrocyte dehydration.
Affected patients exhibit a mild compensated macrocytic hemolyticanemia with
variable degrees of splenomegaly and intermittent jaundice. The MCHC and
MCV are elevated, erythro
cyte osmotic fragility isdecreased, and K +
concentration and total monovalent cation contentare decreased. There are
small numbers
of stomatocytes, target Lcells _ ,and contracted RBCs with
emoglobin puddled
to the side on peripheralblood smear.
-<
30 D) Tangier
rare recessive
disease, familial deficiency of high-density lipoproteins (HDls) is a
disorder that results from mutationsin the cholesterol and
Phospholipid transport protein ABCA1, leading to perturbations of cellular
cholesterol transport, and resulting in the accumulation of cholesterol esters in
??
>t
anY t ssues. Hematologic
' manifestations include a mild to mo erate
° matocytic hemolytic .
anemia and thrombocytopenia Affected patients can
490
also have large orange
tonsils, hepatosplenomegaly, lymphaden0 ,
Dat
, and premature atherosclerosis V' %
corneas, peripheralneuropathy .
B1 (D) Pancytopenia with
bone marrow infiltration can be Jccn |
n *
^
, myelofibrosis , hemophagocytic lymphohisticx
solid tumors
osteopetrosis. * ^
32 (B) Inherited pancytopenias account for approximately 30 % of
pediatric bone marrow failure. Fanconi
^
anemia is considered the 01
mostco
of the inherited pancytopenias. ^
33.( B) Amegakaryocytic
thrombocytopenia with radioulnar syno
characterized by thrombocytopenia with progression to pancytopenia it ^
be associated with acute myeloid leukemia. The gene is HOXA11.
.
34 (C) Pearson marrow -pancreas syndrome is characterized by neutropem
with progression to pancytopenia. The gene is mtDNA deletion. *
,

^
.
35 (E) The most common congenital anomalies in FA are skeletal and include
absence of radii and/or thumbs that are hypoplastic, supernumerary, bifid, or
absent. Anomalies of the feet, congenital hip dislocation, and leg abnormalities
can also be seen.
.
36 (A) Thrombocytopenia, red blood cell (RBC) macrocytosis, and increasec
hemoglobin F, as a result of bone marrow stress, often appear first
Subsequently, patients develop neutropenia and then anemia.
.
37 (C) Complete blood count shows pancytopenia; bone marrow aspiration and
biopsy confirms the hypoplasia of the marrow; chromosomal breakage test is
the most commonly available and detects the increased fragility with and
without addition of the cross -linking agents but still 10-15% have somatic
mosaicism and may not show the high degree of chromosomal fragility; Next -
generation sequencing has largely replaced the need for 2 -step genetic testing
(complementation group testing followed by targeted gene testing) and is most
often used.
38,(B)
.
39 (B) AML, MDS, and rare head, neck and skin tumors.
40.(D) Reticulocytosis and a rise in hemoglobin within cell
1-2 mo. White blood
( WBC) counts may increase next, followed by platelet
counts
41 (B) Oral oxymetholone and danatol are the
androgenic drugs . 2 most commonly ^
42 (E) A response is expected in 70% of patients,
than curative while awaiting for a suitable donor for
treatment is a bridge rathe '
HSCT or while weight
androgen dose can be increased
.
the risks and benefits of transplant If a low dose is
every 3-4 wk as long
initially employed,
ts
as no major side eff
491
*
^ ployed, androgen dosage can be slowly reduced to h
* Kns the required blood counts. Side

afljressiveness
,
mascolinization increased linear
, elevated hepatic enzymes
effect, «
growth
increased
, cholestasi, Mr
* ^ dose
is initial y

m d swings
°° - or-

that
andr0gens incl e
^
tumors. Screening for these should be performed regularly ’‘S**’ tVpica" v
3nd Ver

Stop responding to androgens after several months


marrow failure progresses or as
or ZJ ^ thel r "
'

.
they develop MDS or AML
«10 A heightened risk of expansion of bone
'
,
srrssrc,"
44,(D)
7 -a
"
marrow cells with

not
,

.
45 (B) Neutropenia 90%, anemia 46%, thrombocytop
enia 42%, and
pancytopenia 21%.
46,(A) Exocrine pancreatic insufficiency 98%, elevated
transaminase 61%,
skeletal abnormality 70%, rib cage abnormality 35% and short
stature 66%
47.(C)
48.( C) Although skeletal abnormalities are variable in Shwachman-
diamond
syndrome, classic findings are metaphyseal dysplasia, osteopenia,
delayed
appearance of secondary ossification centers, short or flared
ribs, and thoracic
dystrophy.
49.(A) Isochromosome 7 q is particularly common, suggesting that it is a fairly
specific clonal marker of SDS and might be related to the presence of mutant
SBDS on 7qll. Other clonal chromosome abnormalities include monosomy 7,
i( 7q) combined with monosomy 7, deletions or translocations involving part of
7q, and deletions of 20q [Del(20qH.
50.(A) A diagnostic triad of mucocutaneous features was proposed when the
disorder was first described and included dysptastic nails, lacy reticular
pigmentation of the upper chest and/ or neck, and oral leukoplakia. However,
the triad is not present in all individuals.
51.(D) Skin pigmentation is the commonest ( 89%) followed by nail dystrophy
( 88% ) ,
52.( C) Revesz syndrome has many of the features of DC and presents in early
childhood in addition to Bilateral exudative retinopathy is required to establish
a diagnosis. Patients may also have intracranial calcifications, IUGR,
developmental delay, and bone marrow failure.
.
53 (C) Owing to additional risk of liver and pulmonary fibrosis.

492
neurologic and
cardiac. Find
common anomalie
54.( 0) The most and cerebral
atrophy are frequent, and develops«
heart disease includes atrial *
4i
related to cerebellar feature Congenital
-
delay is a prominent ductus arteriosus, tetralogy of Fallot .TO
'

defects , patent 9

ventricular septal
.
coarctation of the aorta
55 ( ®l , Hepatitis C, Hepatitis non.,
Fostein-Barr, Hepatitis B
56.(0 Cytome® a ° 3 tJe hepatitis) and HIV. Parvovirus B19 is classic
^
asTociatTi
assoaaieo wu isolated
cell disease or
red
immunodeficiency
blood cell ( RBC ) aplasia , but in patients with sic
, it can resujt
jn transient pancytopenia.
,
fluorocytosine ), and dose
J
*
( Chloramphenicol , dapsone ,
57.(B ) Dose dependent
dapsone, sulfonamides, tetracycline
independent (Chloramphenicol,
amphotericin , quinacrine , chloroquine, pyrimethamine).
methicillin,
aplastic anemia is defined as a condition in which > 2 cell
58.( 6 ) Severe
components have become seriously
compromised (absolute neutrophil count
<S00/mm3, platelet count <20 000/
, mm3, reticulocyte count <1% after
biopsy material has
correction for hematocrit ) in a patient whose bone marrow
<30% cellularity .
59.( C)
60.(A)
61.{ C)
62.( E) Presence of JAK 2 or similar mutation, ( A ) is a major criterion if > 25 %.
63 ( A )
64.( A ) Phlebotomy is the initial treatment of choice to alleviate symptoms of
hyperviscosity and decrease the risk of thrombosis.
65.( B } Maintain hemoglobin > 12.0 g/ dl and severe pulmonary disease; maintain
hemoglobin > 12.0 g/ dl during extracorporeal membrane oxygenation; maintain
hemoglobin > 10.0 g / dt and moderate pulmonary disease; maintain hemoglobin
>12.0 g/dl and severe cardiac disease; maintain hemoglobin >10.0 gM
preoperatively and during major surgery;
maintain hemoglobin >7.0 gM
postoperatively; and maintain hemoglobin > 7 0 g dL
. - / and symptomatic anemia.
66.( D ) Another factor is the rapid disappearance of
f

EPO from infant plasma (i.e ,


accelerated metabolism ) .
67.( D )
68 ( C)
69.( E ) In patients with inherited disorders , PLT transf
uslons are justified only if
the risk of significant bleeding is quite high or if
bleeding is overt, because
ns may ieai)
** for infants
below 4 months,
granulocyte , ,
6
neutrophil count < 3.0 x
bacterial infection,
105 /1in lst Jof ran
ionis iindicated
10 * 10 A with
d fU|rninant thereafter
-
Cert ain diseases, such as hypothyroid
71 (C ) ism and
acid , can lowe r VWF levels in affected dedications, such as
v3lproic patients
requ jred to rule out or confirm a diagnosis of VWD RePeat testing may
level . ' Other choices i
crease the
'"
72 -(E) Characterized
by a defect in the ability
of VWF to bind
factor VIII is seen in Type 3. FVill also low
73 |C) 2B and platelet -type VWD.
74.(A) Desmopressin is beneficial in Type 1 and some type 2
VWD with caution.
75.(E)
76.(B)
77.(B) Leukopenia suggests another diagnosis.
78.( A ) Chronic Immune thrombocytopenia is an
indication for autoimmune
assay.
.
79 { B } Isolated enlargement of the spleen sugge
sts the potential for
hypersplenism caused by liver disease or portal vein
thrombosis.
80.( B) Autoimmune thrombocytopenia may be an initial
manifestation of SLE ,
HIV infection, common variable immunodeficiency,
and rarely, lymphoma or
autoimmune lymphoproliferative syndrome.
81.(A ) Observation in this condition is best and inexpensiv
e.
82.1C) Rapid elevation of platelets is needed in cases of social hyperactivity.
83.( B) It gives 80% cure.
84.( B ) The majority of patients have mild sympt
oms despite severe
thrombocytopenia, early therapy does not affect the outcome of disease,
transfused platelets are liable for destruction so as the autologous platelets, and
observation and watchful waiting is the best current strategy in cases of mild
bleeding.
85 -(D ) The role of
splenectomy in ITP should be reserved for 2 circumstances:
U) the older child ( > 4 yr ) with severe ITP that has lasted >1 yr (chronic ITP ) and
whose symptoms are not easily
controlled with therapy and ( 2) when life -
threatening hemorrhage ( ICH) complicates acute ITP, if the plate et count
^nnot be corrected rapidly with transfusion of platelets and administration of
VIG and corti
* costeroids.

494
SLE), chronic infectious disordg
86 (C) Autoimmune disease r
thrombocytopenjai SU£h $ (e «-, h'
3nd
r 'lTvon Willebrand disease, X -linked thrombocytopen;a
, *^*
' ^
plate et‘ , common variable immunode iCien \.
ferative syndrome
IVZoTatacrothrombocytopeni ^ and Wiskott -Aldrich syndrome
^
87.(A)
gg|g|
89 (E) Effective in 80-
95% of patient.
study are diagnostic.
90.( E) Findings in the BMA
to cow ' s milk formula ( present in 50%) may
91 .( A ) Intolerance comP; C. ( ,

management by triggering gastrointestinal bleeding, jncrea


thrombocytopenia ia, eosinophilia, and a leukemoid reaction. >
'
92.(B) MYH9-related thrombocytopeniacomprises a number of divert
hereditary thrombocytopenia syndromes ( e.g.; Sebastian, Epstein, May -Heggim
Fechtner) characterized by autosomal dominant macrothrombocytope
neutrophil inclusion bodies, and a variety of physical anomalies, including
^
sensorineural deafness, renal disease, and eye disease.
9m
94.(D) Splenomegaly may result from obstruction in the hepatic, portal, or
splenic veins leading to hypersplenism. Wilson disease, galactosemia, biliar ,
atresia, and al-antitrypsin deficiency may result in hepatic inflammation
fibrosis, and vascular obstruction.
95.) B)
96.(A) Howell-Jolly bodies is seen in sickle cell disease, basophilic stippling in
lead and heavy metal poisoning, Pappenheimer bodies
in sideroblastic anemia
Cabot rings in megaloblastic anemia, and
hemoglobin crystals in hemoglobin C
disease.
.
97 ( A) Group A streptococci
and Staphylococcus aureus.
98.( B) A firm, fixed
node should always raise the question of malignancy,
rpoarwioec

physical findings .
abMnCe
° f SyStemic sVmPtonns or 0
, "
her ab m> '
99.( D) A prolonged
course is more suggestive of infectious agents rather than
,
bacteria .
100. E )
101.( B ) HHV - 8
maY stimulate excessive production
102.( 0) of interleukin - 6 ( IL-6) -
103.( A)
Cancer and Benign
Tumors
Questions
UASANEIN H. GHALI
1. Which one of the following malignancies is associated
with the use of
alkylating agents ?
A . Brain tumors
B. Hodgkin disease
C. Osteosarcoma
D. Neuroblastoma
E. Hepatoblastoma

2 . Which of the following viruses is associated with the development of Kaposi


sarcoma ?
A. Epstein- Barr virus
B. Human herpes virus
C. Hepatitis B virus
D. Hepatitis C virus
E. Human papilloma virus

3. The development of cancer that is linked to genomic imprinting is best


represented by :
A . Bloom syndrome
B . Trisomy 21
C. Beckwith- Wiedemann syndrome
D. Ataxia telangiectasia
E . Wiskott - Aldrich syndrome

A. Which of the following agents has a side effect of pseudotum


A . Methotrexate
B. Vinblastine
C. Cytarabine
D . Tretinoin
E Etoposide

496
following agents has a significant possibility of a||er
5 Which of the 8ic rea
A. Etoposide %
B . Methotrexate
C. Cisplatin
D. L- Asparaginase
E. Vinblastine

6. Which of the followi ng


cancers is more common in school- age children?
A. Hodgkin lymphoma
B. Neuroblastoma
C. Retinoblastoma
D . Wilms tumor
E. Hepatoblastoma

7. Which of the following lymphadenopathies, region - wise, are highly suggest


^
of malignancy ?
A . Anterior upper cervical
B. Post auricular
C. Supraclavicular
D. Axillary
E. Inguinal

8. A "new - onset asthma " symptoms during adolescence may be a presentation


of
A. acute lymphoblastic leukemia
B. lymphoma
C. neuroblastoma
D. askin tumor
E. brain tumor

9. PET scan as a part of workup in staging of pediatric cancers indicated


is in
which of the following malignancies ?
A. CNS tumor
B. Neuroblastoma
C. Retinoblastoma
D. Hodgkin lymphoma
E. Hepatoblastoma
F analysis as a
part of work up in
10- c5 staging of nediatn
P d
of the following malignancies ? C
cancers is indicated
10 which lymphoma
A Hodgkin
B. Wilms tumor
Q Bladder rhabdomyosarcoma
p Non- Hodgkin lymphoma
E Osteosarcoma

11. Which of the following chromosomal translocations is associated with


acute
promyelocytic leukemia ?
A. t ( 9;22 )
B, t(l; X9)
C t(l5;17)
D. t ( 2;13 )
E. t(ll;22 )

12 . A 6- year- old boy under current treatment of acute lymphoblastic leukemia


has exposed to a brother with active chicken pox .
Of the following, the MOST appropriate management is
A . oral acyclovir
B. intravenous acyclovir
C. varicella - zoster immunoglobulin
D. close observation for any skin lesions
E. close observation for any temperature rise

13 . Among all the aspects of pailiative care, the most serious cause of suffering
of a cancer child is
A. hair loss
B. psychological trauma
C. pain
D . fear of death
E . fear of loss of function

14. Which of the following agents has been associated with neurocognitive
deficits as late effects of cancer treatment ?
A . Cyclophosphamide
B. Etoposide
C. Vincristine
498
0. Daunomycin
E, Methotrexate

following agents has been associated with hearing


15. Which of the f0s 8s
? lau
effects of cancer treatment
A . cyclophosphamide
B. carboplatin
C. etoposide
0 . vincristine
E , daunomycin

16 . Which of the following matches is TRUE regarding late effects of cancer


therapy and uses of chemotherapies ?
A . Renal insufficiency and daunomycin
B . Cardiomyopathy and busulfan
C . Pulmonary fibrosis and melphalan
D. Gonadal dysfunction and procarbazine
E. Peripheral neuropathy and carboplatin

17 . Which of the following matches is TRUE regarding oncological emergencies


and cancers?
A. Hyperkalemia and medulloblastoma
B. Hyponatremia and rhabdomyosarcoma
C . Disseminated intravascular coagulation and lymphoma
D. Spinal cord compression and neuroblastoma
E . Superior vena cava syndrome and hepatoblastoma

18. Which of the following agents has the


LEAST myelosuppressive effect ?
A - Dactinomycin
B - Daunomycin
C - Methotrexate
D- Vincristine
E Carmustine
-

19. What is the MOST common pri


primary
cancer ?
modality of treatment of childhood
A . Chemotherapy
B . Surgery

499
c Radiothera py
agent therapy
p‘ Biologic
therapy
£ Laser

of the following tissue cells is


20 Which MORE susceptible to
therapy ? effects of
rhe^o
A Neurons
B. Muscle cells
C Connective tissue
0. Epidermis
E . Bone

2i. What is the most common adverse effect of radiation therapy in children
?
A . Dermatitis
B . Mucositis
C. Somnolence
D. Alopecia
E . Nausea and diarrhea

22 . Which of the following agents is used as prophylaxis in patients with cancer


to prevent infection with Pneumocystis jiroveci?
A. Amoxiclavulanic acid
B. Trimethoprim- sulfamethoxazole
C. Fluconazole
D. Ciprofloxacin
E. Acyclovir

23 . Patients receiving immunosuppressive therapy should receive irradiated


blood products to prevent
A- graft- versus- host disease
B- transfusion- associated reactions
C- infections
D- febrile neutropenia
E- thrombocytopenia

24. Which of the following presenting features is common in acute


lymphoblastic leukemia as compared to acute myeloid leukemia .

A. Subcutaneous nodules

500
intr ava scu lar coagulation
B. Disseminated
C Granulocytic sarcoma
D. Mediastinal mass
E. Gingival hyp
ertrophy

wing features of acute


myeloid leukemia is
25. Which of the follo Spe
*fic f,
infants ?
A Blueberry muffin lesions
intr ava scu lar coagulation
B. Dis sem inat ed
C. Chloroma
D. Splenomegaly
E. Gingival hypertrophy

is MORE common in acute promyelocyte


26. Which of the following features
myeloid leukemia ?
leukemia rather than other types of acute
A . Subcutaneous nodules
B. Disseminated intravascular coagulati
on
C. Granulocytic sarcoma
D. Splenomegaly
E. Gingival hypertrophy

d with
27 . Which of the following features of acute myeloid leukemia is associate
t { 8;21) ?
A. Subcutaneous nodules
B. Disseminated intravascular coagulation
C. Chloroma
D. Gingival hypertrophy
E - Huge splenomegaly

afu!c
28 . Which of thefollowing chromosomal abnormalities is usually seen in
promyelocytic leukemia ?
A. t ( 8;21)
B. inv { 16 )
C- t ( 15;17 )
D llq23 abnormalities
E. del( 7 q), - 7
„2 which Of leukemia
the following agents has im r

^ dramatically
^ “ the need
Ved

bon.
yelocytic and def the survival
01

^ notion? acute
A All-trans-retinoic acid 'or
harrow
6. Arsenic trioxide
C Cytarabine
0. Anthracycline
E. Etoposide

living enlarged liver and spleen* duMnTtheTr n as


*
states that he is active and good feeder, with no obvious feat '
ures of
mother
General physical and regional examination was normal apartsickness ,

hepatosplenomegaly . An initial complete blood count from


showed Hb 12.0 em/ dl
WBC 55,000 / cmm with many blast cells, and platelets
70,000/cmm. BMA
showed acute leukemia.
Which of the following facts discussed during counseling is inadmissible ?
A. Ten percent of neonates with Down syndrome develop a transient
leukemia .
B. These features usually resolve within the first three months of life
C. At time being, less intensive chemotherapy should be instituted to
induce remission
D. After resolution, 20- 30% will develop acute leukemia in the next 3 years
E. Later on, such cases will develop acute megakaryocytic leukemia

31- Ninety- nine percent of chronic myeloid leukemia cases are characterized by
which of the following specific translocation ?
A . t{ l;19 )
B . t ( 4; ll )
C. t (9;22 )
D. t ( 12;21)
E t ( 10;14 )
physician
32. A 6-year -old boy is referred to the pediatric clinic from a generalexperience
for fever, fatigue and weight loss for the last few weeks The patient , he is
During examination
,
Pain in the abdomen below the left side of the chest
. J f"
.
bdomjna| distension.
.
38.3 C, no lymphadenopathy but - umbj|iCus slightly tender
^
pale > ebnle
Abdominal examination shows a huge spleen reac i
502
,,f ,
'' -
and eurroloeiM e*aminat
on Pa «at\
ipa on. Cardi0Va rll
d count sh hows Hb 9.5 9 " °' * '„ ate no
’ mC 53,000/cmm, plWei
°"
^,
Comp et® . der0 .
560,000/crn The periphera hio0d °\
normocytic
anemia .h teukocytosis 5
% monocv •
% . B 17% ( 0
eosinoph ^
5% notmodw
' metamyetocyt
and 3% basophils ^
^
*"*2J* *,• . ;
; is
« « ,^
*** hvperCeUuW, 4
Jn

increased " .
^
e ce e and P

Of the following, the MOST likely diagnosis is


A . Acute lymphoblastic leukemia
cells. Megakarwi ^th
^° ^ are

B. Toxoplasmosis
C. Acute promyelocytic leukemia
D. Chronic myeloid leukemia
E . Kala azar -

33 . Which of the following genetic syndromes has a predilection for juvenile


myelomonocytic leukemia ?
A . Edward syndrome
B . Noonan syndrome
C . Turner syndrome
D . Klinefelter syndrome
E. Angelman syndrome

34 . Which of the following factors is stratified as high risk in treatment of acute


lymphoblastic leukemia ?
A . Age at diagnosis of 9 years
B WBC at diagnosis of 33, 000/cmm
.

C. Pre- B- cell detected by flowcytometry


D Philadelphia chromosome detected by cytogenetic study
.

E . Rapid response to therapy

during
35 . Which of the following agents is added to the higher risk patient
induction therapy of acute lymphoblastic leukemia ?
A . Prednisolone
B . Daunomycin
C . Asparaginase
D . Vincristine
E. Intrathecal methotrexate
503
36 - Which of the following infecti
,^
lymphLT
Quires
chemotherapy for acute pn b astic leukerrij ,prophylacti»c
° treatment
A. Pneumocystis jiroveci *
viridans
B. Streptococcus
donovani
C. Leishmania
D . Escherichia coli
E. Pseudomonas auroginosa

lymphoblast,cS euTemla >freqUen',


^ ^
37. V
encountered in T -cell rather tha B -cell acu e |
A- Hepatosplenomegaly
B- Respiratory distress
C- Bone pain
D- Pallor
E- Fever

38. Which of the following is a B-symptom in Hodgkin lymphoma ?


A. Pruritus
.
B Anorexia
C. Painful limbs
D. Lethargy
E. Drenching night sweating

39. Which of the following is the typical presentation of Hodgkin lymphoma ?


A. Left cervical progressive swelling for 3 months in a six - year old boy, no
other constitutional symptoms; active daily living is perfect.
rapidly
B. A nine -month old boy with shortness of breath due to

C. ZXeSfJ«
examination showed
r for 4 months
generalized V
hepatosplenomegaly. appearance of
D . Chronic abdominal pain in a ten-year old boy with
lymph node at the
jaundice in the last few weeks, sonogram showed big
hepatis
porta
. -
E. An axillary swelling of few monthS a nine vears old
girl, with fever and
b rcu|0sis patient
,
weight loss; history of contact with tuoer
40. A 10- year-old boy is recently
diagnosed with Hodgkin lymphoma .
h
axillary lymph nodes involvement . No history
^
right sided cervical and of fev
the last 4 rm n
° '
weight loss about 15 % in
the last weeks, with remarkable
identify more involved lymph nodes gr nths
Work up and staging have failed to 0ljps
for this patient ?
What is the correct stage of the disease
A . IB
B . IIA
C . MB
D . IIIA
E . IllB

41. A newly diagnosed Hodgkin disease in a thirteen year old girl with bilateral
-

cervical lymph nodes involvement . The family reported one episode of high
fever ( 40.0C ) in the last days, but without weight loss or sweating, Work up
showed liver multiple hypoechoic lesion by sonography with high metabolic
activity by PET scan suggestive of liver involvement .
What is the correct stage of the disease for this patient ?
A. IIB
B IIIB
.

C . IIIA
D. IVA
E . JVB

42 . What is the cutoff time to use standard treatment rather than myeloablative
autologous stem cell transplantation in children with relapsed Hodghn
lymphoma ?
A . 6 months
B. 12 months
C. 18 months
D . 24 months
E . 30 months

1d
43 . In the perspectives of International Pediatric Non - Hodgkin Lymphon
Staging System, which of the following matches is CORRECT ?
A . Mediastinal tumor is stratified as stage I
!
B. A primary completely resected gastrointestinal tumor is stra tified
stage l
C. Any paraspinal tumor is stratified as
stage II
505
,
Two or more nodal site on the same si
D. Slte f the
as stage III °diaphragm ls
stratified
. A single bone lesion with
n regional
E lymph node
as stage H involvement \ls stratified

AnessVof
ortmother lasTleek '
^
Z examination
On
breath for the

, is febrile ( 38.5C
with bw g
noted that his face had become swollen
he ), no
dTfem'
^^
and
and

pallor, and no skin


'
££Ej
°
bleeding in
moderate respiratory distress, with puffy face and engorged neck vessels; there
are dilated superficial veins on the upper chest. Lymph node examination
revealed enlarged left supraclavicular lymph nodes. Abdominal examination
shows normal soft abdomen with no palpable masses or organomegaly . Initial
radiological evaluation shows big mediastinal mass; Complete blood count and
bone marrow aspiration are normal, liver and renal function are normal, lactate
dehydrogenase is high .
Of the following, the MOST likely diagnosis is
A . acute lymphoblastic leukemia
B. neuroblastoma
C . germ cell tumor
D. sarcoma
E. lymphoblastic lymphoma

*lth the
^^
3te
45. Which of the following types of lymphoma is |
therapeutic approach of childhood acute lymphoblastic leukem
A . Burkitt lymphoma
B. Germinal center B- cell like lymphoma
C. Anaplastic large cell lymphoma
D. Lymphoblastic lymphoma
E. Diffuse large B- cell lymphoma
ic non -Hodgkin
pediatric
diagnosed
46. Which of the following scenarios of newly
lymphoma is indicated for radiotherapy as breathi 8 a n d
A . A six - year old girl with huge jaw
"
swallowing, diagnosed as Burk tt
yrinP pressure e e o
'
B . A three - year old boy with big ab
mass causing
as d > use a
hrosjs diagnosed
hy
the ureters and consequent
B - cell lymphoma
506
intestinal mass with bilateral facja|
C. A two- year boy with Pals
'

diagnosed as Burkitt lymphoma


with acute superior mediastinal syndrome, diaBn
^
- Vear old boy
D. Aten os
as lymphoblastic lymphoma ^
E A six -year - old boy with
including liver , spleen, and lungs.

disseminated anaplastic large cell vmph

recognized complication seen in newly diaSnoseg


47 which of the following is a
,

starting treatment ?
patient with bulky Burkitt lymphoma after
A . Disseminated intravascular coagulopathy
B . Septicemia
C . Tumor lysis syndrome
D . CNS encephalopathy
E . Hemorrhagic pancreatitis

48. A 6- year - old boy is recently diagnosed with Burkitt lymphoma . Treatment is
started with intravenous fluid hydration and chemotherapy . Three day later, the
boy develops tremulousness and unusual spasm of both hands. Review of his
charts shows reduced urine output for the last 12 hours with slightly elevated
blood pressure. The body weight is increased by 500 gm comparing to that at
time of admission. These findings are consistent with :
A . acute renal failure due to chemotherapy
B. tumor lysis syndrome
C . postrenal urinary tract obstruction by the tumor
D . renal failure from lymphomatous infiltration
E . acute renal injury from dehydration

49. What is the most common malignancy below age of 19 years ?


A . Leukemia
B. CNS tumor
C. Rena! Tumors
D Lymphoma
.

E . Sarcomas

"nlS ' the Nlow of the cerebellum )


seen
.
o
A Tuberous sclerosis
^ g

B . Von Hippel- Lindau syndrome

507
C. Li-Fraumeni syndrome
D . Cowden syndrome
E . Turcot syndrome

SJZL’iS'
A. Neurofibromatosis type 1
<> »
* . »

B. Neurofibromatosis type 2
C. Tuberous sclerosis
D. Cowden syndrome
E. Von Hippel- Lindau syndrome

52 . What is the most common primary brain tumor in children ( 0- 14 ) year old ?
A . Pilocytic astrocytoma
B. Glioblastoma
C. Ependymal tumors
D. Meningioma
E. Pituitary tumors

53. What are the most common congenital and neonatal brain tumors ?
A . Teratomas ( mature and immature )
B . Embryonal tumors
C. Astrocytic tumors
D. Neuronal tumors
E. Mixed neuronal-glial tumors

54 . in young children, the diagnosis of brain tumor may be delayed


because
rate
A . the tumor increases in size very slowly due to low proliferation
this age group such as
B . the symptoms are similar to common illnesses in
vomiting
C . there is a good space for these tumors to
grow in before producing
symptoms
family of the child
D . the symptoms are usually neglected by the
this age group
E - the diagnosis for brain tumors is difficult in

frequently seen in supratentorial


55 . Which of the following features is more
tumor as compared to infratentorial tumor ?
A. Headache
508
B. Vomiting
C. Focal motor weakness
D. Nausea
E. Torticollis

standard neuroimaging modality for diagnosis of primarV


56 . What is the bra,,
tumors ?
A. CT scan
B. MRI
C. PET scan
D. MRA
E. Sonography

57. In addition to the routine evaluation of brain tumors, special consideration


should be taken in tumor of Pituitary, suprasellar and optic chiasmal region to
look for
A . metabolic assessment
B. respiratory function evaluation
C. endocrine function
D. hematological evaluation
E. cardiac function

58. What is the preferential site for CNS germ cell


tumors ?
A . Suprasellar
B. Pontine
C. Cerebral cortex
0. Cerebellum
E . Medulla

59. Which of the following


statements regarding ependymomas children is
TRUE ? in
A . The median age of presentation
is 10 years
B. The majority occur in the
subtentorial location
C. Radiologically, it appears like
a well-ci
enhancement. circumscribed lesion with variable
D . Tumor in the posterior fossa is
associated with better
E . Surgery alone is usually curative outcome
is the MOST common CNS P
60 - vvhat mbrVonaltumor ?
A Medulloblastoma
g Supratentorial PNET
C. Ependymoblastoma
D. Medulloepithelioblastoma
, ,
E • AfVPical terato d/rhabdo d tumor

the following malignancies k


61. Which of aSSOCIated with Chioroma ?
A. Acute lymphoblastic leukemia
B. Acute myeloid leukemia
C. Non-Hodgkin lymphoma
D. Neuroblastoma
E. Medulloblastoma

62. What is the most common extracranial solid tumor in children ?


A. Wilms tumor
B. Neuroblastoma
C. Non-Hodgkin lymphoma
D. Hodgkin Lymphoma
E. Rhabdomyosarcoma

63. What is the most commonly diagnosed malignancy in infants?


A. Wilms tumor
.
B Neuroblastoma
.
C Non-Hodgkin lymphoma
D. Hodgkin Lymphoma

« . .
E. Rhabdomyosarcoma

«

limping with fever for the a
weight during this period an
r:
r zr laming

co|or became Igbter


of bone pain and

of his usual activity. Also, '


he has been loped a bluish
she
discoloration around both eyes. ° "“ *ce
he with
skin
bilateral periorbital ecchymoses, n node enlarge
discoloration. Abdominal examination potable for uPp M sjded deep-
seated abdominal mass that is below
and spleen is not
Of the following
palpable.

- .^
*
„„nnsiss is
, the MOST likely d agnos
Liver |S 2 cm below costal
margin

510
A. Wilms tumor
B. non-Hodgkin
lymphoma
C. neuroblastoma
D. leukemia
E . systematic lupus
erythematosus

with massive liver involvement is


65. The syndrome of Neuroblastoma called
A. pepper syndrome
B. Horner syndrome
C. Hutchinson syndrome
D. Kerner- Morrison syndrome
E . Neurocristopathy syndrome

66. Limping and irritability in young child with neuroblastoma associated with
bone and bone marrow metastasis referred to as
A. Pepper syndrome
B. Horner syndrome
C. Hutchinson syndrome
D. Kerner-Morrison syndrome
E. Neurocristopathy syndrome

67 . An 18- month - old boy who is referred to the


pediatric hospital forevaluation
of chronic diarrhea and failure to thrive
for the past few months with no
response to any medical therapy or
therapeutic formulas. All diarrhea -related
lab tests are negative. The accompanied
sonography shows a small right sided
adrenal gland mass, which is confirmed by
CT scan .
Of the following, the MOST likely
cause of diarrhea is
A. food allergy
B . vasoactive intestinal peptide
s
C. associated celiac disea
se
D . small intestinal
bacterial overgrowth
E . pancreatic insufficiency

68 . Spontaneous regression
of
following clinical scenarios ? neuroblastoma is reported in which of the
A. A three - year old boy with
large
vessels with isolated bone
marrow metastasis!
010
" the
* ***
*
-year old girl with small suprarenal
B. A five mass and diffuse bone
irtvolv^nn®^' marrow
-month boy with small
C- A sixteen
subcutaneous tumor nodules, T ' lvem° ent withwideoutsprebonade
massivTliverinvo Um r and

involvement.
0 A f ur'mon
th Old girl with a mediastinal tumor
°
syndrome
and Horner

E. A $ix- vear old girlvvitb causing acute paralysis from


spinal cord compression

69. What is the MOST common metastatic site of Wilms tumor ?


A. Lung
B. Bone marrow
C. Brain
D. Spleen
E. Bone

70. A 9-month old boy develops abdominal distension noticed by the mother
*

atal
for the last few weeks. Past history reveals a birth weight of 5.5 kg and neon
large tongue,
surgical correction of omphalocele. On examination; he has
the lower limbs giving
abdominal distention, and asymmetrical appearance of
side. Ultrasonography
an impression of larger left lower limb than the right
shows large liver and pancreas.
Of the following, the MOST likely diagnosis is
A. WAGR syndrome
B. Denys-Drash syndrome
C. Beckwith- Wiedemann syndrome
D. Perlman syndrome
E. Trisomy 18
,
with fema le external genitalia
71. Features of ( Male pseudohermaphroditism ) are linked to Wilms
gonadoblastoma
focal segmental glomerulosclerosis and
tumor . These features are suggestive of
A. Perlman syndrome
B. Denys-Drash syndrome
C. Frasier syndrome
D. Schimke syndrome
E. Galloway - Mowat syndrome
512
72 . A 10-year -old girl is referred to the hematology/oncology Ur) j
diagnosis of Wilms tumor based on imaging studies. On examine1- after
short stature, microcephaly, and multiple cafe -au-lait s p o t S l
trunk. ****« $
Of the following, the MOST likely diagnosis is
A. Diamond-Blackfan Anemia
B. Oyskeratosis Congenita
C. Shwachman- Diamond syndrome
D. Cartilage hair hypoplasia
E. Fanconi anemia

73. A 3-year -old boy is referred after initial diagnosis of Wilms tu


mor based y
the imaging studies. He has a history of early onset renal
failure and rerta
mesangial sclerosis and pseudohermaphrodism.
Of the following, the MOST likely diagnosis is
A . WAGR syndrome
B. Denys -Drash syndrome
C. Beckwith- Wiedemann Syndrome
D. Frasier Syndrome
E . Perlman Syndrome

74. The clinical characteristics of Polyhydramnios


, macrosomia, distinctive facial
eatures, renal dysplasia, nephroblastomatosis
, and multiple congenita'
anomalies is indicative of
A. Simpson-Golabi-Behmel syndrome
B. Costello syndrome
C. Perlman syndrome
D. Sotos syndrome
E. Beckwith-Wiede
mann syndrome
75. What is the MOSTi
common initial clinical presentation
A. Abdominal pain 4
of Wilms tumor ?
B. Abdominal mass
C. Hematuria
D. Hypertension
E. Fever
conditions ,is Gratified
the following
Which one of
Lims tumor ?
nv
as stage v
disease of
^B. Bilateralrena
Lung metastasis
** ° lvement

c A tumor thrombus extended into the inferior


cava
Q Liver metastasis
E. Lymph node metastasis outside the abdominopelvic
region
MOST common solid renal
77. What is the tumor identified in
the neonatal
period?
A. Mesoblastic nephroma
B. Wilms tumor
C. Clear cell sarcoma
D. Rhabdoid tumor of the kidney
E. Renal cell carcinoma

78. What is the MOST common anatomic site affected by Rhabdomyos


arcoma?
A. Head and Neck
B. Orbit
C. Hepatobiliary
D. Extremities
E. Retroperitoneal

79. Sarcoma botryoides is a grape like mass of Rhabdomyosarcoma locat in the


A. Vagina
B. Rectum
C. Neck
D. Orbit
E - Cheek

8
°, Which of the following is a benchmark
os eogenic
sarcomas
to differentiate between Ewing and
for
A - Age of
onsent is the second decades for Ewing, but the third decade
osteogenic
B• do so in osteogenic
Females gender predominates in Ewing while male
sarcoma metaphysis
Site of predilection is diaphysis of long bones in Ewing while
the long bones in osteogenic
514
it is common in osteogenic
D. Fever is rare in Ewing while
E . Lung and bones are common
metastatic sites in Ewing while ,
|lv

bone marrow in osteogenic sarcoma *^


is a rare condition associated
81. Rothmund- Thomson syndrome with whlch ,,
?
the following group of manifestations
,
.. .
small hands,
. ..
A. Short stature, skin telangiectasia and hypoplasticity of the
thumbs
and feet, and absenCe
B Short stature, skin melanoma, large hands of th ,
kidneys
,
C. Tall stature, skin telangiectasia, large feet and hypoplasticity of
thumbs ^
D Short stature, skin telangiectasia, small hands and feet, and extra .

thumbs
E. Tall stature, skin rash, large hands and feet, and hypoplasticity of the
thumbs

82. A 13- year -old boy presents with 3 -month history of left lower thigh pain and
swelling that grew up slowly . He has no history of trauma or injury to the site
and no fever or other constitutional features. Activity is normal except for the
last few weeks where he started to feel aches and movement restriction. Initial
CBC and other biochemical tests were normal but elevated alkaline
phosphatase. X -ray of the left thigh shows a lytic lesion in the lower left femur
with the classical sunburst pattern.
Of the following, the MOST likely diagnosis is
A . Ewing Sarcoma
B. Osteosarcoma
C. Non-Hodgkin lymphoma
D. Simple bone cyst
E. Histiocytosis

83. What is the MOST important prognostic factor in osteosarcoma ?


A. Age
B. Gender
C. High LDH level
D . Histologic response to chemotherapy
E. Primary site of tumor

515
with bone tumors mav h
84 - Why patients
,
The pathological differentiation of bon ^ diaS
"°sis >
'

A.
B. Patients at £*' *** nostic work

are usually nonsoe


c. Clinical featureslikely
D. Symptoms are to be attributed ° '
tn
*
^ disea itself
*
E. Apart from biopsy, no specific diagno 7

boy presented with 2 - week history of fever, mild lower back


g5 A 12 - year -old
standing and walking with fever at day and night . This
pain, and troublesome
progressed to complete inability to move his lower limbs that ensued suddenly
.
within the last 24 hours There was no antecedent history of trauma. On
examination, there was bilateral spastic paraplegia with only a flicker of
movement at the right ankle joint . Sensory examination revealed mild
hypoesthesia to all sensations below the level of DIO dermatome . Tendon
reflexes of his lower extremities were exaggerated and Babinski response was
in the D 9
extensor bilaterally . Magnetic resonance imaging revealed a lesion
vertebra compressing the spinal cord.
Of the following, the MOST likely diagnosis is
A. Ewing sarcoma
B. rhabdomyosarcoma
C. osteogenic sarcoma
D. leukemia
E. Burkitt Lymphoma

86, What is the characteristic radiographic sign of Ewing sarcoma ^


A. Moth- eaten appearance
B. Codman triangles
.
C Onion skin appearance of periosteum
D. Sunburst appearance of bone lesions
E Popcorn calcifications

87. Which of the following malignancies is radioresistant ?


A. Ewing Sarcoma
B Lymphoma
C. Neuroblastoma
D. Osteosarcoma
E Wilms' Tumor
516
can be diagnosed by radio
following benigH bone lesions
88. Which of the
only ? histiocytosis
A. Langerhans cell
B. Fibrous
dysplasia
C. Osteomyelitis
0 osteochondroma
F. Osteoblastoma
,. lower thigh swelling that has b

foticedT tht ,
hnu nresents with right
las few months. Initial radiograph shows lesion in the r „*'
&hi
femur , suggestive of malignant nature of
is
Which of the following symptoms ^
lesion ?
A. Night sweatig
B. Night pain
C. Poor appetite
D. Local erythema
E. Impaired daily activities

90. Which of the following benign bone lesions can be treated by observation
only ?
A. Osteoblastoma
8. Chondroblastoma
C. Aneurysmal bone cyst
D. Chondromyxoid fibroma
E. Osteochondroma

91. You are the physician responsible for


counselling with a family of a 10-year-
old boy who is recently diagnosed
as having Osteochondroma by radiographic
suggestion.
Which one of the following
, statements regarding his condition is TRUE ?
A. Surgery s not performed
unless the lesion is symptomatic
L! dlaen SreqUlreSCOreneed e biopsy
C. Malignant°degeneration
' S

a P ssibleils
|

D. It is not common
during
E . Usually arise from
°
childhood
squeal in childhood

vertebrae and pelvic bone


Which of the following conditions has features closely resemble bone
infection
*

A Osteochondroma
osteoma
B. Osteoid
c Eosinophilic granuloma
p Angiosarcoma
E Aneurysmal bone cyst

syndrome refers to a condition of angiomas of the soft tissue with


93. Maffucci
multiple
A , Osteochondroma
B. Enchondroma
C. Chondroblastoma
D . Chondromyxoid fibroma
E . Osteoid osteoma

94. What is the classical presenting feature of retinoblastoma ?


A. Pain
B. Decreased vision
C. Strabismus
D . Leukocoria
E. Pupil irregularity
95. The diagnosis of retinoblastoma is essentially established by
A . ophthalmoiogical examination
B . MRl brain
C. CT scan brain
D . local Sonography
E. biopsy

96, A common differential diagnosis of retinoblastoma is


A. glaucoma
B. endophthalmitis for toxoplasma gondi
C. idiopathic retinopathy
D . raised intraocular pressure
E. hyperplastic primary vitreous

associated with increased risk for


97. Which of the following conditions is
mediastinal germ cell tumors ?
MS
A . Down syndrom
e
syndrome
B . Klinefelter
C . Inguinal hernia
D Testicular
microlithiasis
l Cryptorchidism
for teratoma ?
Wha t is the most common site
98 .
A. Ovaries
B. Testes
C. Sacrococcygeal
D. Mediastinal
E. Brain

mon site for Embryon al carcinoma ?


99. What is the MOS T com
A. Ovaries
B. Testes
C. Sacrococcygeal
D. Mediastinal
E. Brain

follo wing gon ada l tumo rs can cause virilization or precocious


100 . Which of the
puberty ?
A . Yolk sac tumor
B. Dysgerminoma
C. Sertoli- Leydig cell tumor
D . Seminoma
E . Gonadoblastoma

follo wing Germ Cell Tumors .


101. Radiotherapy is important in which of the
A . Testicular tumors
B . Ovarian tumors
C . Mediastinal tumors
D . Central nervous system tumors
E. Sacrococcygeal tumors
he
n of <
102 . Which of the following markers provides important confirmati fo mot °
diagnosis of Endodermal sinus tumor as well as to monitor the patient
^
response and recurrence ?

519
- Chorionic Gonadotropin
( jufffsn

Lactate dehydrogenase
c A|pha fetoprotein
0 Ferritin
protein
E C -reactive

/ hich of the following subtypes of


. ^al
103 alpha -fetoprotein level and
worse
hepatoblastoma
\s
norm outcome? associated with
A Fetal
type

B. Embryonal
type
c Mixed histology type
0 small cell undifferentiated type
l Mesenchymal element type

104. Which of the following syndromes is a risk factor for development


hepatocellular carcinoma ? of
A. Down syndrome
B. Fragile X- syndrome
C. Fanconi syndrome
D. Patau syndrome
E. Alagille syndrome

105. What is the MOST common benign tumor of infancy?


A. Neurofibroma
B. Fibrosarcoma
C . Hemangioma
0. Ganglioneuroma
E. Lymphangioma

1Q6. What is hema g


the MOST common complication of cutaneous
A - Ulceration
B. Secondary infections
C* Bleeding
Pain
E. Anemia
thyroid mass
107 . referred as a case of .
^^ 'year -old boy who recently has been preoperative assessment
and
f tumor by sonography for evaluation
° rnn
and physical
examination, you n
Upon further history gathering
ticed ,
affected with thyroid cancer and underwent surgical resect°ion hl s
. , v
^ fpeurofe" arnioat 00' he has fear no. \d features and and
' f"


t

that moves
'
centra 1(1Vme thyroid mutapi.
settingoH* Or
stded prominent
1

.
«
C

ca .

*Of*therSo
A.
to «+ the
” thyroid carcinoma
differentiated
^"^^
S£ho ’ o


*

B. papillary thyroid carcinoma


C. medullary thyroid carcinoma
D. follicular thyroid carcinoma
E. benign thyroid lesion

108. Which of the following organs is highly affected by Langerhans Ce


Histiocytosis?
A. Skin
B. Bones
C. Lymph nodes
D. Liver and spleen
E. Lung
Histiocytes
in Langerhans Cell
109. Which of the following organ involvements
carries a high risk of mortality ?
A . Lungs
B. Pancreas
C. Bone marrow
D. Kidneys
E. Brain

Langerh ans cell histiocytos ^ *


110. Which of the following complications of
irreversible ?
A . Pulmonary fibrosis
B. Pancytopenia
C. Chronic Otitis Media
D. Diabetes Insipidus
F. Primary Hypothyroidism
521
. A 3-year -old boy presented to the
HI weight loss, EPH
malaise, and irritability for
few ** clmie wi
Wth history of
fever
domeI?- ,'
visitingtheir family physician With 00 bVi
after
history showed no other
and
significant data, prim, ° 0us di n
m /eS gat, * °
8 SK

0ff' ear
discharge. He also
but only chm
has long hi«Ory " ' ons The
that was not responding
dermatitis . On
of
to medical
chronic"’ ° * n3 d
recurrent atopic "
dermatologist examination, he was febrile treatment prescribed by
pale, non
growth parameters, and bilateral cervical lymphad -reactive, with |ow
enopathies with
axillary palpable lymph node. The scalp, left sided
features of seborrheic dermatitis.
Of the following, the MOST likely diagnosis is
"
P 5 3 nd di3Per regions
showed

A. Rosai-Dorfman disease
B. non-Hodgkin lymphoma
C. Langerhans Cell Histiocytosis
D. immune deficiency syndrome
E. multicentric Castleman disease

112. What is the most common consistent sign of


Hemophagocytic
Lymphohistiocytosis?
A. Hepatosplenomegaly
B. Lymphadenopathy
C. Respiratory distress
D. Jaundice
E . Neurological signs
Chapter 21
Cancer and benign i umors
Answers
HASANEIN H. GHALI
myeloid leukemia .
l.( C) Osteosarcoma and acute
with lymphoproliferative disorp
2 ( B ) Epstein - Barr virus is associated
Hepatitis B virus and Hepatitis C virus are associated with hepatoceli
carcinoma, and Human papilloma virus with cervical cancer.
.
3 (0
4.(1»
^
5.(D)
6.(A) School-age children might present with lymphoma or with brain tumors
7.(Q
8.(B) Lymphoma, especially during adolescence, often manifests as an anterior
mediastinal mass . Symptoms such as chronic cough, unexplained shortness of
breath, or "new - onset asthma" are typical with this presentation and are often
overlooked.
<
9 * D)
HMD)
ll.(C) t(9;22 ) is for CML, t (l;19 ) is for ALL, t ( 2;13 ) is for rhabdomyosarcoma, and
t (ll;22) is for Ewing sarcoma .
me )
13.(C) Pain is a serious cause of suffering among patients with cancer. It maybe
the result of organ obstruction or compression or bone metastasis, or it may be
neuropathic .
14.(E| Methotrexate and cranial radiation
.
.
15 (B) Cisplatin and Carboplatin
16. ) Renal insufficiency is
JD
carboplatin. Cardiomyopathy
associated with the use of Ifosfamide, cisplatin and
is linked to the use of daunomycin and
,
onarY f brosis is linked to the use of bleomycin, busulfan and
DPu
lomu tirw
^^^
17 ( D ) Hv
eral neur Pathy is linked
° to vincristine and vinblastine.
. and tumor factors (
S promyelocytic
to sepS' to superior mediastinal mass (lymphoma
leukemia), superior
iVorne due ) . vena cava

J9|A.)
usceptible are tissues or organs
St SU
20 (P
)
^° ? 0ral and
ne rfiarroW!h< e to chemotherapymucosa
intestinal
with high rates of
, epidermis, cell turnover
liver, and spermatogonia
sUSC P and radiation .
teast therapy are cells that do not
that replicate slowly, such as
neurons, muscle cells,
•Jjjllbor*- connective

^
is the most common general
<*>
»always adverse effect, because skin

.
eatment field.
in
is

22.(B)
rece ve leukoreduced blood products to prevent
23.(A) They should s uslon
associated reactions and infections.
24.(0) T-cell leukemia
25.(A) Blueberry muffin lesions or subcutaneous nodules
26.(B)
27.(C) They are discrete masses, known as chloromas or granulocytic sarcomas.
These masses can occur in the absence of apparent bone marrow involvement
and typically are associated with a t(8;21) translocation. Chloromas also may be
seen in the orbit and epidural space.
28.(C) PML-RARA, of favorable prognosis, is a gene rearrangement involving the
retinoic acid receptors.
29.( A) Acute promyelocytic leukemia, characterized by a gene rearrangement
involving the retinoic acid receptor (t(15;17); PML-RARA ), is very responsive to
all-trans -retinoic acid (ATRA, tretinoin) combined with anthracyclines and
cytarabine. The success of this therapy makes marrow transplantation in first
remission unnecessary for patients with this d
30.(C) No treatment is indicated .
.
31(C) Philadelphia chromosome t(9;22) (q3 .qii), resulting in a L fusion
,n a
protein. This
translocation, althoug
small percentage of patients with ALL.
*
istic of CML , als
* °
shown to have
* 3*(B) It'
Spor>tanpn4
s assoc at on with Noonan syndrome
' * -
has been
an
,csrion yr or who have
i

older than 10
,
n haf |
ch
eu
wfl0 are younger than 1 yr or
to be high
considered
risk . Additional
^, Count of >50 ,000 / IL are immunophenotype
rt )dr
* cterj st cs ^
that adversely affect outcome
include -
T cell

524
therapy. Chromosomal
, resconse to initial abnormalities u.

include a rapid response


, and 17 ), and rearrangemen
.
to therapy, hyperd pfoidy, trisomy
ts of the ETV6- RUNX 1
chromosom es ( 4, 10
,(0) Daunomycin
( Daunorubicin)
35
usually is related to anemia but can occur in Pat|
37.(B) Respiratory distress
the result of a large anterior mediastinal mass ( e.g., in the thymus or n0(^ j,/
as
This problem is most frequently
seen in adolescent boys with T-ceN ALL . |
TAL
leukocyte count .
also usually has a higher
as B symptoms, that are
38.(E) Systemic symptoms, classified consider
38° C ( 100.4 °F ), weight
important in staging are unexplained fever > loss >io
total body weight over 6 mo, and drenching night sweats .
*
39.(A) Typically, patient presented with cervical or
supra clavicular swelling ,

40.( C ) According to Lugano classification for Hodgkin lymphoma .


41.( 0) According to Lugano classification for Hodgkin lymphoma .
42.(B ) Patients whose disease relapses >12 mo after chemotherapy alone or
combined-modality therapy have the best prognosis, and their relapses usually
respond to additional standard therapy .
43.(E ) Mediastinal tumor is stratified as stage III; a primary completely resected
gastrointestinal tumor is stratified as stage II; any paraspinal tumor is stratified
is
as stage III; two or more nodal sites on the same site of the diaphragm
stratified as stage IL -

44.(E ) Lymphoblastic lymphoma typically manifests as a symptomatic


mediastinal mass and also has a predilection for spreading to the bone marrow
CNS, and testes in males.
45.( 0) Localized or advanced disease requires 12- 24 mo of therapy,
including
chemotherapy, intrathecal therapy, and cranial radiation in CNS-positive
lymphoma
5
46,( D) Radiation therapy is used only in special circumstances, such as CN
r
involvement in lymphoblastic lymphoma or the presence of acute sup ® '
mediastinal syndrome or paraplegias .
47.(C) Newly diagnosed patients, especially those with BL or LBL, are at high ^
for TLS. These patients require vigorous hydration, frequent electro
monitoring, and either an xanthine oxidase inhibitor ( e.g. , allopurinO '
mg/ kg / day orally in 3 divided doses daily ) \e -
or a recombinant urate oxidase
rasburicase, 0.2 mg/ kg/day intravenously once daily for up to 5 days)
48.(6)
49.(6 ) The
CHS tumors
with
overall mortality amo
have
malignancies.
the
^
highest morK,d,' ^ » appro; hes
S 8r u
°
tV Primarj|y n
,.
" , _
iPatients
Prolog c Qf 3 1 ,
with
children
chromosome 10q 23 .
$0.{ D ) On
51 (A) 17 . astrocytoma
qll the gene IMFl .
Pilocytic and medUll.
52.(A)
turtiors are the most common tum
°
7
rS n chi
' ^
°,
blast ma/nr
en
° O- 14 ? neuroectodermal
V< arS
53.(A) During the 1
st yr of | jfe SUpratentoria
| tum
'

often include choroid P eXUS m
54.(6) '
plexus ComP « tumors and
' tT" " ** *"
* » «« and most
55.(0 Supratentorial tumors are frec ntly as,n
weakness , focal sensory change , ^ °
with focal motor
T^ e dls ers focal
asymmetry. The classical triad of headache, vomiting °^ seizures , and reflex
and nausea iis more in
favor of infratentorial tumors
56.(B ) MRI with and without gadolinium is the standard
neuroimagmg .
57.(C)
58.( A ) The suprasellar and pineal regions are preferential
sites for germ cell
tumors.
59.{C) The mean age is 6 years. The majority
occurs in the posterior fossa . The
most important prognostic factor is the
extent of tumor resection . The younger
the patient, the poorer the prognosis. Tumor in
the posterior fossa is associated
with poorer outcome . Surgery is rarely
curative.
60.( A)
6L(B)
62.(B)
63.(B)
W.(C)
65,(A) Massive
involvement of the liver with metastatic disease, with or without
respiratory distress .

67 - (B)
Intractable
secretory diarrhea caused by tumor secretion of vaso ^

'ntestinal peptides. Tumors are generally biologically favorable .


68 (C)
Infants < 18 mo old also can present in unique fashion, termed stage MS
Iprevi0us1
* y 45 - See later ), with widespread subcutaneous tumor nodules,
^ SlVe Ver nvo vernent limited bone marrow disease, and a small primary
*' '
tUrr| r w ,
' -
csan ° thout bone involvement or other metastases. The stage MS disease
spontaneously regress.
526
nodes and liver.
59.( A ) Lung, regional lymph
.
70 ( C)
71.(0
72.( 0 Associated with BRCA 2 and
PALB 2.
73.( B ) WT 1 missense mutation.
74.(0
.
75 ( 0) Hypertension is present in about
25% of patients at presentation
activity . Abdominal pam (
and h
been attributed to increased renin 40%) #

symptoms such as fever, anor


painless hematuria (18%), and constitutional
and weight loss are other findings at diagnosis
. ^
are stage IV while C is stage III.
76.( A) Bilateral renal involvement. B, D, and E
solid renal tumor identifier
77.( A) Mesoblastic nephroma is the most common
benign renal tumor in childhood
in the neonatal period and the most frequent
78.( A ) These tumors may occur at virtually any
anatomic site but are usually
found in the head and neck ( 25%), orbit ( 9%), genitourinary tract 24%), and
(
for the remainder of
extremities ( 19%); retroperitoneal and other sites account
to both patient age
primary sites. The incidence at each anatomic site is related
and tumor type,
a grapelike mass of tumor
79.( A) Vaginal rhabdomyosarcoma may manifest as
botryoides, and
tissue bulging through the vaginal orifice, known as sarcoma
can cause urinary tract or large bowel symptoms.
Vaginal bleeding or
obstruction of the urethra or rectum may occur. Similar findings can
be noted

with uterine primaries.


predilection
80.(C) Both occurred in the second decade, both have a slight male
fever is common in Ewing and rare in osteogenic sarcoma, lung and
bores

common metastatic site for both.


shor .
81.( A) Rothmund Thomson syndrome is a rare condition associated with

stature, skin telangiectasia, small hands and feet, hypoplastidty or absence
the thumbs, and a high risk of osteosarcoma.
bo -
82.( B) The lesion may be mixed lytic and blastic in appearance, but new
formation is usually visible. The classic radiographic appearance
osteosarcoma is the sunburst pattern.
83.(D ) One of the most important prognostic factors in osteosarcoma is
histologic response to chemotherapy; a poor histologic response is Sl via ^
tumor .

84.( 0)
differential diagnosis for a paraspinal/perivertebral mass in 3 chrid 3
85.( A ) The
-
wide and includes benign and malignant etiology. Malignant lesions such
EW,ne sarc0ma
chonrirosarcOITia' ' penpheral
,
oSua
f
and |ly thoW
8& lc
^
oroblastom

>
3 metastasis
bone destruction.
B and D
other
are for
, amongst others

osteosarcoma, while A and


choices are radiosensit we .
neuroectodermal
. Benign

D are for
lesions
,
tumor PNET
JP TJ
chondrosarcoma.
N .

°'
'^
g7-l
0chondroma, nonossifying
g8 (0 O te fibroma, unicameral
3 can be diagnosed by radiographs bone cVst, and
^ pain that
^
'*
9' 8
*
Pain
90$
Night awakens a child :suggests
h aspirin is common with osteoid1 >
choices are locally aggressive
osteomas
only.
maltgnancy, but

and need to be
relief of such

t e m0St common benign lesion treated.


^
wjth very rare malignant
in childhood,
usually arise from
long degeneration the
radiogfaPnh3 linne
unless it is suspicious of malignant . It is diagnosed by
27 \ And Ewing Sarcoma condition .

9 ml It has a high rate of malignant transformation


” 0 Retinoblastoma classically presents with
L Which often is first noticed when
leukocoria, a white
en a red reflex is not present pupillary
at a routine
newborn or well-child examination or in a flash photograph
of the child.
95.(A) The diagnosis is established by the characteristic
ophthalmologic findings
of a chalky, white-gray retinal mass with a soft,
friable consistency. Imaging
studies are not diagnostic, and biopsies are
contraindicated.
96.{E) The differential diagnosis of retinoblastoma
includes other causes of
leukocoria , including persistent hyperplastic primary
vitreous, Coats disease,
vitreous hemorrhage, cataract, endophthalmitis from Toxocora
canis, choroidal
coloboma, retinopathy of prematurity, and familial exudative vitreoretinopathy .
97.( 8 ) All other distractors are
associated with increased risk of testicular
cancer.
98.(C)
99.(B )
Sertoli-Leydig cell tumors and granulosa cell tumors produce hormones
at can cause
virilization, feminization, or precocious puberty , depending on
P efTal
stage and the balance between Sertoli cells ( estrogen production ) and
eydig ce||s ( ancjr 0
gen procjuctjon)
, ,ted^ to^cept for tumors of the central nervous system, radiation therapy
li;rn
* »s
th ,
ose tumors that are not amenable complete excision and are

.
to
refractory to chemotherapy
Wl.iq The
and may be
serum a-fetopr
°
- te

,
is elevate endodetm y(Ws

minimally elevateo teratom3S-


528
.
103 (0) The pure fetal histology subtype predicts a more fay0 |
rab e
norma,
and the small cell undifferentiated subtype is associated with
and predicts a worse outcome. Afp ,
.
104 (E) Hepatocellular carcinoma occurs mostly in adolescents an
associated with hepatitis B or C infection. It also occurs in the chron
hereditary tyrosinemia, galactosemia, glycogen storage disease, QI
C

deficiency, progressive familial intrahepatic cholestasis, and bilia7 %.


Alagille syndrome and aflatoxin B contamination of food are „5S
factors.
* ^"
0Cl
C r
--

.
105 (C) Hemangiomas, the most common benign tumors of infancy *
approximately 5-10% of term infants . °
' CCU‘ .
106.|A) Ulceration may lead to secondary infection.
.
107 (C) Medullary thyroid carcinoma is an uncommon disease i»n
childhOOdtiu;
almost always occurs in the context of an autosomal
dominant
endocrine tumor syndrome that arises secondary to activating
RET protooncogene: multiple endocrine neoplasia type 2a ( mutations rfnty
MEN2A) or type it
( MEN 2B).
.
108 ( B) The skeleton is involved in 80%, Skin in
Hepatosplenomegaly in 20%, and lungs in 10%.
50%, Lymph nodes in ft
.
.
109 {C) The risk organs are liver, spleen,
and the hematopoietic (bone
system . marrovd
.
110 (A) Late (fibrotic) complications
, whether hepatic or pulmonary
irreversible and require organ transplantation are
to be definitively treated.
111(C)
.
112 (A) Physical examination
often reveals hepatosplenomegaly (70 100
lymphadenopathy (20-50%), respiratory - ^
symptoms of CNS distress (40-90%), jaundice, an:
involvement (50%) that are not unlike
meningitis or acute those of asepw
demyelinating encephalomyelitis.
Nephrology
Questions
JnAHTAN ALOBAIDY ,
bloo d passes through the glomerular capillaries plasma is filtered
J the
l When glomerular capillary walls.
through the following P aSma molecules filt freely through the glomeruli?
Which of <he
A.
Peptides ' ^
Albumin
fl.
C. Globulins
-6
D. Interleukin
Myoglobin

Schw artz formula is the most widely used pediatric formula for
2 The "beds ide " ,
, and is based on the an empirical constant, patient height, and
GFR estimation
?
which of the following
A. Inulin
B. Cystatin c
C. Blood urea
D. Serum creatinine
E. Fructose polymer

is considered as significant hematuria


3. Which of the following urinalysis results
in school age children ?
A. > 10 RBCs/HPF
B. > 20 RBCs/HPF
C. > 30 RBCs/HPF
0- > 40 RBCs/HPF
E. > 50 RBCs/HPF

urin
^ Which of the following causes heme-positive red
Homogentisic acid
" • Myoglobin
* ' Methemoglobin
n
o porphyrin
E. Urates
finding in upper urinary tract he
5. What is the
characteristic maturja
in
\
examination? casts
A. Leukocytes
RBCs
B. Isomorphic
C. Bright red urine
in urine
D. Blood clots
mg/di
E . Proteinuria>100
distinguish IgA nephropathy from post
6. Which of the following
glomerulonephritis?
A. Gross hematuria
^
B. Proteinuria
C. Normal C3 level
D. RBC casts
E. Serum IgA level

7. A 13- year- old boy presents with progressive proteinuria ia, his brother or
isalsc
regular hemodialysis sessions, wearing glasses for his eyes problem and
suffering from hearing impairment.
ic for th tf
Which of the following ophthalmological findings is pathognomonic
disease?
A. Macular flecks
B. Recurrent corneal erosions
C. Anterior lenticonus
D. Optic nerve atrophy
E. Retinitis pigmentosa
2
8. A 7-year-old male presents
with sudden onset of gross hematur

and^
Blood pressure is 130/90 mmHg
low C3 level, and urinalysis
acute tonsillitis with penicillin
Which of the following is
shows RBC casts. He had history o
for 10 days.
^^
. Investigations results; blood
atrfientfof

the MOST likely diagnosis?


A. IgA nephropathy
B. Alport syndrome
C. Essential mixed cryoglobulinemia
St infectious glomerulonephritis
0 . P0 branoproliferative glomerulonephritis
E-

which
,
of he following
neurologic sequelae is seen in patients
with post
9- cal glomerulonephritis and severe prolonged hypertension ?
strep tococ
A. Stroke
B Seizures
r Brain edema
n intracranial bleeding
calcification
E Basal ganglions
following can cause low serum complement ( C 3, CH 50) ?
Which of the
A polyaretritis nodosa
B Goodpasture syndrom
C Hypersensitivity vasiculitis
p Henoch-Schonlein Purpura
E. Subacute bacterial endocarditis

three days history of hematuria and mild puffy face.


11 A 4 -year -old boy had
Blood pressure was 140/90 mmHg. He underwent surgery
for hydrocephalus
granular cast +, hyaline cast +,
one month ago. Urinalysis: RBC +++, protein +,
complement level.
blood urea 80 mg/dl, serum creatinine 0.9 mg/dl and low
Which of the following is the MOST likely organism causing this complication ?
A. Streptococcus viridans
B. Staphylococcus epidermidis
C. Staphylococcus aureus
D. Klebsiella pneumoniae
E. Pseudomonas aeruginosa
ed
A 15- year- old female presented with fever, fatigue, and generalized

,
examination you find malar discoid rash, investigation
W 0C 10* 10V,
a t>umin
Platelet 50 loV ’ .
positive
2.2mg/dl, total serum protein 4 / mg
antinuclea
dl .
and
r
s s ws ^° .
antibody est se
,
unnalys s revealed ++

Proteinuria .
Which of the following is the MOST likely class of this disease ?
A I
8 II
c - III
532
D. IV
E. V
, ,
Dir
g a known case of beta thalassemia and |hepatilis r
c
.
i

^. ^
!3 A 10-Yea 0|ood pre 5 SUre was 140/90 mmHg,
eJS Hih

ETiKS » 0 9 m5/dl'

A membranous nephropathy
glomerulonephritis
B. membranoproliferative
C. lupus nephritis
0. post streptococcal
glomerulonephritis
E. Immunoglobulin A nephropathy

Which of the following are the MOST common -


post renal transplant
14 .
bacterial infections ?
A. Urinary tract infections
B. Osteomyelitis
C. Peritonitis
0. Meningitis
E. Sinusitis

15. Which of the following is an option for treatment of acute cellular


rejection
post renal transplant ?
A. Plasmapheresis
B. Intravenous immunoglobulin
C. Thymoglobulin
D. Rituximab
E. Proteasome inhibitors

16. The natural history of most forms of crescentic glomerulonephritis ( CGN ]


rapid and relentless progression to end -stage renal failure.
Which of the following RARELY progresses to CGN?
A. Henoch-Schonlein Purpura
B. Microscopic polyangiitis
C. Immunoglobulin A
nephropathy
.
D Goodpasture disease
E. Systemic lupus erythematosus
the following microorgani
hich of isms ils
l7' ,syndrome ? blamed in C3USin
"A" 5trePtococcus pneumonia
Mycoplasma hominis
** ,
,
b n oly jc

0.
urea plasma urealyticum
C
Staphvl°coccus ePldermidis
° Klebsiella
E-
pneumoniae

. 2-year-old
toddler is admitted to hospital ,
HV
18 - with
local outbreak of E
Lstigations
is a known
showed evidence of acute renal failure
" h initial'«hargy
coll 0l57. 3 M and

and ' S b d
'°°
Lch of the following investigation
prolong prothrombin time
expecteT
results would be
ar
A
B Fragmented red blood cells
c Thrombocytosis
0. Right-shift of the white blood cells
E. Raised serum haptoglobins

19. what is the characteristic lab finding of pneumococci-induced hemolytic-


uremic syndrome?
A. Prolong prothrombin time
B. Macroscopic hematuria
.
C Thrombocytosis
D. Coombs positive hemolytic anemia
E. High-grade proteinuria

20. A 14-year- old girl with cystinuria disease on D-penicillamine treatment for
the last 6 months, presented with generalized edema, decreased urine output,
and blood pressure of 140/95 mmHg. Urinalysis; pus cell ++, RBC +, albumin+++,
blood urea 60 mg/dl, total serum protein 4.5mg/dl, serum albumin 2.2mg/dl,
fc um
' cholesterol 300mg/dl, and normal serum C3 and C4.
0fthe following
, the MOST likely diagnosis is
_B.
focal segmental glomerulonephritis
membarenoprolifrative glomerulonephritis
C . membranous glomerulopathy
minimal change nephrotic syndrome
rapidly progressive glomerulonephritis
bV bo* to a
° ^ ^ - « t ms
L * r 0\d abe
$s \n l e rrftW anK'
-
other presents
wth
b\ood pressure
^
-
. e! „/d\ ttha » U|
^ * .

* maternaise\ *
ote fc\ood
>
«
*“& r’'0 1
**
countV <&^^
fibrinogen.
diagnosis is
Of the following, the MOST likely
A. Wilms tumor
B. multicystic dysplastic kideny
C. renal vein thrombosis
D. polycystic kidney disease
E. neuroblastoma

22. You are reviewing a 15-year- old boy who had recently been diagnosed
^
autosomal dominant polycystic kidney disease ( ADPKD ) in the renal clinic. y0 u
proceed to examine his cardiovascular system .
What other features you expect to find on examination ?
A. Dilated cardiomyopathy
B. Mitral stenosis
C. Aortic stenosis
D. Renal bruit secondary to renal artery stenosis
E. Mitral valve prolapse

-
23. An 8 year-old boy presents with severe anemia and polyuria. Examination
revealed growth failure, retinal degeneration with blood urea of 66 mg/dl and
serum creatinine of 3 mg/dl.
Of the following, the MOST likely diagnosis is
A. infantile polycystic disease
B. autosomal recessive polycystic kidney disease
C. nephronophthisis
D. Bardet- biedl syndrome
-
E. Von hippel lindau syndrome

24. What is the MOST common vi


virus causing hemorrhagic cystitis ?
A. Para influenza
B. Adenovirus
C. Parvo virus
D. Enterovirus
E-
of the fo
pnioovirus
.11 h hun
„ , '
25. Which
gphthousulcers,dyslipip m a nd D
„ - *
- - ‘ ' °SdBpr< ?
druBS had a high
rate of
A
Tacrolimus
Corticosteroids
?
C
CyclosPorine
Mycophenolate mofetil
'
E.
c Sirolimus

Which of the
* Microalbuminuria test
rfithdiabetes mell tus? .
is the BEST method
following
to predict
proteinuria in patient

^B Dipstick testing
test
C Boiling
0. Spot urine
for protein/creatinine ratio
E. 24 hr urine for protein and creatinine excretion

r
^ iallneVH
'
B.
" "8
Dilute urine
0110 1 U s e' n e
^ proteinuria ?

C. Oliguria
D. Hematuria
E. Myoglobulinuria

28. Which of the following conditions can be associated with microalbuminuria ?


A . Obesity
B. Hyperlipidemia
C. Renal vein thrombosis
D- Cirrhosis
E. Leukemia
presented to
ear stero c dependent nephrotic syndrome
the eme^ '* reveals
ever and abdominal pain. Examination
ck ~
Um Wl
* ^^
ascites and abdominal tenderness; aspiration of peritoneal
fluid

fP
ritonea ( teukocyte counts 350 cells/pL
Of the
4
^* w
° ° * , the MOST likely microorganism
ng
rePtococcus viridans
isolated

536
from the ascitic fluid is
staphVlocOCCLJS ep
idermidis
B.
r Escherichia colipneumon a
o . Streptococcus
.
E. Enterococcus
syndrome had
following types of nephrotic the

.
30. Which of the
prognosis ? glomerulosclerosis
£ ”glomerulopathy
C.
Me branoproliferative
C3
glomerulonephnt s

D. Membranous nephropathy
E. Minimal change disease
following
_ predispose to the development of venous
31. Which of the
syndrome .
thromboembolism in patients with nephrotic
A. Decreased platelet number
B. Loss of antithrombin III
C. Hemodilution
D. Loss of fibrinogen
E. Reduced metabolism of vitamin K

32. A 10- year - old child with steroid dependent nephrotic syndrome and
.
frequent relapses, his height below third centile The parents are concerned
about his short stature.
Which of the following is the MOST appropriate strategy to improve linear
growth in their child?
A. Reduce the total dose of steroid
B. Use single dose regimen
C. Start growth hormone therapy
D. Follow steroid- sparing strategies
E . Combine calcium with
steroid therapy
33. 1
A. Neutropenia ^ ^ ' 6*
1 S 3 preventa e Slc e effect of cyclophosphamide ?
^ ^
B. Alopecia
C. Hemorrhagic cystitis
D' Premature gonadal failure
E. Increased risk of future
malignancy
r
cyclophosphamide therapy j„ children

,
11 00
noring

A
8
2 ’,000 / mm 1
3 oOO/mm
'
- ‘ --
if the
fir?
"
,. COur>tm„

"‘ alls below ^


COu
and

4 oOO/mm’
5,0OO/mm
E 6,000/mm^

, which ofinthe
5
following is the MOST
minimal change disease ?
appropriate lnitial
I
treatment
proteinuria to reduce
A protein restriction in diet
0 Lipid lowering agent
C. Angiotensin converting enzyme inhibitor
D. Loop diuretics
E. Prednisolone

36. Which of the following drug is recommended as initial


therapy for children
with steroid-resistant nephrotic syndrome ?
A. Tacrolimus
B. Mycophenolate mofetil
C. Levamisole
0. Chlorambucil
E. Azathioprine

37. A 17-year-old girl


with nephrotic syndrome is noted to have symmetrical
barked loss of adipose tissue, first manifests in the face and then gradually
progresses to the upper
extremities, thorax and upper abdomen. Investigations
eal low complement

^^ B.
level.
type of nephrotic syndrome
is associated with these manifestations?
Membranoproliterative glomerulonephritis type II
Focal rsegmental glomerulonephritis
Minimal change disease
Di Renal vein thrombosis
E.
embranous glomerulonephritis
38.
Which can differentiate classic distal renal
Hu ar acid
| following urine findings
sis (type I) from proximal renal tubular acidosis ( type ) -
°
A' phosphate13
B Aminoaciduriaa
HyPerca c ur
Q
Di Glycosuria
*' '
E. Uricosuria
mpnta||y
handicapped boy presents by his Pare to
" poor vision. Examination shows SeVf
Ye
39 - A 4 - fo ld vere riCk ets and


f
dime aS Inve stiga tions reve al ABG

-
.
. ,£
outpatient ’
and hypoton ia : pH
fj „ C0, 6 mEq/l, «
CTUS n« a
* rh«r «* "
^
*
diagnosis
, the MOST likely
of the following
A. cystinosis
B. Lowe syndrome
C. chronic renal failure
D. vitamin D resistant rickets
E. Dent s disease

follow ing is the BES T way to confirm the diagnosis of


40. Which of the
cystinosis?
A . Detection of cystine crystals in
the cornea
B. Measurement of leukocyte cysteine
content
C. Normal anion gap (hyperchloremic ) metabolic
acidosis
D. Measurementof amino acids in urine
E . Cysteine urine concertation

(type
41. Which of the following findings can differentiate renal tubular acidosis
IV ) from other types?
A. Hyperchloremia
B. Hyperkalemia
C. Nephrocalcinosis
0. Normal anion
ani gap metabolic
acidosis
E. Normal glomerular filtration rate

42. Which of the foil


owing types of renal tubular
acidosis (RTA) typically cause
urine pH > 6?
A. Distal
B. Proximal
oiTibined proximal
and distal

Cn ^HVPerIcalemic
*\
type

Cyittnosis
£
Qf the following types of renal tubular ac dosis (
& ^hichca
Iciuria ? ' RTA ) is associated

^"fi!
K
8
(tVI» RTA
<
proxima| tVPell RTA
»
>
Combined proximal and distal ( type III) RTA
C
nyperkalemic (type IV ) RTA
E-
Nephropathic cystinosis

yvhich of the following causes false low bicarbonate level in interpretation


*f*arterial blood gas analysis ?
° A Traumatic
blood draws
of blood sampling
B Large volumes
C. Addison disease
D Chronic diarrhea
E. Diabetic ketoacidosis

45. A 6- month-old boy presents with polyuria . Examination shows irritability,


tachycardia, tachypnea , and moderate dehydration. Investigations reveal blood
urea 66 mg/dl, serum creatinine 0.5 mg/ dl, A 8G : PH 7.36, HC03 26mEq /l, Na +*
ISSmg/dL, CL 105 mg/dL, K 3.8mg/dl_, lactate 2.9 mmol/ L, serum osmolality 293
+

mOsm/kg, and urine osmolality 280 mOsm/kg .


Ofthe following, the MOST likely diagnosis is
A - Bartter syndrome
B. acute kidney injury
c diabetes insipidus
§* distal renal tubular acidosis
E.
meduJjary sponge kidney
< 6 . A flrst c
usin parents brought their 13-month -old toddler with hyperthermia
and °
^sSive polyuria . Examination reveals failure to thrive® a hycardia
^inlS
"
/H
63, dncl

127me/dL SerUm
norrnal blood
creatinine 0.5
pressure
mg /dL,
.
ABG
Investigations rev
pH 7.55, HC03 32
° .^° ,2
osmolality
Cl 80mg/dl, K * 2.8mg/dL, and lactate 2mmol/L, serum
-
osmolality of 280 mOsm/kg; abdominal
290 mOsm/kg, with a urine
shows renal pyramids calcifications
diagnosis is
Of the following, the MOST likely
A. Bartter syndrome
B. Gitelman Syndrome
C . diabetes insipidus
D. EASTsyndrome
E. medullary sponge kidney

47. Which of the following tubulopathy is characterized by low- 1


weight proteinuria, hypercalciuria, glycosuria, aminoaciduria *
phosphaturia ?
A. Gordon syndrome
B. Cystinuria
C. Liddle syndrome
D. Dent disease
E. Bartter syndrome type V

48. A mother with positive history of polyhydramnios in her pregnancy consults


you about her premature baby boy with massive polyuria, on careful
examination; you find he is suffering from sensory neural deafness.
+2
Investigations reveal arterial pH 7.55, HC03 32 mEq/l, Na 127 mg/dl, Cl
80 mg / dl , K * 2.8 mg/dl.
Of the following, the MOST likely diagnosis is
A. Liddle syndrome
B. Gordon syndrome
C. medullary sponge kidney
D. Bartter syndrome type IV
E. chronic renal failure

49. Which of the following syndromes is characterized by hyperald ste


hypertension, hypokalemia, and alkalosis °
?
A . Liddle
B. Bartter
C. Gordons
D. Gitelman
E. milk- alkali

r
have a « * "' ,, % „ u*ho„ a n<
// from tho ie or hn e
,
.
SO 3
which has
.
been
for
ittlAg/hr A
last12hr
J * , bfY° e >tfJs
Y u days in the ne0nat Car un ' as
before° increased >5 tirhes n01 e
,
Jf 8e of renal /
**
„* " " ^
" < ^ Ut <0S
-

Acute *
rfrte kidneyinJu
* *
* Acuteki Vinu2! gee*23
. A

c
0. Chronic
t
^renal
Chrome
fai
failure
renal
8e1

^
Sta

the following tests is MOST useful in determining


L0f
5]. Whichfrom acute tubular necrosis?
prerenal

,otemiaSerum urea level


3
A.
B
Hemoglobin concentration
. osmolality
C Plasma
D. Urinary
urea
E. Urinary sodium
the emergency department after a crush
boy presents to
52 A 14 - year-old reveals no underlying fractures. One hour
Radiography
his right leg.
n ury to pain, swelling, and paresthesia of the
2 er the injury, he
injured leg
develops
Investigations
worsening
reveal, high lactate, serum creatinine
kinase, and

pH, 7.29; potassium , 5.8 mmol/l; and low glomerular


myoglobin level ; arterial
filtration rate. initial management is to
Of the following, the MOST appropriate
A. estimate urine output
B. start high rate intravenous
fluid
C. give intravenous mannitol 20
%
D. give loading dose furosemide
E. arrange for urgent dialysis
inofollowing
as dark color urine .
5j to emergency unit and hypotension
^ k y presents
'n8estion$ Qf
ln)/est 8ations
' ^ ^
°^
ean' £nomination
d urea
^°° 80
Urirtilysis sho r dd, brownish granular

shows
mg

iurt) an(j ract or>al excretion of sodium


/dl

,
pallor, jaundice 1.3 mg/ dl, and
, serum creatinine
casts
and high urinary
, high both
, protein nilosmolality.
kidney injury
?
urinary

^
^ ,'
ich of 6 0 lowing is the MOST
^
likely cause of his acute

542
A. Hypovolemia
necrosis
B. Acute tubular nephritis
C. Acute interstitial
D. Glomerulonephritis
E . Tubular obstruction

54. A 5 - yearr nld boy

days followm
pr« tc with fever,
entS
'
g ingestion of
ibuprofen
? . .
maculopapular rash,
Invest g ations
and m.
reveal b|
urina|vsls show white 00(j
arthralgia 3
urea 80 mg / dl , serum creatinine
casts.
^ blOOQ
osinophils
_ , and cellular
cells , e _
the MOST likely cause of his acute kidney injury?
following is
Which of the
A. Hypovolemia
B. Acute tubular necrosis
Acute interstitial
nephritis
C.
D. Glomerulonephritis
E . Tubular obstruction

the MOST likely cause of raised phosphate level in


55. Which of the following is
?
patients .with chronic kidney disease
A. Decreased renal excretion
B. Increase gut absorption
C. Hypervitaminosis D
D. Secondry hyperparathyroidism
D
E. Decreased 25 -alpha hydroxylation of vitamin

56. Which of the following is the leading cause of death in childhood chronic
renal failure ?
A. Central nervous disease
B. Cardiovascular disease
C. Respiratory disease
D. Hematological disease
E . Gastrointestinal disease

57. Which of the following is not restricted regarding the dietary managemenl
of chronic renal disease?
A. Protein
B. unsaturated Fat
C. Salt

543
ted Fat
5atura
0
i
fluid
the sufficient level of vitamin 0 «-
hvdro Vcalcifero, ,,
/
^
?
rcnal failure * n
Patients with
ch '’ 20
0 ng/ mL
'A '
j

2 30
ng/ f711-
0
240 ng/nrlL
£ 2 50 / -
ng rr1
» 60 ng/mL
*

-
E

,
. Which of the following is the BEST time for amh ,
• SST
h perte s
“ p
" “" “* •
ra »
JSSL,
^
Pressure

3 Over 24
hr
c Twice/ week
0. weekly
E Every two week

-
60. A 4 -vear old male with stage 4 chronic renal disease serum „
serum
«
phosphorus 7 mg /dl. You advised the patient to follow a
low nh, T* '
and to enhance gastrointestinal phosphate excretion by using phosphite
which of the following is the BEST time to give such
treatment ?
A. Before meals
B. After meals
C. With meals
D - Before sleep
E . Early morning

Which of the following is the BEST level of serum bicarbonate to be


gained14in chronic renal disease?
.
'
*i 16 mmEq/L
Eq/ L
~ 1® h Eq/ L
D ^
~ 20 Eq/L
t
'
"'
?? mEq i
/

544
hemodialysis with refractory
evaluating a child on anemja j
62. Vou are
using iron
is elevated
supplem
in the
entation
last year .
and erythropoietin. His parathyroid h
orrno >
ne I
*
'

following is the MOST common cause of his refractn0rVan mia


Which of the * ?
A. Occult blood loss
B. Chronic infection
C. Folate deficiency
D. Bone marrow fibrosis
E. Vitamin B12 deficiency
antihypertensive medications of choice imall
63. Which of the following are the
children with pediatric CKD?
A. ACE inhibitors
B. P-adrenergic blockers
C. Vasodilators
D. Loop diuretics
E. Calcium channel blockers

64. Which of the following is a modifiable risk factor for CKD progression ?
A. Older age
B . CKD severity
C. Onset of puberty
D. Glomerular etiology of renal disease
E. Persistent nephrotic range proteinuria

65. Which of the following substances is responsible for osmotic gradient


created in the dialysis fluid used in peritoneal dialysis ?
A . Sodium
B. Potassium
C. Protein
D. Dextrose
E. Calcium

66. Parents are concerned about immunization of their son who recently
underwent renal transplant .
Which of the following vaccines should be avoided to this child as well as his
family members ?
A. Varicella- Zoster
545
inI13
Jed influenza
titiS A
c* '
H*P3
MP,
p' ^eas
|
eS
£ ^
j

0f the following immunosuppressant dm


transplant (NODAT)?
.
arL 85
* can
caus
0. ^ new-onset
A
9
Corticosteroids
cvclosporine

^ ycophenolate mofetil
C

E "
Azathioprine
Chapter 22
Nephrology
Answers
QAHTAN ALOBAIDY
( e .g„ electrolytes, glucose, ph0
1.(A ) Small plasma molecules filter freely
urea, creatinine, peptides, low-molecular weight proteins), whereas 3rger
Sph
,
molecules are retained in the circulation ( such as albumin and globulins).
2.(D) Because true measurement of the GFR is expensive and time
the GFR is commonly estimated (eGFR ) by the clearance of endogenouj
consume
creatinine.
The "bedside'" Schwartz formula is the most widely used pediatric formula and
is based on the serum creatinine (Scr ), patient height, and an empirical
constant:
eGFR = 0.413 * height ( cm)/Scr (mg/dL ).
3.(E) Hematuria, defined as the persistent presence of more than 5 red blood
cells (RBC$)/high power field (HPF) in uncentrifuged urine, occurs in 4-6% of
urine samples from school-age children. The presence of 10-50 RBCs/pL may
suggest underlying pathology, but significant hematuria is generally considered
as > 50 RBCs/HPF.
4.(B) Most important causes of heme positive red urine are hemoglobin and
are
myoglobin while the metabolites that cause heme negative red urine
homogentisic acid, melanin, methemoglobin, porphyrin, tyrosinosis and urates
.
,
5.(E) Hematuria from within the glomerulus is often associated with brown
dipstick,
cola- or tea-colored, or burgundy urine, proteinuria > 100 mg/dL via
urinary
urinary microscopic findings of RBC casts, and deformed
system
( particularly acanthocytes ) . Hematuria originating within the tubular
may be associated with the presence of leukocytes or renal tubular casts
. Lower
tuna that
urinary tract sources of hematuria may be associated with gross hema
occurrin g at the end
is bright red or pink, terminal hematuria gross hematu
( ria
mimm
of the urine stream ), blood clots, normal urinary RBC morphology, and
proteinuria on dipstick ( <100 mg/dL).
6.(C) IgA nephropathy is the most common chronic glomerular
disease
nephrop athy help to distinguish this
children. Normal serum levels of C 3 in IgA
fr0m
postinfectious glomerulonephritis. Serurri ,
hav no.

- -
*
d<5

£V*
1-
d . pathognomonic. ' *
'
a
The P «e„Ce:
' *
critic*!
interior
X‘ post-streptococcal glomerulonephritis (APSGN ) « , ,
,
>' *IAC t e nephritic syndrome characterized by he sudd * ns
( the ° derna, hypertension
, and renal dysfunction
i» « °
t 01

1,
^
of gross hematuria in
lomero ar causes
'
^ L '
group
^» * pertension
A
in
p - hemolytic streptococcal
children a n d h a 3 or
is seen in 60% of patients
infection.
*neurologic
associated with
9.|t
rtens
WP en
are °«
®
1
STS'
can lead to intracranial bleeding. ' severe prolonged
*
sequelae

hypertension diseases causes of In ,


Systemic
talU/ Lupus nephritis (focal 75%)
»«
*
,
°" ^
c »Pfement la
/r CHXr
-

/
/
/ Subacute
Shunt ^
nephritis ( 90%")* * tx%)

Essential mixed cryoglobulinemia (85%)


/ Visceral abscess
renal diseases causes of low Serum complement (C3 CH 50) *

While
Acute post infectious glomerulonephritis
/
(>90%)
( - %)
J Membranoproliferative glomerulonephritis 50 80
( B) Glomerulonephritis with hematuria is a irecognized complication of
IL
various chronic infections. Classic examples that
are uncommonly encountered
endocarditis caused by viridans group streptococcii and
today include bacterial
with Staphylococcus
other organisms, and ventriculoatrial shunts infected
epidermidis.
commonly Present with nephrotic
(E) Patients with class V lupus nephritis
^
< Membranoproliferative glomerulonephritis
13 B )
jn 0 jder children
^ngiocapillary glomerulonephritis, most common V . and secondary
?ryoun
^
8 adults. MPGN can be classified into Prim commonly
rrns o are most ,
of glomerular disease. Secondary forms hepatitis 0 and C
5^ ?
Wilis.
subacute and chronic infection
,
shunts, especia
y

Ver>tri subacute bacterial endocarditis


d t

lcul atrial
° shunts (shunt nephritis)
r jtO
.° and °un arv t act infect 00 are the most
; ' corrirnon «s.
. ^„

14.1A) pne*monia . Onn tract infections can progress
transplant ba ri3 infections wit P
|

-rSTl
of acute reject on.
uroseps 5 and
confused
^ Trimeth0
' ,
^ jnfection prophylaxis

^
used for urin as W(, .
hv aKis for 3-6 mo after transplant
pnCUm0
hearaPde election can treated withthymog
.

^ nE/d:
K, 0

%.£ 1 ^
17 (A) HUS
seConf.aa7 istr *****
e (Streptococcus
Neuraminidase P pneumoniae) *******
Human immunodeficiency
y Influenza
/ Human herpes virus 6
S Parvovirus B19
S Malaria
18.(B) Red blood cell ( RBC) destruction may occur in hemolytic anemias because
of mechanical injury as the cells traverse a damaged vascular bed. Damage may
be microvascular when RBCs are sheared by fibrin in the capillaries during
intravascular coagulation or when renovascular disease accompanies the
hemolytic-uremic syndrome ( HUS).
19.(D) Thrombocytopenia is an invariable finding in the acute phase, with
platelet counts usually 20,000-100,000/mm3 . Partial thromboplastin and
prothrombin times are usually normal. The Coombs test is negative, with the
exception of pneumococci- induced HUS, where the Coombs test is usually
positive. Leukocytosis is often present and significant. Urinalysis typically shows
microscopic hematuria and low-grade proteinuria .
20.(C) Nephrotic syndrome has also developed during therapy with numerous
drugs and chemicals. The histologic picture
can resemble membranous
glomerulopathy (penicillamine, captopril, gold, nonsteroidal antiinflammawj
drugs, mercury compounds),
minimal change nephrotic syndrom (probene^
1

ethosuximide, methimazole, lithium ), or proliferative glomeruloneph


(procainamide, chlorpropamide ).
, phenytoin, trimethadione, paramethadione
.
21 (C) The development
of renal vein thrombosis is classically heralded by
en onset of gross
hematuria and unilateral or bilateral flank mas *
owever, patients can also present
with any combination of micros **
^^
b . .
flank Pain' hypertension, or a
ocytopen aorol guria.
prolapse is seen in
microangiooatt .
8 P thlc h
ernolytic
anemia
valve approxim ately
) M U% of
jj.(f artery aneurysms and aortic valve insuffkU
mCY arechildren ;- aortic
noted in and
, .
0u~ .. ociated external findings include
-
retinal de»Pno,a,
affected
*
Z3.I0 e ) cerebellar ataxia ( joubert syndrome ), and °
hepatic f
(Senior -
token
sf \
*° fibrosis (Boiehts

^
*'
Hemorrb agiP cystitis is defined as the
<B r urinary tract symptoms as dysuria presence ol
. sustained K
frequency al
^ ^ ompeten.
He

tunoc
tis in
agic cystitis can occur in response to viruses (
nd influenza
immunos
A). The polyoma
hosts is associated with the
uppresse d patients .
adenovirus
BK virus present types 11 and
development of drug
lTte
8
*'^
„n
See"
°uced
?[ E) Mammalian
Target of Rapamycin (mTOR )
Inhibitors (sirolimus more
LlmonlV than everolimus) are used primarily as
adjunctive
suppression in combination with MMF in order to avoid tacrolimus immune-
toxicity or
lth tacrolimus and MMF to spare steroids.
*
26.(A) Microalbuminuria is defined as the presence of albumin in the
urine
above the normal level but below the detectable range of conventional
urine
dipstick methods. Microalbuminuria in children has been found to be
associated
with obesity and to predict, with reasonable specificity, the development
of
diabetic nephropathy in type 1 diabetes mellitus.
Z7.(B) Total protein concentration in urine can be estimated with chemically
impregnated plastic strips that contain a pH- sensitive colorimetric indicator
that
changes color when negatively charged proteins, such as albumin, bind
to it.
False-negative test results can occur in patients with a low urine pH (<4.5),
dilute urine or a large volume of urine output, or in disease states in which the
predominant urinary protein is not albumin.
* (A)
Spontaneous bacterial peritonitis presents with fever, abdominal pa -
3nd
Per,toneal signs.
Although Pneumococcus is the most frequent
Juntos , Gram-negative
bacteria also are associated with a vg
lnfmber of ses.
^ Children with nephrotic syndrome and fever or 0 ^
Cn " USt be evaluated aggressively, with appropriate

n and
Peritoneal
ocyte
counts > 250 cells/pL are bacterial
Peril nitis.
°
ccn
==S=SS
. 5>
progressive, ultimately
most patients.
,
involving all glomerul w th end-stage rena|

*
disI
31 (B ) Nephrotic syndrome
is a hypercoagulable state resulting fron mulUpt,
factors vascular stasis
from hemoconcentration and intravascular ^
depletion, increased platelet number and aggregability, and
an increase in hepatic
changed
coagulation factor levels. There is production *
fibrinogen along with urinary losses
of antithrombotic factors
suo, 45 “
antithrombin III and protein S.
32.(D) Growth may be affected in children who require long-term corticostero .
therapy. Steroid- sparing strategies may improve linear growth in children who
require prolonged courses of steroids.
33.(C) The potential side effects of the drug (neutropenia, dissemin ec
varicella, hemorrhagic cystitis, alopecia, sterility, increased risk of future *
malignancy ) should be carefully reviewed with the family before initiating
treatment. Attention to adequate hydration can attenuate the risk of
hemorrhagic cystitis.
34.(D)
35.(E)
3 .(A) Calcineurin inhibitors (cyclosporine or tacrolimus) are recommended as
initial therapy for children with steroid-resistant nephrotic syndrome.
37.(A) In many patients with type II MPGN, C3 nephritic factor (antK3
convertase antibody ) is present . This factor activates the alternative
complement pathway. In unusual cases, patients with type |i MPGN
demonstrate an associated systemic disease called partial lipodystrophy, where
there is diffuse loss of adipose tissue and decreased complement in th*
presence of C3 nephritic factor.
38.(C) Renal tubular acidosis ( RTA ) iis a
disease state characterized by a norm
3

anion gap (hyperchloremic)


metabolic acidosis in the setting of normal ornea
is usually present in distalR
normal glomerular filtration rate. Hypercalciuria
an can lead to nephrocalcinosis
or nephrolithiasis. Chronic metabolic a
also impa rs urinary citrate
'
calcium deposition in the
excretion. Hypocitraturia further increases the ns *
tubules.
^
disorder
sZorne
rhlSV * rom ( oculocerebrorenal syndrome
* ^bV C ngenital
° ««
**. — of Lowe ) is a rare XA-Nn
, and Fa
tal retardation
••

-c#
- —-
——
*

r ; ;; - ~*s
^ srproximai rta

«5, With distal RTA should be monitored for the development


* *
43
hVPerCalciuria . Those with symptomatic hypercalciuria (recurrent episodes
turia ), nephrocalcinosis, or nephrolithiasis can require
of
of
gross .hema

tQ decrease
urine calcium excretion .
thiazide

^
!b? d in

Traumatic blood draws ( such as heel stick specimens), small volumes of
adult-size specimen collection tubes, or a prolonged specimen

^
temperature can lead to falsely low bicarbonate levels,
sp0rt time at room
in association with
an elevated serum potassium value.
4S.(C) The
diagnosis is suggested in a male infant with polyuria, hypernatremia.
dilute urine. Simultaneous serum and urine osmolality measurements
'
and
should be obtained. If the serum osmolality value is 290 mOsm/kg or higher
with a simultaneous urine osmolality value of < 290 mOsm/kg, a formal water
deprivation test is not necessary. Because the differential diagnosis includes
causes of central diabetes insipidus, the inability to respond to ADH ( and thus
the presence of NDI ) should then be confirmed by the administration of
vasopressin ( 10- 20 pg intranasally ) followed by serial urine and serum
osmolality measurements hourly for 4 hr.
46.(A) Consanguinity suggests the presence of an autosomal recessive disorder.
Older children can have a history of recurrent episodes of polyuria with
hydration, failure to thrive, nonspecific fatigue, dizziness, and chronic
^
constipation. Older children may also present with muscle cramps ana
usually
Weakness secondary to chronic hypokalemia. The blood pressure is
"ormal- Serum chemistry reveals the classic biochemical abnormal jes
VWkalemk hypochloremic metabolic alkalosis. The renal function is
formal,
« (D) Although some patients develop nephrocalcinosis nephrolithiasis,
with Dent
"" fftsive renal failure, and hypophosphatemic rickets, pa
an# typically do not have proximal renal tubular act extrarenal
"' stations.
s, fr %
results
°
orrtta,,
"
These patients respond to the potassium %
owse
pVninrand levels.
ri aldosterone
« lt amterene and
,
amilorid e ) that inhibit this sodium channel.
KD G0 Stagin8 0f aCUte kidneV injUrV(AK ) -
, ^
as an abrupt (1 to 7
days) and sustaine d ( more than 24 * *
on U
hours) dec
defined *
5l!(ErTh feUn seSi vity and specificity of urine sodium of 20 mEq/L in
<

differentiating prerenalazotemia from acute tubular necrosis are 90 and % 82%,


respectively.
) case of acute lower limb compartment syndrome; his paraesthesia
52 .( B A
kinase, acidosis, and myoglobinaemia
elevated blood lactate, elevated creatine
glomerular filtration rate and
support this diagnosis . Reduced estimated
that acute kidney injury due to
elevated serum creatinine and urea level suggest
rhabdomyolysis.
53.( B)
blood cell casts with low-
54.(C) The presence of white blood cells and white
grade hematuria and proteinuria suggests tubule interstitial
- disease. Urinary
tubule-
eosinophils may be present in some children with drug induced
-

interstitial nephritis.
55.(A)
a 30-fold
56.(B) Childhood-onset ESRD still carries significant morbidity and
increased mortality rate as compared with healthy peers, with cardiovascular
and infectious diseases as the leading causes of death.
57.( A) Dietary protein restriction is not suggested for children with CKD because
,t
of the concern about adverse effects on growth and development; in fact
receiving
recommended protein intake is often 100% (or more for those
dialysis ) of the dietary reference intake for ideal weight for children.
M .( S )
59.(B) Ambulatory blood pressure monitoring ( ABPM ) over 24 hr, the t
standard of blood pressure evaluation, is recommended in patients with
re
disease to diagnose and treat hypertension, especially masked hypertension-
60.( C) CKD patients of all ages should typically follow a low -phosphorus
.I
with the goai of maintaining age -appropriate
should be provided with a low -phosphorus
serum phosphorus vaiues /
60 "
formula ( Similac PM
553
(given with meals) are USed
.M binders

Wper1
A 3 nd
eItcretlhosphatemia.
at present are recommended
to
to be
enhance
the onset 'of
started a .1
G Phospha e

tabolic acidosis develops because of a decrease


*the taS?tablets (650 mg *
u‘ w
= 8
» tq s» Citrat" "Cid eXCre,i0 by
*"mEq of base) dfl "? /
L) "
d to ^
may be S diU
° "
^jfincv .
tf T
bicarbonate level 22 mEq/L.
’ ffatients who appear to be resistant to ESA should h»
maintain
,
the

, or CJroni
^ ^
stS
l0SS 3
. occult bl00edncy bone c ^
infecti or
' °" inflammato
or folate defic
hyperparathyroidism -
marrow fibrosis relate ? 1 ?" to

63 (A) ACE
inhibitors (e.g., enalapril or lisinopril ) and angiotensin
II receptor
( ARB ; e.g., losartan) are the antihypertensive
(totkers medications of choice in
all children with pediatric CKD , irrespective of the level of proteinuric
[ renal
disease , because of their potential ability to slow CKD and their isuperiority in
controlling blood pressure as noted in various observational and research
studies.
64,( E ) Non modifiable risk factors associated with more rapid CKD progression
include older age, glomerular etiology of renal disease, CKD severity, and onset
of puberty. In terms of potential modifiable risk factors (in addition to an
elevated blood pressure ), persistent nephrotic range proteinuria, anemia, and
(falfpidemia, as well as no ACE
/ARB use, were important predictors of CKD
progression.
65 (D) Peritoneal dialysis utilizes the patient s peritoneal membrane to
'
transport fluid and solutes. Excess body water is removed by an osmotic
gradient created by
the relatively high dextrose concentration in the dialysis
fluid; wastes are the
removed by diffusion from the peritoneal capillaries into
dia|ysis fluid.

?- 11 is recommended that all immunizations be current Pr r


'> ° “
D

^? R ation . All live vaccines ( measles-mumps


Prior to transplantation, and antibody titers should b
e because
these vaccines should not be given t0
.
-rubella and «rice a
sh
°
ojtien

•,‘tieJtteMeaSl , m aRe inhaled

P
li «-
nJ ,' ' eS 'niunips'rubel a may be given as
Uated v rus) influenza vaccine should not be
3 rri
early as
g
-ven sp|ant

effect SUch
^ members, or healthcare providers.
acro imus also appears to cause less dyslip * erT1iaa, though other
' tremor
side
, seizure,
as new-onset diabetes after transplant (NO f
alopecia, and sleep disturbance seem to be more c
with tacrolimus than cyclosporine. °mmon lni Pati
** tr,

555
f

ter 23
Urologic Disorders
Questions
MAL QBA IDY
«030 ®
Of a 10-year- old boy with a single kidney ask you if
The Parents tact sports. he can
l
3.te in
con
fticiP arguments is against such participation?
P3
the following
f
A infection
0 Renal injury
c Hypertension
injury
p Vascular
nephropathy
E Reflux

2 vvhat is the
mode of inheritance in multicystic dysplastic kidney (MCDK)?
A. Autosomal recessive
B. Autosomal dominant
C. x-linked recessive
D. X-linked dominant
E. Not inherited

3. Which of the following renal cystic disorders needs surgical removal?


A. Multicystic kidney
B. Benign multilocular cyst
C. Simple cyst
Medullary sponge kidney
E. Calyceal diverticulum

4. Which of the following systems must be checked thoroughly in a newbo


umbilical artery?
*B Centra
.
Renal

l nervous
0 !" *
» »»»
I

gastrointestinal
Eodocrine
556
-
5s ln
ear
the delivery room, the
in 1
anomalies, imperforate
doctor
anus,
consults you about a baby bom
and scoliosis. Your advice is to
with
exC|
.
l "
%
should be done in this newborn?
Which of the following investigations
A . Renal function test
B. Renal sonogram
C. General urine exam
D. Abdominal CT scan
E. Intravenous pyelography

microorganism causing urinary tract


6. What is the MOST common infections?
A. Enterococcus
.
B Pseudomonas
C. Group B streptococcus
.
D Escherichia coli
E. Staphylococcus saprophyticus

7. Which of the following is a second line treatment of oxalate renal stones ?


A. Ferrous sulphate
B. Thiazide diuretics
.
C Lithium
.
D Pyridoxine
E. Allopurinol

8. Which of the following may be the only manifestation of pyelonephritis ?


A. Flank pain
B. Fever
C. Malaise
D. Rigor
E. Vomiting

9. A 7-year -old boy presents with renal mass,


he had complicated pyelonephfit
,V
secondary to renal calculi. Renal histopathology
consult a urological surgeon who
shows foamy histiocytes °°
recommends total nephrectomy.
Of the following, the MOST likely diagnosis
is
A. neuroblastoma
B. Wilms tumor
C. acute lobar nephronia

557
lobar nephritis
0- acute nulomatous pyelonephritis
£
xanthogra
following stones is treated with 0-
of the Penicillamine?
- Whi ^ oxalate
calcium
B indinavir
c CY $tine
0 struvite
E. Uric
ac
'^
nitrites test is of little value in the diag„
11 rhe
sed by which of the
following microorganism ? , 0f uri arV
" infections
* A. E coll
B Proteus
C . Klebsiella
D . Enterococcus
E. Pseudomonas

12.Which of the following is a recognized cause of sterile pyuria ?


A. Renal transplant
B. Ulcerative colitis
C. Kawasaki disease
D. Refrigerated urine
E. Fungal infection

3. Which of the following antibiotics does not achieve significant renal tissue
eyel m the
treatment of pyelonephritis ?
A Cefepime
B. Ciprofloxacin
C.
^phalexin
?E. Nltrofurantoin
'

Gentamicin
j *, ,

n
,
t
W ati,0n of antibiotic therapy for treatment of UTI, urine cultures are
Sat Ve within
A . 6h
B
12 h r
C
ta hr
D. 24 hr
£. 30 hr
of primary vesicoureteral reflux?
mode of inhe ritan ce
the
15. What is
A. Autosomal dominant
recessive
B . Aut oso mal
C. X -linked
dominant
D. X-linked
recessive
B. Multifactorial
treat men t optio n in a patient with symptom3tic
16. What is the primary ^
stone size 2.2 cm
?
A. Shock wave lithotripsy
B. Ureteroscopic removal
C. Percutaneous nephrolithotomy
D. Laparoscopic removal
E. Open surgery

17. Which of the following procedures has the lowest radiation risk for the
diagnosis of vesicoureteral reflux ?
A. Radionuclide cystogram
B. Contrast voiding cystourethrography
C Indirect cystography
D. Renal MRI
E. Renal scintigraphy

18. During routine renal second trimester; antenatal


hydronephrosis was detect *°n°gram in

as severe hydronephrosis?
A* >4 mm ^
B. >6 mm
C. >8 mm
D. >10 mm
E. >12 mm

19.Which of the following


newborn?
iis the MOST
likely cause of urinary ascites i
"
f

i'
^
ega
w 'urethral valvekidney
j dysplastic
C MultlCY peMc junction obstruction
n
C
Ure teF vesical junction obstruction
M tero
t- ^
is associated with patent
,
the following drainine urachus?
20
^
*
U thral obstruction kidney
MuIticystic
dysplastic
B- esical junction obstruction
C Ureterovpelvic junction obstruction
D Uretero
t
Megaureter
was concerned about fetal ultrasound which

k mother
( | pelvis
jhe r( B
. Your advice
findings.
is to repeat sonographic
P tT
s dilation
studY after birth to
natal
2? After first day
thefbllowing is the ideal time to perform postnatal ultrasound?
A
day
0. After third
c After first week
D. After 2 weeks
E. After 1month

, and
12 . A neonate had deficient abdominal muscles, undescended testes
urinary tract abnormalities. His parents worried about the prognosis of
their
baby .
Hie prognosis ultimately depends on severity of
A . pulmonary hypoplasia
B . cardiac abnormalities
t bowel malrotation
& renal dysplasia
t urological abnormalities
uropathy in
What is the MOST common cause of severe obstructive
“'Wren?
A.
8
Urethral atresia
C
Urethral hypoplasia
Posterior urethral valves
rrn
f

l ce of wkMIM r »u, , „ PosK


„ »
M the percentage
?
of
°
urethral valve disease
A. 10%
B. 30%
C. 50%
D. 70%
E. 90%

is associated with bladder diverticula


25. Which of the following syndromes ?
A. Marfan syndrome
B. Fragile X syndrome
C. Prader - willi syndrome
D. EhJers-Danlos syndrome
Eg Rett syndrome

26. A 6- year-old boy presents with suprapubic pain, fever, irritative voiding
symptoms, and an erythematous infraumbrlicat mass.
Which of the following is the MOST likely diagnosis ?
A. Patent urachus
B. Urachal cyst
C. Vesicourachal diverticulum
D. Umbilical-urachal sinus
E. Cloacal exstrophy

27. Which of the following surgical complications develops in augmented


cystoplasty in response to mucus accumulation?
A. Urinary tract infection
B. Metabolic acidosis
C. Spontaneous perforation
D. Bladder calculi
E. Malignant neoplasm

28. Which of the


foilowilng ls
agenesis in a patient
the M0ST appropriate initial test to exclude sa
with imperforate
^
A. Spinal anus?
ultrasound
T

acf 3 I X - raY
0' S
t MRI
.T scan
0 C
Mye ram
f ^
Jt is the
MOST common cause of daytime inctmti,
J9 bladder fience?
A- tractive
postponement
voiding
0 -sphincter discoordination
C. detrusor
15
D. CV51
£ overflow
''
incontinence

exhibit a staccato stream, day


A 6- year - old
boy
and night wetting
and encopresis , recurrent
yT)S constipation
(
, .
the following, the MOST likely diagnosis is
^ A. diurnal urge syndrome
B. nonneurogenic neurogenic bladder
C. posterior urethral valves
D. epispadias
E. ectopic ureter

31 Medical student ask you about the calculation of the


bladder capacity in
milliliters in a 4- year-old boy.
Which of the following equation can be used for this
purpose?
A . age + 1 (in years) X 30
B age + 2 ( in years ) X 30
.

C , a &e + 3 (in years


)X 30
D. age + 4 ( in years
) X 30
E. aSe + 5 ( in
years ) X 30

il Th idea l
3 tar *
a 8e at which active treatment for nocturnal enuresis can be
*
ed is
A-
*
a . 5 Years
Years
c. 6 years
D. 7
E 8
Vears
Vears
' Ledication
33. Desmopressin acet
reduces urine
children and
A. tablet
is
pro
pre
duc
fera
ate is a
tion
ble to be used as ^^
overnight. This
is FDA °ne
A-appr ed%i
approv

B. IM injection
C. IV injection
D. nasal spray
E. skin patch

following types of renal stones are radiolucent 0n Plain


34. Whic h of the
abdominal radiograph ?
A. Calcium oxalate
B. Calcium phosphate
C. Cystine
D. Struvite
E. Uric acid

of priapism in children ?
35 . What is the MOST common cause
A. Sickle cell disease
B. Sildenafil ingestion
C. Leukemia
D. Perineal trauma
E. Idiopathic

36. Which of the following syndromes causes micropenis due to


hypergonadotropic hypogonadism ?
A. Robinow syndrome
B. Kallmann syndrome
.
C Prader - Willi syndrome
D. Bardet-Biedl syndrome
E. Lawrence-Moon-Bardet - Biedl syndrome

37. What is the maximum age the testes can be descended spontaneously in 3
baby with cryptorchidism ?
A. 2 months
B. 4 months
C. 6 months
D. 9 months

63
nths
E 12
-old boy with cryptorchidjsm
a 4< * * Resents with
.
)
iln
6 uinal Pain
pelliW- following is the OST iil d a«n /s and
HfiAc.hofSeminoma
the "'’ '
(
0j

B. femoral hernia
torsion
C Testicular Malignancy
D Germ
fell
l Varicocele
A> -vear-old girl presents with i
* °shows na. Urine dipstick

£renal•
eft
Proteus infect
pelvis.
"
,^^^
mis the MOST likely comp0sitionn of this renal stone
A Calcium oxalate °
+ and protein

?
' ++,
demonstrates a stag-horn calculus iurine
in the

B. Xanthine
C. Cystine
0- Struvite
C Urate

10. Which of the following is the only complication


cystinuria?
A. Renal tubular acidosis
B. Chronic renal failure
C. Renal calculi
D. Nephrocalcinosis
E. Recurrent
UTI
»1.Which of . •
«'thscrotal the following the MOST common organs
A
pain, erythema
, an
Escherichia coh
n a 9'Vear'0
'^^
8
Chlamydia
C.
0. Conococcus
E. Enterovirus
Adenovirus

564
following can differentiate testicuiar
42. Which of the torsion fr
incarcerated hernia ? °rh w
A. Tender testis
B. Swollen scrotum
C. Absent cremasteric reflex
D. Absent swelling in the inguinal area
E. Associated nausea and vomiting

43. Which of the following anatomical sites of renal stones is


i asympto
A. Ureteropelvicjunction ^
atic ?
B. Mid-ureter
C. Ureterovesical junction
D. Urinary bladder
E. Urethra

44. What is the MOST appropriate imaging


modality iin suspected renal
A. KUB colic?
B. U/S
C. spiral CT scan
D. MRI
E. IV pyelography
ter 23
Urologic Disorders
Answers
m ALOBAIDY

^
arsuments against 0such
hvpertr Phic and
participation are
that the r0r,tr .^ "
31 0r,nal
1 (
° " " t aS Weprotected by the
uldhave serious ife oriK consequences.
* *
ribs anH a
' serious renal
JrDK
;^JJU
is a congenital condition in which the kidney is rpnbr u
J £ „J r
» »*
* * , * , ^
?£ ?!
>,, » KMM
«rallv is
» IS WMW
l inherited.

1 in 2.000. MCOK
*" *
Teral

I,
rt|e
AMULTOCULAR cyst (multilocular cystic nephroma ) is a lesion in the kidnev
spectrum of diseases, along with multilocular cyst
Mi falls in a with partially
jfferentiated Wilms tumor , multilocu lar cyst with nodules of Wilms tumor
or
cyst : Wilms tumor. The multilocular cyst is considered benign and is
n
*
the multicystic dysplastic kidney . More than 95 %
unrelated
occur in children < 4 yr, and
most are discovered during evaluation for an abdominal or flank mass
. The
lesion should be removed.
4(A) Upper urinary tract anomalies are more common in children with certain
apical findings. The incidence of renal anomalies is increased
if there is a
> gle umbilical artery and
^ an abnormality of another organ system (congenital
^ disease).
External ear anomalies (particularly if the child has multiple congenital
- rialies ), imperforate
^^
nfar
D)
>ts with th
anus, and scoliosis are associated with renal anomalies.
ese physical findings should undergo a renal sonogram
Urinary tract infections are caused primarily by colonic bacteria.
,

^ richi13 C K
° cause 54-67% of all UTIs, followed by Klebsiella spp, an
known t0 cause
UTh Enteroc occus , and Pseudom onas. Other bacteria
, an ,
Snw
6
StaPhY lococcu s saprophyticus, group B streptococ cus
v' StaPhylococcus spp -
aureus, Candida spp., and Salmonella
^ Fever
Dj

^ mav particula r conside ration should


^rf 0r
Nai^ an ni !
the only manifestation;
mperature
0re than
> 39C without another source
48 hr for females.
lasting more than 24 hr
nvelonephrit is is a rare type of renal H
ulomatous PV jnfjammation with giant cells and %
Xanthogran
9 IE)
‘ by ranutom
» , renal mass or an acute 0r Rh
characterired * . a $

t,
chr

infection-
* " 'SwaW
«. T snd
of this lesl“
T
“' '
wh - T,
“ *
equ * “
* «
' total J C fe5

contribute .
partial nephrectomy chelating agent that binds to cysteine 0t
is a
10. 0
( D-penicillamine
solubility of the product. Although Poorj
the )
homocysteine, increasing it has been reported to be effective in dissolvj
tolerated by many patients when hydration and urinary
(

recurrences

.
in preventing
cystine stones and
alkalinization fail. . often positive in infected urine
talllfnfVte esterase are
of MM to MM.
m H 4 hr fa metabolism
«
may « „ SXtccted io eases of UII
)
if the ocsaoism does oot coneert mtwec »
or if the child has urinary frequency, where
nitrites (most notably e nterococcus .
the conversion to nitrites. In febrile infants
there may not be enough time for
less than 60 days old, the presence of pyuria
, nitrites, or leukocyte esterase ha;
a high sensitivity and specificity for a UTI.
12.(C) Sterile pyuria ( positive leukocytes, negative culture
) may occur in partially
treated bacterial UTIs, viral infections, urolithiasis, renal tuberculosis, renal
abscess, UTI in the presence of urinary obstruction, urethritis as a consequence
of a sexually transmitted infection, inflammation near the ureter or bladder
( appendicitis, Crohn disease ), Kawasaki disease, schistosomiasis, neoplasm,
renal transplant rejection, or interstitial nephritis ( eosinophils).
13.(D) Nitrofurantoin should not be used routinely in children with a febrile UTI,
because it does not achieve significant renal tissue levels.
14.(D) The rate of renal scarring increases
between days 2 and 3 of fever; this

= n
” the prompt
,
rc:xrnegative
invariably negative.
fvaluation
15.{ A) Primary VUR appears to be
variable penetrance.
and appropriate treatment of a recurrent UTI

an autosomal
within 24 hr f
a urine culture during

dominant inherited trait with


of
° “* *
treatment is almost -
16.(C)
17.(A) Radiation exposure durine a
than that from a contrast VCUG lonuclide cystogram is significantly less
18.(0)
19 (B)
r

.( A .*, 0ne natai


oliguria can cause temporary
perform the
decomPassion of
jO «£
** , is ideal
ui jt
to first postnatal US
after the 3rd day
a

^
renal
tfpt uying
' may be impractical.
us
ulti(T)ately depends on the degree of puimon , _ ,
°gnlasia 1

^
af ) Thepr third f Children *'
/A
t ^°rZst common
- 0 ne ° ,
With brune-belly
,
syndro
are
*

> Hie n the feW m0


°!
cause of "severe
fe
becauhSe of Pu nary hypop
Bia
obstructive uropathy in children
is
tfi ZXg 1 ifl 8
°°°
'
m 6
7* S S 6 Vefe f0rm
of obstructive

^
3 ffe urethral atresia , a rare condition.
ajeS is
it!?3
commonly in children with connective tissue disorders,
*1$ They occur
syndrome , Ehlers- Danlos syndrome, and Menkes syndrome.
Williams
'
atent urachus results in continuous urinary drainage from the
A diverticulum, which is a diverticulum of the bladder
us
yesicourachal
#bil ' mbilic3l-urachal s * nus' which is a bJind external sinus that opens at
* u
do *' a 5
" " lesions should be excised.
the umbilicu calculi have developed in as many as 70% of children followed
'

27 (D) Bladder terocystoplasty . The calculi develop in response to mucus that


en
fur 10 yr after as a nidus for stone formation. This
in the bladder and act
accumulstes by daily irrigation of the biadder with sterile
complication can be prevented
saline.
} Appiroximately 30-45% of children with a high imperforate anus have a
28JA
ic biadder , often because of sacral agenesis. Newborns with
neuropathic
undergo a spinal ultrasound during their initial
mperforate anus should

-
evaluation.
are overactive
BW The most common causes of daytime incontinence
ssrsrttiijESSSrr MI
ai|ure of the external sphincter to relax during voiding in c railed
n ur i
with this syndrome also cal
* ° ° abnormalities. Children
gic
stream, day ana
10nneurogenic
neurogenic bladder, typically exhibit staccato
wetting, recurrent UTIs, constipation, and encopresis.
^J! * I* ,
%M
ln children up to the age of 14 yr, the mean bla er
,
0 be
aBe
e 0 mL). *
2 years) times 30 (e.g., the bla er
outers
0f a 6 yr old

3 I*
^
. ^ sejf.limited and
, to

^ assure the child and parents that the condition development


^ Qid psychologica
measures that can affect the child's
age

longer
using the oral tablets

rePSCeCs ere
oatient with persistent
ly ac d
The diagnosis should be
Curate
crys tallu ria SUsPectf!j
.
jn children Is sickle cell
'Vial n dlseaa

^fs*
The mos t com mo cauS
*
35‘
by a pre n n( of sickle cell hemoglobin. As many
h characteri zed
tf
j
develop priapism. The priapism *
^,
% of children with
«ondary to sickling of red biood cells *
1£££ cavernosa during normal erection reS1% .
venous stasis.
36 (A) Primary
Rudimentary
testicular fal ure
testes syndrome aod
,re5U|t from gonadal dysgenesis a
occurs in Robinow syndrom ,

—-
njng q{ {he forearms faM
(characterized by hypoplasbc genita

S2,
37 (B) Spontaneous descent oc
^ ^57EU short broad nose, long

suie (termed a minipuberty) dur g


significant penile growth. If the testis
first 2 mo, which also resu tn
descended by 4 m0, it will remain
undescended.
3B.(C) Inguinal pain and/or swelling in a male with an undescend
ed testis should
raise the suspicion of an inguinal hernia or torsion of the undescend
ed testis.
(most often
39.(D) Urinary tract infections caused by urea-splitting organisms
Proteus spp., and occasionally Klebsiella spp., Escherichia coli, Pseudomon
as
o
spp., and others) result in urinary alkalinization and excessive production
ammonia, which can lead to the precipitation of magnesium ammonium
phosphate (struvite) and calcium phosphate. In the kidney, the calculi
ofteft
relgn
have a staghorn configuration, filling the calyces. The calculi act as f °
kidney
bodies, causing obstruction, perpetuating infection, and causing gradual
damage.
40.(C) Cystinuria is a rare autosomal recessive disorder of the epithelial
the renal tubule that prevents absorption of the four dibasic amino .
( cystine, ornithine, arginine, lysine ) and
results in excessive urinary excreti|on ^
these products. The only known complication of this familial disease S
t

of -
calculi because of the low
solubility nf
Patients

can
eyeals pyurto. Epididymitis can be
< 3


infectious f
UrnalysiS
'
^IVL
| the
) but often
organism V 8 n0C0
0
**L include familial Mediterranean
remains undet
fever, enterovirusTn ^° Ccw or
,
Addi ional
f
"mi testicular
i'* difficult
U *• » » .
torsion produces acute pain and
swellin
» « >1« w i s * t of

. The cremasteric reflex nearly always


»>
often
to examine
of the testis is abnormal, and the testis position
< ! and
on ( lie)
position often
. In addition, often there is associated nausea and * e
scrotum be differentiated from an incarcerate „em,a vomiting. The
condition can because swelling in
inguinal area typically is absent with torsion
the
43.(0) Children with urolithiasis usually have er ,c,oscol hematuria
severe flank pain and can have irritative svmiL!! "
! *
urgencV and .
.
frequency If the calculus is in the distal ureter the C3n have Native
symptoms of dysuria, urgency, and frequency , 1 passes im «« '
bladder, the child usually becomes asymptomSr 2 ^
1
* , °
dysuria and difficulty voiding can result, particularly n ^^
44,{C) In a child with suspected renal colic, there are multiple imaging options.
Hie most accurate study is an unenhanced spiral CT scan of the abdomen and
pelvis. This study takes only a few minutes to perform, has 96% sensitivity and
specificity in delineating the number and location of calculi, and demonstrates
Aether the involved kidney is hydronephrotic.
Chapter 24
Gynecologic Problems ofChf/JJT^

QAHTAN ALOBAIDY
Questions ^
bleeding requires further evaluation if it persists be
1. Neonatal vaginal Vond
A. 5 days
6. 10 days
C. 20 days
0. 30 days
E. 45 days

2. Which of the following types of hymens associated with urinary obstructiion


A. Crescentic ’
B. Redundant
C. Microperforate
0. Septated
E. Imperforate

3. Which of the following ditoral width in a newborn need further evaluation?


A. >2 mm
B. >3 mm
C. >4 mm
D. >5 mm
E. >6 mm

4. What is the MOST


period?
mmon cause of ambiguous genitals in the neonatal
A. Congenital adrenal
hyperplasia
B. Complete androgen
C. Male insensitivity
pseudohermaphrodite
D. True hermaphrodite
E. Idiopathic

571
1
,r 0ld . Presented with sterna! genital
y
»
5' „
A tionrev reveals skin-colored papU|es some Wl Pruritus and s
u/ j
ft
* ^ ashaBgy, Caulifl^in esion
|
.
the MOST likely diagnosis is Wer. ike
' tollo X ^ S
#
° |

llluseum
^ acuminata
#
contagiosum

6 C
ordvlorna
simplex
- tierf#5
D lichen isclerosus
£. contact dermatitis

SjSi infections
Cotrimoxazole
caused by «as ,
first line th apv
^ in
A.
3. Clarithromycin
c Azithromycin
D. Amoxicillin
E . Erythromycin ethyl succinate

7 Which of the following antibiotics iis recommended


as first line therapy in
,<jvovaginal Infections caused by Yersinia ?
A Cotrimoxazole
0 Clarithromycin
C. Penicillin V
0. Amoxicillin
E . Erythromycin ethyl succinate

^uentymorof,th-old girl presents with napkin rash treated by increasing the

^^
diaper changes, allowing her to be diaper free, frequent bathing,
plication of zinc oxide but without significant benefit.
0T
Allowing can decrease the inflammation of his diaper dermatitis
aPPlied topically ?
fungal
Bn .
C.
Ceroid
Coc
D. ° nut oil

E. ^er tion violet


|

Abiotic
following is FIRST-LINE
therapy in patients with lab;a , dh
9. Which of the * \
and difficulty voiding?
A. Topical estrogen
B. Surgical correction
C. Topical antibiotic
D. Systemic antibiotic
E. Systemic anticholinergic

of the following syndromes is the MOST common cause of Va if


10. Which 8 \i
agenesis ?
A. Fraser
B. McKusick -Kaufman
C . Johanson-Blizzard
D. Renal-genital-middle ear anomalies
E. Mayer -Rokitansky -Kuster -Hauser

11. Which of the following dermatologic conditions affect the vulvar area r,
children and can lead to destruction and scarring of normal genita
architecture ?
A. Vulvar psoriasis
B. Lichen sclerosis
C. Idiopathic vulvar aphthoses
D. Contact dermatitis
E. Lichen planus

12. History is often more predictive than physical exam in the diagnosis of which
of the following conditions ?
A. Vaginal foreign body
B. Pinworms infestation
C. Labial agglutination
D. Molluscum contagiosum
E. Condyloma acuminata

discharpp . ^ presents
scharge fa ls to respond to
the following, the MOST
A. vaginal foreign
with blood
hygiene measures
likely diagnosis is
-tinged and foul- smelling vagina!

body
B. acute genital ulcer
simplex infection
herpes comagi0surri
c
0
Museum acuminata
t
condyloma
.
of developmen t of f e brea 15
, c0n ,derec/
'
it Lack hichofthefol ages? delayed
/
,MwnX
) *
lOynr
and
Warrants
A
B 11 year
C 12 Vear
D- 13 year
E l4 year

which of the following is characterized by amastia


nectoralis muscles? and aplasia of
the
Ectodermal dysplasia
& Congenital adrenal hyperplasia
C. Gonadal dysgenesis
0. Hypogonadotropic hypogonadism
l Poland syndrome

16. Which of the following is the


MOST common cause
adolescent' s girls? of breast pain in
A. Exercise
B. Pregnancy
C. Prolactinoma
D. Fibroadenoma
E. Polycystic
ovary syndrome

A tazepam
blowing drugs can cause unilateral
thelarche?
B.
^
^HaTietidine
'
operidol

E.

'Which of
^vproheptadine
ral
Contraceptives

the followi
'

^A adolescent girl? is the MOST common solid mass seen in the breast
hpoma
,

'
574
B. Intraductal papilloma
C. fibroadenoma
D. Breast carcinoma
node
E. Jntramammary lymph

is the MOST common ovarian neon!asfTi


19. Which of the following in chil
and adolescents ? \
A . Dysgerminoma
B. Cystadenomas
C. Gonadoblastoma
D. Gynandrobfastoma
E. Mature cystic teratoma

20. A 10- year old girl presents with acute lower abdominal pain, nausea
*

vomiting. Examination revels abdominal tenderness. Pelvic ultrasound


ane
sho
enlargement of left adnexa and demonstrate the presence of Doppler flow, ^
free
pelvic fluid, and "whirlpool sign". Of the following ,the MOST likely diagnosis
is
A. tubo- ovarian abscess
B. endometriomas
C. adnexal torsion
D. ovarian cyst
E. uterine rhabdomyosarcoma

21. Which of the following heritable syndromes associated with


mullerian
anomalies is X-linked type of inheritance ?
A . Camptobrachydactyly
B. Hand-foot-genital
C. Fraser
D. Uterine hernia
E. Renal- genital-middle ear
anomalies
22. Which of the following is
the MOST sensitive and specific imaging technipue
used for evaluating
mullerian anomalies ?
A. MRI
B . Ultrasound
C. Hysterosalpingogram
D . Sonohysterography
E. CT scan
r
^ 0Vne
Answers -"
ood

TAN ALOBAIOY
^ c,tro«enic
effects commonly notable in nP„„lt .
white vaginal discharge. prorn
'and"ence
Strode
majora and a
slightly from the vestibule. A Z 0f

JdW from endometrial,s


Bleeding that
sloughing following matern hoi " Withd' al
"'
bZ° °Z
0
313 vaginal

^
% occur. excessive or persistent
DeV nd 1 mo 0f |ife

ME) Cervico
evaluation.
”tresfurther mucus
-vaginal
of an
secretions can accumulate beh,n tK
imperforate hymen and manifest as
., 6 Nocked

wrflow tract if urinary obstruction occurs, correction of mucoco P s- In this


mstafice and period is indicated.
J °
he Perforate hymen
„the neonatal '""
KE, The clitoris may appear large in proportion to the other 8 3,
premature infants. If the clitoris annexe
structures
en arged the clitoral
.
especially in
width should be measured; values > 6 mm in
„ . ' ,
^^ ' ^
6 0r
Wher evaluation. If ditoromegaly and ambiguous 8 a
*
obstetrician and pediatrician should immediate^ nK . -
6 present the

- luation of the infant and to coun« TtClen , "^


s
fr
°
SS
.
j apnosjs anH ' ^
sa wastingforrns
subsequent fluid and electrolyte imbalance. Delay in the
reatment of congenital adrenal hyperplasia may be life-
^MB)
eatenineo*

MO)
MA)
increasing the

^^^
A) Pirst-|jne
t lncludes hygiene measures such as
frequent
uency of djapef an eS lowing the infant to be diaper free, If diaper
f n ° ^ /
such as zinc oxide.
- ' & and apniic
Ca n water-repellant barriers the classic satellite
^ '^
the lf)
^ anirnatj0
^
tis perSistS
^
°
•o;ns of ndid are present treatment with a topical
na /
, or if
ese conservative measures antifungal can decrease
9 (A ) First-line therapy in
.
patients with difficulty vo dmg, persistent
or pain includes topical estrogen
steroid (Betamethasone 0.05% ointm ent ) appli ed tw .
( Premarin or Estrace cream 0.01%) mfectl
c e daily
or ,*
to the °Pic. ^|
miril
raphe under gentle traction. Surgical correction is rarely necessaJ?
recurrence is common until the age of puberty . '^ '
10.(E) The most common caus e of vagin al agen esis is MayerRokitansky-KQ
with an incid ence

^"^.
Hauser (MRKH) synd rome , repor ted at 1 in 4,
000
female births. After gonadal dysgenesis, mullerian agenesis is the seco
nd
common cause of primary amenorrhea. The etiology is believed >ost
multigenetic and multifactorial. This condition is present at birth but to b
diagnosed until mid- adolescence.
often not
ll.(B) Lichen sclerosus is commonly seen in the anogenital region
and has
characteristic appearance of white skin changes associated with
erosion, ulceration, and petechiae . This disease can cause seve
areas of
re
and most commonly presents with vulvar or perianal pruritus, discomfort
dysuria, and
constipation.
12.(B) If pinworms are suspected, transparent adhesive
tape or an anal swab
should be applied to the anal region in the morning
before defecation or
bathing and then placed on a slide. Eggs seen
on microscopic examination
confirm the diagnosis, and sometimes the pinworms
can be seen at the anal
verge. Clinical history is often more
indicative of disease than physical exam,
and a negative tape test does not rule out
this pathogen as a cause.
13.( A ) Vaginal foreign bodies are a
common finding in children presenting with
blood-tinged and foul-smelling
discharge. Quick identification and removal of
the foreign body avoids potential
complications, including recurrent urinary
tract infections, dermatologic
abnormalities, vaginal perforation, or fistula
formation. ^ The most common object
toilet paper.
found in the prepubertal vagina is retained
14.{D)

* USl allv unilateral and can be


systemic dkn H
congenital adrp
hypogonadism), or
li
novei
^^^ ^
6 135 3
congenital or associated with
* ectodernrial dysplasia ), endo
crine disorders M;*
' eonadal dysgenesis, hypogonadotrop«
' j
the underlying mesorip mutations- can be associated with anomalies o
SUCh 3 S abnormal
pectoralis muscles seen in P< an
syndrome ( aplasia
of th6 , ^
and radial nerve
16 (A) The
^piasia ) *******
PhaUS?
mUSC es rib deformities, webbed W
'
,
**
^
benign breast changes
ranges. Physiologic breast pajn n adolescents are exercise af
S

swelling and tenderness occur on a cyclic


,, n)Y during the premenstrual phase,

* !,?
and are secondary
1 C< 1
*nalsti
fi on and resulting proliferative changes. to

^Veve' *' ,
161 dn
later3 melarche
b when
m
has also been reported as a side
the medication is stopped.
common solid mass seen in
si effect ot
cimetidiine
adolescent girls js
So The
Loade fibroadenomas are most often located in the upper

^'
0
"
oadraotot ln breast
. The average site is 2-3 cm, and 10-25%
physical examination is usually diagnostic
outer
of patients have
Multiplelesl0 s' The

l9 .(f)
. Sound
usionsare.
cases
^"
^
cifcumscribed, rubbery, mobile, and not tenderbecause
may be helpful in making the diagnosis.
, In

common ovarian neoplasm in children and adolescents is


! teratoma (dermoid cyst). Most are benign and
these
equivocal

the
mature oft ' contain mature
tissue of ec
mesoderrn®
^
^
skin hair, sebaceous glands, neuroectodermal
^
bon 6 cartilage
i , fat, teeth ),
tissue),
and/or endodermal (thyroid,

M , gastrointestinal) origin.
torsion of the ovary and/or fallopian tube is the fifth most
common gyne 8 emergency and occurs more often in children and
mon gynecologic
individuals with normal adnexa but

ovarian (teratoma, cystadenoma) neoplasms.


21 (D)
22.{A) MRI is considered standard of care and best suited for complex
anomalies because of its noninvasive, high- quality capabilities. MRI is the most
sensitive and specific imaging technique used for evaluating mullerian
, blood flow,
anomalies because it can image nearly all reproductive structures
images, and
external contours, junctional zone resolution on T2- weighted with
MRI also has a high correlation
associated renal and other anomalies.
multiplanar capabilities and high spatial
surgical findings because of its
resolution.

578
Chapter 25
The Endocrine System
Questions
HAYDER ALMUSAWI
1. What is the most sensitive test for primary thyroid dysfunction ?
A. FreeT3
B. T4
C. TSH
D. TRH
E. Thyroglobulin

2. Thyroxine-Binding Globulin ( TBG ) level usually decrease in


A. pregnancy
B. administration of glucocorticoids
C. newborn period
D. hepatitis
E. administration of estrogens

3. The result of screening program of a child is low level of total T4 and normal
levels of free T4 and TSH.
What is the most likely diagnosis ?
A. Primary hypothyroidism
B. Secondary hypothyroidism
C. Tertiary hypothyroidism
D. TBG deficiency
E. Thyroditis

4. What is the dose of L-T4 for a 7- year old child with acquired hypothyroidism
A. 1- 2 pg/kg
B. 3- 5 pg/kg
C. 6-8 pg/kg
D. 9-11 pg/kg
E. 12- 15 pg/kg

579
,
rea ed with amiodarone should have serial monitoring
enal' function
laren
5C '"* Ithyroid function
of

B
function
C. liver count
latelet
l blood sugar
h of the following is a characteristic finding jn fa filial
6VS/hic dysalb
uoerthyrox ‘nern urninemic
serum concentrations of T4
A increased
B Decreased serum concentrations of TSH
C Increased serum concentrations of free T4
0 Increased serum concentrations of free T3
E. Increased serum concentrations of TSH

7. What is the most common cause of permanent congenital hypothyroidism?


A. Defects in thyroid hormone synthesis
B. Defective Iodide transport
C. Thyroid dysgenesis
D. Defects of Iodine organification
E. Defects of thyroglobulin synthesis

8. A 10-month-old boy found to have bilateral fallopian tubes and a rudimentary


uterus during repair of herniorrhaphy. Biopsy of the gonads reveals normal
,
testicular tissue. On examination, his phallus is normal in size and appearance
and his testes size is 2 ml for each.
Wat is the MOST likely sex chromosome pattern for this child ?
A. XY
B- XX/ XY
C. XXY
0. XO/ XY
E- XYY
9- GH re lease ?
'
Which of the following play a
Sleep
role in inhibiting

B. Exercise
-
C Physical
stress
Trauma
580
E. Hyperglycemia
following inhibit release of CRH and ACTH ?
10. Which of the
A . Atrial natriuretic
peptide
B. Arginine vasopressin
C. Oxytocin
D. Angiotensin II
E. Cholecystokinin

n A 9 -year - old boy has been hospitalized for 1 month following r0ad
accident. He is in a full body cast and has not been able to ambulate. |n
2 days, he becomes irritable, anorectic, with frequent urination.
Of the following, the MOST likely cause is
A. hyperglycemia
^
B. hyponatremia
C. hypercalcemia
D. hypokalemia
E. cytitis

12. Which of the following is an indication for rhGH treatment to promote


linear
growth?
A. Chronic renal failure after transplantation
B. Idiopathic short stature
C. Constitutional growth delay
D. Edward syndrome
E. Thalassemia

13. Which of the following is a


reasonable Criterion for stopping GH treatment ?
A. A decision by the patient
that he or she is tall enough
B. A growth rate <3 inch yr
/
C. A bone age >16 yr in girls
D. A bone age >18 yr in
boys
E. Recurrent
abdomi nal pain
14' h
Hypertensio
B. Hypocalcaemia
1°W 6
'" * CaUSed by reco"*inant IGF 1?
-

C. Hypoglycemia

581
0-
Anemia!
E- Acidosi
following results can establish the diagnosis of
diabet »,s inspidis
, Which of the

-^-
5 ,
potassium <3.5 mmol/ L an Serum sodiUm l4S .
K Serum potassium >4.5 mmol/i anW,Seru « sod Um l3S
B. Serum osmolality is <270 m0 Ag and the
" "
mmol
/ , "
* Jf
C Serum kg
mOsm/

"*''
a itViS >30
„ ' ;
>
° - ^
sm/kgandth
;o m0sniA!
^ ,ne ^
smo(a//ty is >330
XT
Ur
E. °
initial approach to a patient with hyponatremia begins
K w th
with ,
16. The
determination ofstatus
A. volume
B. urine sodium
C. urine specific gravity
D. serum potassium
E. BUN

following causes of hyponatremia is associated with


17. Which of the
hypovolemia and high urine sodium?
A. Systemic dehydration
B. SIADH
C. Renal primary salt loss
D. Runner's hyponatremia
E. Factitious hyponatremia
( SIADH) is
antidiuretic hormone secretion
Syndrome of inappropriate
characterized by
^ hypernatremia
( 100 mo sm / kg)
concentrated urine >
C- decreased plasma volume
D - low urine sodium
E. high serum uric acid

582
amount of water a person with normal renal
19. What is the
maximum
daily ?
can consume
A. 4 L/m2
B. 6 L/m2
C. 8 L/m2
D. 10 L/m 2
E. 12 L/m2
during childhood?
only diagnosed adenoma
20. What is the MOST comm
A. Prolactinoma
B. Corticotropinoma
C. Somatotropinoma
D. Thyrotropinoma
E. Gonadotropinoma

21. The nurse in NICU called you


because of persistent hypoglycemia in a 3 -day.
the neonate had eye proptosis with
old male neonate, on examination
malformation , earlobe creases,
periorbital fullness, mid-glabellar capillary
macroglossia, hepatosplenomegaly, and omp
halocele.
What is the most likely cause of hypoglycemia ?
A. Poor feeding
B. Systemic infection
C. Adrenal insufficiency
D. Pancreatic 0-cell hyperplasia
E. Fatty acid oxidation disorder

22. An 11- year- old boy presents with history of headache, mood disturbance,
behavioral swings, difficulty with sleep, decrease in attention span, and a
decline in school performance for the last 3 months , examination shows diffuse
smooth goiter.

T .TS " ‘"Pl" B“


B CT brain
4
0P

C. ESR
D- CBC and blood film
E. Neck ultrasound

583
high in patients with Beckwith-
^" *
r sk Wi dema
hey need regular surveillance ^
with abdomi .
sVndroihe until g
A
"**"“Ascend and
0 HCGassaV assay
17 hydroxyprogesteron
u-l-
antitrypsin assay
Q acid
£ serurTHJric
of the following is a diagnostic
criteria f f
2i Which - ° diabet $ mellitus ?
plasma glucose 100 125 mg
A. Fasting /di *
B. 2 hr plasma > -un mg/dL
glucose during OGTT '
C. Hemoglobin
A1C 6.5% °
mg/dL

25. The dearest evidence of a role for viral infection iin human T1DMifs seen
in
A. congenital rubella syndrome
8. enteroviral infection
C. mumps infection
D. RSV infection
E. HSV infection

.- - i2 vears

SL *.£ * * *»" "«


jc old girl presents to the ER with history of repeated vomiting and

and excessive thirst


P
What is the next step in her management?
*
A. Intravenous fluid
B. Analgesia
C. Ultrasound abdomen
0. Blood glucose
E. Surgical
consultation
breathing, his
ll - An 8-year
-old boy abdominal pa and rapid
presents with
blood sugar is 483 mg dl.
/
What« is the MOST appropriate inhismanagementr
next step
^ Subcutaneous insulin1unit/kg
45% glucose saline 20 ml/ s/ r
^ k^
Intravenous insulin 0.2 uoit / &
584
10 ml/ kg/ hr
al saline
o. 0.9% norm g

.. £ Intravenous ana

saline
meq/i.

A. 0.9% normal
B. 0.45% saline
C. 0.18% glucose saline
D. 5% glucose water
E. Ringer lactate
fluid to be used when the blood gluco
29 What is the type
of intrav enou s
tient with diabetic ketoacidosis?
^
treatin g a pa
dropped to 220mg/dl while
A. 0.9% normal saline
B. 45% glucose saline
C. 18% glucose saline
D. 5% glucose water
E. Ringer lactate

in children due to its potential to


30. Propylthiouracil is not recommended
cause
A. renal failure
B. respiratoryfailure
C. agraniolocytosis
D. DIC
E. liver failure

31. In patients who might have transient congenital hypothyroidism, a trial of


stopping L - T4 for 4 weeks may be undertaken after the age of
A. 6 months
B. 9 months
C. 18 months
D. 24 months
E . 36 months

32. What percent of


children with type i diabetes 0
^
autoantibodies ? mellitus may develop thyr

585
6
*
A- 5
*
10
20*
4
a 40’%
l 80

0
Rations
jf ' eS
SSSSS
^
2 reveals low TSH , high free T4 . and
-* 231 radioiodine uptake suppression.
^of the following' disease
MOST likely
, the diagnosis is
a
mood
decYmedisturbance
small thyroid
si.,.,
serum thyrogiobuiin
,
in school

A. Graves
factitia
B. thyrotoxicosis
’s thyroiditis
C. Hashimoto
0. myxedema
E. thyroid carcinoma

34. Which of the following indicate a high likelihood of GH deficiency?


A. Exotropia
B. Proptosis
C. Cleft lip
D. Micrognathia
E. Saddle nose

35. A 1-year- old child presents with severe short stature


withjengths »4 »
below the mean, his birth length was 1 SD below t e me
while insulin-like growth factor (IGF) 1level is low .
Of the following, the MOST likely diagnosis is
A. Laron syndrome
B. Prader Willi syndrome
0. Silver Russell syndrome
0. Noonan syndrome
E. SHOX gene variant
deficiency in a
of growth hormone
^-yearWhich of the following i
is a physical ien
sign

^ -old boy?
A- Proportionate
short stature
586
. -srsss-
D. Low-pitched
E. Large penis
voice

the level of GH in normal


that rapidly increase chii
37. Provocative
include
tests
administration of ^
A. thyroxine
B. arginine
C. metoclopramide
D. somatostatin
E. histadine

rhGH treatment to promote linear


38. Which of the following is an indication for
growth?
A. Down syndrome
B. Digeorge syndrome
C. Noonan syndrome
D. Patue syndrome
E. Edward syndrome

39. Which of the following is a criterion for stopping GH treatment?


A. A growth rate <2 inch/yr
B. A bone age >12 yr in girls
C. A bone age >16 yr in boys
D. Sever bone pain
E. Uncontrolled hypertension

40. All patients diagnosed with GH


deficiency should undergo periodic
evaluation of
A. renal function
8 Hver function
c. PT, PTT
adrenal function
E. lipid profile

Which of the following d


rugs maY nduce
A - Cisplatin
' nephrogenic diabetes insipidus ?

587
ristine
B.
eyelosporine , hamide
c lopr
hosp
uj*
p. Cyc
l Doxorubicin
tonicity is regulated by
Ujjiar fluid
intak
i e and excretion
4 sodium intake and
excre tion
0. pota ssiu m
wate r intake and excretion
C. metabolism
0. calcium
l environmental temperature
ln a full-term
newborn?

B. Proximal tibiaI
C. Proximal ulnar
0. Proximal fibular
[. Proximal radial

44. Acquired nephrogenic diabetes insipidus can result from


A. hypocalcemia
B. hyperkalemia
C. rifampin
D. isoniazid
E. streptomycin

45. Hyponatremia with decrease intravascular volume and low urine sodium is
usually seen in
A. systemic dehydration
B. S1ADH
C. primary
polydipsia
D. cerebral
salt wasting
E.
factitious natremia
hypo
secr inn is characteriz
etion ed
hormone
by ^ ^r°me of inappropriate antidiuretic
g ^Pern atremia
Appropriately diluted urine
n
588
volume
r decrease plasma
o low urine sodium
E low serum

47 Which of the
A Marfan
uric acid

following
syndrome
. overgrowth syndromes need tumor surveili

syndrome
B Loeys-Oiett
,

Q Perlman syndrome
0. Homocystinuria
E . Lujan syndrome

48. What is the FIRST sign of


puberty in female ?
A. Breast bud
B. Pubic hair
C. Axillary hair
D. Menstrual activity
.
E Peak height velocity

49. Peak height velocity occurs in females at breast stage


A. I-II
B. II—III
C. III-IV
.
D IV-V
.
E V

50. The mean age of menarche is about


A. 11.75 yr
B. 12.25 yr
C. 12.75 yr
D. 13.25 yr
E. 13.75 yr

, < >
LL r hrXismine, the recommended' initial dosefor the treatment of cong^
hypothyroid
L 'T4 is 8 ven orally
is
1

A - 2- 5 Mg/kg day
/
B. 5- 10 Ug/kg day
/
C. 10- 15 ug/kg
/day
D- 15- 25 jig kg
/ /day

RQ
Ug/kg/ day
j00-1 50
t-
(L -T4 ) tablets tu ,
Ul i be crUs
. levothyroxine °
of liquid and should notbem .* ^
Us(led and
:^
J
e(1 2 -
/k
^ r
m\,ed With ffiixed with a small
'
A- breastmiinn formula ,
P te ^
0 - soV
cow milk
C
-
°l !
actose free formula
vdroW«d ormu a ,

rge/velocardiofacial syndrome has been reported in
£
^
o

C.
Veated early in pregnancy with
inhibitors
retinoic acid
cocaine
infants bom to

0 thidomide
f warfarin

male diagnosed recently with diabetes iinsipidus


teenager
, he had hist ory
0f TIOM and optic atrophy since early childhood.
Which of the following manifestations is seen in this patient?
A. Deafness
B. Eczema
C. Seizure
D. Anosmia
E. Vetiligo

-
55. A 10-year old boy presented
previous fracture since 2 years w i ot h h. e i g h t and *ei«h* are
. As pa
both the 75th percentile and
at nation resuits are unrern e
.serumcalcium concentration 7.2phjsjj
mgydL serum ,
Phosphatase 845 .
mg/
^
phosphorus concentra n 16
magnesium concentrate^ mg/dL, an<
* ^ ^
W the
following, the MOST appropriate n t step is to mea
^rC
A. serum creatinine
B- serum 1,25-dihydroxyvitamin D
t serum 25-hydroxyvitamin 0
D.
serum parathyroid hormone
590
ratio
to creatinine
E. urine calcium
T1DM since 2 months, he
-old boy diagnosed with establj
56. A 12- year
cont rol in the last 2 weeks.
glucose ?
inves tigati on is recommended now
What
profile
A. Fasting lipid
B. Protein c peptide
.
C Hb Ale
D. Serum creatinin
E. Serum albumin

- -girl prese nts with history of polyuria, polydipsia, polyphagia 3nd


57. A 6 year
reveal blood glucose 321 mg/dl, Hb Ale
decrease body weight, her investigation
8 and ketones in urine -ve.
What is the NEXT step in her management ?
A. Reassurance and reevaluation after 2 weeks
B. Start metformin
C. Check for other autoimmune disease
D. Start intravenous normal saline
E. Start subcutaneous insulin

58. What is the starting subcutaneous insulin dose in newly diagnosed 4- year
old boy with T1DM ?
A. 0.1-0.25 unit \kg\day
B. 0.25-0.5 unit \kg\day
C. 0.5-0.7 unit \kg\day
D. 0.7-1unit \kg\day
E. 1-1.2 unit \kg\day

diabetic ketoacidosis bicarbonate therapy may increase the risk of


A. hypokalemia
B. hyperkalemia
C. hyponatremia
-
D hypernatremia
hypocalcemia

60. What is the dial*1*


patient diet ?
recommended percent of energy from protein in

591
. *-l-°20V»
*B- lZ22-289
C
30 369*t
-
0 -
jg44%
l
meal blood glucose range

the target pre - in an 8- year-
* , old patient
61.
* dl
0 90 150
mg\
- "*'dl
mg dt
c 80-130 \
g 70-120 mg\
dl
mg dl
E. 65-120 \

an essential role in the management of patients


52. Nutrition plays with T1DM
The caloric mixture should comprise approximately
A. 35% carbohydrate, 30% fat, and 35% protein
B. 45% carbohydrate, 30% fat, and 25% protein
C. 55% carbohydrate, 30% fat, and 15% protein
0. 55% carbohydrate, 20% fat, and 25% protein
E. 45% carbohydrate, 25% fat, and 35% protein

63. Diets with high fiber content are useful in improving control of blood
glucose. Daily recommended fiber intake can be determined using the equation:
Gram of fiber daily = age in years +
A- 3
B. 5
C. 7
D. 9
l 11

thalassemic
T. *hat is the most helpful test to assess glycenmcic control in
'

TA HbAlc
Wlth newly diagnosed T1DM?

P Fruttosamine
n Albumin
E, Galactosamine
Hb F
592
in patients with diabetes?
on of exercise
major complicat'
65. what is the
A. HypoglVcemia
B Hypercalcemia
C Hyp«rnatrernia
D. Hypokalemia
E. Hypomagnesaemia
pubertal or adult males with mumps and may
is common in
66. Acute orchitis
lead to subfertility in
A. 3%
B. 8%
C 13%
D. 18%
E. 23%

Transient gynecomastia occurs in 60-90% of male newborns, most cases


67.
resolve within
A . first wk
B. 1- 2 wk
C. 4-8 wk
D. 9- 12 wk
E. 4- 8 mo

68. A 9- month- old girf brought by her mother due to stunted growth, on
examination extremities are short , and head size is normal, the anterior
fontanel is large, protruded tongue, dry scaly pale yellow skin, with clear sclera
Which of the following is the MOST appropriate initial test ?
A . 6H assay
B. Serum calcium
C. TSH
D. LFT
E . CBC

593
25
The Endocrine System
Answers
I

„ cerum TSH levels are the most sensitive test


for nr

^Cssed
P
- rtion. Serum TSH levels are elevated ,
in priman h ^
hyiS
thyroid
in hyperthyroidism. In central ( secondary ) **"serum
’^

^
1(B)
jr
either low or inappropriately in the normal range despite
free T4 level-
a tow serum T4
Estimation of TBG binding, is occasionally necessary because i, is mcreased
decreased « a vanety of cl meal stations, with effects on the level of
TBG binds approximately 70% of T4 and 50% of oral
T4 and T3. T3 TBG levels
increase in pregnancy , in the newborn period, with hepatitis
f , and with
administration of estrogens ( oral contraceptives ), selective estrogen receptor
modulators, heroin or methadone mitotane, and 5- fluorouracil. TBG level
decrease with androgens , anabolic steroids, glucocorticoids, nicotinic acid, and
L asparaginase. These effects are the results of modulation of hepatic synthesis
of TBG.
1(D) Congenital TBG deficiency is an X-linked dominant trait, it is most often
discovered through screening programs for neonatal hypothyroidism. Affected
patients have low levels of total T4 (usually<4pg/dL) and elevated
tmodothyronine ( T3 ) resin uptake, but levels of free T4 and thyrotropin ( TSH )
^ normal.
4 ( B) The
dose on a weight basis gradually decreases with age. For children age
,
13 he average
daily L - T4 dose is 4-6 pg/kg; for age 3-10 yr, 3-5 pg/
kg; and

'" ageAm10, -16 yr, 2-4 pg/kg. of 37%

^chiiT odarone, a drug used for cardiac arrhythmias and


bV Weight * uses hypothyroidism in approximately
^
Children treated with amiodarone should have sen
coning
jtoring 0f

^function. tosomal dominant


abnormal

^n, co"«ntrations of T4. Levels of T3 are norma


1

594
are normal, and affected paN
However , levels of free .
T4 free T3 and TSH
;

most common
cause of permanent c0n,
are euthyroid-dy js
^ of cases. In approximately 335f
,- w
7.(0 Thyroid * for g0 -85%
N
agenesis ). In the other 66% of >
'?
thre1 thyroid is present (

^
hypo
* *• «•«*
. tissue
. s no
2EST ,
teste end i normal
IT Ms
have fallopian tubes
and a rudimentary uterus, has the
discovered syndrome. This is an autosomal recessive disorder and
to
mullerian duct chromosomal pattern. ^
an XY
associated with , physical stress, trauma, acute illness, puberty , fasting 3
9 / £ ) sleep, exercise
hypoglycemia stimulate the releasei

.
of GH, whereas hypergfa

, . pepUm
^,
01 CRH end ACTH Mt
ACTH.
* "'
a

opioids inhibit release of CRH and


can lead to hyperca cemia .
11 (C) Prolonged immobilization
and Drug Administration ( FDA ) has approved 8 pediatric
12 « The U S. Food
promote linear growth. They are GH
indications for rhGH treatment to
deficiency, Turner syndrome, chronic
renal failure before transplantation,
-age short stature, Prader - Willi
idiopathic short stature, small-for-gestational
syndrome.
syndrome, SHOX gene abnormality, and Noonan
decision by the patient that
13.( A ) Criteria for stopping GH treatment include a
he or she is tall enough, a growth rate <1 inch/yr , and a bone
age >14 yr in girls
and >16 yr in boys.
concurrently
14.(C) The risk of hypoglycemia is reduced by giving the injections
with a meal or snack .
>300
15.(D) The diagnosis of Dl is established if the serum osmolality is
mOsm/kg, and the urine osmolality is <300 mOsm/kg. Dl is unlikely if the ser ^
osmolality is < 270 mOsm/kg or the urine osmolality is >600 mOsm/kg-
patient ' s serum osmolality is <300 mOsm/kg (but >270 mOsm/ kg) 3
pathologic polyuria and polydipsia are present, a water deprivation tes
^
•^
indicated to establish the diagnosis of Dl and to differentiate central
nephrogenic causes.
16.(A ) The initial approach to a vvrth
patient with hyponatremia begins
determination of the volume status .

595
,
dehydration „
*** slArlH WithVp
ru

.
systemc
\1
p and low urine
clemia factitious hvn
,/ . natremja ,associated
0

^* um while
0 !urine sodium. ^^ ^^ *^
*
mia
normovolemia
ted
mi J and high with
urine
and

HIH
STJL'ZsrZ ^
tomaSyndrome of ina
T one secretion (
volume, normal to
SIADH) is
concentrated urine (>100
- -highi urine

^
person with
*552* function
«i o m2 - normal renal
can
wsjszz zsiAzr
^
21.(0) Beckwith
•^ft orbital
-Wiedemann syndrome clinical
fullness, mid -
glabellar
features include eye oront ,
capillary malformation (
earlobe creases and pits, and macrosomia, including
nepatosplenomegaly , nephromegaly , and omphalocele
hypoglycemia secondary to hyperinsulinemia as a
nevus flammeus
macroglossia,
. They also have
*
,
result of pancreatic -cell
hyperplasia. fl
22.(A) In Graves disease, serum TSH iis suppressed
and free T4 and T3 are
elevated.
B.(A) Children with Beckwith-Wiedemann syndrome
are predisposed to
embryonal tumors, including Wilms tumor
, hepatoblastoma, neuroblastoma,
and adrenocortical carcinoma . Cancer risk
is high until 8 yr of age, and regular
surveillance with abdominal ultrasound and measurement of a-fetoprotein is
recommended every 3 mo until age 8 yr .
*MQ Prenatal
infection with rubella iis associated with fj-cell autoimmunity in
ft,m<10 with development of T1DM in up to 40% of infected children. TV
The
lae between infection and development of diabetes may be as h »
,
r, lrlteres ingty, there appears to be no increase in risk of a ®
^bella infection develops after birth or when live-virus rube a im
used.
!MDI
'
Qn |S ^
^ID) ..
J t>oluS
\\

Diabetic Ketoacidos
HaO
Wc ><Jsugaror IR then 2n ' 0yA feS
* <250mg/d °
'- 596
Pf°py\.
_ .«
n\e is the
-
s<
' 0
.

, . . 1

3 At
< , s diw
fW ®1
*
thi00 3
'
t Cl as f O
^13

° cofn« 3S% 0f infan' " ° c0 gen hVP' ' 0,”


d e its potent oO »
3 S it,
*
d,sm
*
^
cW dre
d

^
nendaed in
isno ®
-
3 6' , ; * ' re,
vjitH 3

^ ' , , ° '
nave
,
oSient disease an d do
and a nor
< ure ^
16 fh
d od not u
locate 6 thVr
tP
° Rt
W aV be .°aften^ after s vetran t d«sea a uial ' WeH
^
3^ UtoppmU

^^ ^
vjn 0

^
patient
a« assess wiirthtr .u

^
3 Vf e or 3 -4 {
to

32 .( C ) About 20%
_
% d 3t
' o
C ptesen c of Pef
WPe teS toe e
th thvr
Utus^ deveto
ifem^
|
0

about 5% become hypothyroid . r

autoantibodies, and is caused by exogenous thyroid h


.
33 ( B ) When thyrotoxicosis
), levels of free T4 and TSH are the same as thos
factitia
( thyrotoxicosis
Graves disease but , in contrast to Graves disease, thyroid size is small ^ *
is very low , and 123 I radioiodine uptake is suppressed .
thyroglobulin
(cleft lip, palate ) or the finding of a sclita
.
34 (C ) Midline facial anomalies
indicate a high likelihood of GH or other anterior ^
maxillary central incisor
deficiency .
posterior pituitary hormone of the GH receptor. Children wnn
mutations
35.( A ) Laron syndrome involves
clinically resemble those with severe IGHD . Birth length tends to
this condition short stature with lengths >4 %
below the mean , and severe
be about 1 SD
yr of age. Resting and stimulated GH levels tend
below the mean is present by 1
high and insulin - like growth factor (IGF) 1levels are low.
to be ),
may be the only clinical feature present
.
36 (A) Linear growth failure
, low
(
height velocity , weight for length appropriate
proportionate short stature
micropenis in males , small midface , high-pitched voice, and
or increased ,
delayed dental eruption . rapidly increase the
been ised
devis that
.
37 ( B ) A variety of provocative tests
have
administration of insulin , arginine,
level of GH in normal children. These include
clonidine, levodopa, or glucagon.
Administration ( FDA ) has approved
38.(C) The U. S . Food
rhGH
and Drug
treatment to promote linear growth. ^
indications for
deficiency, Turner
idiopathic short stature
syndrome
, small
,
-
chronic
for -
syndrome, SHOX gene abnormality, and Noonan syndrome by the
renal
gestational -
failure
age short
.
stature,
^
before tran g ( jer.0i

t th*
1

39.(C) Criteria for stopping GH treatment include a


enough , a growth rate <1 inch /yr ,
decision
and a bone age ^^
he or she is tall
and >16 yr in boys.

597
,,, evaluation of thyroid and adrenal
funrtin„ IS
. (P> pen0
dnosed with GH deficiency.
"Skated for all
'>tie|fnedS
lacellular ithiUm CiSPlatin and meth
P

^ ^ 0CVCline ; ,
° xvfurane
fluid tonidty is regulated almost
exclusivelv hv er
iJ-lC Lion, whereas extracellular volume is regulated JZ
?


W!
,nd eiCn The control of plasma tonicity and intravascular * intake

--
ntake and
'
5birthJC
-
of endocrine, neural,
integration

" w
NDI can result from hypercalcemia or
10 Acquired lithium, demeclocycline
-
behavioral, and Da

hypokalemia
°
involves
.
3

*abated with , foscarnet ,


clozapine,
and is
jjjthirillin, and rifampin.
dehydration
amphotericin,

«S.(A| 5vstemic) decreased effective plasma


,
volume, primarv sah
us Inonrenal and runner s hyponatremia all of
them
hyponatremia, decrease intravascular volume and low urine sodium

associated !!
K 4E) Syndrome of inappropriate antidiuretic hormone secretion (
by hyponatremia, an
SIAOH) is
characterized inappropriately concentrated urine (>100
mOsm/kg), normal or slightly elevated plasma volume, normal-to-
high urine
sodium, and low serum uric acid.
47.(C) Perlman syndrome associated with macrosomia,
unusual facies,
nephroblastosis, severe hypotonia and very high risk of Wilms
tumor.
*49.(WB)
50.(C) In females, the breast bud (thelarche ) is
usually the first sign of puberty
(10-11 yr ), followed by
the appearance of pubic hair (pubarche) 6-12 mo later .
interval to the onset of menstrual activity (menarche) is usually 2-2.5 yr but
maY as long as 6 yr. Peak height velocity occurs early (at breast stage IHII,
^'P cally between
11 and 12 yr of age ) in females and always precedes
menarche. The mean age of menarche is about 12.75 yr.
5Mc)

.
(
with a small v° tume 1
:£*^ .
T4 tablets

d- L T4
should be crushed and mixed t atecj

tablets should not be mixed with soy protein

Sr “
,,7- **,
, .,.,. .
Ww
to Mm
th« can MM
.T4 on » MPW
this is not practical in an infant.
in 4.000
•* ‘
newborns. In
90% *DiGeor8e/velocardiofacial syndrome occurs of chromosome 22qll- 2 -
in 1 -
this Patients, the condition is caused by a de
e 10
”numbe
,| of patients with a
SVndrorne has also been reported in a small ber
_

598
„,*«**»•ssiS’** of diab ^' 0 mothers, an
^ ^ ,rv
.
ssrs*° „
1

onr r
^
delet't0
^ tele
>ot tre®
* **”
* •**^
£51$ .*^
Wol syndrom (0>0M00-
^
" «
t
rec s disease due predominantlyPtlC
W
**?&I gene and
neurodegeneratwe di
^

- - * * jj ^
1 and optic atrophy.
Catedefinition

'gzz'rssi
-^ fo S P

^°. ’^^ and hypophosphatemia

TO
*„

-
, lamin
prolonged vitamin
0 (25l H D)
. ,
zss&tes-xz
measuring 25'hyd
toraee forrn 0 v t
,!
,! D. Although l25
dihydroxyvitamin
y D U 251OHU0
, „;tarT1in D, measuring >25(0H)2D levels is less useful m*
.
cemja drives parathyroid
d
hormone (P
1,25(0H)20 .^
»
*
increased risk of cardiovascular disease associated with
*
profile in children aOyr

57.(E)
58.( B)
59.( A) Bicarbonate therapy may increase the risk of hypokalemia and cerebral
edema so should be considered only in situations with severe acidosis
unresponsive to standard DKA management .
60.(B) High-protein intakes may contribute to diabetic nephropathy. Low
intakes may reverse preclinical nephropathy. Therefore, 12-20% of energy is
recommended; lower end of range is preferred.
61.(C) <5 yr: 100- 200 mg\d! , 5-11 yr: 80- 150 mg\dl, 12- 15 yr: 80- 130 mg\d
16-18 yr: 70-120 mg\dl.
62.(0 The caloric mixture should comprise approximately 55% carbohydrate
30% fat, and 15% protein, but must be individualized to meet specific Pat e
needs.
'
63.(6)
« .
-(B) HbAlc values mav ho
'' ' tba assemia
*(or other conditions with
with

^
hefno8 hin F) and
gp *° turnout Ce disease r other conditions
^" <° in
th °° red bl d cell
^
tb«e patients.
65.(A) A major complication of
Fru«osamine can be used instead of HbAlc

exercise in patients with diabetes is the * PresenC


of a hypoglycemic
reaction during or within hours after exercise. In Pa tienB

599
e e e. (
ro/,
K.IO Acute orchitis is c
* / c/

< '*°'°os
, * „
Hid to subfertility jn ,
^ // p
f c ert or
'*»•&* ° * ses * ^0/f a/, XjNvfc,
e^ % ^
0 f/) „ 'H .
r
/

^*ofy ' e. %

>^<<
S
^ * / 0(

** °f 6, * 6 * %/f ^ >,%
e r ^ ' * *
stoc . 0/h

C/
°
ca « ** e .n"
(yf
>
8w

/i / >
i(
«
* <°o?

600
Chapter 26
The Nervous System
Questions
ZUHAIR ALMUSAWI
regarded as a discomforting test for the child?
1. Which of the following is
A. Measurement of weight
B. Measurement of length
C. Measurement of head circumference
D. Measurement of body temperature
E. Examination of deep tendon reflexes

2. At What age smell can be tested reliably ?


A. 32nd wk of gestation
B. Full-term newborn
C. Three months
D. Six months
E. One year

3. What is the gestational age at which a premature


infant can blinks in
response to a bright light ?
A. 26 wk of corrected gestational age
B. 28 wk of corrected gestational age
c* 30 wk of corrected gestational age
D. 32 wk of corrected gestational
age
E. 34 wk of corrected gestational
age

nerved
A.
f Nowin6
° ls a sequelae of complete
paralysis of the oculomotor
Ptosis
B. Constriction of the pupil
0‘ Sr "
2 ^°*° f the eV® ward
f the e e ‘
"“
E mpairrnent
' of adductiion *
Pward
following terms is used to
the describe i^voluntary
wfich filiations of the eves? ' ,
chaotic
Xfdular
^
nystagmus
.
f nystagmus
A
» frontal nystagmus

CnD ocular
'
p lor
,
bobbing
US
t 0 ^
hich 0f the
following is usually caused by
unilateral ini ,
n ry of the
6 rt/
7
^ vagus
A Hoarse voice
g Resp ra or
distress
* ^ ^
regurgitation
c Masai
D pooling of
secretions
E. immobile, low
-lying soft palate

7 Which of the following cause an elevated polymorphonuclear WBC count in


the CSF ?
A. Early phase of aseptic meningitis
B. Tuberculous meningitis
C. Fungal meningitis
D . Spinal cord tumor
E. Immunologic disorders

3 The normal CSF protein is 10-40 mg dL in a child and as high as 120 mg/ dL
, /
na neonate . The CSF protein
falls to the normal childhood range by
A - 1mo of age
B* Roofage

^D 4^ mo10 of age
E.

”mo ofOf age
^ age
9
Wh is the ial hemorrhage
intracrama
*
'n inf ? imaging method of choice for
detecting

|SklJllari xai-ray
r r ' ultrasonography
Cranial CT
Crani
nia CT
*angiography
602
E Brain
to detect infarcts |asting
"hr.
ing
imaging procedure
is the preferred
10. What
24
thaA T«ns cranial
Cranial
D PP er .
° aphy
ultrasonogr '
B
C Cranial CT
0, cranial CT angiography
E. Brain MRI
imaging procedur e to detect brain tumor ?
11. What is the preferred
A. Skull x -rays
B. Cranial ultrasonography
C Cranial CT
D, Cranial CT angiography
E. Cranial MRI

What is the preferred brain imaging procedure to detect inborn errors o


12 .
metabolism?
A . Proton MR spectroscopy (MRS)
B. Positron emission tomography ( PET)
C. Cranial CT
0. Cranial CT angiography
E. MRI with gadolinium

13. Generalized epileptiform discharges typically occur in children with


A . cerebral dysgenesis
B. Brain cysts
C. slow growing brain tumors
D . glial scar tissue
E. structurally normal brains

14 . What is the BEST i


investigation for cases of occult spinal dysraphism ?
A. MRI
B. ultrasonography
C. spine x-ray
a scan
E * alpha feto-protein

603
in MR 1
r Brain
procedure to detect infarcts lastjn
imaging 8
ic
is the preferred "
lOr,
10. What
24
'hanA Trans cranial
Cran jal
Doppler
ultrasonography
B.
C Cranial CT
D. cranial CT angiography
E Brain MRI
procedure to detect brain tumor ?
imaging
11. What is the preferred
A. Skull x-rays
8. Cranial ultrasonograp
hy
C. Cranial CT
D. Cranial CT angiography
E. Cranial MRI

brain imaging procedure to detect inborn errors oi


12. What is the preferred
metabolism ?
A. Proton MR spectroscopy (MRS)
B. Positron emission tomography ( PET)
C. Cranial CT
D. Cranial CT angiography
E. MRI with gadolinium

13. Generalized epileptiform discharges typically occur in children with


A. cerebral dysgenesis
B. Brain cysts
C. slow growing brain tumors
D. glial scar tissue
E - structurally normal
brains
14. What is the BEST i
A. MR )
investigation for cases of occult spinal dysraphism ?
B* ultrasonography
C. spine x -ray
D * a scan
E - alpha feto -
protein
}

mother gives birth to a full-


ar -
„old term bov with
. 'A 25-V| cta t to be pregnant again , and she
asked mVelomen ngocele,
£»« " U You ' of
about chance
r answer ?
vourpmpe
< negligee
!
'. u j

B. > 8%
5-
C Ilis
is
PD' it
25%
E - it is

Mate 0131 periconceptional use of folic acid supplementation


l6
neural tube defects (NTDs) in pregnancies at reduces the
inci*ience
of hsk by at least 50%.
, folic acid supplementation
To & effective should he initiated before
concept’011 and continued
,
until at least the
A 4th wk of gestation
B. 8th wk of gestation
C. 12th wk of gestation
D. 16th wk of gestation
E. 20th wk of gestation

17, A 33-year-old woman with history of previously affected child with


myelomeningocele is planning for a new pregnancy, she read about prevention
and she asks you how ?
four appropriate answer is, supplementation should be started 1 mo before the
time of the planned conception with
A. 0.4 mg of folic acid daily
B 0.8 mg of folic acid daily
1 mg of folic acid daily
4 mg of folic acid daily
E 5 mg of folic
.
acid daily
there is absence of
18 w at
i5 the term applied for the condition in
which
^ sylvian fissure .
'^convolutions and a poorly formed
Lissencephaly
Schizencephaly
SePtooptic dysplasia
0 p
rer>cephaly
t °olymicrogyria
^
604
J
,
femal® fant oresents with repeated seizures

19 A 4-month-old
jlalogram resista _,
anticonvulsants, e ec roen
show
*
''
the retina, inciuamg
agenesis of the
and
corpus callosum.
co .
o
shows

boma of
hemihypsarrhythmia
(hemivertebra). Fundoscopy reveals
the optic disc. MR
.
X-ray
abnormal
, ^'
brain *
SV
*
Of the following, the MOST likely diagnosis is
A. Colpocephaly
B. Aicardi syndrome
C. Holoprosencephaly
D. syntelencephaly
E. Sotos syndrome

20. What is the first thing to be done if the cause of microcephaly is unknown?
A. Mother' s serum phenylalanine
B. Chromosome microarray
C. MRI
D. Fasting plasma and urine amino acid analysis
E. Serum ammonia determination

21. What is the approximate total volume of CSF in infants ?


A. 25 mL
B. 50 mL
C. 75 mL
D. 100 mL
E. 150 mL

22. What is the MOST common cause of communicating hydrocephalus?


A. Tumors in the spinal cord
B. Subarachnoid hemorrhage
C. Leukemic infiltrates
D. Pneumococcal meningitis
E. Tuberculous meningitis

23. A 6- month- old Infant presents with a


rapid increase in head size and 3
prominent occiput .
Of the following, the MOST likely diagnosis
is
A. Dandy- Walker malformation
B. type I Chiari malformation

605
Chian i malformation
typeII fossa brain tumor
c posterior
P
l
aqueductal stenosis
- o d obese child presented with high ,
month
,< A 18 ting palpebral
" ' fissures, hypertelorism il nr .
head frontal
' ^

r0 r
° Sowing- the MOST likely diagnosis is
Ofthe ° Golabi- Behmel syndrome
jmpSOn *

A
Fragile X syndrome
B .
syndrome
C Weaver
syndrome
D - Gorlin
syndrome
E . Sotos

MOST common cause of anatomic megalencephaly?


25 What is the
A. Sotos syndrome
B. fragile X syndrome
C. benign familial megalencephaly
D. Weaver syndrome
E. Gorlin syndrome

26. What is the MOST common form of craniosynostosis?


A . Frontal plagiocephaly
B. Occipital plagiocephaly
C. Scaphocephaly
D. Trigonocephaly
E. Turricephaly

.Whatis the
cause of trigon0(
jJ
A.
-B. Premature fusion of t e doid
C. ^° ^
suture
Premature fusion of the cor \ suture re
Premature fusion of enofronta>
D.
E.
Prema ture
ture
fusion of
Prema fusion of the
the
^^^ ^
etopic s r
^ tufe

*cu ^ ^ear child presents with brachycephaly hypertelohsrt -


8. r
underdevelops
1
d orbits,

0 |
3r
Pr
°
^
Ptosis,
'0

hypopla sia of the maxilla, and orbital


606 J
Of the following, the MOST likely diagnosis is
A. Apert syndrome
B. Carpenter syndrome
C. Chotzen syndrome
D. Pfeiffer syndrome
E. Crouzon syndrome

29. Which of the following is a factor increasing the risk for


deforrr>
plagiocephaly ? ationa|
A . Female
B. Postmaturity
C. Sleep position is prone at birth and at 6 wk
D. Breast feeding
E. Tummy time < 3 times/day

30. What is the BEST treatment for a 6-month-old


infant with severe
deformational plagiocephaly ?
A. Watch-and - wait management
B. Repositioning and physiotherapy
C. Increasing tummy time
D. Helmet therapy
E . Surgery

31. Which of the following is a major risk factor for recurrence of febrile
seizures ?
A . Family history of febrile seizures
B. Family history of epilepsy
C. Complex febrile seizure
D. Male gender
E . Age < 1 yr

32 . Which of the following risk factor occurred


s has the highest incidence for
of subsequent epilepsy after a febrile
seizure ?
A . Recurrent febrile seizures
B. Fever < 1 hr before febrile
seizure
C. Family history of epilepsy
D. Complex febrile seizures ( focal)
E . Neurodevelopmental
abnormalities
607
* nth-dd infant
,
develops h gh fever f0||0w


,A « , e than 30 minutes after
vaccination s
' . je * with
-
lower grade of fever

-ssias
and hen
A tO
year of his
is
with0ut
life, atypical absences ‘ fever n

the MOST likely diagnosis is


(rfrf*
A
Iterated
'Let epilepsy with febrile seizures p|us (
syndrome
GEFS+I
‘6 (nporal lobe epilepsy secondary to mesial temporal
C- Te complex febrile seizures eros,s
sclerosis
0. Focal
febrile infection-related ( or refractory ) epilepsy ( FIRES)
E.

is the MOST frequent cause of febrile status


M What epilepticus ?
A Norovirus
5. Enteroviruses
C. Shigella
D. COVID 19
E. HHV- 6B

35. A 10-month-old boy presented with febrile seizure which lasted about 10
minutes with no clinical signs of meningeal irritation, his immunization status
was unknown.
A medical student asks you about lumbar puncture IP for this baby; your
answer should be
A. No need for LP at time being
B IP should be done because his immunization status was unknown
c LP should be done after brain ultrasound
D- LP should be done he developed another seizure within 24 hour
if
E . LP should
be done because he is a male gender

,
nt ro e n the
management of his son.
Yo r Pfoper
^ ‘
answer should be , tinn

6.
Should not be performed as part of seizures
r the future recurrence
G would predict
would predict the future recurrence o
epi
n
EG should be done after 2 wk of a febrile seiz
608
t

after secon d attack of febrile seizure


£. EEG should be done
, blood studies is routinely recommended ,
„ which of he following » th lr
t

A . Serum
electrolytes
B. Calcium
C. Magnesium
0. Complete blood count
£. Blood glucose

following nutritional deficiencies is associated with an


38. Which of the
increased risk of febrile seizures ?
A. Iron
B . Zinc
C. B12
D. B6
E. Folate

seizure with throat


39. A 5- year- old child wakes up at night due to a focal
tingling and clonic contractions of one side of the face, with drooling
and
inability to speak but with preserved consciousness and comprehension. EEG
shows typical wide-based spikes that are markedly increased in frequency
during drowsiness and sleep with normal MRI .
Of the following, the MOST likely diagnosis is
A. atypical benign childhood epilepsy with centrotemporal spikes
B . benign childhood epilepsy with centrotemporal spikes
C. benign epilepsy with occipital spikes
D. nocturnal autosomal dominant frontal lobe epilepsy
E. familial benign epilepsy syndromes

40. Which of the following is TRUE regarding typical absence seizures?


A. They usually start at 3 -4 yr of age
B . They can occur up to hundreds of times per day
C. They usually have an aura
D. They usually have florid automatisms
E * TheV usually have a postictal period
tk
_old infant presents
with epilentir
»uu
spasms fthat
'° .
fl0
A 7
'
deVe|oPniental reere« n and a high usuahy occur
rffd , ' With multifocal
,0U
, spikes EEG . voltage, slow,
rf
m
' aotic
^* 'S
6-
|lowing -

ahara svndrome
he MOST Hkely dia8nosis is

^ myoclonic infantile
arly encephalopathy
'

pravet syndrome
„ west syndrome syndrome
0
E-
. jnrvox
-Gastaut

* nCp ", *and


A5

^^
t0nic
h EEG findings secures that
occtjr 1
m ts
actable
a slow background in
sleep
wakefulnts
despite multiple therapies.
5S ‘
waves, polyspiVe
The spi
secures are
^the following, the MOST likely diagnosis is
Of
A. Ohtahara syndrome
6 . Doose syndrome
C Dravet syndrome
D. West syndrome
E. Lennox-Gastaut syndrome

43 A 3-week-old newborn presents with recurrent generalized tonic


.
seizures
that progress to status epilepticus which is not controlled by
multiple
antiepileptic drugs. The mother reports increased fetal movements in utero.
'iasina and CSF showed elevated a- aminoadipic semialdehyde and pipecolic
aod levels.
01the following, the
MOST like V
K pyridoxine dependent
epdep
*
8. folinlc
acid responsive setz
C. pyrdoxal - neonatal epi ePt,c onreohalop
cerebral folate deficiency
-
phosphate respons
' ^
E.
tetrahydrobiopterin deficiencie
44 Whi
ich of the . seliUres?
A
E.
Oxtarbazepine
following drugs \
°*
the first choice f r

C tthosuximide
Valproate
D•
tamothgine
610
E. Topiramate

45. Which of the following


drugs is MOST often used as initial treatr
" , en
?
absence seizures
A . Valproate
B. Lamotrigine
C. Ethosuximide
0. Acetazolamide
E. Clonazepam

following antiepileptic drugs may cause apnea ?


46. Which of the
A. Benzodiazepines
B. Carbamazepine
C. Lamotrigine
D. Levetiracetam
E. Phenobarbital
may cause irreversible visual field
47. Which of the following antiepileptic drugs
deficit ?
A. Benzodiazepines
B. Carbamazepine
C. Vigabatrin
D. Levetiracetam
E. Phenobarbital

48. Which of the following antiepileptic drugs may cause aplastic anemia ?
A . Benzodiazepines
B. Carbamazepine
C. Vigabatrin
D. Levetiracetam
E. Phenobarbital

49. Hepatic and pancreatic toxicity are


serious side effects in Which of #
following antiepileptic drugs ?
A. Benzodiazepines
B. Valproic acid
C. Vigabatrin
D- Levetiracetam
tin
l 63bape °
f t h e following antiepileptic dru8 s s used m
' i an
epilePtic patile
t with
3n
^'A ' 8enzod 'a ePines
* "
B; V °
lpr ic acid
3
/ jgabatrin
C ^
P Levetirac etam
n
Gabapent
l '
of the following antiepileptic drugs
51 Which is used in an
epileptic obese
patient ?
A Topiramatc
B valproic acid
Q Vigabatrin
D. Levetiracetam
E . Gabapentin

52. Which of the following antiepileptic drugs is better to be used in


i a patient
with both absence and generalized tonic-clonic seizures?
A. Topiramate
B . Valproic acid
C. Phenytoin
D. Ethosuximide
E. Gabapentin

53. Which of the following


is the safest antiepileptic drug to use during
Pregnancy ?
A - Levetirace
tam
B. Valproic
acid
C * phenytoin

^El Ethosuxi mide


Gabapentin
eekly)

^
hich of the following antiepileptic drugs reqU ' S

^° '
n t0r n

B.

'
8 of liver function and blood counts
Levetiracetam
ValProic acid
t tthe
throughout
^
h e therapy
?

612
C Felbamate
D. Ethosuximide
E. Gabapentin

55 . Which of the
following antiepileptic drugs requires giving vVi ,
tamin D
supplementation with it ?
A . Levetiracetam
B. Gabapentin
C. Felbamate
D. Ethosuximide
E. Phenytoin

56. Which of the following is an absolute contraindication to ketogenic diet?


A. Primary carnitine deficiency
B. Myoclonic- astatic epilepsy
C. Tuberous sclerosis complex
D. Rett syndrome
E. Dravet syndrome

57. Discontinuation of antiepileptic drugs AEDs is usually indicated when


children are free of seizures for at least
A. 1 year
B. 2 year
C. 3 year
D. 4 year
E. 5 year

58. Antiepileptic drug AED therapy should be discontinued gradually; often over
a period of
A. 1- 2 months
B . 2-4 months
C. 3- 6 months
D. 4- 8 months
E. 5-10 months

d flirt1
59 . Which of the following neonatal seizures is frequently not associate
electrographic discharges ?
A . Spasms
613
r clonic
1
fota/ tonic
focal myoclonic
C
0-
ized tonic
Generallized
Genera
com mon . ,f n
MOS T Caus
yfiat is the
events *onat Se/'zures?
4. vasc ular
-ischemic encenhzi Pa%
Hypoxic
0 infections
C anialorm
intracrmalf ations
Brain

.
D.
C- Metabolic disturbances
M
« 'i
ypocalcemic
the following , the MOST likely
i
cause is
Of
A hyponatremia
8. hypernatremia
C hypomagnesemia
D. hypoglycemia
E. pyridoxine dependency

«. What is the first-choice long-acting drug in neonatal seizures^


A. Lorazepam
B. Phenobarbital
C. Phenytoin
D. Fosphenytoin
E- Levetiracetam

deve ) 03^ 56 21 6
new orn is well and ready to be discharged from NICU; he
^ a or anc treate^ wit^
^nenob hita * *oa ^iog then maintenance on 5mg/kg/day* ^ twice *daily. activity
' ^ ^
^’
t Ue 0
( An EEG
(j
.
°Parents °re ^'sc ^ar8e and does not show evidence of epileptiform
6

a ou the fate of anticonvulsant and when can be stopped


,

owing°, the^MOS
0fthe f ^ U
^ T appropriate answer is that phenobarbital should be
st PPed
B. ° immediately
aPered at time of discharge
C.
Pered after 3 months
0.
* Pered after 6months
614
months
E. tapered after \ 2
f status epilepticus in children ?
64. what is the ep»«v
convulsive status
A
B febrile status
epileptics
c Nonconvulsive contmua
.
status ep lepricu
DEpilepsia partialis
Refractory status epilepticus
E.
by the American Epilepsy
agent recommended
65 What
es
Guidelin
is the
to
first
control
- line
status epilepticus ? ^
A. Phenobarbital
B. lorazepam
C. Phenytoin
D. Valproate
E. Levetiracetam

is recommended to a 1- year-old boy


66. Which of the following investigations
spells ?
presents with recurrent severe breath holding
-

A. EEG
B. ECG
C. Serum calcium
D. Serum Zinc
E. Echocardiography

67. How you treat a 1-year-old boy presenting with recurrent severe cyanotic
breath-holding spells ?
A. Education and reassurance of the parents
B. Atropine sulfate
C. Anti- seizure drug therapy
D. Cardiac pacemaker
E. Basic cardiopulmonary
resuscitation

collapse lith oss *:on! ou1esft,nee1°"


I d
SWe3ting followed by 3 gradU3
'
movement when taking a hot shower
What is the MOST likely cause of this
86
^ Standlng for 3 lon6 time wlth0°

event ?
A . Complex partial seizure

615
r I syncope
Ws0 0,a| lobe epilepsv
i- P )oced anaphylaxis
c-
Teff' inc

SS""*”*
'

»
t- with postural
boy presents
r Id , imbalance and nVstagmus,
A 5' ,° objects
ye seem to be moving toward him. he
me child aPpears
* htei, ed dUr?
6 e<l tha
jng the
episode with diaphoresis, nausea and vomitin
8 MRl and
KT „' norh1*1' u .MOST likely diagnosis is

w „ nt syndrome

-
A
0ng
°B l paroxysmal vertigo of childhood

^-
-

.C. ** m Wonderland syndrome


| CPe
E. S M
th- old infant presents with paroxysmal episodes of generalized
70. A 6
mon ‘
posturing accompanied by apnea , staring, and
opisthotonic
stiffening an These episodes often occur 30 min after a
extremities.
minimal jerking
f
^
feed. likely cause of these events?
What is the MOST
paroxysms I torticollis of infancy
A Benign
B. Exaggerated
startle reflex
C. stiff baby syndrome
0. Sandifer syndrome
E. Paroxysmal dyskinesia

71. A 4-month-old boy infant


times per day, without
presents
concurrent
about prognosis, your answer is
EEG epileptic- .=
rSr *£S
A. It persists for life support
B. It subsides when child wa Iks without
C. It subsides by 1st year of life
delay
D. It may lead to developmental .

E. It may convert to epileptic myodon


1I
complaining from rf, |
9

I hrought by her mother yth


*llc
1

' 3
perspiration, irregu is the M0ST likely cause of this event ?
Which of the
following
%
A. Tic
B. Mannerism
C. Anxiety attack
D. Infantile gratification
spell
E Hyperventilation

common presenting complaint of intramedullary Sp,


73 what is the MOST
cord tumors ?
A. Gait disturbance
B. Back pain
C. Sensory deficits
D. Scoliosis
E. Bladder disturbance

74 . Status migrainosus is defined as migraine that persists beyond


A. 24 hr
B. 48 hr
C. 72 hr
D. 96 hr
E . one week

75. What is the MOST frequent


symptom of idiopathic intracranial
hypertension ?
A. Vomiting
B. Transient visual
obscuration
,al headache
O.
E*
SrereSSiV
Pulsatile tinnitus
6 fr0n

76. What is the preferred


A . Acetaminophen
initial drUg t0 treat
m graine headache ?
B. Naproxen
sodium
'
C. Aspirin
D. Triptan

617
e lbuProfen

offhe
Which I in location

followine differentiate ten,, n tyPe
'
he )
77
r
'Focaiin
-
d by physical activity
(
dache
from grame ?
^
0 VV
0 rsenebbing quality
Montfiro
C LquentlV associated with nausea
£ Moderate to severe intensity

JS w ,
i£h of the
following neurocutaneous syn(
jrome. is X-
ltnked dominant ?
A' Neurofibromatosi 5 type 2 ( NF2)

0. Tuberous sclerosis complex ( TSC )


C. Sturge- Weber syndrome (SWS)

0, von HippeKindau disease (VHL)


£. Incontinentia pigmenti

» What is the MOST frequent lesion associated with neurofibromatosis


type 2?
A, Bilateral vestibular schwannomas
B optic nerve schwannomas
,

C, Intracranial meningiomas
0. Spinal tumors
E. Peripheral neuropathy

80 Definite tuberous sclerosis complex TSC is diagnosed when at least


two
major or one major plus two minor features are present.
Which of the following is a minor feature ?
A . Subependymal nodules
B. Facial angiofibromas (>3 )
C. Ungual fibromas ( )
22
D. Retinal
achromic patch
£ Shagreen patch

81 What is
M

the MOST common neurologic manifestation of tuberou


complex ?
^
B.
Epilepsy
Nystagmus
C
sleeP disorder
618
I
hyperactivity disorder
D. Atten tion deficit
E. Depression
therapy for infantile spasms in Tuberou Sc|
82 . What is the first -line * er sis
°
Complex T5C? hormone ( ACTH )
A. Adrenocorticotropic
8. Vigabatrin
C. Epilepsy
surgery
D. Valproate
E. Everolimus
ther brought her 6-month-old boy with infantile spa
83. A young mother OB
secondary to tube S ^ ^with good response to vigabam
. ,
his cardiac echo
^ djac

about the fate of this cardiac lesion


Your best answer should be
rhabdomyoma. The mother counsels
.

A. It usually ends with congestive heart failure


**

B. It tends to slowly resolve spontaneously


C It usually ends with arrhythmias
D. If started in fetal life, it is fatal in infancy
E. It usually needs surgical intervention

84. What is the imaging modality of choice for demonstrating the extension of
pial capillary malformation in Sturge - Weber syndrome (SWS ) ?
A. Brain MRI
B. Brain MRI with contrast
C Head CT
D. Head CT with contrast
E. Brain US

85. What is the MOST common cause of


death in von Hippel-Lindau disea
A. Pheochromocytoma
B. Renal carcinoma
C. Cerebellar hemangioblastoma
D. Hemangioblastoma of
the spinal cord
E. Retinal angiomas

ci n
A a 6no a|
^ uSsr* " >« ^,
1
a
8 dls nctive onja,
facial

^Sri ^ of the cerebellum


nesis of vermis
A- malformation
6 tn A..'Walker syndrome
c Dandy 1 diseas
®
p. Leigt
joubert
syndrome
t
. 3-year Old child presents with vomiting, truncal
'
ataxia, h ruontal
* arnUs, and dysarthria. Examination of the cerebrosp
Inal fluid i °
^^
ls normal.
varicella 3 weeks ago. The
js the MOST
likely diagnosis?
infectious cerebellitis
0 Acute
labyrinthitis
C. Neuroblastoma
0. Acute cerebellar ataxia
E Hartnup disease

88. A 6-year-old child presents with steatorrhea, failure to thrive, ataxia,


peripheral neuritis, and retinitis pigmentosa . Blood smear shows acanthocytosis,
Serum chemistries reveal decreased levels of cholesterol and triglycerides and
absent serum (5-lipoproteins.
Of the following, the MOST likely diagnosis is
A. Bassen-Kornzweig disease
B arginosuccinic aciduria
C. Hartnup disease
H Refsum disease
Friedreich ataxia

^S"'
A 12- year- which *
old boy presents with slowly progressive ataxia '^
Cities more than the upper extremities. Examination wtensor
Sl ive
Romberg test, absent ankle deep tendon re ex ' with
r«Ponse.
'
6 1 lntelli
The boy has explosive dysarthric « ** *£"
an a
* 8ence' He also has high-arched feet
Ofthef
’ Rou
Rowing, the MOST likely diagnosis is

*V-Levy disease
620
6. Ramsay Hunt syndrome
C. Friedreich ataxia
D. ataxia-telangiectasia
E . Refsum disease

yesterday morning with c umsiness. dr0pped


90. A 9- year-old girl awakened
involving
^
6
the face with crying
and
abnoma ody movement tibl
speech When the patient asked to hold the upper limbs overhead, arms *
palms turn outward. Examination of
heart is normal. ^
Of the following, the MOST likely
diagnosis is
A . benign hereditary chorea
B. Sydenham chorea
C. chorea secondary to systemic lupus erythematosus
D. chorea secondary to antiphospholipid antibody
syndrome
E. paraneoplastic chorea

91, Which of the following nutritional disorders may cause secondary


intracranial hypertension without an obstructive lesion on MRI ?
A . Nutritional rickets
B. Hypovitaminosis A
C. Zinc deficiency
D. Copper deficiency
E . Vitamin K deficiency

92. A 5- week - old infant presents with repetitive focal myoclonic jerks occurring
during sleep, more prominent in the upper extremities continued for few
seconds, these jerks began during the 1st week of life, waking the baby causes
the movements to abruptly cease . Neurologic examination is normal.
Of the following, the MOST likely cause of these movements is
A . Opsoclonus myoclonus ( ataxia ) syndrome
B. Benign neonatal sleep myoclonus
C. Benign myoclonus of early infancy
0. Hypoglycemia
E , Myoclonic epilepsy with ragged
red fibers ( MERRF )

93 . What is the MOST comm on cause of


spastic diplegia ?
A . Prematurity

621
I
»
C-
uchef*1 3
' nem a
Lerbilirubi
TombophHicdisorders
.
0-
E. biventricular hemorrhagic infarction

common clinical manifestatio


is the MOST n of burn
94,

^Altered states of consciousness encephalopathy?


g Hallucinations
c Coma
0. Seizures
S. Irritability

- year - old girl complained from rapidly progressive anxiety


9S A 14 agitation
thoughts, bizarre behavior, language disintegration
delusional , and insomnia.'
After few weeks, additional symptoms occur, including a decreased level of
consciousness, seizures, choreoathetoid movements, and autonomic instability.
Brain MRI studies shows nonspecific cortical and subcortical T2-fluid attenuated
inversion recovery ( FLAIR ) signal abnormalities, the cerebrospinal fluid shows
moderate lymphocytic pleocytosis, the electroencephalogram (EEG ) shows
diffuse slow activity in the delta and theta ranges while the abdominal
ultrasound revealed teratoma of the ovary.
Of the following, the MOST likely diagnosis is
A. anti-N -Methyl-D- Aspartate Receptor Encephalitis
B. neuroleptic malignant syndrome
C. limbic encephalitis
D. Kleine- Levin syndrome
E encephalitis
.
lethargica
96 A 7- year ive refractory focal seizures,
cognitive
rightbrain
-old child eseots with
deterioration, Presejbal greSd«.
Pf0
neurology deT
atrophy of

01the hemisphere.
following, the MOS diagnoselS
A.
fclckerstaff Encepha s
. acute
B
C. disseminated
Rasmussen
'^' a\ortWe S
encepha
^ ^'
vnEhM
/

D. Ophelia encephalit
syndrome 's
autoimmune \tntbic ha\itis
for control of seizures in Ra
treatment
the MOST effects X
97. What is
encephalitis?
steroids


A. High-dose
exchange
B. Plasma -
C. IVIG

l S5. '
98. A 15-month-
.
old
milestones beginning
a
.
chhurtd Pnresents
with progressive loss
of motor and langUa
agg wjth seizures, macrocephaly, and do .
feveals splenomegaly while fundoscopy shovvs
,
. exa
like face Abdomina brainstem responses shows prolongM
cherry-red spots, ana
me

Sf the following, the MOST


A. Sandhoff disease
likely diagnosis is

B. Juvenile 6M2 gangliosidosis


C. Tay-Sachs disease TSD
D . Krabbe disease
E. Metachromatic leukodystrophy

99. A 10-month-old boy infant presents with excessive irritability and crying,
unexplained episodes of hyperpyrexia, vomiting, and difficulty feeding,
diagnosed by GP doctor as colic with frequent milk formula change with no
improvement, now he develops generalized seizures , alterations in body tone
with rigidity and opisthotonos and visual inattentiveness as a result of optic
atrophy .
Of the following, the MOST likely diagnosis is
A. Sandhoff disease
B. juvenile GM 2 gangliosidosis
C. Tay-Sachs disease
D. Krabbe disease
E. Metachromatic leukodystrophy

100. A 2 -year-old boy has insidiou


with feeding and swallowing impairmen ^ °* ^ distur^ance / frequent falls*
ex rern es are hypotonic, and
the deep- tendon reflexes are
absent
intellectual function and dysarthric «in‘ r
* noticed
*speech
parents '** deterioration in
shows optic atrophy. .
Examination of the retina
623
>
it tion shows slowing of
iolofilC peripheral
hVs' and M ° irnages of the
brain indicate nerve conduction
r ities Ctf the cerebellar and cerebral white
- matter. diffuse
symmetric
jnon
,ngjSase
° likely diagnosis
<". " °tfGM
K
A Sa
V the MOST

dh
jgvenflc
2 gangliosidosis
is

e Sachs disease
c Tay-
Krabbe diseasemetachr0mati
.

p eUko
f. Wte
infantile
^ tr0phy

Id rnicrocephalic female with regression of


r'
A 3 Ve3
101 neS preS

'
°
ts with ataxic gait and fine tremor of
language and motor
hand movements with
ling°
lfiSt ' s and intermittent periods
reSpirat of apnea. The
'°c onic convulsions with autistic behavior.
|
patient develops
MOST likely diagnosis is
occipit3 horn syndrome
B. Rett Syndrome '
C. Menkes disease
0. Tay-Sachs disease
£. Krabbe disease

102. What is the hallmark of acute disseminated encephalomyelitis ADEM ?


A. Headache
B. Meningeal signs
C. Seizures
0. Cranial neuropathies
E. Encephalopathy

103. Which of the following drugs cause secondary intracranial


hypertension without an obstructive es VMR|?

A. Nitrofurantoin
& Ceftriaxone
C. Ampicillin
D.
Cefixime
E. Vancomycin
,head3che after a non -
104, A 9-year
-Wc viral
-old boy presents
illness, few days 3
' ^ 624 '°
deve PS
this visual ?
loss
likely cause of
What isi the MOST
A. Sarcoidosis
B. Behcet disease
C. Lyme disease optic neuropathy
hereditary
D. Leber
E. Optic neuritis
. d bv visual loss;
mo is chafaCtenze ,
„«• behavioral sensorineuralheari
105
disturbances ario

B.
C. Neurosarcoidosis
D. Susac syndrome
E. Hypoxic-ischemic vasculopathies

106. Which of the following imaging studies can demonstrate arterial ischemic
stroke ( AIS) few minutes following the onset ?
A . Brain CT
B. Cerebral MRI
C. Diffusion- weighted MRI
D. MR angiography
E . Ultrasound brain

107. Which of the following may cause


hand dominance within the first year of
life?
A. Perinatal stroke
B. Megaloblastic anemia
C. Sickle cell anemia
D. Moyamoya disease
E. Focal cerebral
arteriopathy
108. What is the MOST
common cause of
stroke? childhood subarachnoid hemorrhage
A. Hereditary
hemorrhagic
B. Intracranial aneurysm telangiectasia
C. Arteriovenous
malformations
D. Idiopathic thromb
ocytopenic purpura

625
1 'Uremic
iC syndrome
l Herno V
0f
' *"
the following CNS infections rnaV have
v\/hich the highest
&? itis
CSF Protein
^A1 tubercU *oUS menin^
ie
fneningitis
0 . Viral meningitis
Acl te bacterial
)
C
partially treated bacterial meningitis
I fungal meningitis

Which of the following viral diseases may cause low gl


110 .
simplex virus
ucose in CSF ?
A Herpes
0 Arboviruses
C Mumps
Q Adenoviruses
E. Rabies

111. Which of the following bacterial meningitis increased in children with


cochlear implants ?
A. Staphylococcal bacterial meningitis
0. Anaerobic bacterial meningitis
C. Gram-negative enteric bacterial meningitis
0. Pneumococcal bacterial meningitis
E . Pseudomonas aeruginosa bacterial meningitis

112. What is the MOST frequently identified pathogen of bacterial meningitis in


children beyond the neonatal period ?
A Streptococcus pneumoniae
Neisseria meningitides

^
Haemophilus influenzae Type b
Staphylococcus aureus
E. pseudomonas
aeruginosa
m
113. What that cause brain abscesses
child are the predominant organisms
ren ?
g StaPMococcus aureus
c'
^
laerr Ophilus spp.
' coli
Escherichia
626
i

« •>~Zp ,
a
l ** s '*
Streptococcu ang no

f the following tests can ,


differentiate between bacteria|
114. Which 0
nS
meningitis ?
A C-reactive
protein
rate
sedimentation
B. Erythrocyte
C Procalcitonin
0. Blood culture
E. WBC count

recommended treatment duration for N. mening


115. What is the
meningitis ? ^
A. 7 days
B. 10 days
C. 14 days
D. 21 days
E. 28 days

116. Steroids reduced hearing loss in children with meningitis due to


A . Streptococcus pneumoniae
8. Neisseria meningitides
C. Haemophilus influenzae Type b
D. Staphylococcus aureus
E . Pseudomonas aeruginosa

117. What is the MOST common neurologic


sequelae from bacterial meningitis ?
A . Hearing loss
8 - Cognitive impairment
C. Recurrent seizures
D. Delay in acquisition of
language
E- Visual impairment

118. What is the


MOST common CNS manifestation of Varfcella-zoster
infection?
A . Acute encephalitis
B. Cerebellar ataxia
C. Deafness

627
MCCid paralysis
0 - eptic meningitis
f
is a recognized
ne3 fness
- ,ons. - COrnplic

i
* tion
1

A# polio
A ^ich of the
Measles foil »
B
, Mumps ° ing ,
v rai
D Rubella
[ Herpes simplex virus

Which of the following v/ra /


encePhaii( is
^

and cell counts?
4 Herpes simplex virus
g. Parecho viruses
C. Epstein -Barr virus
maVhave n
°rrn* l CSf
^coSet
p. Cytomegalovirus
E. Mumps

628
Chapter 26
The Nervous System
Answers
ZUHAIR ALMUSAWI
approach the child slowly, reserving a
1.{C) The physician should
painful, or discomforting tests
for the end of the examinat0 ,
measurement of head circumference , gag reflex). fe.$ °
L( A) Although not a routine
component of the examination, smell can be test
reliably as early as the 32nd wk of gestation
by presenting a stimulus
observing for an alerting response or withdrawal, or both. Care should be taker
to use appropriate stimuli, such as coffee or peppermint, as opposed to strongly
aromatic substances (e.g., ammonia inhalants) that stimulate the trigeminal
^
nerve, Each nostril should be tested individually by pinching shut the opposite
side.
3.(B) Evaluation of vision in the premature infant presents unique challenges. At
28 wk of corrected gestational age, a premature infant blinks in response to a
bright light, and at 32 wk, the infant maintains eye closure until the light source
is removed. The pupil reacts to light by 29 - 32 wk of corrected gestational age;
however, the pupillary response is often difficult to evaluate, because
premature infants resist eye opening and
have poorly pigmented irises. A
normal 37-wk infant turns the head and eyes
toward a soft light, and a term
infant is able to fix on and follow a target, such
as the examiner 's face.
4.(A) The oculomotor nerve
innervates the superior, inferior, and medial recti,
as well as the inferior oblique and
Complete paralysis of the
levator palpebrae superioris muscles.
oculomotor
displacement Of the eye
and elevation.
outward and downward“^ °' **
and impairment of adduction
5.(E) Ocular bobbing is
characterized by a downward jerk followed by a slow
drift back to primary position
and is
describes involuntary, chaotic, associated with pontine lesions Opsoclonus
conjugate oscillations
often seen in the setting of of the eves which are
neuroblastoma
6.(A) Unilateral Injury of the vagus or viral infection
nerve results in
soft palate and a hoarse voice; f the ipsilatefal
bilateral lesi
as a result of vocal cord paralysis, as esions can °
respirat rY distresS
as nasal reg
°
8Urgitatlon of fluids, pooling
629
r

isolated esions fo
«i *
ne
th type 11
chian malformations. s or '
may be
seen
the

^
jn
>fj' of aseptic ted polymorphonuclear count 5ugge5ts b
> l lymphocytosis can .K1JMeningitis or the
meningitis. CSF

< , ^l -
or fungal meningitis; demyelinating disease
no °logiC disorders including collagen
,
" ( following myelogram, intrathecal method *"
irritation
«* L CSF protein falls to the normal childhood range bv ,ate .
3
in ase
0r

)
s ^
al cord
and

* lutein
- Cl maV be elevated in many processes ,11 ° a«e °'
|

nologic. vascular
and degenerative

^ diseases; blockage of
, ous ,

^ nrs of the brain ( CNS tumors, systemic tumor


primary
* ^ ; as well

Cf^infiltrative
‘ r
acute lymphoblastic leukemia ) and
protein is increased
spinal
trailmatk
: , the CSF
tap by approximately 1 mg/dif every
* !
-00 red blood cells/mm3.
vef

cranial ultrasonography is the imaging method of choice


9 iB ) for detecting
hemorrhage , periventricular leukomalacia, and hydrocephalus
,ntracranial in
anterior fontanels.
uf^ts With patent
10.( E) CT or MRI
can detect infarcts more than 24 hr old, although MRI is
genera Hy preferred to avoid exposure to ionizing radiation.
11 ( E) MRI with and without gadolinium is generally preferred because it
provides a more detailed view of the anatomy without exposure to ionizing
radiation.
11(A) Proton MR spectroscopy ( MRS) is a molecular imaging technique in which
the unique neurochemical profile of a preselected brain region is displayed in
the form of a spectrum. Many metabolites can be detected, the most common
,
of which are N -acetylaspartate, creatine and phosphocreatine. choline
areai ca
myoinositol, and lactate. Changes in the spectral pattern of a given
>
Yield dues to the underlying pathology, making MRS useful in the djagnc
mborn errors of metabolism, as well as the preoperative an ° of cortical
as
fcsessment of intracranial tumors. MRS can ^ ar have low N-
Aplasia jn patients with epilepsy, because these patients
.
^Cetylaspartate:creatine ratios. general categories
,e paroxysmal
^^ KG abnormalities can be divided nt0
Ptiform discharges and slowing. EpHePtl^orrT
or sharp waves, often followed by »
1

*
' ges 3

vvhich n
,errupt t !
*£*’„ gener ized „ Focal

^
., ground activity. They may be focal,
d sch
u

arBes are often associated with cerebral OY


630
^ esiS or irritative
s

•"•‘""ST
,
* *•"
JJJjJ
14(A) Occult sP
na1 « (
0f the
is 0fre
spinal
SOciat 1 wl h mu
"“ "
,
cord,
,
including SyriJ
. ,.
%
' Pmtilia .° de a and/
fST 2?
-^
deVe 0
diastema tomyelia P '
f3ttV
show bone defects or may be
°Ka
norf )a|
|
ord A
'

-rav *
spine x

e
MSI s , *^ •?,
are best investigated with MRI. Initial screeri
» H ir
after one affected child is 3-4% and
Sei risk recurrence
of
affected children.
with two prior
increas
*s to
1SIC1 TO be effective, folic
acid supplementation should be initiated bef0re
the 12th wk of gestation,
conception and continued until at least when
which folic acid
neurulation is complete. The mechanisms
by prevents
NTDJ
remain poorly understood.
.
17 (D) The United States Public Health Service recommends that all women of
childbearing age who can become pregnant take 0.4 mg of folic acid daily, |f (

however, a pregnancy is planned in high-risk women (previously affected child),


supplementation should be started with 4 mg of folic acid daily, beginning 1 mo
before the time of the planned conception.
.
1S (A|
.
19 (B| Aicardi syndrome represents a complex disorder that affects many
systems and is typically associated with agenesis of
the corpus callosum,
distinctive chorioretinal lacunae, and infantile spasms. Patients are almost all
female, suggesting a genetic abnormality of the X
chromosome (it may be lethal
in males during fetal life)
. Seizures become evident during the first few months
antic nvulsants. An electroencephalogram shows
indeDendent »rt- ’i** *5 301
°
fr m b th hemispheres
° ° « ^ result of the absent
anTcan
intellectual disability
^
a rbhVPSarrhythmia
" ' A patients have severe

-^ , ^'
only partially
circumscribed pits or lacuna , Abnorma lties of the retina, including

«MA) If the cause of the


a
^
•«
» are the most

phenylalanine level should


bedeterml n Unknown th mother’s serum
'
u
®
an asymptomatic mother can
normal nonphenylketonuric
infant.
L '?
produce
"Vlalanine phe
serum levels in
' brain damage in an otherwise
r
tot
,®volume of CSF approximates
5 o fl
in a
i« ft
* hitructive or communicating , " infant and ISO
/I2 '1.) ^a r rachnoid hemorrhage
Jf jjlt- ( vd
"Itta

'

^ , which is
subn a premature infant. Blood in the
Crr f the cisterns or arachnoid villi and obstTuc? °’ *«
m t
^ m
suh * ^ 'ntra "' °nlV
0)
t

^
^
n d Sp

Slftf ^ °cledevelopmental
in the posterior fossa and midfine cerZ* * • •> or .
failure of the roof of , " "*
>f fa
th
' a he which
>2 '" " siS.
With
tS preS6nt PW 3
" increase K *d"s«*« ini4
1

^
f nt 0CCiPut-
syndrome (cerebral gigantism) is th(,
j

^ common
'
00 phalic syndrome, with 50% of patients having
n„
lenc _
of patients having macrocephaly by age i yr. tartyacrotephaty
^ 2* formalizes by adulthood. postnatal

familial megaleneephaly is easily diagnosed by a careful family


^Terences
#K
*
Benign
J
measurement of the parents' head tircumfefences (occipitofrontal
).
-circu ,
26,1
premature
or
closure of t he sagittal suture produces a long and narrow
scaphocephaly, the most common form of craniosynostosts.
*skui
wl
"'
phaly is associated with a prominent occiput, a broad forehead, and a
C3 P
|| or absent anterior
fontanel. The condition is sporadic, is more common in
ifn 3
causes difficulties during labor because of cephalopetvic
males, and often not produce increased ICP or
disproportion ion Scaphocephaly does
neurologic examination of affected patients are
hydrocephalus, and results of
normal. caused by premature
27,(D) Trigonocephaly is a rare
form of craniosynostosis
fusion of the metopic suture. These
children have a keel-
shaped forehead and
risk for associated developmental abnormalities of the
hypotelorism and are at
more common .
forebrain. Milder forms of metopic ridging are a(|d s
2 (£|Crouzon syndrome is characterized
- by premature
depeM)s on
^
*inherited as an autosomal dominant trait The .
shape o
en
^^ to back- -
the timing and order of suture fusion but most o 0f the coronal
front diameter or brachycephaly resulting from i 3
is is prominent
toSjS
.
CU
^ res. The orbits are underdeveloped, an 0 e epical facial feature ^
P plasia of the maxilla and orbital hypertelorism

^° Actors That Increase the Risk for Deforma


/
Male
i
piagiocephalY

632
congenital torticollis)

t birth
3 ^
6

y aChievernent
y gott e lee
'
lalVg be s,an
^ £
/Tum ac# W
y lo^e
r
ith
*
a to **" 3110"
^5 0Ce
' ett
esthe
** efordr |es ug6 i n u e 0
therapy
mo
should
^
Parentssh
*

30

°
y 5\ eeP'° ,* th se**
aPV
, can the com
rS ganit' »\edootne
5
at a°V

, "
^

1
'
Stlld
and
i W Per
3 m °
*
« d in
ihVoW n liarlce
„pr daV N °
kittle 35c A mo
.
^^
this tre
has
because helme,
documented „

g0% s
31.1e
'
MAIOR Age aV
of .«*
r
n\ ever ^
v <
y Durati
y fever 38
-39 ° -102 -
QO .4
2f
'izures
, e se
M1N08 . . tory of fet>
r
'^
y famf V h
y
' '
of epifeP
,
; rn V '
fa
Te '.
5
2
febrile seizure
y Comp e* '
DaVcare of Prese ntation
y Male gender sodium at time
y LONVer serum )
32 1- «y Simple febrile
seizur<" 1
%
recurren t wit Pm ^
in 1

se es 4% duration or
y
y
Recurrent febrile
Comp eX febrile
6%
y fever < \
'
hr before
18%
^^ res
febrile seizure
5 min in

11%

y
seizures and
y Comp ab
alopd *'13 "°
" M had ’ prolohgdd febrileto have suffered
y Neurodevex
majority
d
33.1« The after vacci
encephal°Pathv 633
pncephalopathy (seizures and psychomotor regression occurring
-
S
and presumed to be caused by it) turn out to
have Dravet
Mutations » the
, indicating
vaccine .
that
This
their
has
disease
raised doubts
is caused by the mutation
about
and
the very existence of
potse ternled vaccine encephalopathy
£0 ,
0t
the e ' tVbrjie seizures often occur in the context of otitis media; roseola and
herpesvirus (HHV) 6 infections; and infections with norovirus,
hunisn , Shigella , or similar agents, making the evaluation
enteroviruses In patients with febrile status epilepticus, HHV-6B
more
(more

^
J and HHV -7 infections account for 30% of the
uentW cases.
ingitis should be considered in the differential diagnosis,
35,(B) Men and lumbar
performed for
puncture should be all infants younger than 6 mo of age
who
present with fever and seizure , if the child is ill-appearing, or at any age if there
are clinical signs or symptoms of concern. A lumbar puncture is an option in a
child 6-12 mo of age who is deficient in Haemophilus influenzae type b and
Streptococcus pneumoniae immunizations or for whom the
immunization
status is unknown . A lumbar puncture is an option in children who have been
pretreated with antibiotics.
J6.( A ) If the child is presenting with the first simple febrile seizure and
is
otherwise neurological healthy, an EEG need not be performed as part of the
^
evaluation. An EEG would not predict the future recurrence of febrile seizures
or epilepsy even if the result is abnormal. Spikes during drowsiness are often
seen in children with febrile seizures, particularly those older than age 4 yr, and
these do not predict later epilepsy. EEGs performed within 2 wk of a febrile
seizure often have nonspecific slowing, usually posteriorly. Thus, in many cases,
if an EEG is indicated, it is delayed until or repeated after more than 2 wk have
Passed. An EEG should, therefore, generally be restricted to special cases in
which epilepsy is highly suspected, and generally, it should be used to delineate
the type of epilepsy rather than to predict its occurrence.
, and
37.(E) Blood studies (serum electrolytes, calcium, phosphorus, of a
a complete blood count ) are not routinely recommen e
measured
child with a first simple febrile seizure . Blood g uc
* e
° ^
oral intake
obtun ation
initially and with prolonged postictal
(prolonged fasting). febrile seizures. and

-
risk of
with an increased
me
Iron deficiency is associated appr 0priate .

SSSSSssssss 634
„ ^kes up at night due to a focal seizure with
drulingtanTlnfbility to speak 'bw Of
one

secondary are markedly increased in frequentaser, .


cen
sleep. MRI is norm al. Patients respond very well to
antie o„"r
rt
ITlAEDs ) such as oxcar bazep ine and carbamazepme. In some patient,
might be h
°
ol have rare and mild
Typic al absen ce
seizures
seizur es
, treatm ent not neede d.
usually start at 5-8 yr of age and are often
o ,
40 (B)
to their brevity, overl
ooked by parent s for many month s even thoug h they c ,
per day. Unlike focal seizures with impajrM
occur up to hundreds of times
for only a few seconds, and are
awareness they do not have an aura, usuall last
y
accompanied by eyelid flutter or upward rolling of the eyes but typically not by
the usually more florid automatisms seen in focal seizures with impaired
awareness (absence seizures can have simple automatisms such as lip smacking
or picking at clothing, and the head can very minimally fall forward). Absence
seizures do not have a postictal period and are characterized by immediate
resumption of what the patient was doing before the seizure. Hyperventilation
for 3-5 min can precipitate the seizures and the accompanying 3 - Hz spike-and-
slow- wave discharges.
41.(D) Patients with cryptogenic West syndrome have normal development
before onset, whereas patients with symptomatic West syndrome have
preceding developmental delay owing to perinatal encephalopathies,
malformations, underlying metabolic disorders, or other etiologies. In males
West syndrome can also be caused by ARX gene mutations (often associated
with ambiguous genitalia and cortical migration abnormalities). West syndrome,
especially in cases of unknown etiology (cryptogenic cases, i.e., cases that are
not symptomatic of a metabolic or structural brain disorder ), is a medica
emergency because a delay in diagnosis of 3 wk or longer can affect the long
term prognosis. The spasms are often overlooked by parents and by physicians,
being mistaken for startles caused by colic or other benign paroxysma
syndromes
42.(E) Lennox-Gastaut syndrome typically starts between the ages of 2 and 10
yr and consists of a triad of developmental delay, multiple seizure types that as
a rule include atypical absences, and myoclonic, astatic, and tonic seizures, as
well as specific EEG abnormalities. The tonic seizures occur either m
wakefulness (causing falls and injuries) or also, typically, in sleep. The third
635
waves.
s» - *
s olyspike
?:st "
SflSsrs
.
rn

^
W
' ar
difficult to
flmonly have

JS JJ** "'
^^^
®1 multiple therapies. Some, but not and i
pite patients
tractable
develop West syndrome, and *
syodrome, then Progress tostart with
Pt 3
drome . Lennox-

:
(

g
| :=SSS , generalized tome seizures, and myoclonus. Seizures proeres n
^
of
r seizures
mot° epilepticus if no pyridoxine is used. Diagnosis is
status confirmed by
plasma urine , and CSF a
presence of elevated
, -aminoadipic semialdehyde
and
and CSF pipecolic acid levels The presence
elevated plasma * of either
or compound heterozygous mutations in ALDH7A1 alleles {which
h0n
J
^
ene the protein
antiquitin) confirms the diagnosis. The use of pyridoxine 100
mg daily orally {
higher doses, up to 500-600 mg/day, have been used) or
mtravenously helps stop the seizures. Mutations of the PROSC gene can also
cause pyridoxine dependent epilepsy.
for focal seizures and epilepsies are
44 (A) In general, the drugs of first choice
oxcarbazepine and levetiracetam.
45.(0 Absence seizures are most often initially
. .
treated with ett osux mide,
; both are more effective
which is as effective as, but less toxic than, valproate
valproate ) *
than lamotrigine {which has fewer side effects than
46.(A) Benzodiazepines side effects:
neurotoxicity { drowsiness sedation, ataxia ),
V Nuisance: dose-related
hyperactivity, drooling, increased secretions
^ Serious: apnea
47.(C) Vigabatrin side effects:
/ Nuisance: hyperactivity
retinopathy that requires
^ Serious: irreversible visual field deficits
follow-up
-

frequent ophthalmologic evaluations a nd


gain, ausea:
Carbamazepine side effects:
Nuisance: tics, transient leukopeniaia; hyponatremia
ia, weight "
,
anemia
dizziness i5, aplastic
agranulocytosis
* Serious: Stevens-Johnson syndrome
liver toxicity
^ 1®) Valproic acid side effects:
*

636
tremor, alopecia,
weight gain; hyperammonemia menstrUai
/

/
Nuisance
irre8Ul
" Serious
presence
SO.(B) The
,
of
OT
.
- tiepatlc ndpantJMtlctonicity
:
patient with epilepsy can lead
migraine in a to the
S-
_ is effective against both conditions, such S ValProjlt
that
of a medication
topiramate, or zonisamide
.
, a medication such as valproate might b
51. ( A) In an obese patient
that decreases the appetite, such as ac
be av
°^
and a medication
i topiramate
zonisamide, might be used instead.
.
52 (6) In ai patient with both absence and
generalized tonic -clonic
seizures
drug that has a broad spectrum of anti - seizure effects, such as lamotrigine J
valproate, could be used rather than medications that have a narrow spectr J
of efficacy, such as phenytoin and ethosuximide.
.
53 (A) Based on available evidence, levetiracetam and lamotrigine are FDA
pregnancy category C drugs and probably the safest AEDs to use during
pregnancy .
.
54 (C) One exception requiring frequent (even weekly) monitoring of liver
function and blood counts throughout the therapy is felbamate, owing to the
high incidence of liver and hematologic toxicity (1in 500 children under 2 yr of
age with complex neurologic disorders who are taking the drug).
.
55 (E) Other potential side effects are rickets from phenytoin, phenobarbital,
primidone, and carbamazepine (enzyme inducers that
reduce the 25-
hyrdroxyvitamin D level by inducing its metabolism).
.
56 (A) The diet is absolutely contraindicated in carnitine deficiency (
primary);
carnitine palmitoyltransferase I or II deficiency; carnitine translocase deficiency;
fi-oxidation defects; medium-chain acyl dehydrogenase deficiency; long-chain
acyl dehydrogenase deficiency; short chain acyl
- dehydrogenase deficiency;
long-chain 3-hydroxyacyl-coenzyme A deficiency; medium
coenzyme A deficiency; pyruvate carboxylase deficiency;
-chain 3- hydroxyacyH

57.(B Discontinuation of AEDs is usually indicated when


> children are free of
seizures for at least 2 yr. In more severe syndromes, such
as temporal lobe
epilepsy secondary to mesial temporal sclerosis, Lennox-Gastaut syndrome
or
severe myoclonic epilepsy, a prolonged period of seizure
freedom with
treatment is often warranted before AEDs are withdrawn
is
withdrawal
self-limited ( benign)
at all.
1 epj|ep
.
° as short as 6 mo.
e,nPted
sV>dromes
be the
Xn
0ften
therapV should be discontinued gradual 1
Often over a [
*"»* or
^
-
(0 A
'
? brupI discontinuation can result in
withdrawal ei ure Of

sgr
and
srsMiwar
as a rule, associated with electrographic
* * * i r «ati
^
)

m
' •! !? * * ( p Wt
sei urC i whereas motor automatisms and subtle, geneiah
*
sd* J * '!, myoclonic episodes are frequently not associated with
^ultifoca thoo8ht Ph
2£ and

3
thus 3
P
A

rather
^ than true "”"‘
S
. HVPOWC
ml -ischemic encephalopathy is the common cause
most of neonatal

56 1
, L accounting for 50 60 of
- % patients
. Seizures secondary to this
halopathy occur within 12 hr of birth.
Hypomagnesemia(<1.5 mg/dL ) is often associated with hypocalcemia
6fllC)
' in infants of malnourished mothers. In this situation, the
and occurs particularly to calcium therapy but respond to intramuscular
seizures are resistant of 04.
magnesium, 0.2 mL/kg of a 50% solution MgS
62 (B) Phenobarbital is
considered by many as the first-choice long-acting drug
in neonatal seizures. Whether
to use a benzodiazepine first depends on the
is 20 mg/kg. If this dosage is not
clinical situation. The usual loading dose a cumulative
effective, then additional doses of 5-10 mg/kg can be given until
dose of 40 mg/kg is reached.
the time of discharge does not show
63,(8) In general, if the EEG before .
d 0t thal lime .

evidence of epileptiform activity, medications delayed until several


decision is usua y
If the EEG remains paroxysmal, the
months after discharge.
M.(B) ommend using elther
65-(B) The American Epilepsy Society SE Guideline midazolam » *
or muscular
intravenous lorazepam, intravenous diazepam, use "
first-line agent. Less evidence supports the
phenytoin
^
/f«P
first-line
agents "
Phenobarbital
, valproate, or levetiracetam warranted
.
Additionally, in some infants, a trial of PVr 0* _., js usually at is needed
- B) Education and reassurance of the
(
because these episodes are, as a rule, se
P® '
^
'
. and are outF **
0 1 *^
within
cardiac
a

.
1
elect
and for "
spe|is are
few vears. However, screening f r
.
lsturbances with an electrocardiogram is
° *%£!**** *
* **
can rarely be the presenting s »
worsened by iron-deficiency anemia and @n of
long-QT syndromes.
.
67 (A) Education and reassurance of
the parents is usually all that is
and are outgrown needeiJl
because these episodes are, as a rule, self -limited
within J
few years. g
.
68 (B) Vasovagal ( neurocardiogenic
long
) syncope
time
is usually triggered
without movement , hot showers,
by
dehydration, heat, standing for a
the sight of blood, pain, swallowing, vomiting, sudden exposure to cold as with
cold water immersion, and a sudden episode of stress. The history is usually the
clue to distinguishing syncope from epileptic seizures: there is initially pallor and
sweating followed by blurring of vision, dizziness, and nausea and then a
gradual collapse with loss of consciousness.
69.(C) Benign paroxysmal vertigo of childhood is a common migraine equivalent
that consists of brief seconds-to-minutes episodes of vertigo that is often
accompanied by postural imbalance and nystagmus. It is important to note that
vertigo does not always refer to a spinning motion; it can also refer to a
backward or forward motion ( vertigo titubans ) where children sometimes
report that objects seem to be moving toward them. The child appears
frightened during the episode. Diaphoresis, nausea, vomiting, and, rarely,
tinnitus may be present. Episodes usually remit by 6 yr of age. MRIs and EEGs
are normal, but caloric testing, if done, can show abnormal vestibular function.
Diphenhydramine, 5 mg/kg/day (maximum of 300 mg/day ) may be used for a
cluster of attacks. Preventive therapy with cyproheptadine may rarely be
needed for frequent attacks.
70.(D) Gastroesophageal reflux in infants may cause paroxysmal episodes of
generalized stiffening and opisthotonic posturing that may
be accompanied by
apnea, staring, and minimal jerking of the extremities. Episodes
often occur 30
min after a feed. In older children, this syndrome
manifests with episodic
dystonic or dyskinetic movements consisting of
laterocollis, retrocollis, r
°
torticollis, the exact pathophysiology of which remains elusive
71.(C) infantile head atonic attacks consist of repeated
, head drops hundreds to
thousands per day usually appearing at 3 -6
mo of life and spontaneously
subsiding by the 1 st yr of life, without concurrent EEG epileotic activity
72.(0) Pleasurable behaviors similar to masturbation may occur from infancy
onward, and may consist of rhythmic rocking movements in the sitting or lying
position or rhythmic hip flexion and adduction. Infantile gratification
more common m girls,
(masturbation), which is usually 0CCurs at 2 -3 vr of a«e
and is often associated with perspiration

639
, irregul3r breathjng
JgJj (lg but
iousness . Occasionally this is associated
„f cons £l with child
abuse with
100 ' hopathologv -
Vf m related to the level of the tumor js a COmm0 presen
^
73.(B> B . |t is likely that this pain will awaken the child
progresses .
fromT" P" «ng
improve

*1& _ headaches may still be migraine but a »


**tlLe,ond 72 hr is classified as a variant termed status
prolonged
frequent symptom is chronic (weeks-tamonths). p,”"

that

-jS
P*
(C) The
most
headache that
may worsen with postural changes or a
vZ£
JjSprefcnoften
has been we documented be effective a dose 7 S-
" ; however acetaminophen 15 mg/kg can be
preferred ,
to at
(
of . 10 0
)
and is
e kg to NSAIDs.
Active in those with a contraindication
Migraines are typically moderate to severe, are focal in location, are
77 (C)
by physical activity or limit physical activity, and have a throbbing
worse ned
-type headaches TTHs
are mild to moderate in severity, are
quality , Tension
, are not affected by activity (although the patient may not
diffuse in location
), and are nonthrobbing (often described as a constant
feel like being active , photophobia, or
frequently associated with nausea
pressure). TTH is much less than one of these at a time
or
more
phonophobia and is never associated with
with vomiting.
im
79 (A)
Bilateral vestibular schwannomas 90-95%
Other cranial nerve schwannomas 24-51%
intracranial meningiomac d5-S8%
Spinal tumors 63-90
Extramedullary 55-9
Intramedullary 18-53%
Peripheral neuropathy P e6%
»W ^
Minor Features of Tube " Sclerosis Co ie*
Cental enamel pits *
Intraoral fibromas 1 2)
^Confetti ^
tinal achromic patch
skin lesions
^Multiple renal
nrenai hamartomas
cysts
neurologic manifestations of tuberous sder
The most comm on cognitive impsirment, and autism s
81.(A)
of epilepsy infantile spasms
complex TSC consist during infancy with and
TSC may present pattern.
disorder .
electroencephalogram infantile spasms . Adren
hypsarrhythmic therapy for
is the first-line with vigabatrin fails.
82.(B ) Vigabatrin be used if treatmen t
( ACTH ) can
tropic hormone children with TSC have cardiac rhabdo
50 % of
83.(8) Approximately in the fetus by an echocardiogram, usually by 20
° ^
which may be detected omas may be numerous and located throueh
gestation. The
rhabdomy
they can cause congestive hea t r
^**
, and although
ventricular myocardium minority of patients, they tend to slowh resotve
'
^
and arrhythmias in a
spontaneously.
84.(B)
related disorders are autosomal recessive
«
864 Joubert syndrome
marked by
and
developm ental delay, hypotonia, abnormal eye
disorders
, abnormal respiration s, and a distinctive malformation of the
movements
molar tooth sign" on axial
cerebellum and brainstem that manifests as the "
MRI.
Children 1-3 Yr of age and is a
diagnosis"of'exclusin. ^ ^“
condr

represent an autoimmune response tn th


0

f*
'", "*
3
varicella virus, coxsackievirus, or echovirus Me t 0n bY 2-3 wk . It is thought
3gent affectinS the cerebellum
to
.

SS- fA) Abetalipoproteinemia ( Bac^ n rn Weig disease) is an autosomal
*
recessive disorder caused by a mutatinn 6 microsomal triglyceride transfer
steatorrhea and failure to

^
P ein (MTP). This disorder begins in ch IHK
|
^
Jnve . A blood smears shows ^ ,- h consists of spicuiated red
7 ° ^ ,
-
SS
olesterol and
ZeT Serum che tnes re e
^ »Poprotetr MSed ' ^ r
eVe S of

SS Sdha *ataTSlsr ' "


t abSent S6rum P-
*
disability
89 (C)
^ °
^ 0 08 signs
Bmentosa> peripheral neuritis
intellectual
n and vibrotion sense m
' UScle weakness, and
,

V ic
* streptococcal infe
Peak incidence is at age 8- 9 yr, with a female
predominance of 2:1- ^ 1

641
Tttf
,oll0
nutritional disorders can cause
thout an obstructive lesion on MRI secondary j
; 'itracraniai
^ ^” ' /
ens
°
afflin° 'ssA
HVP „ A intoxication
Vita '0 ntation in malnourished patient
' "rdependent rickets
HVPera
vita*1’10 Qnata) sleep myoclonus begins
j
ne during the 1st week
0enign
£ c* 2nd mo , and is usually gone before 6 of life,
dim,wishes
"
1
themoS
in
t Vitcely to occur during quiet (non-
, ,emen!S a* ,n ait sleep stages.
?
months of age. The
REM) sleep, but have
^ describeddipieB
p , ia is bilateral spasticity of the legs that is greater than
1
spastic in the
^arm Spastic dipleg13 v associated with damage to the immature white
trong|
5* '
period of immature oligodendroglia between 20
matter
gestation.
^ -

34 wk of the most common clinical manifestation of bum


are
94.(0) Seizures .
encephalopathy, but altered states of consciousness, hallucinations, and coma
rnn

may also occur.


9S.(A ) Anti-N -Methyl -D- Aspartate c
eptor Encephalitis
Recent is considered rh .
autoimmune encephalitis after acute
second most common cause .
disseminated encephalomyelitis in children and adolescents
inflammatory encephalopathy characterized
96.(C) Rasmussen encephalitis is an
refractory focal seizures, cognitive deterioration, and focal
by progressive
gradual atrophy of one brain hemisphere.
neurologic deficits that occur with
- yr old, although
adolescents
The disorder frequently presents in children 6 8
adults can be affected . .
^herJs the seizures is functional
of
The most effective treatment for control of the affected
W c cons*sts of
surgical disconnection
^' ^ levels
by finding deficient by 3
6 ,a nos s ,
Sandhoff disease is established. Children usud
sually die
of hex ^
aminidases A and B in serum and
leukocytes
HEXB gene located on
Vrofa , f f disease is caused by mutations in the
chroS andh°
0sorne 5ql3.

^ ID.fE))

JOO , ranging from


changes m
< f AOEM
^ (E) En Pba 0Pathy is the
* hallmark
^ avjora(
^ Persistent irritability to coma
.
103.(A) The MRlTletracyclines, SuTfonTnlid "Slfc
without an obstruaiv
acid, Fluoroquinolones,
Cytarabine, Cyclosporine
Oral contraceptive
.
pills/
teroid therapy and withdrawal,
^implants.
104 (E) The typical presentation
^
Nitrofurar lr
^
phenytojn Mesalamine, Isotretinoin, Antioa *tta

of optic neuritis is unilateral or bilater


loss over hours to days, abnormal
color visioni, visual field loss, and som r
relative afferent pupillary defect The
visual loss can be quite severe,
majority of children at 20/200 VA or worse.
Periocular pain or pain with ^**
movement and, at times, a headache are common features. *
.
105 (0) Susac syndrome characterized by all mentioned features with
*
following MRI findings ( Focal and small lesions in supratentorial and
infratentorial regions (both white matter and gray matter ); involvement 0f
corpus callosum (snowball lesions ); leptomeningeal enhancement ).
106.(C) Diffusion-weighted MRI demonstrates arterial ischemic stroke (Ais!
from minutes to 7 days following the onset; MR angiography can confirm
vascular occlusion and suggest possible arteriopathy. Diffusion-weighted MRI
can also demonstrate wallerian degeneration in the descending corticospinal
tract, which correlates with chronic hemiparesis.
.
107 (A) Hand dominance within the first year of life is abnormal and may be the
result of perinatal stroke.
.
108 (C) Arteriovenous malformations are the most common cause of childhood
subarachnoid and intraparenchymal hemorrhagic stroke and may occur
anywhere.
.
109 (A) CSF protein (mg/dl) in tuberculous meningitis is
100-3,000; may be
higher in presence of block.
.
110 (C) CSF glucose generally normal; may be
decreased to < 40 in some viral
diseases, particularly mumps (15-20% of cases).
.
111 (D) The risk of pneumococcal bacterial
meningitis was historically increased
by more than 30-fold in children with
cochlear implants, though advances in
implant design have reduced this risk.
112.(A) Although the incidence of pneumococcal
meningitis has been reduced,
.
S pneumoniae remains the most frequently
identified pathogen of bacterial
meningitis in the United States and in other
countries that have adopted similar
vaccination strategies .
.
113 (E) The predominant organisms that cause brain
abscesses are streptococci,
which account for one third of all cases in children, with
members of the

643
ruS
anginosus group (S anginosus. Strep OC0fr ,
,ePtoC rus intermedius
) being the most
common strep, “ Conste «us
' and

0*en and Should not be used to routinely determine whirh ««and viral
.
^
^
U^AI
^ rfrobials
<«£«** currently, the
.
recommended treatment duration for
meningitis is 10 14 days with a rd-
„iae
- 3

uncomplicateds
generatlicn .

pr umon enicillin ( 300 000 400 000 units/kg/day divided JJJtjj?" *


|dC

,„,ravenouS sensitive isolates or


£ , - , ,
P

^
, vancomycin
if the isolate
resistant is
V and cephalosporins. For N. meningitidis meningitis, the
idllins
pded treatment duration is 5 - 7 days with intravenous penicillin

( 300,000
units/kg/day ) for strains with a minimum inhibitory concentration
f penicillin < 0.1 pg/ mL, or ceftriaxone for strains with an MIC of 0.1- 1
IMIC ) 0
Uncomplicated H. influenzae type b meningitis should be treated for 7-
oofmL
days with ampicillin for p-lactamase-negative strains, or a 3rd generation *

10
halosporin for (3- lactamase-positive isolates. Gram-negative bacillary
least 2 wk after CSF sterilization,
meningitis should be treated for 3 wk or at
which may occur after 2- 1 0 days of treatment
.
in meningitis treatment, steroids
116. ( C) In a Cochrane review of using steroids
to H. influenzae type b but
reduced hearing loss in children with meningitis due di not re uce
. The use of steroids in children
not due to other pathogens
mortality rates?
the most common sequela of bactenal
U7.(A) Sensorineural hearing loss is tatjon. It is
meningitis and, usually, is already present at the
a result of cochlear or auditory nerve inflammation
time 0
an
n l
' ' jn as many as
^ menjngococcai
i «j;
^ of patients with pneumococcal
meningitis, and 5-20% of those with H. influenzae W *
M
ent before
with bacterial meningitis should undergo care u au
assessm on an
reassessment
°r SOon after discharge from the hospital. e re hearing deficit.
tpatient basis is indicated for patients who is cerebellar

^ *1® ) The

and
most common manifestation
most severe is acute
o
encephalitis
1
^ mild
involvement
-
but deafness from

! ( C|
d ma MumPS meningoencephalitis
is W **
cranial nerve can occur
e of the 8th .
^
Chapter
^IMeuromuscuie
Questions
ular Disorders

ALMUSAWj
^ vnFR
infant with bulbar and oculomotor Muscle
- old floppy ;

t A 9-month
involvement . likely cause ?
of the following is the MOST
Which
A. Chromosomal disorder
atrophy
B. Spinal muscular
C. Peripheral neuropathy
D. Myasthenia syndrome
E. Congenital myotonic dystrophy

2. A 6-month - old floppy infant presents with generalized weakness which


spares the diaphragm, facial muscles, and sphincters.
Which of the following is the MOST likely cause ?
A. Chromosomal disorder
B. Spinal muscular atrophy
C. Peripheral neuropathy
D. Myasthenia syndrome
E. Congenital myotonic dystrophy

.
3 Ptosis is a prominent feature
in which of the following congenital
myopathies?
A. Central core disease
B. Nemaline myopathy
.
C Myotubular
myopathy

vstrophy type C1C

with generalized hypotonia, weakness, very thin


muscle mass, and arthresents
and opened mouth T, °
The serum CK IPV »I ;15r
,
r eryposis
6 mot 1er
- The head is dolichocephalic with high arched
sported decreased fetal movements.
normal hut the muscle biopsy was diagnostic.
r A r
following' the MOST likely diagnosis is
of tM
'* congenital muscle fiber-type disproportiion
. central
core disease
B
c Nemaline myopathy
rod
p myotubular myopathy
F centronudear myopathy

5. A 19-
month-old child presents with

,
congenital h p dysplasia. His father develooed Jv' '
,,
dis oca ions and
'
P 8nant hVPerthermia with
anesthesia for repair of inguinal hernia.
Of the following, the MOST likely diagnosis is
A. congenital muscle fiber-type disproportion
B. central core disease
C. Nemaline rod myopathy
D. myotubular myopathy
E. centronudear myopathy

6. Central core myopathy is consistently associated with malignant


hyperthermia which is treated with
A. dantrolene
B. salbutamol
C. albuterol
0. N-acetylcysteine
E. correction of a specific gene defect
involved in amyoplasia ?
7. Which of the following muscles is COMMONLY
A. Palmaris longus
B. Sternocleidomastoid muscle
C. Pectoralis major muscle
.
D Flexor carpi radialis
E. Deltoid muscle
always evident by
sign is nearly
8. in Duchenne muscular dystrophY '
age of
A . 1 year
B . 3 year
646
c. 5 Vear

g
D. 7 year
E . 9 year

After
hypertrophy
A, Tongue
Mssissr — " "’“H
B Forearm
C. Neck
0 shoulder
E. Hip
in Duchenne muscular dystrophy is indicated when
10. Initiation of steroids
in -

child shows
A. plateau in development
development
B severe regression in motor
C. cardiomyopathy
D. scoliosis
E. increased CK even if asymptomatic

11. A 3 - year-old child presented with narrow head, high arched palate,
inverted- V-shaped upper lip, thin cheeks, and scalloped concave temporalis
muscles with progressive wasting of distal muscles and flattened thenar and
hypothenar eminences. The child has difficulty with climbing stairs and the
Gowers' sign is positive.
Of the following, the MOST likely diagnosis is
A . Duchenne muscular dystrophy
B. Classic myotonic dystrophy ( type 1)
C. Emery- Dreifuss muscular dystrophy
D. Becker muscular dystrophy
E. Nemaline myopathy

12 . Which of the following is characterized by grip


myotonia ?
A. Duchenne muscular dystrophy
S- Classic myotonic dystrophy ( type 1)
C. Emery - Dreifuss muscular dystrophy
0. Becker muscular dystrophy
E. Nemaline myopathy
1

13
; .
Which of the following endocrine ,
abnormal! ies is common

^^
in mVotonic
^ ’
perthyroidism
Adrenocortical insufficiency
atrophy
c Testicularatrophy
D ovarian
l Hypothyroidism

l4 Which of the following is characterized by weakness and


generalized
muscular hypertrophy so that affected children resemble bodybuilders
(Herculean appearance ?
)
A. Generalized myotonia of Becker
B. Paramyotonia congenita
C. Myotonia congenita (Thomsen disease )
D. Calcium channel periodic paralysis
E. Sodium channel periodic paralysis

15 . Which of the following is the first line treatment for the nondystrophic
myotonias?
A. Carbamazepine
B. Phenytoin
C. Gabapentin
D . Acetazolamide
E. Mexiletine

16. Which of the following can cause rhabdomyolysis


and myoglobinuria ?
A . Statins
B . Colchicine
C. Labetalol
D . Cyclosporine
E . Vincristine

17 . Which of the following can cause hyperkalem


A . Addison disease
B. Thyrotoxicosis
C . Bartter syndrome
D . Laxative abuse
E . Nontropical sprue
648
because of episodic event of i
-year- Id
°
you
child consulted recovers muscle strength during
18. A lO
move after
awakening but
All
gradually
ur
i
extremities are involved and he i
the nei
nor
.
0
,
few minutes
or hours .
*
syndrome
A. Andersen- Tawil
8 periodic paralysis
paralysis
Q thyrotoxic periodic
D. myotonia congenita
E. paramyotonia congenital

19 . A 6- month-old boy presented with poor feeding, hypotonia , respiratory


distress, and hepatomegaly. Chest x -ray revealed cardiomegaly, the serum CK
1

level was greatly elevated, and the ECG showed short P - R interval with wid
QRS complex .
Of the following, the MOST likely diagnosis is
A . Andersen disease
B. McArdle disease
C. Tarui disease
D . Pompe disease
E . Cori- Forbes disease

20. A 10- year-old boy develops muscular cramps,


weakness, and myoglobinuria
after exercise but strength is normal between
attacks. The serum CK level is
elevated only during exercise .
Of the following, the MOST likely diagnosis is
A . Andersen disease
B. McArdle disease
C. Tarui disease
D. Pompe disease
E. Cori-Forbes disease

21. What is the best 'screening test


for diagnosis of mitochondrial
A . Serum lactate cytopathies?
B. Cerebrospinal fluid lactate
C. Serum 3- methyl - glutaconic
acid
D. Hepatic enzymes ( transaminases )
E. Coenzyme - QlO
22- muscle weakness, diplopia, and preserved ^ degree of
‘eOcular
%<
fight He
'
at1d distal muscles of the hands Tend
! "
has poor head control with progressive wJn PUPi arv res »
* to
limb eirri »
stretch reflexes are'
'

^di hed
Rowing, the MOST likely diagnosis is
0
^ A facioscapulohumeral muscular dystrophy
B. spinal muscular atrophy
r motor neuron disease
D. mitochondrial myopathy
c myasthenia gravis

23. A 3 -year-oId myasthenic boy presented with abdominal cramps, diarrhea


profuse sweating, salivation, bradycardia, increased weakness, and miosis for
the last two hours.
Of the following, the MOST likely cause for this presentation is
A. botulism
B. chronic fatigue syndrome
C. myasthenic crisis
D. organophosphorus poisoning
E. cholinergic crisis

diagnostic tests is highly specific for myasthenia


24. Which of the following
gravis ?
A . Electromyography
B. Muscle biopsy
C. Nerve biopsy
D. Anti-AChR antibodies
E. Electroneurography
gravis ?
as a clinica I test for myasthenia
25. Which of the following drugs is
used
A. Atropine sulfate
B. Edrophonium chloride
C. Pyridostigmine
D. Neostigmine
E. Propranolol
be avoided in
following antibiotics should Patients ith
26. which of the ^
myasthenia gravis ?
A. Cephalosporines
B . Penicillins
C. Macrolids
D. Aminoglycosides
E . vancomycin

27 A 7-month-old boy infant


presented with constipation, poor feeding, and a
appears hypotonic, with facial weakness
weak cry. On evaluation, the baby
dysphagia, and a poor gag. The baby was exclusive breast feeding and his
mother started giving him rice soup with honey, his vaccination is up to date
Of the following, the MOST likely diagnosis is
A. viral encephalitis
B. poliomyelitis
C. infantile botulism
D. spinal muscular atrophy
E. myasthenia gravis

28 . What is the common cause of Bell palsy ?


A . Herpes simplex virus type 1
B . Epstein- Barr virus
C. Cytomegalovirus
D. Mumps
E. Human herpesvirus 6

29. A 3 - month- old boy presented with severe hypotonia, symmetric generalized
muscle weakness affecting the lower limbs more than the upper limbs, absence
of deep tendon reflexes with evident tongue fasciculations. He looks alert with
no involvement of the facial and extraocular muscles .
Of the following, the MOST likely diagnosis is
A. Kugelberg- Welander disease
B . poliomyelitis
C. infantile botulism
D. spinal muscular atrophy type I
E. myasthenia gravis

651
neonate presented
3 day-old
with severe
-
feeding problems, and generalized muscle
respiratorv
stre j' 0f decreased intrauterine mov
*^ tion
^6 weeks from presentation . * «*
" There is
baby died
\
^
Ofthe following
, the MOST likely type of SMA is
A I .
B H .
C. HI
D. W
E. 0

.
31 A 2-year- old boy presents with gross motor developmental delay, proximal
muscle weakness more prominent in the lower extremities. He can sit without
support but is unable to walk independently.
Of the following, the MOST likely type of SMA is
A. I
B. 11
C 111
D. IV
E. 0

32. What is the most simple and practical tool for diagnosis of patients with
spinal muscular atrophy SMA ?
A. Electrocardiography
B. Electromyography
C. Nerve conduction study
0. Muscle biopsy
-
E Serum creatine kinase
a
ir
diagnostic test in a patient with
33. What is the MOST definitive first - step
clinical suspicion of spinal muscular atrophy SMA ?
A. Muscle biopsy
B. Genetic marker from blood sample
C - Electromyography
D. Nerve conduction study
E - Serum creatine kinase

652
diSease,
diagnostic for Charcot- Marie- Tooth
ing
followi is
the
34. Which of velocities
A. Nerve conduction
B. Electromyography
biopsy
C. Muscle
kinase
D. Serum creatine
biopsy
E . Sural nerve

A 7- year -old girl


presents with intermittent motor and sensory ne
35.
neurosensory hearing loss, retinitis pigmentosa
ataxia, progressive
malformatio "S tor
l<>SS 01
night vision, ichthyosis, liver
and skeletal
dysfunction, "
and sensory nerve conduction velocities
are delayed. ^°
'

Of the following , the MOST likely diagnosis is


A. Refsum disease
B. Roussy-Levy syndrome
C. Dejerine-Sottas disease
D. Peroneal muscular atrophy
E. Charcot- Marie- Tooth disease

36. A 13- year- old boy presents with recurrent episodes of burning pain and
paresthesias of the feet and lower legs so severe that he is unable to walk,
these episodes are often precipitated by physical activity. Raised red- black
lesions are seen in the buttocks, and periumbilical area.
Of the following, the MOST likely diagnosis is
A. Refsum disease
B. Roussy-Levy syndrome
C. Dejerine - Sottas disease
D. Fabry disease
E. Charcot - Marie - Tooth disease

37. A 4-year- old child present with nystagmus, progressive ataxia, wea
dysphagia, dysarthria, optic neuropathy, and kinky hair.
Of the following, the MOST likely diagnosis is
A . Hypermyelinating ( Tomaculous ) Neuropathy
B. Giant axonal neuropathy
C. Refsum disease
D. Roussy- Levy syndrome
E. Menkes disease
the following may cause peripheral
of neuropathy ?
38. which
, Nalidixic acld
Jc'
Suiphamethaxazole
cefixime
D Amoxicillin
E. Nitrofurantoin

" properly tfgood eT


diagnosed as pneumonia and treated
later he
developed numbness
^ ^^ ^
and paresthesia , followed
by symmetric
Wd

weakness in the lower extremities and progressively involving the trunk and the
upper limbs with loss of ability to walk.
Of the following, the MOST likely diagnosis is
A . acute transverse myelitis
B. poliomyelitis
C. acute disseminated encephalomyelitis
D. Guillain-Barre syndrome
E. acute flaccid myelitis

, ataxia,
40. An 8- year- old girl presented with acute external ophthalmoplegia
mild lower extremity
areflexia with involvement of 6th cranial nerve, and very
weakness.
Of the following, the MOST likely diagnosis is
A . acute motor axonal neuropathy
B . Bickerstaff brainstem encephalitis
C. Miller Fisher syndrome
-

D . polyneuritis cranialis
E . acute transverse myelitis
peripheral nerve
detect early signs of
41. What is the MOST sensitive test to
?
inflammation in Guillain- Barre syndrome
A. Serum antiganglioside antibodies
B . Nerve conduction study infections
and Helicoba cter
C . Serologic testing for Campylobacter
D . Serum creatine kinase level
E . CSF study

6S 4
42. What is the first line treatment for Guillain- Barr4
rapidly progressive muscle weakness ?
syndrome
A Plasmapheresis
**>> *
VWi

B Immunosuppressive drugs
C. Intravenous immunoglobulin
0 Sterotds
f Gabapentin

655
ter 27
ChaP
Neuromuscular Disorders
Answers
jfti' .igfTTiflUj
j SAW "
(Mvastheniia syndromes
, Infantile
0r mLJScles exhibit
greater degree of
involvement.
2.(B)
W)
4.(C) In the infantile form, generalized hypotonia and weakness
, which can
include bulbar-innervated and respiratory muscles, and a very thin
muscle mass
are characteristic. The head is dolichocephalic, and
the palate high arched or
even cleft. Muscles of the jaw may be too weak to hold it closed.
Decreased
fetal movements are reported by the mother, and neonates suffer from hypoxia
and dysphagia; arthrogryposis may
5 (B) The phenotypical spectrum
.
severe Hypotonia, joint laxity, mo
bethies ranges from mild to
pmental delay, hip girdle or axial
muscle weakness, orthopedic comp »ca > g$ recurrent shoulder or
patellar dislocations, congenital °
hip dislocation or dyspdysplasia, Qr foot deformities
may be presenting features .
MAI Central core myopathy CCD 1 „cictpntlv associated with malignant
hyperthermia (MH), which can prece ® of CCD. All patients and
asymptomatic carriers should be in terms of a Potentia
.

. *
adverse reaction to volatile anest e i sC\e relaxants. reo

..
tive
^
anesthetic consultation in patients known to subject to genera a . hesia
* "
.
.
should be considered Treatment o dantrolene and add t onal
supportive care measures.
muscles is common «n ^
W Congenital absence of often
asymmetric. A common aplasia is J "
a r i s longus
enUa|
forearm, which is absent in 30% of norm subjects and is fu V
by other flexors of
the wrist .
^
8.(Cl An early - but ne3 f 3iWays is evident
Gowers sign
by age 5 or 6 yr.
' may be seen^ age 3 Yr ^
most com mon site of muscular typert
s, the next r
9.(A) After the calve
followed by mus cles of the forea rm .
when a child sh WS 3
°ty
the tongue, indic ated
10.(A ) Initia tion of steroids is
motor deve lopm ent° plakj
development and / or a regression in
- . *° C mpsretf
yr old Recommended d Ses are Pred *ith
*
peers. This typically
occu rs by 4
0.9
6
mg / kg/day. ° niSone
day or deflazacort
0.75 mg/kg/ dystrophy (type 1 ) (DM1, orSteinen n
11.(8 ) Classic
appear almost
myot
norm
i

early expressions of the


12.(B) During
onic
al at birth
disease.
, or facial wasting and hypoto

physical examination, myotonia


^ ca°f
may be demonstrated b
alre»dyI
*

the patien
myotonia) .
t to make tight fists and then to quickly open the h

13.(E) Endocrine abnormalities


involve many glands and appear at an tin*
wip
^
during the course of the disease so that
the endocrine status ml'?
annua lly. Hypo thyro idism is comm on; hyperthyroidism occ '
reeva luate d
rarely. Adrenocortical insufficiency can lead to an addis
onian crisis even in
infancy. Diabetes mellitus is common in patients with myotonic dystrophy;
some children have a disorder of insulin release rather than defective insulin
production. The onset of puberty may be precocious or, more often, delavea
Testicular atrophy and testosterone deficiency are common in adults and are
responsible for a high incidence of male infertility. Ovarian atrophy is rare
m
Frontal baldness is also characteristic in male patients and often begins
adolescence. . .
14.(C) Myotonia congenita (Thomsen disease), a type of chann elopa t y, is

most common of the nondystrophic myotonia syndromes and is characterize


by weakness and generalized muscular hypertrophy so that affected chi ^
resemble bodybuilders (Herculean appearance).
15.(E) Treatments for the nondystrophic myotonias include mexiletine *s
first line (both for sodium channel and chloride channel myotonias). Mex
has been shown to improve stiffness as well as decrease handgrip myotonia
16.(A) The following can cause rhabdomyolysis and myoglobinuria
Cholesterol- lowering drugs (especially statins )
Alcohol
Heroin
Amphetamine
Toluene
Cocaine
e- Aminocaproic acid

657
pentazocine
phencycfidine
.( AI Causes of hyperkalemic periofU ,
a c Paralysis
17
Addison disease
Hypoaldosteronism suppiem
ium
Excessive potass
sparing diuretics
Potassium-
Chronic renal failure
,
1S.(B) Muscles that remain active
rapM
extraocular

""* ". ,
are normal between attacks, but in adult life
,„ ,
d h d sorder cap p ,
* ^
, ,,
Patient5
more

ZZZT" " *** ^ “““"


19.(D) The infantile form is a severe generalized myopathy and cardiomyopathy .
Patients have cardiomegaly and hepatomegaly and are diffusely hypotonic and
weak. The serum CK level is greatly elevated. A muscle biopsy specimen reveals
a vacuolar myopathy with abnormal lysosomal enzymatic activities such as acid
and alkaline phosphatases.
20.( B) Glycogenosis V ( McArdle disease ) is caused by muscle glycogen
phosphorylase deficiency inherited as an autosomal recessive trait at locus
llql3, encoded by the PMGM gene. Exercise intolerance is the cardinal clinical
, but
feature. Physical exertion results in cramps, weakness, and myoglobinuria .
is normal between attacks. The serum CK level is elevated only during exercise
21.(C) Serum 3-methyl-glutaconic acid often is increased in
mitochondrial
different genetic
cytopathies in general, demonstrated in more than 50
; it rarely is increase
mutations, and hence is a good screening measurement
other metabolic diseases.
22.(E) In juvenile autoimmune MG,
asymmetric ptosis and some degree
earliest and most constant signs. Extraocu
muscles innervated by just one or two
nuclei; it is progressive. Older children
children might hold open their eyes with t
o
-
unilate

mJW
ral or

weaknes
three
bilatera
s
s is
"“
l


but usually

corresponding brains
mpjain of diplop a »
or thumbs if'
an
the
are the
.

.^
ptos
.
«

s ,s
.
severe enough to obstruct vision. with anticholinesterase
overdos ing
23.(E) Cholinergic crisis secondary to cramps, diarrhea,
a bdomi nal
medications. The muscarinic effects include weakne ss , and miosis.
profuse sweating, salivation, bradyca , increased

658
Cholinergic crisis
cholinergic
requires
drugs , and
only supportive care and withholding 0f
it passes within a few hours; the dose of
. v
of , unless the patient had taken
be restarted should be reconsidered ‘
that was not prescribed.
is one of the few neuromuscular
24 ( A ) Myasthenia Gravis diseases
electromyography ( EMG ) is
biopsy. A decremental response is
more specifically diagnostic than
seen to repetitive nerve J
a mUS(le

muscle potentials diminish rapidly in amplitude until the


stimulat^
muscle b ' ^
refractory to further stimulation.
25.(8) A clinical test for MG is administration of a short -acting cholin
inhibitor, usually edrophonium chloride. Ptosis and ophthalmoplegia if
%0Vf.
within a few seconds, and the fatigability of other muscles decreases.
.
26 (D) Certain antibiotics can potentiate myasthenia and should be
Voided
these include the aminoglycosides, beta blocking agents,
pmcainamice.
chloroquine, and fluoroquinolones.
27.(C) In infantile botulism, which classically presents between the ages of
4 anc
7 mo, honey as well as spores from dirt (e.g., near construction
sites) are
common sources of contamination.
28.(A) Numerous viruses have been linked with Bell palsy. Active
reactivations of herpes simplex or varicella -zoster virus are probably the most
common causes of Bell palsy.
.
29 (D)
30.(E) Infants who are symptomatic prenatally or at birth are classified as having
a rare phenotype, SMA type 0 {<1%); they can present with severe muscle
weakness, respiratory distress, feeding problems, and cranial nerve
involvement. Congenital contractures, ranging from simple clubfoot to
generalized arthrogryposis, occur in approximately 10% of severely involved
neonates. There is a perception of decreased intrauterine movements by the
mother, and these infants usually die within the first months of life.
31.( B) In type II SMA, affected infants are usually able to suck and swallow, and
respiration is adequate in early infancy Developmental delay in gross motor
.
milestones or stagnation of motor development between the ages of 6 and 18
mo is rather typical for this form .
.
92 ( A) Electrocardiography ( EKG )\ may serve as a simple and practical tool in
patients with SMA to
demonstrate a baseline tremor as an artefact representing
muscle fibrillations more prominent on lead II. Although seen in mainly lower
motor neuron diseases, including poliomyelitis, recognition of this EKG pattern
«
” 'Saw *"* ** l«CWn & 5£**°'
(
W VOgraphy [EMq
) and

Cellarti eenetic marker tHe


in bl00d
for homozygous -UT “
the
^ is a
y . , ZTZIZ
H

£*T ’ are reduced “


« »
* •* *
,
««
* u
in number collagen is
K .
increased
? ** *
, -
my faated
and k

£pathologic
^^^
of proliferatedSchwann
formations cell cytoplasm
surr . "
finding is ca lied interstitial
hypertrophic *
neuropathy.
15 (A)
36.(0) The presentation is in late childhood or adolescence, with recurrent
episodes of burning pain and paresthesias of the feet and lower legs so severe
that patients are unable to walk . These episodes are often precipitated by fever
or by physical activity . Objective sensory and motor deficits are not
demonstrated on neurologic examination, and reflexes are preserved.
Autonomic nerve involvement is almost universal and may cause cardiac rhythm
abnormalities, cutaneous mottling, and gastrointestinal peristaltic
abnormalities, but autonomic expression is variable between patients .
Characteristic skin lesions are seen in the perineal region, scrotum , buttocks,
and periumbilical zone as flat or raised red- black telangiectasias known as
angiokeratoma corporis diffusum , Hypohidrosis may be present. Recurrent
strokes result from vascular wall involvement.
37.(B) Giant axonal neuropathy is a rare autosomal recessive disease with onset
in early childhood. It is a progressive mixed peripheral neuropathy and
degeneration of central white matter, similar to the leukodystrophies. Ataxia
and nystagmus are accompanied by signs of progressive peripheral
A arge majority
neuropathy
kinky hair, which
' of affected children have frizzy or
microscopically shows variation in the diameter of the shaft and twisting,psimi r
sc
to that in
Menkes disease; hence , microscopic examination of a ew
Provides a simple screening tool in suspected cases. .
Colchicine
•(E)
Amiodarone, Chloramphenicol . Chloroquine
Psone Ethambutol, Ethanol, Fluoroquinolones
, , Gold, v
,
azine sonialid,
es f0vira| agents
.
etronidazole Nitrofurantoin, Nitrous oxide, Nucleosi penicillamine,
1
eoxycytidine [ddC), didanosine Iddl ], d4T' ° c,i| , Suramin,
biJmidine
. '" -
Stat S
entamidine, Phenytoin, Pyridoxine ( excessive), ’
d , Tn/ptophan
mjde
Tacrolimus , Taxanes ( paclitaxel, docetaxel ) Thahdo
( eosinoph] jia
^
myalgia syndrome), Vincristine .

660
tomS include
39.(0) initi®1
back pain and
weakness' Ra aWeakness
usually begins in
, «
^S^*
be very 3
affected childreJ n„ can sivelV the trunk, the upper limbs, and *,
extremities and P
bulbar muse *•
*
rnares
weakneSs
but
is so
many K
,
.imes proximally prominent. fin
$ deve|op acja|
weakness. %?
,
Extra ,
muscleinvolve
s
^is essentially sVm Weakn ess progre sses over
days,
P3
tif'ihTclinical nadir 0» « than 4 wk . Appr oxim ately
*""
^
weekS' 60% of
point in their illness; a
.
children mce the ability to walk
progress to flaccid tetraplegia-
small proper

•» M
eLrnal svndrome (MFS) . uncommon GBS variant associate;

.
al) ophthalmoplegia, ataxia,

-
nally
with acute , (and occasio am

..
areflexia. The th cranial nerve is m
jnv0|ved in MFS. Although
patients have no o
. vefy mi|d |0wer
W
“ ”
!
* , extreme
* » »G S M
****££%, mon in MFS. Urinary
20% o «« «

conduction studies
5!
%
Mprve

of peripheral nerve inflammation in Motor and sensory nerve

^ reflecting the
patejt
may show acute denervation of muscle .
42.(C) Patients with milder weaknp« ^ Pr greSSIOn may be treaW
expectantly, with observation for
Severe or raDidlv nmor
stabifcatiln °
bilization and spontaneous remission.

ssrzr? r
zirssz
Plasmapheresis and/or
immunosuppressive drugs are altpm /^ ° S ineffeCtiVe' Ster idS ^
"‘
0

within^ '
f

effective for weakness but may help °

661
Disorders of the Eye
Questions

wtiatage , the cornea in newborns attains the adult size?


«
1
A. 2 Years
6. 4 years
C. 8 years
0. 12 years
E. 16 years

l At what age, persistent deviation of an eye in an infant requires evaluation ?


ft . Birth
B. 3 months
C. 6 months
D . 9 months
E . 12 months

3 - What is the approximate visual acuity in newborns ?


A* 20 / 20
B. 20/ 50
C‘ 20/100
D- 20/ 200
E 20 /400

4 At what age, most children attain 20 / 20 vision ?


A * 2 years
&
^ years
c 6 years
0, 8 years
E - 10 years
m infants
s - Which is more common
f the following abnormalities of refraction *
°
history of retinopathy of prematurity^-
A. Hyperopia
B. Myopia
C. Astigmatism
D. Anisometropia
E. Amblyopia

way to treat children with amblyopia ?


6. What is the best
A , Part -time patching
B. Use of atropine drops
C. Full-time occlusion
D. Surgery
E. Watchful observation

7. Which of the following is an important cause of progressive loss of vision


children?
A. Craniopharyngioma
B. Hypertensive encephalopathy
C. Optic neuritis
D. Leukemia
E. Increased intracranial pressure

8. What may be the first clue to amaurosis in an infant ?


A. Timidity
B. Clumsiness
C. Strabismus
D. Excessive cry
.
E Behavioral change

.
9 Which of the following is a cause of prominent
anisocoria in darkness than
light ?
A. Third nerve palsy
B. Tonic pupil
C. Pharmacologic mydriasis
D. Disorder of the iris
E. Horner syndrome

10. What is the most common ophthalmic finding in


shaken baby syndrome ?
A. Ecchymosis of the lids

663
about the eye
Hemorrhage
0.
Refinal
detachment
C of the orbit
0. fracturehemorrhage
1

E. Retinafl .

to denote to 2
h term is used irides of different color ?
A. Heterochromiata iridium
8 Heterochromia iridis
C , Dyscoria
D . Corectopia
E. Anisocoria

12. A young mother brought her 3- week-old well looking baby complaining from
intermittent misalignment of the eyes, she seeks your advice regarding the
prognosis of this problem.
Othe following, the MOST appropriate advice is that it usually
A . resolves without treatment before 6 wk of age
B. resolves without treatment before 12 wk of age
C. resolves without treatment before 24 wk of age
0. resolves without treatment before 1 year of age
E . persists for life

13 . Which of the following can cause severe visual impairment in childhood


A. Rifampicin
B . Isoniazid
C. Streptomycin
D. Ethambutol
E. Pyrazinamide

has excessive problems


W ; The mother of an 18-month-old boy noticed that he
comfortably w h e n the
, while
he sleeps easily and

^ hVzrTZ ?calkex — cooperative


00
a well built,
cheerful,
CUn ammation shows
normal baby . is
Of the following, the MOST like y c al ,e Qf this behavior
.

A. separation anxiety
B. primary retinal degeneratio
C. night blindness
D ,
autism
664
in A deficiency
E. vitamin
with outw ard drifting of right eye
old boy presents deviation is
which
15. A 2- year
-
fixating at distance. The more frequent
is
occurs when he. The visual acuity is good in both eyes. %
fatigueor illness likely cause
is
, the MOST
Of the following
A. infantile esotropia
esotropia
B. accommodative
exotropia
C. intermittent
D. pseudostrabismus
E . heterophoria
te with mild crossed eyes with
., » VQune mother brought her 3-day- old neona
ability to move the right eye laterally. The baby is a product of diffcw
I mited
delive ry admi tted to NICU for 24 hours and discharged home in good
vaginal
condition.
Of the following, the MOST likely cause
is
A. congenital 6th nerve palsy
B. transient 6th nerve paresis
C. sequel of hypoxic ischemic encephalopathy
deficiency
D. sequel of intracranial hemorrhage due to vitamin K
E. damage to lateral rectus ocular muscle

17. A mother brought her 4 -year- old boy complaining from


and expressionless face. Examination reveals incomplete, i a er
facial paresis; with ptosis and esotropia .
Of the following, the MOST likely diagnosis is
A. Duane syndrome
B. Mobius syndrome
C. Brown syndrome
D. Parinaud syndrome
E. sylvian aqueduct syndrome
head
18. A 14-month-old boy presents with fine, rapid, pendular nys
nodding, and torticollis.
Of the following, the MOST likely diagnosis is
A . opsoclonus
B. gaze- paretic nystagmus
mus
t nvsta£
C- Irrus nutan
,ngs
D
Jolarbobb
, following may be the first sign of neuroblastoma
n

^
which
. „
,
0fT he
C 0nUS

A

0.
OpS
° r dysmetria
Ocula
Q Ocular flutter
D ocular bobbing
E ocular myoclonus
infant with left upper lid ptosis, the mother noticed that the
4- month- old
20 ^ lid raises up while the baby sucks from the bottle.
Effected MOST likely cause of this ptosis is
Of the following, the
A. myasthenia gravis
B. muscular dystrophy
C. blepharophimosis syndrome
D. Marcus Gunn jaw- winking phenomenon
E. congenital 3 rd nerve palsy

the MOST approp riate initial treatment of rapidly


21. Which of the followin g is
expanding lid capillary hemangioma ?
A . Topical timolol
B. Intratesional corticosteroid
C. Systemical corticosteroid
D * Systemic propranolol
E . Surgical excision
hyp
22. What is the usual presentation of children with
A . Loss of vision
B. Subconjuctival hemorrhage
C. Strabismus
D - Leukocoria
E . Local pain

in infants?
23. What is the MOST common cause of tearing
A- Nasolacrimal obstruction
B. Glaucoma
666
c intraocular inflammation
in
«on
abra
0. Corneal
body
l Foreign
with diagnosis of late onset
to NICU
day-oId newborn admitted
edema, erythema of the lids,
24. A 10- develops
the patient ^
1 day later
discharge, pannus
septic shock .
Of the following,
formation

the MOST
, and endophthalmitis with rapid detenoratl

likely pathogen is
^
A. Staphylococcus aureus
B. Chlamydia trachomatis
C. N. gonorrhoeae
D. Pseudomonas aeruginosa
E. E . coli

2S. What is the MOST frequent cause of bacterial purulent conjunctivitis r


children?
A. Pneumococci
B. Staphylococci
C. Gonococci
D. Meningococci
E . Nontypable haemophilus influenza

26. A 10 year-old boy presents with low grade fever, sore throat, sensation of a
*

foreign body beneath the lids, itching, with edema and photophoba
Examination reveals large oval follicles within the conjunctiva, preauricula '
adenopathy, and a pseudomembrane on the conjunctival
surface.
Of the following, the MOST
likely pathogen is
A. enterovirus
B. coxsackie virus
C. adenovirus
D - diphtheria
£ chlamydia

.
A 12- year - otd hn
V COmes to outpatient clinic in summer complaining frorf
|

intense itchino i
01 h« «
A- mast cell
2 edema .
appropriate initial treatment is topical
stabilizers
667
glandin inhibitors
prostaL
B.
C. corticosteroidsdrops
D. 3ntihistamine
E. antibiotics
of the following should be avoided in endnt••,c , •

28. which keratitis?


Mechanical debridement
A.
B. Topical ganciclovir
C. Systemic acyclovir
D. Cycloplegic agent
E. Topical corticosteroids

29. Which of the following may cause cataract ?


A. Spherocytosis
B. Thalassemia
C. Sickle cell disease
D. G6PD deficiency
E. Iron deficiency anemia

with proptosis, painful


30. An 8- year -old febrile toxic looking boy presents
visual acuity, together
limitation of movement of the right eye, and decreased
eyelids.
with edema of the conjunctiva and swelling of the
Of the following, the MOST likely diagnosis is
A . idiopathic orbital inflammation
B. granulomatous vasculitis
C. orbital cellulitis
D. idiopathic orbital inflammation
E . metastatic tumor
may remain asymptomatic ?
uveitis
31. Which of the following causes of anterior
A. Kawasaki disease
B. Ulcerative colitis
C. Brucellosis
D. Herpes simplex
E . Juvenile idiopathic arthritis retinopathy of
risk factor in
represent a maj r
°
32. Which of the following
prematurity ?
668
A Oxygenation
,

B. Retinal immaturity
C Apnea
D. Hypercarbia
E . Anemia

is the initial sign


in the majority of patients with retinoblastomj
33 . What Is ?
A. Strabismus
B Pseudohypopyon
C Hyphema
,

0 . Leukocoria
E . Vitreous hemorrhage

34 . What is the first clinical manifestation of retinitis pigmentosa ?


A. Impairment of night vision
B. Progressive loss of peripheral vision
C . Loss of central vision
0. Strabismus
E . Leukocoria

35. The diagnosis of which of the following disorders, in a patient with


presumed retinitis pigmentosa, is important ?
A . Scheie syndrome
B. Sanfilippo syndrome
C. Kearns- Sayre syndrome
D. Laurence-Moon syndrome
E. Refsum disease

36. A cherry-red spot typically occurs in which of the following conditions ?


A . Metachromatic leukodystrophy
B. Niemann - Pick disease
C. Tay -Sachs disease
D. Gafactosialidosis
E. Mucolipidoses

37. What is the MOST frequent presenting sign of Coats disease ?


A. Blurring of vision
B Rubeosis of the iris
,

669
C Glaucoma
0. Cataract hemorrhage
E- Vitreous
,
ich
c of the following hypertensive retinal changes is irreversible ?

38. Wh constriction of the arterioles


A Generalized
hemorrhages
B fiame- shaped
C Cotton- wool spots
0 Papilledema
E Copper- wire appearance

following is a principal sign of orbital tumors ?


39. Which of the
A. Ptosis
B . Optic nerve head congestion
C . Optic atrophy
D . Proptosis
E. Loss of vision

headach e, fever, severe ma a *e'


,
40. A 2- year old child
- presents with severe
rigidity, P * tlve * *
decreased vision. Examination reveals neck
.
° ‘resu . .
gia
bilateral ptosis, proptosis, swelling, and ophthalmople is
eye lesion
Of the following, the MOST likely cause of the
A. orbital cellulitis
B . subperiosteal abscess
C. orbital abscess
D. septic cavernous sinus thrombophlebitis
E. inflammatory edema
tearing,
swelling, increase in
A 4- year-old child presents with pain, redness, associated
the right upper eyelid
and discharge over the lateral one third of
thy on the same side.
with concurrent preauricular lymphadenopa
Of the following, the MOST likely diagnosis is
A . dacryocystitis
B . dacryoadenitis
C . preseptal cellulitis
D . hordeolum
E . chalazion

670
Chapter 28
Disorders of the Eye
Answers
ZUHAIR ALMUSAWI
is relatively large in newborns ( averaging lo mm ) and ,
!!
dul size ( nearly
2.(C) Many normal
12 mm ) by the age of 2 yr or earlier .
infants may have imperfect coordination of the
atta n.

early days and weeks, but eye


movements and alignment during the
coordination should be achieved by 3 - 6 mo, usually sooner. Persistent deviat *0
of an eye in an infant at 6 mo of age requires evaluation.
3.(E) The visual acuity in newborns is estimated to be approximately 20/400
This poor vision is a result of the immature, multilayered foveal anatomy.
4.(C) visual acuity of 20/40 is generally accepted as normal for 3 yr old children
At 4 yr of age, 20/30 is acceptable. By 5 or 6 yr of age, most children attain
20/20 vision.
5.( B) Myopia is infrequent in infants and preschool-age children. It is more
common in infants with a history of retinopathy of prematurity.
6.(C) Although full- time occlusion has historically been considered the best way
to treat children with amblyopia, a series of prospective studies has shown that
some children can achieve similar results with part -time patching or the use of
atropine drops.
7.( A) Gliomas of the optic nerve and chiasm and craniopharyngiomas are
primary diagnostic considerations in children who show progressive loss of
vision.
8.(C) The first clue to amaurosis in an infant may be nystagmus or strabismus,
with the vision deficit itself passing undetected for some time. Timidity ;

clumsiness, or behavioral change may be the initial clues in the very young
Deterioration in school progress and indifference to school activities 3
common signs in an older child.
9.(E) Patient who has more prominent anisocoria in light than darknes
indicating that the pupillary constrictor of the larger pupil is abnormal ( • "
'*
fails to constrict in light )as in third nerve palsy, tonic pupil, pharmaco 6
mydriasis, or a disorder of the prom ^
^
iris while a patient who has more
anisocoria in darkness than light, indicating that the pupillary dilator 0 ^
671
ii is abnormal (i.c., it fails to dilate in
»isKo - 1v
^1 hemorrhage is the most common
.
ophtha|mic
firHlini Jlwl
be ««
" '
* of the retina The pattern of hemorrhage
.
*
,*
diso
, W« other oeos « of retinal
. ,
hemorrhage o from
heterochromia, the 2 rides are of different
helm H
accioeml "' *
& - * T?. . r
tl A ) in

iheterochromia
12.( 0) Between
ir d s).
<
* ” "* **•
2 and 4 mo of age, many
» rotor from
color (heterochromia

infants have infantile esotropia


(neonatal misalignments ) , which in most resolve
spontaneously. Those that
resolve without treatment do so before 10-12 wk of age and
have iintermittent
or variable deviations. Those most likely to benefit from
active treatment have
persistent esotropia (10 wk-6 mo of age) and
( 4 PD in
combination with a refractive error +3 00 D
absence f ° '
Prematurity, *
developmental delay meningitis
^ 6 3n malieS and
°
paralytic strabismus ' ' ^ ° ' or
13.(0)
14.(C) Nyctalopia, or night
blindness, is vision that is defective in reduced
illumination. It generally implies impairment in function of the rods, particularly
in dark adaptation time and perceptual
threshold Stationary congenital night
,

blindness may occur as an autosomal dominant, autosomal recessive, or X -


linked recessive condition. It may be associated with myopia and nystagmus.
Children may have excessive problems going to sleep in a dark room, which may
mistaken for a behavioral problem. Progressive night blindness usually
lndicates r seconc arY retinal, choroidal, or vitreoretinal degeneration;
It occurs °
R V am n , *
deficiency or as a result of retinotoxic drugs such as
quinine * ' ^
seconcl most common type of misalignment.
The
!!^' *'
0 3 0115 arG
divergent eV ation exotropia is
maY be intermittent or constant . Intermittent
the most by outward
exodeviation in childhood. It is characterized
driftin QfCommon
°^ usually occurs when a child is
fixating at distance. The
,, bright
deviatio0 * S gener3 V rnore frequent with fatigue or illness. Exposure toinitially
light m the eyes
Cause reflex closure of the exotropic eye. Because
can beakVrT!? stra Sbt most of the time, visual acuity tends
to be good in both
eYes a
e otroP a
'
,nocular vision is initially normal. The age at onset of intermittent
4 yr.
* * varies but is often between age 6 mo and
'
17
16.(8)

ass0031
.(B)

asymmetric
m

The

ic,
^
distin
and
weak
,
ctive
and abduction often c0 p
--store
ness
'" "
s
- The
zsz
£of
js commonly bilateral, frequ

tef)d ng t0
|
w
spare
kerato pathy
the lower
may lop
deve . The
entiy
.
face .and
..

^
P •
olatvsma.
Ectrop ion ,
may be '
eP
un 3
, or bj|ateral. Esotropia is common,
abduction defect ' tpe 0f acquired nystagmus in childhood. | „
18.(0) Spasmus
its complete form
..
.
nutans *

c oHis .
char

11»
a
3
P

V
by the trjad of pendular nystagmus, head
is characteristically very fine, very rapid,
nodding, and tort
horizontal
usually
develop
, and
develo p
pendu
within
ar,
the
at various times. In
^^
many
V
asyrTimetric, sometimes unilateral. Signs

cases
years.
Qf |jfe components of the triad may
^ the condition is benign and self-limited.
usually lasting a few months, sometimes
with infectious or autoimmune
19 (A) opsoclonus is most often associated
encephalitis. It may be the first sign of neuroblasto
ma or other tumors
producing a paraneoplastic syndrome.
20.(D) Marcus Gunn jaw -winking ptosis (maxillopalpebral
synkinesis) accounts
for 5% of ptosis in children. In this syndrome, an abnormal synkinesis exists
between the 5th and 3rd cranial nerves; this causes the eyelid to elevate with
movement of the jaw . The wink is produced by chewing or sucking and may be
more noticeable than the ptosis itself.
21.(D) In the case of a rapidly expanding lesion, which may cause amblyopia by
obstructing the visual axis or inducing astigmatism, treatment should be
considered. Systemic propranolol has been shown to be an effective treatment
without the risks associated with corticosteroid use. Other treatment options
include topical timolol, corticosteroids (systemically or by direct injection), and
surgical excision.
22.(A) Hyphema is the presence of blood in the anterior chamber of the eye -
may occur with either a blunt or perforating injury and represents a situation
that may threaten vision. Hyphema appears as a bright or dark red fluid leve
between the cornea and iris, or as a diffuse murkiness of the aqueous humor.
Children with hyphema present with acute loss of vision, with or without pain.
23.( A )
24.(D) Conjunctivitis caused by Pseudomonas aeruginosa is
uncommon,
acquired in the nursery, and a potentially serious process. It is
characterized by
^^
^hieapa
^ °form *he


lidS Purulent
•££2* , , ion
'
frequent c u «
'
,te
SySftS? 0
^• Js2
Sc Epidtransmitted
)
an
^:
d i$
i keratoconjunctivitis is caused by
by direct contact.
adenovirus serotype 8 19
°*
,ls „
or -
37 ,
Z7.|D) Allergic
,
conjunct vitis is usually accompanied
by intense itching dear
watery discharge , and conjunctival edema { chemosis ). It is commonly
seasonal
(spring - summer ). Cold compresses and topical antihistamine drops give
symptomatic relief. Topical mast cell stabilizers or prostaglandin inhibitors may
also help. In selected cases, topical corticosteroids are used under an
ophthalmologist ' s supervision but should not be used routinely or for a long
time.
28.(E ) Topical use of corticosteroids causes exacerbation of superficial herpetic
disease of the eye and may lead to corneal perforation; eye drops combining
steroids and antibiotics are therefore to be avoided in treatment of red eye,
unless there are clear- cut indications for their use and close supervision during
therapy.
29.(A) Miscellaneous disorders causing cataract
/ Atopic dermatitis
Drugs ( corticosteroids )
* Radiation
Trauma
Juvenile idiopathic arthritis
Retinopathy of prematurity

».(C) Inflammation of the tissues of the orbit,


Proptosis, painful limitation of movement of the eve; an )
of the conjun tiva ( chemosis
visual acuity, is termed orbital cellulitis. Edema
and inflammation and swelling of the eyelids may be idiopathic arthritis
wit
Iridocyclitis that occurs in children uveitis, it rarely creates
o an ^
deserves special mention. Unlike most forms mav not be noticed
pain, photophobia , or conjunctival hyperemia - o ^
until severe and irreversible damage has occUf fully known, but
prematurity
The risk factors associated with ROP
not_
represent the major factors.
and the associated retinal immaturity at .. heart disease, infection,
ra
Oxvgenation, respiratory distress , apnea,
674
need for transfusion are ,
lr «, ht

~jr«~
is a vrefiex <
reflection oflight off
eukocoria
|

the white tumor .


|- Leuk c na re uits because of 'ents
The
° ° ;
most frequent
second
A
initial sig
,
t noblastoma is strabismus
. "
of

34 ( A ) impairment of night vision or dark adaptation is often the first clinic ,,


peripheral vision, often in the form of
manifestation. Progressive loss of
expanding ring scotoma or concentric contraction
of the field, is usual. Then,
may be loss of central vision.
35.(E) The retinal manifestations of abetalipoproteinemia ( Bassen-Kornzweig
syndrome; and Refsum disease are also similar to those found in retinitis
pigmentosa. The diagnosis of the latter 2 disorders in a patient with presumed
retinitis pigmentosa is important because treatment is possible.
36.(C) A cherry- red spot typically occurs in certain sphingolipidoses, principally
in Tay-Sachs disease ( GM 2 type 1), in the Sandhoff variant ( GM 2 type 2), and in
generalized gangliosidosis (GM1 type 1) . Similar but less distinctive macular
changes occur in some cases of metachromatic leukodystrophy ( sulfatide
lipidosis ), in some forms of neuronopathic Niemann - Pick disease, in
galactosialidosis, and in certain mucolipidoses.
37.( A) Coats disease is exudative retinopathy of unknown nonhereditary cause
is characterized by telangiectasia of retinal vessels with leakage of plasma to
form intraretinal and subretinal exudates and by retinal hemorrhages and
detachment. The condition is usually unilateral. It predominantly affects boys,
usually appearing in the 1st decade . The condition is nonfamilial and, for the
most part, occurs in otherwise healthy children. The most frequent presenting
signs are blurring of vision, leukocoria, and strabismus. Rubeosis of the iris,
glaucoma, and cataract may develop . Treatment with photocoagulation or
cryotherapy may be helpful.
3R.( E) In long- standing hypertension, irreversible changes may
occur. Thickening
of the vessel wall may produce a silver - or copper-
wire appearance
Hypertensive retinal changes in a child should alert the physician to renal
disease, pheochromocytoma, collagen disease, and cardiovascular disorders,
particularly coarctation of the aorta.
39.( D) The effects of orbital tumors vary with their locations and growth
patterns . The principal signs are proptosis, resistance to
retroplacement of the
675
impairment of eye m
eVe - and signs are ptosis nn, ° -
Vemer>t A Da , by ,
significant M »
« .M ** " ^ mport ,
; 3nd

"*
t dues5 t0 Vascu/ar
lesions
^


iO.(D) , .' „ *
-
Dacryoadenitis is inflammation f the ac > al fi|anH
* 1(8)
occurs in the pediatric population
pathogens . ^
Pain
° ""
" °f °
S me V ung adults 7 COmm
°^
to a

^
variety of infectious Crease in

^^
' '
discharge over the lacrimal g arid are n
| and usually visible
third of the upper eyelid; concurrent preauricular lymphadenopathy may be
noted. It may occur with mumps (in which case it is usually acute and bilateral,
subsiding in a few days or weeks ) , with influenza, infectious mononucleosis, and
herpes zoster . Staphylococcus aureus may produce a suppurative
dacryoadenitis, and other bacterial causes include streptococci and Neisseria
gonorrhoeae.
Chapter 29
The Ear
Questions
MUSTAFA ALANBAKI
1. what is the MOST common cause of otalgia in children?
A. Foreign body in the auditory canal
B. Trauma
C. Otitis externa
D. Acute otitis media
E. Mastoiditis

2. What is the MOST common cause of conductive hearing loss in children?


A . Tympanic membrane perforation
B. Middle ear effusion
C. Impacted cerumen
D . Foreign body in the auditory canal
E . Cholesteatoma

3 . What is the MOST common infectious cause of congenital sensorineural


hearing loss ?
A. Congenita! rubella
B. Congenital toxoplasmosis
C. Congenital syphilis
D. Congenital cytomegalovirus
E . Congenital herpes simplex

4. Which of the following is the average threshold level ( db ) of severe heanrtf


loss?
A. 16- 25
B. 26- 30
C. 31- 50
D. 51-70
E- More than 70

677
the correct answer re ar<Jinp
3
is
5. What assessment?
^
udioiogic of single words nth ge 0f 12
A. No
use
-»ord vocabul
' **
^ hr referral of a child
for

le B ofl8 omhs
B. Sin
C. No
* ^
f 0ns; unintelligible iin
' the age of 30
months
D. <600 word
s; clarity 580% in the age of 36 months
telegraphic sentences in the age of 48 months
E. No use of

6. At what
age a child with a known risk factor for hearing loss should be
evaluated?
A. First week of life
B. First month of life
C. First 3 months of life
D. First 6 months of life
E. First year of life

7. What is the most common cause of vertigo in children?


A . Acute serous labyrinthitis
B. Acute suppurative labyrinthitis
C. Vestibular neuritis
D. Vestibular migraine
E. Benign paroxysmal vertigo

nt of otitis externa ?
8. What is the most common causative age
A. Staphylococcus Aureus
B. Pseudomonas aeruginosa
C. Proteus mirabilis
D. Klebsiella pneumoniae
E. Candida albicans
externa and otitis media
between otitis
9. Which of the following differentiate
n children?
A . Otitis externa is more com
mon
prom inent in otitis externa in otitis externa
B. Fever is more is more com mon be normal in
C. Periauricula r lymp hade nitis mem brane shou ld
tympanic
D . Otoscopic exam ination of the
otitis externa
678
Hearing loss is more
common in otitis externa
E.
.,bv his Parennt,.vuith 5 days history ot
10. A 2- year-o
Id
of
°
the right *°S
toddler is br
: assoc iate '
* h re jness o« (
right
tever
eye .
severe earach e , 28, BP 100m
greenishsecretions from it. Exam:
to his
TeroPeratandUreoVer^
ot
ufr0Undings
100 RR pR
ise in no distress, . /65
.
He is active, alert
tympanic membrane is
erythem atous and bulging, with poor moMlty
f .

pneumatic otoscopy.
What is the most common
causative agent?
A. Streptococcus pneumoniae
B. Nontypeable Haemophilus influenzae
C. Moraxella catarrhalis
.
0 Chlamydia trachomatis
E . Staphylococcus aureus

11. What is the most specific tympanic membrane finding in acute otitis media ?
A. Erythema
B. Bulging
C. Opacification
D. Perforation
E. Immobility

12. A 7-month-old infant presented to you 3 days ago with second attack o'
acute otitis media and you gave him azithromycin suspension orally as he hac
urticarial rash following amoxicillin therapy in the first attack, but 1C
improvement of his symptoms, so you want to change his antibiotic.
Of the following, the MOST appropriate choice is
A. trimethoprim-sulfamethoxazole
B. cefdinir
C. levofloxacin
D. ceftriaxone
.
E clindamycin

13. What is the antibiotic


of choice for acute otitis media ( AOM), i P
producing pathogen is suspected?
A. Ceftriaxone
B. Cefdinir
C. Amoxicillin-
clavulanate
679
Levofloxacin
l clarithromycin

What is the most common causative pathogen in


uncomplicated
mastoiditis in children? aureus acute
A staphylococcus
B streptococcus pyogenes
r pseudomonas aeruginosa
0. Haemophilus influenzae
E Streptococcus pneumoniae

15. A 12- year-old boy presented to the emergency room with a 5-day-history
of
left sided otalgia , with 2 days of left temporal and parietal headache,
vomiting,
and vertigo. He received co-amoxiclav with a diagnosis of "ear infection. On
examination, temp B9.8°C. He has neck stiffness but no focal neurological signs
but left side papilledema . The left tympanic membrane is red and bulging. There
are retroaurcular redness, tenderness and swelling.
Of the following, the MOST appropriate next step is
A. change the antibiotic to IV ceftriaxone and follow up after 72 hours
B. change the antibiotic to IV ceftriaxone and vancomycin and perform
ultrasound to the left mastoid area .
C. change the antibiotic to IV ceftriaxone and vancomycin and perform CT
scan of the left temporal bone
vancomycin and perform
D . change the antibiotic to IV ceftriaxone and
contrast-enhanced CT scan of the brain.
vancomycin and perform MRI
E. change the antibiotic to IV ceftriaxone and
and MRV of the brain .
recently a cochlear implant due to severe
16. An 18- month- old toddler had vaccination .
suggest pneumoc occal
sensorineural hearing loss, you
What is the recommended schedule ? conjugate ine ( PCV13 ), 2 months
vaccine
A. 3 doses of 13- valent pneumococcal
apart . conjugate ine ( PCV13 ), 2 months
vaccine
B. 3 doses of 13- valent pneum
ococcal
at 24 months of
_
- age .
13
apart, with 1 extra dose o
f PCV
conjugate vaccine ( PCV 13 ), 2 months
coccal polysaccharide vaccine
C. 3 doses of 13- valent pneumo pneum ococcal
-valent
apart, with 1 dose of 23
age .
(PPV 23) at >24 months of
680
n ? doses of 13- valent pneumococcal
conjugate vaccine (PCV131,
n with 1 extra dose of PCV
13 at 224 months of age. %(,;
pneumococcal conjugate vaccine (PCV131
E 2 doses of 13-valent 2 fT)
apart, with 1 dose of 23 - valent pneumococcal
polysaccharide
( PPV23 ) at £ 24 months of
age .

681
Chapter 29
The Ear
Answers
MUSTAFA ALANBAKI
1(D) Otalgia
usually is associated with inflammation „f . ,
^
r 3b
50% of cases) or external ear, but it can represent o “ ° ut

—-
pharJnx" ^ fr°m
Zr ~
involvement of the teeth, temporomandibular joint or
e h rir!

, ICHl1 !
i

01

<m m

*
4.(D) Hearing loss is categorized according to average threshold level (db) at
500- 2,000 hz ( ansi) into the following levels:
Normal range 0-15 db
Slight hearing Loss 16- 25 db
Mild hearing Loss 26- B0 db
Moderate hearing Loss 31- 50 db
Severe hearing Loss 51- 70 db
Profound hearing Loss > 70 db
MQ The Criteria for Referral for Audiologic Assessment is as following.
12 months: No differentiated babbling or vocal imitation
^
At 18 months: No use of single words
At 24 months: Single - word vocabulary of S 10 words
; unintelligible
At B0 months: <100 words; no evidence of 2 - word combinations
clarity < 50%
At 36 months: <200 words; no use of telegraphic sentences;
At 48 months: <600 words; no use
6.(0) Any child with a known risk
^ for
the first 6 months of life. in pediatric
0f vertigo
n
° lasting seconds
Benign paroxysmal vertigo, the most c mf
Patients, is characterized by short periods of
to a few
minutes and associated with imba
^^
vert
an
g dizziness
nystagmus; tinnitus
or

) is
hearing loss is unusual. tcur ithout swimming
B.fB) External otitis (swimmer s ea although it can 60% ), 5 . aureus,
' sa lup
caused most commonly bY aefUgjn0
682
aerogenes, Proteus mirabilis, Klebsiella pneumoniae , streptv
Enterobacter diphtheroids , and fungi such as
negative staphylococci i, # Candid%
coagulase -

also may be isolated


.
Aspergillus
more common , than otitis
externa and is a leadi
ne
media is
) Otitis and figures importantly n th
9 (C for use of antibiotics
,
and
for physician visits
of fever . The predominant symptom of otitis e- tema e. '
differential diagnosis ) of ear pain (otalgia ), 0ft *
|

typically within 48 hr
acute rapid onset
(
of the pinna or by pressure on the tra
^8
accentuated by manipulation
the pain and tenderness ( tragus or ^
pin ndbv
severity of ' °
^
f
jaw motion . The
, If the tympanic membrane (TM ) can be v
may be disproportionate
otitis externa ), it may
appear either normal or opaque TM '^
mobTh
may be reduced in r ,
*
'^
,
**
lln

normal or . if the TM is thickened , mobility


physical findings may included*
positive and negative pressure. Other
and tender lymph nodes in the periauricular region, and
erythema an T*
of the pinna and periauricular skin. Conductive hearing In , (CHI 1* %
T"
from edema of the skin and tympanic membrane (TM serous or
secretions, or the canal skm thickening associated with chronic external ^
but on the other hand middle ear disease and on the top of the list otitis media
otil '
,
with effusion is the most common cause of conductive hearing loss .
10.( B Conjunctivitis-associated otitis media, simultaneous appearance of
purulent and erythematous conjunctivitis with an ipsilateral OM is a well
recognized presentation, caused by nontypeable H. Influenzae in most children
rhe disease often is present in multiple family members and affects young
children and infants.
< >
f A0M 97% but h3 s lower
”n(2 (Chi51%,d)ren ith n n_tVpe reaCti n in which oss reactivity with
vi t U8
y ^f
^
“ °
ceohCi?„' ' 0
° 3
*
approirL, e , h
, "
^
T
theraPV With cefdinir

th re re far ' fewer choices°.""Resistance


^ reaction or kn
3 tyPe
be
sensitivity to
30

trimeir Ifamethoxazole
cephab porin h
T "' bV ma"V strains of both nontypeable H.
f 6 3

,
to

influenzae
^ ' U

, High c inical fal e r3te in


"
"
children with AOM e3 ed ,n,t,ally With thiS anti > crobial argue against its use.
: :r " arc .*
similarly, increasing rates of macrolide resistance
azithromycin. Although not approved by the FDA for
' , ? „
[u
,
clinicians have used qumolones in this patient pooZ '" Res
USe
' ,n
o ,stanCe W
macrolides also results in resistance to clindamycin !!. 0therw ,se 15
generally effective against resistant strains of S. pneumoniae
683
cases
I
of antibiotic resistance which i
Old "
inhibit

lcs
tact *fftasei
,1
e b¥
a
Action of p.
P lactamase
.
|E The most common causative oathn
'

Ustoiditis -12 . Staphylococcus aureuspneunT" °mplicated „,*


in children is Streptococcus Unc
ar
2-10% Pstu oStreptococc

^^
%)
pyogenes Haemophilus
(0 (
jjOK , and
) influenzae (2-3 ). % eudomonas “ aeruginosa
provisional diagnosis of
15.(0) The this case is acute otitis
acute mastoiditis and intracranial (mostly epidural) ahsr , COmplicated by
help l to differentiate postauricular erythema from a
^ f
postauSrT
can be
ess and
* * ^ tk ° radiation exposure. However ,
ultrasound not as
sensitive as CT scanning and will underdiagnose postauricular abscess
form !
and will provide no information as to whether there is an
intracranial
complication present such as a brain abscess . CT scan of the temporal bone
can
confirm the diagnosis of acute mastoiditis, if there is any concern about the
possibility of an intracranial complication ( as in this case ), a contrasted CT scan
is the most sensitive test readily available and should be ordered upon
presentation. Magnetic resonance imaging is generally reserved for patients in
whom there is a suspected intracranial complication like sagittal vein
thrombosis. Regarding antibiotic therapy If intracranial infection is suspected,
broader spectrum antimicrobial coverage ( e.g ., vancomycin plus a 3rd
-

generation cephalosporin) should be initiated.


16,IE) — -

Recommended pneumococcal vaccination schedule for persons with


jcochlear implants PPV 23 dose
primary PCV 13 additional
~

Age at first pcvl3 PCV 13


dose ( months) senes dose at 24
at 12 -15 Indicated *
2-6 3 doses, 2 1 dose months of aje
months of age —
at 24
months apart
2 1 dose at 12
- 15 indicated
a i_
_of |
*
7 -11 2 doses ,
of age months
months idicated at 24 *
12-23
months
2
apart
doses ,
_
2 Not indicated
m months ofafi? ^ —
months apart indicated
2 No t indicated
24- 59 2 doses,
months apart
Indicated ——
Not indicated
^

m
>60 Not indicated

684
Chapter 30
Skin
Questions

scalp hair growth per month?


1. What is the usual
A. 0.5 cm
B. 1cm
C. 1.5 cm
D. 2 cm
E. 2.5 cm

2. How long it takes for a complete fingernail to regrow ?


A. 2 months
B. 4months
C. 6 months
D. 9 months
E. 12 months

3. A 2-year- old boy diagnosed as acute myelogenous leukemia 2 months ago,


and now on treatment, presents with fever and oral ulcers; with tender,
erythematous, and edematous pustular nodules on the arms, legs, and trunk.
CBC reveals anemia, and absolute neutrophilia .
Of the following, the MOST likely diagnosis is
A. pyoderma gangrenosum
B. necrolytic migratory erythema
C. langerhans cell histiocytosis
D. Behcet disease
E. Sweet syndrome

4. When medication eruptions usually begin after exposure .


A. Immediately
B. 1- 3 days
C. 4-6 days
D. 7-10 days

685
l lVl4 davs
"old
boy with p-thalassemia major
5. A 3'veaf |<5 ago, presents with fever,
underwent bone marrow
2" ee
tIi isPlant
, * mucositis, diarrhea, erythematous
3 eruption that starts at the head and
ffaculoPaPu neck including the ears,
,
t Spread to the trunk, extremities, palms,
,
ikeiv diaen s s is
and soles.

^; sweet syndrome
°
Jc acUte graft versus host disease
drug eruption
D necrolytic migratory erythema
£ Langerhans cell histiocytosis

6. A 4 - year-old boy diagnosed 5 weeks ago as focal epilepsy treated with


carbamazepine with good response. Today presents with fever, malaise, diffuse
exanthem of pruritic, morbilliform papules on the head, upper trunk, and arms,
with facial edema and cervical lymphadenopathy. CBC shows eosinophilia and
atypical lymphocytosis with elevated liver transaminase.
Of the following, the MOST likely diagnosis is
A. medication reaction
B. Sweet syndrome
C. DRESS syndrome
D. serum sickness-like reaction
E. Stevens-Johnson syndrome
-drug
7. Which of the following is a common offending medication causing fixed
eruption?
A. Penicillin
B. Sulfonamide
C. Macrolide
D. Cephalosporin
E . Aminoglycoside
throat with low

regions .
686
likely diagnosis is
,
of he following, the MOST
pustular psoriasis
A generalized
B. bullous impetigo
C. IgA pemphigus
dermatosis
D. subcorneal pustular
E. acute generalized
exanthematous pustulosis
topical medications vehicle is preferable for dry
Which of the following
9.
skin ? ^
A. Lotion
B. Cream
C. Ointment
D. Gel
E. Solution

10. Which of the following topical antibiotics is MOST effective in treatment of


moderate impetigo ?
A . Neomycin
B. Nitrofurazone
C. Fusidic acid
D. Polysporin
E. bacitracin

11. Which of the following is a medium - potency topical corticosteroid ?


A . Mometasone furoate
B. Hydrocortisone
C. Desonide
D. Fluocinonide
E. Betamethasone dipropionate

12. A 3-day- fullterm, well looking boy brought by


his mother because she
noticed numerous firm, yellow - white, 1- 2 mm papules
and pustules over his
body surface. Intralesional aspirate
demonstrates eosinophils with sterile
cultures.
Of the following, the MOST likely diagnosis is
A . pyoderma
B. candidiasis
C. transient neonatal pustular melanosis

687
Q rniliaria
toxicum
E erythema

iC rrn. ’
. rrr
:r 56 »
rom episodes of
t0J conventional
management. Exammat on shows dry hypopigmented
flattened
sparse
nasal
scalp hair,
bridge, thick
eyebrows
everted
and lashes.
lips

Of the following, the MOST likely diagnosis is


,
^ frontal bossing, a
and prominent low - set
ears, with

A. hypohidrotic ectodermal dysplasia


B. hidrotic ectodermal dysplasia
C. Fabry disease
D. Sjogren syndrome
E. hereditary sensory autonomic neuropathy

14. A 9- month-old infant brought by his mother complaining from a macular,


sharply circumscribed, pink lesion localized to the right forehead and upper
eyelid, with repeated seizures and left hemiparesis. The mother seeks your
advice about the skin lesion.
Your advice is to
A. start pulsed-dye laser therapy
B. start topical corticosteroid ointment
C. start calcineurin-inhibiting antiinflammatory agents
D. reassure the mother that it will fade with time
E. advise the mother to use masking cosmetics

15. Which of the following sites of infantile hemangioma


seem to persist most
often?
A . Lip
B. Check
C. Scalp
D . Back
E. Anterior chest
infantile hemangioma of the
enlarging
16 . A 6- month- old infant presents With
.
lower eyelid obscuring the medial canthus for this infant is
trea tment
Of the following, the MOST appropriate
A . expectant observation
688
B. topical timolol solution
C. oral propranolol
D. oral corticosteroids injection
E. intralesional corticosteroid
following drugs decreases the blood level of propranolol?
17. Which of the
A. Cimetidine
B. Ciprofloxacin
C. Isoniazid
D. Theophylline
E . Phenytoin

pallor, ecchymoses, and petechiae with


18. A 9-month-old boy presents with
rapid increase in the size of a large vascular lesion
on axilla. The platelet count is
depressed, but the bone marrow contains increased
numbers of normal
megakaryocytes.
Of the following, the FIRST-line therapy is
A. surgical excision
B. systemic steroid
C. antiplatelet
D. antifibrinolytic
E. sirolimus

19. A 5- year- old child presents with a small red, glistening, sessile papule with a
weeping and crusted surface on left check.
Of the following, the MOST likely diagnosis is
A. spider angioma
B. angiokeratoma of Mibelli
C. pyogenic granuloma
D. spindle cell hemangioma
E. congenital hemangioma

20. A 10- year- old boy presents with 1- 3 mm red to blue -black papules with a
hyperkeratotic surface over the genitalia and in the umbilical and inguinal
regions. The boy gives history of recurrent episodes of fever, psrestheste,
agonizing pain, cyanosis and flushing of the hands and feet with hypohidrosis.
Of the following, the MOST likely diagnosis is
A . Fabry disease
689
Osler -Weber-
Rendu disease
l Maffucci syndrome
angiokeratoma of Mibelli
*

£ Kaposiform hemangioendothelioma

A B-year-old macules of few mill,meters in


boy presents with pigmented
21 -
, size
n the lips,
‘ buccal mucosa nose, hands, feet, and around
the mouth, with
of the nails. The mother gave
diffuse hyperpigmentation history of episodic
abdominal pain , diarrhea, and melena.
Of the following, the MOST likely diagnosis is
A. Gardner syndrome
.
B Laugier -Hunziker syndrome
C. Peutz-Jeghers syndrome
D. Cronkhite-Canada syndrome
E. multiple lentigines syndrome

22 . Which of the following has strong association with cafe- au- lait macules ?
A . Ataxia telangiectasia
B. Bloom syndrome
C. Fanconi anemia
D . Russell-Silver syndrome
E. McCune- Albright syndrome

congenital hip dysplasia, hirsutism,


syndromes .
.
mental retardat on, and cafe
- au spo

are features of which of the following


A. Johanson- Blizzard syndrome
B. Rubinstein-Taybi syndrome
C. Kabuki syndrome
D. Russell-Silver syndrome
E. Bloom syndrome
worldwide?
form of albinism seen
24. What is the MOST common 1)
albinism tyPe 1 ( OCA
A . Oculocutaneous
type 2 < OCA 2 )
B . Oculocutaneous albinism
C. Brown Oculocutaneous
albinism ( rufous albinism
)
D . Oculocutaneous albinism type ^
690
type 4 0 CA 4
albinism
E. oculocutaneous
ion of the skin,
• abnormal natural killer
.
eyes, and hair; prolonged bleeding t mes a
cell function - a
* /
and
easy bruising which of the following syndromes
features of ?
neuropathy are
periipheral
A. Hermansky-Pudlak syndrome
B. Chediak-Higashi syndrome
C. Bloom syndrome
syndrome
D. Cross-McKusick -Breen
E. Johanson-Blizzard syndrome

sharply demarcated amelanotic patches on


26. A 5- year- old boy presents with
the forehead and anterior scalp with a white
forelock, there are Islands of
darker pigmentation within the amelanotic areas.
Of the following, the MOST likely cause of this disorder is
A. piebaldism
B. vitiligo
C. nevus depigmentosus
D. Waardenburg syndrome
E. oculocutaneous albinism type 2

27. What is the hallmark of Waardenburg syndrome type 1?


A . White forelock
B. Depigmented skin
C. Deafness
D. Heterochromia irides
E. Limb abnormalities

28. An 8- year- old boy presents with bizarre patterned, hypopigmented macules
arranged over his abdomen in sharply demarcated whorls and streaks. The
palms, soles, and mucous membranes are spared with no history of
inflammatory or vesicular lesions preceding the development of the pigmentary
changes. The patient is hypotonic with nystagmus and intellectual disability.
Of the following, the MOST likely cause of this disorder is
A . oculocutaneous albinism type 3 OCA 3 (rufous albinism )
B. hypomelanosis of Ito
C. systematized nevus depigmentosus
D. incontinentia pigmenti
691
j-Harada syndrome

-
- K Vanagl
l V0
gt
°

t,v «*calcium.
**" ° *•
' **
* "'PO PM I SMStie . ***
vea
following, the MOST appropriate treatment is
0 the
A. topical steroid
8. topical pimecrolimus
C narrow-band ultraviolet light
0. cover-up cosmetics
E. minimizing sun exposure of affected areas

30. Which of the following is the MOST


common implicated factor in the
etiology of erythema multiforme?
A. Sulfonamides
B. Herpes simplex virus
C. Mycoplasma pneumoniae
D. Amoxicillin
E- Acetaminophen

31. A 12-year-old boy develops abrupt, fixed papules with a central dusky zone
0n the palms
and extensor upper extremities, followed by oral lesions on
vermilion border of the lips and buccal mucosa. The boy gives history of
herpetic mouth lesion 10 days ago.

°* the following, the MOST likely diagnosis is


A. Stevens-Johnson syndrome
B. pemphigus vulgaris
C. erosive lichen planus
D. erythema multiforme
E. Behcet syndrome
necrolysis from
differentiate toxic epidermal
32. Which of the following
.
Stevens-Johnson syndrome
A. Skin tenderness
B. Corneal ulceration
C. Pneumonitis
D. Mouth ulcers 692
E Genital ulcers

33. Which of the


foMowing should
be considered in early management
,«veri,
of S
Johnson syndrome ?
A. Systemic antibiotics
B. Corticosteroids
C. IV immunoglobulin
D. Azathioprine
E. Dapsone

34. A 2 -year -old child presents with easy blistering on palms and sol
manifested when the child begins to walk, also the patient has oral ^ mucosa
erosions.
Of the following, the MOST likely diagnosis is
A. epidermolysis bullosa simplex generalized (formerly Koebner )
B. epidermolysis bullosa simplex localized ( formerly Weber -Cockayne )
C. epidermolysis bullosa simplex Dowling-Meara
D. epidermolysis bullosa simplex with mottled pigmentation
E. epidermolysis bullosa simplex superficialis

35. A 5 -year -old boy with past history of painful oral ulcers for the last 6
weeks
presents with large, flaccid bullae on nonerythematous skin
of the face and
trunk. The lesions rupture and enlarge peripherally, producing painful, raw,
denuded areas that have little tendency to heal. Examination shows malodorous
granulomatous lesions at sites of ruptured bullae
with positive Nikolsky sign.
Of the following, the MOST likely diagnosis is
A erythema multiforme
B. bullous pemphigoid
C. Stevens-Johnson syndrome
D. pemphigus vulgaris
E. toxic epidermal necrolysis

.
36 What is the best initial
treatment for pemphigus vulgaris ?
A . Systemic methylprednisolone
B. Azathioprine
C. Cyclophosphamide
D. Methotrexate
E. IVIG

693
. A 7-ycar-old boV presents with symmetric, grouped’ small • 6 .
and vesicles on both knees and elbows with ha ' nselV P^ritic
i
^ ^
° '
eSi
and soles. The boy looks short, thin, with chronic bd m,nal °ns on
Pain and
rrtiea -
^Of the following, the MOST likely diagnosis is
A. scabies
B. papular urticaria
C dermatitis herpetiformis
0. contact dermatitis
E. papular eczema

38. What is the MOST common type of eczema in children?


A. Seborrheic dermatitis
B. Irritant contact dermatitis
C. Nummular eczema
0. Atopic dermatitis
E. Acute palmoplantar eczema

39. What is the first-line therapy for diaper dermatitis?


A. Topical sucralfate
B. Low-potency topical corticosteroids
C. Zinc oxide ointment
D. Topical anticandidal agent.
E. Topical preparations containing
,
ino one-nystatin

40. What is the recommended, ,, „e {0r earlobes piercing?


ag
A. immediately after birth
B. One month
C. First birth day
O. After 10 year of age

——
t. No age limit
cause of allergic contact
common
, the
-
. following th® MOST

irsi* •

iSS» 694
temporary tattoos
E Dye in
presents with hypopigmented, ill-defined r
42. A 5- year -old finely scaly patches on the face with minimal
boy

erythematous, and
eruption is exacerbated
dryness .
by
ltc nSi
^ J t

, the MOST likely diagnosis is


Of the following
A. vitiligo
8. tinea versicolor
C. tinea corporis
D. pityriasis alba
E. contact dermatitis

43. A young mother brought her 3 -month- old well looking smiley baby
with
diffuse scaling and crusting of the scalp followed by a greasy, «scaly
,
erythematous papular non-pruritic dermatitis involving the face, neck,
and
retroauricular areas. The mother is asking about the prognosis of this lesion
What is your MOST appropriate answer ?
.
A It resolves by 1 year
B. It resolves by late childhood
C. It will persist through adulthood
D. It reflects systemic dysfunction of the immune system
E. It may progress to psoriasis

44. Which of the following drugs may cause photoallergic drug


eruptions ?
A. Nalidixic acid
B. Furosemide
C. Sulfonamides
D. Naproxen
.
E Piroxicam

45. A 2-year -old child presents


, with erythema, scaling, crusting, freckles, and
eratosis on sun - exposed areas of the
body following minimal UV exposure, the
patient also has
photophobia, lacrimation, blepharitis, and sensorineural
deafness.
Of the following, the MOST
likely diagnosis is
A. Cockayne syndrome
B. trichothiodystrophy
C. xeroderma pigmentosum

695
D. ctinic prurigo
3

l polymorphous light eruption


boy had history 0f p|
A 5-year old
-
46. aques f
on the face, neck, dorsal portions
thTh„! ;‘
^
ln 3
distribution of butterfly


age of 3
started fromatrophic, months. These lesions are re i W and buttocks
,
reticulated hyperpigmented and gradually by
« telangiectatic
e
eyelashes
cstsracts.
, and
^
“* alopecia.
" with
Eye examination

Of the following, the MOST likely diagnosis is


I sparse eyebrows
reveals subcapsular
bilateral
A. Bloom syndrome
B. Rothmund-Thomson syndrome
C. cockayne syndrome
D. xeroderma pigmentosum
E. actinic prurigo

47. A 10-year-old boy presents with an eczematous eruption on the face and
extremities in a glove-and- stocking photo distribution followed by
hyperpigmentation and hyperkeratosis intensified by further exposure to
sunlight . He is mentally intact with episodic cerebellar ataxia. The mother
mentioned that these cutaneous signs initially developed during the early
months of life. The urine contains increased amounts of monoamine
monocarboxylic amino acids.
Of the following, the MOST likely diagnosis is
A. pellagra
B. Hartnup disease
C. ataxia- telangiectasia
D. xeroderma pigmentosum
E. hydroa vacciniforme

48. A young child, after few


papules
acute eruption of many ierythematous
_
and face . with sharply demarcated,
the trunk result in pinpoint
papules smaller than 1.5 cm on , when iremoved
silvery scale
irregular borders and a thick
bleeding. •
is
likely diag
Of the following, the MOST
A . viral exanthems
696
B. secondary syphilis
C. pityriasis rosea
D. pityriasis lichenoides chronica
E. guttate psoriasis

topical agent for psoriatic skin lesions ?


49. What is the first line
-
A. Tazarotene
B. Tar preparations
C. Salicylic acid
D. Calcineurin inhibitors
E. calcitriol

50. A 15- year-old dark- skinned boy develops a widespread, symmetric erupts
involving mainly the trunk and proximal limbs that are round, pink to brown <1
cm in diameter, slightly raised, and covered by a fine scale. He also gave history
of a 7 cm annular patch with raised border and fine, adherent scales on the
chest, 7 days prior to this eruption.
Of the following, the MOST likely diagnosis is
A. drug eruptions
B. viral exanthem
C. guttate psoriasis
D. nummular dermatitis
E. pityriasis rosea

51. A 13- year-old boy presents with small, firm, skin-colored, warty papuie.
changed to yellow, pruritic, malodorous, greasy crusts coalesced forming la
symmetrical, gray- brown, plaques on the scalp, face, chest, limb flexures,
groin with hyperkeratosis of the palms and soles and nail dystrophy ,

lesions worsen in summertime.


Of the following, the MOST likely diagnosis is
A. seborrheic dermatitis
B. acanthosis nigricans
C. Hailey-Hailey disease
D. Darier disease
E. flat warts

52. A 4- year-old child, 1 week after a non - specific viral illness , develops
numerous, flat- topped, red variable size papules on the face, buttocks, and
697
, sparing the trunk and scalp. There k ,
aiso a of papules on
limbs of minor local trauma. the right
in at site
shin
following, the MOST likely djapnr
800,SISls
Of the
A. Gianotti-
Crosti syndrome
B. erythema infectiosum
C. lichen planus
D. erythema multiforme
£. Henoch-Schonlein purpura

53. A 10-year -obese female with diabetis mellitus presents with symmetric,
hyperpigmented, velvety, hyperkeratotic plaques with exaggerated skin lines in
the posterior neck and axillae .
Of the following, the MOST likely diagnosis is
A. reticulated papillomatosis
B. Addison disease
C. acanthosis nigricans
D. pellagra
E. erythrasma

in January
54. An 11- month-old female baby brought by her mother
surface and scaling most
complaining from slight roughening of the skin
abdomen, neck, and face.

organ systems.
is
Of the following, the MOST likely diagnosis
A. X-Linked Ichthyosis
B. ichthyosis vulgaris
C . Harlequin ichthyosis
D. lamellar ichthyosis
E. epidermolytic ichthyosis
pruritic ichthyosis accentuated
genpralized
.
.
55 . A 2-year- old child with
history o
Dajmoplantar hyperkeratosis retinal
on the flexures and the lower
abdomen . speech and primary
diplegia ( eukotriene B
4 ( LTB4).
The patient has spastic of
e
degeneration. Urinalysis reveals diagnosis 1S•

likely
Of the following, the MOST
A . Netherton syndrome
B. Sjogren- Larsson syndrome
698
C. Refsum syndrome
D. chondrodysplasia punctate
E. CHILD syndrome
history of generalized ichthyosis from
56 A 15-year-old boy had early
paralysis ataxia retmitfe
childhood, presents with progress

values. ^
Of the following, the MOST likely diagnosis is
A. Netherton syndrome
^ ^
piigmentos
*

B. Sjogren-Larsson syndrome
C. Refsum syndrome
D. chondrodysplasia punctate
E. CHILD syndrome

57. A 6- month-old infant presents with mild hyperkeratosis, cataract,


hypertelorism, optic nerve atrophy, saddle nose, frontal bossing,
disproportionate shortening of the proximal extremities, psychomotor
retardation, failure to thrive, and spasticity; X - ray shows stippled humeral head
epiphysis.
Of the following, the MOST likely diagnosis is
A. chondrodysplasia punctate
B. keratitis with ichthyosis and deafness ( KID syndrome )
C. Chanarin-Dorfman syndrome
D. Papillon-Lefevre Syndrome
E. Richner-Hanhart syndrome

58. A 15- year-old diabetic female presents


with irregularly shaped, sharply
demarcated, yellow, sclerotic plaques with and a
central telangiectasia
violaceous border located on the shins . These
skin lesions started as
erythematous papules 3 months ago.
Of the following, the MOST likely diagnosis is
A. necrobiosis lipoidica
B. xanthomas
C. morphea
D. granuloma annulare
E. pretibial myxedema

699
'old b V
0 presents With
-Vear brawnv ^
*
5 A 7
ds rapidly
to involve the thorax and
. The involved areas feel induraw
armTrT °face' thehas Ja
and necl( »•
*
**
t not
i
-
ppearance
*?' 1

7 days ago.
pharyngitis
*
historv of
^
« atrophic. The patient gives n. T h, V 8 skin « normal , 3renot
r of Proceeding
^ febri(
color and
rept roccai " °
Of the following, the MOST likely diagnosis ls
A. morphea
B. myxedema
C. scleredema of Buschke
D. dermatomyositis
E. subcutaneous fat necrosis

60. What is the classic sign of lipoid proteinosis?


A. Hoarseness
B. Beaded papules on the eyelids
C. Patchy alopecia
D. Enlarged firm tongue
E. Diffusely thickened palms

of
61. What is the MOST common symptom seen in the classic infantile type
urticaria pigmentosa ?
A. Intense pruritus
B. Headache
C. Colic
D. Diarrhea
E. Flushing
mediator release in
mast cell
62. Which of the following may exacerbate
patients with mastocytosis ?
A . Macrolids
B . Cephalosporins
C. Vancomycin
D. Penicillins
E. Sulphonamides , Joints
bruising,, velvet skin
K Ttv
easy
of
: SkinJ hyperelasticity, joint
63 .
hVper *° prolapse are features
mitra
hypermobility, scoliosis, and
700
syndrome
A. Loeys-Dietz
B. MASS syndrome
r cutis laxa
o pseudoxanthoma elasti cum
Syndrome
E. classic Ehlers-Danlos
agents is a common cause of
64 . which of the following Infectious erytheinj
nodosum ?
A. Group A streptococcus
B. Yersinia
C. Mycoplasma
D. leprosy
E. Brucellosis

65. A 2-week -old full-term infant, product of difficult vaginal labor with previous
admission to NICU for 3 days, presents with asymptomatic indurated,
erythematous sharply demarcated plaque on the right cheek.
Of the following, the MOST likely diagnosis is
A. sclerema neonatorum
B. panniculitis
C. cellulitis
D. hematoma
E. subcutaneous fat necrosis

66. A 7- week -old infant with perinatal history of


difficult vaginal breech delivery,
repeated seizures 12 hours after birth contro
lled by phenobarbitone, developed
indurated, erythematous sharply demarcated plaque
on the right shoulder,
presents with lethargy, poor
feeding, vomiting, failure to thrive, irritability, and
seizure, electrocardiography reveals shortening
of the QT interval.
Of the following, the MOST approp
riate investigation is
A. lumbar puncture
B. blood glucose
C. serum calcium
D. blood culture
E. ultrasound brain

67.Munchausen syndrome by proxy should be considered in


which of the following conditions ? young children in

701
FactitiaI panniculitis
^B. Pernio
C. Cold
urticarial
D. Cold panniculitis
E. Buccal cellulitis
. 10-year-olci diabetic girl on insulin presents
68 with thin emaciated
rties and gluteal region with normal appearance of
e<t* face, neck, and trunk
r with golden yellow papules on both elbows. Blood
tests reveal
hvoerlipidentia.
Of the following, the MOST likely diagnosis is
A Type l familial partial lipodystrophy
B. Type 2 familial partial lipodystrophy
C. Type 3 familial partial lipodystrophy
0. Type 4 familial partial lipodystrophy
E. Type 5 familial partial lipodystrophy

69. Which of the following intoxications produces necrosis of sweat glands,


resulting in anhidrosis with or without erythema an
A. Lead
B. Atropine
C. Scopolamine
0. Diazepam
E . Salicylate

70. Which of the following drugs may cause hyperhidro


A. Ciprofloxacin
B. Theophylline
C. Azithromycin
D. Diazepam
E. Valproic acid
?
21. Which of the following have drama * response in miliaria
A. Cooling
B. Antipyretics
C. Topical steroids
D. Topical clindamycin
E. Topical aluminum
702
. <eroa\e Pte ents mutepe
72.
'
esses then aw

e
**
*V

73. Which of the following drugs causes hypertrichosis ?


A. Phenytoin
B. Valproate
C. Phenobarbitone
0. Diazepam
E. Ethosuximide

74. Which of the following conditions may be complicated by trichobezoars ?


A. Traction alopecia
B. Hair pulling
C. Trichotillomania
D. Tinea capitis
E. Alopecia areata

75. A 4-month- old infant brought by his young mother with comp am
fussiness and loss of hair from occipital area for the last 2 weeks, the a ^
bottle fed, diagnosed to have atopic dermatitis of the face, 1month ago a
low potent topical steroid.
Of the following, the MOST likely cause of this hair loss is
A. anagen effluvium
B. telogen effluvium
C. alopecia ophiasis
D. alopecia areata
E. anhidrotic ectodermal dysplasia

703
the following is a possible etiology of
76 - Which of telogen effluvium ?
Chemotherapy
l Radiation therapy
c. tead
p Colchicine
l discontinuation of high-dose corticosteroids
following cause diffuse milky white nails?
77, Which of the
A. Anemia
B. Hypocalcemia
C. Malnutrition
D. Pellagra
E. Zinc deficiency

78. Which of the following conditions is characterized by large nails ?


A. Rubinstein-Taybi syndrome
B. Ectodermal dysplasias
C. Ellis-van Creveld
D. Incontinentia pigmenti
E . Otopalatodigital
by pseu doclubbing?
79. Which of the following conditions is characterized
A. Gardner' s syndrome
B. Maffucci' s syndrome
C Apert ' s syndrome
0. Diamond' s syndrome
E. Pachydermoperiostosis
nails (half -and
-
with Lindsay
80 . Which of the following conditions is associated
half nails)?
A. Renal disease
B. Cirrhosis
C. congestive heart failure
D. Hypoalbuminemia
E. Lichen planus ?
nail syndrome
3 ted with yellow
81. Which of the following diseases
« «8
°
A , Subacute bacterial endocardi
70^
histiocytosis
Langerhans cell
B.
C. Evere
rheumatoid arthritis
D. Cirrhosis
E. Bronchiectasis
ing
followm drugs is a cause of onycholysis ?
82 . Which of the
A. Cyclophosphamide
B . Adriamycin
C. Vincristine
D. Methotrexate
E. 6-mercaptopurine

83. A 10-year-old boy presents with longitudinal ridging, pitting fr


thinning, distal notching, and opalescent discoloration of all the
patient has no associated skin or systemic diseases.
S
'
Of the following, the MOST likely cause of these nail changes is
A. Trachyonychia
B. Onychomycosis
C. Acute paronychia
D. Onycholysis
E. Koilonychia

84. A 5- year-old girl presents with multiple sharply


demarcated, irregular,
smooth red patches surrounded by an elevated
white border on the dorsum of
e tongue. The pattern of lesions
changes over days and is asymptomatic but
may cause a burning sensation to
cold foods.
Of the following, the MOST
likely cause of these tongue changes is
A. black hairy tongue
B. geographic tongue
C. fissured tongue
D. Canker sores
E. oral hair leukoplakia

85- A 5-year-old
boy presents with a
an insect bite, tiny pustule on the skin of right check a
which rapidly progress
into a honey-colored crusted P a<fu
cm in diameter
a enopjithy,
with little pain, no
CBC reveals WBC
'
constitutional symptoms but submandib
^
the following, 16000/cmm with 70% neutrophils,
the MOST likely diagnosis
is
705
herpes simP ex^
^0 varicella - zoster

C tinea
corporis
D. kerion
£ nonbullous impetigo

86. Which empirical antibiotics are used iin


neonates with cellulitis?
A. IV vancomycin and gentamicin
B. IV gentamicin and cefotaxime
C. IV ampicillin and gentamycin
D. IV nafcillin and vancomycin
E. 1 V clindamycin

87. A 2-year-oid boy develops


cellulitis of left palm after an accidental trauma,
he has no fever, lymphadenopathy
, or other constitutional signs.
What is the initial antibiotics
treatment?
A. clindamycin
B. trimethoprim-
sulfamethoxazole
.
C cephalexin
D. amoxicillin
E. azithromycin

i ico fever irritability, and exquisite

of diffuse flaccid blisters and eros ons. followed by large

. .,
and
crusting and fissuring around the eye ,
sheets of epidermis peel away.
^
Of the following, the MOST likely d agnos* s s
_
A. bullous impetigo
B . epidermolysis bullosa
C . pemphigus
D . staphylococcal scalded skin syndrom
E . toxic epidermal necrolysis
boy develops a red macule on the
89. A poorly controlled 10- year - oId diabetic with black, depressed, crusted
right buttock that vesiculates and then ulcerates

706
surrounding im of raised violaceous edges. CBC
rim
center and a ' eveais
°
Ofthe foTo w'
3
ngd JeMOSTlikely cause of this lesion is
A. S. aureus
B. Enterobacter spp.
C. P. aeruginosa
D. Burkholderia cepacia
E. Serratia marcescens

90. What is the MOST common cause of blistering distal dactylitis?


A . Group A streptococcus
B . S. aureus
C. H. influenza
D. Salmonella
E. S. epidermidis

91. A 7- year-old boy presents with superficial, erythematous, well marginated


nonindurated, tender, pruritic, and confluent rash from the anus outward. The
boy has rectal pain described as burning inside the anus during defecation, with
blood- streaked stools. His 5- year-old brother has the same complaint.
Of the following, the MOST likely diagnosis is
A . psoriasis
B. seborrheic dermatitis
C. pinworm infestation
D. perianal infectious dermatitis
E. sexual abuse

92. Which of the following furuncles sites may cause intense pain ?
A. Face
B . Axillae
C. Buttocks
D. Groin
E. External auditory canal
5 a
93. A 10- year -old obese diabetic boy presents with mildly pruritic,
mn
^^
demarcated, irregularly bordered, and slightly scaly brownish red pate
axilla . The diagnosis is readily made on Wood lamp examination.
Of the following, the MOST likely diagnosis is
707
Vsipel ,cl
e(
° a
B '
efVt iraSfT
* ,
tinea versicolor
C-
D seborrheic dermatitis
E erysipelas

04 A younB
child develops cervical lymphadenitis in the submandibular region
several weeks, ulcerate, and drain. The local reaction is
^ ich enlarges over
nontender and circumscribed with no constitutional symptoms or evidence of
lung or other organ
involvement .
Of the following, the MOST likely cause is
A. Mycobacterium scrofulaceum
B. syphilitic gumma
C. deep fungal infection
D. actinomycosis
E. hidradenitis suppurativa

95. A malnourished white adolescent male develops reddish brown macules


on the
covered with a fine scale enlarged forming non pruritic confluent patches
neck and upper chest.
Of the following, the MOST likely diagnosis is
A . seborrheic dermatitis
B - pityriasis alba
C. tinea versicolor
0 , pityriasis rosea
E . secondary syphilis

%. A 6- year -old black boy develops a small


tKerr) atous
follicle then spreads peripherally, forming a " erJpd broken (
with a confluent
plaque within which the hairs become britt
Pruritic patch of alopecia .
f the following, the MOST likely diagnosis is
° A. seborrheic dermatitis
B. psoriasis
C* alopecia areata
D - tinea capitis
E- trichotillomania

708
nf? with a dry, mildly erythematous, elevated, Sc,|v
boy centrally forming an annular lesion 0n
97. A young cent 6
and clears
plaque spreadin g 2 weeks The lesion
does not fluoresce vyith
right antecu bital area ro ^
Wood lamp - , MOST likely diagnosis is
, the
Of the following
A. granuloma
annulare
8 nummular
eczema
C. pityriasis rosea
D. tinea corporis
migrans
E erythema chronicum

diaper dermatitis ?
98. What is the hallmark of candidal
A. Intensely erythematous confluent plaque
B. Satellite pustules
C. Involvement of inguinal folds
D. Involvement entire scrotum and penis
E. Involvement the vaginal mucosa and labia

99. Which of the following is key feature uniformly observed in kwashiorkor ?


A. Moon facies
B. Diffuse fine reddish brown scaling
C. Linear fissures and erosions
.
D Thin soft nails
E. Sparse, thin, and depigmented hairs

100. A 3-year-old boy presents with 4 discrete, pearly, skin-colored, dome-


shaped, papules, with central umbilication 2 to 4 mm in diameter on the right
check.
Of the following, the MOST likely diagnosis is
A. ectopic sebaceous glands
B. keratoacanthoma
C. juvenile xanthogranulomas
D. molluscum contagiosum
E. warty dyskeratoma

101. What is the classic lesion of scabies ?


A . Excoriated red papules
B. Threadlike burrows
709
C- Bullae
p. Wheals
E. Vesicles
year -old boy presents with intense generalized
l02. A 7- nruritus,
inflammatory pruritic excoriated papules localized worse at night;
to flexor aspects
axillae, and buttocks. of wrists,
Of the following, the MOST likely diagnosis is
A. dermatitis herpetiformis
B. eczema
C. pediculosis corporis
D. pityriasis rosea
E. scabies vulgaris

103. What is the treatment of choice for


scabies ?
A. Permethrin 5% cream
B. Sulfur ointment 5-10%
C. Crotamiton 10% lotion
D - Lindane 1% lotion
E - Benzoyl benzoate 10- 25% lotion

104. What is the treatment of choice for pediculosis capitis ?


A. Malathion
B. Spinosad
C. Benzyl alcohol lotion
D. Ivermectin
E. Permethrin
monotherapy for mild
as a topical
105 . Which of the following iis< recommended
acne?
A . Benzoyl peroxide
B . Erythromycin
C. Clindamycin
D . Retinoid
treatment of
£ Adapalene effective in the
ts MOST
of the following
antibiotics
106 . Which
acne ? 710
A . Tetracycline
B. Doxycydine
Q Erythromycin
p. Azithromycin
E Amoxicillin
following vitamins can induce acneiform lesions in susceptibi
107. Which of the *
individuals ?
A. B 12
B. E
C. C
D. D
E. A

108. A 2-year -old otherwise normal white boy brought to you because of a firm,
dome- shaped, orange papule, 0.7cm in diameter on upper trunk with few cafe-
au-lait macules and normal blood lipid values.
Of the following, the MOST likely diagnosis is
A. papulonodular urticarial pigmentosa
B. dermatofibromas
C. xanthomas of hyperlipoproteinemia
D. juvenile xanthogranuloma
E. dermatofibroma

109. A 19-month-old female presents with asymmetric truncal lipomatous mass


since birth with macrodactyly, linear epidermal nevus, and scoliosis.
Of the following, the MOST likely diagnosis is
A. Proteus syndrome
B. Klippel-Trenaunay syndrome
C. Bannayan- Riley- Ruvalcaba syndrome
D. CLOVES syndrome
E. Gorlin syndrome

weanec recently from breast milk to cow's milk due to


maternal m I lgnancy
^
°^ *
° Presents with chronic diarrhea, stomatitis, failure to
thrive and CU aneous' eruPtion in the form of vesiculobullous, eczematous, dry,
scalv |PC Symmetrica|ly distributed in the perioral, acral, and perineal areas
,
with uS
sparse hair. He also has conjunctivitis and blepharitis.

711
Of the following, the MOST lit
A. biotin elvnUtrient
B. vitamin B2 deficiency is
C . vitamin B6
0. essentially
E . zinc

712
Chapter 30j \
Skin
Answers

.
l(B) . ^, complete fingernail regrowth takes 6
relatively slow
2 (C) Na I V ^th is
' ° an regrowth req
_
12 18 mo. m0
while complete
, “loc at ed ^ tlmvelogenous
^
3 (E) Mal g a commonly asso
° cies, especially a leukemia It.
ciated with

^" TJe during, or after the

.
manifesu

—sss
COurse ancj 1S
characterized
v ematous, edematow M
tender, eryth that may be pustular b *
panied by teve ,


^ and leukocytosis Oral ulcers
; ?'S i« syndrome than in
other

4 (D) The majority



are mild morbilliform or
exanthematous eruptions of little clinical consequence
. Identifying the suspect
medication may be difficult owing to the
many medications used in
immunosuppressed patients. Features
that may help identify suspect
medications include rash onset relative to expo
sure, character of distribution
and spread, associated symptoms,
and laboratory data. Medication eruptions
begin on the trunk 7- 10 days
after exposure; they spread peripherally and are
associated with pruritus and, less
commonly, with fever, arthralgia, and
lymphadenopathy. Eosinophilia may
support a diagnosis of drug eruption but
may be absent in the
setting of bone marrow suppression. Penicillins, sulfa
drugs, cephalosporins,
nonsteroidal antiinflammatory drugs, anticonvulsants,
and aminoglycosides
are common offenders. Medicatio eruptions may resolve
despite continued n
the offending agent, or they may progress to more
severe involvement.
.
5 (B)
.
6 (C) Drug rash ,
with s nophilia and
classically rseen 2-6 systemic symptoms (DRESS syndrome ) It is
(carbamazepine wk after initial exposure to an anticonvulsant
, phenobarbital
, phenytoin, lamotrigine) or other drugs

713
minocycline, sulfonamides

^
Idapsone,
ot CS d
0ften manifests as the triad of
fever, rash,sulf asalazine], other
V ' 'T
' e med icati ons inclu de andhepatitis,
7.1») Otfend torv drugs INSAlDs), and sulfonamides, tetra
' » cyclines,

3 w<
lflarnrTia acetaminophen.
neraUzed exanthematous pustulosis
nonsteroidal

rA. ^^
%.$ frcule is often drug-
aj inopenicillins, macrolides, related (most
commonW exposure. It is
sulf ona mid es ), occurring
n hours
10 ^ cha racterized by many nonfollicwithi ular sterile
ith underlying edema and erythema,
C typically
ntertriginous regions. Neutrophilia and fever 3 beginnin on the face
g

; .
eosinophil 3 is less common than in DRESS syndrome. The rash
'
uCous membrane involvement is rare and often mild.
re common, whereas
may burn or itch;
Internal organ
invol veme nt is not common and often iis asymptomatic. A pustular smear is
always indicated to rule out infection in the setting of leukocytosis, fever , and a
pustular rash. Therapy consists of stopping the causative drug and offering
symptomatic relief with moist dressings, emollients, and mid-potency topical
corticosteroids (applied twice daily for 1wk).
9.(C) Acute weeping lesions respond best to wet compresses, followed by
lotions or creams. For dry, thickened, scaly skin, or for treatment of a contact
a topical
allergic reaction possibly the consequence of a component of
helps to occlude a
medication, an ointment base is preferable, as it
usefu l for the scalp"and other
the affected area. Gels and solutions are most
hairy areas because of their faster absorption. h as neomycin
topic al
10.(C) The sensitizing potential of certain
and nitrofurazone, should be kept
in
most effective
^ ^
topical agents
^| erythromycin in'
Mupirocin, fusidic acid, and retaPal
currently available and are as
mild to moderate impetigo. Po
e
YS ’
(
as

^^*ora
baci traci
7 different
n are not as e
categ
° " ,
®
.
.
tajjs of
^moderate.
divi ded
J
^ can be used
11.(A) Cor tico ster oids can be *
put e e|,K n0
Strength, but for practical
high, and super

betamethasone,
,
^
^ ^^
^^" - m-
atl0ns inclu
poten

topic31
cy ‘
d
ds include

clobeta 5
one
' sndd
and

nCv

^ ^
-
triamcinolone. High P care -
toxicum occursless
in
diproPf crlbed with
are affected with
pustules
papules or

lesions
commonly . The
several sites or w » peak incidence occurs on the 2nd day of
:S
' ‘simay
^
S0 eS
the first few days as the rash waxes
lesion P during . and
for a few days to weeks in premature ,
inf
ApnChidrotlc
^Zl££ ectodermal dyspl asia, affec ted patients are.
episodes of high fever m warm environ
and may experience
be mistakenly considered
particularly comm on in infanc
appreciated. Diagnosis at this time
to be fevers of unknown origin.
y, when
may be
the
made
facial chan ges
using the starch-
**
This
are not
iodine
^^C1 ’
palmar or scalp biopsy . Scalp biops y is the most sensi tive and is 100% specifi . "
(
14.( A) The most effective treatment for capillary malformations is
with tt
pulsed-dye laser. This therapy is targeted to hemoglobin within the
lesion anc*
avoids thermal injury to the surrounding normal tissue. After such
treatment
the texture and pigmentation of the skin are generally normal without
scarfy
Therapy can begin in infancy, when the surface area of
involvement is smaller
There may be advantages to treating within the 1st year of life
. Although thrs
approach is quite effective, redarkening of the stain
may occur over time
making ongoing treatments useful. Masking cosmetics
may also be used.
15 .( A ) The course of a particular lesion
is unpredictable, but approximately 60%
of these lesions reach maximal involu
tion by 5 yr of age, and 90-95% by 9 yr.
Spontaneous involution canno
t be correlated with size or site of involvement,
but lip lesions seem to persist most
often.
16.( C) In a disfiguring, life-
or vision- threatening, or ulcerated IH that is not
responding to other treatment, oral
propranolol is the first -line treatment.
17.( E) Increased propra
nolol levels occur with inhibi CYP 2D6 (cimetidine,
tors of
amiodarone, fluoxetine, quinidine
, ritonavir ) and CPY 1A2 (cimetidine
ciprofloxacin, isoni
azid-, ritonavir, theophylline ); decreased blood levels oc<- u
.r' r"
ift
.
a abach
hepat c dru8 metabolism
‘ ( rifampin, phenytoin, phenobarbita
Phenomenon is a life- threatening combination o
^ 'iC Merritt
'

ranirii
ar8 ng Kaposiforrn
microlrngenTth hemangioendothelioma ( KHE ), thrombocyte
anemia, and an acute or chronic consump
1
*
^
^
hemo1
coa
ThZ ^
cllnical manifestations are usually evident u
”"
infancv
ThP
Vascular lesi n is usuall
viscera e a S 0 ° y cutaneous and is only rarev
e ^
ate<( in
.age
lesion^
ciated thrombocytopenia
accomoA may lead to precipitous 0f the
vascular
V ecchvmoses Petec
> hiae, and a rapid increase in t« iysis
evere anemia from hemorrhage or microangiopa hefn0
715
-( ( gatrnent includes surgical
excision of small |
,y el 11 * , because of coagulopathy . Additional
*difficult
'
jsions
, although
thiIS is
^
often ic steroids with or without vincri
Pharmacologic t
rist ne as first j, B reatments
me lude * ne therapy in most
cases - A py lemc granulomas (PG) are relatively
19.(0 ° the face , arms, and hands. Such commn .
on
partrcularlyappear as a subcutaneous
Jjnd may
a lesion
nodule. PGs may arise at si
loJedon",
6S m| ^ °r
history of trauma often cannot be elicited. urv’ but
A g,0k C rP riS D,ffUSUm (Fabpf
3

" ;rhaTa ° ? disease) is an inborn error of


metabolism (( c-galactosidase), angiokeratoma
corporis diffusum a
linked recessive disorder that is fully penetrant in males and is of
variable
.
penetrance in carrier females. Angiokeratomas appear before puberty
occur n profusion over the genitalia, hips, buttocks, and thighs and in the
umbilical and inguinal regions. They consist of 0.1-3.0 mm red to blue-black
and

papules that may have a hyperkeratotic surface. Telangiectasias are seen in the
mucosa and conjunctiva. Additional clinical manifestations include recurrent
episodes of fever and agonizing pain, cyanosis and flushing of the acral limb
areas, paresthesias of the hands and feet, corneal opacities detectable on slit *

lamp examination, and hypohidrosis. Renal involvement and cardiac


involvement are the usual causes of death.
21.(C) The Peutz-Jeghers syndrome is characterized bv melanotic macules on
fhe lips and mucous membranes and by gastrointestinal (61) polyposis it is
,

s oted
inherited as an autosomal dominant trait (STK11 gene). Onset ' " jn
infancy and early cbildbood
buccal mucosa
large as 1-2 cm.
ar|d vaginal mucosa . Cutaneous, lesions may
de p
- ^
" Ltm as

a| bands 0r
; an a
feet; around the mouth, eyes, and umbilicus olves the jejunum
diffuse hyperpigmentation of the nails . Polypos 5 us a v

' on ,
and rectum.

and ileum but may also occur in


Episodic abdominal pain , diarrhea
the St
, 01
°
^
'
n maCha 3Uii"
usception are frequent

complications.
22.(E) Strong association
tyPe
Neurofibromatosisafe
Multiple familial *“
^
'

syn e
legius (NF -l-hke )
syndrome
McCune- Albright sy odrome
repair cancer
Mismatch 716
S Ringc
y LEOPARD/ multiple (multiple hamartoma syndrome)
Cowden syndrome syndrome
Ruvalcaba
s Bannayan - Riley - f
23 (C ) nearly normal to closely resembling t
.
24 (8) OCA 2 ranges from
of albinism seen worldwid e ^
Little T 00 fto
,,
1 alb r
This is the most common
form .

is present at birth
, but pigment, particularly red - yellow *° meW
ma.
accumulate during childhood to
Pigmente d nevi
produce straw -colored or I h
may develop. Progressiv e ,* ,J
slc n
white individuals .
mProvemem in
visual acuity and nystagmus occurs with aging.
25.(8)
26.(A) A congenital autosomal dominant disorder, piebaldism is charact *
by sharply demarcated amelanotic patches that occur most frequently ^'
or
forehead, anterior scalp (producing a white forelock), ventral trunk, elbowT ^
within the amelanotic areas.
.
-
knees. Islands of normal or darker than- normal pigmentation mav

27 ( A) Waardenburg syndrome also manifests at birth as


depigmented skin and hair. There are four types of
^
be ore nt

localized areas of
Waardenburg syndrome.
The hallmark of Waardenburg type 1 is the white forelock,
which is seen in 20-
60% of patients. Only 15% of patients have areas of
depigmented skin. Deafness
occurs in 9-37%, heterochromia irides in 20%, and
unibrow (synophrys) in 17-
69% of those affected. Dystopia canthorum
(i.e., telecanthus ) is seen in all
patients with Waardenburg type
1.
28.(B) Hypomelanosis of Ito is a
rare congenital skin disorder affecting children
of both sexes that can
have associated defects in several organ systems.
Hypomelanosis of Ito is currently
a descriptive rather than definitive diagnosis.
The skin lesions of hypomelanosis
of Ito are generally present at birth but may
be acquired in the first yr
2 of life. The hypopigmentation remains unchanged
throughout childhood but
fades during adulthood.
29.(A) Localized
areas of vitiligo may respond to potent topical steroid, topi .
tacrolimus, or topical pimecrolimus ,
. n Patients with more extensive
involvement, narrnuu L.3n , ultrav olet light
B (UVB) (UVB311J is the treatment of
choice.
30. ( B ) Among the
'
mu tiforme (EM),
numerous factors implicated in the etiology of erythema
infection with herpes simplex virus (HSV) is the most common
(D ) EM is
characterized by an abrupt, symmetric cutaneous eruption, mos
mmonly on the
extensor upper extremities; lesions are relatively sparse

717
legs. Lesions can be seen
trunk, and n the palms and
3
lhe n often appears
01
initially as red macules or ° soles. The
erupt
' lv to form lesions up to 2 cm in
tent 11ifuga
-

',
^ ( A) pain

33 (C) IV
f 0m mucosal
absent

in early disease . Total dose


““'
>2
ration is often severe
in SJS, in contrast to pain in
minimal immunoglobulin (
to TEN.
IVIG; 1,5 -2 0 g/ kg/day x 3
g/kg has shown improved
but skin tenderness is

days) should be
but
considered
not static - „
significant outcomes in children compared with adults
34.( 8) EBS-localized (formerly Weber -Cockayne )
' t tlcally

predominantly affects the


hands and feet and often manifests when a child begins to walk;- onset
delayed until puberty or early adulthood, when heavy may be
shoes are worn or the
feet are subjected to increased trauma . Bullae 3re usually
. . ., restricted to the
hands and feet; rarely, they occur elsewhere, such as the
dorsal aspect of the
arms and the shins. The disorder ranges from mildly
incapacitating to crippling
at times of severe exacerbations,
35,10)
36.( A) The disease is best treated initially with systemic methylprednisolone 1- 2
m6/kg/ day . Azathioprine , cyclophosphamide, mycophenolate mofetil, and
methotrexate therapy all have been useful in maintenance regimens. IVIG given
•n cycles may be beneficial to patients whose disease does not respond to
steroids. Rituximab with IVIG replacement has been effective in the
management of severe pemphigus,
37.(C)
3fl.(D)
M.(C) First-line therapy for diaper dermatitis is application of a prctedree
barrier agent (ointment or paste) containing petroleum or zinc
diaper change . res

^
contact with jewelry, metal
40- (D) Nickel dermatitis develops from
clothing, or even cell phones• 5 provoking eruptions . The
gold jewelry
with even the trace amounts found in the earlobes from
nickel
most frequently involved sites from jew ry are
of sensitiiation, and
it
is
n® isk

^ ^
* ' '
containing earrings . Early ear pl
delay pierang (t vr of age. topical antibiotic
recommended to non prescription
41.1A) Neomycin sulfate is present
in
at an early age
,
_
e V exposed
are fre eventually disappear , and
preparations, and thus children
42.(0) Pityriasis alba lesions
normal pigmentation often
wax and
takes months ^
718
wan but
to return.
may begin in the 1st month of life and
.(A) Seborrheic
dermatitis
ic
43
self-resolves
by lYr
eruptions:
SJ delude
4410 Ph0t° tetracyclines, psoralens, chlorothiazides, sulf0n
, quinidine, phenothiazines
Slates. griseofufvin, thiazides

7 ''
' includenaproxen
^ nalidixic
. .
acid furosemide, no

^
, pirox cam), and h gh doses of
antiinflammatory agents (
causing photoallergic eruptions
is a rare autosomal recessive disorder
.
45 (C) Xeroderma pigmentosum
excision repair. Skin changes are
that
results from a defect in nucleotide first not ed
during infancy or early childhood in sun-
exposed areas though lesions may
occur at other sites, including the scalp. The skin lesions consist of erythema
scaling, bullae, crusting, ephelides (freckles), telangiectasia, keratoses, basal and
squamous cell carcinomas, and malignant melanomas. Interestingly, although
most patients experience exaggerated acute sunburn reactions following
minimal UV exposure, up to half of affected patients do not and instead develop
progressive freckling. This difference in presentation depends on genetic
subtype. Ocular manifestations include photophobia, lacrimation, blepharitis,
symblepharon, keratitis, corneal opacities, tumors of the lids, and possible
eventual blindness. Neurologic abnormalities such as cognitive deterioration
and sensorineural deafness develop in approximately 20% of patients.
.
46 (B) Rothmund-Thomson syndrome is also known poikiloderma congenital
because of the striking skin changes. It is inherited as an autosomal recessive
trait.
.
47 (B) Hartnup disease is a rare inborn error of metabolism with autos
recessive inheritance. Neutral amino acids, including tryptop an >
transported across the brush border epithelium of the intestine an 1
^
0f
to mutation of the SLC6A19 gene encoding .
the transporter Admmis tjon
nicotinamide and protection from sunlight results in improvemen
cutaneous and neurologic manifestations .
^^
48.(E) Guttate psoriasis has
also been observed after perianal streptncocdlCj
infection, viral infections, sunburn, and withdrawal of systemic cortic er 0
therapy or tumor necrosis factor ( ^
TNF) -a inhibitors.
.
49 (E) The first-line topical
agents for lesions on the body are emollientS
/

vitamin D analogs (calcipotriene or


for children), and mid- to high-
calcitriol, although calcitriol is less i
potency corticosteroids.
^
719
— ^
, common eruption
This benign

-
occurs
::
: zz:zss
m

<%
pate
the
to
- ~
body - Herald patches are generally |arger than
,
10 cm n diameter' ev are annu|ar in
h fine adherent
* patch
,
1
scales .
configuration JUT '
Approximately 5- 10 dav I
, a widespread, symmetric eruption
UZ
uch

" Snd Varv


:rr
ssasa
aV occur anywhere
from 1
a raised border
t
appeara « of
Ve
and

Qn

herald
evident. Lesions may appear 1
in T
," the

SUB) Darier disease usually occurs in late childhood "° ** *" days.
proximal limbs becomes
for ^ PS
runk and

life Several exacerbating triggers have been identified


. ‘ throughout
exposure heat, friction surgery, and
JSZttZSf *
chrome relapsing course that usually worsens in summertime
52.IA) Gianotti-Crosti Syndrome (Papular Acrodermatitis),this
eruption is benign and predominantly occurs in children younger
distinctive
than 5 yr old
about 1 wk after a viral illness. Cases are usually sporadic, but epidemics have
been recorded. Lines of papules (Koebner phenomenon) may be noted on the
extremities following minor local trauma. The eruption resolves spontaneously
but may take up to 2 mo.
53.(C) The clinical severity and histopathologic features of acanthosis nigricans
correlate positively with the degree of hyperinsulinism and with the degree of
obesity.
54.(B) Ichthyosis vulgaris is the most common of the disorders of keratinization,
yr of life.
with an incidence of 1/ 250 live births. Onset generally occurs in the 1st
of ichthyosis,
$5.(B) The clinical picture of Sjogren- larsson syndrome consists

lcht S.
rr
° “ thyVs
-rsCES
in the 2nd or 3rd decade of life. The
mild, and resembles ichthyosis vulgaris
the palms and soles. Chronic polyneuritis
.
^
whh l
progress
may a|S0 be localized to
ive para|ysis and ataxia
and
,

retinitis pigmentosa, anosmia,


electrocardiographic changes are
57.( A) These heterogeneous disonmi
changes. Nearly all patients with
the

t e
mos
^^ gracteristic features.

d
marke £ j by ichthyosis
(jminant lorw

cutaneous
and bone
and approximately
lesions, ranging rom
type have
25% of those with the recessive hyperkefatosi5. Rhizomeltc
an ^ ^ , optic nerve
atrop y ,
severe, generalized erythema
cataract VP ,
hypertelorism
eKtremities, psychomotor
CPD is associated with
disproportionate shortening
^ 720
^
thrive, and spasticity; most affected
retardation, failure to Patient d
infancy.
%
manifests as erythematous papules that
SSLJ ‘shapefsharplv evolve In
demarcated, yellow, sclerotic plaques with .
,
,<i and a violaceous border . Scaling, crusting, and i
,Ce-
trj ^
"
e ang
' deration
'
frequent. Lesions develop most
commonly on the shins.

59.(C)
60.(0) Lipoid proteinosis
may be noted initially in early infancy as
hoarse
"
Skin lesions appear during childhood and consist of yellowish papules
nodules that may coalesce to form plaques. The classic sign is beaded Papules
on the eyelids.
61.(E) Systemic signs of histamine release, such as anaphylaxis -like episodes
^
^

hypotension, syncope, headache, episodic flushing, tachycardia, wheezing, colic


and diarrhea, are uncommon and occur most frequently in the more severe
types of mastocytosis. Flushing is by far the most common symptom seen.
62.(C) Pharmacologic Agents and Physical Stimuli That May Exacerbate Mast
Cell Mediator Release in Patients With Mastocytosis
Immunologic Stimuli
Venoms (immunoglobulin E-mediated bee venom )
Complement-derived anaphylatoxins
Biologic peptides ( substance P, somatostatin)
J Polymers (dextran)
Nonimmunologic Stimuli
Physical stimuli (heat, cold, rubbing, trauma, sunlight )
S Acetylsalicylic acid and related
nonsteroidal analgesics *
S Thiamine
* Ketorolac tromethamine
4 Alcohol
^
v
Vancomycin
Dextromethorphan
Narcotics ( codeine, morphine) *
Radiographic dyes (iodine containing)
Emotional Issues
Anxiety
Sleep deprivation
* Stress
63.(E) EDS e disorders,
the hereditary conne
represents a portion of
many of which have unique
features that enable clinica
721
^ The
gSSHSSSS7 ft,'ci
^ changes), cutis laxa, and pseudoxanthoma elasticum ’’ keleta chan««.
$

cutis laxa hangs in redundant folds whpJl" ge eral he ski o


eSk" f ,hwewith
"'
atients with
'
afi is hype
that
rexte nsib
impac t
le
the
and snap
integ rity
s
of
back into niacp
the connective
'" ° K
0,her
disorders and osteogenesis imperfecta or man SUC a$ exposure
t0 corticosteroids
(Bethlem myop
indistingu ishab le
athy, Ullrich congenital
in the early stages of disease.
muscular V
* ,_ “ ”•*
d 0fd
phy can

64.( A ) Etiology of Erythema Nodosum


Viruses
Epstein-Barr, hepatitis B mumps
;

Fungi
Coccidioidomycosis, histoplasmosis, blastomycosis, sporotrichosis
Bacteria and Other Infectious Agents
Group A streptococcus,* tuberculosis,* Yersinia, cat -scratch disease, leprosy,
leptospirosis, tularemia, mycoplasma, Whipple disease, lymphogranuloma
venereum, psittacosis, brucellosis
Other
Sarcoidosis, inflammatory bowel disease,* estrogen-contain
ing oral
syndrome, severe acne,
contraceptives,* systemic lupus erythematosus, Behcet
, echinacea, Sweet
Hodgkin disease, lymphoma, sulfonamides, bromides
syndrome, pregnancy, idiopathic *
(SCFN) is unknown. The disease
m
.
65 (E) The cause of subcutaneous fat necro sis
various perinatal compkcaom
infants may be a result of ische mic
,
injury
birth
from
trauma, asphyx , and
, proksnged
such as maternal preeclampsia
neonata encep a
.
hypothermi Whole -
a body cooling for
associated with SCFN. comp licatio n of subcutaneous fat
necrosis
66.(C) A rare but potentially serious
(SCFN) is hypercalcemia . addre ss the primary reason for
67.(A) Treatment of factitial pann
iculitisis must by proxy
act Munchausen syndrome
the performance of this self -dest
ructive
children .
should be considered in young
«MA) antic holin ergic s ( drugs such as atropine
of the swea t gia , Acute intox ,cation with
69.(D) At the level t e
g|ands .
and scopolamine ) may paral
yze

722
/ infantile scurvy
ovascular:
rardi
C
/ Heart failure
/ Shock
/ vasomotor
/ Cold injury
/ Raynaud phenomenon
/ Rheumatoid arthritis
Neurologic:
/ Abscess
/ Familial dysautonomia
/ Postencephalitic
/ Tumor
Miscellaneous:
^ Ch£diak-Higashi syndrome
^SCompensatory
Lymphoma
^
Phenylketonuria

*
Vitiligo
J Frey syndrome
Medullary
S Physiologic gustatory sweating
S Encephalitis
S Granulosis rubra nasi
S Syringomyelia imury
Thoracic sympathetic trunk
Spinal
Cord transection
Syringomyelia
Changes in Blood Flow
Maffucci syndrome
S Arteriovenous fistula
Klippel-Tr£ naunay syndrome
of the patient by
Glomus tumor nevus syndrome t0 COoling clothing;
Blue rubber bleb of excessive
All
-
respond d r a
forms of miliaria temperature
" d by removal
.IA)
regulation of environmental
71
724
diazepam has produced
necrosis of sweat glands r
barbiturates or ^u|tl
erythema and bullae .
%
anhidrosis with or WJithout
70 (A) Causes of
Hyperhidrosis
Cortical
s Emotional
S Familial dysautonomia
Congenital ichthyosiform erythroderma
S Epidermolysis bullosa
s Nail-patella syndrome
S Jadassohn- Lewandowsky syndrome
Pachyonychia congenita
Pa Imoplantarkeratoderma
/ Stroke
Hypothalamic
Drugs:
/ Alcohol
S Antipyretics
S Cocaine
S Emetics
S Insulin
Opiates (including withdrawal)
S Ciprofloxacin
S Exercise
Infection:
S Defervescence
J Chronic illness
Metabolic:
^ Carcinoid syndrome
Debility
s Diabetes mellitus
Hyperpituitarism
1/
Hyperthyroidism
Hypoglycemia
Obesity
S Pheochromocytoma
Porphyria
Pregnancy
Rickets
723
of antipyretics is also beneficial
to patients wth
wi
admmrstration
may exacerbate the eruption.
fever T%
are usually ineffective and
j
agents
724 )
°
73.(A) Drugs causes hypertrichosis: Diazoxide, phenytoin, cortic0 er
corttsporin. cyclosporine , androgens, anabolic agents, hexachloro °
minoxidil, psoralens, penicillamine, and streptomycin.
Ms
teiy
^
74JC) Trichophagy, resulting in trichobezoars, may complicate trichotiliontow
£ t 1
75.(B) Telogen effluvium accounts for the loss of hair by infants in the ist f
months of life; fnction from bed sheets, particularly in infants with Pruritic **
atopic skin , may exacerbate the problem. There is no inflammatory
reaction;
the hair follicles remain intact, and telogen bulbs can be demonstratec
microscopically on shed hairs. Because >50% of the scalp hair is rarely involved
alopecia is usually not severe. Parents should be reassured that normal hair
growth will return within approximately 3 - 6 mo.
7S.(E) Telogen effluvium manifests as sudden loss of large amounts of hair
often with brushing, combing, and washing of hair. Diffuse loss of scalp hair
occurs from premature conversion of growing, or anagen, hairs, which normally
constitute 80-90% of hairs, to resting, or telogen, hairs. Hair loss is noted 6 wk
to 3 mo after the precipitating cause, which may include childbirth; a febrile
episode; surgery; acute blood loss, including blood donation; sudden severe
weight loss; discontinuation of high-dose corticosteroids or oral contraceptives,
hypo- or hyperthyroidism; and psychiatric stress.
77 D) White Nail or Nail Bed Changes
^
DISEASE CLINICAL APPEARANCE
Anemia Diffuse white
Arsenic Mees lines.transverse white lines
Cirrhosis Terry nails:most of nail, zone of pink at distal end
Darier disease Longitudinal white streaks
Half-and-half nail Proximal white, distal pink azotemia
High fevers ( some diseases ) Transverse white lines
bands
Hypoalbuminemia Muehrcke lines: stationary paired transverse
Hypocalcemia Variable white
Malnutrition Diffuse white
Pellagra Diffuse milky white
Punctate leukonychia Common white spots
Tinea and yeast Variable patterns
Thallium toxicity(rat poison ) Variable white
striations
Trauma Repeated manicure: transverse
725
SjSSifc
7
;
seen in
pachyonychia congenita
Diffuse white

/ gubinstein-Taybi syndrome
/ hemihypertrophy
79 (C) Pseudoclubbing is seen in
/ Apert's syndrome
/ Pfeiffer 's syndrome

r~pSrLir,°:
/ Rubinstein-Taybi syndrome

-sarr
^- ~
»
** * »««
nrsis
81.(E) Yellow nail
syndrome
h

*
may

growing yellow nails manifests as thickened, excessively


without lunulae. All nails are curved, slow
Associated systemic diseases include bronchiectasis,
affected in
cases.most
chylothorax, and focal recurrent
edema of the limbs and face. Deficient bronchitis ,
drainage, caused by hypoplastic lymphatic
lymphatic vessels, is believed to lead to the
manifestations of this syndrome.
Splinter hemorrhages .
most often result from mmorOTuma b
associated with subacute bacterial ce||
histiocytosis, severe rheumatoid a
chronic glomerulonephritis, cirrhosis,
'
^
^
tPrjchjnosis hypertens on.
malignant neoplasms,
-
and psoriasis.
82.(C) Onycholysis Indicates separa ion nf the nail plate from the distal nail bed

^^^
Common causes are trauma, t

cosmetics, psoriasis, fungal


^(distal onycholysis),
^
^ ,
agents,
vincristine
dermatitis, porphyria, d g
indomethacin, chlorpromazme rThorazine
from tetracyclines or ch or,
]
jc0|.
), and
^ photot0XlCltY

, ridging, pitting, fragility.


83.(A) Trachyonychia is
thinning, distal notching,
can have no associate s
defects.
^
^
^
£ ^
gCterized
a
,lescen«
system* dl
by
* a'"
on
t ori of all the nails. Pa
and no other ectodermal

viruses (herpes
84.(B)
. , ,agfung n
diagnosis. Of °
85.(0 The different a d
nbolloos
n
impet K01
, ker on)'
' ' hropod bites and
,

simplex, varicella- zos'


(tioea
poris
corP '
726
( scabies, pediculosis capitis), all of which may k
ir |nfestations
paras itfc ^OfT,t
impetigintfed. should receive an intravenous antibiotic with
Neonates with cellulits
85 ( A) , antistaphylococcal antibiotic such as nafcillin, cefat0| 1
lactamase - stable
aminoglycoside such as gentamicin or a 3rd-gen 01
lnS
vancomycin, and an In
cephalosporin such as cefotaxime .
older than 2 mo with mild to moderate
°
87.(C) In infants and children infection

. J
, lymphadenopathy , and other constitutional
particularly if fever signs
may be initiated orally on an outpatient
absent, treatment of cellulitis basis with
a penicillinase - resistant penicillin such as dicloxac l lin or a lst -generation
cephalosporin such as cephalexin or, if MRSA is suspected, with clindamycin.
88,(0)
89.(C) Ecthyma gangrenosum is a necrotic ulcer covered with a gray-black
eschar. It is usually a sign of P. aeruginosa infection, most often occurring m
immunosuppressed patients. Neutropenia is a risk factor for ecthyma
gangrenosum.
90.(A ) Blistering distal dactylitis is a superficial blistering infection of the volar
fat pad on the distal portion of the finger or thumb that typically affects infants
and young children. More than 1 finger may be involved, as may the volar
surfaces of the proximal phalanges, palms, and toes. Blisters are filled with a
watery purulent fluid; polymorphonuclear leukocytes and Gram -positive cocci
are identified on Gram stain .
91.(D) Perianal infectious dermatitis presents most commonly in boys (70% of
cases ) between the ages of 6 mo and 10 yr as perianal dermatitis (90% of cases)
and pruritus ( 80% of cases). In boys, the penis may be involved. Fecal retention
is a frequent behavioral response to the infection. Although local induration or
edema may occur, constitutional symptoms, such as fever, headache, and
malaise, are absent, suggesting that subcutaneous involvement, as in cellulitis,
is absent. Familial spread of
perianal infectious dermatitis is common,
particularly when family members bathe together or use the same water.
92.( E) Sites of predilection are the hair - bearing areas on the face, neck, axillae,
buttocks, and groin. Pain may be intense if the lesion is situated in an area
where the skin is relatively fixed, such as in the external auditory canal or over
the nasal cartilages.
93 . ( B ) Erythrasma is a benign chronic superficial infection caused by
Corynebacterium minutissimum. Predisposing factors include heat, humidity,
obesity, skin maceration, diabetes mellitus, and poor hygiene. Approximately

727
gspps5§r
enlarge over several
and circumscribed
n0 evidence
of lung
weeks, ulcerate, and drain. The local
, constitutional symptoms are absent, and
or other organ
^
reaction is nontende
there generally is
involvement. Other atypical mycobacteria
may cause a similar presentation, including Mycobacterium avium complex ,
Mycobacterium kansasii, and Mycobacterium fortuitum, Treatment is
accomplished by excision and administration of antituberculous drugs.
9$.(C) The lesions of tinea versicolor vary widely in color . In white individuals,
they are typically reddish brown, whereas in black individuals they may be
!

either hypopigmented or hyperpigmented. The characteristic macules are


covered with a fine scale . They often begin in a perifollicular location, enlarge,
and merge to form confluent patches, most commonly on the neck, upper
chest, back, and upper arms.
96.(D) Tinea capitis can be confused with seborrheic dermatitis, psoriasis,
alopecia areata, trichotillomania, and certain dystrophic hair disorders . When
inflammation is pronounced, as in kerion, primary or secondary bacterial
infection must also be considered.
97.(D) Tinea corporis, defined as infection of the glabrous skin, excluding the
Palms, soles, and groin, can be caused by most of the dermatophyte species
,
prevalent
although T. rubrum and Trichophyton mentagrophytes are the most .
also common
etiologic organisms. In children, infections with M. canis are
persons or by contact
corporis can be acquired by direct contact with infected
surfaces. M. canis
with infected scales or hairs deposited on environmental
' infections are usually acquired from infected pets
,
of
stud the contiguo skin, are a hallmark
98.(B) Satellite pustules, those that
localized Candida infections. may closely resemble
those of
99.(A) The cutaneous manifestations face
, e ema of the extremities and
acrodermatitis enteropathica ; however omen belly" ) are key features
( "moon facies" ) and a protuberant ab
uniformly observed in kwashiorkor . pearly , skin-colored, smooth
,
are
100.( D ) Molluscum contagiosum typjCa||y have a central
dome - shaped, papules vary in s < ze
umbilication from which a plug °
fromhe material can be expressed. The
t e face, eyelids,
neck, axillae ,
papules may occur anywhere on
the ^ but ^ found in clusters on the
and thighs are sites of predilection ^
728
3nd may be associated
of adolescents with
*^
in the groin individuals- Lesions commonly invoi
genitals or sexually active
venereal diseases
in cquired by sexual transmission m «%
, but are not *.
genita area
'
n
in children
,,h05 scabies is frequently heralded by inte
" Hia

? ni (B1 in an
iromunocompeten
^^ Qf the infestation often consists
«v
pruritus, particularly* "* . ,
afe excoriated crusted, or scaling. Thre d|
^ T
burrows are the
5
c as
^
2 mm red paP ' *° lesion 0f scabies but may not be seen in ,„ lk(
infant . |
and pw are relatively common. The eruption may ahc
infants , bullae
& an(J a SUperimposed eczematous dermatitis
include wheals , papu e •
, ’
Q dassic scabies (scabies vulgaris)
,

include
'
Intense aeneralized pmritus

l0C
2
,gtlo Ler . .
worse at night; inflammatory pruritic papules
Webs. flexor aspects of wrists, elbows, ax llae, buttocks.
* female breasts; lesions and pruritus spare the face, head, and neck,
^erondarv lesions include eczematization, excoriation, and impetigo.
103 ( A) The treatment of choice for scabies is permethrin 5% cream (Elimite)
anolied to the entire body from the neck down, with particular attention to
intensely involved areas. Scabies is frequently found above the neck in infants
(younger than 2 yr old), necessitating treatment of the scalp. The medication is
left on the skin for 8- 12 hr and should be reapplied in 1 wk for another 8-12 hr
period.
104,( A) Malathion 0.5% in isopropanol is the treatment of choice and should be
applied to dry hair until hair and scalp are wet, and left on for
12 hr. A second
application, 7-9 days after the initial
treatment may be necessary . This product
is flammable, so care should be taken
to avoid open flames.
105.1A) Benzoyl peroxide or combinations with erythromycin or clindamycin are
effective acne treatments and are
or in conjunction with
recommended as monotherapy for mild acne,
a topical retinoid, or systemic antibiotic therapy f r
moderate to severe acne. °
106.(B) Doxycycline
and minocycline are more effective than tetracycline
Although oral erythromycin
or azithromycin can be effective in treating acne, its
use should be
limited to
women or children <8 yr those who cannot use the tetracyclines (i.e., pre8nan
of age). Erythromycin use should be restricted because
,
r sk 0< bacterial
ru s
«*
» «. . uaA
.indivi
,
, ^ can Induce acneiform
SOniazid’ Pbenytoin, phenobarbital lesions in
susceptible
androgens (anabolic , trimethadione lithium carbonate
. .
steroids), and vitamin B12.

729
J Xanthogranulomatous infiltrates occur occasionally
,
This
s
in ocu|ar ti
heterochromia Irid .
location may heighten concerns for

^^ "
ions
and periocular intra ent .
nvoivement
appears
- to be an association ramong juvenile
There>
xanthogranuloma ,
neurofibromatosis , and childhood leukemia,
myelogenous
. .
leukemia . TU
There
most
is no need to remove ^
equently juvenile chronic
the benign lesions of
juvenile xanthogranuloma because most of them regress spontaneously in the
first few years . Residual dyspigmentation and atrophy may result .
109,(0) CLOVES syndrome ( congenital lipomatous overgrowth, vascular
malformations, epidermal nevi, and scoliosis/skeletal and spinal anomalies ) is
usually a sporadic disorder caused by a mutation in the PIK 3CA gene with an
asymmetric truncal lipomatous mass present at birth. Additional features
include macrodactyly , vascular malformations ( low flow ), linear epidermal
nevus, and renal anomalies.
110.(E ) Acrodermatitis enteropathica is a rare autosomal recessive disorder
caused by an inability to absorb sufficient zinc from the diet . The genetic defect
is in the intestinal zinc- specific transporter gene SLC 39A 4. Initial signs and
symptoms usually occur in the 1st few months of life, often
after weaning from
breast milk to cow ' s milk . The cutaneous eruption consists of
vesicuiobullous.
eczematous, dry, scaly, or psoriasiform skin lesions
symmetrica y is n
cheeks knees,* ,,
the perioral, acral, and perineal areas and on the '
characteristic. Ocular man f .
The hair often has a peculiar. redds
estatio
dysuop V

ab|
photophohia, conjunctivitis
ePbyl ,
slit. amp examination
.
,

blepharitis, and corneal diarrhea . stomatitis, glossitis,


Associated manifestations lnC Ude
paronychia, nail dystrophy , '
growth retardation
and
, irritability, delayed
superinfection with Candida
wound

healing, intercurrent bacterial infections


albicans.

730
Chapter 31
Bone and Joint Disorders
Questions

_ head height to total height at birth ?


io of
ratio
1. What is approximate
A. 1:3
B. 1:4
C. 1:5
D. 1:6
E. 1:7

2. By what age, birth height usually doubles?


A. B yr
B. 4 yr
C 5 yr
D. 6 yr
E. 7 yr

-
3. Which of the following is a cause of painless limp in a 2 -year old toddler ?
A. Septic arthritis
B. Transient synovitis
C. Intervertebral diskitis
.
D Malignancy
E. Cerebral palsy

4. Which of the following imaging modalities is BEST for diagnosis of early septic
arthritis?
A. Bone scan
B. Computed Tomography
C. Plain X - ray
D. Ultrasonography
E. Magnetic Resonance Imaging
of the following congenital foot
5- with developmental dysplasia of deformities has the highest
Ration the hip?
aSSA Metatarsus adductus
calcaneovalgus foot
B
C Congenital talipes equinovarus
D congenital vertical talus
E . Flatfoot

6. A 2- year- old toddler has a normal longitudinal f oot arch when


examined in a
non-weight-bearing position or standing on the toes, but the arch
disappears
when standing flat .
Of the following, the MOST likely diagnosis is
A. Hypermobile pes planus
B. Tarsal coalition
C. Cavus foot
D. Persian slipper foot
E. Rocker-bottom foot

-
1 What is the MOST likely cause of unilateral cavus foot ?
A . Tethered cord
B. Myelodysplasia
C. Occult intraspinal anomaly
D. Charcot-Marie-Tooth
E . Refsum disease

8. What is the MOST common cause of heel pain in


children ?
A. Iselin disease
B. Kohler disease
C. Freiberg infraction
D. Sever disease
E. Pescavus
- formed extra
digit of foot ?
removal of well
9 . What is the proper time for
A. At birth
of age
B . Between 3 and 6 mo
of age
C. Between Band 9 mo
D . Between 9 and 12
mo of age
E . Between 12 and 18
mo of 3Be
732
sssssi -
the MOST
Of the following,
A. mallet toe
B. hammertoe
C. claw toe
*
likely
; —
diagnosis is
Sr;
fc.

D. macrodactyly
E. subungual exostosis

11. Which of the following is the MOST common cause of foot pain in an 8 year

-
old child ?
A. Poorly fitting shoes
B. Foreign body
C. Osteomyelitis
0. Plantar fasciitis
E. Dactylitis

12. A young parents brought their 2-year-old 1st boy


concerned with his
"bowed" appearance of legs and in- toeing gait
. On examination he had also
congenital metatarsus varus.
Of the following, the MOST likely cause of
his gait is
A. femoral retroversion
B. external femoral torsion
C. slipped capital femoral
epiphysis
D. medial (internal) tibial
torsion
E. active rickets

13. An 18-month- old


boy brought by his concerned young mother because o -
his bow legs, on
examination in recumbent position, legs do not appear bowe-
but in erect position
during weight bearing and with ankles in apposition, the
legs are bowed.
Of the following,
the MOST likely cause of boys
A. Blount bow legs is
disease
B. nutritional
rickets
C. hypophosphatasia
D. physiologic
bowleg
E- metaphyseal
dysplasia
, 0« the following is a secondary
form of osten
1
Which 160porosis
Gaucher disease in children?
0
osteogenesis imperfecta
c Ehlers-Danlos syndrome
0 Marfan syndrome
l Homocystinuria

15- The appearance of symmetric bilateral genu va|B„m


physiologic process of leg development is most oron
ounced
part of the

normal
A. 2 years
H
' around the age of
B. 4 years
C. 6 years
D. 8 years
E. 10 years

16. Which of the following


conditions causes lower extremity shortening?
A. Rheumatoid arthritis
B. Hemarthrosis (hemophilia)
C . Osteomyelitis
0. Neurofibromatosis
E. Tuberculosis

IT . A 12-year-old boy presents with a fairly large asymptomatic mass behind the
pop itea
knee which is firm but compressible located medially and distal to the
crease. The mass is
Of the following, the only necessary diagnostic
Knee
A. CT
B. radiographs
C. ultrasonography
D. MW
E. aspiration above scenario at the
time
boy in the
ma nagement for the
18 What
,
is the BEST
being?
prance
A . Reass ovation
and e g e
B Rest
C. ASP
iration' 73
*
o. Corticosteroid injection
E . Surgical excision

19 A 13- year-old boy


presents with anterior knee pain, over the tibia ) tutte
aggravated by sports activities but
may often persist with regular daily
reveals point tenderness over the tibial
acJ*
Physical examination tubercle and th
. Radiographs reveal fragmentation *
distal portion of the patellar tendon of the
tibial tubercle and soft tissue swelling. There is no acute traumatic inoting
event. '
Of the following, the MOST likely diagnosis is
A . osteochondritis dissecans
B. Osgood- Schlatter disease
C. Sinding- Larsen-Johansson syndrome
D. patellofemoral pain syndrome
E. acute patellofemoral dislocation

20. An adolescent female complains from pain beneath the patella aggravated by
walking up and down stairs, and when sitting in a flexed knee position for an
extended period of time . Pain is often relieved through knee extension. The
onset of symptoms was gradual with no history of trauma . Physical exam
reveals tenderness with palpation about the medial aspect of the patella.
Of the following, the MOST likely cause of pain is
A. osteochondritis dissecans
B. Osgood-Schlatter disease
C. Sinding- Larsen-Johansson syndrome
D. patellofemoral pain syndrome
E. acute patellofemoral dislocation

21. Which of the following is a risk factor regarding developmental dysplasia of


the hip ( DDH ) ?
A. Male gender
B. Cephalic presentation
C. Polyhydramnios
D. Small birth weight
E. First pregnancy
?
22. What is the MOST reliable sign of a dislocated hip in a 4 -month - oId infant
A . Limitation of abduction

735
Apparent shortening of the thigh
B-
location of the greater
c proximal trochanter
of the gluteal or thigh folds
0 Asymmetry
E Hip click

23. What is
the presenting complaint of DDH in a , walking child?
A. Limp
B. Waddling gait
C. Excessive lordosis
D. Leg-length discrepancy
E. Toe-walks on the affected side

24. A 4- week -old newborn during routine examination found to be Ortolani


positive (dislocated but reducible left hip). Ultrasonographic image of the left
hip demonstrates an a angle of 42 degree.
Of the following, the MOST appropriate treatment at this age is
A. Pavlik harness
B. triple diapers
.
C abduction diapers
D. Frejka pillow
E. von Rosen splint

rafrbri wirhistorv
^^ ^^^ ^ ^
"
25. A 6-year-old girl presents with ' she
of
a
she is able to bear weight on the jon 10 days ag0.
Erythrocyte
trac counts are
nonspecific upper respiratory and white b|0od cell
sedimentation rate, serum.
^ a small joint effusion
C'
7
,

hip demonstrates
UltrasonograP V ° the
normal , and is
Of the following, the MOST
A. transient
synovitis
B•
SePtiCca?"
C. Legg-
v - Perthes
disease

njdar° tsyndrorr'
^ ^
e
D . osteoc - Ja | a
reported to describe
E schwartz ^
vTperthes disease?
26. Which
"he 3 Dee
A
^
" 30 ^^ £^^
eased
he

taterauzation
-
following
red
«
radioBraPhL
°mora head
of me e
,
nter
|

B-
C. Subchondral fracture
D . Lateral extrusion
of the epiphysis
E . Physeal irregularity

abnormality noted on physical exam in


27. What is the earliest pat

scoliosis? 'ents With


A Posterior rib hump on the convex side of the spinal curve
B Asymmetry of the posterior chest wall on forward bending
C. Apparent leg-length discrepancy due to pelvic obliquity
D. Shoulder imbalance
E. Lateral shift of the trunk

28. What are the MOST common congenital abnormalities identified iin
with congenital scoliosis ?
children
A. Cardiac anomalies
B. Intraspinal anomalies
C. Skin tags
D. Genitourinary abnormalities
E. Hemangiomas

29. An adolescent male brought by his father becau


se of kyphosis which can be
corrected voluntarily, a standing lateral radiograph
was ordered and showed an
increase in kyphosis but no pathologic changes of
the involved vertebrae. The
patient and his father are concerned
regarding the cosmetic appearance and
ask for treatment.
Of the following, the MAINSTAY of
treatment is
A. physical therapy
B. bracing
C. surgery
D. reassurance
E. core strengthening

30. A 14- year- old boy


presents with hyperkyphosis of the thoracic spin
associated with a sharp contour .
The boy is unable to correct the deform
voluntarily. He has mild interm
ittent pain near the apex of the kyphosis wit ^
other neurologic symptoms
. Standing lateral radiograph showed wedging o
vertebrae at T6, T7, T8, and T9. ^^
Of the following, the
MOST likely diagnosis is

737
A. spondylolisthesis
B. Scheuermann disease
C. postural kyphosis
D. congenital kyphosis
£. spondylolysis

is a common cause of back


A. Vertebral osteomyelitis
B. Transverse myelitis
C. Diskitis
D. Perinephric abscess
E. Pancreatitis

32. A 14- year-old wrestler presents with low back pain radiate to the
buttocks,
exaggerated with spinal hyperextension. On examination there is spasm of the
hamstring muscles with discomfort on palpation of the spinous process of the
lower lumbar vertebrae but no any other neurologic symptoms.
Of the following, the MOST likely cause of this back pain is
A . spondylolisthesis
B . Scheuermann syndrome
C. spondylolysis
D. herniated disk
E . overuse syndrome
pain, fever, and
33. A 4 - year - old boy presents with back pain, abdominal over
refuses to walk , with local point tenderness
malaise for the last 4 days. He the pam
he asked to pick up an object from the ground,
the middle back . When
increased in intensity * test is
Of the following, the MOST helpful diagnostic
A. plain radiograph
B. technetium bone scan
C. MRI
D. Blood culture
E . open biopsy of the disc
space
infections ?
organism causing spine
34 . What is the MOST common
A . Kingella kingae
738
0cc^s
streptoc
_
6.
Group A
coli
C. Escherichimarcescens
a aureus .

D. Serratia
Staphylococcus diagnosis of a disc herniation?
the
E. « establish
BeststodV °
t is the raph
35 - ha
^A. Plain
radiofc
bone scan
B. Technetium
C. MRI
D . CT scan . her father complaining from

- . *
rs* :.i
'
s

v„
„ and evidence
congenita fus on
Ur
hairline
^
*,s , cervical sp'


thp
Radiograph s«s is
°
’ ***
diagn
of spin 1* * M 0ST likely

"ttSSarsvn ,e

£ Ss ^
^ °
^
! '
B.
c. gl ppe '
p
0r
'

^ -O

the
. h of syndrome
37 . Whic
A. Apert
following

syndrome
ndrorne
syndromes associated with polydactyly
?

B. Holt-Oram
C. Trisomy
21
-Moon -Biedl syndrome
D . Laurence syndrome
E. Ellis- van Creveld limb in childre ° '

bone of the lower


is the MOST commonly fractured
38. What
A. Fibula
B. Tibia
C. Femur
D. Tibia and fibula
E. Calcaneus
739
the middle area of tita,
3
* bear We ght,
' graphs
Point(
^ trauma. Radi

°
0 osteomyelitis
A.
g toddler 's fracture of the tibia
C. Osgood-Schlatter syndrome
D. green stick fracture of tibia
E. Tillaux fracture

40. What is the MOST common infecting organism in


osteomyelitis among all
age groups ?
A. Group B streptococcus
B. Escherichia coli
C. Staphylococcus aureus
D. Group A streptococcus
E. Pseudomonas aeruginosa

41. Which of the following organisms is difficult to detect in osteomyelitis unless


polymerase chain reaction ( PCR ) testing is used?
A. Kingella kingae
B. Streptococcus pneumoniae
C. Bartonella henselae
D. Group A streptococcus
E. Pseudomonas aeruginosa
if
treatment of osteomyelitis
for
is recommended ?
42. Which antibiotic icillin were recovered
is
to p
pneumococcal isolates resistance
A. Clindamycin
B. Cefotaxime
C. Vancomycin
D. Cefazolin bacterial
E. piperacillin- tazobactam „ hematogenous
im
joint involved
the MOST common
43. What is
arthritis?
A . Knee
B. Hip 740
c. Ankle
D Elbow
,

£ . Wrist
positive blood cultures cases of proven
in
percent 0f
is the
44 . What ic %

arthritis ?
A. 10%
B. 20%
C. 30%
D. 40%
E. 50%

45. A 2 - year-old child presents


with fever, pain, swelling, erythema of
days Lab
fenee
joint, with refusal to walk for the last 3 . . investigations
revealed
normal leukocyte counts , slightly raised ESR, normal CRP . Plain films showec
widening of the joint capsule while ultrasonograp hy detected joint effusion.
Of the following, the MOST likely diagnosis is
A . proximal tibial osteomyelitis
B . pauciarticular rheumatoid arthritis
C . Lyme arthritis
D. septic arthritis
E. pyomyositis

46. Which of the following structural or acquired heart disease


can participate
in sports ?
A . Hypertrophic cardiomyopathy ( symptomatic )
B. Dilated cardiomyopathy ( symptomatic )
C. Mitral valve prolapse
D. Coronary artery anomalies
E. Acute rheumatic fever with
carditis

during sports ? ^ f°*l Wing cardiovascular diseases can result in sudden death
0

^
ertrophlc cardiomyop
R athy
Mal g ant ventricular
" '
c. Hypertension arrhythmias

?
E.
• Coneenital heart disease
Symptomatic Wolff -
Parkinson - White syndrome
741
of sprain?
4$- What is the definition
muscle or
to a tendon
A. Injury
injury to any soft tissue
B. Crush
C. injury to a ligament or joint capsule
D. Repetitive microtrauma that

^ 'T y's rate of repair


E. Evidence of microscopic trauma to endon

49. Why children are more vulnerable to heat illness than adults ?
A. They have greater ratio of surface area to body
mass
B. They produce lesser heat per kilogram of
body weight during activity.
C. The sweat rate is greater in children
D. The temperature at which sweating occurs is lower
E. Children have a sensitive response

50. What is the character of the skin in heat stroke ?


A. Sweating skin
B. Profuse sweating skin
C. Hot dry skin
D. Hot moist skin
E. Flushed skin

51. Treatment of heat exhaustion includes moving to a cool environment.


placing ice over the
cooling the body with fans, removing excess clothing, and
A . head
B. groin and axillae
C. dorsum of hands and feet
D. chest and abdomen
E. whole body
performance -enhancing substances is
following ccommon
52. Which of the Olympic committee?
allowed by international
androgenic steroids
A . Anabolic-
B. Creatine
Human growth hormone
C. /stimulants
O. Amphetamines blood doping
£
Erythropoietin/

742
, r adverse effect
of which of the
foil
is a rT al
° °W |

•£
?
* *****
.
54.
white hte P»«

C. hVP ch dr0

” i'5“
° °"
sss'5s- ri
,
B. achondroP asia
^
with norh ah

Phvseal oysp'as’a
( (nteUlgence.
- - — *•

^
55. A 9-year-old boy brought by his father due to poor school performance. On
examination, he looks a little bit short stature affecting the limbs with a stocky
build and slight frontal bossing of the head.
Of the following, the MOST likely diagnosis is
A. thanatophoric dysplasia
B. achondroplasia
C. hypochondroplasia
0. Jansen Metaphyseal Dysplasia
.
E Schmid metaphyseal dysplasia

56. A 3- week -old boy with severe shortening of limb, clubfoot, associated with
an unusual facial appearance. His serum calcium is 14mg dL. Radiographs show
/
short tubular bones with characteristic metaphyseal abnormalities that include
flaring, irregular mineralization, fragmentation, and widening of the physea
space.
Of the following, the MOST likely diagnosis
is
A. thanatophoric dysplasia
B. achondroplasia
C. hypochondroplasia

743
D. Jansen Metaphyseal Dysplasia
E. Schmid metaphyseal dysplasia

57. in which of the following conditions, the


anterior fontanelle is
might remain open ? • wide and
A. Rickets
B. Down syndrome
C. Hypochondroplasia
D. Cleidocranial dysplasia
E. Campomelic dysplasia

58. A 6-month- old boy presents with macrocephaly, hepatosplenomegaly


,
deafness, blindness, and severe anemia . Radiographs reveal diffuse bone
sclerosis and blood test showed hypocalcemia . Most of the bone manifestations
in this disease can be prevented or reversed by
A. hematopoietic stem cell transplantation
B. Calcitriol
C. interferon- y
D. blood transfusions
E. vitamin D and calcium supplements

with short limbs, postaxial polydactyly of the


radiographs reveal short
hands, and dental anomalies , skeletal
atrial septa
tubular bones with clubbed ends, echo revealed
Of the following, the MOST likely diagnosis
is

A . Asphyxiating thoracic dystrophy


B. Ellis-van Creveld syndrome
C. Metatropic dysplasia
D. thanatophoric dysplasia
E. achondroplasia
following Juvenile
region in which of the
60. Os calcaneus is the affected
Osteochondroses?
A . Legg- Calve- Perthes disease
B . Osgood- Schlatter disease
C. Sever disease
D. Freiberg disease
E . Scheuermann disease
744
.
,„h bv his mother with
. painful mandibular
SWM
. ° gsJ JS
-old boybr like in consistency
61. A 4-month |irlg is wood - with
duration
for one month
a a ion the baby is irritable, feverish and .Poor
warmth or redness
feeder. Laboratory
i " disp|ay elevated erythrocyte sedimen
jncreased serum alkaline ph0Sph 0
ati .
'
rate
« ^ ^« * «
ata$e
* ““ c rticai
°
A. vitamin A intoxication
B. neuroblastoma
C. Caffey Disease
D. osteoid osteoma
E. scurvy

62. A 6- year - old child presents with short stature, blue scleral, blue-gray teeth
that break easily, hearing loss, and kyphoscoliosis. Radiographs show Lower
long bones osteoporosis, with metaphyseal flaring, and "popcorn" formation at
growth plates .
Of the following, the MOST likely diagnosis is
A. Osteogenesis Imperfecta Type I
B. Osteogenesis Imperfecta Type II
C. Osteogenesis Imperfecta Type III
.
D Osteogenesis Imperfecta Type IV
E. Osteogenesis Imperfecta Type V

63. What iIS the MOST


obvious skeletal manifestation of Marfan syndrome
( MFS) ?
A. Pectus carinatum
B. Dolichostenomelia
-
C Pes planus
D. Arachnodactyly
E. Amptodactyly

Marfan syndrome ( MPS) ^6 a d mortality in young children with


? °
LTa
VentriCUlar arrhythmias
B Mitral, valve
dysfunction
c- Ventricular
dysrhythmias
745
p. prolonged QT interval.
E. Dilated cardiomyopathy
the MOST common
65. What is ocular
(MFS)?
manifestation iin Marfan syndrome
A. Severe myopia
B. Flat cornea
C. Ectopia lentis
D. Hypoplastic iris
E. ciliary muscle hypoplasia

66. A 3-month-old infant presents


with irritability, and failure to
laboratory results revealed hypercalcemia thrive,
, hypercalciuria, and low alkaline
phosphatase, ultrasonography shows nephrocalcinosis
, X-rays reveal irregular
ossification, punched-out areas, and metaphyseal cupping.
Of the following, the MOST likely diagnosis is
A. infantile hypophosphatasia
B. perinatal hypophosphatasia
C. hypophosphatasia tarda
D. pseudohypophosphatasia
E. metaphyseal Dysostosis

Of the following, the MOST likely diagnosis


A. infantile hypophosphatasia
.
B perinatal hypophosphatasia
.
C hypophosphatasia tarda
D. pseudohypophosphatasia
E . metaphyseal Dysostosis
5,000
. p hjch Serum alkaline phosphatase
, * “
^
68. An 18-month-old child found to
ncldenno ,
jndj during screening laboratory
IU/L, which is detected as an laboratory signs of
are Qther c injca| or
evaluations for bow legs. There of: calcium, phosphorus, Cr,
assessments
Laboratory yv amin D are normal.
hepatic or bone disease.
AST, ALT, GGT, bilirubin, PTH, and 25 - hydrc
^
Of the following, the MOST likely diag
746
A. familial hyperphosphatasia
B. juvenile paget disease
C. familial expansile osteolysis
D. autosomal dominant hypophosphatemic rickets
E. transient hyperphosphatasia

747
chapter 31
Bone and Joint Disorders
f Answers
ZUHAIR ALMUSAWI
1.(B) The head is disproportionately large at birth, and the ratio of head height
to total height is approximately 1:4 at birth, which changes to 1:7.5 at skeletal
maturity .
2.(C) By age 5 yr, birth height usually doubles and the child is approximately
60% of adult height. The child is approximately 80% of final height at 9 yr.
During puberty, the standing height increases by approximately 1 cm/mo.
3.(E)
Painful limp
Septic arthritis and osteomyelitis
Transient synovitis
S Occult trauma ( "toddler ' s fracture" )
S Intervertebral diskitis
S Malignancy
S Abuse
Painless limp
•/ Developmental dysplasia of the hip
S Neuromuscular disorder
S Polio
S Cerebral palsy
inequality
V Lower extremity length n rather than pure anatomy
ays phys iolog ic infor matio
4.(A) A bone scan displ energ y from the nucleotide injected
into the
of
and relies on the emission early septic arthritis, osteomyelitis, avascular
patient. Indicationias
include occult and stress
osteo ma ), metastatic lesions,
oid
necrosis, tumors oste
i(

, and case s of child abuse. dysplasia of the hip (DDH) with


fractures for deve lopm ental
comparing risk calcaneovalgus has the highest
5.(B) When , cong enita l
foot deformities
other congenital % of patien ts having coexisting DDH
.
19.4 up to 23% of
association, with is a com mon diagn osis; it has been estimated that
6 ( A ) Flatfoot depen ding on the diagnostic criteria. Although
may be affected ,
the public
748
«• •* represents a common source of
concern.
* . *
lanus
hvPermobile or ch ldren are rarely symptomatic. Flatfeet are
olrents . these .
d with phys ologic hgamentous
common *
laJ
neonates and
improvement
10 yr of age. ^
may

7.(C) Although a
intraspinal
be

umla
seen

.
era
anomaly DiBtera
us
longitudinal arch develops between


foot is most likely to result from an
5

ment usua v suggests an underlying occuh


J
or muscle disease. djsease) js the most common cause of
8.(0) Calcaneal apophysn 1 he ,
pain in children; treat
activjty modification, *
nonsteroidal
" *

-
medications heel co
, stretching exercises, and heel
antunflammatory cushions

ss.? r a
or arch supports.
digit may be either rudimentary or well formed, and P|ain
*
rr*
2nd toe is most commonly involved, and a painful callus may develop over
dorsum of the toe where it rubs on the shoe.
11. (A) Differential Diagnosis of Foot Pain According to Age
6-12 yr
Poorly fitting shoes
Trauma ( fracture, sprain)
Juvenile idiopathic arthritis (enthesopathy)
Puncture wound
Sever disease (calcaneal apophysitis)
Accessory tarsal navicular bone
Hypermobile flatfoot
Tarsal coalition
Oncologic ( Ewing sarcoma, leukemia )
12.(D) Medial (internal) tibial torsion is
manifests with in- toeing galt It (
commonly associated with congenital q
metatarsus varus, genu valgunr
femoral anteversion. This condition is usually

749
seen during the second y
^^
life-
pare^
"omation that is secnnH,
lopment can be anticipated. Persistede of
^
be pathologic
.
Secondary forms of osteoporosis
14.|A| iinclude

disorders, chronic illness, endocrine disorders and drug
grrors of metabolism, including lysinuric protein - J!?*"?
neurom«cular
induced and inborn
disease. intolerance and Gaucher
15.(B) The appearance of symmetric bilateral rgenu
valgum most pronounced
aroiind age 4 is part of the normal physiologic
' Process of leg development,
However, variation of up to 15 degrees of valgus
is possible until 6 yr of age. The
majority of physiologic valgus has a good
chance of correction until this age.
1MC)
17.(C) Ultrasound can be used to confirm a simple cystic
lesion in the expected
anatomical location and is often the only diagnostic test necessary to confirm
the diagnosis with these reassuring findings.
18.(A) In most cases, reassurance is all that is needed for popliteal cysts because
they often resolve spontaneously .
19.(B) Localized soft tissue swelling, along with an eventual firm and fixed
increased prominence at the tibial tubercle, may occur with OS disease and may
inciting
also be part of the initial complaint . There is typically no acute traumatic
e V e n t* . Bent
20.(D) Pain beneath or near the P 3 e
' hTpatella
knee activities, such as walking up
compressive loads and tend to aggravat *
vigorous physical activities also exace
an
^ pu
^ under high
cauattinR running, and other
anterior
,
knee pain. Sitting in
tj(ne he s0.called theater
a
sign,
e
flexed knee position for an exten
complaint . of affected patients.
is another common 33%
thought to be
history which ar
21.(E) A positive family (80%), * hormoncs,
DDH is more common _J% of all
because of the greater
such as relaxin, which ^Lsentation
babies are born tn» breech
*
SU5 C
nP
’ *‘
,
presen
condition
of

,
female fetuses
hgamentous
incidence
,
"
B
^m l0n may he
*"

associated
2
in these patients
spaCe and,
with DDH.
is 16 - 25% .
consequently
Any
, less room for normat
fetal ° ‘
750
, large birth
weighs and
include
oligohydramnios s
These conditions rd mo of life, the soft tissue S
3
no longer reliable. L **
pregnancy baby enters the 2
. nd and
22.(A) As the Barlow tests are a« '"
O 0(
to

tighten, and the


and
Ortolani sign of a dislocated hip in this age grou "'
most reliable develops secondary to altered hip m c a ics
abduction is the
lordosis , which complaint . ^^ (t

23.(C) Excessive presenting positive ( reduced but disloca


is often the
common and
24.(A) Newborns
hips that are Barlow
but reducible
) should
dislocated diagnosis is made .
(
Cafa J 0
^*
generally be tre8 ed itlj r
a
Ortolani positive ,
°* fT,ateV
soon as the age groups A r
Pavlik harness as
synovitis can occur in all
Although transient consist of pain jn
25 .( A)
symptoms often develop acutely
and usually
referred from the hip.
°
, which may be
anterior thigh, or knee or laterally rotated unless
flexed , abducted ,
The hip is not held afebrile or have a lowgrade
present . They are often
effusion is
„hic signs have been reported
( 100.4'F). „ that describe a
ra l0g
26.(0) Several classic v Laterat extrusion of the epiphysis, a
"head at risk" for
severe '
o {he epiphysis, subluxation of the hip,
horizontal physis, calcifica V in the latera
of the physis (Gage's sign)
and a radiolucent horizontal
are all associated with a physjCal exam in patients with scoliosis
,
i
^ .
27.(B) The earliest abnormal
ft 1K»
ly P

b>
bending. This test, called
»«
apex
inclination measuring 7 degrees or
more has been
« suggested
«
rr r x
as the cutoff for

orthopedic referral .
are identified in 20 - 40 % of children with
28.(D) Genitourinary abnormalities duplication.
congenital scoliosis and include unilateral renal is
agenesis , ureteral
horseshoe kidney, and genital anomalies.
cosmetic concern and is most often
29. D
( ) Postural kyphosis is a common
recognized by parents or peers. The mainstay of treatment is reassurance
.

30.(B) Scheuermann disease is the most common form


of stru ^
hyperkyphosis and is defined by wedging of >5 degrees of three
or ^
consecutive vertebral bodies at the apex of the deformity on
a
than ^
radiograph. In addition, the apex of the thoracic kyphosis is lower
expected.

751
(C) Differential Diagnosis of Back Pain
inflammatory Diseases
Diskitis *
/ Vertebral osteomyelitis (pyogenic, tuberculosis)
/ spinal epidural abscess
/ Transverse myelitis
pyelonephritis *
perinephric abscess
Pancreatitis
/ Paraspinal muscle abscess, myositis
Psoas abscess
/ Endocarditis
S Pelvic osteomyelitis or myositis
S Pelvic inflammatory disease
32.(C) Spondylolysis is common in athletes who engage in repetitive spinal
hyperextension, especially gymnasts, football interior linemen, weight lifters,
and wrestlers. Approximately 4-8% of the entire pediatric population is
ts when a
affected, making it the most common cause of back pain in adolescen
diagnosis can be established.
.( ) A high inde x of susp icion is required to establish the diagnosis of
33 C a
spon dylit is. The diag nosi s may be established earlier using either
infec tious
or MRI ; MRI is the most sensitive and specific imaging to
technetium bone scan and/or neural compression.
and to ident ify absc esse s
diagnose osteomyelitis mon organism causing spine
is the most com
34.(E) Staphylococcus aureuss kinga e, and less often
group A
de King ella
infections. Other organisms inclu caus ® of vertebral
bone infection
streptococcus, and Escherichia
coli . Rare * , and cat-scratch disease.
, bruc ellos is -
1
open biopsy of
PCR is indicated
for the
Blood cultures have a the time ;
only 50-85% of
the disc space is positive . CT is
diagnosis of Kingella . diag nosi s of a disc herniation
to establish
the ass ocia ted with a
best study fragm ent
35.(C) MRI is the _ ified
oss
visu alize a partially <
especially helpful to
a low posterior
slipped apophysis. the class ic triad of
ic
( KFS ) includes
36.(C) Klippel -Feil synd rome
ical range of motion.
ease d cerv
hairline , short neck, and decr
Polydactyly
Syndromes Associated With
v' Carpenter syndrome
752
Ellis -van Creveld syndrome
Meckel-Gruber syndrome
J Polysyndactyly
S Trisomy 13
s Orofaciodigital syndrome
/ Rubinstein -Taybi syndrome
Syndromes Associated With Syndactyly
S Apert syndrome
S Carpenter syndrome
de Lange syndrome
s Holt- Oram syndrome
^ Orofaciodigital syndrome
Polysyndactyly
*^ Trisomy 21
^ Fetal hydantoin syndrome
^ Laurence-Moon-Biedl syndrome
^ Fanconi pancytopenia
^ Trisomy 13
^
B8.( B)
Trisomy 18

Physical exan ination. The classic


symptom is refusal to bear weieht
3 ected extremity or florid disD
lav nf
tenderness at the fracture site The A
^?
Wh > C

° her
'
Can mani est as pulling up the
^ » P<
*t
fibula*migh
ateral v e‘ s of the tibia-*
s ow a nondisplaced spiral frartur , '^
d Stal tibiaj metaPMs. An oblique
view is often helpful because
*
the frart
three views. Often the fracturp r
*l , ,
,ne may be visib e in on V one ofthe
Visualized until 2-3 wk later , when

periosteal reaction and


resoroti ^
racture s,te allow better visualization
inflammatory markers may bp * *^
ered to rule out infectious processes if the
diagnosis is in doubt .
40-(C) Staphylococcus aureus js th
e m St common infecting organism
steomyelitis among all age Pm, °
ndudin6 newborns.
41-(A) Kingella kingae is the ' most Common cause of osteomyelitis
children younger than 4 Vr JeCOnd
The 0 r anism s established as a cause
o
osteomyelitis, spondylodiskitis
^ ^ *
an septic arthritis in this age group, espec
753
iafo
is a subacute presentation
when there L,
tc k
polymerase chain reaction (PCRU , n8ae can be difficult
unless detect t0
42.(8) Cefotaxime
or ceftriaxone is '°
est n is u
^ d. 8

^
to penicillin and for m0
mmended for pneumococcal
with resistance isolates
43.(A) Joints of the lower extremity coni ' "
0
,Ute 75%3 spp.
ofa» cases of^
(Knee ~35, Hip ~25, Ankle ~im TK
S
involved in approximately 25% 0f cases d 311 Joints are
^ #
and st )oul
uncommonly ^ ImtsT
infected, except in gonococcal arthritis
44.(B) Blood cultures should be nerf ,1
3 Casesof susPected septic
arthritis but are positive in 20% or
,, cases proven or probable septic

- .
arthritis. Cervical anal anH u

« sus|
*S d *h
*" “' * * *« »» » be obtained when

V g Tr*^ “ » »» «
d symptoms may be subtle, particularly in neonates . Older
infants and children might have fever and pain, with localizing signs such as
swelling, erythema, and warmth of the affected joint. With involvement of
joints of the pelvis and lower extremities, limp or refusal to walk often occurs.
widening of
Plain films can suggest the diagnosis of septic arthritis by showing:
of normal fat lines.
the joint capsule, soft tissue edema, and obliteration
helpful in detecting joint effusion and fluid
Ultrasonography is particularly
regions.
collection in the soft tissue and subperiosteal
46.(C) from undetected cardiac disease
sports can result
47.( A ) Sudden death during , anomalous
coronary vessels,
cardiomyopathies
such as hypertrophic or other . In many cases, the underlying
heart
syndrome
or a ruptured aorta in Marfan is the first sign of heart disease
disease is not suspected, and
death
48.(C) or joint capsule.
- A sprain is an injury to a ligament
to .
a muscle or tendon.
- A strain is an injury
the body's
- A contusion is a crush i iU
- Overuse injuries are
"
caused by reTeie^crotrauma
tendons
that exceeds

.
rate of repair . ,
because they
- Tendinosis is evidence
more
of m r
* ‘£
0S C
°'
t
vulnerable "aheat
illness
and
than
produce
adults
greater heat per
49.( A) Children are in children, and
have greater ratio
area to
°
of surface activity’ me sweat rate
is lower
can take longer to
during higher. Children have a blunted
kilogram of body weight sweating occurs is
at which nments Children also
the temperature enviro
humid
acclimatize to warmer, more 754
thirst response compared with adults and
might not consume enoueb
during exercise to prevent dehydration.
^
50.( C) Heat Stroke: MEDICAL EMERGENCY
Core temperature of 40*C (104 °F ) or higher
/ Hot dry skin
Multiple system failure
/ Delirium
Convulsions
*v ' Abnormal vital signs
51.( B)
.
52 { B) Creatine increase in strength, power output, sprint performance, total
work to fatigue, peak force/ power; decrease lactate threshold; increase weight
and lean body mass.
53.( A ) Anabolic-androgenic steroids
MAJOR ADVERSE EFFECTS
Testicular atrophy, CV disease, atherosclerosis, myocardial disease, liver
dysfunction, cancer
MINOR ADVERSE EFFECTS
Acne, gynecomastia
54.( B) Infants usually exhibit delayed motor milestones, often not walking alone
untill8- 24 mo. This is because of hypotonia and mechanical difficulty balancing
the large head on a normal-sized trunk and short extremities. Intelligence is
normal unless central nervous system complications develop. As the child
begins to walk, the gibbus usually gives way to an exaggerated lumbar lordosis.
55.(C) Hypochondroplasia resembles achondroplasia but is milder. Usually, itis
not apparent until childhood, when mild short stature affecting the limbs
becomes evident. Children have a stocky build and slight frontal bossing of the
head. Learning disabilities may be more common in this condition. Radiographic
changes are mild and consistent with the mild achondroplastic phenotype
Complications are rare; in some patients the condition is never diagnosed. Adu
heights range from 116 to 146 cm. Recombinant growth hormone therapy m ^ V
enhance growth and improve body disproportion.
56.(0) Jansen metaphyseal chondrodysplasia is a rare, dominantly inherit ^
d with
chondrodysplasia characterized by severe shortening of limbs associate
o and
an unusual facial appearance. Sometimes it is accompanied by club
hypercalcemia with serum calcium values of 13 -15 mg/dL

7S5
57.(0) Cleidocranial dySp|asja might be rec
drooping shoulders, open fontanelles and n °gnited jn infants , •

because of
The shoulders of patients with Cleidocrani | °mment forehead.
dVSP asia can meet
Radiographs reveal hypoplastic or
cranial bones with multiple ossification ^(
centers ' '
absent I^ 65, delayed in the midline
ossification of the
,

ossification of pelvic bones. The anterior wormian bones), and


fontanelle is wide and might delayed
open.
remain
58 ( A) Most of the bone
manifestations- im ^vere osteopetrosis
osteoclast defects can be prevented due to intrinsic
or
10 61 _ stem cell
transplantation (HSCT), if
carried out before
secondary complications, such as
development '
of
^
irreversible
visual impairment.
S9.(B) The Ellis—van Creveld syndrome
, also known as
dysplasia, is a skeletal and an chondroectodermal
ectodermal dysplasia. The skeletal dysplasia
presents at birth with short
limbs, especially the middle and distal segments,
accompanied by postaxial polydactyly of the
hands and sometimes of the feet.
Nail dysplasia and dental anomalies (including neonatal, absent, premature
loss
of teeth, and upper lip defects) constitute the ectodermal dysplasia . Common
manifestations also include atrial septal defects and other congenital heart
defects.
60. (C)
Legg-Calve- Perthes disease = Capital femoral epiphysis
Osgood-Schlatter disease = Tibial tubercle
Sever disease = Os calcaneus
Freiberg disease = Head of second metatarsal
Scheuermann disease = Vertebral bodies 1 .
Blount disease = Medial aspect of proxima ons
Osteochondritis dissecans =
Subchondraeg .
‘ elbow, and ankie
rera „ „,
)
61.(C) Prenatal and more often Pos " re (autosomal recess** -
lesions.
onset may be mild ( autosomal
« ^ ^
distre ’ pr * ,
Severe prenatal disease

^

jevwe respiratory
6 mo; a**r
“ k is m0st
polyhydramnios, hvdropS
f cy ( younger than
of
*
bility swelIing of
high mortality . Onset m
in
, u sudden onset underlying
manifestations ncprece the cortical ‘ "» '" od-like induration
common;
contiguous soft tissue
that
. . . pai lful wrth
on
(

^^
absent . There
are
anore*
^ ^
supp to 3 mo . The
bones, fever, and can last 2 wk
but with minimal w a r m a p s e s; a n * P«° de
unpredictable remissions
756
bones involved
include the mandible ( 75%), the clavicle, an
most common %
ulna .
62 .
( C ) 01 type
.
significant physic
.. is the most severe nonlethal form of 01 and result .
disability - Birthweight and length are often l0w nc
_
Fractures
Postn atally. fractur es o
,
relative macrocephaly and
from inconsepuentia trauma and e
trian
31

facies .
^ a **nh
appearance aUhTmetaphyses . The rib cage has flaring at the base, and

^J
deformity is frequent. Virtually all type III patients
have scoliosis and
vert
compression. Growth falls below the curve by the 1 st yr, all type III patjenis
.
have extreme short stature Scleral hue range s from white to
blue
Dentinogenesis imperfecta, hearing loss, and kyphoscoliosis may be presenter
.

develop over time.


63.(B) Overgrowth of the long bones ( dolichostenomelia ) is often the
most
obvious manifestation of MFS and may produce a reduced upper segment
to
lower segment ratio ( UL/ LS) or an arm span to height ratio >1.05 times.
.
64 (B) Insufficiency of the mitral valve can lead to heart failure,
pulmonary
hypertension, and death in infancy; this manifestation is the leadin
g cause of
morbidity and mortality in young children with the disorder.
65.(C) Dislocation of the ocular lens ( ectopia lentis) occurs
in around 60-70% of
patients, although it is not unique to the disord
er. Other ocular manifestations
include early and severe myopia, flat cornea,
increased axial length of the globe,
hypoplastic iris, and ciliary muscle hypop
lasia, causing decreased miosis.
Patients are also predisposed to retinal
detachment, early cataracts, and
glaucoma .
66 ( A ) Infantile hypophosphatasia
is next on the continuum. These infants
present prior to 6 months
of age with hypercalcemia
nephrocalcinosis ), premature cranial / hypercalciuria (leading to
suture fusion (that can lead
JSSSzsris «““!,oh , ' *-
to increased
h - »

^ T1Ssienificant ,itv
theraPY P ent with asfotase alfa, mortality « »
.
estimated at 5n% *
S3KJXS •

disabi -
teeth ( with the root
intact due
skeletal deformities fracture
toT"°' '
P
W th premature exfoliation of pnmW
r V mmerall2ed denta
|

l cementum),
H ""
skeletal pain
"tongues" of radiolucency L ng ^ ^
bone °"
s can have
S e ^
characterise
°
757
.
68 (E) In children between 6 mo

incidental ‘duZ
'
detected as an alkaline
^,’,'
in
evaluations performed to assess
phosphatase values as high as 3 nnn
s
/
8 SCreen n

,
Specific '
6 laboratory
comnla /m
s usually

',
eva uat ons or
bone isoenzyme fractions am K , 6,000 UA may be enr erUm a kafine
laboratory signs “ pl '
h * k

,««„;laboratT “*
careful clinical history plus
.
Dla8nos|s can
be j”^ w
Ait ALT, GGT, b
related to excess sialylation
,
AWn phosphatase should
IS documented .
py

^* “* **
of
7 ;
» nh

«
"*

phosphorus, Cr,
0 » cause may

se, which slows
serially (every 2*3 mo)
Resolution usually occurs within 4
-6 mo.
be
clearance.
until resolution

758
Chapter 32
Rehabilitation Medicine
Questions
AQEEL MAHDI
mended duration of prophylactic anti-epileptic medicati
1. What is the recom ons
seve re traumatic brain injury ?
in seizures associated with
A . No prophylaxis
B. 1 week
C. 2 weeks
D. 6 weeks
E. 12 weeks

2. You examine a 10- year -old boy with history of traumatic brain injury due tc
car accident. He had hyperthermia, tachycardia, tachypnea, diaphoresis, and
increased muscle tone.
Of the following, the MOST likely diagnosis is
A. paroxysmal sympathetic hyperactivity
B. pulmonary embolism
C. sepsis
D . elevated intracranial pressure
E. acute painful episode

3 . Which of the following benzodiazepines is MOST commonly used to treat


spasticity in a 4- year-old child ?
A . Diazepam
B. Clonazepam
C. Lorazepam
D . Chlordiazepoxide
E. Midazolam

Y at> Ut the M0ST appropriate treatment for thei \

^ ^ °" ^*
boy who is a° known | f
0
ar old 11
yvear
-0 L
spasticitV
case of cerebral palsy with severe
t 0 oral medication associated with &&
droolingof saliva .

7 CO
following, the MOST appropriate
Of the answer is
A intramuscular botulinum toxin
B phenol neurolysis
C . intrathecal baclofen
0. selective dorsal rhizotomy
E. Hemispherectomy

5. Which of the following is the MOST


following traumatic brain injury TBI ?
common neuroendocrine disorder
A . Growth hormone deficiency
B. Cerebral salt wasting
C. Hyperpituitarism
D. Diabetes insipidus
E. Syndrome of inappropriate secretion of
antidiuretic hormone
6. Which of the following oral antispasmodic medications is a preferred
agent in
the treatment of spasticity of spinal origin ?
A. Baclofen
B. Diazepam
C. Clonidine
D. Dantrolene sodium
E . Tizanidine

is acting by peripheral
7. Which of the following oral antispasmodic medications
calcium blocking?
A . Baclofen
B. Diazepam
C. Clonidine
D. Dantrolene sodium
E. Tizanidine
, injuries has the best functional
cervical spina Cord
following
8 . Which of the
outcome ?
A . C4
B . C5
C . C6
D . C7
E . C8
760
pou ndin g fron tal headache. Examinatj0
. , h presents with hea rt 60 bpm
n
flushing^
rate
9. A 9
face, bloo d Pressure 135/70 ,
and
of the spinal cord injury involving the Uppet
shows a histo ry of
had
distended bladder, He
thoracic vertebrae. diagnosis is
the follow ing, the MOST likely
Of
A. migraine
B. essential hypertensio
n
C pheochromocytoma
D. intracranial hyperte
nsion
E, autonomic dysreflexia

Which of the following factors con


sidered protective against development
10.
of birth brachial plexus palsy ?
A. Multiparous mothers
B. Mothers with excessive weight gain
C. Diabetic mothers
D. Cephalic presentation
E. Twins

11. Which of the following types of birth brachial plexus


injury is the LEAST
severe form?
A. Neurapraxia
B. Neurotmesis
C. Axonotmesis
D. Avulsion
E. Rupture

a
12. What is the recommended time for surgical intervention in patients with
complete brachial plexus palsy ?
A. As soon as possible
B. 3 months
C 6 months
D . lyear
E. 2 years

carry the
13.Which of the following neurological level of meningomyelocefe
highest risk of developing scoliosis?
A. Thoracic
761
Uppe r lumbar ( L1- L2 )
B.
lum bar ( L3 )
c Mid
( L4- L 5 )
p Lower lumbar
E. Sacral

l4 Which of the following neurological level of meningomyelocele carry the


i

highest risk of developing hip dislocation?


A . Thoracic
B. Upper lumbar ( L1-L2 )
C. Midlumbar ( L 3 )
D. Lower lumbar ( L4-L5)
E. Sacral

15. Which of the following is the mainstay of management in patients with


meningomyelocele and neurogenic bladder ?
A . Clean intermittent catheterization
B . Prophylactic antibiotics
C. Oxybutynin
D. Bladder augmentation
E. Urethral surgeries
t of menin gomy eloce le will have
cord segmen
16. Which of the following spinal
minimal motor dysfunction?
A. L1- L 2
B. L3 L4
C. L5
D. SI
S 2 S3 power wheelchair
E.
, aod elect ric
-

where a manua I
inima l as *- imit
17. What is the
mm
en ?
can be used by childr
A. 2 years
B . 3 years
Q 4 years
p 5 years
g year 5
E
762
18. Which of the following is the MOST
common and long- lasting Seque|
ae oi
traumatic brain injury ?
A . Cognitive-behavioral disorders
B. Syndrome of inappropriate secretion of antidiuretic hormone
C. Posttraumatic seizure
D. Paroxysmal sympathetic hyperactivity
E . Spasticity
Chapter 32
Rehabilitation Medicine
Answers
AQEEL MAHDI
treatment with an anti- epileptic medication for 7 days after a
1.(B) Prophylactic
traumatic brain injury is commonly prescribed. However, treatment with an
anti- epileptic medication beyond 1 week offers no further benefit as a
prophylactic agent .
2.(A) Paroxysmal sympathetic hyperactivity PSH is thought to be due to
disruption of the inhibitory function of the mesencephalon on the
diencephalon. Some drug-related symptoms may mimic the features of PSH for
example, haloperidol and chlorpromazine may cause neuroleptic malignant
syndrome, phenytoin may precipitate a fever, and cimetidine may produce
extrapyramidal symptoms.
3.( A) Diazepam is the most commonly used medication to treat spasticity
. In
because of its long half -life and need for less- frequent administration
the availability of
children <2 yr of age, clonazepam is a good option because of
a liquid formulation and dosing guidelines.
in treating severe spasticity.
4.(C) Intrathecal baclofen ( ITB) is highly effective
via a surgically implanted infusion pump
ITB is delivered to the intrathecal space
method of delivery confers an advantage over enteral
and catheter. This
nervous system depressive effects are minimized and
baclofen, in that central
effect .
dosages can be titrated to functional are the most common
of growth hormone and gonadotropin
5.{ A) Deficiencies growth retardation and precocious puberty
resulting in
disorders following TBI ,
with severe TBI sustain chronic pituitary
8% of children
respectively . About
dysfunction. , inhibitory effect on both
monosynaptic and
exerts an
6.(A) Baclofen , Unfortunately,
. supraspinal receptor sites also exist,
polysynaptic spinal reflexes GABAergic medications.
, which is common to all
resulting in sedation
<
7. D) centrally acting, structural analog of
GABA . Bind to GABAB
/ Baclofen :
y

causing presynaptic inhibition of mono/ polysynaptic spinal


receptors
reflexes.
764
S Diazepam: Centrally acting, binds to GABAA receptors mediaf
n8
presynaptic inhibition in brain stem reticul ar forma tion and $ pjna|

polysynaptic pathways.
Clonidine: Centrally acting mixed alpha adrenoceptor agonist .
J Dantrolene sodium : Peripheral action, blocking release of calcium from
sarcoplasmic reticulum with uncoupling of nerve excitation and skeletal
muscle contraction.
Tizanidine: Centrally acting, alpha - 2 adrenoceptor agonist activity
both spinal and supraspinal sites.
8.(E) At C8 level patient can do feeding, grooming, upper extremity dressing
lower extremity dressing, bathing, bed mobility, weight shifts, transfer
independently.
9.( E ) Children with neurologic levels of injury at T6 or above are at particular risk
for interruption and decentralization of the autonomic nervous system .

Autonomic dysreflexia AD is a sustained sympathetic response as a result of a


noxious stimulus below the level of injury . Symptoms resulting from AD typicaffy
include hypertension, bradycardia , headache, and flushing of the skin above the
level of injury, although vague symptoms such as fatigue, irritability, or crying
may be the presenting symptoms in younger patients. Noxious stimuli are most
often localized to bladder or rectal distention, but may include a number of
other causes.
10.(E) Risk factors for birth brachial plexus injury include prior infants with birth
brachial plexus palsy BBPP, shoulder dystocia, birth weight >4 kg, multiparous
Delivering
mothers, mothers with excessive weight gain, and diabetic mothers.
as protective
twins or triplets, as well as cesarean sections, have been described
from BBPP .
includ e neurap raxia , neurot mesis , and axonotmesis.
11.( A) Nerve injuries
Neurapraxia is the least severe of these types and is a revers
ible loss of nerve
severe and is a total
conduction. This type will recover. Neurotmesis is the most
ion of the nerve. An avulsio n descri bes a neurotmesis of a
and comple te disrupt
lesion , and a rupture describ es the same event in a postganghor ^
preganglionic
is the interm ediate form and the most difficult to delineate
lesion. Axonotmesis in muscle strength ^

under debate. are under


specific criteria and timing remain jon
flaccid arm an a
brachial plexus palsy with a
surgery at 3 mo
considerationareforconsidered 0f aEe and
A
those with upper -plexus
involvement between 3 and 6 f ' 9) m of age
13-(A) The development of scoliosis has an assn
^ ° -
-lonT
^
the neurol « c level
Children with thoracic level defects have an 80 °'
rlsk Pereas those with a
,

sacral level are at very low risk. “'

f , Wed
14.(C) The development of the hip is also influent K eurolo8ic evel. The risk
thJ
for dislocation is highest for those with lesions at ' " , '
Unilateral hip dislocations should be fixed sureicallv 1 °' by L1 -
0
^
sszis:sr «ar *

M
nerally do

16 (E)
17.(A) Children as young as age 2 year can self -propel a manual wheelchair and
operate a power wheelchair.
memory -learning and
18.( A) Cognitive and behavioral impairments (poor
deficits) are the most
executive skills, hyperactivity, depression, awareness
These deficits can inhibit successful
common and long-lasting sequelae of TBI.
school re-entry and participation in social
activities.
Chapter 33
Environmental health
Questions
DARA.
following conditions is linked to environmental rn¥co> 0
1. Which of the xin
exposures ?
A. Cerebral palsy
8. Neural tube defects
C. Thyroid cancer
0. Kidney stones
E. Chloracne

2. A 6-year - old boy presents to the E/D with cyanosis, headache, dizziness and
generalized weakness two hours following eating excessive amount of cured
meat. He vomited twice during transport to the hospital. Physical examination
. .
shows HR 120 bpm, RR 30 bpm, Sp02 85% ECG and CXR are normal An arterial
blood sample { obtained for ABG analysis) shows chocolate brown- colored
blood. Ingestion of contaminated food is suspected.
Of the following, the MOST likely food contaminant is
A. nitrite
B. antimony
C. arsenic
D. mercury
E. copper

3. Which of the following imaging tests


has the HIGHEST average radiation
closes?
A. Computed tomography: brain
B. Computed tomography:
chest
C. Computed tomography
: abdomen/pelvis
D. Nuclear medicine ( mTc
E. Positron emission " methylene diphosphonate Bone) -
tomography ( lsF -FDG; whole body)
T

4 Which of the following is the MOST radiosensiti,ve tissue ?


A. Brain
B. Skin
C. Lung
D. Liver
E. Gonads

5 . Which of the following age groups is MOST


carcinogenesis? sensitive to radiation-induced
i
F . Neonates
G. Infants
H. Toddlers
I . Preschool children
J. School children

6 . Radiation induced cell injury


i results primarily from damage to
A. cell membrane
B. cytoplasmic reticulum
C. RNA
D. DNA
E . ribosomes

7. Which of the following effects of radiation therapy is the MOST severe acute
reaction?
A. Pneumonitis
B. Dermatitis
C. Mucositis
D. Esophagitis
E . Cerebral edema

primary malignancies has the HIGHEST cumulative


8. Which of the following
neoplasm ?
incidence of a second
A . Soft tissue sarcoma
B. CNS cancers
C. Leukemia
p Hodgkin disease
Hodgkin disease lymphoma
E. Non-

768
,
hn(iv irradiation, the expected OU

^
e
9 . After acute penetrating
who
coun

LC )
Come *
depends on absolute lymphocyte within first 48 hours
Which of the after
and fair prognosis?
exposure is associa
A. 1,000- 3 ,000
B. 1,000- 1,500
C. 500- 1,000
D. 100- 500
E. < 100

10. What is the MOST common site for accidental localized irradiation injuries ?
A . Face
B. Hand
C. Foot
D . Thigh
E. Buttocks

11. Which of the following radiopharmaceutical agents requires complete


cessation of breast feeding if administered to a lactating mother ?
A . 131- l Sodium iodide
B. 99mTc pertechnetate
123
C. -l Sodium iodide

D.
E.
"
51
m
-Tc MDP
CrEDTA

12. A 2 - month- old boy was breast fed during the last
night by his mother who
received - I Sodium iodide before 4 days as a part
of investigations for her
thyroid nodule. The mother was advised
previously to stop breast feeding but
she believed that 3 days would be enough
to clean her body .
Of the following, the MOST appropriate
treatment is
A . no treatment is required
B. dilution therapy with forcing
fluids
C. blocking therapy with potassium
iodine
D. chelation therapy with calcium
diethylene triamine pentaacetic acid
E. removal treatment with
Prussian blue
13. Which of the following
chemical pollutants is associated with increased risk
of sudden infant death syndrome
?
769
A Benzene
3. Environmental tobacco smoke
C. Lead
Q . Methyl mercury
E. Organophosphate insecticides

14 . Which of the following is the principal


source of air pollution?
A . Agricultural use of pesticides
B. Industrial waste
C. Military sources
D. Natural sources
E. Fuel combustion

15. Which of the following chronic toxic effects is caused by


organophosphate
insecticides ?
A. Cancer
B. Hormonal disruption
C. Reproductive impairment
D. Polyneuropathy
E. Pulmonary fibrosis

16. Children suffering from more than simple urticaria (e.g., wheezing, evidence
) following hymenoptera
of laryngeal edema or cardiovascular instability
) envenomation are at high risk for
(including the stinging ants, bees, and wasps
progressing to
Of the following
,reatment that can reduce this risk is
A. Antihistamines
B. Regular epinephrine
C. Immunotherapy
D. Systemic corticosteroids
E. Leukotriene modifiers
in Middle East
regularly causes fatalities
following insects
17. Which of the
children ?
A. Ants
B. Bees
C. Spiders
D. Scorpions
770
E. Wasps

18. Tobacco smoke exposure can


be considered a modifiable risk
fact 0r for
children with
A. sickle cell disease
B. immune thrombocytopenia
C. type I diabetes mellitus
D. hypothyroidism
E. rickets

19. Which of the following is the MOST appropriate method to treat children's
second-hand tobacco smoke ( SHS) exposure ?
A. Parents smoking outside the home
B. helping parents quit smoking by the 5 A's method
C. wearing a smoking jacket
D. helping parents quit smoking by the Ask, Advise, Refer model
E. helping parents quit smoking by the CEASE program

20. Which of the following heavy metals has the MOST


prevalent human
exposure and requires a screening program ?
A. Lead
B. Mercury
C. Arsenic
D. Cadmium
E. Thallium

21. Which of the following is MOST


commonly required for the treatment of
scorpion stings in children ?
A . Intravenous opioids
B. Oral pain killers
C. Intravenous benzodiazepines
D. Scorpion antivenom
E. Local anesthesia

22. Which of the following


is the classical presentation exposure to
arsine gas ? after toxic
A. Encephalopathy
B. Acute hemolysis

771
Cardiac arrhythmias
0 Acute tubular
necrosis
E. Hemorrhagic
gastroenteritis

girl lives in an
23 . A 5-year-old industrial a

SSfSs
f. oJ
Of the .
B mercury
,
owing, the MosT jke|y
lender is
exposure to heavy
^
pruritic
openia,
metals is

.
C arsenic
.
0 cadmium
thallium
24. Definitive diagnosis of
acute arsenic intoxication depends on
analysis of arsenic in quantitative

A. blood
B. urine
C. saliva
.
D hair
E. nail

25. Which of the following routes of exposure is usually associated with


elemental mercury poisoning ?
A. Oral
B. Dermal
C. Inhalation
D. Parenteral
E. Sublingual

responsible for the toxic effects of ingestion of a


26 What is the heavy metal
button battery ?
A . Organic mercury
B . Organic lead
772
C.
0
' 'eao'c'ead
P0
t-
^. ote$eo« »,he W Pfta *',h- vemot. neutop <c.
'
^ *,. aPd jnpakeA mwwft* .Ku
ar 0\d boV p
, rib uwm
jisW ba
' **- £' **,*oe*'** *o«wemo; . flnsenbatte » ^**
^
Jtttis Hs ,
af nl
edte
s nso
he

^^
0
- ^ ** «
^ ^
*
it rt
"* ° r

0
A ea(
n
' *
arseo
* a ' or
\urf>
C
I

^
suspected

o. c Pper'° °
°
^ (rvercvjr
^
28. A 3- year- old girl presents to the E/D two hours following inges
tion of
removal powder frequently used as folk remedy by her grandmothe a hair -
r. The girl
vomited twice before hospital arrival and the parents
describe a history of
convulsion during transport . On arrival she is semi
conscious and crying while
holding her stomach with bloody diarrhea and
moderate dehydration. ECG
reveals QT interval prolongation and polymorphous
ventricular tachycardia, CXR
shows diffuse pulmonary edema, while plain
abdominal X - ray is normal.
After stabilizing the patient, the MOST appropriate
next step management is
A. collecting urine for 24 hours and
send for arsenic level
B. performing gastric lavage
C. administration of activated charc
oal
D . whole- bowel irrigation
E. starting chelation therapy with
dimercaprol
29. Which of the following
chelators is preferred for chronic inorganic mere
poisoning?
A . Succimer (DMSA )
B. Dimercaprol ( BAL)
C. D-Penicillamine
D. N-acetyl-D,L-penicillamine
E. Calcium disodium
edetate (CaNa 2EDTA )
773
of the following is the STANDARD specimen for estimating
30. Which lead
toxicity?
A . urine
B. blood
C. saliva
D. hair
E. nails

31. A 9- year- old boy presented to the ER two hours after a rattle snake bite in
his right leg. His leg was progressively swollen, tender , and erythematous with
localized area of ecchymosis over the bitten site. Few minutes following
removing the field-placed tourniquet, the boy developed severe epistaxis
requiring nasal packing. Investigations revealed WBC 6000/ mm 3 , platelet
3
40,000/ mm , INR 2.5, fibrinogen 60 mg/ dL, and activated partial thromboplastin
time (aPTT) 40 sec. Following these finding, Crotalinae polyvalent immune Fab
antivenom was administered
Of the following, the MAIN indication for antivenin administration is
A . low platelet count
B. high INR
C. epistaxis
D. high aPTT
E. low fibrinogen level
the
32. Which of the following is the MOST common pathway for lead to enter
body?
A. Drinking contaminated water
B. Eating contaminated food inorganic lead compounds
contamination with
C. Cutaneous activity of young children
han d - to - mou th
D. Nonnutritive with organic lead compounds
E. Cutaneous contamination
Gl lead absorption?
enha nce
the following factors can
33. Which of
large paint
chips
of
A. Ingestion lead with » meal
of
B. Ingestion
medium ,n meal
C. Alkaline calcium
D. Presence of
deficiency
E . Iron
following is the MOST effective first aid measure afte
34. Which of the <
envenomation ?
A . Local application of ice
tourniquets in the bitten limb
B. Application of proximal
limb
C. Immobilization of the bitten
D. Incision and squeezing of the
wound
E. Wound suction

35. A 5- year- old boy lived in an old house presents


with chronic constipati
and lethargy . The boy had two attacks of

on
associated with headache generated
seizures last week. On examination he looks pale and confused
ophthalmoscopy reveals papilledema . Serum ferritin 12 ng/mL, Hb 9 g/
Erythrocyte protoporphyrin level 40 pg/dL and radiographs of long bones show ^
dense bands at the metaphyses. Chronic exposure to a heavy metal is
suspected.
Of the following, the MOST likely offender is
A . lead
B. arsenic
C. aluminum
D . cadmium
E. mercury

36. Which of the following is the earliest manifestation of neurotoxicity after


neurotoxic snake envenomation ?
A. Upper limb weakness
B. Ptosis
C. Lower limb weakness
D. Respiratory muscles weakness
E. Flaccid paralysis

37. Which of the following drug combinations is indicated for children with lea
encephalopathy ?
A . CaNa 2 EDTA plus DMSA
B. CaNa 2 EDTA plus BAL
C. DMSA plus BAL
D . D- Penicillamine plus DMSA
E . D- Penicillamine plus BAL

775
following mushroom species is the MOST fatal ?
38 which of the
A Gyromitra
B, Boletus
Q Cortinarius
D, Amanita
E Psilocybe

39 . A B- year - old boy presents to the E / D with delirium for the last 30 minutes
The boy was completely healthy before one hour when he ingests a wild
mushroom from the adjacent garden. Physical examination shows plethoric
face, dilated pupils, hot and dry skin, HR 150 bpm, RR 24 bpm, and temp 38 C
Of the following, the MOST appropriate management is
F . administration of activated charcoal
G. performing gastric lavage
H . whole- bowel irrigation
I. administration of cathartics
J . induction of emesis

40 . A 13- year -old boy presents to the E/ D with generalized erythema and
shortness of breath, one hour after having dinner in a seafood restaurant . The
symptoms started 20 min after eating a fish dish as abdominal pain and diarrhea
associated with oral numbness. On examination there is generalized urticarial
rash, diaphoresis , facial swelling, and tachycardia .
Of the following the MOST likely diagnosis is
A. shellfish poisoning
B. scombroid fish poisoning
C . ciguatera fish poisoning
D . pufferfish poisoning
E . Rudderfish poisoning
to infant
classical toxic effect of illegal addition of melamine
41. What is the
formula?
A . Encephalopathy
B . Gall stone formation
C. Chemical hepatitis
D Renal stone formation
E . Hemorrhagic gastroenteritis
biologic terrorism, patients can be classified as havi
42 . In chemical and dermatologic mg
primarily respiratory, neuromuscular
, or manifestations . |n ..
u
"

regard, which of the following


agents has prominent dermatologic findings? *
A. Mustard
B. Anthrax
C. Plague
D. Chlorine
E. Phosgene

43. The nerve agents (tabun, sarin, soman, and VX ) antidote kits consisting 0f
prefilled autoinjectors designed for the rapid administration of
A . Pralidoxime and pyridostigmine
B . Pralidoxime and diazepam
C. Atropine and pralidoxime
0 . Atropine and diazepam
E. Atropine , pralidoxime, and diazepam

44 . What is the MAJOR presentation of envenomations due to snakes, spiders,


scorpions, and other venomous animals ?
A . Acute compartment syndrome
B . Coagulopathy
C. Anaphylaxis
D. Localized pain and swelling
E . Infected wounds

45. A 10- year- old boy presents to the E/ D with large lacerated wound in his left
thigh due to dog bite . After appropriate material has been obtained for culture ,
the wound is anesthetized, cleaned , and prepared for vigorous irrigation.
Of the following, the MOST appropriate irrigation method is
A . copious irrigation with sterile saline
B. Irrigation with antibiotic-containing solutions
C. irrigation with catheter
D. blunt- tipped needle irrigation
E . high- pressure irrigation
>r
46 . Which of the following antimicrobials can be considered as FIRST CHOICE
the treatment of most infected wounds caused by animal and human
"
A . Clindamycin
B.
Trimethoprim/ sulfamethoxazole
Q Ciprofloxacin
p , cefotaxime
E Amoxicillin/clavulanic acid

47. Which of the


following is the drug of CHOICE for
the treatment of rat bite
fever?
A. Penicillin G
B. Doxycydine
C. Gentamicin
D. Streptomycin
E. Clindamycin
health
Chapter
^ Environmental
Answers
HAIDAR A N . ABOOP ,
been link d o at leas 2 COrldit|
. to mycotoxins
have
'
and acute pulmonary hemorrhage. 0
i,m Exposures
tube
children: neural to other
defects
environment exposure; cerebral pa >°
conditions are linked
methylmercury, thyroid
cancer to radiation , kidney stones to melamine ,
'
.
chloracne to dioxin not only help kill bacteria
, but also prodtJ
2,(AJ Nitrates and nitrites
give meat a pink or red color . Nitrite ^ a
salts are most
characteristic flavor and
. Nitrite poisoning usually occurred 1-2 hr f0|| W ng
often used in curing meat
foods with nitrates, or spinach eXposed °'
eating cured meals, any contaminated
Signs and symptoms include nausea
to excessive nitrification .
, loss of consciousness
cyanosis, headache, dizziness, weakness
brown-colored blood . It is usually self -limited
. Treatment is support - ,
^ ^
,,
methylene blue may be required to correct methemoglobinemia .
3. E Average radiation doses (mSv) by Imaging test for pediatric population
/ Interventional fluoroscopy: AP & Lat abdomen
P
*
( 0.2-11
-

mSv/min)
/ Interventional fluoroscopy: head (0.02-0,08
mSv/min)
Interventional fluoroscopy: cardiac _ (0.1- 1
mSv/min)
Digital radiography: 2 view chest (0.04-0.061
/ Digital radiography
: 2 view abdomen (01-0.4)
/ Computed
tomography: brain (0.8- 4 )
/ Computed
tomography: chest (1-4)
Computed tomography;
/ abdomen/pelvis ( 2 -7)
Nuclear medicine { <3mTc methylene
!

/ Positron diphosphonate-Bone ) ( 5- 7)
emission tomography ( F-FDG;
18
( 3- 15 )
There iis clear whole body )
evidence of radiation-
posure >100 m$
v, whereas no
induced cancer risk for whole ho V
direct epidemiological data support 5 exposure

779
is unclear is the canror r.'sk for
,0 «sv »What calculated
and 100 m5v.
radiosensitive weight , f 0fs
^'
^ ct

r<ssue
HO of radiological protects
t*nmiss* ' * *
<
103 .'^* * *
r) acr^
^ to mtefnacors
-marrow, colon lUn sto
Red bone ^ _ "tach. 5^
>

_
’ remainder tissues 0 12
)
/ Gonads (0.08
c Bladder, oesophagus, liver' th»~ (0 04)
,
Bone surface
^ Compared
^ .
, brain saliva Sk n <
°°•« ,«
1)

“ ““-
with middle-aged
S.(A)
more ISSidti ** ' ,
carcmogenesis * 4r
* * *^
. and neonates are more
sensitive than older children.
6.(0)
7,(A) One of the most severe acute reactions is pneumonitis. It can be manifest
within 24 hr of irradiation when there is an exudation of proteinaceous materia)
into the alveoli and intraalveolar edema. Most often, radiation pneumoratrs
begins 2-6 mo after the beginning of radiation with a clinical presentation of
fever, cough, congestion, and pleuritic pain.
8.(0) Radiation therapy increases the risk of second cancers in a dose -
dependent manner for nongenetic neoplasms. Almost 70% of the secono
neoplasms are in the field of the original irradiation.
Primary malignancies with
second neoplasm in the orderof
the highest cumulative incidence of a
frequency are Hodgkin disease ( 7.6 .
) soft tissue cancers erf bone
.
.
i .
(3.3), leukemia
i1 . n
( + )v central nervous system
2.1 -
( CNS) cancers ( 2.1) and non
Thjs refjects an overall standard incidence rate
(
Hodgkin disease lymphoma are bon^ breast, thyroid. and CNS
secono t

of 6.38%. The most prevalent


lesions. .on absolute lymphocyte count after
ha< ed
acute
9.(0 Expected radiation:
penetrating w ^ Prognosis
°
ALC wit
.
Minimal
first 48 hr after oou
, range) No significant injury
^
Significant but probably nonlethal injury ,
i 000- 3
. '
.
a OOO -1.50 °
good prognosis Severe injury, fair prognosis
8
500'1.000 Very severe injury, poor prognosis
^ 100- 500 Lethal without compatible bone marrow
v'
<ioo
donor
site foraccidental localized irra(Jja
11 n <K)r,
the mo
$t COO
'’ or playing with
lost radiation
sour < v
hand result
io.l»> usually
Thc
injure,
~i* ^
as a
|S

°'
common Rental
a
cted
; ' * * is th
butt *
highlV radioactive sources i ,?
th gh

most unsuspe the


(1

^
.
The second
"
nantW °
m placing
,orv
L'

guidelines on breastfeeding during th6


recommended complete cessatf0
poeMO-
1L|A| Noce
period o 1nlK
SS ' sodium in.|
|a
e
^
r medicine *
lodl
pertechnetate
^ other distracters.
administered
°
to a lactating mother
is used, while
^ of
Jrl

3 Son
"Tust
.
breast
jSm most
in
effectwe
res
re
knowledge
be instituted
*
feeding W

^
of both <
^
of internal contamination
and

quicklyjo
the 3
chemical form
r
radionuclide
Treatment
with

Blocking therapy is the


(i i n e containing
,

administration
compounds to patients
or
of potassium
iodine with radioactive iodine. The stable iodine
with know intemaUont jts effectiveness decreases rapidly as
effective
^ inCrrDollution
time elapses after
13.(B) There is

“ '"f
the

^^
^ o^
den infant death syndrome and asthma in
and environmental tobacco smoke. Benzene is
infants exposed to lead with neurobehavioral toxicity, while
childhood cancer
associated with organophosphate insecticides are associated with
methyl mercury, and
developmental neurotoxicity. of air pollution. In high- and
principal
is the source
14.(E) Fuel combustion
countries, combustion of fossil fuels (coal, oil, and gas) accounts
middle-income
. In low - and lower - middle-income countries, the major
for most air pollution
biomass : wood , dung, straw , and charcoal . Coal is the single
source is burning of
highly polluting fossil fuel and also the most important source of the
most

effeCtS that indudf


Ts °stemT'
f (or anoPhosPha (e!)

bowonT ? 8 and C6ntral ne


0n and reProductive *° VS
^ *
100
lmpairrne t (DDT,
kepone
dbromochloroDrUPt° "
' ); cancer (a|dnn . , Chlorophenoxv herbicides
pr pane
12',4,5. . . ^
^
f brosis (Paraquat)
16 <c) lmmunoPUh. DnarV '
stmgs in high.rk
°apv
a reduces the risk svstemic 2anaphylaxis from future
<5%-
3 % and 609610
17-(t>) Most scomMt' ,
3nd Natalities are
ents from somewhn
occu n ! ° States
southwestern UnitedAmerica
e* n other regions 0 the world -especially Latin
Middle East, and A$ia -a number of scorpions
the regularly cause
fric3 r
^
‘ tobaccosickle farm,, r
^
i
c
with
are exposed
smoke exposure can be considered a modifiable risk

to
cell disease and with CF.
second-hand
Children with sickle
tobacco smoke have increased
JiH

JS?
°
>
5
callV increased rates of Emergency Department visits and hospitalize '
crisis and acute chest syndrome. In addition,
ions

^
( vaso-
exposure
ocduswe
is also associated with pulmonary
tobacco smoke
function abnormalities among
children with sickle cell disease, independent of their baseline disease.
19.(E) The best method to treat children's SHS exposure is to eliminate this
exposure by helping parents quit smoking. Methods to reduce exposure such as
'smoking outside" or wearing a "smoking jacket" have not been shown to
eliminate biochemically confirmed SHS exposure. The 5 A's method (Ask,
Advise, Assess, Assist, and Arrange) was developed for adults. The pediatric
model has been abbreviated to the Ask, Advise, and Refer. The Clinical Effort
Against Secondhand Smoke Exposure ( CEASE) is a program that trains
pediatricians and their office staff to systematically provide cessation counseling
and interventions to parents and other adults who smoke, while offering more
assistance in quitting smoking than Ask /Advise/Refer.
, four of the World Health
20 (A) Lead, mercury, arsenic, and cadmium
.
of greatest public health concern, are the
Organization's ( WHO) "Ten chemicals
to humans. The most prevalent of

. ***£* jjj31
heavy metals posing t 99% of lead-poisoned children are
these exposures is lead It is er than through clinical recognition of
“ ^
rAt
identified by screening proce to thallium is relatively rare,
lead related symptoms. Expos o nQt produce severe effects and require only
21.( B) Most scorpion
wound care and
more severe symptoms
stings
orally admin
m v
^ ^^ medlcations. However, patients with
jntravenous opioids for analgesia and
sc) e spasm or agitation
. The antivenom is
SeVe e ^ neurotoxicity or other severe
benzodiazepines forcriticaHy ill patients with
for pamin that is not responsive to adequate doses
recommended
including
jntractable
odorless, nonirritating, and highly toxic. After
symptom ' esjcs -
c
of opioi
r
ga5 is
absorbed °arsine enters RBCs and is oxidized to arsenic
22.(B ) afsine arsenic. Complexing of these derivatives with red
cell
exposure
^ instability and massive hemolysis.
elen
uits in cell membrane

d hYd rvl er UpStlnic cause the other distractors.
' ^
sulf
Oth^
^ ^
Cm farSen
r
782
by prolonged fatj
,
X arsenic
is characterized
exposure chron,c encephalopathy,
0

23.(0 Subacute
o - anemia, thrombocytopenia, ch
malaise, weight
neuropa V,
sensorimotor " |jnes { transverse white striae on the naik ,
e ^ in approximately 5% of patieJ
cough, and ga
trof' m nths
^
5
after exposure
1-2
become apparent u|ceratjon perjphera| edema
Dermatologic findings ^ )

desquamation. Chronic arsenic toxicity cause^,


pruritic macular ra ‘ hjkJren resulting in skin lesions, lung disease, and
SSTSS fusion -
usually from environmental or
. Chronic exposure to low levels of arsenic
occupational
24,( B| Because urinary excretion
sources.
of arsenic is intermittent, definitive diagnosis
greater than 50 pg/L in a 24

depends on a 24 hr urine collection. Concentrations


intoxication. Blood arsenic levels
hr urine specimen are consistent with arsenic
rarely are helpful because of their high variability and the
rapid clearance of
arsenic from the blood in acute poisonings. Elevated arsenic values in the hair or
nails must be interpreted cautiously because of the possibility of external
contamination.
25.(C) Mercury exists in three forms: elemental mercury, inorganic mercury
salts, and organic mercury. Elemental mercury is present in thermometers,
sphygmomanometers, barometers, batteries, and gold or silver smelting
processes. Dental amalgams containing elemental mercury release trace
amounts of mercury. Severe inhalation poisonings have resulted from attempts
to separate gold from gold ore by heating mercury and forming a gold mercury
amalgam. Inhaled elemental mercury vapor is 80% absorbed by the lungs and is
distributed rapidly to the central nervous system because of its high lipid
solubility. Elemental mercury liquid Is poorly
absorbed from the gastrointestinal
tract, with less than 0.1% being
absorbed.
26.(D) Acute ingestion of inorganic
mercury salts (typically secondary to
StroemerLs
hemX!
8
,*7 !?bv
" *
58
* hemat
*metallic manifest in few hours as corrosive
a
nausea
taste< oropharyngeal burns, ,

cardiovascular cX4 !
,
^
a
" ,hPain'
e de
che ia
'
tubulaf
0 2 acu e
. Chronic
27 ( E )
consisting of tremnr
inorganic
no
mprnm, . , . ,
mtox cation produces the classic triad
; .

syndrome may resuli ^^g ostomatitis


J
- - *—
r!
"“ * i s s- *,r ss.
athetosis and spasmodic ballismus. Mixed
,* ,
5! t
0 C
visual disturbances may also be present. The sensorimotor
,, ce5 ^ |0de emotional lability, delirium, headachesneuropsychiatric
^^ roP3 hv anjnc
P

Jfsti rba> ' , memory


n0rexia , and fatigue. Renal dysfunction ranges from asymptomatic
loss,
nephr t c syndrome.
prote'nuriaw
n °'
te ingestion of inorganic arsenic and mercury
(E) salts results in
^ 3 gastroenteritis
of the
, cardiovascular collapse, and multi-organ
corrosive effects of these compounds
dysfunctio Because

^
, induced
recommended, and endoscopy may be considered
efneSlS & pnic
lavage-
r<',
and mercury are not well adsorbed .
before gastric
activated charcoal
Whole -bowel irrigation is used to remove anv raHi„
mercury poisoning is
os Soon as possible after the exposure .
Chelation should be continued until 24 hr urinary arsenic or mercury levels
return to normal (<50 pg/L for arsenic and < 20 pg/L for mercury), the patient is
symptom-free, or the remaining toxic effects are believed to be irreversible.
Dimercaprol, also known as British antilewisite ( BAL), is the chelator of choice
for a patient who cannot tolerate oral therapy, as often is true for critically ill
patients and after ingestion of the corrosive inorganic arsenic and mercury salts.
29.( A) Oral chelating agents are used to replace the painful BAL injections when
the patient is stable enough to tolerate oral therapy and prolonged chelation is
necessary . Succimer , also known as 2,3 -dimercaptosuccinic acid (DMSA), is an
BAL. 0- Penicillamine is an orally
administered water- soluble derivative of
orallyV aummibiereu
for less-severe mercury poisoning
administered chelator that can considered poisoning, but its use is largely
^
or as an adjunct to BAL therapy in
arsenic
of the potential for significant leukopenia,
restricted because ia. A newer investigational
analog, N-acetyl-
thrombocytopenia , and proteinuria
with variable success in mercury poisoning.
.
D L-penicillamine, is used
lead poisoning.
CaNa EDTA , is used for
in snake envenomation are:
30 (B)
a tjvenin administration
for
31.(C) indications el "
sjgnjticant Weeding
^ 25 QOO / mm^
Evidence of '"
*

y
Platelet
Activated
Fibrinogen
^
internatio"P33'
»
aHzed ratio (1NR ) >3
thromboplastin time (aPTT) >50 sec
/ dL

7 ftA
placed in the field, they should be cautiously removed
If tourniquets are adverse effects that may follow after
due to possible
venous access is obtained systemic circulation .
fr0 fT]
into the
a sudden release of venom
hand - to- mouth activity of young children is the
32 .{ D) The nonnutritive ^ ost
~

for lead to enter the body . In most cases, lead is ingest “


'
common pathway
either as a component of dust licked off
of surfaces or in swallowed paint ch *
through water contaminated by its flow through lead pipes or brass fixtures '^
from contaminated foods or liquids. Cutaneous contamination with inorga
lead compounds, such as those found in pigments, does not result in 3
substantial amount of absorption. Organic lead compounds, such as tetraeth
lead, may penetrate through skin, however .
^
33 (E)
34 (C) Prehospital care should focus on rapid transport to the emergency
department while providing supportive care. Constrictive clothing, jewelry, and
watches should be removed, and the injured body part should be immobilizea
in a position of function at the level of the heart. Many popularized field
treatments for snake bites — such as tourniquets, ice, electric shock, incision,
and suction —have proven ineffective or deleterious.
35.(A) Gastrointestinal symptoms of lead poisoning include
anorexia, abdominal
pain, vomiting, and constipation, often occurring
and recurring over a period of
weeks. Children with blood lead levels (BLLs) >
20 pg/dL are twice as likely to
have gastrointestinal complaints as those with
lower BLLs. Central nervous
system symptoms are related
to worsening cerebral edema and increased
intracranial pressure. Headaches, change
in mentation, lethargy, papilledema,
seizures, and coma leading to death
are rarely seen at levels lower than 100
pg/dL but have been reported
in children with a BLL as low as 70 pg/dL
Erythrocyte protoporphyrin
levels > 35 pg/dL are abnormal and are consistent
with lead poisoning,
iron deficiency, or recent inflammatory disease
Radiographs of long bones may
cases of anemia in lead-poisoned show dense bands at the metaphyses. Most
iron deficiency and children are a result of other factors, such as
36.(B) Following r
hemoglobinopathies.
neurotoxic snake snvenomation, manifestations of
neurotoxicity generally arecaus
junction* Symptoms usually
d hy CUrare-
like blockade at the neuromuscular
be ’ W ,
cran a nerve palsies such as ptosis,
dysarthria, and dysphagia
paralysis.
andrrf ^ ^ *
Pro8ress to respiratory failure and complete
.
37 (B) Children with BLLs (
blood Iead levels) of 44-70 pg dL may betreated
a single drug, preferably
DMSA . Those with BLLs of pg /
70 /dL or greater re
785
thoseeocephalopathv-

~rrc,c
>-(>»1 Poisonings by species of Amanita (death can m u

?
'

*jccount fatalities from II ^ and Gal ina


^
, ,General management of mushroom ingestion-
for 95% of the mushroom into*
39. A
Determine history of ingestion
Attempt to determine the possible syndromes
/ Administer activated charcoal.
If feasible and when indicated; identification of
mycologist.
mushroom by
Try to perform a preliminary identification of
mushroom and spores.
J Maintain supportive measures
^ Avoid antispasmodics for gastrointestinal symptoms.
* Anticipate the clinical course.
40.(B) Scombroid (pseudoallergic) fish poisoning: Ingestion of members of
the
Scombridae families, including albacore, mackerel, tuna, bonita, and kingfish,
have been linked to pseudoallergic fish poisoning. The bacterial transformation
of histidine to histamine (if refrigeration is inadequate ) is responsible for the
clinical syndrome. Histidine is found in high concentrations in the flesh of
scombroid fish. The onset of clinical manifestations is acute and occurs within
10 min to 2 hr of ingestion. The most common symptoms and signs are
diarrhea, erythema, sweating, flushing, diaphoresis, urticaria, nausea, and
or numbness, dizziness,
headache. Abdominal pain, tachycardia, oral burning
swelling also occur. The illness is usually
respiratory distress, hives, and facial
self-limited, terminating within
gen tsy mass. The illegal
nitr0 addition of
41.(0) Melamine contains ^
the formula a milky appearance and falsely
melamine to infant formu
a
a
^
^asured by nitrogen testing. Melamine, combined
raise the protein content * ufate crvstals in the kidneys . Along with protein,
with cyanuric acid,
uric acid, and phosphat
42.(A)
attacks,
Following *
dermatology
^orrnSi
, e|arnine

SfUnd
ngs
inent
^.
forms renal calculi,
mustard ( a blistering agent ) during
e p° ^ ,^ are prominent , while for other
terrorist
distracters
fmdmgs

rr.ssr “ ** *• * *.•rsss:
respiratory
, spiders, scorpions, and
43.(C) due to snakes
u other
Enveno mations > • >«
44.(0) can cause sign *
animalsn»Y oca Itzed pain
an

causeo ‘ 786
, p 3c been bia n d (0r culture, the
° ‘\
khef aPPf0pf e materia
' letit '
^' '
c e3 d, and nUsW «f 6ated wth
'
animal
nta«Vmg solutions « »» ^
no
(tb aot<b ot c
' ' *“ cauSe local *
prnvw
irritation*«
lS
'
cal
a
h el e Led < evaaUlC deb
«<
"gan«Sww
^° * 4- “^
of
b
* fjeas
fluctuant a«a incised and '
e4W
to£man bites -
m
^ c nMcWS
be 3
*' slterrestrial
animal

^,^
oyAcml
^»nbeorconS levofloxa
cr
e er
1

u
..
^
cin or
M
ooxVtVclne
W»««tacm '
'
In cases where intravenous antibiotics are deemed necessary, single
antimicrobial choices can include ampicillin
ertapenem, or moxifloxacin.
/sulbactam, cefoxitin,
47.( A ) Penicillin is the drug of
choice for both forms of rat
Intravenous penicillin G or intramuscular penicillin bite fever ,

for 7-10 days. Doxycycline, gentamicin G procaine is recommended


, or streptomycin
alternatives for penicillin allergic patients represents effective
.

787
Laboratory Medicine
Questions
. N. ABOOD
of the following variables has no considerable effects when developing
^
1.
reference laboratory values for pediatric population ?
composition
A Genetic
B. physiologic development
C. Environmental influences
D. Subclinical disease
E. Socioeconomic status

2. Which of the following terms is a measure of the nearness of a test


result to
the actual value?
A. Precision
B. Accuracy
C. Sensitivity
0, Specificity
E- Reproducibility

Assume you have the results of PCR test for COVID- 19; the number of positive
tests is 95 collected from a 100 truly-infected children.
The sensitivity of the tests is
A . 5%
B . 15%
C. 75%
D. 85%
E . 95%
further confirmative investigations
4. Which
in later Ufe
A, Phenyike^
mir a
actosemia
. diagnosis through neonatal screening programs?

B Ga
'
c Hvpothyroidism
788
D. Cystic fibrosis
disease
E. Sickle cell
following analVtes
,
iss measured in
serum ?
5. Which of the E
A. Immunoglobulinpartial pressure
,
B. Carbon dioxide
hormone
C. Antidiuretic
D. Ammonia
E. Magnesium
lytical technique for neonatal screening program fQr
ana
6. What is the best ?
inborn error of metabolism
immunoassay
A. Enzyme-amplified
.
B Atomic absorption
liquid chromatography (HPLC)
C. High performance
-
(MS/MS)
D. Tandem mass spectrometry
spectrometry (GC/MS)
E. Gas chromatography mass

7. Which of the following analytes is


measured in plasma ?
A. Bilirubin
B. Aspartate transaminase
C Albumin
D. C-reactive protein
E. Base excess

8. Assume you are evaluating the specificity of an immunoassay test for rapid
diagnosis of COVID -19; the number of norma! children tested is 150 and the test
is positive in 30 children.
The specificity of the tests is
A . 30%
B. 50%
C. 70%
D. 80%
E. 90%

9. What is the best analytical


technique for HIV screening?
A. Atomic absorption
B. ELISA
f

c. High-performance liquid chrnf atog


p. PCR ^ raphv (
Flowcytometry
HPLC)

)0. Which of the following analyte


A. Iron
\s
measuredin who,e
B. TSH ' blood?
C. Uric acid
0. Lead
B. Potassium

790
Chapter 34
Laboratory Medicine
Answers
HAIDAR A. N. ABOOD
1 ( E ) Normal values (reference intervals ) 3re difficult to establish within
the
pediatric population, Differences in genetic composition, physiologic
development, environmental influences, and subclinical disease are
variables
that need to be considered when developing reference intervals.
2.( B) Accuracy is a measure of the nearness of a test result to the actual
value,
whereas precision is a measure of the reproducibility ( how close are repetitive
measurements of a single sample) of a result. Sensitivity is ability to
detea a
disease and specificity is ability to define the absence of a disease.
.
3 ( E) Sensitivity = number positive by test x 100 total number positi
/ ve
4.( D) The results of neonatal screening for cystic
fibrosis show clear benefits
associated with preclinical diagnosis, but some inher
ent difficulties associated
with genetic screening for complex autosomal
recessive diseases that are
common and are caused by a large number of
mutations (>1,500) of a single
gene. The definitive diagnostic test for CF
is the measurement of concentrations
of chloride in sweat, a test that is not practical
during the first week of life.
5.( A) Immunoglobulin E.
Carbon dioxide,

““ ,
IMS/MSI i, lechmcali advanced me had in
, ,
1 cZTn/ *‘
-
WWW d «»
W «
is based on character
m60 ragmented “ * ' "
a 8ain - Identification of compounds
Bro
7.(C) Albumin . ““^
nb0rn errors of metabolism
- *
Bilirubin (serum),
Base excess (wholebfcTdf
8.(D ) Specificity - number negative
» -
tranSaminase ( serum C-reactive Pr tein(serum),

by test x 100/total number without


°
9 ( B) disease.
10.(D) Lead.
Iron (plasma), TSH (serum ), Uric acid (
serum), Potassium (serum).
791

You might also like